You are on page 1of 497

41 Years’

CHAPTERWISE TOPICWISE
SOLVED PAPERS
2019-1979

IITJEE
(JEE Main & Advanced)

Chemistry
Ranjeet Shahi

Arihant Prakashan (Series), Meerut


Arihant Prakashan (Series), Meerut
All Rights Reserved

© Author
No part of this publication may be re-produced, stored in a retrieval system or distributed
in any form or by any means, electronic, mechanical, photocopying, recording, scanning,
web or otherwise without the written permission of the publisher. Arihant has obtained
all the information in this book from the sources believed to be reliable and true. However,
Arihant or its editors or authors or illustrators don’t take any responsibility for the absolute
accuracy of any information published and the damages or loss suffered there upon.

All disputes subject to Meerut (UP) jurisdiction only.

Administrative & Production Offices


Regd. Office
‘Ramchhaya’ 4577/15, Agarwal Road, Darya Ganj, New Delhi -110002
Tele: 011- 47630600, 43518550; Fax: 011- 23280316
Head Office
Kalindi, TP Nagar, Meerut (UP) - 250002
Tele: 0121-2401479, 2512970, 4004199; Fax: 0121-2401648

Sales & Support Offices


Agra, Ahmedabad, Bengaluru, Bareilly, Chennai, Delhi, Guwahati,
Hyderabad, Jaipur, Jhansi, Kolkata, Lucknow, Meerut, Nagpur & Pune

ISBN : 978-93-13196-96-9

For further information about the books published by Arihant


log on to www.arihantbooks.com or email to info@arihantbooks.com

/arihantpub /@arihantpub Arihant Publications /arihantpub


CONTENTS

1. Some Basic Concepts of Chemistry 1-21 19. Extraction of Metals 262-273

2. Atomic Structure 22-38 20. Qualitative Analysis 274-285

3. Periodic Classification and 21. Organic Chemistry Basics 286-309


Periodic Properties 39-44
22. Hydrocarbons 310-326
4. Chemical Bonding 45-62
23. Alkyl Halides 327-339
5. States of Matter 63-78
24. Alcohols and Ethers 340-352
6. Chemical and Ionic Equilibrium 79-102
25. Aldehydes and Ketones 353-371
7. Thermodynamics and
Thermochemistry 103-122 26. Carboxylic Acids and
their Derivatives 372-387
8. Solid State 123-131
27. Aliphatic Compounds
9. Solutions and Colligative Properties 132-146 Containing Nitrogen 388-397

10. Electrochemistry 147-167 28. Benzene and Alkyl Benzene 398-412

11. Chemical Kinetics 168-184 29. Aromatic Compounds


Containing Nitrogen 413-426
12. Nuclear Chemistry 185-187
30. Aryl Halides and Phenols 427-438
13. Surface Chemistry 188-193
31. Aromatic Aldehydes, Ketones
14. s-block Elements 194-203 and Acids 439-452
15. p-block Elements-I 204-212 32. Biomolecules and Chemistry
16. p-block Elements-II 213-232 in Everyday Life 453-468

17. Transition and 33. Environmental Chemistry 469-472


Inner-Transition Elements 233-240 Ÿ JEE Advanced Solved Paper 2019 1-14
18. Coordination Compounds 241-261
SYLLABUS
JEE MAIN
Section A : PHYSICAL CHEMISTRY
UNIT I Some Basic Concepts in Chemistry model of hydrogen atom - its postulates, derivation of
Matter and its nature, Dalton's atomic theory; Concept the relations for energy of the electron and radii of the
of atom, molecule, element and compound; Physical different orbits, limitations of Bohr's model; dual
quantities and their measurements in Chemistry, nature of matter, de-Broglie's relationship,
precision and accuracy, significant figures, S.I. Units, Heisenberg uncertainty principle.
dimensional analysis; Laws of chemical combination; Elementary ideas of quantum mechanics, quantum
Atomic and molecular masses, mole concept, molar mechanical model of atom, its important features,
mass, percentage composition, empirical and ψ and ψ2, concept of atomic orbitals as one electron
molecular formulae; Chemical equations and wave functions; Variation of ψ and ψ2 with r for 1s and
stoichiometry. 2s orbitals; various quantum numbers (principal,
angular momentum and magnetic quantum numbers)
UNIT II States of Matter and their significance; shapes of s, p and
Classification of matter into solid, liquid and gaseous d - orbitals, electron spin and spin quantum number;
states. rules for filling electrons in orbitals – aufbau principle,
Gaseous State Measurable properties of gases; Gas Pauli's exclusion principle and Hund's rule, electronic
laws - Boyle's law, Charle's law, Graham's law of configuration of elements, extra stability of half-filled
diffusion, Avogadro's law, Dalton's law of partial and completely filled orbitals.
pressure; Concept of Absolute scale of temperature;
Ideal gas equation, Kinetic theory of gases (only UNIT IV Chemical Bonding and
postulates); Concept of average, root mean square and Molecular Structure
most probable velocities; Real gases, deviation from Kossel Lewis approach to chemical bond formation,
Ideal behaviour, compressibility factor, van der Waals' concept of ionic and covalent bonds.
equation, liquefaction of gases, critical constants. Ionic Bonding Formation of ionic bonds, factors
Liquid State Properties of liquids - vapour pressure, affecting the formation of ionic bonds; calculation of
viscosity and surface tension and effect of temperature lattice enthalpy.
on them (qualitative treatment only). Covalent Bonding Concept of electronegativity,
Solid State Classification of solids: molecular, ionic, Fajan's rule, dipole moment; Valence Shell Electron Pair
covalent and metallic solids, amorphous and crystalline Repulsion (VSEPR) theory and shapes of simple
solids (elementary idea); Bragg's Law and its molecules.
applications, Unit cell and lattices, packing in solids Quantum mechanical approach to covalent bonding
(fcc, bcc and hcp lattices), voids, calculations involving Valence bond theory - Its important features, concept
unit cell parameters, imperfection in solids; electrical, of hybridization involving s, p and d orbitals;
magnetic and dielectric properties. Resonance.
UNIT III Atomic Structure Molecular Orbital Theory Its important features,
Discovery of sub-atomic particles (electron, proton and LCAOs, types of molecular orbitals (bonding,
neutron); Thomson and Rutherford atomic models and antibonding), sigma and pi-bonds, molecular orbital
their limitations; Nature of electromagnetic radiation, electronic configurations of homonuclear diatomic
photoelectric effect; spectrum of hydrogen atom, Bohr molecules, concept of bond order, bond length and
bond energy.
Elementary idea of metallic bonding. Hydrogen ionization of water, pH scale, common ion effect,
bonding and its applications. hydrolysis of salts and pH of their solutions, solubility
of sparingly soluble salts and solubility products, buffer
UNIT V Chemical Thermodynamics solutions.
Fundamentals of thermodynamics System and
surroundings, extensive and intensive properties, UNIT VIII Redox Reactions and
state functions, types of processes. Electrochemistry
First law of thermodynamics Concept of work, heat Electronic concepts of oxidation and reduction, redox
internal energy and enthalpy, heat capacity, molar heat reactions, oxidation number, rules for assigning
capacity, Hess's law of constant heat summation; oxidation number, balancing of redox reactions.
Enthalpies of bond dissociation, combustion, Eectrolytic and metallic conduction, conductance in
formation, atomization, sublimation, phase transition, electrolytic solutions, specific and molar conductivities
hydration, ionization and solution. and their variation with concentration: Kohlrausch's
Second law of thermodynamics Spontaneity of law and its applications.
processes; ΔS of the universe and ΔG of the system as Electrochemical cells - Electrolytic and Galvanic cells,
o
criteria for spontaneity, ΔG (Standard Gibb's energy different types of electrodes, electrode potentials
change) and equilibrium constant. including standard electrode potential, half - cell and
cell reactions, emf of a Galvanic cell and its
UNIT VI Solutions
measurement; Nernst equation and its applications;
Different methods for expressing concentration of Relationship between cell potential and Gibbs' energy
solution - molality, molarity, mole fraction, percentage change; Dry cell and lead accumulator; Fuel cells;
(by volume and mass both), vapour pressure of Corrosion and its prevention.
solutions and Raoult's Law - Ideal and non-ideal
solutions, vapour pressure - composition plots for UNIT IX Chemical Kinetics
ideal and non-ideal solutions. Rate of a chemical reaction, factors affecting the rate of
Colligative properties of dilute solutions - relative reactions concentration, temperature, pressure and
lowering of vapour pressure, depression of freezing catalyst; elementary and complex reactions, order and
point, elevation of boiling point and osmotic pressure; molecularity of reactions, rate law, rate constant and its
Determination of molecular mass using colligative units, differential and integral forms of zero and first
properties; Abnormal value of molar mass, van't Hoff order reactions, their characteristics and half - lives,
factor and its significance. effect of temperature on rate of reactions - Arrhenius
theory, activation energy and its calculation, collision
UNIT VII Equilibrium theory of bimolecular gaseous reactions (no
Meaning of equilibrium, concept of dynamic derivation).
equilibrium.
UNIT X Surface Chemistry
Equilibria involving physical processes Solid -liquid,
Adsorption - Physisorption and chemisorption and
liquid - gas and solid - gas equilibria, Henry's law,
their characteristics, factors affecting adsorption of
general characteristics of equilibrium involving
gases on solids- Freundlich and Langmuir adsorption
physical processes.
isotherms, adsorption from solutions.
Equilibria involving chemical processes Law of
Catalysis Homogeneous and heterogeneous, activity
chemical equilibrium, equilibrium constants (K and K)
and selectivity of solid catalysts, enzyme catalysis and
and their significance, significance of ΔG and ΔGo in
its mechanism.
chemical equilibria, factors affecting equilibrium
concentration, pressure, temperature, effect of Colloidal state distinction among true solutions,
catalyst; Le -Chatelier's principle. colloids and suspensions, classification of colloids -
lyophilic, lyophobic; multi molecular, macromole-
Ionic equilibrium Weak and strong electrolytes,
cular and associated colloids (micelles),
ionization of electrolytes, various concepts of acids
preparation and properties of colloids Tyndall effect,
and bases (Arrhenius, Bronsted - Lowry and Lewis) and
Brownian movement, electrophoresis, dialysis,
their ionization, acid-base equilibria (including
coagulation and flocculation; Emulsions and their
multistage ionization) and ionization constants,
characteristics.
Section B INORGANIC CHEMISTRY
UNIT XI Classification of Elements and Group 14 Tendency for catenation; Structure,
Periodicity in Properties properties and uses of allotropes and oxides of
Periodic Law and Present Form of the Periodic Table, s, p, d carbon, silicon tetrachloride, silicates, zeolites and
and f Block Elements, Periodic Trends in Properties of silicones.
Elementsatomic and Ionic Radii, Ionization Enthalpy, Group 15 Properties and uses of nitrogen and
Electron Gain Enthalpy, Valence, Oxidation States and phosphorus; Allotrophic forms of phosphorus;
Chemical Reactivity. Preparation, properties, structure and uses of
ammonia nitric acid, phosphine and phosphorus
UNIT XII General Principles and Processes of halides,(PCl3, PCl5); Structures of oxides and
Isolation of Metals oxoacids of nitrogen and phosphorus.
Modes of occurrence of elements in nature, minerals, ores; Group 16 Preparation, properties, structures and
steps involved in the extraction of metals - concentration, uses of dioxygen and ozone; Allotropic forms of
reduction (chemical and electrolytic methods) and sulphur; Preparation, properties, structures and uses
refining with special reference to the extraction of Al, Cu, of sulphur dioxide, sulphuric acid (including its
Zn and Fe; Thermodynamic and electrochemical industrial preparation); Structures of oxoacids of
principles involved in the extraction of metals. sulphur.
UNIT XIII Hydrogen Group 17 Preparation, properties and uses of
Position of hydrogen in periodic table, isotopes, chlorine and hydrochloric acid; Trends in the acidic
preparation, properties and uses of hydrogen; physical nature of hydrogen halides; Structures of
and chemical properties of water and heavy water; Interhalogen compounds and oxides and oxoacids
Structure, preparation, reactions and uses of hydrogen of halogens.
peroxide; Classification of hydrides ionic, covalent and Group 18 Occurrence and uses of noble gases;
interstitial; Hydrogen as a fuel. Structures of fluorides and oxides of xenon.

UNIT XIV s - Block Elements UNIT XVI d–and f–Block Elements


(Alkali and Alkaline Earth Metals) Transition Elements General introduction, electronic
Group 1 and 2 Elements configuration, occurrence and characteristics,
general trends in properties of the first row
General introduction, electronic configuration and
transition elements - physical properties, ionization
general trends in physical and chemical properties of
enthalpy, oxidation states, atomic radii, colour,
elements, anomalous properties of the first element of
catalytic behaviour, magnetic properties, complex
each group, diagonal relationships.
formation, interstitial compounds, alloy formation;
Preparation and properties of some important Preparation, properties and uses of K2 Cr2 O7 and
compounds - sodium carbonate, sodium chloride, sodium KMnO4.
hydroxide and sodium hydrogen carbonate; Industrial
Inner Transition Elements
uses of lime, limestone, Plaster of Paris and cement;
Biological significance of Na, K, Mg and Ca. Lanthanoids - Electronic configuration, oxidation
states, chemical reactivity and lanthanoid
UNIT XV p - Block Elements contraction. Actinoids - Electronic configuration
Group 13 to Group 18 Elements and oxidation states.
General Introduction Electronic configuration and general UNIT XVII Coordination Compounds
trends in physical and chemical properties of elements Introduction to coordination compounds, Werner's
across the periods and down the groups; unique theory; ligands, coordination number, denticity,
behaviour of the first element in each group.Group wise chelation; IUPAC nomenclature of mononuclear
study of the p – block elements coordination compounds, isomerism; Bonding
Group 13 Preparation, properties and uses of boron and Valence bond approach and basic ideas of Crystal
aluminium; structure, properties and uses of borax, boric field theory, colour and magnetic properties;
acid, diborane, boron trifluoride, aluminium chloride and importance of coordination compounds (in
alums. qualitative analysis, extraction of metals and in
biological systems).
UNIT XVIII Environmental Chemistry Stratospheric pollution Formation and breakdown
of ozone, depletion of ozone layer - its mechanism
Environmental pollution Atmospheric, water
and effects.
and soil.
Water pollution Major pollutants such as,
Atmospheric pollution - Tropospheric and stratospheric.
pathogens, organic wastes and chemical pollutants
Tropospheric pollutants Gaseous pollutants Oxides of their harmful effects and prevention.
carbon, nitrogen and sulphur, hydrocarbons; their
Soil pollution Major pollutants such as: Pesticides
sources, harmful effects and prevention; Green house
(insecticides, herbicides and fungicides), their
effect and Global warming; Acid rain;
harmful effects and prevention.
Particulate pollutants Smoke, dust, smog, fumes, mist;
Strategies to control environmental pollution.
their sources, harmful effects and prevention.

Section C ORGANIC CHEMISTRY


UNIT XIX Purification & Characterisation UNIT XXI Hydrocarbons
of Organic Compounds Classification, isomerism, IUPAC nomenclature,
Purification Crystallisation, sublimation, distillation, general methods of preparation, properties and
differential extraction and chromatography principles reactions.
and their applications. Alkanes Conformations: Sawhorse and Newman
Qualitative analysis Detection of nitrogen, sulphur, projections (of ethane); Mechanism of
phosphorus and halogens. halogenation of alkanes.
Quantitative analysis (basic principles only) Estimation Alkenes Geometrical isomerism; Mechanism of
of carbon, hydrogen, nitrogen, halogens, sulphur, electrophilic addition: addition of hydrogen,
phosphorus. halogens, water, hydrogen halides (Markownikoff's and
Calculations of empirical formulae and molecular peroxide effect); Ozonolysis, oxidation, and
formulae; Numerical problems in organic quantitative polymerization.
analysis. Alkenes acidic character; addition of hydrogen,
halogens, water and hydrogen halides;
UNIT XX Some Basic Principles of
polymerization.
Organic Chemistry
Aromatic hydrocarbons Nomenclature, benzene
Tetravalency of carbon; Shapes of simple molecules
structure and aromaticity; Mechanism of electrophilic
hybridization (s and p); Classification of organic
substitution: halogenation, nitration, Friedel – Craft's
compounds based on functional groups:
alkylation and acylation, directive influence of
—C=C—,—C=C— and those containing halogens,
functional group in mono-substituted benzene.
oxygen, nitrogen and sulphur, Homologous series;
Isomerism - structural and stereoisomerism. UNIT XXII Organic Compounds
Nomenclature (Trivial and IUPAC) Containing Halogens
Covalent bond fission Homolytic and heterolytic free General methods of preparation, properties and
radicals, carbocations and carbanions; stability of reactions; Nature of C—X bond; Mechanisms of
carbocations and free radicals, electrophiles and substitution reactions.
nucleophiles. Uses/environmental effects of chloroform, iodoform,
Electronic displacement in a covalent bond Inductive freons and DDT.
effect, electromeric effect, resonance and
hyperconjugation. UNIT XXIII Organic Compounds
Common types of organic reactions Substitution, Containing Oxygen
addition, elimination and rearrangement. General methods of preparation, properties, reactions
and uses. Alcohols, Phenols and Ethers
Alcohols Identification of primary, secondary and only), denaturation of proteins, enzymes. Vitamins
tertiary alcohols; mechanism of dehydration. Classification and functions.
Phenols Acidic nature, electrophilic substitution Nucleic Acids Chemical constitution of DNA and RNA.
reactions: halogenation, nitration and sulphonation, Biological functions of Nucleic acids.
Reimer - Tiemann reaction.
UNIT XXVII Chemistry in Everyday Life
Ethers: Structure
Chemicals in medicines Analgesics, tranquilizers,
Aldehyde and Ketones Nature of carbonyl group; antiseptics, disinfectants, antimicrobials, antifertility
Nucleophilic addition to >C=O group, relative drugs, antibiotics, antacids, antihistamins - their
reactivities of aldehydes and ketones; Important meaning and common examples.
reactions such as - Nucleophilic addition reactions Chemicals in food Preservatives, artificial sweetening
(addition of HCN, NH3 and its derivatives), Grignard agents - common examples.
reagent; oxidation; reduction (Wolff Kishner and
Clemmensen); acidity of α - hydrogen, aldol Cleansing agents Soaps and detergents, cleansing
condensation, Cannizzaro reaction, Haloform reaction; action.
Chemical tests to distinguish between aldehydes and Unit XXVIII Principles Related to
Ketones.
Practical Chemistry
Carboxylic Acids Acidic strength & factors affecting it.
— Detection of extra elements (N, S, halogens) in
UNIT XXIV Organic Compounds organic compounds; Detection of the following
Containing Nitrogen functional groups: hydroxyl (alcoholic and
General methods of preparation, properties, reactions phenolic), carbonyl (aldehyde and ketone), carboxyl
and uses. and amino groups in organic compounds.

Amines Nomenclature, classification, structure basic Chemistry involved in the preparation of the
character and identification of primary, secondary and following
tertiary amines and their basic character. — Inorganic compounds Mohr's salt, potash alum.
Diazonium Salts Importance in synthetic organic — Organic compounds Acetanilide,
chemistry. p-nitroacetan ilide, aniline yellow, iodoform.
— Chemistry involved in the titrimetric excercises -
UNIT XXV Polymers
Acids bases and the use of indicators, oxali acid vs
General introduction and classification of polymers, KMnO4, Mohr's salt vs KMnO4.
general methods of polymerization-addition and
condensation, copolymerization; Natural and — Chemical principles involved in the qualitative salt
synthetic rubber and vulcanization; some important analysis
polymers with emphasis on their monomers and uses - — Cations — Pb2+ , Cu2+, Al3+, Fe3+, Zn2+, Ni2+, Ca2+, Ba2+ ,
polythene, nylon, polyester and bakelite. Mg2+ NH4+. Anions – CO32-, S2-, SO42-, NO2, NO3, Cl -, Br-,
I- (Insoluble salts excluded).
UNIT XXVI Biomolecules
— Chemical principles involved in the following
General introduction and importance of
experiments
biomolecules.
1. Enthalpy of solution of CuSO4
Carbohydrates Classification aldoses and ketoses;
monosaccharides (glucose and fructose), constituent 2. Enthalpy of neutralization of strong acid and
monosaccharides of oligosacchorides (sucrose, lactose, strong base.
maltose) and polysaccharides (starch, cellulose, 3. Preparation of lyophilic and lyophobic
glycogen). sols.
Proteins Elementary Idea of α-amino acids, peptide 4. Kinetic study of reaction of iodide ion with
bond, . polypeptides; proteins: primary, secondary, hydrogen peroxide at room temperature.
tertiary and quaternary structure (qualitative idea
JEE ADVANCED
PHYSICAL CHEMISTRY Chemical Kinetics Rates of chemical reactions, Order of
reactions, Rate constant, First order reactions,
General Topics Concept of atoms and molecules, Dalton's
Temperature dependence of rate constant (Arrhenius
atomic theory, Mole concept, Chemical formulae,
equation).
Balanced chemical equations, Calculations (based on
mole concept) involving common oxidation-reduction, Solid State Classification of solids, crystalline state, seven
neutralisation, and displacement reactions, Concentration crystal systems (cell parameters a, b, c), close packed
in terms of mole fraction, molarity, molality and normality. structure of solids (cubic), packing in fcc, bcc and hcp
lattices, Nearest neighbours, ionic radii, simple ionic
Gaseous and Liquid States Absolute scale of temperature,
compounds, point defects.
ideal gas equation, Deviation from ideality, van der Waals'
equation, Kinetic theory of gases, average, root mean Solutions Raoult's law, Molecular weight determination
square and most probable velocities and their relation from lowering of vapour pressure, elevation of boiling
with temperature, Law of partial pressures, Vapour point and depression of freezing point.
pressure, Diffusion of gases.
Surface Chemistry Elementary concepts of adsorption
Atomic Structure and Chemical Bonding Bohr model, (excluding adsorption isotherms), Colloids, types,
spectrum of hydrogen atom, quantum numbers, methods of preparation and general properties,
Wave-particle duality, de-Broglie hypothesis, Uncertainty Elementary ideas of emulsions, surfactants and micelles
principle, Qualitative quantum mechanical picture of (only definitions and examples).
hydrogen atom, shapes of s, p and d orbitals, Electronic
Nuclear Chemistry Radioactivity, isotopes and isobars,
configurations of elements (up to atomic number 36),
Properties of rays, Kinetics of radioactive decay (decay
Aufbau principle, Pauli's exclusion principle and Hund's
series excluded), carbon dating, Stability of nuclei with
rule, Orbital overlap and covalent bond; Hybridisation
respect to proton-neutron ratio, Brief discussion on fission
involving s, p and d orbitals only, Orbital energy diagrams
and fusion reactions.
for homonuclear diatomic species, Hydrogen bond,
Polarity in molecules, dipole moment (qualitative aspects
only), VSEPR model and shapes of molecules (linear, INORGANIC CHEMISTRY
angular, triangular, square planar, pyramidal, square Isolation/Preparation and Properties of the following Non-
pyramidal, trigonal bipyramidal, tetrahedral and metals Boron, silicon, nitrogen, phosphorus, oxygen,
octahedral). sulphur and halogens, Properties of allotropes of carbon
(only diamond and graphite), phosphorus and sulphur.
Energetics First law of thermodynamics, Internal energy,
Preparation and Properties of the following Compounds
work and heat, pressure-volume work, Enthalpy, Hess's
Oxides, peroxides, hydroxides, carbonates, bicarbonates,
law, Heat of reaction, fusion and vaporization, Second law
chlorides and sulphates of sodium, potassium,
of thermodynamics, Entropy, Free energy, Criterion of
magnesium and calcium, Boron, diborane, boric acid and
spontaneity.
borax, Aluminium, alumina, aluminium chloride and
Chemical Equilibrium Law of mass action, Equilibrium alums, Carbon, oxides and oxyacid (carbonic acid), Silicon,
constant, Le-Chatelier's principle (effect of concentration, silicones, silicates and silicon carbide, Nitrogen, oxides,
temperature and pressure), Significance of DG and DGo in oxyacids and ammonia, Phosphorus, oxides, oxyacids
chemical equilibrium, Solubility product, common ion (phosphorus acid, phosphoric acid) and phosphine,
effect, pH and buffer solutions, Acids and bases (Bronsted Oxygen, ozone and hydrogen peroxide, Sulphur,
and Lewis concepts), Hydrolysis of salts. hydrogen sulphide, oxides, sulphurous acid, sulphuric
Electrochemistry Electrochemical cells and cell reactions, acid and sodium thiosulphate, Halogens, hydrohalic acids,
Standard electrode potentials, Nernst equation and its oxides and oxyacids of chlorine, bleaching powder, Xenon
relation to DG, Electrochemical series, emf of galvanic fluorides.
cells, Faraday's laws of electrolysis, Electrolytic Transition Elements (3d series) Definition, general
conductance, specific, equivalent and molar conductivity, characteristics, oxidation states and their stabilities,
Kohlrausch's law, Concentration cells. colour (excluding the details of electronic transitions) and
calculation of spin-only magnetic moment; Coordination (boiling points, density and dipole moments), Acidity of
compounds: nomenclature of mononuclear coordination alkynes, Acid catalysed hydration of alkenes and alkynes
compounds, cis-trans and ionisation isomerisms, (excluding the stereochemistry of addition and
hybridization and geometries of mononuclear elimination), Reactions of alkenes with KMnO4 and ozone,
coordination compounds (linear, tetrahedral, square Reduction of alkenes and alkynes, Preparation of alkenes
planar and octahedral). and alkynes by elimination reactions, Electrophilic
Preparation and Properties of the following Compounds. addition reactions of alkenes with X2, HX, HOX and H2O
(X=halogen), Addition reactions of alkynes, Metal
Oxides and chlorides of tin and lead, Oxides, chlorides and
acetylides.
sulphates of Fe2+, Cu2+ and Zn2+, Potassium permanganate,
potassium dichromate, silver oxide, silver nitrate, silver Reactions of Benzene Structure and aromaticity,
thiosulphate. Electrophilic substitution reactions, halogenation,
nitration, sulphonation, Friedel-Crafts alkylation and
Ores and Minerals Commonly occurring ores and minerals
acylation Effect of o-, m- and p-directing groups in
of iron, copper, tin, lead, magnesium, aluminium, zinc and
monosubstituted benzenes.
silver.
Phenols Acidity, electrophilic substitution reactions
Extractive Metallurgy Chemical principles and reactions
(halogenation, nitration and sulphonation), Reimer-
only (industrial details excluded), Carbon reduction
Tiemann reaction, Kolbe reaction.
method (iron and tin), Self reduction method (copper and
lead), Electrolytic reduction method (magnesium and Characteristic Reactions of the following (including those
aluminium), Cyanide process (silver and gold). mentioned above) Alkyl halides, rearrangement reactions
of alkyl carbocation, Grignard reactions, nucleophilic
Principles of Qualitative Analysis Groups I to V (only Ag+,
substitution reactions, Alcohols, esterification,
Hg2+, Cu2+, Pb2+, Bi3+, Fe3+, Cr3+, Al3+, Ca2+, Ba2+, Zn2+, Mn2+ and
dehydration and oxidation, reaction with sodium,
Mg2+), Nitrate, halides (excluding fluoride), sulphate and
phosphorus halides, ZnCl2/concentrated HCl, conversion
sulphide.
of alcohols into aldehydes and ketones, Ethers,
Preparation by Williamson's Synthesis, Aldehydes and
ORGANIC CHEMISTRY Ketones, oxidation, reduction, oxime and hydrazone
Concepts Hybridisation of carbon, Sigma and pi-bonds, formation, aldol condensation, Perkin reaction, Cannizzaro
Shapes of simple organic molecules, Structural and reaction, haloform reaction and nucleophilic addition
geometrical isomerism, Optical isomerism of compounds reactions (Grignard addition), Carboxylic acids, formation
containing up to two asymmetric centres, (R,S and E,Z of esters, acid chlorides and amides, ester hydrolysis.
nomenclature excluded), IUPAC nomenclature of simple Amines, basicity of substituted anilines and aliphatic
organic compounds (only hydrocarbons, mono-functional amines, preparation from nitro compounds, reaction with
and bi-functional compounds), Conformations of ethane nitrous acid, azo coupling reaction of diazonium salts of
and butane (Newman projections), Resonance and aromatic amines, Sandmeyer and related reactions of
hyperconjugation, Keto-enol tautomerism, Determination diazonium salts, carbylamine reaction, Haloarenes,
of empirical and molecular formulae of simple nucleophilic aromatic substitution in haloarenes and
compounds (only combustion method), Hydrogen bonds, substituted haloarenes (excluding Benzyne mechanism
definition and their effects on physical properties of and Cine substitution).
alcohols and carboxylic acids, Inductive and resonance Carbohydrates Classification, mono and disaccharides
effects on acidity and basicity of organic acids and bases, (glucose and sucrose), Oxidation, reduction, glycoside
Polarity and inductive effects in alkyl halides, Reactive formation and hydrolysis of sucrose.
intermediates produced during homolytic and heterolytic
Amino Acids and Peptides General structure (only primary
bond cleavage, Formation, structure and stability of
structure for peptides) and physical properties.
carbocations, carbanions and free radicals.
Properties and Uses of Some Important Polymers Natural
Preparation, Properties and Reactions of Alkanes
rubber, cellulose, nylon, teflon and PVC.
Homologous series, physical properties of alkanes
(melting points, boiling points and density), Combustion Practical Organic Chemistry Detection of elements (N, S,
and halogenation of alkanes, Preparation of alkanes by halogens), Detection and identification of the following
Wurtz reaction and decarboxylation reactions. functional groups, hydroxyl (alcoholic and phenolic),
carbonyl (aldehyde and ketone), carboxyl, amino and
Preparation, Properties and Reactions of Alkenes and
nitro, Chemical methods of separation of mono-functional
Alkynes Physical properties of alkenes and alkynes
organic compounds from binary mixtures.
1
Some Basic Concepts
of Chemistry
6. The percentage composition of carbon by mole in methane is
Topic 1 Mole Concept (2019 Main, 8 April II)
(a) 75% (b) 20% (c) 25% (d) 80%
Objective Questions I (Only one correct option)
7. 8 g of NaOH is dissolved in 18 g of H 2O. Mole fraction of
1. 5 moles of AB2 weight 125 ´ 10-3 kg and 10 moles of A2 B2
NaOH in solution and molality (in mol kg- 1 ) of the solution
weight 300 ´ 10-3 kg. The molar mass of A ( M A ) and molar
respectively are (2019 Main, 12 Jan II)
mass of B ( M B ) in kg mol -1 are (2019 Main, 12 April I) (a) 0.2, 11.11 (b) 0.167, 22.20
(a) M A = 10 ´ 10-3 and M B = 5 ´ 10-3 (c) 0.2, 22.20 (d) 0.167, 11.11
(b) M A = 50 ´ 10-3 and M B = 25 ´ 10-3 8. The volume strength of 1 M H 2O2 is
-3 -3
(c) M A = 25 ´ 10 and M B = 50 ´ 10 (Molar mass of H 2O2 = 34 g mol- 1 ) (2019 Main, 12 Jan II)
(d) M A = 5 ´ 10-3 and M B = 10 ´ 10-3 (a) 16.8 (b) 22.4 (c) 11.35 (d) 5.6
2. The minimum amount of O2 ( g ) consumed per gram of 9. The amount of sugar (C12H 22O11 ) required to prepare 2 L of
reactant is for the reaction (Given atomic mass : Fe = 56, its 0.1 M aqueous solution is (2019 Main, 10 Jan II)
O =16, Mg = 24, P =31, C =12, H =1) (2019 Main, 10 April II) (a) 17.1 g (b) 68.4 g (c) 136.8 g (d) 34.2 g
(a) C3H 8 ( g ) + 5O2 ( g ) ¾® 3CO2 ( g ) + 4H 2O( l ) 10. For the following reaction, the mass of water produced from
(b) P4 ( s ) + 5O2 ( g ) ¾® P4O10 ( s ) 445 g of C57 H110 O6 is :
(c) 4Fe( s ) + 3O2 ( g ) ¾® 2Fe2O3 ( s ) 2C57 H110 O6 ( s ) + 163O2 ( g ) ® 114CO2 ( g ) + 110 H2 O ( l )
(d) 2Mg( s ) + O2 ( g ) ¾® 2MgO( s ) (2019 Main, 9 Jan II)

3. At 300 K and 1 atmospheric pressure, (a) 490 g (b) 495 g (c) 445 g (d) 890 g
10 mL of a hydrocarbon required 55 mL of O2 for complete 11. A solution of sodium sulphate contains 92 g of Na + ions per
combustion and 40 mL of CO2 is formed. The formula of the kilogram of water. The molality of Na + ions in that solution
hydrocarbon is (2019 Main, 10 April I) in mol kg-1 is (2019 Main, 9 Jan I)
(a) C4H7Cl (b) C4H 6 (c) C4H10 (d) C4H 8
(a) 16 (b) 4 (c) 132 (d) 8
4. 10 mL of 1 mM surfactant solution forms a monolayer
12. The most abundant elements by mass in the body of a healthy
covering 0.24 cm 2 on a polar substrate. If the polar head is human adult are oxygen (61.4%), carbon (22.9%), hydrogen
approximated as a cube, what is its edge length? (10.0 %), and nitrogen (2.6%). The weight which a 75 kg
(2019 Main, 9 April II)
person would gain if all 1 H atoms are replaced by 2 H atoms is
(a) 2.0 pm (b) 0.1 nm (c) 1.0 pm (d) 2.0 nm (2017 JEE Main)
5. For a reaction, (a) 15 kg (b) 37.5 kg
N 2 ( g ) + 3H 2 ( g ) ¾® 2NH 3 ( g ), identify dihydrogen (H 2 ) (c) 7.5 kg (d) 10 kg
as a limiting reagent in the following reaction mixtures. 13. 1 g of a carbonate (M 2 CO3 ) on treatment with excess HCl
(2019 Main, 9 April I)
produces 0.01186 mole of CO2 . The molar mass of M 2 CO3
(a) 56 g of N 2 + 10 g of H 2 (b) 35 g of N 2 + 8 g of H 2 in g mol -1 is (2017 JEE Main)
(c) 14 g of N 2 + 4 g of H 2 (d) 28 g of N 2 + 6 g of H 2 (a) 1186 (b) 84.3 (c) 118.6 (d) 11.86
2 Some Basic Concepts of Chemistry

14. At 300 K and 1 atm, 15 mL of a gaseous hydrocarbon 24. The normality of 0.3 M phosphorus acid (H3PO3) is
requires 375 mL air containing 20% O2 by volume for (1999, 2M)
complete combustion. After combustion, the gases occupy (a) 0.1 (b) 0.9 (c) 0.3 (d) 0.6
330 mL. Assuming that the water formed is in liquid form 25. In which mode of expression, the concentration of a solution
and the volumes were measured at the same temperature and remains independent of temperature? (1988, 1M)
pressure, the formula of the hydrocarbon is (2016 JEE Main) (a) Molarity (b) Normality (c) Formality (d) Molality
(a) C3 H8 (b) C4 H8 (c) C4 H10 (d) C3 H6 26. A molal solution is one that contains one mole of solute in
15. The molecular formula of a commercial resin used for (1986, 1M)
exchanging ions in water softening is C8 H7 SO3 Na (a) 1000 g of solvent (b) 1.0 L of solvent
(molecular weight = 206). What would be the maximum (c) 1.0 L of solution (d) 22.4 L of solution
uptake of Ca 2+ ions by the resin when expressed in mole per 27. If 0.50 mole of BaCl 2 is mixed with 0.20 mole of Na 3 PO4 ,
gram resin? (2015 JEE Main) the maximum number of moles of Ba 3 (PO4 )2 that can be
1 1 2 1 formed is (1981, 1M)
(a) (b) (c) (d)
103 206 309 412 (a) 0.70 (b) 0.50 (c) 0.20 (d) 0.10
16. 3 g of activated charcoal was added to 50 mL of acetic acid 28. 2.76 g of silver carbonate on being strongly heated yields a
solution (0.06 N) in a flask. After an hour it was filtered and residue weighing (1979, 1M)
the strength of the filtrate was found to be 0.042 N. The (a) 2.16 g (b) 2.48 g (c) 2.32 g (d) 2.64 g
amount of acetic acid adsorbed (per gram of charcoal) is 29. When the same amount of zinc is treated separately with
(2015 JEE Main) excess of sulphuric acid and excess of sodium hydroxide, the
(a) 18 mg (b) 36 mg (c) 42 mg (d) 54 mg ratio of volumes of hydrogen evolved is (1979, 1M)
17. The ratio mass of oxygen and nitrogen of a particular gaseous (a) 1 : 1 (b) 1 : 2 (c) 2 : 1 (d) 9 : 4
mixture is 1 : 4. The ratio of number of their molecule is 30. The largest number of molecules is in (1979, 1M)
(2014 Main)
(a) 1 : 4 (b) 7 : 32 (c) 1 : 8 (d) 3 : 16 (a) 36 g of water
(b) 28 g of CO
18. The molarity of a solution obtained by mixing 750 mL of
(c) 46 g of ethyl alcohol
0.5 M HCl with 250 mL of 2 M HCl will be (2013 Main)
(d) 54 g of nitrogen pentaoxide (N2 O5 )
(a) 0.875 M (b) 1.00 M (c) 1.75 M (d) 0.0975M
31. The total number of electrons in one molecule of carbon
19. Dissolving 120 g of urea (mol. wt. 60) in 1000 g of water dioxide is (1979, 1M)
gave a solution of density 1.15 g/mL. The molarity of the (a) 22 (b) 44 (c) 66 (d) 88
solution is (2011)
32. A gaseous mixture contains oxygen and nitrogen in the ratio
(a) 1.78 M (b) 2.00 M (c) 2.05 M (d) 2.22 M
of 1:4 by weight. Therefore, the ratio of their number of
20. Given that the abundances of isotopes 54 Fe, 56 Fe
and 57 Fe molecules is (1979, 1M)
are 5%, 90% and 5%, respectively, the atomic mass of Fe is (a) 1 : 4 (b) 1 : 8 (c) 7 : 32 (d) 3 : 16
(2009)
(a) 55.85 (b) 55.95 Numerical Value Based Questions
(c) 55.75 (d) 56.05 33. Galena (an ore) is partially oxidised by passing air through it
21. Mixture X = 0.02 mole of [Co(NH3 )5 SO4 ]Br and 0.02 mole at high temperature. After some time, the passage of air is
of [Co(NH3 )5 Br]SO4 was prepared in 2 L solution. stopped, but the heating is continued in a closed furnace such
1 L of mixture X + excess of AgNO3 solution ¾® Y that the content undergo self-reduction. The weight (in kg) of
Pb produced per kg of O 2 consumed is ……… .
1 L of mixture X + excess of BaCl 2 solution ¾® Z
(Atomic weights in g mol -1 : O = 16, S = 32, Pb = 207)
Number of moles of Y and Z are (2003, 1M)
(2018 Adv.)
(a) 0.01, 0.01 (b) 0.02, 0.01 34. To measure the quantity of MnCl 2 dissolved in an aqueous
(c) 0.01, 0.02 (d) 0.02, 0.02 solution, it was completely converted to KMnO4 using the
22. Which has maximum number of atoms? (2003, 1M) reaction,
(a) 24 g of C (12) (b) 56 g of Fe (56) MnCl 2 + K 2 S2 O8 + H2 O ¾® KMnO4 + H2 SO4 + HCl
(c) 27 g of Al (27) (d) 108 g of Ag (108) (equation not balanced).
23. How many moles of electron weighs 1 kg? Few drops of concentrated HCl were added to this solution
1 and gently warmed. Further, oxalic acid (225 mg) was added
(a) 6.023 ´ 1023 (b) ´ 1031 (2002, 3M)
in portions till the colour of the permanganate ion
9.108
6.023 1 disappeared. The quantity of MnCl 2 (in mg) present in the
(c) ´ 1054 (d) ´ 108 initial solution is ……… .
9.108 9.108 ´ 6.023
(Atomic weights in g mol -1 : Mn = 55, Cl = 35.5) (2018 Adv.)
Some Basic Concepts of Chemistry 3

35. In the following reaction sequence, the amount of D (in 44. In a solution of 100 mL 0.5 M acetic acid, one gram of active
gram) formed from 10 moles of acetophenone is ……. charcoal is added, which adsorbs acetic acid. It is found that the
concentration of acetic acid becomes 0.49 M. If surface area of
(Atomic weights in g mol -1 : H = 1, C = 12, N = 14,
charcoal is 3.01 ´ 102 m2 , calculate the area occupied by single
O = 16, Br = 80. The yield (%) corresponding to the acetic acid molecule on surface of charcoal. (2003)
product in each step is given in the parenthesis)
45. Find the molarity of water. Given: r = 1000 kg/m3 (2003)
O
46. A plant virus is found to consist of uniform cylindrical particles
NaOBr NH3, D Br2/KOH of 150 Å in diameter and 5000 Å long. The specific volume of
A B C
H3O+
(60%) (50%) (50%)
the virus is 0.75 cm 3 /g. If the virus is considered to be a single
particle, find its molar mass. (1999, 3M)
Br2(3 equivalent )
AcOH
D 47. 8.0575 ´ 10-2 kg of Glauber’s salt is dissolved in water to
(100%)
obtain 1 dm 3 of solution of density 1077.2 kg m -3 . Calculate
(2018 Adv.)
the molality, molarity and mole fraction of Na 2 SO4 in solution.
(1994, 3M)
Fill in the Blanks 48. A is a binary compound of a univalent metal. 1.422 g of A reacts
36. The weight of 1 ´ 1022 molecules of CuSO4 × 5H2 O is completely with 0.321 g of sulphur in an evacuated and sealed
tube to give 1.743 g of a white crystalline solid B, that forms a
…………. . (1991, 1M)
hydrated double salt, C with Al 2 (SO4 )3 . Identify A, B and C.
37. 3.0 g of a salt of molecular weight 30 is dissolved in 250 g (1994, 2M)
water. The molarity of the solution is ………. (1983, 1M)
49. Upon mixing 45.0 mL 0.25 M lead nitrate solution with
38. The total number of electrons present in 18 mL of water is 25.0 mL of a 0.10 M chromic sulphate solution, precipitation of
…………. . (1980, 1M) lead sulphate takes place. How many moles of lead sulphate are
39. The modern atomic mass unit is based on the mass of formed? Also calculate the molar concentrations of species left
…………. . (1980, 1M)
behind in the final solution. Assume that lead sulphate is
completely insoluble. (1993, 3M)

Integer Answer Type Questions 50. Calculate the molality of 1.0 L solution of 93% H2 SO4 ,
(weight/volume). The density of the solution is 1.84 g/mL.
40. The mole fraction of a solute in a solution is 0.1. At 298 K, (1990, 1M)
molarity of this solution is the same as its molality. Density
51. A solid mixture (5.0 g) consisting of lead nitrate and sodium
of this solution at 298 K is 2.0 g cm-3 . The ratio of the
nitrate was heated below 600°C until the weight of the residue
æ m ö
molecular weights of the solute and solvent, çç solute ÷÷ is ... was constant. If the loss in weight is 28.0 per cent, find the
è msolvent ø amount of lead nitrate and sodium nitrate in the mixture.
. (1990, 4M)
(2016 Adv.) 52. n-butane is produced by monobromination of ethane followed
41. A compound H2 X with molar weight of 80 g is dissolved by Wurtz’s reaction.Calculate volume of ethane at NTP
-1 required to produce 55 g n-butane, if the bromination takes
in a solvent having density of 0.4 g mL . Assuming no
change in volume upon dissolution, the molality of a 3.2 place with 90% yield and the Wurtz’s reaction with 85% yield.
(1989, 3M)
molar solution is (2014 Adv.)
53. A sugar syrup of weight 214.2 g contains 34.2 g of sugar
42. 29.2% (w/W ) HCl stock solution has density of 1.25g mL
-1 (C12 H22 O11 ). Calculate (i) molal concentration and (ii) mole
. The molecular weight of HCl is 36.5 g mol - 1 . The fraction of sugar in syrup. (1988, 2M)
volume (mL) of stock solution required to prepare a 200
mL solution 0.4 M HCl is (2012)
54. An unknown compound of carbon, hydrogen and oxygen
contains 69.77% C and 11.63% H and has a molecular weight
of 86. It does not reduces Fehling’s solution but forms a
Subjective Questions bisulphate addition compound and gives a positive iodoform
43. 20% surface sites have adsorbed N2 . On heating N2 gas test. What is the possible structure(s) of unknown compound?
(1987, 3M)
evolved from sites and were collected at 0.001 atm and 298
K in a container of volume is 2.46 cm 3 . Density of surface 55. The density of a 3 M sodium thiosulphate solution ( Na 2 S2 O3 )
sites is 6.023 ´ 1014 /cm 2 and surface area is 1000 cm 2 , find is 1.25 g per mL. Calculate (i) the percentage by weight of
sodium thiosulphate (ii) the mole fraction of sodium
out the number of surface sites occupied per molecule of
thiosulphate and (iii) the molalities of Na + and S2 O2-
3 ions.
N2 . (2005, 3M)
(1983, 5M)
4 Some Basic Concepts of Chemistry

56. (a) 1.0 L of a mixture of CO and CO2 is taken. This mixture 58. In the analysis of 0.5 g sample of feldspar, a mixture of
is passed through a tube containing red hot charcoal. The chlorides of sodium and potassium is obtained, which weighs
volume now becomes 1.6 L. The volumes are measured 0.1180 g. Subsequent treatment of the mixed chlorides with
under the same conditions. Find the composition of silver nitrate gives 0.2451 g of silver chloride. What is the
mixture by volume. percentage of sodium oxide and potassium oxide in the
(b) A compound contains 28 per cent of nitrogen and sample? (1979, 5M)
72 per cent of a metal by weight. 3 atoms of metal 59. The vapour density (hydrogen = 1) of a mixture consisting of
combine with 2 atoms of nitrogen. Find the atomic NO2 and N2 O4 is 38.3 at 26.7°C. Calculate the number of
weight of metal. (1980, 5M)
moles of NO2 in 100 g of the mixture. (1979, 5M)
57. 5.00 mL of a gas containing only carbon and hydrogen were 60. Accounts for the following. Limit your answer to two
mixed with an excess of oxygen (30 mL) and the mixture sentences, “Atomic weights of most of the elements are
exploded by means of electric spark. After explosion, the fractional”. (1979, 1M)
volume of the mixed gases remaining was 25 mL.
61. Naturally occurring boron consists of two isotopes whose
On adding a concentrated solution of KOH, the volume atomic weights are 10.01 and 11.01. The atomic weight of
further diminished to 15 mL, the residual gas being pure natural boron is 10.81. Calculate the percentage of each
oxygen. All volumes have been reduced to NTP. Calculate isotope in natural boron. (1978, 2M)
the molecular formula of the hydrocarbon gas. (1979, 3M)

Topic 2 Equivalent Concept, Neutralisation and Redox Titration


Objective Questions I (Only one correct option) of the watch glass is 10 cm. What is the height of the
1. An example of a disproportionation reaction is monolayer? [Density of fatty acid = 0.9 g cm -3 ; p = 3]
(2019 Main, 12 April I) (2019 Main, 8 April II)
-6 -4
(a) 2MnO-4 - +
+ 10I + 16H ¾® 2Mn 2+
+5I2 + 8H 2O (a) 10 m (b) 10 m
(b) 2NaBr + Cl2 ¾® 2NaCl + Br2 (c) 10-8 m (d) 10-2 m
(c) 2KMnO4 ¾® K 2MnO4 + MnO2 + O2 4. In order to oxidise a mixture of one mole of each of FeC2 O4 ,
(d) 2CuBr ¾® CuBr2 + Cu Fe2 (C2 O4 )3 , FeSO4 and Fe2 (SO4 )3 in acidic medium, the
number of moles of KMnO4 required is (2019 Main, 8 April I)
2. In an acid-base titration, 0.1 M HCl solution was added to
(a) 2 (b) 1 (c) 3 (d) 1.5
the NaOH solution of unknown strength. Which of the
following correctly shows the change of pH of the titration 5. 100 mL of a water sample contains 0.81 g of calcium
mixture in this experiment? (2019 Main, 9 April II) bicarbonate and 0.73 g of magnesium bicarbonate. The
hardness of this water sample expressed in terms of
equivalents of CaCO3 is (molar mass of calcium bicarbonate
is 162 g mol-1 and magnesium bicarbonate is 146 g mol-1 )
pH pH (2019 Main, 8 April I)
(a) 5,000 ppm (b) 1,000 ppm
(c) 100 ppm (d) 10,000 ppm
V(mL) V(mL)
(A) (B) 6. 50 mL of 0.5 M oxalic acid is needed to neutralise 25 mL of
sodium hydroxide solution. The amount of NaOH in 50 mL
of the given sodium hydroxide solution is
(2019 Main, 12 Jan I)
pH pH (a) 40 g (b) 80 g (c) 20 g (d) 10 g
7. 25 mL of the given HCl solution requires 30 mL of 0.1 M
V(mL) V(mL)
sodium carbonate solution. What is the volume of this HCl
(C) (D)
solution required to titrate 30 mL of 0.2 M aqueous NaOH
(2019 Main, 9 April II)
solution? (2019 Main, 11 Jan II)
(a) (D) (b) (A) (a) 75 mL (b) 25 mL (c) 12.5 mL (d) 50 mL
(c) (B) (d) (C)
8. In the reaction of oxalate with permanganate in acidic
3. 0.27 g of a long chain fatty acid was dissolved in 100 cm 3 of medium, the number of electrons involved in producing one
hexane. 10 mL of this solution was added dropwise to the molecule of CO2 is (2019 Main, 10 Jan II)
surface of water in a round watch glass. Hexane evaporates (a) 2 (b) 5 (c) 1 (d) 10
and a monolayer is formed. The distance from edge to centre
Some Basic Concepts of Chemistry 5

9. The ratio of mass per cent of C and H of an organic 18. The number of moles of KMnO 4 that will be needed to react
compound (Cx H y O z ) is 6 : 1. If one molecule of the above completely with one mole of ferrous oxalate in acidic
compound (Cx H y O z ) contains half as much oxygen as medium is (1997)
required to burn one molecule of compound Cx H y 2 3 4
completely to CO2 and H2 O. The empirical formula of (a) (b) (c) (d) 1
5 5 5
compound Cx H y O z is (2018 Main)
(a) C3 H6 O3 (b) C2 H4 O (c) C3 H4 O2 (d) C2 H4 O3 19. The number of moles of KMnO 4 that will be needed to react
with one mole of sulphite ion in acidic solution is
10. An alkali is titrated against an acid with methyl orange as (1997)
indicator, which of the following is a correct combination? 2 3 4
(2018 Main) (a) (b) (c) (d) 1
5 5 5
Base Acid End point
20. For the redox reaction
(a) Weak Strong Colourless to pink
(b) Strong Strong Pinkish red to yellow MnO-4 + C2 O42 - + H+ ¾® Mn 2+ + CO2 + H2 O
(c) Weak Strong Yellow to pinkish red The correct coefficients of the reactants for the balanced
(d) Strong Strong Pink to colourless reaction are

11. From the following statements regarding H2 O2 choose the MnO-4 C2 O24 - H+ (1992)

incorrect statement. (2015 Main)


(a) 2 5 16
(b) 16 5 2
(a) It can act only as an oxidising agent
(c) 5 16 2
(b) It decomposed on exposure to light
(d) 2 16 5
(c) It has to be stored in plastic or wax lined glass bottles in
dark 21. The volume strength of 1.5 N H2 O2 is (1990, 1M)
(d) It has to be kept away from dust (a) 4.8 (b) 8.4 (c) 3.0 (d) 8.0
12. Consider a titration of potassium dichromate solution with 22. The oxidation number of phosphorus in Ba(H2 PO2 )2 is
acidified Mohr’s salt solution using diphenylamine as (a) +3 (b) +2 (1988)
indicator. The number of moles of Mohr's salt required per (c) +1 (d) –1
mole of dichromate is (2007, 3M)
23. The equivalent weight of MnSO 4 is half of its molecular
(a) 3 (b) 4 (c) 5 (d) 6
weight, when it converts to (1988, 1M)
13. In the standardisation of Na 2 S2 O3 using K 2 Cr2 O7 by
(a) Mn 2 O3 (b) MnO2
iodometry, the equivalent weight of K 2 Cr2 O7 is (2001, 1M)
(a) (molecular weight)/2 (b) (molecular weight)/6 (c) MnO-4 (d) MnO2-
4
(c) (molecular weight)/3 (d) same as molecular weight
14. The reaction, 3ClO - (aq) ¾® ClO3– (aq) + 2Cl - (aq) is an
Objective Question II (More than one correct option)
example of (2001) 24. For the reaction, I- + ClO-3 + H2 SO4 ¾® Cl - + HSO4- + I2
(a) oxidation reaction the correct statement(s) in the balanced equation is/are
(b) reduction reaction (a) stoichiometric coefficient of HSO-4 is 6 (2014 Adv)
(c) disproportionation reaction (b) iodide is oxidised
(d) decomposition reaction (c) sulphur is reduced
15. An aqueous solution of 6.3 g oxalic acid dihydrate is made up (d) H2 O is one of the products
to 250 mL. The volume of 0.1 N NaOH required to
completely neutralise 10 mL of this solution is (2001, 1M) Numerical Value Based Question
(a) 40 mL (b) 20 mL (c) 10 mL (d) 4 mL 25. The ammonia prepared by treating ammonium sulphate with
16. Among the following, the species in which the oxidation calcium hydroxide is completely used by NiCl 2 × 6H2 O to
number of an element is + 6 (2000) form a stable coordination compound. Assume that both the
reactions are 100% complete. If 1584 g of ammonium
(a) MnO-4 (b) Cr(CN)3-
6
sulphate and 952 g of NiCl 2 × 6H2 O are used in the
(c) NiF62- (d) CrO2 Cl 2 preparation, the combined weight (in grams) of gypsum
17. The oxidation number of sulphur in S 8 , S 2 F 2 , H 2 S and the nickel-ammonia coordination compound thus
produced is____
respectively, are (1999)
(a) 0, +1 and –2 (b) +2, +1 and –2 (Atomic weights in g mol -1 : H = 1, N = 14, O = 16, S = 32,
(c) 0, +1 and +2 (d) –2, +1 and –2 Cl = 35.5, Ca = 40, Ni = 59) (2018 Adv.)
6 Some Basic Concepts of Chemistry

Assertion and Reason 35. To a 25 mL H2 O2 solution, excess of acidified solution of


Read the following questions and answer as per the direction potassium iodide was added. The iodine liberated required
given below : 20 mL of 0.3 N sodium thiosulphate solution. Calculate the
(a) Statement I is true; Statement II is true; Statement II is the volume strength of H2 O2 solution. (1997, 5M)
correct explanation of Statement I. 36. A 3.00 g sample containing Fe3 O4 , Fe2 O3 and an inert
(b) Statement I is true; Statement II is true; Statement II is not impure substance, is treated with excess of KI solution in
the correct explanation of Statement I. presence of dilute H2 SO4 . The entire iron is converted into
(c) Statement I is true; Statement II is false. Fe2+ along with the liberation of iodine. The resulting
(d) Statement I is false; Statement II is true. solution is diluted to 100 mL . A 20 mL of the diluted
solution requires 11.0 mL of 0.5 M Na 2 S2 O3 solution to
26. Statement I In the titration of Na 2 CO3 with HCl using
reduce the iodine present. A 50 mL of the dilute solution,
methyl orange indicator, the volume required at the
after complete extraction of the iodine required 12.80 mL of
equivalence point is twice that of the acid required using 0.25 M KMnO4 solution in dilute H2 SO4 medium for the
phenolphthalein indicator.
oxidation of Fe2+ . Calculate the percentage of Fe2 O3 and
Statement II Two moles of HCl are required for the Fe3 O4 in the original sample. (1996, 5M)
complete neutralisation of one mole of Na 2 CO3 . (1991, 2M)
37. A 20.0 cm 3 mixture of CO, CH4 and He gases is exploded by
Fill in the Blanks an electric discharge at room temperature with excess of
27. The compound YBa 2 Cu 3 O7 , which shows super oxygen. The volume contraction is found to be 13.0 cm 3 .
conductivity, has copper in oxidation state ………. Assume A further contraction of 14.0 cm 3 occurs when the residual
that the rare earth element yttrium is in its usual + 3 oxidation gas is treated with KOH solution. Find out the composition
state. (1994, 1M) of the gaseous mixture in terms of volume percentage.
(1995, 4M)
Integer Answer Type Questions 38. A 5.0 cm 3 solution of H2 O2 liberates 0.508 g of iodine from
28. The difference in the oxidation numbers of the two types of an acidified KI solution. Calculate the strength of H2 O2
sulphur atoms in Na 2 S4 O6 is (2011) solution in terms of volume strength at STP. (1995, 3M)
29. Among the following, the number of elements showing only 39. One gram of commercial AgNO3 is dissolved in 50 mL of
one non-zero oxidation state is O, Cl, F, N, P, Sn, Tl, Na, Ti
(2010)
water. It is treated with 50 mL of a KI solution. The silver
iodide thus precipitated is filtered off. Excess of KI in the
30. A student performs a titration with different burettes and filtrate is titrated with (M/10) KIO 3 solution in presence of
finds titrate values of 25.2 mL, 25.25 mL, and 25.0 mL. The 6 M HCl till all I- ions are converted into ICl. It requires
number of significant figures in the average titrate value is 50 mL of (M/10) KIO 3 solution, 20 mL of the same stock
(2010)
solution of KI requires 30 mL of (M/10) KIO3 under similar
Subjective Questions conditions. Calculate the percentage of AgNO3 in the
sample.
31. Calculate the amount of calcium oxide required when it
Reaction KIO3 + 2KI + 6HCl ¾® 3ICl + 3KCl + 3H2 O
reacts with 852 g of P4 O10 . (2005, 2M)
(1992, 4M)
32. Hydrogen peroxide solution (20 mL) reacts quantitatively
with a solution of KMnO4 (20 mL) acidified with dilute
40. A 2.0 g sample of a mixture containing sodium carbonate,
sodium bicarbonate and sodium sulphate is gently heated till
H2 SO4 . The same volume of the KMnO4 solution is just
the evolution of CO2 ceases. The volume of CO2 at 750 mm
decolourised by 10 mL of MnSO4 in neutral medium
Hg pressure and at 298 K is measured to be 123.9 mL. A 1.5 g
simultaneously forming a dark brown precipitate of hydrated
of the same sample requires 150 mL of (M/10) HCl for
MnO2 . The brown precipitate is dissolved in 10 mL of 0.2 M
complete neutralisation. Calculate the percentage
sodium oxalate under boiling condition in the presence of
composition of the components of the mixture. (1992, 5M)
dilute H2 SO4 . Write the balanced equations involved in the
reactions and calculate the molarity of H2 O2 . (2001) 41. A 1.0 g sample of Fe2 O3 solid of 55.2% purity is dissolved in
33. How many millilitres of 0.5 M H2 SO4 are needed to dissolve acid and reduced by heating the solution with zinc dust. The
resultant solution is cooled and made up to 100.0 mL. An
0.5 g of copper (II) carbonate? (1999, 3M)
aliquot of 25.0 mL of this solution requires for titration.
34. An aqueous solution containing 0.10 g KIO3 Calculate the number of electrons taken up by the oxidant in
(formula weight = 214.0) was treated with an excess of KI the reaction of the above titration. (1991, 4M)
solution. The solution was acidified with HCl. The liberated
I2 consumed 45.0 mL of thiosulphate solution decolourise 42. A solution of 0.2 g of a compound containing Cu 2+ and
the blue starch-iodine complex. Calculate the molarity of the C2 O42- ions on titration with 0.02 M KMnO4 in presence of
sodium thiosulphate solution. (1998, 5M) H2 SO4 consumes 22.6 mL of the oxidant. The resultant
Some Basic Concepts of Chemistry 7

solution is neutralised with Na 2 CO3 , acidified with dilute balanced equations for all the three half reaction. Find out the
acetic acid and treated with excess KI. The liberated iodine volume of 1M K 2 Cr2 O7 consumed, if the same volume of the
requires 11.3 mL of 0.05 M Na 2 S2 O3 solution for complete reducing agent is titrated in acid medium. (1989, 5M)
reduction. Find out the mole ratio of Cu 2+ to C2 O42- in the 46. A sample of hydrazine sulphate ( N2 H 6 SO4 ) was dissolved in
compound. Write down the balanced redox reactions 100 mL of water, 10 mL of this solution was reacted with
involved in the above titrations. (1991, 5M) excess of ferric chloride solution and warmed to complete
43. A mixture of H2 C2 O4 (oxalic acid) and NaHC2 O4 weighing the reaction. Ferrous ion formed was estimated and it,
required 20 mL of M/50 potassium permanganate solution.
2.02 g was dissolved in water and the solution made up to one
Estimate the amount of hydrazine sulphate in one litre of the
litre. Ten millilitres of the solution required 3.0 mL of 0.1 N
solution.
sodium hydroxide solution for complete neutralisation. In
another experiment, 10.0 mL of the same solution, in hot Reaction 4Fe3+ + N2 H4 ¾® N2 + 4Fe2+ + 4H+
dilute sulphuric acid medium, required 4.0 mL of 0.1 N MnO4- + 5Fe2+ + 8H+ ¾® Mn 2+ + 5Fe3+ + 4H2 O
potassium permanganate solution for complete reaction. (1988, 3M)
Calculate the amount of H2 C2 O4 and NaHC2 O4 in the 47. 5 mL of 8 N nitric acid, 4.8 mL of 5 N hydrochloric acid and
mixture. (1990, 5M) a certain volume of 17 M sulphuric acid are mixed together
44. An organic compound X on analysis gives 24.24 per cent and made up to 2 L. 30 mL of this acid mixture exactly
carbon and 4.04 per cent hydrogen. Further, sodium extract neutralise 42.9 mL of sodium carbonate solution containing
of 1.0 g of X gives 2.90 g of silver chloride with acidified one gram of Na 2 CO3 × 10H2 O in 100 mL of water. Calculate
silver nitrate solution. The compound X may be represented the amount in gram of the sulphate ions in solution.
by two isomeric structures Y and Z. Y on treatment with (1985, 4M)
aqueous potassium hydroxide solution gives a dihydroxy 48. 2.68 ´ 10-3 moles of a solution containing an ion A n+ require
compound while Z on similar treatment gives ethanal. Find 1.61 ´ 10-3 moles of MnO-4 for the oxidation of A n+ to A O-3
out the molecular formula of X and gives the structure
in acidic medium. What is the value of n ? (1984, 2M)
of Y and Z. (1989, 5M)
49. 4.08 g of a mixture of BaO and unknown carbonate MCO3
45. An equal volume of a reducing agent is titrated separately
with 1 M KMnO4 in acid, neutral and alkaline medium. The was heated strongly. The residue weighed 3.64 g. This was
volumes of KMnO4 required are 20 mL in acid, 33.3 mL in dissolved in 100 mL of 1 N HCl. The excess acid required
neutral and 100 mL in alkaline media. Find out the oxidation 16 mL of 2.5 N NaOH solution for complete neutralisation.
state of manganese in each reduction product. Give the Identify the metal M. (1983, 4M)

Answers
Topic 1 55. (i) 37.92, (ii) 0.065, (iii) 7.73m 56. (a) 0.6, (b) 24
1. (d) 2. (c) 3. (b) 4. (a) 58. (i) 0.0179 g, (ii) 10.6 % 59. (0.437) 61. 20 %
5. (d) 6. (b) 7. (d) 8. (c) Topic 2
9. (b) 10. (b) 11. (b) 12. (c) 1. (d) 2. (b) 3. (a) 4. (a)
13. (b) 14. (*) 15. (d) 16. (d) 5. (d) 6. (*) 7. (b) 8. (c)
17. (b) 18. (a) 19. (c) 20. (b) 9. (d) 10. (c) 11. (a) 12. (d)
21. (a) 22. (a) 23. (d) 24. (d) 13. (b) 14. (c) 15. (a) 16. (d)
25. (d) 26. (a) 27. (d) 28. (a) 17. (a) 18. (b) 19. (a) 20. (a)
29. (a) 30. (a) 31. (a) 32. (c) 21. (b) 22. (c) 23. (b) 24. (a, b, d)
33. (6.47kg) 34. (126 mg) 35. (495 g) 36. (4.14 g) 25. (2992) 26. (b) 27. 7/3 28. (5)
37. (0.4) 38. (6.023´10 24 ) 39. C-12 isotope 40. 29. (2) 30. (3) 31. (1008 g) 33. (8.096 mL)
(9)
34. (0.062 M) 35. (1.334 V) 39. (85%) 41. (1.04 ´ 10 4 )
41. (8) 42. (8 mL) 43. (2) 44. (5 ´ 10 -19 m 2 )
42. (1:2) 45. (16.67 mL) 46. (6.5gL -1) 47. (6.5376 g)
45. (55.56 mol L-1) 46. (70.91 ´ 10 6g) 47.(4.3 ´ 10 -3) 50. (10.42)
48. (2) 49. (Ca)
51. (1.7 g) 52. (55.55 L) 53. (9.9 ´ 10 -3)
Hints & Solutions
Topic 1 Mole Concept 3. In eudiometry,
300 K
æ yö y
CxH y + ç x + ÷ O2 ¾¾¾® x CO2 + H2O
Key Idea To find the mass of A and B in the given question, è 4ø 1 atm 2
1. æ yö
mole concept is used. 1 mol ç x + ÷ mol x mol
è 4ø
given mass (w)
Number of moles( n) = 1 mL
æ yö
ç x + ÷ mL x mL
molecular mass (M ) è 4ø
æ yö
10 mL ç x + ÷ ´ 10 mL 10x mL
è 4ø
Compound Mass of A (g) Mass of B (g)
Given, (i) VCO2 = 10x = 40 mL Þ x = 4
AB2 MA 2 MB æ yö
(ii) VO2 = 10 ç x + ÷ mL = 55 mL
A 2B 2 2 MA 2 MB è 4ø
æ yö
We know that, Þ 10 ç 4 + ÷ = 55 [Q x = 4]
è 4ø
given mass (w)
Number of moles (n) = y ´ 10
molecular mass (M ) Þ 40 + = 55
4
n ´M = w …(A) 10 4
Using equation (A), it can be concluded that Þ y ´ = 15 Þ y = 15 ´ = 6
4 10
5(M A + 2M B ) = 125 ´ 10-3 kg …(i) So, the hydrocarbon (CxH y ) is C4H6.
-3
10(2M A + 2M B ) = 300 ´ 10 kg …(ii) 4. Given, volume = 10 mL
From equation (i) and (ii) Molarity = 1 mM = 10-3 M
1 (M A + 2M B ) æ 125 ö \Number of millimoles = 10 mL ´ 10-3 M = 10-2
=ç ÷
2 (2M A + 2M B ) è 300 ø
Number of moles = 10-5
On solving the equation, we obtain
Now, number of molecules
M A = 5 ´ 10-3
= Number of moles ´ Avogadro’s number
and M B = 10 ´ 10-3 = 10-5 ´ 6 ´ 1023 = 6 ´ 1018
So, the molar mass of A (M A ) is Surface area occupied by 6 ´ 1018 molecules = 0.24 cm2
5 ´ 10-3 kg mol -1 and B (M B ) is 10 ´ 10-3 kg mol -1.
\Surface area occupied by 1 molecule
2. (a) C3H8 (g ) + 5O2 (g ) ¾® 3CO2 (g ) + 4H2O(l ) 0.24
= = 0.04 ´ 10-18 cm2
44g 160g 6 ´ 1018
160 As it is given that polar head is approximated as cube. Thus,
Þ 1g of reactant = g of O2 consumed = 3.64 g
44 surface area of cube = a2, where
(b) P4 (s) + 5O2(g ) ¾® P4O10 (s) a = edge length
124g 160g \ a2 = 4 ´ 10-20 cm2
160
Þ1 g of reactant = g of O2 consumed = 129
. g a = 2 ´ 10-10 cm = 2 pm
124
(c) 4Fe(s) + 3O2(g ) ¾® 2Fe2O3 (s) 5. Key Idea The reactant which is present in the lesser amount,
244g 96g 96 i.e. which limits the amount of product formed is called
Þ 1 g of reactant = g of O2 consumed = 0.43 g limiting reagent.
224
(d) 2Mg(s) + O2(g ) ¾® 2MgO(s) When 56 g of N2 + 10 g of H2 is taken as a combination then
48 g 32 g dihydrogen (H2 ) act as a limiting reagent in the reaction.
32 N2 (g ) + 3H2 (g ) ¾® 2NH3 (g ) …(I)
Þ 1 g of reactant = g of O2 consumed = 0.67 g 2 ´ 14 g 3 ´ 2g 2(14 + 3) g
48
28g 6g 34g
So, minimum amount of O2 is consumed per gram of reactant
(Fe) in reaction (c). 28g N2 requires 6g H2 gas.
6g
56g of N2 requires ´ 56 g = 12g of H2
28 g
Some Basic Concepts of Chemistry 9

12g of H2 gas is required for 56g of N2 gas but 8. Concentration of H2O2 is expressed in terms of volume strength,
only 10 g of H2 gas is present in option (a). i.e. “volume of O2 liberated by H2O2 at NTP”. Molarity is
Hence, H2 gas is the limiting reagent. connected to volume strength as:
In option (b), i.e. 35g of N2 + 8 g of H2. Molarity (M) =
x
or x = Molarity ´ 11.2
As 28 g N2 requires 6g of H2. 112
.
6g where, x = volume strength
35g N2 requires ´ 35 g H2 Þ 7.5 g of H2. So, for 1 M H2O2, x = 1 ´ 112. = 112.
28 g
Among the given options, 11.35 is nearest to 11.2.
Here, H2 gas does not act as limiting reagent since 7.5 g of H2
gas is required for 35g of N2 and 8g of H2 is present in reaction Number of moles of solute (n)
9. Molarity =
mixture. Mass of H2 left unreacted = 8 - 7.5 g of H2. Volume of solution (in L)
= 0.5 g of H2. wB (g)
Also, n =
Similarly, in option (c) and (d), H2 does not act as limiting M B (gmol-1 )
reagent. w / MB
For 14 g of N2 + 4 g of H2. \ Molarity = B
V
As we know 28g of N2 reacts with 6g of H2.
Given, wB = mass of solute ( B ) in g
6
14 g of N2 reacts with ´ 14 g of H2 Þ 3g of H2. M B = Gram molar mass of B (C12H22O11 ) = 342 g mol -1
28
For 28g of N2 + 6 g of H2, i.e. 28g of N2 reacts with 6g of H2 Molarity = 01. M
(by equation I). Volume (V ) = 2 L
w / 342
Þ . = B
01 Þ wB = 01
. ´ 342 ´ 2 g = 68.4 g
6. Key Idea The percentage composition of a compound is given 2
by the formula. 10. 2 C57 H110O6 (s) + 163 O2 (g ) ¾® 110H2O(l ) + 114 CO2 (g )
% composition = [Composition of a substance in a compound /
Molecular mass of C57H110O6
Total composition total of compound] ´100
= 2 ´ (12 ´ 57 + 1 ´ 110 + 16 ´ 6) g = 1780 g
In CH4 , Molecular mass of 110 H2O = 110 (2 + 16) = 1980 g
mole of carbon = 1 1780 g of C57H110O6 produced = 1980 g of H2O.
mole of hydrogen = 4 1980
445g of C57H110O6 produced = ´ 445 g of H2O
1 1780
\ % of carbon by mole in CH4 = ´ 100 = 20%
1+ 4 = 495of H2O
Number of moles of solute
7. Mole fraction of solute 11. Molality (m) = ´ 1000
number of moles of solute + number of moles solvent Mass of solvent (in g)
= Mass of solute (in g) ´ 1000
number of moles of solute =
w Solute é Molecular weight of solute ù
êë ´ mass of solvent (in g)úû
n Solute Mw Solute
cSolute = = wNa+ ´ 1000 92 ´ 1000
n Solute + nSolvent w Solute w
+ Solvent = = = 4 mol kg - 1
Mw Solute MwSolvent M Na+ ´ wH 2 O 23 ´ 1000

Given, wSolute = wNaOH = 8 g 12. Given, abundance of elements by mass


Mw Solute = MwNaOH = 40g mol - 1 oxygen = 614. %, carbon = 22.9%, hydrogen = 10% and
nitrogen = 2.6%
w Solvent = w H2 O = 18g Total weight of person = 75 kg
Mw Solvent = 18 g mol- 1 75 ´ 10
Mass due to 1 H = = 7.5 kg
8 / 40 0.2 0.2 100
\ cSolute = c NaOH = = = = 0167
.
8 18 0.2 + 1 12
. 1
H atoms are replaced by 2 H atoms,
+
40 18 Mass due to 2 H = (7.5 ´ 2) kg
Moles of solute \ Mass gain by person = 7.5 kg
Now, molality (m) =
Mass of solvent (in kg)
13. M 2CO3 + 2HCl ¾® 2M Cl + H2O + CO2
w Solute 8 1g 0.01186
mole
Mw Solute 40
= ´ 1000 = ´ 1000 Number of moles of M 2CO3 reacted = Number of moles of CO2
wSolvent (in g ) 18
evolved
0.2
= ´1000 = 11.11 mol kg - 1 1
= 0.01186 [M = molar mass of M 2CO3]
18 M
Thus, mole fraction of NaOH in solution and molality of the 1
solution respectively are 0.167 and 11.11 mol kg - 1 . M = = 84.3 g mol - 1
0.01186
10 Some Basic Concepts of Chemistry

Moles of solute
14. CxH y (g ) + æç x + ö÷ O2 (g ) ¾® xCO2 (g ) + H2O(l )
y y
19. Molarity =
è 4ø 30 mL
2 Volume of solution (L)
75 mL
120
O 2 used = 20% of 375 = 75 mL Moles of urea = =2
60
Inert part of air = 80% of 375 = 300 mL
Weight of solution = Weight of solvent + Weight of solute
Total volume of gases = CO2 + Inert part of air
= 1000 + 120 = 1120 g
= 30 + 300 = 330 mL
1120 g 1
x 30 Þ Volume = ´ = 0.973 L
= Þx=2 1.15 g / mL 1000 mL / L
1 15
2.000
x+
y Þ Molarity = = 2.05M
4 = 75 Þ x + y = 5 0.973
1 15 4 20. From the given relative abundance, the average weight of Fe can be
Þ x = 2, y = 12 Þ C2 H12 calculated as
54 ´ 5 + 56 ´ 90 + 57 ´ 5
15. We know the molecular weight of C8 H7 SO3Na A= = 55.95
100
= 12 ´ 8 + 1 ´ 7 + 32 + 16 ´ 3 + 23 = 206
we have to find, mole per gram of resin.
21. 1.0 L of mixture X contain 0.01 mole of each [Co(NH3 )5 SO4 ]Br
and [Co(NH3 )5 Br]SO4. Also, with AgNO3, only
\ 1g of C8 H7 SO3Na has number of mole [Co(NH3 )5 SO4 ]Br reacts to give AgBr precipitate as
weight of given resin 1
= = mol [Co(NH3 )5 SO4 ]Br + AgNO3 ® [Co(NH3 )5 SO4 ]NO3 + AgBr
Molecular, weight of resin 206 1.0 mol Excess 1.0 mol

Now, reaction looks like With BaCl 2, only [Co(NH3 )5 Br]SO4 reacts giving BaSO4
precipitate as
2C8 H7 SO3Na + Ca 2+ ¾® (C8 H7 SO3 )2 Ca + 2Na
[Co(NH3 )5 Br]SO4 + BaCl 2 ® [Co(NH3 )5 Br]Cl 2 + BaSO4
Q 2 moles of C8 H7 SO3Na combines with 1 mol Ca 2+ 1.0 mol Excess 1 mol

1 Hence, moles of Y and Z are 0.01 each.


\1 mole of C8 H7 SO3Na will combine with mol Ca 2+
2 22. Number of atoms = Number of moles
1 ´ Avogadro’s number (N A)
\ mole of C8 H7 SO3 Na will combine with
206 24
Number of atoms in 24 g C = ´ NA = 2NA
1 1 1 12
´ mol Ca 2+ = mol Ca 2+ 56
2 206 412 Number of atoms in 56 g of Fe = NA = NA
56
16. Given, initial strength of acetic acid = 0.06 N Number of atoms in 27 g of Al =
27
NA = NA
Final strength = 0.042 N; Volume = 50 mL 27
108
\Initial millimoles of CH3COOH = 0.06 ´ 50 = 3 Number of atoms in 108 g of Ag = NA = NA
108
Final millimoles of CH3COOH = 0.042 ´ 50 = 2.1 Hence, 24 g of carbon has the maximum number of atoms.
\ Millimoles of CH3COOH adsorbed = 3 - 2.1 = 0.9 mmol
23. Mass of an electron = 9.108 ´ 10-31 kg
= 0.9 ´ 60 mg = 54 mg
(mO 2 ) Q 9.108 ´ 10-31 kg = 1.0 electron
nO 2 (M O 2 ) 1 1031 1
17. = \ 1 kg = -31
electrons = ´
nN 2 (mN 2 ) 9.108 ´ 10 9.108 6.023 ´ 1023
(M N 2 ) 1
= ´ 108 mole of electrons
where, mO 2 = given mass of O2 , mN 2 = given mass of N2 , 9.108 ´ 6.023
M O 2 = molecular mass of O2 , M N 2 = molecular mass of N2 , 24. Phosphorus acid is a dibasic acid as :
nO 2 = number of moles of O2 , nN 2 = number of moles of N2 O
é mO ù 28 1 28 7 ½½
=ê 2ú = ´ = H—P — OH only two replaceable hydrogens
êë mN 2 úû 32 4 32 32 ½
OH
M 1V1 + M 2V2
18. From the formula, M f = Therefore, normality = molarity ´ basicity = 0.3 ´ 2 = 0.60
V1 + V2
25. Molality is defined in terms of weight, hence independent of
Given, V1 = 750 mL, M 1 = 0.5 M temperature. Remaining three concentration units are defined in
V2 = 250 mL, M 2 = 2 M terms of volume of solution, they depends on temperature.
750 ´ 0.5 + 250 ´ 2 875
= = = 0.875 M 26. Molality of a solution is defined as number of moles of solute
750 + 250 1000 present in 1.0 kg (1000 g) of solvent.
Some Basic Concepts of Chemistry 11

27. The balanced chemical reaction is w (N2 ) N (N2 )


And = …(ii)
3BaCl 2 + 2Na 3PO4 ¾® Ba 3 (PO4 )2 + 6NaCl 28 NA
In this reaction, 3 moles of BaCl 2 combines with 2 moles of Dividing Eq. (i) by Eq. (ii) gives
Na 3PO4. Hence, 0.5 mole of of BaCl 2 require N (O2 ) w (O2 ) 28
2 = ´
´ 0.5 = 0.33 mole of Na 3PO4. N (N2 ) w (N2 ) 32
3
1 28 7
Since, available Na 3PO4 (0.2 mole) is less than required mole = ´ =
(0.33), it is the limiting reactant and would determine the 4 32 32
amount of product Ba 3 (PO4 )2. 33. The equations of chemical reactions occurring during the process
Q 2 moles of Na 3PO4 gives 1 mole Ba 3 (PO4 )2 are
1 In the presence of oxygen
\ 0.2 mole of Na 3PO4 would give ´ 0.2 = 0.1 mole Ba 3 (PO4 )2
2 2PbS + 3O2 ¾® 2PbO + 2SO2 …(i)
28. Unlike other metal carbonates that usually decomposes into By self reduction
metal oxides liberating carbon dioxide, silver carbonate on 2PbO + PbS ¾® 3Pb + SO2
heating decomposes into elemental silver liberating mixture of
carbon dioxide and oxygen gas as : Thus 3 moles of O2 produces 3 moles of Pb
i.e. 32 ´ 3 = 96 g of O 2 produces 3 ´ 207 = 621 g of Pb
Heat 1
Ag2CO3 (s) ¾® 2Ag (s) + CO2 (g ) + O (g ) So 1000 g (1kg) of oxygen will produce
2 2
MW = 276 g 2 ´ 108 = 216 g 621
´ 1000 = 6468.75 g
Hence, 2.76 g of Ag2CO3 on heating will give 96
216 = 6.4687 kg » 6.47 kg
´ 2.76 = 2.16 g Ag as residue.
276 Alternative Method
From the direct equation,
29. The balanced chemical reaction of zinc with sulphuric acid and
NaOH are PbS + O2 ¾® Pb + SO2
32 g 207 g
Zn + H2SO4 ¾® ZnSO4 + H2 (g ) ­
Zn + 2NaOH + 2H2O ¾® Na 2[ Zn(OH)4 ] + H2 (g ) ­ So, 32 g of O 2 gives 207 g of Pb
207
Since, one mole of H2 (g ) is produced per mole of zinc with both 1 g of O 2 will give g of Pb
32
sulphuric acid and NaOH respectively, hydrogen gas is
207
produced in the molar ratio of 1:1 in the above reactions. 1000g of O 2 will give ´ 1000 = 6468.75 g
32
30. Number of molecules present in 36 g of water = 6.46875 kg » 6.47kg
36
= ´ N A = 2N A 34. The balanced equations are
18
28 (1) 2MnCl 2 + 5K 2S 2O 8 + 8H 2O ®
Number of molecules present in 28 g of CO = ´ NA = NA 2KMnO 4 + 4 K 2SO 4 + 6H 2SO 4 + 4HCl
28
46 (2) 2KMnO 4 + 5H 2C 2O 4 + 3H 2SO 4 ®
Number of molecules present in 46 g of C2H5OH = ´ NA = NA
46 K 2SO 4 + 2MnSO 4 + 8H 2O+10CO 2
54
Number of molecules present in 54 g of N2O5 = ´ N A = 0.5 N A Given, mass of oxalic acid added = 225mg
108
225
Here, NA is Avogadro’s number. Hence, 36 g of water contain So, millimoles of oxalic acid added = = 2.5
the largest (2NA ) number of molecules. 90
Now from equation 2
31. In a neutral atom, atomic number represents the number of Millimoles of KMnO 4 used to react with oxalic acid=1 and
protons inside the nucleus and equal number of electrons around Millimoles of MnCl 2 required initially=1
it. Therefore, the number of total electrons in molecule of CO2
\ Mass of MnCl 2 required initially = 1 ´ (55 + 71) = 126mg
= electrons present in one carbon atom Alternative Method
+ 2 ´ electrons present in one oxygen atom
m moles of MnCl 2 = m moles of KMnO 4 = x (let)
= 6 + 2 ´ 8 = 22.
and M eq of KMnO 4 = M eq of oxalic acid
Weight of a compound in gram (w) 225
32. = Number of moles (n) So, x´ 5= ´2
Molar mass (M ) 90
Number of molecules (N ) Hence, x = 1
=
Avogadro number (NA ) \ m moles of MnCl 2 = 1
w (O2 ) N (O2 )
Þ = …(i) Hence mass of MnCl 2 = ( 55 + 71) ´ 1 = 126 mg.
32 NA
12 Some Basic Concepts of Chemistry

35. Given, Number of moles of solute


37. Molarity =
O Volume of solution in litre
Weight of solute 1000
= ´
NaOBr NH3/D Br2/KOH Br2(3-eqiv.) Molar mass Volume in mL
A B C D 3 1000
H3O+ (60%) (50%) (50%)
AcOH
(100%) = ´ = 0.4 M
30 250
The products formed are 38. Considering density of water to be 1.0 g/mL, 18 mL of water is
O HO O H2N O 18 g (1.0 mol) of water and it contain Avogadro number of
molecules. Also one molecule of water contain
NaOBr NH3/D 2 ´ (one from each H-atom) + 8 ´ (from oxygen atom)
H3O+ = 10 electrons.
Benzoic acid Benzamide
Þ 1.0 mole of H2O contain = 10 ´ 6.023 ´ 1023
Acetophenone Br2/KOH
10 mol (60%)
i.e.,
(50%)
i.e.,
= 6.023 ´ 1024 electrons.
6 mol 3 mol
(A) (B) 39. Carbon-12 isotope. According to modern atomic mass unit, one
NH2 NH2 atomic mass unit (amu) is defined as one-twelfth of mass of an
Br Br atom of C-12 isotope, i.e.
Br2(3-eqiv.) 1
1 amu (u) = ´ weight of an atom of C-12 isotope.
AcOH 12
w w
Br
Aniline 40. Moles of solute, n1 = 1 ; Moles of solvent, n2 = 2
(50%) m1 m2
2,4,6-tribromo aniline i.e.,
(100%) 1.5 mol c 1 (solute) = 0.1 and c 2 (solvent) = 0.9
i.e., (C)
c 1 n1 w1 m2 1
1.5 mol \ = = × =
(D) c 2 n2 m1 w2 9
NH2 Solute (moles) w1 ´ 1000 ´ 2
Br Br Molarity = =
Volume (L) m1 (w1 + w2 )
So, 1.5 mol of are produced from Total mass of solution æ w1 + w2 ö
Note Volume = =ç ÷ mL
Density è 2 ø
Br Solute (moles) w1 ´ 1000
Molality = =
10 moles of acetophenone. Solvent (kg) m1 ´ w2
NH2 Given, molarity = molality
Br Br 2000 w1 1000 w1
hence, =
Molar mass of = 240 + 14 + 4 + 72 = 330 m1 (w1 + w2 ) m1 w2
w2 1
\ = Þ w1 = w2 = 1
Br w1+ w2 2
NH2 w1 m2 1 m1 (solute)
\ = Þ =9
Br Br m1 w2 9 m2 (solvent)
Hence, amount of produced is 330 × 1.5 = 495 g
41. PLAN This problem can be solved by using concept of conversion of
molarity into molality.
Br Molarity = 3.2 M
Let volume of solution = 1000 mL = Volume of solvent
36. Molar mass of CuSO4 × 5H2O
Mass of solvent = 1000 ´ 0.4 = 400 g
= 63.5 + 32 + 4 ´ 16 + 5 ´ 18
Since, molarity of solution is 3.2 molar
= 249.5 g
\ n solute = 3.2 mol
Also, molar mass represents mass of Avogadro number of 3.2
molecules in gram unit, therefore Molality (m) = =8
400 / 1000
Q 6.023 ´ 1023 molecules of CuSO4 × 5H2O weigh 249.5 g
Hence, correct integer is (8).
249.5
\ 1022 molecules will weigh ´ 1022 = 4.14 g 42. Mass of HCl in 1.0 mL stock solution
6.023 ´ 1023
29.2
= 1.25 ´ = 0.365 g
100
Some Basic Concepts of Chemistry 13

Mass of HCl required for 200 mL 0.4 M HCl 80.575


Þ Mole of Na 2SO4 × 10H2O in 1.0 L solution = = 0.25
200 322
= ´ 0.4 ´ 36.5 = 0.08 ´ 36.5 g
1000 Þ Molarity of solution = 0.25 M
\ 0.365 g of HCl is present in 1.0 mL stock solution. Also, weight of 1.0 L solution = 1077.2 g
0.08 ´ 36.5 weight of Na 2SO4 in 1.0 L solution = 0.25 ´ 142 = 35.5 g
0.08 ´ 36.5 g HCl will be present in = 8.0 mL
0.365 Þ Weight of water in 1.0 L solution = 1077.2 – 35.5 = 1041.7 g
0.25
43. Partial pressure of N2 = 0.001 atm, Þ Molality = ´ 1000 = 0.24 m
1041.7
T = 298 K, V = 2.46 dm 3. Mole of Na 2SO4
Mole fraction of Na 2SO4 =
From ideal gas law : pV = nRT Mole of Na 2SO4 + Mole of water
pV 0.001 ´ 2.46
n(N2 ) = = = 10-7 0.25
RT 0.082 ´ 298 =
1041.7
0.25 +
Þ Number of molecules of N2 = 6.023 ´ 1023 ´ 10-7 18
= 6.023 ´ 1016 = 4.3 ´ 10-3.
Now, total surface sites available 48. Compound B forms hydrated crystals with Al 2 (SO4 )3. Also, B is
= 6.023 ´ 1014 ´ 1000 = 6.023 ´ 1017 formed with univalent metal on heating with sulphur. Hence,
20 compound B must has the molecular formula M 2SO4 and
Surface sites used in adsorption = ´ 6.023 ´ 1017 compound A must be an oxide of M which reacts with sulphur to
100
give metal sulphate as
= 2 ´ 6.023 ´ 1016
A + S ¾® M 2SO4
Þ Sites occupied per molecules B

Number of sites 2 ´ 6.023 ´ 1016 Q 0.321 g sulphur gives 1.743 g of M 2SO4


= = = 2 \ 32.1 g S (one mole) will give 174.3 g M 2SO4
Number of molecules 6.023 ´ 1016
Therefore, molar mass of M 2SO4 = 174.3 g
44. Initial millimol of CH3COOH = 100 ´ 0.5 = 50 Þ 174.3 = 2 ´ Atomic weight of M + 32.1 + 64
millimol of CH3COOH remaining after adsorption Þ Atomic weight of M = 39, metal is potassium (K)
= 100 ´ 0.49 = 49 K2SO4 on treatment with aqueous Al 2 (SO4 )3 gives potash-alum.
Þ millimol of CH3COOH adsorbed = 50 – 49 = 1 K2SO4 + Al 2 (SO4)3 + 24H2O ¾® K2SO4Al 2 (SO4)3 × 24H2O
B C
Þ number of molecules of CH3COOH adsorbed If the metal oxide A has molecular formula MOx, two moles of it
1 combine with one mole of sulphur to give one mole of metal
= ´ 6.023 ´ 1023 = 6.023 ´ 1020 sulphate as
1000
2KOx + S ¾® K2SO4
3.01 ´ 102 Þ x = 2, i.e. A is KO2.
Þ Area covered up by one molecule =
6.02 ´ 1020
49. The reaction involved is
= 5 ´ 10-19 m 2 3Pb(NO3 )2 + Cr2 (SO4 )3 ¾® 3PbSO4 (s) ¯ + 2Cr(NO3 )3
45. Mass of 1.0 L water = 1000 g millimol of Pb(NO3 )2 taken = 45 ´ 0.25 = 11.25
1000
Þ Molarity = = 55.56 mol L-1 millimol of Cr2 (SO4 )3 taken = 2.5
18 Here, chromic sulphate is the limiting reagent, it will determine
the amount of product.
46. Volume of one cylinderical plant virus = pr2l
Q 1 mole Cr2 (SO4 )3 produces 3 moles PbSO4.
= 3.14 (75 ´ 10-8 )2 ´ 5000 ´ 10-8 cm 3 = 8.83 ´ 10-17 cm 3
\ 2.5 millimol Cr2 (SO4 )3 will produce 7.5 millimol PbSO4.
Volume of a virus
Þ Mass of one virus = Hence, mole of PbSO4 precipitate formed = 7.5 ´ 10-3
Specific volume
Also, millimol of Pb(NO3 )2 remaining unreacted
8.83 ´ 10-17 cm 3
= = 1.1773 ´ 10-16 g 11.25 – 7.50 = 3.75
0.75 cm 3 g-1 Þ Molarity of Pb(NO3 )2 in final solution
Þ Molar mass of virus millimol of Pb(NO3 )2 3.75
= = = 0.054 M
= Mass of one virus ´ Avogadro’s number Total volume 70
= 1.1773 ´ 10-16 ´ 6.023 ´ 1023 g Also, millimol of Cr(NO3 )2 formed
= 70.91 ´ 10 g6 = 2 ´ millimol of Cr2 (SO4 )3 reacted
5
47. Molar mass of Glauber’s salt (Na 2SO4 × 10H2O) Þ Molarity of Cr(NO3 )2 = = 0.071 M
70
= 23 ´ 2 + 32 + 64 + 10 ´ 18 = 322 g
14 Some Basic Concepts of Chemistry

50. 93% H2SO4 solution weight by volume indicates that there is Mole of sugar
(ii) Mole fraction of sugar =
93 g H2SO4 in 100 mL of solution. Mole of sugar + Mole of water
0.1
If we consider 100 mL solution, weight of solution = 184 g = = 9.9 ´ 10-3
Weight of H2O in 100 mL solution = 184 – 93 = 91 g 0.1 + 10
Moles of solute 54. From the given elemental composition, empirical formula can
Þ Molality = ´ 1000
Weight of solvent (g) be derived as :
93 1000
= ´ = 10.42 Element C H O
98 91
Weight % 69.77 11.63 18.60
51. Heating below 600°C converts Pb(NO3 )2 into PbO but to Mole % 5.81 11.63 1.1625 (obtained by
NaNO3 into NaNO2 as dividing from M )
D 1
Pb(NO3 )2 ¾® PbO(s) + 2NO2 ­ + O ­ Simple ratio 5 10 1
2 2
MW : 330 222 Hence, empirical formula is C5H10O and empirical formula
D 1
NaNO3 ¾® NaNO2 (s) + O ­ weight is 86.
2 2
MW : 85 69 Since, empirical formula weight and molecular weight both are
28 (86), empirical formula is the molecular formula also.
Weight loss = 5 ´ = 1.4 g
100 Also, the compound does not reduce Fehling’s solution,
Þ Weight of residue left = 5 – 1.4 = 3.6 g therefore it is not an aldehyde, but it forms bisulphite, it must be
Now, let the original mixture contain x g of Pb(NO3 )2. a ketone.
Q 330 g Pb(NO3 )2 gives 222 g PbO Also, it gives positive iodoform test, it must be a methyl ketone.
222 x O
\ x g Pb(NO3 )2 will give g PbO
330 ½½
Similarly, 85 g NaNO3 gives 69 g NaNO2 C3H7 — C — CH3
69 (5 - x ) Based on the above information, the compound may be one of
Þ (5 – x) g NaNO3 will give g NaNO2
85 the following :
222 x 69 (5 - x ) O CH3 O
Þ Residue : + = 3.6 g ½½ ½ ½½
330 85 CH3CH2CH2— C — CH3 or CH3 — CH— C — CH3
Solving for x gives, x = 3.3 g Pb(NO3 )2 2-pentanone 3-methyl -2-butanone
Þ NaNO3 =1.7 g.
55. (a) Let us consider 1.0 L solution for all the calculation.
52. Reactions involved are (i) Weight of 1 L solution = 1250 g
C2H6 + Br2 ¾® C2H5Br + HBr Weight of Na 2S2O3 = 3 ´ 158 = 474 g
2C2H5Br + 2Na ¾® C4H10 + 2NaBr 474
Þ Weight percentage of Na 2S2O3= ´ 100 = 37.92
Actual yield of C4H10 = 55 g which is 85% of theoretical yield. 1250
55 ´ 100 (ii) Weight of H2O in 1 L solution = 1250 - 474 = 776 g
Þ Theoretical yield of C4H10 = = 64.70 g
85 3
Mole fraction of Na 2S2O3 = = 0.065
Also, 2 moles (218 g) C2H5Br gives 58 g of butane. 776
3+
Þ 64.70 g of butane would be obtained from 18
2 + 3 ´2
´ 64.70 = 2.23 moles C2H5Br (iii) Molality of Na = ´ 100 = 7.73 m
58 776
Also yield of bromination reaction is only 90%, in order to have 56. (a) After passing through red-hot charcoal, following reaction
2.23 moles of C2H5Br, theoretically occurs
2.23 ´ 100 C(s) + CO2 (g ) ¾® 2CO(g )
= 2.48 moles of C2H5Br required. If the 1.0 L original mixture contain x litre of CO2, after
90
Therefore, moles of C2H6 required = 2.48 passing from tube containing red-hot charcoal, the new
volumes would be :
Þ Volume of C2H6 (NTP) required = 2.48 ´ 22.4 = 55.55 L.
2x (volume of CO obtained from CO2) + 1
34.2
53. Moles of sugar = = 0.1 – x (original CO) = 1 + x = 1.6 (given)
342
Þ x = 0.6
Moles of water in syrup = 214.2 – 34.2 = 180 g
Hence, original 1.0 L mixture has 0.4 L CO and 0.6 L of CO2 ,
Moles of solute
Therefore, (i) Molality = ´ 1000 i.e. 40% CO and 60% CO2 by volume.
Weight of Solvent (g)
(b) According to the given information, molecular formula of
0.1 the compound is M 3N2. Also, 1.0 mole of compound has 28 g
= ´ 1000 = 0.55
180 of nitrogen. If X is the molar mass of compound, then :
Some Basic Concepts of Chemistry 15

28 Now, let us consider 1.0 mole of mixture and it contains


X ´ = 28
100 x mole of NO2.
Þ X = 100 = 3 ´ Atomic weight of M + 28 Þ 46 x + 92 (1 - x ) = 76.6 Þ x = 0.3348
72 100
Þ Atomic weight of M = = 24 Also, in 100 g mixture, number of moles =
3 76.6
57. In the present case, V µ n (Q all the volumes are measured under 100
Þ Moles of NO2 in mixture = ´ 0.3348 = 0.437
identical conditions of temperature and pressure) Hence, the 76.6
reaction stoichiometry can be solved using volumes as : 60. Most of the elements found in nature exist as a mixture of
æ yö y isotopes whose atomic weights are different. The atomic weight
CxH y (g ) + ç x + ÷ O2 (g ) ¾® x CO2 (g ) + H2O (l ) of an element is the average of atomic weights of all its naturally
è 4ø 2
occurring isotopes.
volume of CO2 (g ) + O2 (g ) (remaining unreacted) = 25
Þ Volume of CO2 (g ) produced
61. Average atomic weight
S Percentage of an isotope ´ Atomic weight
= 10 mL (15 mL O2 remaining) =
100
Q 1 mL CxH y produces x mL of CO2
10.01x + 11.01 (100 - x)
\ 5 mL CxH y will produce 5 xmL of CO2= 10 mL Þ 10.81 = Þ x = 20%
100
Þ x=2
Therefore, natural boron contains 20% (10.01) isotope and 80%
æ yö
Also, 1 mL CxH y combines with ç x + ÷ mL of O2 other isotope.
è 4ø
æ yö Topic 2 Equivalent Concept, Neutralisation
5 mL CxH y will combine with 5 ç x + ÷ mL of O2
è 4 ø and Redox Titration
æ yö
Þ 5 ç x + ÷ = 15 (15 mL of O2 out of 30 mL) 1. In disproportionation reactions, same element undergoes
è 4ø oxidation as well as reduction.
(remaining unreacted)
Þ y = 4, hence hydrocarbon is C2H4. e.g. Reduction
+1 +2 0
58. Oxides of sodium and potassium are converted into chlorides 2CuBr CuBr2 + Cu
according to following reactions :
Oxidation
Na 2O + 2HCl ¾® 2NaCl + H2O
Here, CuBr get oxidised to CuBr2 and also it get reduced to Cu.
K2O + 2HCl ¾® 2KCl + H2O
Other given reactions and their types are given below.
Finally all the chlorides of NaCl and KCl are converted into Reduction
AgCl, hence +7
– – + +2
moles of (NaCl + KCl) = moles of AgCl 2 MnO4 + 10I + 16 H 2Mn + 5I2 + 8H2O
(one mole of either NaCl or KCl gives one mole of AgCl) Oxidation
Now, let the chloride mixture contain x g NaCl. In the given reaction, MnO-4 get oxidised to Mn 2+ and I- get
x 0.118 - x 0.2451
Þ + = reduced to I2. It is an example of redox reaction. The reaction
58.5 74.5 143.5 takes place in acidic medium.
Solving for x gives x = 0.0338 g (mass of NaCl) 2KMnO4 ¾¾® K 2MnO4 + MnO2 + O2
Þ Mass of KCl = 0.118 – 0.0338 The given reaction is an example of decomposition reaction.
= 0.0842 g Here, one compound split into two or more simpler compounds,
1 atleast one of which must be in elemental form.
Also, moles of Na 2O = ´ moles of NaCl
2 2NaBr + Cl2 ¾¾® 2NaCl + Br2
1 0.0338 The given reaction is an example of displacement reaction. In
Þ Mass of Na 2O = ´ ´ 62 = 0.0179 g this reaction, an atom (or ion) replaces the ion (or atom) of
2 58.5
1 0.0842 another element from a compound.
Similarly, mass of K2O = ´ ´ 94 = 0.053 g
2 74.5 2. The graph that shows the correct change of pH of the titration
0.0179 mixture in the experiment is
Þ Mass % of Na 2O = ´ 100 = 3.58 %
0.5
0.053
Mass % of K2O = ´ 100 = 10.6 %
0.5
pH
59. From the vapour density information
Molar mass = Vapour density ´ 2 (Q Molar mass of H2 = 2)
= 38.3 ´ 2 = 76.6 V(mL)
16 Some Basic Concepts of Chemistry

In this case, both the titrants are completely ionised. V H 2 O = 100mL


+ -
HCl + NaOH - N a Cl + H O 2 Now, neq (CaCO3) = neq [Ca(HCO3 )2 ] + neq [Mg(HCO3 )2 ]
Å
+ W 0.81 0.73
As H is added to a basic solution, [ OH ] decreases and [ H ]
È
´2= ´2+ ´2
increases. Therefore, pH goes on decreasing. As the equivalence 100 162 146
point is reached, [OHÈ ] is rapidly reduced. After this point [OH È] W
\ = 0.005 + 0.005
decreases rapidly and pH of the solution remains fairly constant. 100
Thus, there is an inflexion point at the equivalence point. W = 0.01 ´ 100 = 1
The difference in the volume of NaOH solution between the end 1
Thus, hardness of water sample = ´ 106 = 10,000 ppm
point and the equivalence point is not significant for most of the 100
commonly used indicators as there is a large change in the pH
value around the equivalence point. Most of them change their 6. The reaction takes place as follows,
colour across this pH change. H2C2O4 + 2NaOH ¾® Na 2C2O4 + 2H2O
3 Now, 50 mL of 0.5 M H2C2O4 is needed to neutralize 25 mL of
3. 100 mL (cm ) of hexane contains 0.27 g of fatty acid.
In 10 mL solution, mass of the fatty acid, NaOH.
0.27 \ Meq of H2C2O4 = Meq of NaOH
m= ´ 10 = 0.027 g
100 50 ´ 0.5 ´ 2 = 25 ´ M NaOH´ 1
Density of fatty acid, d = 0.9 g cm-3 M NaOH = 2M
\Volume of the fatty acid over the watch glass, Number of moles
Now, molarity =
m 0.027 Volume of solution (in L)
V = = = 0.03 cm3
d 0.9 Weight / molecular mass
Let, height of the cylindrical monolayer = h cm =
Volume of solution (in L)
Q Volume of the cylinder = Volume of fatty acid wNaOH 1000
2= ´
40 50
2 ´ 40 ´ 50
wNaOH = = 4g
h cm 1000
Thus, (*) none option is correct.
10 cm 7. The reaction of HCl with Na 2CO3 is as follows:
Þ V = pr2 ´ h 2HCl + Na 2CO3 ¾® 2NaCl + H2O + CO2
V 0.03 cm3 We know that, M eq of HCl = M eq of Na 2CO3
Þ h= =
pr 2
3 ´ (10)2 cm2 25 30
´ 1 ´ M HCl = ´ 0.1 ´ 2
1000 1000
= 1 ´ 10-4cm = 1 ´ 10-6 m
30 ´ 0.2 6
M HCl = = M
4. The oxidation of a mixture of one mole of each of 25 25
FeC2O4 , Fe2 (C2O4 )3 FeSO4 and The reaction of HCl with NaOH is as follows:
Fe2 (SO4 )3 in acidic medium with KMnO4 is as follows :
NaOH + HCl ¾® NaCl + H2O
FeC2O4 + KMnO4 ¾¾® Fe3 + + CO2 + Mn 2+ …(i)
Also, M eq of HCl = M eq of NaOH
Fe2(C2O4 )3 + KMnO4 ¾¾® Fe3 + + CO2 + Mn 2+ …(ii)
6 V 30
FeSO4 + KMnO4 ¾¾® Fe3 + + SO24- + Mn 2+ …(iii) ´1 ´ = ´ 0.2 ´ 1
25 1000 1000
Change in oxidation number of Mn is 5. Change in oxidation
number of Fe in (i), (ii) and (iii) are +3, + 6, + 1, respectively. V = 25 mL
neq KMnO 4 = neq [ FeC 2O 4 + Fe 2 ( C 2O 4 ) 3 + FeSO 4 ] 8. Reaction of oxalate with permanganate in acidic medium.
n ´ 5 = 1 ´ 3 + 1 ´ 6 + 1 ´1 5C2O24- + 2MnO-4 ¾® 10CO2 + 2Mn 2+ + 8H2O
\ n=2
n-factor : (4 - 3 ) ´ 2 = 2 (7 - 2 ) = 5
5. Given, W Ca (HCO 3) 2 = 0.81 g Number of mole 5 2 10
-
W Mg (HCO = 0.73 g Þ 5C2O42– ions transfer 10e to produce 10 molecules of CO2.
3) 2
-1
M Ca (HCO 3 ) 2 = 162 g mol , So, number of electrons involved in producing 10 molecules of
CO2 is 10. Thus, number of electrons involved in producing 1
M Mg(HCO 3 ) 2 = 146 mol -1 molecule of CO2 is 1.
Some Basic Concepts of Chemistry 17

9. We can calculate the simplest whole number ratio of C and H 12. n-factor of dichromate is 6.
from the data given, as Also, n-factor of Mohr’s salt is 1 as :
Element Relative Molar Relative Simplest whole O. A
FeSO4 (NH4 )2 SO4× 6H2O ¾® Fe3+
mass mass mole number ratio Mohr’s salt
C 6 12 6 0.5
= 0.5 =1 Q 1 mole of dichromate = 6 equivalent of dichromate.
12 0.5
H 1 1 1 1 \ 6 equivalent of Mohr’s salt would be required.
=1 =2
1 0.5 Since, n-factor of Mohr’s salt is 1, 6 equivalent of it would also
be equal to 6 moles.
Alternatively this ratio can also be calculated directly in the
terms of x and y as Hence, 1 mole of dichromate will oxidise 6 moles of Mohr’s salt.
12x 6 13. The following reaction occur between S2O32- and Cr2O72- :
= (given and molar mass of C = 12, H = 1)
y 1 26H+ + 3S2O32- + 4Cr2O72- ¾® 6SO24- + 8Cr 3+ + 13H2O
Now, after calculating this ratio look for condition 2 given in the
Change in oxidation number of Cr2O72- per formula unit is 6 (it is
question i.e. quantity of oxygen is half of the quantity required
to burn one molecule of compound C xH y completely to CO2 and always fixed for Cr2O72- ).
H2O. We can calculate number of oxygen atoms from this as Molecular weight
consider the equation. Hence, equivalent weight of K2Cr2O7 =
6
é yù y
CxH y + ê x + ú O2 ¾® xCO2 + H2O 14. It is an example of disproportionation reaction because the same
ë 4û 2 species (ClO- ) is being oxidised to ClO-3 as well as reduced to Cl - .
é yù é yù
Number of oxygen atoms required = 2 ´ ê x + ú = ê 2x + ú 15. Oxalic acid dihydrate H2C2O4 × 2H2O : mw = 126
ë 4û ë 2û
It is a dibasic acid, hence equivalent weight = 63
1é yù é yù
Now given, z = ê 2x + ú = ê x + ú 6.3 1000
2ë 2û ë 4û Þ Normality = ´ = 0.4 N
63 250
Here we consider x and y as simplest ratios for C and H so now Þ N 1V1 = N 2V2
putting the values of x and y in the above equation.
Þ 0.1 ´ V1 = 0.4 ´ 10
é yù é 2ù
z = ê x + ú = ê1 + ú = 1.5 Hence, V1 = 40 mL
ë 4û ë 4û
Thus, the simplest ratio figures for x , y and z are x = 1, y = 2 and 16. In MnO-4 , oxidation state of Mn is +7
z = 15
. In Cr(CN)63- , oxidation state of Cr is +3
Now, put these values in the formula given i.e.
In NiF62- , Ni is in + 4 oxidation state.
CxH yOz = C1H2O1.5
So, empirical formula will be [C1H2O1.5 ] ´ 2 = C2H4O3 In CrO2Cl 2, oxidation state of Cr is +6.

10. Methyl orange show Pinkish colour towards more acidic 17. In S8 , oxidation number of S is 0, elemental state.
medium and yellow orange colour towards basic or less acidic In S2F2, F is in – 1 oxidation state, hence S is in + 1 oxidation
media. Its working pH range is state.
In H2S, H is in +1 oxidation state, hence S is in – 2 oxidation
Pinkish
3.9 –4.5
Yellow state.
Red orange
18. The balanced redox reaction is :
Weak base have the pH range greater than 7. When methyl 3MnO-4 + 5FeC2O4 + 24H+ ¾® 3Mn 2+ + 5Fe3+
orange is added to this weak base solution it shows yellow + 10CO2 + 12H2O
orange colour. Q 5 moles of FeC2O4 require 3 moles of KMnO4
Now when this solution is titrated against 3
strong acid the pH move towards more acidic \ 1 mole of FeC2O4 will require mole of KMnO4.
1 5
range and reaches to end point near 3.9 where
HN NH2
yellow orange colour of methyl orange 19. The balanced chemical reaction is :
In conjugation
changes to Pinkish red resulting to similar 2MnO4- + 5SO32- + 6H+ ¾® 2Mn 2+ + 5SO24- + 3H2O
change in colour of solution as well.
Q 5 moles SO2-
3 reacts with 2 moles of KMnO 4
11. H2O2 acts as an oxidising as well as reducing agent, because 2
\ 1 mole of SO2-
3 will react with mole KMnO4.
oxidation number of oxygen in H2O2 is -1. So, it can be oxidised 5
to oxidation state 0 or reduced to oxidation state –2. 20. The balanced redox reaction is :
H2O2 decomposes on exposure to light. So, it has to be stored in 2MnO-4 + 5C2O42- + 16H + ¾® 2Mn 2+ + 10CO2 + 16H2O
plastic or wax lined glass bottles in dark for the prevention of
exposure. It also has to be kept away from dust. Hence, the coefficients of reactants in balanced reaction are 2, 5
and 16 respectively.
18 Some Basic Concepts of Chemistry

21. Volume strength of H2O2 = Normality ´ 5.6 = 1.5 ´ 5.6 = 8.4 V and 24 moles of NH3 = 24 ´17 = 408 g
Further, as given in question,
22. In Ba(H2PO2 )2, oxidation number of Ba is +2. Therefore, 24 moles of NH3 produced in reaction (i) is completly utilised
H2PO2- : 2 ´ (+1) + x + 2 ´ (-2) = - 1 by 952g or 4 moles of NiCl 2 × 6H2O to produce 4 moles of
Þ x=+1 [Ni(NH3 )6 ] Cl 2.
So, 4 moles of [Ni(NH3 )6 ] Cl 2 = 4 ´ 232 = 928gms
23. Equivalent weight in redox system is defined as :
Hence, total mass of gypsum and nickel ammonia coordination
Molar mass compound [Ni(NH3 )6 ] Cl 2 = 2064 + 928 = 2992
E=
n-factor
26. Both assertion and reason are factually true but the reason does
Here n-factor is the net change in oxidation number per formula not exactly explain the assertion. The correct explanation is,
unit of oxidising or reducing agent. In the present case, n-factor methyl orange and phenolphthalein changes their colour at
is 2 because equivalent weight is half of molecular weight. Also, different pH.
1
n-factor MnSO4 ¾® Mn 2O3 1 (+ 2 ¾® + 3) 27. If x is the oxidation state of Cu then :
2
7
MnSO4 ¾® MnO2 2 (+ 2 ¾® + 4) 3 + 2 ´ 2 + 3x + 7 ´ (- 2) = 0 Þ x =
3
MnSO4 ¾® MnO-4 5 (+ 2 ¾® + 7)
28. Na2S4O6 is a salt of H2S4O6 which has the following structure
MnSO4 ¾® MnO24- 4 (+ 2 ¾® + 6)
O O
Therefore, MnSO4 converts to MnO2. (0) (v)
HO S S S S OH
24. PLAN This problem includes concept of redox reaction. A redox
reaction consists of oxidation half-cell reaction and reduction
half-cell reaction. Write both half-cell reactions, i.e. oxidation O O
half-cell reaction and reduction half-cell reaction.Then balance Þ Difference in oxidation number of two types of sulphur = 5
both the equations.
Now determine the correct value of stoichiometry of H2SO4. 29. Only F and Na show only one non-zero oxidation state.
Oxidation half-reaction, 2 I - ¾® I2 + 2 e - …(i) O = O- , O2- , O2+ ;
Here, I - is converted into I2. Oxidation number of I is increasing Cl = - 1 to + 7
from –1 to 0 hence, this is a type of oxidation reaction. N = - 3 to + 5
Reduction half-reaction P = - 3 to + 5
6H+ + ClO-3 + 6e- ¾® Cl - + 3H2O …(ii) Sn = + 2, + 4

Here, H2 O releases as a product. Hence, option (d) is Tl = + 1, + 3 (rare but does exist)
correct. Ti = + 2, + 3, + 4
Multiplying equation (i) by 3 and adding in equation (ii)
30. Average titrate value is 25.15, but the number of significant
6I- + ClO3- + 6H+ ¾® Cl - + 3I2 + 3H2O figure cannot be greater than the same in either of them being
manipulated.
6I- + ClO3- + 6H2SO4 ¾® Cl - + 3I2 + 3H2O + 6HSO-4
31. The balanced reaction is

Stoichiometric coefficient of HSO-4 is 6.
6CaO + P4O10 ¾® 2Ca 3 (PO4 )2
Hence, option (a), (b) and (d) are correct.
852
Moles of P4O10 = =3
25. Balanced equations of reactions used in the problem are as 284
follows Moles of CaO required = 3 ´ 6 = 18
(i) ( NH4)2SO 4 + Ca(OH)2 ¾® CaSO 4 × 2H2O + 2NH3 Mass of CaO required = 18 ´ 56 = 1008 g
1 mol 1 mol 2 mol
132 g 172 g (2 ´ 17) = 34 g
32. Meq of oxalate = 10 ´ 0.2 ´ 2 = 4
(ii) NiCl 2 × 6H2O + 6NH3 ¾® [Ni(NH3)6] Cl 2 + 6H2O Meq of MnO2 formed = Meq of oxalate = 4
1 mol 6 mol 1 mol
238 g 102 g 232 g Meq of KMnO4 in 20 mL = 4
Now, in Eq. (i) Þ Normality of H2O2 ´ 20 = 4
if, 1584 g of ammonium sulphate is used. Þ Normality of H2O2 = 0.20 N
1584 0.20
i.e., 1584 g (NH4 )2 SO4 = = 12 mol Þ Molarity of H2O2 = = 0.10 M
132 2
So, according to the Eq. (i) given above 12 moles of (NH4 )2 SO4 The balanced reactions are
produces 2KMnO4 + 5H2O2 + 3H2SO4 ¾® 2MnSO4 + 5O2
(a) 12 moles of gypsum + K2SO4 + 8H2O
(b) 24 moles of ammonia MnO2 + Na 2C2O4 + 2H2SO4 ¾® MnSO4 + Na 2SO4
Here, 12 moles of gypsum = 12 ´172 = 2064 g + 2CO2 + 2H2O
Some Basic Concepts of Chemistry 19

33. The balanced chemical reaction is The first step volume contraction can be calculated as :
CuCO3 + H2SO4 ¾® CuSO4 + H2O + CO2 æ x ö
0.5 ´ 1000 ç x + + y + 2 y÷ - (x + y) = 13
millimol of CuCO3 = = 4.048 è 2 ø
123.5 Þ x + 4 y = 26 …(i)
Þ Millimol of H2SO4 required = 4.048 The second volume contraction is due to absorption of CO2.
Q Millimol = Molarity ´ Volume (in mL) Hence, x + y = 14 …(ii)
4.048
Þ Volume = = 8.096 mL
0.50 Now, solving equations (i) and (ii),
34. The redox reaction involved are x = 10 mL, y = 4 mL and volume of He = 20 – 14 = 6 mL
IO-3 + 5I- + 6H+ ¾® 3I2 + 3H2O 10
Þ Vol % of CO = ´ 100 = 50%
20
I2 + 2S2O32- ¾® 2I- + S4O62- 4
Vol % of CH4 = ´ 100= 20%
0.1 20
millimol of KIO3 used = ´ 1000 = 0.467
214 Vol % of He = 30%
Þ millimol of I2 formed = 3 ´ 0.467 = 1.4
Þ millimol of Na 2S2O3 consumed = 2 ´ 1.4 = 2.8
38. The redox reaction involved is :
2.8 H2O2 + 2I- + 2H+ ¾® 2H2O + I2
Þ Molarity of Na 2S2O3 = = 0.062 M
45 If M is molarity of H2O2 solution, then
35. Meq of H2O2 = Meq of I2 = Meq of Na 2S2O3 0.508 ´ 1000
5M = (Q 1 mole H2O2 ºº 1 mole I2)
254
If N is normality of H2O2, then
Þ M = 0.4
N ´ 25 = 0.3 ´ 20
Also, n-factor of H2O2 is 2, therefore normality of H2O2 solution
Þ N = 0.24 is 0.8 N.
Þ Volume strength = N ´ 5.6 = 1.334 V Þ Volume strength = Normality ´ 5.6 = 0.8 ´ 5.6 = 4.48 V
36. Let the original sample contains x millimol of Fe3O4 and 39. The reaction is
y millimol of Fe2O3. In the first phase of reaction, KIO3 + 2KI + 6HCl ¾® 3ICl + 3KCl + 3H2O
Fe3O4 + I- ¾® 3Fe2+ + I2 (n-factor of Fe3O4 = 2) KIO3 required for 20 mL original KI solution = 3 millimol.
Fe2O3 + I- ¾® 2Fe2+ + I2 (n-factor of Fe2O3 = 2) Þ 7.5 millimol KIO3 would be required for original 50 mL KI.
Þ Meq of I2 formed = Meq (Fe3O4 + Fe2O3 ) Þ Original 50 mL KI solution contain 15 millimol of KI.
= Meq of hypo required
After AgNO3 treatment 5 millimol of KIO3 is required, i.e. 10
Þ 2x + 2 y = 11 ´ 0.5 ´ 5 = 27.5 …(i) millimol KI is remaining.
Now, total millimol of Fe2+ formed = 3x + 2 y. In the reaction
Þ 5 millimol KI reacted with 5 millimol of AgNO3.
Fe2+ + MnO-4 + H+ ¾® Fe3+ + Mn 2+
5
Þ Mass of AgNO3 = ´ 170 = 0.85 g
n-factor of Fe2+ = 1 1000
Þ Meq of MnO-4 = Meq of Fe2+ Þ Mass percentage of AgNO3 = 85%
Þ 3x + 2 y = 12.8 ´ 0.25 ´ 5 ´ 2 = 32 …(ii) 40. CO2 is evolved due to following reaction :
Solving Eqs. (i) and (ii), we get
2NaHCO3 ¾® Na 2CO3 + H2O + CO2
x = 4.5 and y = 9.25
pV
4.5 Moles of CO2 produced =
Þ Mass of Fe3O4 = ´ 232 = 1.044 g RT
1000
1.044 750 123.9 1
% mass of Fe3O4 = ´ 100 = 34.80% = ´ ´
3 760 1000 0.082 ´ 298
Mass of Fe2O3 =
9.25
´ 160 = 1.48 g = 5 ´ 10-3
1000
Þ Moles of NaHCO3 in 2 g sample = 2 ´ 5 ´ 10-3 = 0.01
1.48
% mass of Fe2O3 = ´ 100 = 49.33% Þ millimol of NaHCO3 in 1.5 g sample
3
0.01
37. The reaction involved in the explosion process is = ´ 1.5 ´ 1000= 7.5
1 2
CO(g ) + O (g ) ¾® CO2 (g ) Let the 1.5 g sample contain x millimol Na 2CO3, then
2 x2
x mL mL x mL
2 2x + 7.5 = millimol of HCl = 15
CH4 (g ) + 2O2 (g ) ¾® CO2 (g ) + 2H2O(l ) Þ x = 3.75
y mL 2 y mL y mL
20 Some Basic Concepts of Chemistry

7.5 ´ 84 Þ Mass of H2C2O4 = 90 ´ 0.01 = 0.9 g


Þ Mass of NaHCO3 = = 0.63 g
1000 Mass of NaHC2O4 = 112 ´ 0.01 = 1.12 g
3.75 ´ 106 35.5
Mass of Na 2CO3 = = 0.3975 g 44. Mass of chlorine in 1.0 g X = ´ 2.9 = 0.717 g
1000 143.5
0.63
Þ % mass of NaHCO3 = ´ 100 = 42 % Now, the empirical formula can be derived as :
1.50
0.3975 C H Cl
% mass of Na 2CO3 = ´ 100
1.5 % wt : 24.24 4.04 71.72
= 26.5%
Mole : 2 4 2
41. Mass of Fe2O3 = 0.552 g Simple ratio : 1 2 1
0.552
millimol of Fe2O3 = ´ 1000 = 3.45 Þ Empirical formula = CH2Cl.
160
During treatment with Zn-dust, all Fe3+ is reduced to Fe2 + , Because X can be represented by two formula of which one
hence gives a dihydroxy compound with KOH indicates that X has two
chlorine atoms per molecule.
millimol of Fe2 + (in 100 mL) = 3.45 ´ 2 = 6.90
6.90 Þ X = C2H4Cl 2 with two of its structural isomers.
Þ In 25 mL aliquot, = 1.725 millimol Fe2+ ion. Cl— CH2— CH2 — Cl and CH3— CHCl 2
4
I II
Finally Fe2+ is oxidised to Fe3+ , liberating one electron per Fe2+
ion. Therefore, total electrons taken up by oxidant. On treatment with KOH, I will give ethane-1, 2-diol, hence it is
Y. Z on treatment with KOH will give ethanal as
= 1.725 ´ 10-3 ´ 6.023 ´ 1023
ClCH2CH2Cl + OH- ¾® CH2 — CH2
= 1.04 ´ 1021 ½ ½
OH OH
42. With KMnO4, oxalate ion is oxidised only as : (Y )
5C2O42- + 2MnO-4 + 16H+ ¾® 2Mn 2+ + 10CO2 + 8H2O –H2 O
CH3CHCl 2 + KOH ¾® CH3CH(OH)2 ¾¾® CH3CHO
Let, in the given mass of compound, x millimol of C2O42- ion is Unstable (Z )
present, then 45. Let the n-factor of KMnO4 in acid, neutral and alkaline media
-
Meq of C2O2-4 = Meq of MnO 4 are N 1, N 2 and N 3 respectively. Also, same volumes of reducing
Þ 2x = 0.02 ´ 5 ´ 22.6 Þ x = 1.13 agent is used everytime, same number of equivalents of KMnO4
At the later stage, with I - , Cu 2+ is reduced as : would be required every time.
100 5
2Cu 2+ + 4I - ¾® 2CuI + I2 Þ 20N 1 = N 2 = 100N 3 Þ N 1 = N 2 = 5N 3
3 3
and I2 + 2S2O23- ¾® 2I- + S4O62- Also, n-factors are all integer and greater than or equal to one but
Let there be x millimol of Cu 2+ . less than six, N 3 must be 1.
Þ Meq of Cu 2+ = Meq of I2 = meq of hypo Þ N 1 = 5, N 2 = 3
\ In acid medium MnO-4 ¾® Mn 2+
Þ x = 11.3 ´ 0.05 = 0.565
In neutral medium MnO-4 ¾® Mn 4+
2+
Þ Moles of Cu : moles of C2O2-
4 = 0.565 : 1.13 = 1 : 2 In alkaline medium MnO-4 ¾® Mn6+
43. Let us consider 10 mL of the stock solution contain x millimol Þ meq of K2Cr2O7 required = 100
oxalic acid H2C2O4 and y millimol of NaHC2O4. Þ 100 = 1 ´ 6 ´ V (n-factor = 6)
When titrated against NaOH, basicity of oxalic acid is 2 while Þ V = 100/ 6 = 16.67 mL
that of NaHC2O4 is 1. 1
Þ 2x + y = 3 ´ 0.1 = 0.3 … (i) 46. Meq of MnO-4 required = 20 ´ ´ 5 = 2
50
When titrated against acidic KMnO4, n-factors of both oxalic Þ Meq of Fe2+ present in solution = 2
acid and NaHC2O4 would be 2. Þ millimol of Fe2+ present in solution = 2 (n-factor = 1)
Þ 2x + 2 y = 4 ´ 0.1 = 0.4 … (ii) Also,
Solving equations (i) and (ii) gives Q 4 millimol of Fe2+ are formed from 1 millimol N2H4
y = 0.1, x = 0.1 1 1
\ 2 millimol Fe2+ from ´ 2 = millimol N2H4
0.1 4 2
Þ In 1.0 L solution, mole of H2C2O4 = ´ 100 = 0.01
1000 Therefore, molarity of hydrazine sulphate solution
0.1 1 1 1
Mole of NaHC2O4 = ´ 100 = 0.01 = ´ =
1000 2 10 20
Some Basic Concepts of Chemistry 21

1 49. During heating MCO3 is converted into MO liberating CO2


Þ In 1 L solution mol N2H6SO4 is present.
20 while BaO is remaining unreacted :
1
Þ Amount of N2H6SO4 = ´ 130 = 6.5 gL-1 Heat
MCO3 (s) ¾® MO(s) + CO2 (g ) ­ 0.44 g = 0.01 mol
20
BaO(s) BaO(s)
47. Molecular weight of Na 2CO3 ×10H2O = 286
4.08 g 3.64 g
1 1000
Þ Molarity of carbonate solution = ´ = 0.035 From the decomposition information, it can be deduced that the
286 100
original mixture contained 0.01 mole of MCO3 and the solid
Þ Normality of carbonate solution = 2 ´ 0.035 = 0.07 N residue, obtained after heating, contain 0.01 mole (10 millimol)
8´5 of MO.
In acid solution : Normality of HNO3 = = 0.02 Also, millimol of HCl taken initially = 100
2000
5 ´ 4.8 millimol of NaOH used in back-titration = 16 ´ 2.5 = 40
Normality of HCl = = 0.012 Þ millimol of HCl reacted with oxide residue = 60
2000
HCl reacts with oxides as :
Let normality of H2SO4 in final solution be N.
MO + 2HCl ¾® MCl 2 + H2O
Þ (N + 0.02 + 0.012) ´ 30 = 0.07 ´ 42.9 10 millimol 20 millimol

Þ N = 0.0681 BaO + 2HCl ¾® BaCl 2 + H2O


Þ Gram equivalent of SO2-
4 in 2 L solution = 2 ´ 0.0681 60 – 20 = 40 millimol
= 0.1362 Therefore, the residue contain 20 millimol of BaO.
96 Also, molar mass of BaO = 138 + 16
Þ Mass of SO2-
4 in solution = 0.1362 ´ = 6.5376 g = 154
2
154 ´ 20
Þ Mass of BaO = = 3.08 g
48. For the oxidation of A n+ as : 1000
A n+ ¾® AO-3 n-factor = 5 – n Þ Mass of MCO3 = 4.08 – 3.08 = 1.0 g
Þ Gram equivalent of A n+ = 2.68 ´ 10-3 (5 - n) Q 0.01 mole of MCO3 weight 1.0 g
Now equating the above gram equivalent with gram equivalent \ 1 mole of MCO3 = 100 g
of KMnO4 :
2.68 ´ 10-3 (5 - n) = 1.61 ´ 10-3 ´ 5 Þ 100 = (Atomic weight of metal) + (12 + 3 ´ 16)
Þ n=+2 Þ Atomic weight of metal = 40, i.e. Ca

Download Chapter Test


http://tinyurl.com/y2r7vwy5 or
2
Atomic Structure

Topic 1 Preliminary Developments and Bohr’s Model


Objective Questions I (Only one correct option) 5. Which of the graphs shown below does not represent the
1. Which one of the following about an electron occupying the relationship between incident light and the electron ejected
1s-orbital in a hydrogen atom is incorrect? (The Bohr radius from metal surface? (2019 Main, 10 Jan I)

is represented by a0 ) (2019 Main, 9 April II)


K.E. of K.E. of
(a) The electron can be found at a distance 2a0 from the e ss e ss
nucleus.
(b) The magnitude of the potential energy is double that of
its kinetic energy on an average. (a) (b)
(c) The probability density of finding the electron is 0 0
maximum at the nucleus. Energy of light Intensity of light
(d) The total energy of the electron is maximum when it is
at a distance a0 from the nucleus. Number K.E. of
of ess e ss
2. If p is the momentum of the fastest electron ejected from a
metal surface after the irradiation of light having wavelength
l, then for 1.5 p momentum of the photoelectron, the (c) (d)
wavelength of the light should be 0 0
(Assume kinetic energy of ejected photoelectron to be very Frequency of light Frequency of light
high in comparison to work function) (2019 Main, 8 April II)
4 3 6. A stream of electrons from a heated filament was passed
(a) l (b) l between two charged plates kept at a potential difference V
9 4
esu. If e and m are charge and mass of an electron,
2 1
(c) l (d) l respectively, then the value of h/ l (where, l is wavelength
3 2 associated with electron wave) is given by (2016 Main)
3. What is the work function of the metal, if the light of (a) 2 meV (b) meV
wavelength 4000 Å generates photoelectron of velocity
6 ´ 105 ms -1 from it? (c) 2meV (d) meV

(Mass of electron = 9 ´ 10-31 kg 7. Rutherford’s experiment, which established the nuclear


Velocity of light = 3 ´ 108 ms -1 model of the atom, used a beam of (2002, 3M)

Planck’s constant = 6.626 ´ 10 -34


Js (a) b-particles, which impinged on a metal foil and got
-19 absorbed
Charge of electron = 1.6 ´ 10 JeV-1 ) (2019 Main, 12 Jan I)
(b) g-rays, which impinged on a metal foil and got
(a) 4.0 eV (b) 2.1 eV scattered
(c) 0.9 eV (d) 3.1 eV
(c) helium atoms, which impinged on a metal foil and got
4. The ground state energy of hydrogen atom is -13.6 eV. The scattered
energy of second excited state of He+ ion in eV is (d) helium nuclei, which impinged on a metal foil and got
(2019 Main, 10 Jan II)
scattered
(a) -54.4 (b) -3.4
(c) -6.04 (d) -27.2
Atomic Structure 23

8. Rutherford’s alpha particle scattering experiment eventually 17. When alpha particles are sent through a thin metal foil, most
led to the conclusion that (1986, 1M) of them go straight through the foil, because (1984, 1M)
(a) mass and energy are related (a) alpha particles are much heavier than electrons
(b) electrons occupy space around the nucleus (b) alpha particles are positively charged
(c) neutrons are burried deep in the nucleus (c) most part of the atom is empty space
(d) the point of impact with matter can be precisely (d) alpha particles move with high velocity
determined
18. Many elements have non-integral atomic masses, because
9. The radius of an atomic nucleus is of the order of (1985, 1M) (a) they have isotopes (1984, 1M)
(a) 10-10 cm (b) 10-13 cm (b) their isotopes have non-integral masses
(c) 10-15 cm (d) 10-8 cm (c) their isotopes have different masses
(d) the constituents, neutrons, protons and electrons,
10. Bohr’s model can explain (1985, 1M)
combine to give fractional masses
(a) the spectrum of hydrogen atom only
(b) spectrum of an atom or ion containing one electron only Match the Columns
(c) the spectrum of hydrogen molecule
19. According to Bohr’s theory,
(d) the solar spectrum
En = Total energy K n = Kinetic energy
11. The increasing order (lowest first) for the values of e/m Vn = Potential energy rn = Radius of nth orbit
(charge/mass) for electron ( e ), proton (p), neutron (n) and Match the following : (2006, 6M)
alpha particle (a) is (1984, 1M)
(a) e, p, n, a (b) n, p, e, a Column I Column II
(c) n , p , a , e (d) n , a, p , e A. Vn / K n = ? p. 0
12. Rutherford’s scattering experiment is related to the size of B. If radius of nth orbit µ E nx , x = ? q. –1
the (1983, 1M)
(a) nucleus (b) atom (c) electron (d) neutron Angular momentum in lowest
C. r. –2
orbital
13. Rutherford’s experiment on scattering of a-particles showed
for the first time that the atom has 1
(1981, 1M)
D. µ Zy, y = ? s. 1
(a) electrons (b) protons rn
(c) nucleus (d) neutrons
Fill in the Blanks
Objective Questions II 20. The light radiations with discrete quantities of energy are
(One or more than one correct option) called ................ . (1993, 1M)
14. The energy of an electron in the first Bohr orbit of H-atom is 21. The mass of a hydrogen is …… kg. (1982, 1M)
–13.6 eV. The possible energy value(s) of the excited state(s) 22. Isotopes of an element differ in the number of …… in their
for electrons in Bohr orbits of hydrogen is (are) (1988) nuclei. (1982, 1M)
(a) - 3.4 eV (b) - 4.2 eV (c) - 6.8 eV (d) + 6.8 eV
23. Elements of the same mass number but of different atomic
15. The atomic nucleus contains (1988, 1M) numbers are known as …… . (1983, 1M)
(a) protons (b) neutrons
(c) electrons (d) photons Subjective Questions
16. The sum of the number of neutrons and proton in the isotope 24. With what velocity should an a-particle travel towards the
of hydrogen is (1986, 1M) nucleus of a copper atom so as to arrive at a distance 10-13 m
(a) 6 (b) 5 (c) 4 (d) 3 from the nucleus of the copper atom ? (1997 (C), 3M)
24 Atomic Structure

Topic 2 Advanced Concept (Quantum Mechanical Theory)


Electronic Configuration and Quantum Number
Objective Questions I (Only one correct option) The correct order of their increasing energies will be
(2019 Main, 8 April I)
1. Among the following, the energy of 2s-orbital is lowest in
(2019 Main, 12 April II)
(a) IV < III < II < I (b) I < II < III < IV
(a) K (b) H (c) Li (d) Na (c) IV < II < III < I (d) I < III < II < IV

2. The electrons are more likely to be found 7. If the de-Broglie wavelength of the electron in n th Bohr orbit in
. pa0 (a0 is Bohr radius), then
a hydrogenic atom is equal to 15
the value of n / Z is (2019 Main, 12 Jan II)
a Y (x) (a) 1.0 (b) 0.75 (c) 0.40 (d) 1.50
8. The de-Broglie wavelength ( l ) associated with a photoelectron
b x
–x varies with the frequency (n ) of the incident radiation as, [n 0 is
threshold frequency] (2019 Main, 11 Jan II)
c 1 1
(a) l µ 1
(b) l µ 3

(2019 Main, 12 April I) (n - n 0 ) 4 (n - n 0 ) 2


(a) in the region a and c (b) in the region a and b 1 1
(c) l µ (d) l µ
(c) only in the region a (d) only in the region c (n - n 0 ) 1
(n - n 0 ) 2
3. The ratio of the shortest wavelength of two spectral series
of hydrogen spectrum is found to be about 9. The spectral 9. Which of the following combination of statements is true
series are (2019 Main, 10 April II) regarding the interpretation of the atomic orbitals?
(a) Lyman and Paschen (b) Brackett and Pfund (2019 Main, 9 Jan II)

(c) Paschen and Pfund (d) Balmer and Brackett I. An electron in an orbital of high angular momentum stays
away from the nucleus than an electron in the orbital of
4. The graph between | y |2 and r (radial distance) is shown lower angular momentum.
below. This represents (2019 Main, 10 April I) II. For a given value of the principal quantum number, the size
of the orbit is inversely proportional to the azimuthal
quantum number.
2 III. According to wave mechanics, the ground state angular
|Y| h
momentum is equal to .
2p
IV. The plot of y vs r for various azimuthal quantum numbers,
shows peak shifting towards higher r value.
r (a) I, III (b) II, III (c) I, II (d) I, IV
(a) 1s-orbital (b) 2 p-orbital 10. Heat treatment of muscular pain involves radiation of
(c) 3s-orbital (d) 2s-orbital wavelength of about 900 nm. Which spectral line of H-atom is
5. For any given series of spectral lines of atomic hydrogen, suitable for this purpose? [RH = 1´ 105 cm–1 ,
let Dn = n max - n min be the difference in maximum and h = 6.6 ´ 10-34 Js, c = 3 ´ 108 ms -1 ] (2019 Main, 11 Jan I)
minimum frequencies in cm -1 . The ratio (a) Paschen, 5 ®3 (b) Paschen, ¥ ® 3
D nLyman / D nBalmer is (2019 Main, 9 April I) (c) Lyman, ¥ ® 1 (d) Balmer, ¥ ® 2
(a) 27 : 5 (b) 5 : 4 (c) 9 : 4 (d) 4 : 1
6. The quantum number of four electrons are given below: 11. For emission line of atomic hydrogen from ni =8 to n f = n, the
1 æ 1 ö
I. n = 4 , l = 2, ml = - 2, ms = - plot of wave number (n ) against ç 2 ÷ will be (The Rydberg
2 èn ø
1
II. n = 3, l = 2, ml = 1, ms = + constant, RH is in wave number unit)
2 (2019 Main, 9 Jan I)
1 (a) non linear
III. n = 4 , l = 1, ml = 0, ms = +
2 (b) linear with slope -RH
1
IV. n = 3, l = 1, ml = 1, ms = - (c) linear with slope RH
2 (d) linear with intercept -RH
Atomic Structure 25

12. The radius of the second Bohr orbit for hydrogen atom is 18. The number of radial nodes in 3s and 2p respectively are
(Planck’s constant ( h ) = 6.6262 ´ 10- 34 Js; mass of (a) 2 and 0 (b) 0 and 2 (2005, 1M)
- 31 (c) 1 and 2 (d) 2 and 1
electron = 91091
. ´ 10 kg ; charge of electron
19. Which hydrogen like species will have same radius as that of
( e ) = 160210
. ´ 10- 19 C; permitivity of vacuum
Bohr orbit of hydrogen atom? (2004, 1M)
(Î0 ) = 8.854185 ´ 10- 12kg - 1 m - 3A 2 ) (2017 Main) (a) n = 2, Li 2+ (b) n = 2, Be3+
(a) 1.65 Å (b) 4.76 Å (c) 0.529 Å (d) 2.12 Å (c) n = 2, He+ (d) n = 3, Li 2+
13. P is the probability of finding the 1s electron of hydrogen 20. If the nitrogen atom had electronic configuration 1s7 , it would
atom in a spherical shell of infinitesimal thickness, dr, have energy lower than that of the normal ground state
at a distance r from the nucleus. The volume of this shell is configuration 1s2 2s2 2 p 3 , because the electrons would be
4 pr2 dr. The qualitative sketch of the dependence of P on r is closer to the nucleus, yet 1s7 is not observed, because it
(2016 Adv.)
violates
(a) Heisenberg uncertainty principle (2002, 3M)
P P (b) Hund’s rule
(c) Pauli exclusion principle
(a) (b) (d) Bohr postulate of stationary orbits
1 1
21. The quantum numbers + and - for the electron spin
2 2
0 r 0 r represent (2001, 1M)
(a) rotation of the electron in clockwise and anti-clockwise
direction respectively
(b) rotation of the electron in anti-clockwise and clockwise
P P
direction respectively
(c) magnetic moment of the electron pointing up and down
(c) (d) respectively
(d) two quantum mechanical spin states which have no classical
analogue
0 r 0 r 22. The wavelength associated with a golf ball weighing 200 g and
moving at a speed of 5 m/h is of the order (2001, 1M)
14. Which of the following is the energy of a possible excited
(a) 10-10 m (b) 10-20 m
state of hydrogen? (2015 Main)
(a) + 13.6 eV (b) – 6.8 eV (c) 10-30 m (d) 10-40 m
(c) –3.4 eV (d) + 6.8 eV 23. The number of nodal planes in a px orbital is (2001, 1M)
15. The correct set of four quantum numbers for the valence (a) one (b) two
electrons of rubidium atom ( Z = 37 ) is (2013 Main) (c) three (d) zero
1 1
(a) 5, 0, 0, + (b) 5, 1, 0, + 24. The electronic configuration of an element is
2 2
1 1 1s2 , 2s2 2 p 6 , 3s2 3 p 6 3d 5 , 4 s1 . This represents its (2000, 1M)
(c) 5, 1, ,1, + (d) 5, 0, 1, + (a) excited state (b) ground state
2 2
(c) cationic form (d) anionic form
16. Energy of an electron is given by
æ Z2 ö 25. The electrons, identified by quantum numbers n and l,
E = - 2.178 ´ 10-18 J ç 2 ÷ (i) n = 4, l = 1, (ii) n = 4, l = 0, (iii) n = 3, l = 2, (iv) n = 3, l = 1
çn ÷
è ø (2013 Main) can be placed in order of increasing energy, from the lowest to
Wavelength of light required to excite an electron in an highest, as (1999, 2M)
hydrogen atom from level n = 1 to n = 2 will be (a) (iv) < (ii) < (iii) < (i) (b) (ii) < (iv) < (i) < (iii)
(h = 6.62 ´ 10-34 Js and c = 3.0 ´ 108 ms -1 ) (c) (i) < (iii) < (ii) < (iv) (d) (iii) < (i) < (iv) < (ii)

(a) 1.214 ´ 10-7 m (b) 2.816 ´ 10-7 m 26. The energy of an electron in the first Bohr orbit of H-atom is
–13.6 eV. The possible energy value(s) of the excited state(s)
(c) 6.500 ´ 10-7 m (d) 8.500 ´ 10-7 m
for electrons in Bohr orbits of hydrogen is (are) (1998, 2M)
17. The kinetic energy of an electron in the second Bohr orbit (a) –3.4 eV (b) – 4.2 eV (c) – 6.8 eV (d) + 6.8 eV
of a hydrogen atom is [a0 is Bohr radius] (2012)
27. For a d-electron, the orbital angular momentum is (1997, 1M)
h2 h2 h2 h2 æ hö æ hö æ hö æ hö
(a) (b) (c) (d) (a) 6 ç ÷ (b) 2 ç ÷ (c) ç ÷ (d) 2 ç ÷
4p2 ma02 16p2 ma02 32p2 ma02 64p2 ma02 è 2p ø è 2p ø è 2p ø è 2p ø
26 Atomic Structure

28. The first use of quantum theory to explain the structure of 38. The ratio of the energy of a photon of 200 Å wavelength
atom was made by (1997, 1M) radiation to that of 4000 Å radiation is (1986, 1M)
(a) Heisenberg (b) Bohr 1
(a) (b) 4
(c) Planck (d) Einstein 4
29. Which of the following has the maximum number of unpaired 1
(c) (d) 2.
electrons ? (1996, 1M) 2
(a) Mg2+ (b) Ti 3+ (c) V3+ (d) Fe2+ 39. Which one of the following sets of quantum numbers
30. The orbital angular momentum of an electron in 2s-orbital is represents an impossible arrangement? (1986, 1M)
(1996, 1M) n l m s
1 h 1
(a) + × (b) zero (a) 3 2 –2
2 2p 2
h h 1
(c) (d) 2 × (b) 4 0 0
2p 2p 2
31. Which of the following relates to photons both as wave 1
(c) 3 2 –3
motion and as a stream of particles ? (1992, 1M) 2
(a) Interference (b) E = mc2 1
(d) 5 3 0 -
(c) Diffraction (d) E = hn 2
32. Which of the following does not characterise X-rays ? 40. Electromagnetic radiation with maximum wavelength is
(a) The radiation can ionise gases (1992, 1M) (a) ultraviolet (b) radio wave (1985, 1M)
(b) It causes ZnS to fluoresce (c) X-ray (d) infrared
(c) Deflected by electric and magnetic fields 41. Which electronic level would allow the hydrogen atom to
(d) Have wavelengths shorter than ultraviolet rays absorb a photon but not to emit a photon? (1984, 1M)
33. The correct set of quantum numbers for the unpaired electron (a) 3s (b) 2p
of chlorine atom is (1989, 1M) (c) 2s (d) 1s
n l m n l m 42. Correct set of four quantum numbers for the valence
(a) 2 1 0 (b) 2 1 1 (outermost) electron of rubidium ( Z = 37 ) is (1984, 1M)
(c) 3 1 1 (d) 3 0 0 1 1
(a) 5, 0, 0, + (b) 5, 1, 0 , +
34. The correct ground state electronic configuration of 2 2
1 1
chromium atom is (1989, 1M) (c) 5, 1, 1, + (d) 6, 0, 0, +
(a) [ Ar ] 3d 5 4 s1 (b) [ Ar ] 3d 4 4 s2 2 2

(c) [ Ar ] 3d 6 4 s0 (d) [ Ar ] 4 d 5 4 s1 43. The principal quantum number of an atom is related to the
(a) size of the orbital (1983, 1M)
35. The outermost electronic configuration of the most (b) spin angular momentum
electronegative element is (1988, 90, 1M) (c) orientation of the orbital in space
(a) ns2 np3 (b) ns2 np4 (d) orbital angular momentum
(c) ns2 np5 (d) ns2 np6 44. Any p-orbital can accommodate upto (1983, 1M)
36. The orbital diagram in which the Aufbau principle is violated (a) four electrons
(1988, 1M) (b) six electrons
(c) two electrons with parallel spins
(a)
(d) two electrons with opposite spins

(b)
Objective Questions II
(One or more than one correct option)
(c)
45. The ground state electronic configuration of nitrogen atom
(d) can be represented by (1999, 3M)

(a)
37. The wavelength of a spectral line for an electronic transition is
inversely related to (1988, 1M) (b)
(a) the number of electrons undergoing the transition
(b) the nuclear charge of the atom (c)
(c) the difference in the energy of the energy levels involved in
the transition (d)
(d) the velocity of the electron undergoing the transition
Atomic Structure 27

46. Which of the following statement (s) is (are) correct ? Passage Based Questions
(1998, 2M)
(a) The electronic configuration of Cr is [Ar] 3d 4 s (atomic 5 1 The hydrogen-like species Li2+ is in a spherically symmetric
number of Cr = 24) state S 1 with one radial node. Upon absorbing light the ion
undergoes transition to a state S 2 . The state S 2 has one radial
(b) The magnetic quantum number may have a negative value
node and its energy is equal to the ground state energy of the
(c) In silver atom, 23 electrons have a spin of one type and 24 of
hydrogen atom.
the opposite type. (atomic number of Ag = 47)
(d) The oxidation state of nitrogen in HN3 is – 3 49. The state S 1 is (2010)
76 (a) 1s (b) 2s (c) 2p (d) 3s
47. An isotone of 32 Ge is (1984, 1M)
77 77 50. Energy of the state S 1 in units of the hydrogen atom ground
(a) 32 Ge (b) 33 As
77 78
state energy is (2010)
(c) 34 Se (d) 34 Se (a) 0.75 (b) 1.50 (c) 2.25 (d) 4.50
51. The orbital angular momentum quantum number of the state
Assertion and Reason S 2 is (2010)
Read the following questions and answer as per the direction (a) 0 (b) 1 (c) 2 (d) 3
given below :
(a) Both Statement I and Statement II are correct; Statement Match the Columns
II is the correct explanation of Statement I Answer Q. 52, Q. 53 and Q. 54 by appropriately matching
(b) Both Statement I and Statement II are correct; Statement the information given in the three columns of the
II is not the correct explanation of Statement I following table.
(c) Statement I is correct; Statement II is incorrect The wave function, y n , l , ml is a mathematical function
(d) Statement I is incorrect; Statement II is correct whose value depends upon spherical polar coordinates
48. Statement I The first ionisation energy of Be is greater than ( r, q , f ) of the electron and characterised by the quantum
that of B. number n , l and ml . Here r is distance from nucleus, q is
colatitude and f is azimuth. In the mathematical functions
Statement II 2p-orbital is lower in energy than 2s. (2000)
given in the Table, Z is atomic number and a0 is Bohr radius.
(2017 Adv.)

Column 1 Column 2 Column 3

3 æ Zr ö yn, I, ml (r)
æ Z ö 2 - çç a ÷÷
(I) 1s-orbital (i) yn, l , ml µ çç ÷÷ e è 0 ø (P)
è a0 ø
0
r/a0

1
(II) 2s-orbital (ii) One radial node (Q) Probability density at nucleus µ
a03
5 æ Zr ö
æ Z ö 2 - çç a ÷÷
(III) 2 pz-orbital (iii) yn, l ml µ çç ÷÷ re è 0 ø cosq (R) Probability density is maximum at nucleus
è a0 ø

Energy needed to excite electron from n = 2 state to n = 4 state


27
(IV) 3 dz2-orbital (iv) xy-plane is a nodal plane (S) is times the energy needed to excite electron from n = 2
32
state to n = 6 state

52. For He+ ion, the only INCORRECT combination is


(a) (I) (i) (S) (b) (II) (ii) (Q) (c) (I) (iii) (R) (d) (I) (i) (R)
53. For the given orbital in Column 1, the Only CORRECT combination for any hydrogen-like species is
(a) (II) (ii) (P) (b) (I) (ii) (S) (c) (IV) (iv) (R) (d) (III) (iii) (P)
54. For hydrogen atom, the only CORRECT combination is
(a) (I) (i) (P) (b) (I) (iv) (R) (c) (II) (i) (Q) (d) (I) (i) (S)
28 Atomic Structure

55. Match the entries in Column I with the correctly related 68. In an atom, the total number of electrons having quantum
quantum number(s) in Column II. (2008, 6M) numbers (2014 Adv.)
1
Column I Column II n = 4, | ml | = 1 and ms = - is
2
A. Orbital angular momentum p. Principal 69. The atomic masses of He and Ne are 4 and 20 amu,
of the electron in a quantum respectively. The value of the de-Broglie wavelength of He
hydrogen-like atomic orbital. number gas at -73°C is ‘M’ times that of the de-Broglie wavelength
B. A hydrogen-like one-electron q. Azimuthal of Ne at 727°C. M is (2013 Adv.)

wave function obeying quantum 70. The work function (f) of some metals is listed below. The
Pauli’s principle. number number of metals which will show photoelectric effect when
light of 300 nm wavelength falls on the metal is (2011)
C. Shape, size and orientation r. Magnetic
of hydrogen-like atomic quantum Metal Li Na K Mg Cu Ag Fe Pt W
orbitals. number F (eV) 2.4 2.3 2.2 3.7 4.8 4.3 4.7 6.3 4.75

D. Probability density of s. Electron spin 71. The maximum number of electrons that can have principal
electron at the nucleus in quantum quantum number, n = 3 and spin quantum number,
hydrogen-like atom. number ms = -1 / 2 , is (2011)

Fill in the Blanks Subjective Questions


56. The outermost electronic configuration of Cr is 72. (a) Calculate velocity of electron in first Bohr orbit of hydrogen
atom (Given, r = a0 ).
.......................... . (1994, 1M)
(b) Find de-Broglie wavelength of the electron in first Bohr
57. 8 g each of oxygen and hydrogen at 27°C will have the total orbit.
kinetic energy in the ratio of .......... . (1989, 1M)
(c) Find the orbital angular momentum of 2p-orbital in terms of
58. The uncertainty principle and the concept of wave nature of h / 2p units. (2005, 2M)
matter were proposed by ............ and .............respectively.
(1988, 1M)
73. (a) The Schrodinger wave equation for hydrogen atom is
3/ 2
1 æ 1ö æ rö
59. Wave functions of electrons in atoms and molecules are y 2s = çç ÷÷ çç 2 - ÷÷ e-r / 2a 0
called .............. . (1993, 1M) 4 (2p )1/ 2 è a0 ø è a0ø

60. The 2 px, 2 p y and 2 p z orbitals of atom have identical shapes where, a0 is Bohr’s radius. Let the radial node in 2s be at r0.
Then, find r in terms of a0.
but differ in their ........... . (1993, 1M)
(b) A base ball having mass 100 g moves with velocity
61. When there are two electrons in the same orbital, they have 100 m/s. Find out the value of wavelength of base ball.
…… spins. (1983, 1M) (2004, 2M)
74. The wavelength corresponding to maximum energy for
True/False hydrogen is 91.2 nm. Find the corresponding wavelength for
62. In a given electric field, b-particles are deflected more than He+ ion. (2003, 2M)
a-particles in spite of a-particles having larger charge. 75. Calculate the energy required to excite 1 L of hydrogen gas
(1993, 1M) at 1 atm and 298 K to the first excited state of atomic
63. The electron density in the XY-plane in 3d x 2 - 2 orbital is hydrogen. The energy for the dissociation of H—H bond is
y
zero. (1986, 1M) 436 kJ mol -1 . (2000)
64. The energy of the electron in the 3d-orbital is less than that in 76. An electron beam can undergo diffraction by crystals.
the 4s-orbital in the hydrogen atom. (1983, 1M) Through what potential should a beam of electrons be
65. Gamma rays are electromagnetic radiations of wavelengths accelerated so that its wavelength becomes equal to 1.54 Å.
(1997 (C), 2M)
of 10-6 to 10-5 cm. (1983, 1M)
77. Consider the hydrogen atom to be proton embedded in a
66. The outer electronic configuration of the ground state
cavity of radius a0 (Bohr’s radius) whose charge is
chromium atom is 3d 4 4 s2 . (1982, 1M)
neutralised by the addition of an electron to the cavity in
vacuum, infinitely slowly. Estimate the average total energy
Integer Answer Type Questions of an electron in its ground state in a hydrogen atom as the
67. Not considering the electronic spin, the degeneracy of the work done in the above neutralisation process. Also, if the
second excited state ( n = 3 ) of H-atom is 9, while the magnitude of the average kinetic energy is half the
degeneracy of the second excited state of H- is (2015 Adv.) magnitude of the average potential energy, find the average
potential energy. (1996, 2M)
Atomic Structure 29

78. Calculate the wave number for the shortest wavelength 84. What is the maximum number of electrons that may be
transition in the Balmer series of atomic hydrogen.(1996, 1M) present in all the atomic orbitals with principal quantum
number 3 and azimuthal quantum number 2 ? (1985, 2M)
79. Iodine molecule dissociates into atoms after absorbing light
to 4500Å. If one quantum of radiation is absorbed by each 85. Give reason why the ground state outermost electronic
molecule, calculate the kinetic energy of iodine atoms. configuration of silicon is (1985, 2M)
(Bond energy of I2 = 240 kJ mol –1 ) 3s 3p 3s 3p
(1995, 2M)
and not
80. Find out the number of waves made by a Bohr’s electron in
one complete revolution in its 3rd orbit. (1994, 3M) 86. The electron energy in hydrogen atom is given by
81. What transition in the hydrogen spectrum would have the 21.7 ´ 10-12
En = - erg. Calculate the energy required to
same wavelength as the Balmer transition n = 4 to n = 2 of n2
He+ spectrum? (1993, 3M) remove an electron completely from the n = 2 orbit. What is
82. Estimate the difference in energy between 1st and 2nd the longest wavelength (in cm) of light that can be used to
Bohr’s orbit for a hydrogen atom. At what minimum atomic cause this transition? (1984, 3M)
number, a transition from n = 2 to n = 1 energy level would 87. Calculate the wavelength in Angstroms of the photon that is
result in the emission of X-rays with l = 3.0 ´ 10–8 m ? emitted when an electron in the Bohr’s orbit, n = 2 returns to
Which hydrogen atom-like species does this atomic number the orbit, n = 1in the hydrogen atom. The ionisation potential
correspond to? (1993, 5M) of the ground state hydrogen atom is 2.17 ´ 10-11 erg per
83. According to Bohr’s theory, the electronic energy of atom. (1982, 4M)
hydrogen atom in the nth Bohr’s orbit is given by : 88. The energy of the electron in the second and third Bohr’s
-21.7 ´ 10 -19 orbits of the hydrogen atom is - 5.42 ´ 10-12 erg and
En = J
n2 - 2.41 ´ 10-12 erg respectively. Calculate the wavelength of
Calculate the longest wavelength of electron from the third the emitted light when the electron drops from the third to the
Bohr’s orbit of the He+ ion. (1990, 3M) second orbit. (1981, 3M)

Answers
Topic 1 33. (c) 34. (a) 35. (c) 36. (b)
1. (d) 2. (a) 3. (b) 4. (c) 37. (c) 38. (d) 39. (c) 40. (b)
5. (d) 6. (c) 7. (d) 8. (b) 41. (d) 42. (a) 43. (a) 44. (d)
9. (b) 10. (b) 11. (d) 12. (a) 45. (a,d) 46. (a,b,c) 47. (b,d) 48. (c)
13. (c) 14. (a) 15. (a,b) 16. (d) 49. (b) 50. (c) 51. (b) 52. (c)
17. (a,c) 18. (a,c) 19. A ® r; B ® q; C ® p, D ® s 53. (a) 54. (d)
20. (photons) 21. (1 .66 ´ 10 –27 kg) 22. (neutrons) 55. A ® q; B ® p, q, r, s C ® p, q, r D ® p, q, r
23. (isobars) 24. 6.3 ´ 10 6
56. Cr = [Ar] 3d 5 , 4s 1 57. 1 : 16

Topic 2 58. Heisenberg, de-Broglie. 59. orbital


1. (a) 2. (a) 3. (a) 4. (d) 60. Orientation in space 61. opposite
5. (c) 6. (c) 7. (b) 8. (d) 62. True 63. False 64. True 65. False
9. (d) 10. (b) 11. (c) 12. (d) 66. False 67. (3) 68. (6) 69. (5)
13. (c) 14. (c) 15. (a) 16. (a) 70. (4.14 eV) 71. (9) 74. (22.8 nm) 75. (98.44 kJ)
17. (c) 18. (a) 19. (b) 20. (c) 76. (63.56 V) 78. (2.725´ 10 6 M -1)
21. (d) 22. (c) 23. (a) 24. (b) 79. (2.16 ´ 10 20 J/atom) 83. (471 nm) 84. (10)
25. (a) 26. (a) 27. (a) 28. (b) -5
86. (3.66 ´ 10 cm) 87. (1220 Å) 88. (660 nm)
29. (d) 30. (b) 31. (a) 32. (c)
Hints & Solutions
Topic 1 Preliminary Developments l2 æ p ö
2
æ 2ö 4
2
Þ =ç ÷ =ç ÷ =
and Bohr’s Model . ´ p ÷ø
l çè 15 è 3ø 9
1. Statement (d) is incorrect. For 1s-orbital radial probability 4 éQ l 1 = l ù
density (R 2 ) against r is given as: Þ l2 = l
9 êë p1 = p úû

3. Work function of metal (f) = hn 0


where, n 0 = threshold frequency
1
Also, mev 2 = hn - hn 0
R2 2
1s
1
or mev 2 = hn - f …(i)
2
r 1 hc
mev 2 = -f …(ii)
2 l
For 1s-orbital, probability density decreases sharply as we move
away from the nucleus. Given : l = 4000 Å = 4000 ´ 10-10m
The radial distribution curves obtained by plotting radial v = 6 ´ 105 ms -1,
probability functions vs r for 1s-orbital is
me = 9 ´ 10-31 kg, c = 3 ´ 108 ms -1
h = 6.626 ´ 10-34 Js
Thus, on substituting all the given values in Eq. (i), we get
4prR2

1
´ 9 ´ 10-31 kg ´ (6 ´ 105 ms -1 )2
2
6.626 ´ 10-34 J s ´ 3 ´ 108 ms -1
= -f
4000 ´ 10-10 m
r \ . ´ 10-21 kgm 2s -2 - 4.96 ´ 10-19 J
f = 162
The graph initially increases and then decreases. It reaches a = 3.36 ´ 10-19 J [1 kg m2s -2 = 1J]
maximum at a distance very close to the nucleus and then = 21
. eV
decreases. The maximum in the curve corresponds to the
distance at which the probability of finding the electron in 4. The ground state energy of H-atom is + 13.6 eV.
maximum. For second excited state, n = 2 + 1 = 3
2. The expression of kinetic energy of photo electrons, Z2
\ E3 (He+ ) = - 13.6 ´ 2 eV [Q for He+ , Z = 2]
1 n
KE = mv 2 = E - E0
2 22
= - 13.6 ´ 2 eV = - 6.04 eV
When, KE > > E0, the equation becomes, 3
1
KE = mv 2 = E 5. For photoelectric effect,
2
(i) KE = E - E0
1 2 hc p2 hc
Þ mv = Þ =
2 l 2m2 l
1 1
Þ l = hc ´ 2m2 ´ 2 Þ l µ 2
p p |Y|2 For 1s-orbital
number of radial node = 1–0–1=0
hc
E= = energy of incident light.
l
E0 = threshold energy or work functions, r

1 2 1 (mv )2 1 p2 where,
mv = ´ = ´ 2 KE = Kinetic energy of ejected electrons.
2 2 m2 2 m
E = Energy of incident light = hn
Q p = momentum = mv
E0 = Threshold energy = hn0
As per the given condition, n = Frequency of incident light
2
l 2 æ p1 ö n0 = Threshold frequency
=ç ÷
l 1 çè p2 ÷ø
Atomic Structure 31

8. According to Rutherford’s model, there is a heavily positively


charged nucleus and negatively charged electrons occupies space
KE around it in order to maintain electro-neutrality.
0
E0 E 9. Radius of a nucleus is in the order of 10-13 cm, a fact.
10. Bohr’s model is applicable to one-electron system only.
11. Neutron has no charge, hence e/ m is zero for neutron. Next,
Slope = ± 1, intercept = - E0 a-particle (He2+ ) has very high mass compared to proton and
electron, therefore very small e/ m ratio. Proton and electron
So, option (a) is correct.
have same charge (magnitude) but former is heavier, hence has
(ii) KE of ejected electrons does not depend on the intensity of
smaller value of e/ m.
incident light.
e
: n<a < p<e
KE m
12. The negligibly small size of nucleus compared to the size of
atom was first established in Rutherford’s experiment.
13. The most important findings of Rutherford’s experiment is
0 discovery of nucleus.
Intensity of light
14. Energy of electron in H-atom is determined by the expression:
So, option (b) is correct. 13.6
(iii) When, number of ejected electrons is plotted with frequency En = - eV where, n =1, 2, 3, ....
n2
of light, we get 13.6
In excited states, E2 = - = - 3.4 eV
4
13.6
Number E3 = - = - 1.51 eV etc.
of e–’s 9
15. Nucleus is composed of neutrons and protons.
0
Frequency of light (n)
16. The isotopes of hydrogen are 1 H2 and 1H3.

So, option (c) is also correct. 17. Alpha particles passes mostly undeflected when sent through
(iv) KE = hn - hn 0 thin metal foil mainly, because
(i) it is much heavier than electrons.
(ii) most part of atom is empty space.
KE 18. Many elements have several isotopes. For such elements,
0 n0 n atomic mass is average of the atomic masses of different
isotopes, which is usually non-integral.
1 æ Ze2 ö
19. A. Vn = - ç ÷
4 pe0 ç r ÷
Slope = + h, intercept = - hn 0. So, option (d) is not correct. è ø

6. 1 æ Ze2 ö
Plan As you can see in options, energy term is mentioned hence, we Kn = ç ÷
have to find out relation between h / l and energy. For this, we 8pe0 ç r ÷
è ø
shall use de-Broglie wavelength and kinetic energy term in eV.
Vn
de-Broglie wavelength for an electron (l ) =
h Þ = - 2 ¾ (r )
p Kn

Þ p=
h Ze2
…(i) B. En = - µ r -1
l 8pe0r
Kinetic energy of an electron = eV Þ x = - 1 ¾ (q )
p2 h
As we know that, KE = C. Angular momentum = l (l + 1) = 0 in 1s-orbital
2m 2p ¾ ( p).
p2 a0n2 1
\ eV = or p = 2 meV …(ii) D. rn = Þ µ Z ¾ (s)
2m Z rn
h
From equations (i) and (ii), we get = 2 meV 20. Photons have quantised energy.
l
7. Rutherford used a-particle (He 2+
nuclei) in his experiment. 10-3
21. Mass of one H-atom = kg = 1.66 ´ 10-27 kg
6.023 ´ 1023
32 Atomic Structure

22. Isotopes have different number of neutrons. 3. According to Rydberg’s equation,


23. Isobars have same mass number but different atomic numbers. 1 RH æ 1 1ö 1 æ 1 1ö
= ç 2 - 2 ÷ or µ çç 2 - 2 ÷÷
l hc çn ÷ l è n1 n2 ø
24. When a-particle stop at 10-13m from nucleus, kinetic energy is è 1 n2 ø
zero, i.e. whole of its kinetic energy at the starting point is now For shortest wavelength, i.e. highest energy spectral line, n2 will
converted into potential energy. be (¥ ).
Potential energy of this a-particle can be determined as For the given spectral series, ratio of the shortest wavelength of
two spectral series can be calculated as follows :
Z1 ´ Z2e2
PE = - 1 1 1
(4 pe0 ) r - -0
l L 32 ¥ 2 9 1
(a) = = =
(Z1 = + 2, Z2 = + 29, e0 = 8.85 ´ 10-12 J-1 C2 m -1, lP 1
-
1 1 - 0 9
r = 10-13 m) 12 ¥ 2
-19 2 1 1
2 ´ 29 ´ (1.6 ´ 10 ) - 2
Þ | PE | = J l Bk 2
¥ = 1 ´ 16 = 16
4 ´ 3.14 ´ 8.85 ´ 10-12 ´ 10-13 (b) = 5
l Pf 1
-
1 25 1 25
= 1.33 ´ 10-13 J 42 ¥ 2
= kinetic energy of a-particle at t = 0 1 1
- 2
lP 2
¥ = 1 ´9= 9
Þ
1
KE = mv 2 = 1.33 ´ 10-13 (c) = 5
2 l Pf 1
-
1 25 1 25
32 ¥ 2
2 ´ 1.33 ´ 10-13
Þ v= = 6.3 ´ 106 ms-1 1
- 2
1
4 ´ 1.66 ´ 10-27 lB 2
¥ = 1 ´4 = 1
(d) = 4
l Bk 1
-
1 16 1 4
Topic 2 Advanced Concept (Quantum Mechanical 22 ¥ 2
Theory) Electronic Configuration and Note Lyman = L (n1 = 1), Balmer = B (n1 = 2)
Quantum Number Paschen = P (n1 = 3), Brackett = Bk (n1 = 4 )
1. The energy of 2s-orbital is lowest in K(potassium). An orbital Pfund = Pf (n1 = 5)
gets larger as the principal quantum number n increases.
Correspondingly, the energy of the electron in such an orbital
4. The graphs between | y |2 and r are radial density plots having
becomes less negative, meaning that the electron is less strongly (n - l - 1) number of radial nodes. For 1s, 2s, 3s and 2 p-orbitals
bound and has less energy. The graph of principal quantum these are respectively.
number with atomic number is
Principal quantum

2 |Y|2 For 2s-orbital


number of radial node = 2–0–1=1
number

r
1 2s

1 25 50 75 100
Atomic number |Y|2 For 3s-orbital
number of radial node = 3–0–1=2
2. The electrons are more likely to be found in the region a and c. At
b, wave function becomes zero and is called radial nodal surface
or simply node. r

a
Y (x)
b +x ®
¬ –x |Y|2 For 2p-orbital
number of radial node = 2–1–1=0

c
r
The graph between wavefunction (y ) and distance (r) from the
nucleus helps in determining the shape of orbital. Thus, the given graph between | y |2 and r represents 2s-orbital.
Atomic Structure 33

5. For any given series of spectral lines of atomic hydrogen. Thus, Eq. (i) becomes
Let Dn = nmax - nmin be the difference in maximum and n2
2pa0 = nl ...(ii)
minimum frequencies in cm-1. Z
For Lyman series, n2
\ 2pa0 = n (1.5 pa0) [Given, l = 1 .5 pa0]
Dn = nmax - nmin Z
General formula: n 15 . pa0 1.5
= = = 0.75
é 1 1ù Z 2pa0 2
n = 109677 ê 2 - 2 ú
êë ni nf úû 8. de-Broglie wavelength (l) for electron is given by
For Lyman n1 = 1, n2 = 2, 3, K l=
h
...(i)
æ1 1 ö æ1 ö 2 m K.E
nmax = 109,677ç - ÷ = 109,677 ç - 0÷
è1 ¥ ø è1 ø Also, according to photoelectric effect
= 109,677 KE = hn - hn 0
æ1 1 ö On substituting the value of KE in Eq (i), we get
nmin = 109,677ç - 2 ÷ h
è 1 (2) ø l=
2m ´ (hn - hn 0 )
DnLyman = nmax - nmin
é 109,677 ´ 3 ù 109,677 1
= 109,677 - = \ lµ
êë 4 úû 4 (n - n 0 )1/ 2
For Balmer series, nh
9. (I) Angular momentum, mvr =
æ 1 1ö 109677 2p
nmax = 109,677ç 2 - ÷ Þ
è (2) ¥ ø 4 Þ mvr µ n
µ distance from the nucleus
æ 1 1 ö 109677 ´ 5
nmin = 109,677ç 2 - 2 ÷ Þ (II) This statement is incorrect as size of an orbit
è (2) (3) ø 36
µ Azimuthal quantum number (l )
Dn = nmax - nmin (Q n = constant)
109,677 é 109,677 ù æ 1ö
DnBalmer = - ´ 5 = 109,677 ç ÷ (III) This statement is incorrect as at ground state,
4 êë 36 úû è 9ø n = 1, l = 0
D nLyman 109,677 / 4 Þ Orbital angular momentum (wave mechanics)
=
D nBalmer 109,677 / 9 h
= l (l + 1) =0 [Q l = 0 ]
D nLyman 9 2p
= (IV) The given plot is
D nBalmer 4
D nLyman
\The ratio of is 9 : 4. l=0 (n=1)
D nBalmer l=1 (n=2)

6. Smaller the value of (n + l ), smaller the energy. If two or more l=2 (n=3)
sub-orbits have same values of (n + l ), sub-orbits with lower y
values of n has lower energy. The (n + l ) values of the given l=3 (n=4)
options are as follows :
I. n = 4, l = 2 ; n + l = 6
II. n = 3, l = 2; n + l = 5
r
III. n = 4 , l = 1, n + l = 5
IV. n = 3, l = 1, n + l = 4 1 é æ 1 1ö ù
10. DE = hc ´ = hc ´ ê RH çç 2 - 2 ÷÷ ´ Z 2 ú
Among II and III, n = 3 has lower value of energy. Thus, the l ë è n1 n2 ø û
correct order of their increasing energies will be
1 1 hc
IV < II < III < I Þ 2- 2= [for H, atom Z = 1]
n1 n2 RH ´ l ´ Z 2 ´ hc
Circumference 2pr
7. Number of waves = Þ n= 1 1 1
Wavelength l = = ´
RH ´ l (1 ´ 10 7 m-1 ) (900 ´ 10-9 m)
\ 2pr = nl ...(i)
1 1 1
Also, we know that radius (r) of an atom is given by Þ 2
- 2=
n1 n2 9
a n2
r= 0 1 1 1 1 é\ n1 = 3,ù
Z So, in option (b) - = -0=
32 ¥ 2 9 9 ëê n2 = ¥ ûú
34 Atomic Structure

11. According to Rydberg’s formula, 15. Given, atomic number of Rb, Z = 37


é 1 1ù Thus, its electronic configuration is [ Kr ]5s1. Since, the last
wave number (n) = RH Z 2 ê 2 - 2 ú
êë ni nf úû electron or valence electron enter in 5s subshell.
Given, ni = n, nf = 8 [Q it is the case of emission] So, the quantum numbers are n = 5, l = 0, (for s-orbital) m = 0
(Q m = + l to -l), s = + 1 / 2 or - 1 / 2.
é 1 1ù
n = RH ´ (1)2 2 - 2 é Z2 ù
êë n 8 úû 16. Given, in the question E = - 2.178 ´ 10-18 J ê 2 ú
ën û
é 1 1 ù RH RH For hydrogen Z = 1,
n = RH - = -
êë n2 64 úû n2 64 é 1ù
So, E1 = - 2.178 ´ 10-18 J 2
On comparing with equation of straight line, y = mx + c, we get êë 1 úû
-RH é 1ù
Slope = RH , intercept = . E2 = - 2.178 ´ 10-18 J 2
64 êë 2 úû
1 Now, E1 - E2
Thus, plot of wave number (n) against 2 will be linear with
æ1 1 ö hc
n i.e. DE = 2.178 ´ 10-18 ç 2 - 2 ÷ =
slope (+ RH ). è1 2 ø l
12. Bohr radius (rn ) = Î0 n2h2 æ1 1 ö 6.62 ´ 10-34 ´ 3.0 ´ 108
2.178 ´ 10-18 ç 2 - 2 ÷ =
n2h2 è1 2 ø l
rn = -7
4p 2me2kZ \ l » 1.21 ´ 10 m
1 17. According to Bohr’s model,
k=
4 p Î0 nh n2h2
mvr = Þ (mv )2 =
2 2
n h Î0 a 2p 4 p 2r2
\ rn = = n2 0
2
pme Z Z 1 n2h2
Þ KE = mv 2 = 2 2 …(i)
where, m = mass of electron 2 8p r m
e = charge of electron Also, Bohr’s radius for H-atom is, r = n2a0
h = Planck’s constant Substituting ‘r’ in Eq. (i) gives
k = Coulomb constant h2 h2
KE = when n = 2 , KE =
n2 ´ 0.53 8p 2n2a02m 32p 2 a02 m
rn = Å
Z 18. The number of radial nodes is given by expression (n - l - 1).
Radius of nth Bohr orbit for H-atom For 3s, number of nodes = 3 - 0 - 1 = 2
= 0.53 n Å2
[Z = 1 for H-atom] For 2p, number of nodes = 2 - 1 -1 = 0
a0n2
\Radius of 2 nd
Bohr orbit for H-atom 19. Expression for Bohr’s orbit is, rn = = a0
Z
= 0.53 ´ (2)2 = 212
. Å when n = 2, Z = 4.
13. This graph shows the probability of finding the electron within 20. 1s7 violate Pauli exclusion principle, according to which an
shell at various distances from the nucleus (radial probability). orbital cannot have more than two electrons.
The curve shows the maximum, which means that the radial 1 1
probability is greatest for a given distance from the nucleus.
21. + and - just represents two quantum mechanical spin states
2 2
This distance is equal to Bohr’s radius = a0 which have no classical analogue.
22. Using the de-Broglie’s relationship :
P h 6.625 ´ 10-34
1s
l= = = 2.3 ´ 10-30 m
mv 0.2 ´ 5
r 60 ´ 60

(a) It is for 2s-orbital. 23. Nodal plane is an imaginary plane on which probability of
(b) It is radial wave function for 1s. finding an electron is minimum. Every p-orbital has one nodal
plane :
(c) Correct
(d) Probability cannot be zero at a certain distance from nucleus.
13.6 px
14. \ En = - eV where, n = 1, 2, 3 ...
n2
-13.6 YZ-plane, a nodal plane
In excited states, E2 = = -3.4 eV
4
Atomic Structure 35

24. 1s2 2 s2 2 p6 3s2 3 p6 3d 5 4 s1 is ground state electronic configuration 36. Option (b) is wrong representation according to aufbau
of Cr. principle. A high energy atomic orbital (2p) cannot be filled
unless the low energy orbital (2s) is completely occupied.
25. (i) n = 4 , l = 1 Þ 4 p-orbital
(ii) n = 4 , l = 0 Þ 4s-orbital æ 1 1 ö hc
37. Transition energy (DE ) = kZ 2 çç - 2 ÷÷ =
è n1 n2 ø l
2
(iii) n = 3, l = 2 Þ 3d-orbital
(iv) n = 3, l = 1 Þ 3d-orbital 1
i.e. DE µ
According to Aufbau principle, energies of above mentioned l
orbitals are in the order of hc
(iv) 3p < (ii) 4s < (iii) 3d < (i) 4 p 38. E=
l
26. The energy of an electron in a Bohr atom is expressed as E l
Þ 1 = 2 =2
kZ 2 where, k = Constant, E2 l 1
En = - Z = Atomic number,
n2 39.
n = Orbit number n l m s
= - 13.6 eV for H (n = 1) 1
3 2 -3
- 13.6 2
when n = 2 , E2 = eV = - 3.40 eV This is the wrong set of quantum number because | m | cannot be
22
greater than l.
(n can have only integral value 1, 2, 3,…… ¥)
40. The wavelength order is
h
27. The orbital angular momentum (L) = l (l + 1) X-ray < ultraviolet < infrared < radio wave
2p
h 41. When electron jumps to lower orbit photons are emitted while
= 6 (l = 2 for d - orbital ) photons are absorbed when electron jumps to higher orbit.
2p
1s-orbital is the lower most, electron in this orbital can absorb
28. Bohr first made use of quantum theory to explain the structure of photons but cannot emit.
atom and proposed that energy of electron in an atom is
quantised. 42. The valence shell configuration of Rubidium (Rb) is
1 1
[ Kr ] 5s1 n = 5, l = 0, m = 0, s = + or -
29. Mg2+ = 1s2 2s2 2 p6 no unpaired electron 2 2
Ti 3+ = 1s2 2 s2 2 p6 3s2 3 p6 3d 1 one unpaired electron 43. The principal quantum number ‘n’ represents orbit number
3+ 2 2 6 2 6 2 hence, determine the size of orbitals.
V = 1s 2 s 2 p 3s 3 p 3d two unpaired electrons
44. According to Pauli exclusion principle, an atomic orbital can
Fe2+ = 1s2 2 s2 2 p6 3s2 3 p6 3d6 four unpaired electrons
accommodate at the most, two electrons, with opposite spins.
30. Expression for orbital angular momentum (L) is 45. Both (a) and (d) are correct. The three electrons in the
h 2p-orbitals must have same spin, no matter up spin or down spin.
L = l (l + 1) = 0 for 2s-electrons
2p 46. (a) Cr = [Ar] 3d 5 4 s1 , an exception to aufbau principle.
Q For s-orbital, l = 0. (b) For a given value of l, m can have any value from
31. Diffraction is property of wave, E = mc2 determine energy of (-l to + l), so can have negative value.
particle and E = hn determine energy of photon. Interference (c) Ag is in copper group with d 10s1 configuration,
phenomena is exhibited by both matter and waves. i.e. 46 electrons are spin paired.
32. X-rays is electrically neutral, not deflected in electric or 47. Isotones have same number of neutrons.
magnetic fields. 76 77
and 34Se78 have same number (44) of neutrons,
32 Ge , 33As
33. Cl (17) = 1s2 2 s2 2 p6 3s2 3 p5 hence they are isotones.
48. Assertion is correct Be(1s2 , 2s2 ) has stable electronic
configuration, removing an electron require more energy than
The last, unpaired electron has, n = 3, l = 1( p) and m can have the same for B(2 p1 ). Reason is incorrect
any of the three value (- 1, 0, + 1). (Aufbau principle).
34. Cr (24) = 1s2 2 s2 2 p6 3s2 3 p6 3d 5 4 s1 49. S 1 is spherically symmetrical state, i.e. it correspond to a
1442 443
Ar s-orbital. Also, it has one radial node.
The above configuration is exception to Aufbau’s principle. Number of radial nodes = n - l - l
35. Fluorine, a halogen, is the most electronegative atom, has the Þ n - 0 -1=1
electronic configuration 2s2 2 p5 (valence shell). Þ n = 2 i.e. S 1 = 2s-orbital.
36 Atomic Structure

50. Ground state energy of electron in H-atom (EH ) 67. In an one electron (hydrogenic) system, all orbitals of a shell
2 remains degenerate, hence in second excited state, the
kZ
EH = = k (Z = 1, n = 1) degeneracy of H-atom is nine
n2 3s 3p 3d
For S 1 state of Li 2+ , H(1s1) Second excited
Ground state state of H-atom
k (3)2 9 All are degenerate
E= = k = 2.25 k
22 4 degeneracy = 9

51. In S 2 state, E (Li 2+ ) = K (given) In case of many electrons system, different orbitals of a shell are
non-degenerate. Hence,
qk
K =
n2 1s 1s 2s 2p 1s 2s 2p
Þ n=3

H
Second excited
Since, S 2 has one radial node. Ground state First excited state
state only three p-orbitals
3 - l -1 = 1 (2px, 2py, 2pz) are
l =1 degenerate.

52. In the wave function (y ) expression for 1s-orbital of He+ , there 68. PLAN This problem is based on concept of quantum number. Follow
should be no angular part. Hence (iii) can’t be true for y 1s of the following steps to solve this problem.
He+ . Write all possible orbitals having combination of same principal,
azimuthal, magnetic and spin quantum number.
53. Correct : 2s orbital has one radial node. Then count the all possible electrons having given set of
No of radial node = n - l - 1 = 2 - 0 - 1 = 1 quantum numbers.
Also, when radial part of wave function (y ) is plotted against For n = 4, the total number of possible orbitals are
‘‘r’’, wave function changes its sign at node. 4s 4p 4d 4f
54. i is the correct expression of wave function for 1s-orbital of 0 –1 0 +1 –2 –1 0 +1 +2 –3 –2 –1 0 +1 +2 +3
hydrogenic system.
According to question | m l | = 1 , i.e. there are two possible
55. A. Orbital angular momentum values of m l , i.e. +1 and –1 and one orbital can contain
h 1
(L) = l (l + 1) maximum two electrons one having s = + and other having
2p 2
i.e. L depends on azimuthal quantum number only. s = - 1/ 2 .
B. To describe a one electron wave function, three quantum So, total number of orbitals having {| m l | = 1} = 6
numbers n, l and m are needed. Further to abide by Pauli Total number of electrons having
exclusion principle, spin quantum number(s) is also needed. 1
{| m l | = 1 and ms = - } = 6
C. For shape, size and orientation, only n, l and m are needed. 2
D. Probability density (y 2 ) can be determined if n, l and m are 69. PLAN KE = 1 mv 2 = 3 RT
known. 2 2
\ m2v 2 = 2mKE \ mv = 2mKE
56. Cr = [Ar] 3d 5 4 s1
h h h
l (wavelength) = = µ
57. 1 : 16 mv 2mKE 2m(T )
58. Heisenberg proposed uncertainty principle and de-Broglie where, T = Temperature in Kelvin
proposed wave nature of electron. h
l(He at -73° C = 200 K) =
59. orbital 2 ´ 4 ´ 200
60. 2 px ,2 py and 2pz have different orientation in space. l (Ne at 727°C = 1000 K) =
h
61. Two electrons in same orbital must have opposite spin. 2 ´ 20 ´ 1000

62. Very large mass of alpha particles than beta particles is l (He) 2 ´ 20 ´ 1000
\ =M = =5
responsible for less deflection in former case. l (Ne) 2 ´ 4 ´ 200
63. 3dx2 - y2 orbital lies in XY-plane. Thus, M =5
64. Aufbau principle. 70. Energy of photon
65. This is the wavelength of infrared radiation. hc hc 6.625 ´ 10–34 ´ 3 ´ 108
= J= eV = = 4 . 14 eV
66. Cr = 3d 5 4 s1. l el 300 ´ 10–9 ´ 1.602 ´ 10–19
Atomic Structure 37

For photoelectric effect to occur, energy of incident photons Number of H-atoms produced after dissociation
must be greater than work function of metal. Hence, only Li, Na, = 2 ´ 0.0409 ´ 6.023 ´ 1023 = 4.93 ´ 1022
K and Mg have work functions less than 4.14 V.
æ 1ö
71. When n = 3, l = 0, 1, 2 i.e. there are 3s, 3p and 3d-orbitals. If all Transition energy/atom = 2.18 ´ 10-18 ç1 - ÷ J
these orbitals are completely occupied as è 4ø
3
= ´ 2.18 ´ 10-18 J
4
1 Þ
Total 18 electrons, 9 electrons with s = + and 9 with Total transition energy
2 3
1 = ´ 2.18 ´ 10-18 ´ 4.93 ´ 1022 J
s=– . 4
2
= 80.60 ´ 103 J = 80.60 kJ
Alternatively In any nth orbit, there can be a maximum of 2n2
electrons. Hence, when n = 3, number of maximum electrons = Therefore, total energy required
1 = dissociation energy + transition energy
18. Out of these 18 electrons, 9 can have spin –
2
1 = (17.84 + 80.60) kJ = 98.44 kJ
and remaining nine with spin + .
2
nh 76. If accelerated by potential difference of V volt, then
72. (a) mvr = 1
2p mv 2 = eV
-34 2
nh 6.625 ´ 10
Þ v= = p2
2pmr 2 ´ 3.14 ´ 9.1 ´ 10-31 ´ 0.529 ´ 10-10 Þ = eV , here p = momentum (mv )
2m
= 2.18 ´ 106 ms - 1
h h
Using de-Broglie equation, l = =
h 6.625 ´ 10-34 p 2meV
(b) l = = = 0.33 ´ 10-9 m
mv 9.1 ´ 10-31 ´ 2.18 ´ 106
6.625 ´ 10-34
Þ 1.54 ´ 10-10 =
(c) Orbital angular momentum (2 ´ 9.1 ´ 10-31 ´ 1.6 ´ 10-19 V )1/ 2
h æ hö
(L) = l (l + 1) = 2ç ÷ Solving for V gives : V = 63.56 V.
2p è 2p ø
77. The work done in the given neutralisation process is
[Q For p-orbital, l = 1 ]
¥ e2
73. (a) At radial node, y 2 must vanishes, i.e. W =-ò F dr and F =
a0 4pe0r2
2 2 r0
é 1 ù æ r0 ö - a 0 e2
¥ 2
y 22s = 0 = ê ú çç 2 - a ÷÷ e é 1ù e
Þ W = =- = Total energy (E )
ë 4 2p û è 0ø 4 pe0 êë r úû
a0 4 pe0r
r0
Þ 2- = 0 Þ r0 = 2a0 Now, if ‘V’ is magnitude of potential energy, then according to
a0 given information, kinetic energy (Ek ) is V / 2. Therefore,
h 6.625 ´ 10-34 E = -V +
V
(b) l= = = 6.625 ´ 10-35 m (PE is always negative)
mv 100 ´ 10-3 ´ 100 2
V
= 6.625 ´ 10-25 Å (negligibly small) =-
2
74. The general Rydberg’s equation is - e2
Þ V = - 2E =
1 æ 1 1ö 2pe0r
n= = R (Z )2 ç 2 - 2÷
l çn ÷
è 1 n2ø
78. The Rydberg’s equation for H-atom is
1
Þ µ Z2 1 æ 1 1ö
l = n (wave number) = RH çç 2 - 2 ÷÷
l è 1
n n2ø
l (He+ ) Z (H)2 1
Þ = =
l (H) Z (He+ )2 4 For Balmer series, n1 = 2 and n2 = 3, 4 , 5, ..., ¥
l (H) 91.2 For shortest l, n2 has to be maximum, i.e. infinity. Then
Þ l (He+ ) = = nm = 22.8 nm
4 4 æ1 1ö R 1.09 ´ 107
n = RH ç - ÷ = H = = 2.725 ´ 106 m -1
pV 1 ´1 è 4 ¥ ø 4 4
75. Moles of H2 = = = 0.0409
RT 0.082 ´ 298
Þ Bond energy = 0.0409 ´ 436 = 17.84 kJ
38 Atomic Structure

79. After breaking of the bond of I2 molecule, the remaining energy For longest wavelength transition from 3rd orbit, electron must
would be distributed uniformly to iodine atoms as their kinetic jump to 4th orbit and the transition energy can be determined as
energy, i.e. æ1 1 ö
DE = + 4 ´ 21.7 ´ 10-19 ç - ÷ J = 4.22 ´ 10-19 J
E (energy of photon) = Bond energy + 2 ´ kinetic energy è 9 16 ø
6.625 ´ 10-34 ´ 3 ´ 108 240 ´ 103 Also, Q DE =
hc
Þ -10
= + 2 ´ Ek l
4500 ´ 10 6.023 ´ 1023
hc 6.625 ´ 10-34 ´ 3 ´ 108
Þ Ek = 2.16 ´ 1020 J/atom \ l= = m
DE 4.22 ´ 10-19
80. The Bohr de-Broglie relationship is
= 471 ´ 10-9 m = 471 nm
2pr = nl = circumference of Bohr’s orbit.
i.e. number of complete waves formed in one complete 84. Ten, the given value of n and l correspond to 3d-orbital which
revolution of electron in any Bohr orbit is equal to orbit number, has five fold degeneracy level.
hence three. 85. The 2nd configuration is against Hund’s rule of maximum
81. The expression for transition wavelength is given by Rydberg’s multiplicity which states that the singly occupied degenerate
equation : atomic orbitals must have electrons of like spins.
1 æ 1 1ö 86. The required transition is n1 = 2 to n2 = ¥ and corresponding
= RH Z 2 çç 2 - 2 ÷÷
l è n1 n2 ø transition energy is
Equating the transition wavelengths of H-atom and He ion, + æ 1 1ö
DE = 21.7 ´ 10-12 çç 2 - 2 ÷÷ erg
æ 1 1ö æ4 4ö è 1
n n2ø
RH çç 2 - 2 ÷÷ = RH ç 2 - 2 ÷ 21.7
è 2 4 ø = ´ 10-12 erg = 5.425 ´ 10-12 erg
è 1
n n2ø 4
The longest wavelength that can cause above transition can be
Equating termwise on left to right of the above equation gives
determined as :
n1 = 1 and n2 = 2
hc 6.625 ´ 10-34 ´ 3 ´ 108
82. For H-atom, the energy of a stationary orbit is determined as l= =
DE 5.425 ´ 10-12 ´ 10-7
k
En = - where, k = constant (2.18 ´ 10-18 J)
n2 = 3.66 ´ 10-7 m = 3.66 ´ 10-5 cm
æ 1ö 3
Þ DE (n = 2 to n = 1) = k ç1 - ÷ = k 87. Ionisation potential of H-like species
è 4 ø 4 = E1 = 2.17 ´ 10-11 erg
-18
= 1.635 ´ 10 J
æ 1ö
For a H-like species, energy of stationary orbit is determined as Þ DE = 2.17 ´ 10-11 ç1 - 2 ÷ ´ 10-7 J
kZ 2 è 2 ø
En = - 2
hc
n = 1.6275 ´ 10-18 J Þ l =
where, Z = atomic number DE
æ 1 1ö 6.625 ´ 10-34 ´ 3 ´ 108
Þ DE = kZ 2 çç 2 - 2 ÷÷ = m
è n1 n2 ø 1.6275 ´ 10-18
1 DE k 2 æ 1 1 ö 3 = 122 ´ 10-9 m = 1220 Å
Þ = = Z ç - ÷ = RH Z 2 ´
l hc hc è1 4 ø 4
88. Transition energy = [ - 2.41 - (- 5.42)] ´ 10-12 erg
4 4
Þ Z2 = = = 4.05
3RH l 3 ´ 1.097 ´ 107 ´ 3 ´ 10-8 = 3.01 ´ 10-12 erg

Þ Z = 2 (He+ ) = 3.01 ´ 10-19 J [Q 1 erg = 10-7 J ]


hc
83. For H-like species, the energy of stationary orbit is expressed as Also, DE =
2
E ( X ) = Z ´ E (H ) l

Þ For He+ (Z = 2) 6.625 ´ 10-34 ´ 3 ´ 108


Þ l= m
3.01 ´ 10-19
4 ´ 21.7 ´ 10-19
E=- J
n2 = 660 ´ 10-9 m = 660 nm

Download Chapter Test


http://tinyurl.com/y49o45j8 or
3
Periodic Classification and
Periodic Properties

w
Flo
Topic 1 History and Periodic Classification

ree
Objective Question I (Only one correct option) (c) the first ionisation energies of elements along a period do not

F
vary in a regular manner with increase in atomic number
1. The IUPAC symbol for the element with atomic number 119
(d) for transition elements the d-subshells are filled with electrons
would be (2019 Main, 8 April II)
monotonically with increase in atomic number

or
(a) unh
ur
(b) uue
(c) uun (d) une

f
Objective Question II
2. The element with Z = 120 (not yet discovered) will be an/a (One or more than one correct option)
ks
(2019 Main, 12 Jan I)
Yo
(a) transition metal (b) inner-transition metal 4. The statements that is/are true for the long form of the
oo
(c) alkaline earth metal (d) alkali metal periodic table is/are (1988, 1M)
(a) it reflects the sequence of filling the electrons in the order of
3. The statement that is not correct for the periodic
eB

sub-energy level s, p, d and f


classification of elements, is (1992, 1M)
(b) it helps to predict the stable valency states of the elements
(a) the properties of elements are the periodic functions of their
(c) it reflects tends in physical and chemical properties of the
atomic numbers
r

elements
(b) non-metallic elements are lesser in number than metallic
ou
ad

(d) it helps to predict the relative ionicity of the bond between any
elements
two elements
Y
nd

Topic 2 Periodic Properties


Re
Fi

Objective Questions I (Only one correct option) 4. The correct order of the atomic radii of C, Cs, Al and S is
(2019 Main, 11 Jan I)
1. The group number, number of valence electrons and valency
(a) C < S < Al < Cs (b) C < S < Cs < Al
of an element with atomic number 15, respectively, are
(2019 Main, 12 April I) (c) S < C < Cs < Al (d) S < C < Al < Cs
(a) 16, 5 and 2 (b) 15, 5 and 3 5. In general, the properties that decrease and increase down a
(c) 16, 6 and 3 (d) 15, 6 and 2 group in the periodic table, respectively are
(2019 Main, 9 Jan I)
2. The element having greatest difference between its first and
(a) electronegativity and atomic radius
second ionisation energy, is (2019 Main, 9 April I)
(b) electronegativity and electron gain enthalpy
(a) Ca (b) Sc
(c) electron gain enthalpy and electronegativity
(c) Ba (d) K
(d) atomic radius and electronegativity
3. The correct option with respect to the Pauling
electronegativity values of the elements is 6. The ionic radii (in Å) of N3- ,O2- and F- respectively are
(2019 Main, 11 Jan II) (2015 Main)

(a) P > S (b) Si < Al (a) 1.36, 1.40 and 1.71 (b) 1.36, 1.71 and 1.40
(c) Te > Se (d) Ga < Ge (c) 1.71, 1.40 and 1.36 (d) 1.71, 1.36 and 1.40
40 Periodic Classification and Periodic Properties

7. Which one of the following alkaline earth metal sulphates 19. The first ionisation potential of Na, Mg, Al and Si are in the
has its hydration enthalpy greater than its lattice enthalpy? order (1988, 1M)
(a) CaSO4 (b) BeSO4 (2015 Main) (a) Na < Mg >Al < Si (b) Na > Mg > Al >Si
(c) BaSO4 (d) SrSO4 (c) Na < Mg <Al >Si (d) Na > Mg > Al <Si
8. Which among the following is the most reactive? 20. The electronegativity of the following elements increases in
(a) Cl 2 (b) Br2 (2015 Main) the order (1987, 1M)
(c) I2 (d) ICl (a) C, N, Si, P (b) N, Si, C, P
9. Which one has the highest boiling point? (c) Si, P, C, N (d) P, Si, N, C
(a) He (b) Ne (c) Kr (d) Xe 21. Atomic radii of fluorine and neon in Angstrom units are
10. The first ionisation potential of Na is 5.1 eV. The value of respectively given by (1987, 1M)
electron gain enthalpy of Na + will be (2013 Main) (a) 0.72, 1.60 (b) 1.60, 1.60

w
(a) - 2.55 eV (b) - 5.1 eV (c) 0.72, 0.72 (d) None of these
(c) - 10.2 eV (d) + 2.55 eV 22. The first ionisation potential in electron volts of nitrogen and
11. Which of the following represents the correct order of oxygen atoms are respectively given by (1987, 1M)

Flo
increasing first ionisation enthalpy for Ca, Ba, S, Se and Ar? (a) 14.6, 13.6 (b) 13.6, 14.6
(2013 Main) (c) 13.6, 13.6 (d) 14.6, 14.6
(a) Ca < S < Ba < Se < Ar (b) S < Se < Ca < Ba < Ar 23. The hydration energy of Mg 2+ is larger than that of

ree
(c) Ba < Ca < Se < S < Ar (d) Ca < Ba < S < Se < Ar (1984, 1M)
12. Identify the least stable ion amongst the following. (a) Al 3+ (b) Na + (c) Be2+ (d) Mg3+

F
(a) Li + (b) Be- (2002, 3M) 24. The element with the highest first ionisation potential is
(c) B- (d) C - (1982, 1M)
(a) boron (b) carbon

or
ur
13. The set representing the correct order of first ionisation
(c) nitrogen (d) oxygen
potential is (2001, 1M)

f
25. The correct order of second ionisation potential of carbon,
(a) K > Na > Li (b) Be > Mg > Ca nitrogen, oxygen and fluorine is (1981, 1M)
ks
(c) B > C > N (d) Ge > Si > C (a) C > N > O > F (b) O > N > F > C
Yo
(c) O > F > N > C (d) F > O > N > C
oo
14. The correct order of radii is (2000, 1M)
(a) N < Be < B (b) F - < O2- < N3- Objective Questions II
eB

(c) Na < Li < K (d) Fe3+ < Fe 2+ < Fe4+ (One or more than one correct option)
15. The incorrect statement among the following. (1997(C), 1M) 26. The option(s) with only amphoteric oxides is(are)(2017 Adv.)
r

(a) The first ionisation potential of Al is less than the first (a) NO, B2O3 , PbO, SnO2 (b) Cr2O3 , CrO, SnO, PbO
ou
ad

ionisation potential of Mg (c) Cr2O3 , BeO, SnO, SnO2 (d) ZnO, Al 2O3 , PbO, PbO2
(b) The second ionisation potential of Mg is greater than the
27. Ionic radii of (1999, 3M)
Y

second ionisation potential of Na


(a) Ti 4+ < Mn 7 + (b) 35
Cl - < 37
Cl -
(c) The first ionisation potential of Na is less than the first
ionisation potential of Mg (c) K + > Cl - (d) P 3+ > P 5+
nd
Re

(d) The third ionisation potential of Mg is greater than third


28. The first ionisation potential of nitrogen and oxygen atoms
ionisation potential of Na
are related as follows.
Fi

(1989, 1M)
16. Which of the following has the maximum number of (a) The ionisation potential of oxygen is less than the ionisation
unpaired electrons ? (1996, 1M) potential of nitrogen
(a) Mg2+ (b) Ti 3+ (b) The ionisation potential of nitrogen is greater than the
(c) V 3+ (d) Fe2+ ionisation potential of oxygen
(c) The two ionisation potential values are comparable
17. Amongst the following elements (whose electronic (d) The difference between the two ionisation potential is too large
configurations are given below), the one having the highest
ionisation energy is (1990, 1M) 29. Sodium sulphate is soluble in water whereas barium sulphate
2
(a) [Ne] 3s 3 p 1 2
(b) [Ne] 3s 3 p 3 is sparingly soluble because (1989, 1M)
2 2 10 2 3
(a) the hydration energy of sodium sulphate is more than its lattice
(c) [Ne] 3s 3 p (d) [Ar] 3d 4s 4 p energy
18. Which one of the following is the smallest in size? (b) the lattice energy of barium sulphate is more than its hydration
(1989, 1M) energy
(a) N3- (b) O2- (c) the lattice energy has no role to play in solubility
(c) F - (d) Na + (d) the hydration energy of sodium sulphate is less than its lattice
energy
Periodic Classification and Periodic Properties 41

Assertion and Reason 37. On Mulliken scale, the average of ionisation potential and
electron affinity is known as ................ (1985, 1M)
Read the following questions and answer as per the
direction given below : 38. The energy released when an electron is added to a neutral
(a) Statement I is true; Statement II is true; Statement II is gaseous atom is called …… . (1982, 1M)
the correct explanation of Statement I.
(b) Statement I is true; Statement II is true; Statement II is True/False
not the correct explanation of Statement I. 39. The basic nature of the hydroxides of group 13 (III B)
(c) Statement I is true; Statement II is false. decreases progressively down the group. (1993, 1M)
(d) Statement I is false; Statement II is true.
40. The decreasing order of electron affinity of F, Cl, Br is
30. Statement I Nitrogen and oxygen are the main F > Cl > Br. (1993, 1M)
components in the atmosphere but these do not react to

w
41. In group IA of alkali metals, the ionisation potential decreases
form oxides of nitrogen.
down the group. Therefore, lithium is a poor reducing agent.
Statement II The reaction between nitrogen and oxygen
(1987, 1M)
requires high temperature. (2015 Main)

Flo
31. Statement I Pb 4+ compounds are stronger oxidising 42. The softness of group IA metals increases down the group with
4+ increasing atomic number. (1986, 1M)
agents than Sn compounds.

ree
Statement II The higher oxidation states for the group 14
elements are more stable for the heavier members of the
Subjective Questions
group due to ‘inert pair effect’. (2008, 3M) 43. Arrange the following ions in order of their increasing radii

F
32. Statement I Band gap in germanium is small. Li + , Mg 2+ , K + , Al 3+. (1997, 1M)
Statement II The energy spread of each germanium 44. Compare qualitatively the first and second ionisation

or
ur
atomic energy level is infinitesimally small. (2007, 3M) potentials of copper and zinc. Explain the observation.

f
33. Statement I The first ionisation energy of Be is greater (1996, 2M
than that of B. 45. Arrange the following as stated :
ks
Statement II 2p-orbital is lower in energy than 2s. “Increasing order of ionic size’’ N3– ,Na + , F- , O2 - , Mg 2+
Yo
(2000, (S), 1M)
oo
(1991, 1M)
34. Statement I F-atom has a less negative electron affinity 46. Explain the following :
eB

than Cl-atom.
Statement II Additional electrons are repelled more “The first ionisation energy of carbon atom is greater than that
effectively by 3 p-electrons in Cl-atom than by 2 p-electrons of boron atom whereas, the reverse is true for the second
in F-atom. ionisation energy.’’ (1989, 2M)
r

(1998, 2M)
ou

47. Arrange the following in the order of their increasing size:


ad

Fill in the Blanks Cl - , S2 - , Ca 2+ , Ar (1986, 1M)


Y

4+
35. Compounds that formally contain Pb are easily reduced 48. Arrange the following in order of their
2+
to Pb . The stability of the lower oxidation state is due to (i) decreasing ionic size Mg 2+ , O2 - , Na + ,F-
nd

…… . (1997, 1M)
Re

(ii) increasing first ionisation energy Mg, Al, Si, Na


36. Ca 2+ has a smaller ionic radius than K + because it has
(iii) increasing bond length F2 , N2 , Cl 2 , O2 (1985, 3M)
Fi

............ (1993, 1M

Answers
Topic 1 21. (a) 22. (a) 23. (b) 24. (c)
1. (b) 2. (c) 3. (d) 4. (b,c,d) 25. (c) 26. (a,b) 27. (d) 28. (a,b,c)
29. (a,b) 30. (a) 31. (c) 32. (c)
Topic 2
33. (c) 34. (c)
1. (b) 2. (d) 3. (d) 4. (a)
35. (inert pair effect)
5. (a) 6. (c) 7. (b) 8. (d)
36. (higher effective nuclear charge)
9. (d) 10. (b) 11. (c) 12. (b)
37. (electronegativity) 38. (electron affinity)
13. (b) 14. (b) 15. (b) 16. (d)
17. (b) 18. (d) 19. (a) 20. (c) 39. F 40. F 41. F 42. T
Hints & Solutions
Topic 1 History and Periodic Classification First ionisation enthalpy (I.E.) of K is lowest among the given
1. Atomic number (119) = 1 1 9 options. Here, the energy required to remove an electron from 4 s1
un un en is least as only one electron is present in the outermost shell. I.E.
So, symbol of the element = uue (I) is comparatively high for Mg and Sr and two electrons
Name of the element = ununennium (fully-filled) are placed in s-orbital. Second ionisation enthalpy of
K is highest among the given options.
It is expected to be s-block element an alkali metal and the first
element in eighth period. It is the lightest element that has not Now, removal of an electron occur from p6 (fully-filled). So, high
yet been synthesised. energy is required to remove the electron. From the above
discussion, it can be concluded that (I.E 2 - I.E 1 ) value is
2. The element with Z = 120 will be an alkaline earth metal.

w
maximum for K (potassium).
Recently, oganesson (Og) with atomic number 118 is named
by IUPAC is a noble gas and placed just two place before 120. 3. The electronegativity values of given elements on the Pauling scale
So, the general electronic configuration is represented as can be shown as follows:

Flo
[noble gas]ns2 and element with Z = 120 exist as an alkaline
Period No. Group 13 Group 14 Group 15 Group 16
earth metal.
3 Al (1.5) Si (1.8) P (2.1) S (2.5)

ree
3. (a) Correct statement According to Moseley’s law, the 4 Ga (1.6) Ge (1.8) Se (2.4)
properties of elements are the periodic function of their 5 Te (2.01)
atomic numbers.

F
(b) Correct statement The whole s-block, d-block, f -block On moving from left to right across a period, i.e. from Ga to Se,
and heavier p-block elements are metal. the effective nuclear charge increases and size decreases.
As a result, the value of electronegativity increases due to

or
(c) Correct statement Trend is not regular, Be has higher
ur
first ionisation energy than B, nitrogen has higher first increase in the attraction between the outer electrons and the
nucleus. Whereas on moving down the group, (i.e. from Se to Te),

f
ionisation energy than oxygen.
the atomic size increases.
(d) Inccorrect statement d-subshells are not filled
ks
monotonically, regularity break at chromium and copper. As a result, the force of attraction between the outer electron and
Yo
the nucleus decreases. Hence, the electronegativity decreases.
oo
4. (a) Incorrect Electrons are not filled in sub-energy levels
s, p, d and f in the same sequence.
4. Element Period Group No.
eB

(b) Correct Number of valence shell electrons usually No.


determine the stable valency state of an element. C 2nd 14
(c) Correct Physical and chemical properties of elements Al 3rd 13 Along the period atomic radius
are periodic function of atomic number which is the basis of S 3rd 16 decreases, so, radii : Al > S.
r

modern, long form of periodic table.


ou

Cs 6th 1
ad

(d) Correct Relative ionicity of the bond between any two With the addition of a new shell, period number as well as atomic
Y

elements is function of electronegativity difference of the radius increases. It is because of the successive addition of one
bonded atoms which in turn has periodic trend in long form extra shell of electrons. So, the order of the atomic radii of the
of periodic table. given elements will be: C < S < Al < Cs
nd
Re

Topic 2 Periodic Properties 5. The summary of variation of periodic properties is given in table
below:
Fi

1. The group number, number of valence electrons and valency of


an element with atomic number 15 are 15, 5 and 3 respectively. S.No. Periodic property Variation
Modern periodic table is based on the atomic number. Number Along a period Along a group
of valence electrons present in an atom decides the group
1. Atomic radius Decreases Increases
number. Electronic configuration of element having atomic
2. Electron gain Increases Decreases
number 15 = 1s2 2s2 2 p6 3s2 3 p3
enthalpy
Valence electrons
3. Electronegativity Increases Decreases
As five electrons are present in valence shell, its group number
is 15. Valency of element having atomic number 15 is +3 Thus, electronegativity decreases and atomic radius increases
(8 - 5 = 3). down a group in the periodic table.

2. The electronic configuration of given elements are as follows : 6. Number of electrons in N3- , = 7 + 3 = 10
K(19) = 1s2 2s2 2 p6 3s2 3 p6 4 s1 Number of electrons in O2- = 8 + 2 = 10
Mg(12) = 1s2 2s2 2 p6 3s2 Number of electrons in F- = 9 + 1 = 10
Sr(38) = 1s2 2s2 2 p6 3s2 3 p6 4 s2 3d 10 4 p6 5s2 Since, all the three species have each 10 electrons, hence they are
isoelectronic species.
Sc(21) = 1s2 2s2 2 p6 3s2 3 p6 4 s2 3d 1
Periodic Classification and Periodic Properties 43

It is considered that, in case of isoelectronic species as the 15. (a) Correct statement In a period, element of 2nd group has
negative charge increases, ionic radii increases and therefore the higher first ionisation potential than element of group 13.
value of ionic radii are (b) Incorrect statement Mg+ require less energy for further
N3- = 1.71 (highest among the three) ionisation than Na + because of noble gas configuration of Na + .
O2- = 1.40 F- = 136. (lowest among the three) (c) Correct statement Ionisation energy increases from left
Time Saving Technique There is no need to mug up the radius to right in a period.
values for different ions. This particular question can be solved
through following time saving. 16. Mg2+ = 1s2 2 s2 2 p6 = no unpaired electron
Trick The charges on the ions indicate the size as Ti 3+ = 1s2 2 s2 2 p6 3s2 3 p6 3d 1 = one unpaired electron
N3- > O2- > F- . Thus, you have to look for the option in which V3+ = 1s2 2 s2 2 p6 3s2 3 p6 3d 2 = two unpaired electrons
the above trend is followed. Option(c) is the only one in which
Fe2+ = 1s2 2 s2 2 p6 3s2 3 p6 3d6 = four unpaired electrons
this trend is followed. Hence, it is the correct answer.

w
7. As we move down the group, size of metal increases. Be has 17. [Ne] 3s2 3 p3 has highest ionisation energy, periodic trend.
lower size while SO2-
4 has bigger size, that’s why BeSO4 breaks 18. Among isoelectronic species, the relation in size is
easily and lattice energy becomes smaller but due to lower size cation < neutral < anion
+

Flo
of Be, water molecules are gathered around and hence hydration Hence, Na has smallest size.
energy increases. 19. Ionisation energy increases from left to right in a period.
On the other hand, rest of the metals, i.e Ca, Ba, Sr have bigger However, exception occur between group 2 and group 13

ree
size and that’s why lattice energy is greater than hydration energy. elements on account of stability of electronic configuration of
Time Saving Technique In the question of finding hydration valence shell.
energy only check the size of atom. Smaller sized atom has more IE

F
hydration energy. Thus, in this question Be is placed upper most Group 2 = > Group 13 =
in the group has lesser size and not comparable with the size of
ns2 ns2 np1

or
sulphates. Hence, BeSO4 is the right response.
ur Þ The desired order is Na < Mg > Al < Si
8. Cl 2, Br2 and I2 are homonuclear diatomic molecule in which

f
electronegativity of the combining atoms is same, so they are 20. Electronegativity increases from left to right in a period and
decreases from top to bottom in a group. Variation is more rapid
ks
more stable and less reactive, whereas, I and Cl have different
electronegativities and bond between them are polarised and in group than in a period, hence the desired order is
Yo
reactive. Therefore, interhalogen compounds are more reactive. Electronegativity : Si < P < C < N
oo
Time Saving Technique In this type of question of halogen, 21. Atomic radius of noble gases are greater than halogens of same
only go through the polarity of the molecules. As we know,
eB

period, hence (a) is the correct answer.


diatomic molecule does not have polarity but molecules with 22. First ionisation energy of oxygen is less than that of nitrogen on
dissimilar sizes have polarity resulting in more reactivity. the ground of stability of valence shell configuration, hence (a)
9. As we move down the group of noble gases, molecular mass is the correct answer.
r

increases by which dipole produced for a moment and hence 23. Hydration energy depends on charge of ion and ionic radius.
ou
ad

London forces increases from He to Xe. Higher the charge, greater the hydration energy. On the other
Therefore, more amount of energy is required to break these hand, smaller the size, greater the hydration energy. Charge is
Y

forces, thus boiling point also increases from He and Xe. considered first for comparison. Hence, Mg2+ has higher
10. Na ¾® Na + + e- First IE hydration energy than Na + .
nd

Na + + e- ¾® Na
Re

24. Nitrogen has highest ionisation potential due to exceptional


+ stability of its valence shell configuration mentioned in question 21.
Electron gain enthalpy of Na is reverse of (IE)
Fi

Because reaction is reverse so DH (eq) = - 5.1 eV 25. For second ionisation potential, electron will have to be
removed from valence shell of the following ions:
11. Ionisation energy increases along a period from left to right and
decreases down a group. The position of given elements in the C+ (5e-) = 1s2 2s2
periodic table is as 2p
N+ (6e-) = 1s2 2s2
Group No. 2 16 18
2p
Ca S Ar
Ba Se O+ (7e-) = 1s2 2s2
2p
Thus, the order of increasing DH IE1 is Ba < Ca < Se < S < Ar
F+ (8e-) = 1s2 2s2
12. Be- is the least stable ion, Be (1s2 2s2 ) has stable electronic 2p
configuration, addition of electron decreases stability. In general, ionisation energy increases from left to right in a
13. In a group, ionisation energy decreases down the group period. However, exception occur between adjacent atoms in a
Be > Mg > Ca period, greater amount energy is required for removal of
electron from completely half-filled or completely filled orbital
14. Among isoelectronic species, greater the negative charge, than the same for adjacent atom with either less than completely
greater the ionic size, hence F- < O2- < N3- . half-filled or less than completely filled orbital. Therefore,
44 Periodic Classification and Periodic Properties

ionisation potential of O+ is greater than that of F+ . Also ionisation F atom has slightly lower affinity for the electron than chlorine.
potential of N+ is greater than C+ but less than both O+ and F+ It is due to the reason that additional electrons are repelled more
(periodic trend). Hence, overall order is 2nd IP : O > F > N > C. effectively by 2p-electrons in F than by 3p-electrons in Cl-atom.
26. (c) is incorrect because NO is neutral oxide. 35. Inert pair effect-favours lower oxidation state.
(d) is incorrect because CrO is basic oxide. 36. Higher effective nuclear charge due to greater p/e ratio.
27. (a) Ti 4+ > Mn 7+ is the correct order of size due to lower positive IP + EA
37. Electronegativity = (Mulliken formula)
2
charge on Ti 4+ .
(b) 37 Cl - = 37 Cl - : Isotopes with same charge have same size 38. Electron affinity–definition.
because isotopes differ in compositions of nuclei which do 39. Basic nature of hydroxides increases down a group.
not affect the atomic/ionic radius. 40. Cl has maximum electron affinity, hence the correct order is
(c) K+ < Cl - is the correct order. Among isoelectronic species, Cl > F > Br

w
anion has greater size than cation. 41. Ionisation potential decreases down the group but this is not the
(d) P3+ > P5+ is the correct order. For the same elements, lower only criteria of reducing power.
the positive charge, larger the ions. 42. In a group, size increases from top to bottom.

Flo
28. (a) and (b) are infact the same statements and both are correct. N 43. Li + < Al 3+ < Mg2+ < K+ . Size decreases from left to right in a
has slightly greater ionisation energy than oxygen which is period and it increases from top to bottom in a group. Variation

ree
against periodic trend. This exception is due to completely is more pronounced in group than in period.
half-filled (2 p3 ) orbital in nitrogen that makes ionisation 44. Zn = 3d 10 4 s2 , Cu = 3d 10 4 s1

F
slightly difficult than oxygen. The first ionisation energy is greater for Zn but reverse is true for
(c) Also correct : Although N has greater first ionisation 2nd ionisation energy.
potential than oxygen, two values of ionisation potentials are

or
ur
comparable since they are adjacent in a period, i.e. electrons 45. Ionic size Mg2+ < Na + < F- < O2- < N3-
are removed from same orbit during ionisation.

f
Already explained in question 1 (i).
(d) Incorrect – opposite to (c). of the bonded atoms which in
ks
turn has periodic trend in long form of periodic table. 46. The first ionisation energy of carbon is greater than the same of
Yo
boron as predicted from periodic trend. However, for 2nd
29. (a) Correct For greater solubility, hydration energy must be
oo
greater than lattice energy. B+ = 1s2 ; more stable than C+ =1s2 2s2
(b) Correct Greater lattice energy discourage dissolution of a salt.
eB

2s2 2p1
(c) Incorrect When a salt dissolve, energy is required to break
the lattice, which comes from hydration process. ionisation trend is reversed due to stability of completely filled
(d) Incorrect Explained in (A). 2s-orbital of B+ :
r

47. Size Ca 2+ < Ar < Cl - < S2- . Explained in (i), question 6.


ou

30. Statement I and II are true and Statement II is the correct


ad

explanation of statement I.
48. (i) Mg2+ , O2- , Na + and F - are all isoelectronic, has
Y

31. Statement I is true. Stronger oxidising agent is one which itself


can easily be reduced. Pb4+ is unstable, due to inert pair effect, 10 electrons each. Among isoelectronic species, the order of
can easily be reduced to stable Pb2+ , hence a stronger oxidising size is cation < neutral < anion.
nd

Also, between cations, higher the charge, smaller the size


Re

agent than Sn 4+ .
and between anions, greater the negative charge, larger the
Statement II is false. Due to inert pair effect, the higher size. Therefore, the decreasing order of ionic radii :
Fi

oxidation states of group 14 elements becomes less stable for


O2- > F- > Na + > Mg2+
heavier member.
(ii) First ionisation energy increases from left to right in a period.
32. Both statements I and II are true and Statement II is the correct
explanation of statement I. However, exception occur between group 2 and 13 and group 15
and 16 where trend is reversed on the grounds of stability of
33. Statement I is true Be has higher first ionisation energy than B completely filled and completely half-filled orbitals. Therefore,
which is against periodic trend.
Ionisation energy (1st) : Na < Al < Mg < Si
Statement II is false 2s-orbital is lower in energy than 2p,
(iii) If the atoms are from same period, bond length is inversely
Aufbau’s principle.
proportional to bond order. In a group, bond length is related
34. Statement I is true; Statement II is false. directly to atomic radius. Therefore,
bond length N2 < O2 < F2 < Cl 2

Download Chapter Test


http://tinyurl.com/yxga4gah or
4
Chemical Bonding

w
Topic 1 Preliminary Concepts of Electrovalent and

Flo
Covalent Bonding

ree
Objective Questions I (Only one correct option) 7. The number and type of bonds between two carbon atoms in
1. The isoelectronic set of ions is (2019 Main, 10 April I) CaC2 are (1996, 1M)

F
- + +
(a) F , Li , Na and Mg 2+ (a) one sigma ( s ) and one pi ( p ) bonds
(b) one sigma ( s ) and two pi ( p ) bonds
(b) N3 - , Li + , Mg 2+ and O2 - (c) one sigma ( s ) and one half pi ( p ) bonds

or
(c) Li + , Na + , O2 - and F-
ur (d) one sigma ( s ) bond

f
8. The molecule which has zero dipole moment is
(d) N3 - , O2 - , F- and Na + (1989, 1M)
(a) CH2 Cl 2 (b) BF3 (c) NF3 (d) ClO2
ks
2. Which of the following compounds contain(s) no covalent
Yo
bond(s)? 9. Element X is strongly electropositive and element Y is
oo
KCl, PH3 , O2 , B2 H6 , H2 SO4 (2018 Main) strongly electronegative. Both are univalent. The compound
(a) KCl, B2 H6 , PH3 (b) KCl, H2 SO4 formed would be (1980, 1M)
eB

+ - - +
(c) KCl (d) KCl, B2 H6 (a) X Y (b) X Y (c) X -- Y (d) X ® Y
3. The intermolecular interaction that is dependent on the 10. Which of the following compound is covalent? (1980, 1M)
inverse cube of distance between the molecules is (2015 Main) (a) H2 (b) CaO
r
ou

(a) ion-ion interaction (b) ion-dipole interaction (c) KCl (d) Na 2 S


ad

(c) London force (d) hydrogen bond 11. The total number of electrons that take part in forming the
Y

4. The nodal plane in the p-bond of ethene is located in bonds in N2 is (1980, 1M)
(a) the molecular plane (2002, 3M) (a) 2 (b) 4 (c) 6 (d) 10
nd
Re

(b) a plane parallel to the molecular plane 12. The compound which contains both ionic and covalent bonds
(c) a plane perpendicular to the molecular plane which is (1979, 1M)
bisects the carbon-carbon s-bond at right angle
Fi

(a) CH4 (b) H2 (c) KCN (d) KCl


(d) a plane perpendicular to the molecular plane which
contains the carbon-carbon s-bond
Objective Questions II
5. Amongst H2 O, H2 S, H2 Se and H2 Te, the one with the highest
boiling point is (2000, 1M) (One or more than one correct option)
(a) H2 O because of hydrogen bonding 13. Dipole moment is shown by (1986, 1M)
(b) H2 Te because of higher molecular weight (a) 1, 4-dichlorobenzene (b) cis-1, 2-dichloroethene
(c) H2 S because of hydrogen bonding (c) trans-1, 2-dichloroethene (d) trans-1, 2-dichloro-2- pentene
(d) H2 Se because of lower molecular weight
6. Arrange the following compounds in order of increasing Numerical Value
dipole moment, toluene (I), m-dichlorobenzene (II), 14. Among the species given below, the total number of
o-dichlorobenzene (III), p-dichlorobenzene (IV) (1996, 1M) diamagnetic species is____
(a) I < IV < II < III (b) IV < I < II < III H atom, NO2 monomer, O-2 (superoxide), dimeric sulphur in
(c) IV < I < III < II (d) IV < II < I < III vapour phase, Mn 3 O4 ,( NH4 )2 [ FeCl 4 ], ( NH4 )2 [ NiCl 4 ],
K 2 MnO4 , K 2 CrO4 (2018 Adv.)
46 Chemical Bonding

Assertion and Reason Subjective Questions


Read the following questions and answer as per the direction 19. Arrange the following ions in order of their increasing radii:
given below: Li + , Mg 2+ , K + , Al 3+ . (1997, 1M)
(a) Statement I is true; Statement II is true; Statement II is + +
20. Between Na and Ag , which is stronger Lewis acid and
the correct explanation of Statement I
why? (1997, 3M)
(b) Statement I is true; Statement II is true; Statement II is -
not the correct explanation of Statement I 21. In the reaction, I + I2 ¾® I-3 , which is the Lewis acid?
(1997, 1M)
(c) Statement I is correct; Statement II is incorrect
(d) Statement I is incorrect; Statement II is correct 22. Explain the difference in the nature of bonding in LiF and
LiI. (1996, 2M)
15. Statement I LiCl is predominantly a covalent compound. -29
23. The dipole moment of KCl is 3.336 ´ 10 C-m which

w
Statement II Electronegativity difference between Li and indicates that it is a highly polar molecule. The interatomic
Cl is too small. (1998, 2M) distance between K + and Cl - in this molecule is
2.6 ´ 10-10 m. Calculate the dipole moment of KCl molecule

Flo
Fill in the Blank if there were opposite charges of one fundamental unit
16. There are …… p bonds in a nitrogen molecule. (1982, 1M) located at each nucleus. Calculate the percentage ionic
character of KCl.

ree
(1993, 2M)
True/False 24. Give reasons in two or three sentences only for the following :
17. All molecules with polar bonds have dipole moment.

F
“Hydrogen peroxide acts as an oxidising as well as a
1
(1985, M) reducing agent.’’ (1992, 1M)
2

or
25. State four major physical properties that can be used to
ur
18. Linear overlapping of two atomic p-orbitals leads to a sigma
distinguish between covalent and ionic compounds. Mention
bond.

f
(1983, 1M)
the distinguishing features in each case. (1978, 2M)
ks
Yo
oo
Topic 2 VBT, Hybridisation and VSEPR Theory
eB

Objective Questions I (Only one correct option) 4. The size of the iso-electronic species Cl - , Ar and Ca 2+ is
1. The correct statements among I to III are : affected by (2019 Main, 8 April I)
(a) azimuthal quantum number of valence shell
r

I. Valence bond theory cannot explain the color exhibited


ou

(b) electron-electron interaction in the outer orbitals


ad

by transition metal complexes.


II. Valence bond theory can predict quantitatively the (c) principal quantum number of valence shell
Y

magnetic properties of transition metal complexes. (d) nuclear charge


III. Valence bond theory cannot distinguish ligands as weak 5. In which of the following processes, the bond order has
nd

and strong field ones. increased and paramagnetic character has changed to
Re

(2019 Main, 9 April II)


(a) II and III only diamagnetic? (2019 Main, 9 Jan II)
(b) I, II and III
Fi

(a) O2 ® O2+ (b) N2 ® N2+


(c) I and II only (c) O2 ® O22 - (d) NO ® NO+
(d) I and III only
6. Total number of lone pair of electron in I-3 ion is (2018 Main)
2. The correct statement about ICl 5 and ICl -4 is
(2019 Main, 8 April II) (a) 3 (b) 6
(a) ICl 5 is square pyramidal and ICl -4 is tetrahedral (c) 9 (d) 12

(b) ICl 5 is square pyramidal and ICl -4 is square planar 7. The group having isoelectronic species is (2017 Main)
2- - + 2+ - - +
(a) O , F , Na , Mg (b) O , F , Na, Mg
(c) Both are isostructural
2- -
(d) ICl 5 is trigonal bipyramidal and ICl -4 is tetrahedral (c) O , F , Na, Mg 2+
(d) O- , F- , Na + , Mg 2 +

3. The ion that has sp 3 d 2 hybridisation for the central atom, is 8. The correct statement for the molecule, CsI3 is (2014 Main)
(2019 Main, 8 April II) (a) it is a covalent molecule
(a) [ICl 2 ]- (b) [BrF2 ]- (b) it contains Cs + and I-3 ions
(c) it contains Cs 3+ and I- ions
(c) [ICl 4 ]- (d) [IF6 ]-
(d) it contains Cs + , I- and lattice I2 molecule
Chemical Bonding 47

9. The species having pyramidal shape is (2010) (a) sp, sp 3 and sp 2 respectively
(a) SO3 (b) BrF3 (c) SiO2-
3 (d) OSF2 (b) sp, sp 2 and sp 3 respectively
(c) sp 2 , sp and sp 3 respectively
10. Assuming that Hund’s rule is violated, the bond order and
(d) sp 2 , sp 3 and sp respectively
magnetic nature of the diatomic molecule B 2 is (2010)
(a) 1 and diamagnetic (b) 0 and diamagnetic 22. In the compound CH2 == CH ¾ CH2 ¾ CH2 ¾ C ºº CH, the
(c) 1 and paramagnetic (d) 0 and paramagnetic C2 ¾ C3 bonds is of (1999, 2M)
(a) sp - sp 2 (b) sp 3 - sp 3
11. The species having bond order different from that in CO is (c) sp - sp 3 (d) sp 2 - sp 3
(a) NO- (b) NO+ (2007, 3M)
23. The geometry of H2 S and its dipole moment are (1999, 2M)
(c) CN- (d) N2
(a) angular and non-zero (b) angular and zero
12. Among the following, the paramagnetic compound is (c) linear and non-zero (d) linear and zero

w
(2007, 3M)
(a) Na 2 O2 (b) O3 (c) N2 O (d) KO2 24. The geometry and the type of hybrid orbital present about the
central atom in BF3 is (1998, 2M)
13. Which of the following contains maximum number of lone (a) linear, sp (b) trigonal planar, sp 2

Flo
pairs on the central atom? (2005, 1M)
(c) tetrahedral, sp 3 (d) pyramidal, sp 3
(a) ClO -3 (b) XeF 4 (c) SF 4 (d) I -3
25. Which one of the following compounds has

ree
14. Number of lone pair(s) in XeOF4 is/are (2004, 1M) sp 2 - hybridisation? (1997, 1M)
(a) 0 (b) 1 (c) 2 (d) 3 (a) CO2 (b) SO2

F
15. Which of the following are isoelectronic and isostructural ? (c) N2 O (d) CO
NO–3 , CO2– –
3 , ClO3 , SO3 (2003, 1M)
26. Among KO2 , AlO2- , BaO2 and NO2+ , unpaired electron is
(a) NO–3 , CO2- (b) SO3 , NO3–

or
3
ur present in (1997 C, 1M)
(c) ClO–3 , CO2– (d) CO2–
3 , SO3
3 (a) NO+2 and BaO2 (b) KO2 and AlO-2
16. Among the following, the molecule with the highest dipole
f
(c) Only KO2 (d) Only BaO2
ks
moment is (2003, 1M) -
27. The cyanide ion CN and N2 are isoelectronic, but in contrast
Yo
(a) CH 3 Cl (b) CH 2 Cl 2
to CN- , N2 is chemically inert because of
oo
(1997 C, 1M)
(c) CHCl 3 (d) CCl 4
(a) low bond energy
eB

17. Which of the following molecular species has unpaired (b) absence of bond polarity
electron (s)? (2002, 3M) (c) unsymmetrical electron distribution
(a) N2 (b) F2 (c) O–2 (d) O2–
2 (d) presence of more number of electron in bonding orbitals
r

18. Specify the coordination geometry around and hybridisation 28. Among the following species, identify the isostructural pairs.
ou
ad

of N and B atoms in a 1 : 1 complex of BF3 and NH3 . NF3 , NO-3 , BF3 , H3 O+ , N3 H (1996, 1M)
(a) N : tetrahedral, sp 3 ; B: tetrahedral, sp 3
Y

(2002, 3M)
(b) N : pyramidal, sp 3 ; B: pyramidal, sp 3 (a) [NF3 ,NO-3 ] and [BF3 ,H3 O+ ]
(c) N: pyramidal, sp 3 ; B: planar, sp 2 (b) [NF3 , N3 H] and [NO-3 ,BF3 ]
nd
Re

(d) N: pyramidal, sp 3 ; B: tetrahedral, sp 3 (c) [ NF3 , H3 O+ ] and [NO–3 , BF3 ]


19. The correct order of hybridisation of the central atom in the (d) [NF3 , H3 O+ ] and [N3 H, BF3 ]
Fi

2-
following species NH3 , [PtCl 4 ] , PCl 5 and BCl 3 is
29. Which one of the following molecules is planar? (1996, 1M)
(a) dsp 2 , dsp 3 , sp 2 and sp 3 (2001, 1M)
(a) NF3 (b) NCl 3 (c) PH3 (d) BF3
(b) sp 3 , dsp 2 , sp 3 d and sp 2
(c) dsp 2 , sp 2 , sp 3 and dsp 3 30. The maximum possible number of hydrogen bonds a water
(d) dsp 2 , sp 3 , sp 2 and dsp 3 molecule can form is (1992, 1M)

20. The common features among the species CN – , CO (a) 2 (b) 4 (c) 3 (d) 1
and NO+ are (2001, 1M)
31. The type of hybrid orbitals used by the chlorine atom in
(a) bond order three and isoelectronic ClO-2 is (1992, 1M)
(b) bond order three and weak field ligands (a) sp 3 (b) sp 2
(c) bond order two and acceptors (c) sp (d) None of these
(d) isoelectronic and weak field ligands
32. The molecule which has pyramidal shape is (1989, 1M)
21. The hybridisation of atomic orbitals of nitrogen in (a) PCl 3 (b) SO3
NO+2 , NO-3 and NH+4 are (2000, 1M) (c) CO2– (d) NO–3
3
48 Chemical Bonding

33. Which of the following is paramagnetic? (1989, 1M) Objective Questions II


(a) O–2 (b) CN– (One or more than one correct option)
(c) CO (d) NO+ 45. The molecules that will have dipole moment are (1992, 1M)
34. The Cl—C—Cl angle in 1, 1, 2, 2-tetrachloroethene and (a) 2, 2-dimethyl propane (b) trans-2-pentene
tetrachloromethane respectively will be about (1988, 1M) (c) cis-3-hexene (d) 2,2,3,3-tetramethyl butane
(a) 120° and 109.5° (b) 90° and 109.5° 46. Which of the following have identical bond order?
(c) 109° and 90° (d) 109.5° and 120° (a) CN– (b) O–2 (1992, 1M)
35. The molecule that has linear structure is (1988, 1M) (c) NO+ (d) CN+
(a) CO2 (b) NO2 (c) SO2 (d) SiO2 47. The linear structure assumed by (1991, 1M)
(c) NO+2 (d) NCO–

w
2
36. The species in which the central atom uses sp -hybrid (a) SnCl 2 (b) CS2
orbitals in its bonding is (1988, 1M)
48. CO2 is isostructural with
(a) PH3 (b) NH3 (c) CH+3 (d) SbH3 (1986, 1M)
(a) HgCl 2 (b) C2 H2 (c) SnCl 2 (d) NO2

Flo
37. Of the following compounds, which will have a zero dipole
moment ? (1987, 1M)
Match the Columns

ree
(a) 1, 1-dichloroethylene
(b) cis-1, 2-dichloroethylene 49. Match the orbital overlap figures shown in Column I with the
description given in Column II and select the correct answer
(c) trans-1, 2-dichloroethylene

F
using the codes given below the Columns. (2014 Adv.)
(d) None of the above
Column I Column II
38. The hybridisation of sulphur in sulphur dioxide is (1986, 1M)

or
(a) sp
ur
(b) sp 3 A. 1. p-d p antibonding

f
2
(c) sp (d) dsp 2
ks
39. The bond between two identical non-metal atoms has a pair B. 2. d-d s bonding
Yo
of electrons (1986, 1M)
oo
(a) unequally shared between the two
eB

(b) transferred fully from one atom to another C. 3. p-dp bonding


(c) with identical spins
(d) equally shared between them
r

40. On hybridisation of one s and one p-orbital we get D. 4. d-d s antibonding


ou
ad

(a) two mutually perpendicular orbitals (1984, 1M)


(b) two orbitals at 180°
Y

(c) four orbitals directed tetrahedrally Codes


(d) three orbitals in a plane A B C D A B C D
nd

(a) 4 3 2 1 (b) 1 2 3 4
Re

41. Carbon tetrachloride has no net dipole moment because of


(c) 2 3 1 4 (d) 4 1 2 3
(a) its planar structure (1983, 1M)
Fi

50. Match each of the diatomic molecules in Column I with its


(b) its regular tetrahedral structure
property/properties in Column II. (2009)
(c) similar sizes of carbon and chlorine atoms
(d) similar electron affinities of carbon and chlorine Column I ColumnII
A. B2 p. Paramagnetic
42. The ion that is isoelectronic with CO is (1982, 1M)
B. N2 q. Undergoes oxidation
(a) CN- (b) O+2
C. O-2 r. Undergoes reduction
(c) O-2 (d) N+2
D. O2 s. Bond order ³ 2
43. Among the following, the linear molecule is (1982, 1M)
t. Mixing of ‘s’ and ‘p’ orbitals
(a) CO2 (b) NO2
(c) SO2 (d) ClO2 Codes
44. If a molecule MX 3 has zero dipole moment, the sigma A B C D
(a) q, r, s p, r, t, s q, r, t p, q, t
bonding orbitals used by M (atomic number < 21) are
(b) p, q, r, t q, r, s, t p, q, r, t p, r, s, t
(a) pure p (b) sp-hybridised (1981, 1M) (c) q, r, s, t p, q, r r, s, t p, q, r, t
(c) sp 2 -hybridised (d) sp 3 -hybridised (d) p, q, s, t p, q, s p, t q, r, t
Chemical Bonding 49

Fill in the Blanks atom does not have contribution from the d-orbital(s) is
[atomic number of S = 16 , Cl = 17 , I = 53 and Xe = 54]
51. Among N2 O, SO2 , I+3 and I–3 , the linear species are …… and (2015 adv.)
…… (1997 C, 1M) 68. A list of species having the formula XZ4 is given below
52. When N2 goes to N+2 , the N ¾ N bond distance … , and when (2014 Adv.)

O2 goes to O+2 the O ¾ O bond distance …… XeF4 , SF4 , SiF4 , BF4- , BrF4- , [Cu(NH3 )4 ] 2+ , [FeCl 4 ] 2- ,
(1996, 1M)
[CoCl 4 ] 2- and [PtCl 4 ] 2-
53. The two types of bonds present in B2 H6 are covalent and
Defining shape on the basis of the location of X and Z atoms,
…… (1994, 1M)
the total number of species having a square planar shape is
54. The kind of delocalisation involving sigma bond orbitals is
69. The total number of lone-pair of electrons in melamine is
called................. (1994, 1M)
(2013 Adv.)

w
55. The valence atomic orbitals on C in silver acetylide is 70. Based on VSEPR theory, the number of 90° F—Br—F
.............hybridised. (1990, 1M) angles in BrF5 is (2010)
56. The shape of CH3 + is …… . (1990, 1M)

Flo
Subjective Questions
57. …… hybrid orbitals of nitrogen atom are involved in the
formation of ammonium ion. (1982, 1M) 71. Predict whether the following molecules are isostructural or

ree
not. Justify your answer.
58. Pair of molecules which forms strongest intermolecular (i) NMe3 (ii) N(SiMe3 )3 (2005, 2M)
hydrogen bonds is ……… . (SiH4 and SiF4 , acetone and

F
CHCl 3 , formic acid and acetic acid) (1981, 1M) 72. On the basis of ground state electronic configuration, arrange
the following molecules in increasing O—O bond length
59. The angle between two covalent bonds is maximum in …… . order. KO2 , O2 , O2 [AsF6 ] (2004, 2M)

or
(CH4 , H2 O, CO2 )
ur (1981, 1M)
73. Draw the shape of XeF4 and OSF4 according to VSEPR

f
True/False theory. Show the lone pair of electrons on the central atom.
ks
(2004, Main, 2M)
60. The dipole moment of CH3 F is greater than that of CH3 Cl.
Yo
(1993, 1M) 74. Using VSEPR theory, draw the shape of PCl 5 and BrF5 .
oo
61. H2 O molecule is linear. (2003, 2M)
(1993, 1M)
eB

62. The presence of polar bonds in a polyatomic molecule 75. Draw the molecular structures of XeF 2 , XeF 4 and XeO2 F2 ,
indicating the location of lone pair(s) of electrons. (2000, 3M)
suggests that the molecule has non-zero dipole moment.
(1990, 1M) 76. Interpret the non-linear shape of H2 S molecule and
r

3 non-planar shape of PCl 3 using valence shell electron pair


63. sp hybrid orbitals have equal s and p character.
ou

repulsion (VSEPR) theory. (Atomic number : H = 1, P = 15,


ad

(1987, 1M)
S = 16, Cl = 17) (1998, 4M)
Y

64. In benzene, carbon uses all the three p-orbitals for


77. Using the VSEPR theory, identify the type of hybridisation
hybridisation. (1987, 1M)
and draw the structure of OF2 . What are the oxidation states
nd

65. SnCl 2 is a non-linear molecule. of O and F ? (1997, 3M)


Re

1
(1985, M)
2 78. Write the Lewis dot structural formula for each of
Fi

the following. Give also, the formula of a neutral molecule,


Integer Type Questions
which has the same geometry and the same arrangement of
66. The sum of the number of lone pairs of electrons on each the bonding electrons as in each of the following. An
central atom in the following species is example is given below in the case of H3 O+ and NH3 .
[TeBr6 ]2 - , [BrF2 ]+ , SNF3 and [XeF3 ]- +
é H ù é H ù
(Atomic numbers : N = 7, F = 9, S = 16, Br = 35, ê ···· ú ê ···· ú
Te = 52, Xe = 54) (2017 Adv.) êH · O · Hú êH · N · Hú
ê ·· ú ê ·· ú
67. Among the triatomic molecules/ions BeCl 2 , N-3 , N2 O, NO+2 , êë úû êë úû
O3 , SCl 2 , ICl -2 , I-3 and XeF2 , the total number of linear Lewis dot. structure .
Neutral molecule (1983, 4M)
molecules(s)/ion(s) where the hybridisation of the central (i) O2- (ii) CO2- (iii) CN -
(iv) NCS-
2 3
50 Chemical Bonding

Topic 3 Resonance, LCAO, MOT, Other Bonding Types


Objective Questions I (Only one correct option) 13. Hyperconjugation involves overlap of which of the
1. During the change of O2 to O-2 , the incoming electron goes to following orbitals? (2008, 3M)

the orbital. (2019 Main, 10 April I)


(a) s - s (b) s - p (c) p - p (d) p - p
(a) p2 px *
(b) p 2 px (c) p2 p y (d) s* 2 p z 14. According to MO theory, (2004, 1M)
(a) O+2 is paramagnetic and bond order greater than O2
2. HF has highest boiling point among hydrogen halides,
(b) O+2 is paramagnetic and bond order less than O2
because it has (2019 Main, 9 April II)
(a) lowest ionic character (c) O+2 is diamagnetic and bond order is less than O2
(b) strongest van der Waals’ interactions (d) O+2 is diamagnetic and bond order is more than O2

w
(c) strongest hydrogen bonding 15. Molecular shape of SF4 , CF4 and XeF4 are
(d) lowest dissociation enthalpy (2000, 1M)
(a) the same, with 2, 0 and 1 lone pair of electrons respectively
3. Among the following species, the diamagnetic molecule is

Flo
(2019 Main, 9 April II)
(b) the same, with 1, 1 and 1 lone pair of electrons respectively
(a) CO (b) B2 (c) NO (d) O2 (c) different, with 0, 1 and 2 lone pair of electrons respectively

ree
(d) different, with 1, 0 and 2 lone pair of electrons respectively
4. Among the following, the molecule expected to be stabilised
by anion formation is C2 , O2 , NO, F2 . (2019 Main, 9 April I) 16. In compounds of type ECl 3 , where E = B, P, As or Bi, the
angles Cl—E—Cl is in order

F
(1999, 2M)
(a) C2 (b) F2
(a) B > P = As = Bi (b) B > P > As > Bi
(c) NO (d) O2
(c) B < P = As = Bi (d) B < P < As < Bi

or
5. Among the following molecules/ions, C2-
ur 2- 2-
2 , N2 , O2 , O2 17. The correct order of increasing C¾ O bond length of

f
Which one is diamagnetic and has the shortest bond length? CO, CO23 - , CO2 is (1999, 2M)
(2019 Main, 8 April II)
CO23 -
ks
(a) < CO2 < CO (b) CO2 < CO2– < CO
(a) C2-
2 (b) O2 (c) O2-
2 (d) N 2-
2
3
Yo
(c) CO < CO2– < CO2 (d) CO < CO2 < CO2–
oo
3 3
6. Two pi and half sigma bonds are present in
(2019 Main, 10 Jan I) 18. Which contains both polar and non-polar bonds? (1997, 1M)
eB

(a) O+2 (b) N2 (c) N+2 (d) O2 (a) NH4 Cl (b) HCN (c) H2 O2 (d) CH4
7. According to molecular orbital theory, which of the following 19. Which one among the following does not have the hydrogen
is true with respect to Li +2 and Li -2 ? bond? (1983, 1M)
r

(2019 Main, 9Jan I)


(a) Phenol (b) Liquid NH3
ou

(a) Both are unstable


ad

(b) Li+2 is unstable and Li-2 is stable (c) Water (d) HCl
Y

(c) Both are stable Objective Question II


(d) Li+2 is stable and Li-2 is unstable (One or more than one correct option)
nd
Re

8. According to molecular orbital theory, which of the 20. According to molecular orbital theory, which of the
following will not be a viable molecule? (2018 Main) following statements is(are) correct? (2016 adv.)
Fi

(a) He2+
2 (b) He+2 (c) H-2 (d) H2-
2 (a) C2-
2 is expected to be diamagnetic

9. Which of the following species is not paramagnetic? (b) O2+


2 is expected to have a longer bond length than O2
(2017 Main) (c) N+2 and N-2 have the same bond order
(a) NO (b) CO (c) O2 (d) B2
(d) He+2 has the same energy as two isolated He atoms
10. Assuming 2s-2p mixing is not operative, the paramagnetic
21. Hydrogen bonding plays a central role in which of the
species among the following is (2014 Adv.)
following phenomena? (2014 Adv.)
(a) Be2 (b) B2 (c) C2 (d) N2
(a) Ice floats in water
11. Stability of the species Li 2 , Li -2 and Li +2 increases in the order (b) Higher Lewis basicity of primary amines than tertiary
of (2013 Main) amines in aqueous solutions
(a) Li 2 < Li 2+ < Li 2- (b) Li –2 < Li +2 < Li 2 (c) Formic acid is more acidic than acetic acid
(c) Li 2 < Li -2 < Li 2+ (d) Li -2 < Li 2 < Li +2 (d) Dimerisation of acetic acid in benzene
12. In which of the following pairs of molecules/ions both the 22. Which one of the following molecules is expected to exhibit
species are not likely to exist? (2013 Main)
diamagnetic behaviour? (2013 Main)
- 2- 2+ (a) C 2 (b) N 2 (c) O 2 (d) S 2
(a) H+2 , He2-
2 (b) H 2 , He 2 (c) H2 , He 2 (d) H-2 , He2+
2
Chemical Bonding 51

Assertion and Reason Column I Column II

Read the following questions and answer as per the direction given
C. MnO-4 + NO2- 3.
Dimeric bridged
below: + H+ ¾® tetrahedral metal ion
(a) Statement I is correct; Statement II is correct; Statement II is D. NO-3 + H2 SO4 4.
the correct explanation of Statement I. 2+ Disproportionation
+ Fe ¾®
(b) Statement I is correct; Statement II is correct; Statement II is
not the correct explanation of Statement I.
(c) Statement I is correct; Statement II is incorrect. Codes
(d) Statement I is incorrect; Statement II is correct. A B C D A B C D
(a) 2 1, 4 3 4 (b) 1, 4 3 1, 2 1
23. Statement I The electronic structure of O 3 is
(c) 2 3 1 4 (d) 3 4 2, 3 1

w
+
O
·
··
·
-
·
Integer Questions
·O ·O· .
·· ·· 25. Among H2 ,He+2 , Li 2 , Be2 , B2 , C2 , N2 , O-2 and F2 , the

Flo
O number of diamagnetic species is
··
·
O ·· structure is not allowed (Atomic numbers : H = 1, He = 2, Li = 3, Be = 4, B = 5, C = 6,
Statement II ·O

ree
·· ·· N = 7, O = 8, F = 9) (2017 Adv.)
because octet around O cannot be expanded.
Subjective Questions

F
(1998, 2M)

Match the Columns 26. Write the MO electron distribution of O2 . Specify its bond

or
ur
24. Match the reactions in Column I with nature of the order and magnetic property. (2000, 3M)
reactions/type of the products in Column II. (2007, 6M) 27. Arrange the following as stated.
Column I Column II
f
“Increasing strength of hydrogen bonding ( X ¾ H ¾ X ).”
ks
A. O-2 ¾® O2 + O22 - 1. Redox reaction O, S, F, Cl, N (1991, 1M)
Yo
oo
B. 2. One of the products has 28. What effect should the following resonance of vinyl chloride
CrO24 - + H+ ¾® have on its dipole moment?
trigonal planar structure (1987, 1M)
eB

+
CH2 ==CH ¾ Cl ¬¾® CH–2 ¾ C HCl

Answers
r
ou
ad

Topic 1
Y

1. (d) 2. (c) 3. (b) 4. (a) 49. (c) A ® 2; B ® 3; C ®1; D ®4


5. (a) 6. (b) 7. (b) 8. (b) 50. (b) A ® p, q, r, t; B ® q, r, s, t; C ® p, q, r, t; D ® p, r, s, t
9. (a) 10. (a) 11. (c) 12. (c) 51. N 2O, I -3 52. increases, decreases
nd
Re

13. (b, d) 14. (1) 15. (c) 16. (2) 53. three centre bond-two electrons
17. F 18. T 23. (80.2%)
Fi

54. hyperconjugation
55. sp 56. Triangular planar 57. sp 3
Topic 2
1. (d) 2. (b) 3. (c) 4. (d) 58. HCOOH and CH3COOH 59. CO2 60. F
5. (d) 6. (c) 7. (a) 8. (d) 61. F 62. F 63. F 64. F
9. (d) 10. (a) 11. (a) 12. (d) 65. T 66. (6) 68. (4) 69. (6)
13. (d) 14. (b) 15. (a) 16. (a)
Topic 3
17. (c) 18. (a) 19. (b) 20. (a)
1. (b) 2. (c) 3. (a) 4. (a)
21. (b) 22. (d) 23. (a) 24. (b)
5. (a) 6. (c) 7. (d) 8. (d)
25. (b) 26. (c) 27. (b) 28. (c)
9. (b) 10. (c) 11. (b) 12. (c)
29. (d) 30. (b) 31. (a) 32. (a)
13. (b) 14. (a) 15. (d) 16. (b)
33. (a) 34. (a) 35. (a) 36. (c)
17. (a) 18. (c) 19. (d) 20. (a, c)
37. (c) 38. (c) 39. (d) 40. (b)
21. (a, b, d) 22. (a,b) 23. (a)
41. (b) 42. (a) 43. (a) 44. (c)
24. (b) A ® 1, 4; B ® 3; C ®1, 2; D ®1 25. (6)
45. (b, c) 46. (a, c) 47. (b, c, d) 48. (a, b)
Hints & Solutions
Topic 1 Preliminary Concepts of Electrovalent and F =
2q2r – 4 q2a
(In case of dipole-dipole interaction)
and Covalent Bonding 4 p Î0 r3
1. Key Idea Isoelectronic species contains same number of From the above, it is clear that the ion-dipole interaction is the
electrons. better answer as compared to dipole-dipole interaction, i.e.
hydrogen bonding.
The species with its atomic number and number of electrons are
as follows :
H H
Species (ions) At. no. ( Z ) No. of electrons
4. C—–C
N3- 7 + 3 = 10

w
7
O2- 8 8 + 2 = 10 H H
-
F 9 9 + 1 = 10
Pi bond is formed by the p-orbitals whose lobes have minima in
Na + 11 11 - 1 = 10

Flo
+
the plane of molecule, hence molecular plane is the nodal plane
Li 3 3 -1 = 2 of pi-bond.
Mg 2+
12 12 - 2 = 10

ree
5. H-bond is the strongest intermolecular force.
Thus, option (d) contains isoelectronic set of ions. All are different with 1, 0 and 2 lone pairs of electrons at central
2. KCl is the only ionic compound. The structure of PH3, O2, B2H6 atom.

F
and H2SO4 are given below 6. p-dichlorobenzene is non-polar.

or
P
ur
Covalent
bond
Cl— —Cl

f
O O
H H (O2) p-dichlorobenzene
H
ks
(PH3) Covalent The two dipole vectors cancelling each other giving zero
Bonds resultant dipole moment. o-dichlorobenzene has greater
Yo
H H 1.33Å Å H
1.19
oo
120° B 97° B dipole moment than meta-isomer.
H H H Cl
eB

H
Cl m1 > m2
H H
H H H m1 m2
B B B B
r

H H H Cl Cl
ou
ad

H H (B2H6) H Banana bond (o-dichlorobenzene) (m-dichlorobenzene)


O dipole vectors are at 60° angle dipole vectors are at 120° angle
Y

S Toluene is less polar than both ortho and para


HO OH O dichlorobenzene. Therefore, the increasing order of dipole
nd
Re

Sulphuric acid moment is


(H2SO4) p-dichlorobenzene (IV) < toluene (I) < m-dichlorobenzene (II)
Fi

All bond between S and O atom are covalent bonds. < o-dichlorobenzene (III)

3. Ion-ion interaction is dependent on the square of distance, i.e. 7. The carbide (C2-
2 ) ion has the following bonding pattern:
1 – –
ion-ion interaction µ : C ºº C : one sigma and two pi bonds.
r2
1 8. BF3 has triangular planar arrangement. F
Similarly, ion-dipole interaction µ
r3 Three identical vectors acting in outward F—B 120°
1 1
London force µ and dipole-dipole interaction µ direction at equal angles in a plane cancel each F
r6 r3 other giving zero resultant, hence non-polar. sp2
Superficially it seems as both ion-dipole interaction and 9. Strongly electropositive, univalent X will form
hydrogen bonding vary with the inverse cube of distance an 1 : 1 ionic compound with strongly electronegative, univalent
between the molecules but when we look at the exact Y.
expressions of field (force) created in two situations,
it comes as X + Y ¾® X +Y -
2| P | 10. H2 is a covalent, diatomic molecule with a sigma covalent bond
| E | or | F | = (In case of ion-dipole interaction)
4 p Î r3 between two hydrogen atoms.
Chemical Bonding 53

11. N2 has triple bond and each covalent bond is associated with one 3d 4s 4p
pair of electrons, therefore, six electrons are involved in forming
sp3
bonds in N2.
Cl– Cl– Cl– Cl– hybridisation
12. In KCN, the bonding between potassium ion and cyanide ion is 4 unpaired
electrons
ionic while carbon and nitrogen are covalently bonded in cyanide
ion as: (vii) (NH4)2NiCl 4 has Ni as central metal atom with +2 oxidation
Covalent bonds state. The electronic configuration of Ni 2+ in the complex is
+ –
[K] [ C ººN] 3d 4s 4p

Ionic bond sp3


Cl– Cl– Cl– Cl– hybridisation
13. 1,4-dichlorobenzene is non-polar, individual dipole vectors 2 unpaired

w
electrons
cancel each other.
m¹0 (viii) In K2MnO4 central metal atom Mn has +6 oxidation state
Cl Cl H Cl ClCH2 H with following structure
C==C C==C C==C

Flo
O–
H H Cl H Cl C2H5
m=0 2K+ Mn
Polar Polar

ree
Non-polar O –
O O
14. Among the given species only K2CrO4 is diamagnetic as central

F
metal atom Cr in it has [ Ar ]3d 0 electronic configuration i.e., all Electronic configuration of Mn6+ is
paired electrons. The structure and oxidation state of central
3d 4s
metal atom of this compound are as follows

or
O
ur
f
one unpaired
Structure K+ Cr , Oxidation state Cr6+ electron
ks
O –
O– O 15. Statement I is correct but Statement II is incorrect. The
Yo
covalency in LiCl is due to small size of Li + ion which brings
oo
Rest all the compounds are paramagnetic. Reasons for their
paramagnetism are given below about large amount of polarisation in bond.
eB

(i) H-atom have 1s1 electronic configuration, i.e. 1 unpaired 16. These are 2p-bonds in a nitrogen molecule.
electron. 17. The resultant of individual bond dipoles may or may not be
non-zero.
N
r

(ii) NO2 , i.e. in itself is an odd electron species. 18. Linear overlapping of p-orbitals form sigma bond while
ou

O O sidewise overlapping of two p-orbitals forms a pi bond.


ad

(iii) O-2
(Superoxide) has one unpaired electron in p * molecular
19. Li + < Al 3+ < Mg2+ < K+
Y

orbital.
(iv) S2 in vapour phase has O2 like electronic configuration i.e., 20. Ag+ is stronger Lewis acid because it can easily accommodate
have 2 unpaired electrons in p * molecular orbitals. lone pair of electrons from Lewis base. On the other hand, Na +
nd

has noble gas configuration, cannot accept lone pair of


Re

(v) Mn 3O4 has following structure


electron, not at all a Lewis acid.
O O
21. I2 is Lewis acid because I - coordinate its one lone pair to I2.
Fi

+2 +4 +2
Mn Mn Mn
O O 22. Both LiF and LiI are expected to be ionic compounds.
However , LiI is predominantly covalent because of small size
Thus, Mn is showing +2 and +4 oxidation states. The outermost of Li + and large size of iodide ion. A smaller cation and a larger
electronic configuration of elemental Mn is 3d 5 4 s2. Hence, in anion introduces covalency in ionic compound.
both the above oxidation states it has unpaired electrons as
23. Dipole moment is calculated theoretically as
3d 4s
Mn2+ m = q×d
Here, q = 1.6 ´ 10-19 C and d = 2.6 ´ 10-10 m
5 unpaired electrons
3d 4s m Theo = 1.6 ´ 10-19 ´ 2.6 ´ 10-10 = 4.16 ´ 10-29 cm
Mn4+ m obs 3.336 ´ 10-29
% ionic character = ´ 100 = ´ 100
3 unpaired electrons m Theo 4.16 ´ 10-29
= 80.2%
(vi) (NH4)2FeCl 4 has Fe as central metal atom with +2 oxidation
state. The electronic configuration of Fe 2+ in the complex is
54 Chemical Bonding

24. In hydrogen peroxide (H2O2 ), oxygen is in –1 oxidation state, The hybridisation of given species are as follows :
can be oxidised to O2 (zero oxidation state) or can be reduced to ●
For [ ICl2 ]- and [ BrF2 ]-
H2O (–2 oxidation state of oxygen).
1
H = (7 + 2 - 0 + 1) = 5 (sp3d )
Hence, H2O2 can act as both oxidising agent and reducing agent. 2
With strong oxidising agent like KMnO4, H2O2 acts as a
reducing agent while with strong reducing agent like H2C2O4 , it

For [ ICl4 ]- ,
acts as an oxidising agent. 1
H = (7 + 4 - 0 + 1) = 6 (sp3d 2 )
2
25. (i) Melting points Ionic compounds have higher melting
points than covalent compounds. ●
For [ IF6 ]- ,
(ii) Boiling points Ionic compounds have higher boiling points 1
than covalent compounds. H = (7 + 6 - 0 + 1) = 7 (sp3d 3 )
2

w
(iii) Solubility Ionic compounds have greater solubility in water
than a covalent compound. 4. The radius of isoelectronic species is inversely proportional to
their nuclear charge or atomic number (Z). Thus, greater the
(iv) Conductivity in aqueous solution Ionic compounds have value of Z, lesser the radii of isoelectronic species.

Flo
greater electrical conductivity in aqueous solution while
covalent compounds are usually non-conducting. 5.
Species Valence MOs Bond Order Paramagnetic/

ree
æ N b - N a ö Diamagnetic
Topic 2 VBT, Hybridisation and çç ÷÷
è 2 ø Nature
VSEPR Theory NO(15e- ) 6-1

F
[ 8 e- ] p 2 px2 = p 2 py2s 2 pz2 Paramagnetic
= 2.5
1. Among the given statements, correct statements are I and III 2
only. Valence bond theory (VBT) cannot explain the colour –e- p * 2 p1x =p * 2 py0s * 2 pz 0

or
6-0
ur
exhibited by transition metal complexes. This theory cannot [ 8e ] -
p 2 px2 = p 2 py2s 2 pz2 =3 Diamagnetic
distinguish ligands as weak and strong field ones. NO+ (14e- ) 2

f
·· p * 2 px0 = p * 2 py0s * 2 pz0
2. For ICl5
ks
N 2 (14 e- ) [ 8 e- ]p 2 px2 = p 2 py2s2 pz2 6-0 Diamagnetic
=3
1 –e-
Yo
2
H = (7 + 5 - 0 + 0) = 6 (sp3d 2 ) 5-0
N +2 (13 e- ) p * 2 px0 = p * 2 py0 , s 2 pz0
oo
2 Paramagnetic
= 2.5
2
Cl [ 8 e- ]p 2 px2 = p 2 py2s 2 pz 1
eB

Cl Cl sp 3 d 2 hybridised
Geometry : Octahedral
p * 2 px0 = p * 2 py0s * 2 pz0
I Shape / Structure : Square pyramidal
O2 (16 e- ) [ 8 e- ] s 2 pz2p 2 px2 = p 2 py2 6-2 Paramagnetic
r

Cl Cl =2
2
ou
ad

p * 2 p1x = p * 2 p1y s * 2 pz0


For &&ICl-4 –e-
O+2 (15 e- ) [ 8 e- ] s 2 pz2p 2 px2 = p 2 py2
Y

1
H = (7 + 4 - 0 + 1) = 6 (sp3d 2 ) 6-1
2 p * 2 p1x = p * 2 py0s * 2 pz0 = 2.5 Paramagnetic
+2e- 2
-
nd

[ 8e ] s 2 pz2p 2 px2 = p 2 py2


Re

Cl Cl sp 3 d 2 hybridised
s O22- (18 e- )
Geometry : Octahedral
I
p * 2 px2 = p * 2 py2s * 2 pz0 6 - 4 = 1 Diamagnetic
Fi

Shape/Structure : Square planar


Cl Cl 2
So, only in the conversion of NO ® NO+, the bond order has
So, ICl5 and ICl-4 are isolobal but not isostructural.
increased (2.5 ® 3) and paramagnetic character has changed to
3. diamagnetic.
Key Idea The hybridisation for a central atom in a species
can be calculated using formula 6. The structure of I-3 ion is
1 –
H = (V + M - C + A )
2 I
where, H = No. of hybridised orbitals used by central atoms.
V = No. of valence electrons of the central atom.
M = No. of mono-valent atoms (bonded). I
C = No. of cationic (positive) charge.
A = No. of anionic (negative) charge. I

Hence, 9 is the correct answer.


Chemical Bonding 55

7. Isoelectronic species are those which contains same number of SF4 : F one lone pair at S.
electrons. F
S
Species Atomic number Number of electrons F
F
O2 - 8 10
·· ·· -
F- 9 10 I3- : I¾ I ·
· ¬¾ ·
· I ·· three lone pairs at central iodine.
·· ··
+
Na 11 10 O
2+ F ½½ F
Mg 12 10
14. Xe At central atom (Xe), there is one lone pair.
· ·
O- 8 9 F F
Na 11 11
15. NO-3 and CO2-
3 both have 32 electrons, central atom

w
+
Mg 12 11 sp2 hybridised, triangular planar.
\Option (a) is correct which contains isoelectronic species 16. CH3Cl has the highest dipole moment.
O2 - , F - , Na + , Mg 2 + . 17. O-2 has odd number(17) of electrons, therefore it must contain at

Flo
8. I -3 -
is an ion made up of I 2 and I which has linear shape. least one unpaired electron.
+ é ù

ree
While Cs is an alkali metal cation. F H
ê ½ ½ ú
9. 18. ê F ¾ B- ¬¾ N+ ¾ Hú Both ‘B’ and ‘N’ sp3 tetrahedral.
ê ú

F
F—S==O S ½ ½
ê F H ú

F O ë û
F F
19. NH3 = sp3 , [ PtCl 4 ]2- = dsp2 , PCl 5 = sp3d , BCl 3 = sp2

or
S is sp3 hybridised Pyramidal
ur
SO3 is planar (S is sp2 hybridised), BrF3 is T-shaped and SiO2- 20. All three have 14 electrons (iso electronic) with bond order of

f
3 is
planar (Si is sp2 hybridised). three.
ks
O H
10. For molecules lighter than O2, the increasing order of energies
Yo
+ +
of molecular orbitals is
oo
– H
O==N==O, O—N==O, N
é* 21.
é p2 p y ù êp 2 p y ..... H
eB

s1s s
*
1s s 2s s
*
2sê ú s 2 p s
*
2 p sp sp2 sp3 H
ë p2 p z û
x x
ê*
êëp 2 p z 1 2 3 4 5 6
22. CH2 == CH ¾ CH2 ¾ CH2 ¾ C ºº CH
where, p2 p y and p2 p z are degenerate molecular orbitals,
r

Hybridisation at C2 = sp2 and at C3 = sp3.


first singly occupied and then pairing starts if Hund’s rule is
ou
ad

obeyed. If Hund’s rule is violated in B2 , electronic 23. H2S has sp3 hybridised sulphur, therefore, angular in shape with
arrangement would be
Y

non-zero dipole moment.


ép2 p 2y
s1 s2 s
*
1s2 s2s2 s
*
2s2 ê ... H
ë p2 p z
nd
Re

S
No unpaired electron-diamagnetic.
H
Fi

bonding electrons - antibonding electrons


Bond order = (Non-linear, polar molecule)
2
6-4 F
= =1
2 24. F—B
11. The bond order of CO = 3. NO+ , CN- and N2 are isoelectronic F
with CO, have the same bond order as CO. NO- (16e- ) has bond sp2
order of 2. (Trigonal planar)
12. O-2 in KO2 has 17 electrons, species with odd electrons are 25. Sulphur in SO2 is sp2-hybridised.
always paramagnetic.
S
··
13. ClO-3 -
: O ¾ Cl == O one lone pair at Cl. O O
½½ Electron pair = 2 (s-bonds) + 1 (lone pair) = 3
O Hybridisation = sp 2
XeF4 : F Xe two lone pairs at Xe. Carbon in CO2 is sp-hybridised, N in N2 O is sp-hybridised,
carbon in CO is sp-hybridised.
F F
F
56 Chemical Bonding

· ·
26. Molecular orbital electronic configuration are 31. -
O ¾ Cl == O
· ·
*
p2 p 2y p 2 p 2y electron pairs at Cl = 2 (s-bonds) + 2 (lone-pairs) = 4
KO2 (O-2 ) : s1s2 s
* 2 *
1s s2s2 s 2s2 s 2 px2 *
s 2 px0
p 2 p 2z * Hybridisation at Cl = sp 3
p 2 p1z
*
32. PCl 3 has sp3-hybridised phosphorus, with one lone pair.
Has one unpaired electron in p 2 p orbital. Therefore, molecule has pyramidal shape like ammonia.
AlO-2 has both oxygen in O2- state, therefore, no unpaired
33. O-2 has odd number of electrons, hence it is paramagnetic.
electron is present.
BaO2 (O22- ) Cl
* Cl Cl 109°
* 2 *
p 2 p 2y p 2 p 2y *
s1s2 s 1s s2s2 s 2s2 s2 px2 s 2 px0 120°

w
C==C C Cl
34.
p 2 p 2z *
p 2 p 2z Cl Cl Cl Cl
Has no unpaired electron. sp2-hybridised
+ sp3-hybridised
NO+2 has [O== N==O] bonding, hence no unpaired electron.

Flo
35. CO2 is linear because carbon is sp-hybridised.
27. N2 is a neutral, non-polar, inert molecule while CN - is a highly
36. In CH+3 , there are only three electron pairs around carbon

ree
polar, highly active ion.
atom giving sp2-hybridisation state.
F

F
NF3 : N F BF3 : F—B +
H
28.
F H—C
F
F H

or
Triangular planar
Pyramidal
ur (B-sp2) sp2-hybridised
(N-sp3)

O

f
37. Dipole vectors in trans-1, 2-dichloroethylene are at 180° and
ks
+
– + O directed in opposite direction, cancelling each other.
NO3 : H3O : H
Yo
O==N H Cl
H
oo
O H
Pyramidal C==C dipole moment = 0
Triangular planar
(O-sp3)
eB

(N-sp2) Cl H

·· + ·· 38. In SO2, the Lewis-dot structure is


N3H ·· · N == N == N ¾ H
· · ·
r

Central nitrogen is sp -hybridised O == S == O


ou
ad

Therefore, NF3 , H3O+ and BF3 , NO3- pairs have same shape. Electron pairs at S = 2 (s-bonds) + 1 (lone-pair) = 3
sp2 hybridised.
Y

29. BF3 has triangular planar arrangement.


NOTE
F p-bonded electrons are not present in hybrid orbitals, therefore
nd
Re

not counted in electron pairs. Rather p bonds are formed by


F—B 120° lateral overlapping of pure p-orbitals.
F
Fi

sp2-hybridised
39. Bonds between identical non-metal is purely covalent due to
same electronegativities of the bonded atoms. Hence, the
There identical vectors acting in outward direction, at equal bonded atoms have equal holds on the shared pair of electrons.
angles in a plane, cancel each other giving zero resultant, hence 40. Hybridisation of one ‘s’ and one ‘p’ orbitals gives two
non-polar. sp hybrid orbitals oriented linearly at 180°.
30. A water molecule can form at the most four H-bonds. s + p ¾® 2 sp hybrid orbitals
41. CCl 4 has a regular tetrahedral shape.
H
Cl
O
m
H C m Cl Net dipole = 0
Four sites of H-bonding Cl
Cl
Chemical Bonding 57

42. CO has a total of 14 electrons and CN- also has 14 electrons. On the basis of above two concepts, correct matching can be done
- - - - - as shown below:
C (6e ) + N (7e ) + e ¾® CN (14 e )
43. CO2 is a linear molecule because of sp-hybridisation around
carbon atom. A. d-d s bonding
44. For non-polar MX 3, it must have triangular planar
arrangement, i.e. there should be sp2-hybridisation around M.
CH3
½ H3C H B. p-d p bonding
45. H3C ¾ C ¾ CH3 C == C
½ H CH2CH3
CH3 Polar

w
Symmetric, non-polar
C. p-d p antibonding
CH3 CH3
CH3H2C CH2CH3 ½ ½
C == C H3C ¾ C ¾ C ¾ CH3

Flo
H H ½ ½
CH3 CH3
Polar Symmetric, non-polar d-d s antibonding

ree
D.
- +
46. CN and NO are isoelectronic, have the same bond order
of 3.

F
+ \ A ® 2, B ® 3, C ® 1, D ® 4
-
47. S== C == S O== N == O O ¾ C ºº N Hence, (c) is the correct option.
Linear Linear Linear

or
·· ··
ur 50. (A) B2 : s1s2 s
*
1s2 s2 s2 s
*
2s2
p 2 p1y
paramagnetic.
Sn S

f
Cl Cl O O p 2 pz1
Bent Bent 6-4
ks
Bond order = =1
Yo
48. CO2 , HgCl 2 , C2H2 are all linear. 2
oo
Bond is formed by mixing of s and p orbitals.
49. PLAN This problem includes basic concept of bonding. It
B2 undergoes both oxidation and reduction as
can be solved by using the concept of molecular orbital
eB

theory. Heat
B2 + O2 ¾¾® B2 O3 (Oxidation)
+ ve phase B2 + H2 ¾¾® B2 H6 (Reduction)
r

p2 p 2y
ou

* *
(B) N2 : s1s2 s 1s2 s2 s2 s 2 s2 s2 px2 diamagnetic.
ad

– ve phase 10 - 4 p 2 p 2z
Bond order = =3>2
Y

Any orbital has two phase +ve and –ve. In the following 2
diagram, +ve phase is shown by darkening the lobes and –ve N2 undergoes both oxidation and reduction as
nd

by without darkening the lobes.


Re

D
N2 + O2 ¾¾¾® NO
Catalyst
N2 + 3H2 ¾¾¾® NH3
Fi

In N2 , bonds are formed by mixing of s and p orbitals.


Bonding MO Antibonding MO p 2 p2y p
*
2 p2y
(C) O2- : s1s2 s
*
1s2 s2 s2 s
*
2 s2 s2 px2 s
*
2 px0
When two same phase overlap with each other, it forms p 2 pz2 p
*
2 pz1
bonding molecular orbital otherwise antibonding. -
Paramagnetic with bond order = 1.5. O2 undergoes both oxidation
s-bond and reduction and bond involves mixing of s and p-orbitals.
p 2 p2y p
*
2 p1y
(D) O2 : s1s2 s
*
1s2 s2 s2 s
*
2 s2 s2 px2 s
*
2 px0
p-bond p 2 pz2 p
*
2 pz1
Paramagnetic with bond order = 2.
O2 undergoes reduction and the bond involves mixing of s
and p-orbitals.
51. N2O and I-3 are linear species.
58 Chemical Bonding

52. Bond order in N2 is 3 while same in N+2 is 2.5, hence bond 68. PLAN This problem includes concept of hybridisation using VBT,
distance increases as N2 goes to N+2 . VSEPR theory, etc.,
Bond order in O2 is 2 while same in O+2 is 2.5, hence bond XeF4 , BrF4- ,[Cu(NH3 )4 ]2+ , [PtCl 4 ]2- are square planar as shown
distance decreases as O2 goes to O+2 . below:
53. Three centred-2 electrons.
s
54. Hyperconjugation involves delocalisation of s-electrons. F F F F
55. sp-hybridised. Xe Br
F F F F
56. Triangular planar. Carbon in CH+3
is sp hybridised. 2

3
57. sp -hybrid orbital holding the lone pair is involved in formation
of ammonium ion. 2+ 2–
H3N NH3 Cl Cl
O O
Cu Pt

w
½½ ½½
58. H ¾ C ¾ OH and CH3 ¾ C ¾ OH . Both are capable of forming H3N NH3 Cl Cl
H-bonds. SF4 (See-saw) as shown below:
59. CO2, it is 180°.

Flo
F
60. Dipole moment (m ) = q.d F
S
Since electronegativity of F and Cl are very close, it is the

ree
F
internuclear distance (d) that decides dipole moment here. F
Hence, C ¾ Cl bond has greater dipole moment the C-F bond.
SiF4 , BF4- , [FeCl 4 ]2- , [CoCl 4 ]2- are tetrahedral as shown below:

F
61. H2O is V-shaped molecule.
s
O F F

or
H
V-shaped
urH
Si B

f
62. False F F F F
F F
ks
63. In sp3-hybrid orbital, there is 25 % s-character and 75 %
Yo
2– 2–
p-character. Cl Cl
oo
64. Carbon in benzene is sp2-hybridised, i.e. uses only two of its
p-orbitals in hybridisation. Fe Co
eB

65. Sn in SnCl 2 has sp2-hybridisation. Cl Cl Cl Cl


Cl Cl
66. S.N. Species No. of s-bonds No. of L.P at
Hence, correct integer is 4.
r

with central atom central atom


ou

2-
(i) In [ TeBr6 ] 6 1 69.
ad

PLAN Melamine is a heterocyclic compound.


(ii) In [ BrF2 ]+ 2 2
H2N N NH2
Y

(iii) In SNF3 4 0
(iv) In [ XeF3 ]- 3 3 N N
nd
Re

··
– NH2
67. Cl ¾ Be ¾ Cl N ºº N ¾® N ··
­ ­ ··
Fi

sp Each nitrogen atom has one pair of lone pair. Thus, in all six
sp
lone pairs.
·· +
Nºº N ® O ·· O == N == O 70. F
­ ·· ­ F F
sp sp
Br
All the above mentioned molecules/ions have sp-hybridised
central atom and no one pair at central atom, hence linear also.
F F
Others are :
O S Lone pair would push the Br—F bond pairs in upward direction
[Cl ¾ I ¾ Cl] and all Br—F bond angles will contract.
O O Cl Cl
sp2-bent sp3-V-shaped sp3d-linear 71. No, (i) NMe3 is pyramidal while (ii) N(SiMe3 )3 is planar. In the

[I ¾ I ¾ I] F ¾ Xe ¾ F latter case, pp - dp back bonding between N and Si makes N
sp2-hybridised.
sp3d-linear sp3d-linear
– 72. Bond order : O2- = 1.5, O2 = 2, O2+ = 2.5
[Although ICl 2 , I–3 and XeF2 all also are linear but in them
Bond length : O+2 < O2 < O2-
d-orbital contribute in hybridisation.]
Chemical Bonding 59

73. F Topic 3 Resonance, LCAO, MOT,


F
F F Other Bonding Types
Xe O S 1. The change of O2 to O-2 can be as follows:
+e – –
O2 (16 e –) O2 (17 e–)
F F [Dioxygen] [Super-oxide]
F F
Energy Energy
Square planar Trigonal bipyramidal
[8e– ] s2p2z p2px2 [8e– ] s2p2z p2p2x = p2p2y p2p
* 2
x
= p2py2 p2p
*
x = p2py
1 * 1
= p2p
* 1
74. Cl Cl F y

F F Half-filled
anti-bonding
Cl P Br p*-MOs

w
F F So, in the formation of O-2 from O2, the 17th electron goes to the
Cl Cl * *
p 2px or p 2py molecular orbital (anti-bonding) which is
Trigonal bipyramidal Square pyramidal

Flo
(P is sp3d-hybridised) (Br is sp3d 2 -hybridised) half-filled in O2.
75. 2. HF has highest boiling point among hydrogen halides because

ree
F F it has strongest hydrogen bonding. Here, the hydrogen bond
F F O exists between hydrogen of one molecule and fluorine atom of
another molecule as shown below.

F
Xe Xe Xe d+ d- d+ d- d+ d-
…H¾ F…H¾ F…H¾ F
F F In this molecule, hydrogen bond behaves like a bridge between

or
F
ur F
O
two atoms that holds one atom by covalent bond and the other

f
Linear Square planar See-saw shaped by hydrogen bond.
ks
3
76. In H2S, S is sp -hybridised with two lone pairs of electrons on it 3. Key Idea Magnetic nature can be detected by molecular
orbital theory. Presence of unpaired electrons means
Yo
3
giving V-shaped (water like) shape. In PCl 3 , P is sp -hybridised
paramagnetic and absence of unpaired electrons means
oo
with one lone pair of electrons on it. Therefore, PCl 3 is pyramidal diamagnetic in nature.
in shape.
eB

+2
Among the given options, CO is a diamagnetic molecule. It can
´´ ·· ´´ O be proved by molecular orbital (MO) theory. The electronic
´ ´ -1
77. ´ F ´· O ·´F ´ F F -1 configuration of given diatomic molecules are given below.
´´ ·· ´´ V-shaped
CO (Number of electrons = 14)
r


ou
ad

Electron pair = P = 2 + 2 = 4 Electronic configuration = s1s2 , s* 1s2 , s2s2 , s* 2s2,


Hybridisation = sp3 s2 pz2 , p 2 px2 » p 2 p2y
Y

2- Since, there is no unpaired electron in the CO molecule, so it is


é ·· ·· ù ·· ··
diamagnetic.
78. (i) O2- : ê ·· O ·¾· O ·· ú and · ·
· Cl ·¾· Cl · (Cl 2 )
nd

2
Re

ë · · ·· û ·· ·· ●
NO (Number of electrons = 15)
é ù
2- Electronic configuration = s1s2 , s* 1s2 , s2s2,
Fi

· ·
ê ·O· ú s* 2s2 , s2 pz2 , p 2 px2 » p 2 p2y, p* 2 px1 » p* 2 p0y
··
ê ·· ú
2-
(ii) CO3 : ê C ú and F · · B · · F (BF3 ) Since, NO has one unpaired electron in p* 2 px1 orbital, so it is
ê · · · · ú ·
· paramagnetic.
ê · O · · O ·ú F
êë · · · · · · · · úû ●
B2 (Number of electrons = 10)
- Electronic configuration = s1s2 , s* 1s2 , s2s2, s* 2s2, p 2 px1 » pp1y
é ··
· ·ù ·· ··
(iii) CN - : ê ·· C · · N ú and C ·· ·· O ·· (CO) Since, two unpaired electrons are present in p2 px1 and p2 p1y
··
ë û
orbital. So, it is paramagnetic.
-
é ·· ·· ù ··
·· O2 (Number of electrons = 16)
(iv) NCS- : ê ·· S Nú and ·· Cl · · C · · N (ClCN)

··C ··
·· ··
ë ·· û · ·
Electronic configuration = s1s2 , s* 1s2 , s2s2 , s* 2s2,
s 2 pz2 , p 2 px2 » p 2 p2y, p* 2 px1 » p* 2 p1y
Since, two unpaired electrons are present in p* 2 px1 and p* 2 p1y
orbital. So, it is also paramagnetic.
60 Chemical Bonding

4. C2 will be stabilised after forming anion. The electronic configuration of carbon is 1s2 2s2 2 p2. There are twelve electrons in C2. After
forming anion (i.e. C–2 ), the electronic configuration is

C-2 : (s1s) 2 (s*1s) 2 (s2 s) 2 (s * 2 s) 2 (p 2 px2 = p 2 p 2y ) (s2 pz1 ) or KK (s2 s) 2 (s* 2 s) 2 (p 2 px2 = p 2 p 2y ). s2 pz1
1 1
Bond order = (N b - N a ) = (9 - 4 ) = 2.5
2 2
For other options such as F2- , O-2 , NO- , the electronic configurations are as follows :
F2- : (s1s)2 (s* 1s)2 (s2s)2 (s* 2s)2 (s2 pz )2 (p 2 px2 = p 2 py2 ) (p* 2 px2 = p* 2 p2y )(s* 2 pz1 )
Bond order = 1 / 2(N b - N a ) = 1 / 2(10 - 9) = 0.5

O2- : (s1s)2. (s* 1s)2 (s2s)2 (s* 2s)2 (s2 pz )2(p 2 px2 = p 2 p2y ) (p* 2 px2 = p* 2 p1y )

w
1 1
Bond order = (N b - N a ) = (10 - 7) = 15 .
2 2
-

NO : (s1s) (s 1s) (s 2s) (s * 2s) (s2 pz )2 (p 2 px2 = p 2 p2y ) (p* 2 px1 = p* 2 p1y )
2 * 2 2 2

1 1

Flo
Bond order = (N b - N a ) = (10 - 6) = 2
2 2
The value of bond order of C-2 is highest among the given options. Bond order between two atoms in a molecule may be taken as an

ree
approximate measure of the bond length.
The bond length decreases as bond order increases. As a result, stability of a molecule increases.

F
5. Species MO energy order Bond order (BO) n, number of unpaired e- Magnetic character
C22- (14e- ) [ 8 e ]p =p 2s
6-0 0 Diamagnetic
=3

or
2p x2 2p z2
ur2p y
2
O2 (16e- ) [ 8 e ]s p =p p* = p* 6-2 2 Paramagnetic
=2

f
2p z2 2p x2 2p y2 2p1x 2p1y
2
O22- (18e- ) 6-4
ks
[ 8 e ]s p =p p* *
=p =1 0 Diamagnetic
2p z2 2p x2 2p y2 2p x2 2p y2
2
Yo
N22- (16e- ) 6-2
oo
* * 1
=2 2 Paramagnetic
[ 8 e ]p =p s p =p 2p y
2p x2 2p y2 2p z2 2p1x 2
eB

1
Bond length µ . So order of bond length C22- < O2 = N22- < O22-
BO (Bond order) (BO = 3) (BO = 2) (BO = 1)

The diamagnetic species with shortest bond length is C2-


2 (option-a).
r
ou
ad

6. The energy order of MOs of the given species are as follows:


O2 (16 e- ’s) = s1s2 , s* 1s2 , s2s2 , s* 2s2 , s2 p2z ,
Y

p 2 p2x = p 2 p2y , p* 2 p1x = p* 2 p1y ,


O+2 (15e- ’s ) = s1s2, s* 1s2 , s2s2, s* 2s2 , s2 p2z ,
nd
Re

p 2 p2x = p 2 p2y , p* 2 p1x » p* 2 p0y


Fi

N2 (14 e- ’s ) = s1s2 s* 1s2 , s 2s2 , s* 2s2


p 2 p2x = p 2 p2y , s2 p2z
N2+ (13e- ’s) = s1s2s* 1s2 s2s2 s* 2s2
p 2 p2x = p 2 p2z s2 p1z
Thus, in case of N+2 , two p-bonds and half s-bond are present in the bonding MOs.
7. Considering molecular orbital theory (MOT) :
The electronic configuration of Li2+ (Z = 5) = s1s2 , s* 1s2 , s2s1
Nb - Na 3 - 2 1
Bond order (BO) = = =
2 2 2
The electronic configuration of Li-2 (Z = 7) = s1s2 , s* 1s2 , s2s2 , s* 2s1
N - Na 4 - 3 1
Bond order (BO) = b = =
2 2 2
For the species having the same value of BO, the specie having lesser number of antibonding electrons [ N a ] will be more stable.
Chemical Bonding 61

Here, N a of Li+2 (2) < N a of Li -2 (3) . So, their order of stability * 2s2 , s2 p2 , p 2 px ,
* 1s2 , s2s2 , s
2
(d) N2 ® s1s2 , s
will be Li+2 > Li-2. z
p 2 py2
* 2 p0
p
8. Key Idea According to M.O.T, the viability of any molecule can x , s * 2 p0 (diamagnetic)
z
be judged through the calculation of bond order. *
p 2 p0y
Electronic Configuration Bond order
Hence, (c) is the correct choice.
He+2 s s* 2-1
= 0.5
1s 2 1s1
2 11. Li 2 (3 + 3 = 6) = s1s2 , s* 1s2 , s2s2
H-2 s 2 s* 1 2-1 Nb - Na 4 - 2
1s 1s = 0.5 Bond order = = =1
2 2 2
H2- s s* 2-2 Li +2 (3 + 3 - 1 = 5) = s1s2 , s* 1s2 , s2s1
2 1s 2 1s 2 =0
2

w
3-2 1
He2+ s 2-0 Bond order = = = 0.5
2 1s 2 =1 2 2
2
Li -2 (3 + 3 + 1 = 7) = s1s2 , s*1s2 , s2s2 s* 2s1
The molecule having zero bond order will not be viable hence,

Flo
4-3 1
H2-
2 (option d) is the correct answer. Bond order = = = 0.5
2 2
9. To identify the magnetic nature we need to check the molecular Stability order is Li -2 < Li +2 < Li 2 (because Li -2 has more number

ree
orbital configuration. If all orbitals are fully occupied, species is
of electrons in antibonding orbitals which destabilises the
diamagnetic while when one or more molecular orbitals is/are
species).
singly occupied, species is paramagnetic.

F
(a) NO (7 + 8 = 15) - s1s2 , s* 1s2 , s2s2 , s* 2s2 , 12. Species having zero or negative bond order do not exist.
H2+
2 (1 + 1 - 2 = 0) = s1s
0
p 2 px2 = p 2 p2y , p 2 pz2 , *
p 2 px1 *
=p 2 p0y

or
ur Bond order = 0
One unpaired electron is present. Hence, it is paramagnetic.
* 1s2

f
He2 (2 + 2 = 4 ) = s1s2 , s
(b) CO (6 + 8 = 14 ) - s1s2 , s* 1s2 , s2s2 , s* 2s2 ,
N - Na 2 - 2
ks
p 2 px2 = p 2 p2y, s2 pz2 Bond order = b = =0
2 2
Yo
oo
No unpaired electron is present. Hence, it is diamagnetic. So, both H2+
2 and He2 do not exist.
(c) O2 (8 + 8 = 16) - s1s2 , s* 1s2 , s2s2 , s* 2s2 , s2 pz2 , H H
eB

p 2 px2 = p 2 px2 , p* 2 px1 = p* 2 p1y ½ ½ -


Two unpaired electrons are present. 13. H ¾ C ¾ CH == CH2 ¬® H ¾ C == CH ¾ CH2
½ +
Hence, it is paramagnetic. H
H
r

(d) B2(5 + 5) - s1s2 , s* 1s2 , s2s2 , s* 2s2 , p 2 px1 = p2 p1y (I) (II)
ou
ad

Two unpaired electrons are present. I and II are hyperconjugation structures of propene and involves
Hence, it is paramagnetic. s-electrons of C—H bond and p-orbitals of pi bond in
Y

delocalisation.
10. PLAN This problem can be solved by using the concept involved in
molecular orbital theory. Write the molecular orbital electronic p 2 p2y p
*
2 p1y
nd

14. O+2 (15e- ) : s1s2 s


* 2
1s s2s2 s
*
2s2 s2 px2 s
*
2 px0
Re

configuration keeping in mind that there is no 2s-2p mixing,


then if highest occupied molecular orbital contain unpaired p 2 pz2 p
*
2 pz0
10 - 5
Fi

electron then molecule is paramagnetic otherwise diamagnetic. Bond order = = 2.5; paramagnetic.
Assuming that no 2s-2p mixing takes place the molecular orbital 2
electronic configuration can be written in the following p 2 p2y p
*
2 p1y *
O2 (16e- ) : s1s2 s
* 2
1s s2s2 s *
2s2 s2 px2 s 2 px
sequence of energy levels of molecular orbitals p 2 pz2 p
* 1s, s2s, s
* 2s, s2 p , p 2 p º p 2 p , p
* 2p º p
* 2p , s
* 2p
*
2 pz1
s1s, s 10 - 6
z x y x y z
Bond order = =2
* 1s2 , s2s2 , s
* 2s2 (diamagnetic) 2
(a) Be2 ® s1s2 , s
Hence, (a) is the correct answer.
0
* 2s2 , s2 p2 , p 2 px (diamagnetic)
* 1s2 , s2s2 , s
(b) B2 ® s1s2 , s z F F
p 2 py0 F F F
* 2s2 , s2 p2 , p 2 px ,
1

(c) C2 ® s1s2 , s * 1s2 , s2s2 , s Xe


15. S C
p 2 p1y
z
F
* 2 p0 F F F
p * 2 p0 (paramagnetic)
x , s
F
z F F
*
p 2 p0 See-saw shape Tetrahedral Square planar
y
molecule
62 Chemical Bonding

16. When E = B in BCl 3 , bond angle is 120°. When E = P, As or Bi It is also a diamagnetic species because of the absence of
unpaired electrons.
in ECl 3 , hybridisation at E will be sp3. Also, if central atoms are
O2 (8 + 8 = 16)
from same group, bond angle decreases down the group
provided all other things are similar. Hence, the order of bond or S2 = s1s2 , s* 1s2 , s2s2 , s* 2s2 ,
angles is BCl 3 > PCl 3 > AsCl 3 > BiCl 3 * 2 p1 » p
s 2 p 2 , p2 p 2 » p2 p 2 p * 2 p1
z x y x y
1
17. Bond length µ Due to the presence of two unpaired electrons, O2 and S2 both
Bond order are paramagnetic molecules.
1 4
Bond order : CO2 = 2, CO = 3 , CO23- = 1 + = 23. Statement I is correct, given structure is one of the resonance
3 3 structure of ozone.
Therefore, order of bond length is CO23- < CO2 < CO + +
O ¬¾® O

w
18. H2O2 H
O O- -O O
O—O Statement II is also correct because oxygen cannot expand its
polar bond
H octet. It is also the explanation for the given structure of ozone.

Flo
Non-polar bond
24. (A) In the reaction : O-2 ¾® O2 + O22-
19. HCl does not form hydrogen bond. For formation of hydrogen 1
Oxygen on reactant side is in - oxidation state. In

ree
bond, atleast one hydrogen atom must be bonded to one of the 2
product side, one of the oxygen is in zero oxidation state,
three most electronegative atom O , N and F.
i.e. oxidised while the other oxygen is in –1 oxidation

F
20. state, i.e. reduced. Hence, in the above reaction, oxygen
Species Electrons MOEC N B N A BO Magnetic
( O-1 / 2 ) is simultaneously oxidised and reduced
disproportionated.

or
C2- 14 s1s2 , s * 1s2 ,
ur
10 4 3
character
Diamagnetic (B) In acidic medium, CrO2- 2-
4 is converted into Cr2 O7 which
2

f
is a dimeric, bridged tetrahedral.
s 2s2 , s * 2s2 , O– O–
ks
p 2 px2 » - p 2 p2y ,
Yo
Cr Cr
s 2 pz2 O
oo
O O
O O
O2+
2
14 As above 10 4 3 Diamagnetic
eB

according to (C) MnO-4 + NO-2 + H+ ¾® Mn 2+ + NO-3


O2 16 number of 10 6 2 Paramagnetic
The above is a redox reaction and a product NO-3 has
N+2 13 electrons 9 4 2.5 Paramagnetic
trigonal planar structure.
r

N -2 15 10 5 2.5 Paramagnetic
(D) NO-3 + H2SO4 + Fe2+ ¾® Fe+ + NO
ou
ad

He+2 3 2 1 0.5 Paramagnetic The above is a redox reaction.


Y

25. H2 , Li 2 , Be2 , C2 , N2 and F2 are diamagnetic according to


Thus, (a) is correct.
molecular orbital theory.
(b) Bond order O2+ 2 > O 2 thus,
nd
Re

Bond length of O2+


2 < O 2 thus, incorrect.
p 2 p2y p
*
2 p1y
26. O2 : s1s2 s
* 2
1s s2s2 s
*
2s2 s2 px2
Fi

(c) N+2 and N-2 have same bond order thus correct. p 2 pz2 p
*
2 pz1
(d) He+2 with bond order = 0.5 is more stable thus, less energy 10 - 6
Bond order = = 2, paramagnetic.
than isolated He atoms. Thus, (d) is incorrect. 2
21. PLAN This problem can be solved by using concept of H-bonding and 27. Strength of hydrogen bonding in X—H—X depends on
applications of H-bonding. electronegativity as well as size of X . X with higher
22. C2 (6 + 6 = 12) = s1s2 , s* 1s2 , s2s2 , s* 2s2 , p2 px2 » p2 p2y electronegativity and smaller size forms stronger H-bond.
Hence, increasing order of strength of H-bond is
Since, all the electrons are paired, it is a diamagnetic species.
S < Cl < N < O < F
* 1s2, s2s2 ,
N2 ( 7 + 7 = 14 ) = s1s2 , s
28. Resonance in vinyl chloride increases polar character of the
* 2s2 , p2 px2 » p2 p 2y , s2 p 2z
s molecule.

Download Chapter Test


http://tinyurl.com/y3mna4bw or
5
States of Matter

w
Topic 1 Gaseous State 3. At a given temperature T, gases Ne, Ar, Xe and Kr are

Flo
found to deviate from ideal gas behaviour. Their equation
Objective Questions I (Only one correct option) RT
of state is given as, p = at T.
1. Points I, II and III in the following plot respectively correspond V -b

ree
to (vmp : most probable velocity) (2019 Main, 10 April II)
Here, b is the van der Waals’ constant. Which gas will
exhibit steepest increase in the plot of Z (compression

F
factor) vs p? (2019 Main, 9 April II)
Distribution function f( v)

(a) Xe (b) Ar

or
ur (c) Kr (d) Ne

f
4. Consider the van der Waals’ constants, a and b, for the
following gases.
ks
Yo
Gas Ar Ne Kr Xe
oo
a/(atm dm 6 mol -2 ) 1.3 0.2 5.1 4.1
I II III b/(10-2 dm 3 mol -1 )
eB

3.2 1.7 1.0 5.0


Speed, v
Which gas is expected to have the highest critical
(a) vmp of H 2 ( 300 K ); vmp of N 2(300 K); vmp of O2(400 K) temperature ? (2019 Main, 9 April I)
r

(b) vmp of O2 ( 400 K ); vmp of N 2(300 K); vmp of H 2(300 K) (a) Kr (b) Xe
ou
ad

(c) vmp of N 2(300 K); vmp of O2 ( 400 K ); vmp of H 2(300 K) (c) Ar (d) Ne
Y

(d) vmp of N 2(300 K); vmp of H 2(300 K); vmp of O2(400 K)


5. The combination of plots which does not represent
2. Consider the following table. isothermal expansion of an ideal gas is
nd

(2019 Main, 12 Jan II)


Re

6 -1 3 -1
Gas a/(k Pa dm mol ) b/(dm mol )
A 642.32 0.05196
Fi

B 155.21 0.04136 p p

C 431.91 0.05196
D 155.21 0.4382 O 1/Vm O Vm
(A) (B)
a and b are van der Waals’ constants. The correct statement
about the gases is (2019 Main, 10 April I)
(a) gas C will occupy lesser volume than gas A; gas B will be pVm
lesser compressible than gas D U
(b) gas C will occupy more volume than gas A; gas B will be
more compressible than gas D O O
p Vm
(c) gas C will occupy more volume than gas A; gas B will be
(C) (D)
lesser compressible than gas D
(d) gas C will occupy more volume than gas A; gas B will be (a) ( A ) and (C ) (b) ( B ) and (C )
lesser compressible than gas D (c) ( B ) and ( D ) (d) ( A ) and ( D )
64 States of Matter

6. An open vessel at 27ºC is heated until two fifth of the air 13. For one mole of a van der Waals’ gas when b = 0 and
(assumed as an ideal gas) in it has escaped from the vessel. T = 300 K, the pV vs 1/V plot is shown below. The value of
Assuming that the volume of the vessel remains constant, the the van der Waals’ constant a (atm L mol - 2 ) (2012)
temperature at which the vessel has been heated is
(2019 Main, 12 Jan II)
24.6
(a) 750 K (b) 500 K

pV (L atm mol–1)
23.1
(c) 750ºC (d) 500ºC
21.6
7. The volume of gas A is twice than that of gas B. The
compressibility factor of gas A is thrice than that of gas B at 20.1
same temperature. The pressures of the gases for equal
number of moles are (2019 Main, 12 Jan I)

w
(a) p A = 2 pB (b) 2 p A = 3 pB
(c) p A = 3 pB (d) 3 p A = 2 pB 0 2.0 3.0
8. A 10 mg effervescent tablet containing sodium bicarbonate 1/ V (mol L –1)

Flo
and oxalic acid releases 0.25 mL of CO2 at T = 29815
. K and (a) 1.0 (b) 4.5 (c) 1.5 (d) 3.0
p = 1 bar. If molar volume of CO2 is 25.0 L under such 14. The term that corrects for the attractive forces present in a

ree
condition, what is the percentage of sodium bicarbonate in real gas in the van der Waals’ equation is (2009)
each tablet? (a) nb (b) n 2 a / V 2
[Molar mass of NaHCO 3 = 84 g mol -1 ] (2019 Main, 11 Jan I)

F
(c) - ( n 2 a / V 2 ) (d) - nb
(a) 8.4 (b) 0.84 (c) 16.8 (d) 33.6
15. The given graph represent the variations of Z

or
ur
9. 0.5 moles of gas A and x moles of gas B exert a pressure of 200
(compressibility factor ( Z ) =
pV
) versus p, for three real
Pa in a container of volume 10m 3 at 1000 K. Given R is the

f
nRT
gas constant in JK -1 mol-1 , x is (2019 Main, 9 Jan I)
gases A, B and C. Identify the only incorrect statement.
ks
(2006, 5M)
2R 4 -R 4 +R 2R
Yo
(a) (b) (c) (d) C A
4-R 4+R
oo
2R 2R A Ideal gas
B
10. Two closed bulbs of equal volume (V) containing an ideal gas
eB

1 C
initially at pressure pi and temperature T1 are connected
B
through a narrow tube of negligible volume as shown in the Z
figure below. The temperature of one of the bulbs is then
r

0 p (atm)
raised to T2 . The final pressure p f is
ou

(2016 Main)
ad

(a) For the gas A, a = 0 and its dependence on p is linear at all


æ T1 ö æ T ö pressure
Y

(a) 2 pi çç ÷÷ (b) 2 pi çç 2 ÷÷ (b) For the gas B, b = 0 and its dependence on p is linear at all
è T1 + T2 ø è T1 + T2 ø
pressure
æ TT ö æ TT ö
nd

(c) 2 pi çç 1 2 ÷÷ (d) pi çç 1 2 ÷÷ (c) For the gas C, which is typical real gas for which neither a
Re

è T1 +T2 ø è T1 +T2 ø nor b = 0 . By knowing the minima and the point of


intersection, with Z = 1, a and b can be calculated
Fi

11. If Z is a compressibility factor, van der Waals’ equation at low (d) At high pressure, the slope is positive for all real gases
pressure can be written as (2014 Main)
RT a 16. If helium and methane are allowed to diffuse out of the
(a) Z = 1 + (b) Z = 1 - container under the similar conditions of temperature and
pb VRT
pressure, then the ratio of rate of diffusion of helium to
pb pb methane is (2005)
(c) Z = 1 - (d) Z = 1 +
RT RT (a) 2.0 (b) 1.0
12. For gaseous state, if most probable speed is denoted by C *, (c) 0.5 (d) 4.0
average speed by C and root square speed by C, then for a 17. For a monatomic gas kinetic energy = E. The relation with
large number of molecules, the ratios of these speeds are rms velocity is (2004, 1M)
(a) C * : C : C =1.225 : 1.128 : 1 (2013 Main) æ 2E ö
1/ 2
æ 3E ö
1/ 2

(b) C * : C : C =1.128 : 1.225 : 1 (a) u = ç ÷ (b) u = ç ÷


è mø è 2m ø
(c) C * : C : C =1 : 1.128 : 1.225 1/ 2 1/ 2
æ E ö æ E ö
(d) C * : C : C =1 : 1.225 : 1.128 (c) u = ç ÷ (d) u = ç ÷
è 2m ø è 3m ø
States of Matter 65

18. Positive deviation from ideal behaviour takes place because 27. Equal weights of ethane and hydrogen are mixed in an empty
of (2003, 1M) container at 25°C. The fraction of the total pressure exerted
(a) molecular interaction between atom and pV / nRT > 1 by hydrogen is (1993, 1M)
(b) molecular interaction between atom and pV / nRT < 1 (a) 1 : 2 (b) 1 : 1 (c) 1 : 16 (d) 15 : 16
(c) finite size of atoms and pV / nRT > 1 28. At constant volume, for a fixed number of moles of a gas the
(d) finite size of atoms and pV / nRT < 1 pressure of the gas increases with rise of temperature due to
19. Which of the following volume (V ) temperature (T ) plots (a) increase in average molecular speed (1992, 1M)
represents the behaviour of one mole of an ideal gas at the (b) increase rate of collisions amongst molecules
atmospheric pressure? (2002, 3M) (c) increase in molecular attraction
(a) V(L) (38.8 L, (b) V(L) (d) decrease in mean free path
373 K) (28.6 L,
29. According to kinetic theory of gases, for a diatomic molecule

w
373 K)
(22.4 L, (20.4 L,
273 K) 273 K) (1991, 1M)
(a) the pressure exerted by the gas is proportional to mean
T(K) T(K) velocity of the molecule

Flo
(c) V(L) (d) V(L) (b) the pressure exerted by the gas is proportional to the root
(30.6 L,
373 K) mean velocity of the molecule
(22.4 L, (22.4 L, (c) the root mean square velocity of the molecule is inversely

ree
273 K) 273 K) (14.2 L,
373 K) proportional to the temperature
(d) the mean translational kinetic energy of the molecule is

F
T(K) T(K) proportional to the absolute temperature
20. The root mean square velocity of an ideal gas at constant 30. The rate of diffusion of methane at a given temperature is
pressure varies with density (d ) as twice that of a gas X. The molecular weight of X is

or
(2001, S, 1M)
(a) d 2 (b) d
ur
(c) d (d) 1 / d (1990, 1M)

f
21. The compressibility of a gas is less than unity at STP. (a) 64.0 (b) 32.0 (c) 4.0 (d) 8.0
Therefore, 31. The density of neon will be highest at
ks
(1990, 1M)
(a) Vm > 22.4 L (b) Vm < 22.4 L (a) STP (b) 0°C, 2 atm
Yo
(2000, 1M)
oo
(c) Vm = 22.4 L (d) Vm = 44.8 L (c) 273°C, 1 atm (d) 273°C, 2 atm
22. The rms velocity of hydrogen is 7 times the rms velocity of 32. The value of van der Waals’ constant a for the gases O2 , N2 ,
eB

nitrogen. If T is the temperature of the gas (2000, 1M) NH3 and CH4 are 1.360, 1.390, 4.170 and 2.253L2 atm mol -2
(a) T (H2 ) = T (N2 ) (b) T (H2 ) > T (N2 ) respectively. The gas which can most easily be liquefied is
(c) T (H2 ) < T (N2 ) (d) T (H2 ) = 7 T (N2 ) (a) O2 (b) N2 (c) NH3 (d) CH4
r

23. A gas will approach ideal behaviour at 33. A bottle of dry ammonia and a bottle of dry hydrogen
ou

(1999, 2M)
ad

(a) low temperature and low pressure chloride connected through a long tube are opened
simultaneously at both ends the white ammonium chloride
Y

(b) low temperature and high pressure


(c) high temperature and low pressure ring first formed will be (1988, 1M)

(d) high temperature and high pressure (a) at the centre of the tube
nd
Re

24. According to Graham’s law, at a given temperature the ratio (b) near the hydrogen chloride bottle
rA (c) near the ammonia bottle
Fi

of the rates of diffusion of gases A and B is given by (d) throughout the length of the tube
rB
(where, p and M are pressures and molecular weights of 34. In van der Waals’ equation of state for a non-ideal gas, the
gases A and B respectively) term that accounts for intermolecular forces is (1988, 1M)
(1998, 2M)
1 1 æ a ö
æp ö æ MA ö2 æM ö æ pA ö2 (a) (V - b ) (b) RT (c) ç p + 2 ÷ (d) ( RT )-1
(a) çç A ÷÷ çç ÷÷ (b) çç A ÷÷ çç ÷÷ è V ø
è pB ø è MB ø è MB ø è pB ø
1 1 35. The average velocity of an ideal gas molecule at 27°C is
æp ö æ MB ö2 æM ö æ pB ö2 0.3 m/s. The average velocity at 927°C will be (1986, 1M)
(c) çç A ÷÷ çç ÷÷ (d) çç A ÷÷ çç ÷÷
è pB ø è MA ø è MB ø è pA ø (a) 0.6 m/s (b) 0.3 m/s (c) 0.9 m/s (d) 3.0 m/s
25. The compressibility factor for an ideal gas is (1997, 1M) 36. Rate of diffusion of a gas is (1985, 1M)
(a) 1.5 (b) 1.0 (c) 2.0 (d) ¥ (a) directly proportional to its density
26. The ratio between the root mean square speed of H2 at 50 K (b) directly proportional to its molecular weight
and that of O2 at 800 K is (1996, 1M)
(c) directly proportional to the square root of its molecular weight
1 (d) inversely proportional to the square root of its
(a) 4 (b) 2 (c) 1 (d)
4 molecular weight
66 States of Matter

37. Equal weights of methane and hydrogen are mixed in an 45. A gas described by van der Waals’ equation (2008, 4M)
empty container at 25°C. The fraction of the total pressure (a) behaves similar to an ideal gas in the limit of large
exerted by hydrogen is (1984, 1M) molar volumes
1 8 1 16 (b) behaves similar to an ideal gas in the limit of large pressures
(a) (b) (c) (d)
2 9 9 17 (c) is characterised by van der Waals’ coefficients that are
38. When an ideal gas undergoes unrestrained expansion, no dependent on the identity of the gas but are
cooling occurs because the molecules (1984, 1M) independent of the temperature
(a) are above the inversion temperature (d) has the pressure that is lower than the pressure exerted
(b) exert no attractive forces on each other by the same gas behaving ideally
(c) do work equal to loss in kinetic energy 46. If a gas is expanded at constant temperature (1986, 1M)
(d) collide without loss of energy (a) the pressure decreases

w
39. Helium atom is two times heavier than a hydrogen molecule. (b) the kinetic energy of the molecules remains the same
At 298 K, the average kinetic energy of a helium atom is (c) the kinetic energy of the molecules decreases
(a) two times that of a hydrogen molecule (1982, 1M) (d) the number of molecules of the gas increases

Flo
(b) same as that of a hydrogen molecule
(c) four times that of a hydrogen molecule Numerical Value Based Question

ree
(d) half that of a hydrogen molecule 47. A closed tank has two compartments A and B, both filled
40. Equal weights of methane and oxygen are mixed in an empty with oxygen (assumed to be ideal gas). The partition
container at 25°C. The fraction of the total pressure exerted separating the two compartments is fixed and is a perfect heat

F
by oxygen is (1981, 1M)
insulator (Fig. 1). If the old partition is replaced by a new
1 1 2 1 273 partition which can slide and conduct heat but does not allow
(d) ´

or
(a) (b) (c) the gas to leak across (Fig. 2), the volume (in m3 ) of the
3 2
ur
3 3 298
compartment A after the system attains equilibrium is ____.

f
41. The temperature at which a real gas obeys the ideal gas laws
over a wide range of pressure is
ks
(1981, 1M)
(a) critical temperature (b) Boyle temperature 1 m3, 5 bar
3 m3, 1 bar, 300 K
Yo
400 K
(c) inversion temperature (d) reduced temperature B
oo
A
42. The ratio of root mean square velocity to average velocity of
eB

a gas molecule at a particular temperature is (1981, 1M) Fig. 1


(a) 1.085 : 1 (b) 1 : 1.086 (c) 2 : 1.086 (d) 1.086 : 2
r

A B
Objective Questions II
ou
ad

(One or more than one correct option)


Y

43. One mole of a monoatomic real gas satisfies the equation Fig. 2 (2018 Adv)
p(V - b)= RT where, b is a constant. The relationship of Assertion and Reason
interatomic potential V(r) and interatomic distance r for gas
nd

Read the following questions and answer as per the direction given
Re

is given by (2015 Adv.)


below :
(a) Statement I is correct; Statement II is correct;
Fi

V(r) V(r)
(a) 0 (b) 0
Statement II is the correct explanation of Statement I
r r (b) Statement I is correct; Statement II is correct; Statement
II is not the correct explanation of Statement I
V(r) V(r) (c) Statement I is correct; Statement II is incorrect
(d) Statement I is incorrect; Statement II is correct
(c) 0 r
(d) 0 r 48. Statement I The pressure of a fixed amount of an ideal gas is
proportional to its temperature.
44. According to kinetic theory of gases (2011) Statement II Frequency of collisions and their impact both
(a) collisions are always elastic increase in proportion to the square root of temperature.
(2000)
(b) heavier molecules transfer more momentum to the wall
of the container 49. Statement I The value of van der Waals’ constant ‘a’ is
(c) only a small number of molecules have very high velocity larger for ammonia than for nitrogen.
(d) between collisions, the molecules move in straight lines Statement II Hydrogen bonding is present in ammonia.
with constant velocities (1998)
States of Matter 67

Passage Based Questions True / False


Passage 59. A mixture of ideal gases is cooled up to liquid helium
X and Y are two L = 24 cm temperature (4.22 K) to form an ideal solution. (1996, 1M)
volatile liquids with æ n2a ö
molar weights of 60. In the van der Waals’ equation, çç p + 2 ÷÷ (V - nb ) = nRT
è V ø
10 g mol -1 and 40 g
the constant ‘a’ reflects the actual volume of the gas
mol -1 respectively. d Initial formation Cotton wool molecules. (1993, 1M)
Two cotton plugs, Cotton wool
of the product soaked in Y
soaked in X 61. A gas in a closed container will exert much higher pressure
one soaked in X and
due to gravity at the bottom than at the top. (1985, 1/2 M)
the other soaked in Y, are simultaneously placed at the ends of a
tube of length L = 24 cm, as shown in the figure. 62. Kinetic energy of a molecule is zero at 0°C. (1985, 1/2 M)

w
The tube is filled with an inert gas at 1 atm pressure and a
temperature of 300 K. Vapours of X and Y react to form a product Integer Answer Type Questions
which is first observed at a distance d cm from the plug soaked in X. 63. The diffusion coefficient of an ideal gas is proportional to its
Take X and Y to have equal molecular diameters and assume ideal

Flo
mean free path and mean speed. The absolute temperature of
behaviour for the inert gas and the two vapours. (2014 Adv.) an ideal gas is increased 4 times and its pressure is increased
50. The experimental value of d is found to be smaller than the 2 times. As a result, the diffusion coefficient of this gas

ree
estimate obtained using Graham’s law. This is due to increases x times. The value of x is ... (2016 Adv.)

(a) larger mean free path for X as a compared of that of Y 64. A closed vessel with rigid walls contains 1 mole of 238
92 U and

F
(b) larger mean free path for Y as compared to that of X 1 mole of air at 298 K. Considering complete decay of
238 206
(c) increased collision frequency of Y with the inert gas as 92 U to 82 Pb, the ratio of the final pressure to the initial

or
ur
compared to that of X with the inert gas pressure of the system at 298 K is (2015 Adv.)
(d) increased collision frequency of X with the inert gas as 65. If the value of Avogadro number is 6.023 ´ 10 23
mol -1 and

f
compared to that of Y with the inert gas the value of Boltzmann constant is 1.380 ´ 10 JK -1 , then
-23
ks
51. The value of d in cm (shown in the figure), as estimated from the number of significant digits in the calculated value of the
Yo
Graham’s law, is universal gas constant is (2014 Adv.)
oo
(a) 8 (b) 12 (c) 16 (d) 20 66. To an evacuated vessel with movable piston under external
pressure of 1 atm, 0.1 mole of He and 1.0 mole of an
eB

Match the Columns


unknown compound (vapour pressure 0.68 atm at 0°C ) are
52. Match the gases under specified conditions listed in introduced. Considering the ideal gas behaviour, the total
Column I with their properties/laws in Column II. volume (in litre) of the gases at 0°C is close to (2011)
r
ou

Column I Column II
ad

Subjective Questions
A. Hydrogen gas ( p = 200 atm, p. compressibility
Y

T = 273 K) factor ¹ 1 67. At 400 K, the root mean square (rms) speed of a gas X
(molecular weight = 40) is equal to the most probable speed of
B. Hydrogen gas ( p ~ 0, T = 273 K) q. attractive forces
gas Y at 60 K. The molecular weight of the gas Y is (2009)
nd

are dominant
Re

C. CO2 ( p = 1 atm, T = 273 K) r. pV = nRT 68. The average velocity of gas molecules is 400 m s - 1 , find the
rms velocity of the gas. (2003, 2M)
Fi

D. Real gas with very large molar s. p (V - nb ) = nRT 69. The density of the vapour of a substance at 1 atm pressure
volume
and 500 K is 0.36 kg m–3 . The vapour effuses through a small
(2007, 6M) hole at a rate of 1.33 times faster than oxygen under the same
Fill in the Blanks condition.
53. The absolute temperature of an ideal gas is …… to/than the (i) Determine, (a) molecular weight (b) molar volume
average kinetic energy of the gas molecules. (1997, 1M) (c) compression factor (Z) of the vapour and (d) which
54. 8 g each of oxygen and hydrogen at 27°C will have the total forces among the gas molecules are dominating, the
kinetic energy in the ratio of …… . (1989, 1M)
attractive or the repulsive?
(ii) If the vapour behaves ideally at 1000 K, determine the
55. The value of pV for 5.6 L of an ideal gas is ......... RT, at NTP.
(1987, 1M) average translational kinetic energy of a molecule.
(2002, 5M)
56. The rate of diffusion of a gas is .......... proportional to both
.......... and square root of molecular mass. (1986, 1M) 70. The compression factor (compressibility factor) for one mole
57. C p - CV for an ideal gas is …… of a van der Waals’ gas at 0° C and 100 atm pressure is found
(1984, 1M)
to be 0.5. Assuming that the volume of a gas molecule is
58. The total energy of one mole of an ideal monoatomic gas at negligible, calculate the van der Waals’ constant ‘a’.
27°C is …… cal. (1984, 1M) (2001, 5M)
68 States of Matter

71. Calculate the pressure exerted by one mole of CO2 gas at the root mean square (rms) speed and the temperature of the
273 K if the van der Waals’ constant a = 3.592 dm 6 atm gas molecules. If the ratio of the most probable speed to root
mol -2 . Assume that the volume occupied by CO2 molecules mean square speed is 0.82, calculate the most probable speed
is negligible. (2000) for these molecules at this temperature. (1993, 4M)

72. (i) One mole of nitrogen gas at 0.8 atm takes 38 s to diffuse 81. At room temperature, the following reaction proceed nearly
through a pin-hole, whereas one mole of an unknown to completion. 2NO + O2 ¾® 2NO2 ¾® N2 O4
compound of xenon with fluorine at 1.6 atm takes 57 s to The dimer, N2 O4 , solidifies at 262 K. A 250 mL flask and a
diffuse through the same hole. Calculate the molecular 100 mL flask are separated by a stopcock. At 300 K, the
formula of the compound. nitric oxide in the larger flask exerts a pressure of 1.053 atm
and the smaller one contains oxygen at 0.789 atm.
(ii) The pressure exerted by 12 g of an ideal gas at
temperature t °C in a vessel of volume V litre is one atm. The gases are mixed by opening the stopcock and after the

w
When the temperature is increased by 10°C at the same end of the reaction the flasks are cooled to 220 K. Neglecting
volume, the pressure increases by 10%. Calculate the the vapour pressure of the dimer, find out the pressure and
temperature t and volume V. composition of the gas remaining at 220 K. (Assume the

Flo
(Molecular weight of the gas = 120) (1999, 5M) gases to behave ideally). (1992, 4M)

73. Using van der Waals’ equation, calculate the constant a 82. At 27° C, hydrogen is leaked through a tiny hole into a vessel
when two moles of a gas confined in a four litre flask exert a for 20 min. Another unknown gas at the same temperature

ree
pressure of 11.0 atm at a temperature of 300 K. The value of and pressure as that of hydrogen is leaked through same hole
b is 0.05 L mol –1 . (1998, 4M) for 20 min. After the effusion of the gases the mixture exerts

F
74. An evacuated glass vessel weighs 50.0 g when empty 148.0 g a pressure of 6 atm. The hydrogen content of the mixture is
0.7 mole. If the volume of the container is 3 L. What is the
when filled with a liquid of density 0.98 g mL–1 and 50.5 g

or
molecular weight of the unknown gas?
ur
when filled with an ideal gas at 760 mm Hg at 300 K.
(1992, 3M)

Determine the molar mass of the gas. (1998, 3M) 83. Calculate the volume occupied by 5.0 g of acetylene gas at

f
50° C and 740 mm pressure. (1991, 2M)
75. A mixture of ideal gases is cooled up to liquid helium
ks
temperature (4.22 K) to form an ideal solution. Is this 84. The average velocity at T1 K and the most probable at T2 K of
Yo
statement true or false ? Justify your answer in not more than CO2 gas is 9.0 ´ 104 cm s –1 . Calculate the value of T1 and T2
oo
two lines. (1996, 1M)
(1990, 4M)
eB

76. The composition of the equilibrium mixture (Cl 2 s 2Cl)


85. A spherical balloon of 21 cm diameter is to be filled up with
which is attained at 1200° C, is determined by measuring the hydrogen at NTP from a cylinder containing the gas at
rate of effusion through a pin-hole. It is observed that at 20 atm at 27° C. If the cylinder can hold 2.82 L of water,
1.80 mm Hg pressure, the mixture effuses 1.16 times as fast
r

calculate the number of balloons that can be filled up.


ou

as krypton effuses under the same conditions. Calculate the


ad

(1987, 5M)
fraction of chlorine molecules dissociated into atoms (atomic 86. Calculate the root mean square velocity of ozone kept in a closed
Y

weight of Kr = 84) (1995, 4M)


vessel at 20° C and 82 cm mercury pressure. (1985, 2M)
77. A mixture of ethane (C2 H6 ) and ethene (C2 H4 ) occupies 40 L
87. Give reasons for the following in one or two sentences.
nd

at 1.00 atm and at 400 K. The mixture reacts completely with


Re

(i) Equal volumes of gases contain equal number of moles.


130 g of O2 to produce CO2 and H2 O. Assuming ideal gas
(1984, 1M)
behaviour, calculate the mole fractions of C2 H4 and C2 H6 in
Fi

(ii) A bottle of liquor ammonia should be cooled before


the mixture. (1995, 4M)
opening the stopper. (1983, 1M)
78. An LPG (liquefied petroleum gas) cylinder weighs 14.8 kg
88. Oxygen is present in one litre flask at a pressure of 7.6 ´ 10-10
when empty. When full it weighs 29.0 kg and shows a
mm Hg. Calculate the number of oxygen molecules in the
pressure of 2.5 atm. In the course of use at 27° C, the weight of
flask at 0°C. (1983, 2M)
the full cylinder reduces to 23.2 kg. Find out the volume of the
gas in cubic metres used up at the normal usage conditions, 89. When 2 g of a gas A is introduced into an evacuated flask
and the final pressure inside the cylinder. Assume LPG to the kept at 25°C, the pressure is found to be one atmosphere. If
n-butane with normal boiling point of 0° C. (1994, 3M) 3 g of another gas B is then added to the same flask, the total
89. A 4 : 1 molar mixture of He and CH4 is contained in a vessel pressure becomes 1.5 atm. Assuming ideal gas behaviour,
calculate the ratio of the molecular weights M A : M B .
at 20 bar pressure. Due to a hole in the vessel, the gas
(1983, 2M)
mixture leaks out. What is the composition of the mixture
effusing out initially? (1994, 2M)
90. At room temperature, ammonia gas at 1 atm pressure and
hydrogen chloride gas at p atm pressure are allowed to effuse
80. A gas bulb of 1 L capacity contains 2.0 ´ 1021 molecules of through identical pin holes from opposite ends of a glass tube
nitrogen exerting a pressure of 7.57 ´ 103 Nm–2 . Calculate of one metre length and of uniform cross-section.
States of Matter 69

Ammonium chloride is first formed at a distance of 60 cm 93. A hydrocarbon contains 10.5 g of carbon per gram of
from the end through which HCl gas is sent in. What is the hydrogen. 1 L of the vapour of the hydrocarbon at 127°C and
value of p ? (1982, 4M) 1 atm pressure weighs 2.8 g. Find the molecular formula of
91. Calculate the average kinetic energy, in joule per molecule in the hydrocarbon. (1980, 3M)

8.0 g of methane at 27°C. (1982, 2M) 94. 3.7 g of a gas at 25°C occupied the same volume as 0.184 g of
hydrogen at 17°C and at the same pressure. What is the
92. The pressure in a bulb dropped from 2000 to 1500 mm of
molecular weight of the gas ? (1979, 2M)
mercury in 47 min when the contained oxygen leaked
through a small hole. The bulb was then evacuated. A 95. 4.215 g of a metallic carbonate was heated in a hard glass
mixture of oxygen and another gas of molecular weight 79 in tube, the CO2 evolved was found to measure 1336 mL at
the molar ratio of 1 : 1 at a total pressure of 4000 mm of 27°C and 700 mm of Hg pressure. What is the equivalent
mercury was introduced. Find the molar ratio of the two weight of the metal ? (1979, 3M)

w
gases remaining in the bulb after a period of 74 min. 96. Calculate the density of NH3 at 30°C and 5 atm pressure.
(1981, 3M) (1978, 2M)

Topic 2 Liquid State

Flo
Objective Questions I (Only one correct option) (d) CH3OH KCl CH3 (CH2 )11 OSO-3 Na +

ree
1. The qualitative sketches I, II and III given below show the 2. At 100°C and 1 atm if the density of the liquid water is
variation of surface tension with molar concentration of three 1.0 g cm–3 and that of water vapour is 0.0006 g cm-3 , then the

F
different aqueous solutions of KCl, CH3 OH and
volume occupied by water molecules in 1 L of steam at this
CH3 (CH2 )11 OSO-3 Na + at room temperature. (2016 Adv.) temperature is (2000, 1M)

or
ur (a) 6 cm 3 (b) 60 cm 3
Surface tension

Surface tension

Surface tension

I II III

f
(c) 0.6 cm 3 (d) 0.06 cm 3
ks
3. The critical temperature of water is higher than that of O2
because the H2O molecule has
Yo
(1997)
oo
(a) fewer electrons than O2
Concentration Concentration Concentration
(b) two covalent bonds
eB

The correct assignment of the sketches is (c) V-shape


I II III (d) dipole moment
(a) KCl CH3OH CH3 (CH2 )11 OSO3- Na +
4. A liquid is in equilibrium with its vapour at it’s boiling point.
(b) CH3 (CH2 )11 OSO3- Na + CH3OH
r

KCl On the average, the molecules in the two phases have equal
ou
ad

(c) KCl CH3 (CH2 )11 OSO3- Na + CH3OH (a) inter-molecular forces (b) potential energy
(c) kinetic energy (d) total energy
Y

Answers
nd
Re

Topic 1
1. (c) 2. (b) 3. (a) 4. (a)
Fi

5. (c) 6. (b) 7. (b) 8. (a) 53. less 54. 1 : 16 55. 0.25


9. (b) 10. (b) 11. (b) 12. (c) 56. inversely, time 57. R 58. 900
13. (c) 14. (b) 15. (b) 16. (a) 59. F 60. F 61. F 62. F
17. (a) 18. (a) 19. (c) 20. (d) 63. (4) 64. (9) 65. (4) 66. (7 L)
21. (b) 22. (c) 23. (c) 24. (c) 67. (4) 68. 435 ms -1 70. (1025) 71. (0.99 atm)
25. (b) 26. (c) 27. (d) 28. (a) 73. (6.46) 74. (123 g mol -1) 76. (0.14)
29. (d) 30. (a) 31. (b) 32. (c) 78. (2.46 m 3) 79. (8:1) 80. (407 ms -1) 81. (0.221 atm)
82. (1020 g mol -1) 83. (5.23 L) 85. (10)
33. (b) 34. (c) 35. (a) 36. (d)
86. (390.2 ms -1) 88. (2.7 ´ 1010 g mol -1)
37. (b) 38. (b) 39. (b) 40. (a)
89. (1 : 3) 90. (2.20 atm) 91. (6.2 ´ 10 -21 J/molecule)
41. (b) 42. (a) 43. (c) 44. (a)
94. (41.32g) 95. (12.15) 96. (3.42 gL -1)
45. (a,c) 46. (a,b) 47. (2.22) 48. (d)
49. (a) 50. (d) 51. (c) Topic 2
52. A ® p,s; B ® r; C® p, q; D ® r 1. (d) 2. (c) 3. (d) 4. (c)
Hints & Solutions
Topic 1 Gaseous State pV - pb = RT Þ pV = RT + pb
1. pV pb
Key Idea From kinetic gas equation, =1+
RT RT
2RT
Most probable velocity (vmp ) = As, Z=
pV
M RT
where, R = gas constant, T = temperature, pb
so, Z =1+ Þ y = c + mx
M = molecular mass RT
The plot of z vs p is found to be

w
2RT T
vmp = , i.e. vmp µ
M M b
slope =
Gas M T ( K) T/M RT

Flo
Z
H2 2 300 300 / 2 = 150 …III (Highest)
N2 28 300 300 / 28 = 10.71 …I (Lowest)

ree
O2 32 400 400 / 32 = 12.5 … II p

F
So, The gas with high value of b will be steepest as slope is directly
I. corresponds to vmp of N2 (300 K) proportional to b. b is the van der Waals’ constant and is equal to
four times the actual volume of the gas molecules. Xe gas

or
ur
II. corresponds to vmp of O2 (400 K)
possess the largest atomic volume among the given noble gases
III. corresponds to vmp of H2 (300 K) (Ne, Kr, Ar). Hence, it gives the steepest increase in the plot of Z
2. For 1 mole of a real gas, the van der Waals’ equation is
f
(compression factor) vsp.
ks
æ a ö 4. Critical temperature is the temperature of a gas above which it
Yo
ç p + 2 ÷ (V - b) = RT cannot be liquefied what ever high the pressure may be. The
oo
è V ø
kinetic energy of gas molecules above this temperature is
The constant ‘a’ measures the intermolecular force of attraction sufficient enough to overcome the attractive forces. It is
eB

of gas molecules and the constant ‘b’ measures the volume represented as Tc.
correction by gas molecules after a perfectly inelastic binary 8a
collision of gas molecules. Tc =
27Rb
r

For gas A and gas C given value of ‘b’ is 8 ´ 13


.
0.05196 dm3 mol -1. Here, For Ar, Tc = = 0.0144
ou
ad

27 ´ 8.314 ´ 3.2
a µ intermolecular force of attraction
8 ´ 0.2
Y

µ compressibility µ real nature For Ne, Tc = = 0.0041


27 ´ 8.314 ´ 17
.
1
µ 8 ´ 51
.
For Kr, Tc = = 018
.
nd

volume occupied
Re

27 ´ 8.314 ´ 1.0
Value of a/(kPa dm6 mol -1) for gas A (642.32) > gas C (431.91) 8 ´ 4.1
For Xe, Tc = = 0.02
Fi

So, gas C will occupy more volume than gas A. Similarly, for a 27 ´ 8.314 ´ 5.0
given value of a say 155.21 kPa dm6 mol -1 for gas B and gas D
The value of Tc is highest for Kr (Krypton).
1
µ intermolecular force of attraction 5. In isothermal expansion, pVm = K (constant)
b
µ compressibility µ real nature This relation is plotted in graph ‘C’
1 K
µ Likewise, p=
volume accupied Vm
b/(dm 3 mol -1) for gas B (0.04136) < Gas D (0.4382) This relation is plotted in graph “A”.
So, gas B will be more compressible than gas D. Thus, graph B and D are incorrect.
For them the correct graphs are:
3. Noble gases such as Ne, Ar, Xe and Kr found to deviate from
ideal gas behaviour.
Xe gas will exhibit steepest increase in plot of Z vs p.
p for graph B and U for graph D
Equation of state is given as:
RT
p= Þ p(V - b) = RT
(V - b) Vm Vm
States of Matter 71

6. Given, temperature (T1 ) = 27° C = 273 + 27 = 300 K 10. Initially,


Volume of vessel = constant piV
Number of moles of gases in each container =
Pressure in vessel = constant RT1
2 3 piV
Volume of air reduced by so the remaining volume of air is . Total number of moles of gases in both containers = 2
5 5 RT1
Let at T1 the volume of air inside the vessel is n so at T2 the pf V
3 After mixing, number of moles in left chamber =
volume of air will be n. RT1
5 pf V
Now, as p and V are constant, so Number of moles in right chamber =
RT2
3
n ×T1 = n T2 ...(i) pf V pf V pf V æ 1 1ö
5 Total number of moles = + = çç + ÷÷

w
RT1 RT2 R è T1 T2 ø
Putting the value of T1 in equation (i) we get,
3 As total number of moles remains constant.
n ´ 300 = n ´T2
5 2 piV pf V pf V æ T2 ö
Hence, = + Þ pf = 2 pi çç ÷÷

Flo
or
5
T2 = 300 ´ = 500 K
RT1 RT1 RT2 è T1 + T2 ø
3
11. PLAN To solve this problem, the stepwise approach required, i.e.

ree
7. Given, ZA = 3ZB (i) Write the van der Waals’ equation, then apply the condition
pV that at low pressure, volume become high,
Compressibility factor (Z) = [for real gases]
V - b~
-V

F
nRT i.e.
On substituting in equation (i), we get (ii) Now calculate the value of compressibility factor (Z ).
pAVA 3p V [ Z = pV / RT ]

or
= B B
nARTA nB RTB
ur …(ii) According to van der Waals’ equation,
æ a ö

f
Also, it is given that ç p + 2 ÷ (V - b) = RT
è V ø
VA = 2VB , nA = nB and TA = TB
ks
\ Eq. (ii) becomes æ a ö
At low pressure, ç p + 2 ÷ V = RT
Yo
pA ´ 2VB 3 pBVB è V ø
oo
= a a
nB RTB nB RTB Þ pV + = RT or pV = RT -
eB

V V
Þ 2 pA = 3 pB
pV a
Divide both side by RT, =1-
8. 2NaHCO3 + H2C2O4 ¾® 2CO2 + Na 2C4O4 + H2O RT RTV
r

2 mol 1 mol 2 mol


2RT
Þ In the reaction, number of mole of CO2 produced. 12. C * = Most probable speed =
ou
ad

M
pV 1 bar ´ 0.25 ´ 10-3 L
n= =
RT 0.082 L atm K-1mol -1 ´ 298.15 K
Y

8RT
C = Average speed =
-5
= 1.02 ´ 10 mol pM
Weight of NaHCO3 C = Root square speed corrected as root means square speed,
nd

Number of mole of NaHCO3 =


Re

Molecular mass of NaHCO3 i.e.


. ´ 10-5 ´ 84 ´ 103 mg 3RT
Fi

\ wNaHCO3 = 102 rms = and as we know C* < C < C


M
= 0.856 mg
0.856 * 4 3
Þ NaHCO 3 % = ´ 100 = 8.56% C : C : C =1 : : = 1 : 1.128 : 1.225
10 p 2

9. From the ideal gas equation, NOTE


As no option correspond to root square speed, it is understood as
pV = SnRT ...(i)
misprint. It should be root mean square speed.
Given: p = 200 Pa, V = 10 m3 , T = 1000 K
13. The van der Waals’ equation of state is
nA = 0.5 moles, nB = x moles
æ n2a ö
On substituting the given values in equation (i), we get çp+ ÷ (V - nb) = nRT
ç V 2 ÷ø
200 ´ 10 = (nA + nB ) ´ R ´ 1000 è
200 ´ 10 For one mole and when b = 0, the above equation condenses to
0.5 + x =
R ´ 1000
æ a ö
1 2 2 1 4-R ç p + 2 ÷ V = RT
+x= = - = è V ø
2 R R 2 2R
72 States of Matter

a 14T (H2 )
Þ pV = RT - …(i) Þ 7=
V T (N2 )
1 Þ T (N2 ) = 2T (H2 ) i.e. T (H2 ) < T (N2 )
Eq. (i) is a straight equation between pV and whose slope is ‘
V
- a’. Equating with slope of the straight line given in the graph.
23. At high temperature and low pressure, the gas volume is
infinitely large and both intermolecular force as well as
20.1 - 21.6
-a= = - 1.5 molecular volume can be ignored. Under this condition
3- 2 postulates of kinetic theory applies appropriately and gas
Þ a = 1.5 approaches ideal behaviour.
14. In the van der Waals’ equation 24. Rate of effusion µ pi ; pi = Partial pressure of ith component
æ n aö
2 1
çp+ ÷ (V - nb) = nRT µ
ç V 2 ÷ø M

w
è
V
n2a 25. Compressibility factor (Z ) = =1
The additional factor in pressure, i.e. corrects for Videal
V2
intermolecular force while b corrects for molecular volume. Q For ideal gas V = Videal

Flo
3RT
15. Option (b) is incorrect statement because at high pressure slope 26. Expression of rms is, urms =
M
of the line will change from negative to positive.

ree
3R ´ 50
r(He) 16
16. = =2 :1 urms (H2 at 50 K) 2
r(CH4 ) 4 Þ =
urms (O2 at 800 K) 3R ´ 800

F
3 32
17. Kinetic energy (E ) = kT
2 50 32

or
3kT
ur = ´
2 800
=1
RMS speed (u) =

f
m
27. Let x g of each gas is mixed.
2E
ks
Þ u= x
Mole of ethane =
m 30
Yo
oo
x
18. Positive deviation corresponds to Z > 1 Mole of hydrogen =
2
pV pV
eB

Q Z= , for positive deviation, > 1. x


nRT nRT 2 15
Mole fraction of hydrogen = =
19. Option (b) and (d) are ruled out on the basis that at the initial x x 16
+
point of 273 K, 1 atm, for 1.0 mole volume must be 22.4 L, and 2 30
r
ou

it should increase with rise in temperature. Partial pressure of H2


ad

Þ = Mole fraction of hydrogen


Option (a) is ruled out on the basis that initial and final points are Total pressure
not connected by the ideal gas equation V µ T , i.e. V /T do not
Y

= 15 : 16
have the same value at the two points.
8RT
In option (c), at the initial point, the volume is 22.4 L as required 28. Average speed =
pM
nd

by ideal gas equation and (V /T ) have the same value at both


Re

initial and final points. i.e. at constant volume, for a fixed mass, increasing temperature
increases average speeds and molecules collide more frequently
Fi

3RT
20. Root mean square velocity (urms ) = to the wall of container leading to increase in gas pressure.
M
Also, p × M = dRT 29. The mean translational kinetic energy (Î) of an ideal gas is
3
Substituting for RT / M in urms expression gives, Î= k BT ; T = Absolute temperature, i.e. Î µ T
2
3p 1
urms = Þ urms µ rCH 4 MX
d d 30. =2= Þ M X = 64
rX 16
V
21. Compressibility factor (Z ) = < 1 (given) w
Vid 31. The ideal gas equation, pV = nRT = RT
M
Þ V < 22.4 L æ wö
Q Vid (1 mol ) = 22.4 L at STP Þ pM = ç ÷ RT = dRT (d = density)
èV ø
3RT pM
22. Root mean square speed urms = Þ d=
M RT
urms (H2 ) T (H2 ) 28 i.e. density will be greater at low temperature and high
Þ = 7= ´
urms (N2 ) 2 T (N2 ) pressure.
States of Matter 73

32. The ease of liquefication of a gas depends on their x


intermolecular force of attraction which in turn is measured in 32 1
Þ Mole fraction of oxygen = =
terms of van der Waals’ constant a. Hence, higher the value of a, x x 3
+
greater the intermolecular force of attraction, easier the 32 16
liquefication. According to law of partial pressure
In the present case, NH3 has highest a, can most easily be Partial pressure of oxygen ( pO 2 ) = Mole fraction ´ Total
liquefied. pressure
33. HCl will diffuse at slower rate than ammonia because rate of pO 2 1
1 Þ =
effusion µ . p 3
M
Therefore, ammonia will travel more distance than HCl in the 41. It is the Boyle temperature TB . At Boyle temperature, the first

w
same time interval and the two gas will first meet nearer to HCl virial coefficient ( B ) vanishes and real gas approaches ideal
end. behaviour.
a
34. In van der Waals’ equation of state TB =
Rb

Flo
æ a ö
ç p + 2 ÷ (V - b) = RT (For 1 mole) Here, a and b are van der Waals’ constants.
è V ø
42. The two types of speeds are defined as;

ree
The first factor ( p + a/V 2 ) correct for intermolecular force while
the second term (V - b) correct for molecular volume. 3RT
Root mean square speed (urms ) =
M

F
8RT
35. Expression for average velocity is uav = 8RT
pM Average speed (uav ) =
pM

or
ur
For the same gas but at different temperature
For the same gas, at a given temperature, M and T are same,
uavg (T1 ) 300 1

f
T therefore
= 1 = =
uavg (T2 ) T2 1200 2 urms 3RT 8RT
ks
= :
uav M pM
Þ uav (927° C) = 2 ´ uav (27° C) = 0.6 ms-1
Yo
oo
8
1 = 3: = 3 : 2.54 = 1.085 : 1
36. Rate of effusion µ , p
eB

M
43. Equation of state p(V - b) = RT indicates absence of
37. Let x grams of each hydrogen and methane are mixed,
intermolecular attraction or repulsion, hence interatomic
x
Moles of H2 = potential remains constant on increasing ‘p’ in the beginning. As
r

2 the molecules come very close, their electronic and nuclear


ou
ad

x repulsion increases abruptly.


Moles of CH4 =
16
Y

x 44. (a) According to a postulate of kinetic theory of gases, collision


8 between the molecules as well as with the wall of container
Þ Mole fraction of H2 = 2 = is perfectly elastic in nature.
nd

x x 9
+
Re

2 16 (b) If a gas molecule of mass m moving with speed u collide to


Partial pressure of H2 8 the wall of container, the change in momentum is
Fi

Þ = Mole fraction of H2 = Dp = – 2mu. Therefore, heavier molecule will transfer more


Total pressure 9
momentum to the wall as there will be greater change in
38. According to postulates of kinetic theory, there is no momentum of the colliding gas molecule. However, this is
intermolecular attractions or repulsions between the molecules not postulated in kinetic theory.
of ideal gases. (c) According to Maxwell-Boltzmann distribution of molecular
3 speed, very few molecules have either very high or very low
39. According to kinetic theory, average kinetic energy (E ) = k BT speeds. Most of the molecules moves in a specific,
2
intermediate speed range.
where, k B is Boltzmann’s constant. Since, it is independent of
(d) According to kinetic theory of gases, a gas molecule moves
molar mass, it will be same for He and H2 at a given temperature.
in straight line unless it collide with another molecule or to
40. If x g of both oxygen and methane are mixed then : the wall of container and change in momentum is observed
x only after collision.
Mole of oxygen =
32 45. Option (a) is correct because in the limit of large volume, both
x intermolecular force and molecular volume becomes negligible
Mole of methane =
16 in comparison to volume of gas.
74 States of Matter

Option (b) is wrong statement because in the limit of large 51. PLAN This problem can be solved by using the concept of Graham’s law
pressure Z > 1. of diffusion according to which rate of diffusion of non-reactive
gases under similar conditions of temperature and pressure are
Option (c) is correct statement. For a van der Waals’ gas,
inversely proportional to square root of their density.
van der Waals’ constants a and b are characteristic of a gas, 1
independent of temperature. Rate of diffusion µ
molar weight of gas
Option (d) is wrong statement because Z can be either less or
Let distance covered by X is d, then distance covered by Y is
greater than unity, hence real pressure can be less or greater than
24 – d.
ideal pressure.
If rX and rY are the rate of diffusion of gases X and Y,
46. Pressure is inversely proportional to volume at constant rX d 40
temperature, hence (a) is correct. = = =2
rY 24 - d 10
Average kinetic energy of a gas is directly proportional to
[Q Rate of diffusion µ distance travelled]

w
absolute temperature, hence (b) is correct.
Expansion at constant temperature cannot change the number of d = 48 - 2d
molecules, hence (d) is incorrect. Þ 3d = 48 Þ d = 16 cm
Hence, (c) is the correct choice.
47. Given p1 = 5 bar, V1 = 1 m 3, T1 = 400 K

Flo
So, n1 =
5
(from pV = nRT ) 52. A. At p = 200 atm, very high pressure, Z > 1. Also, at such a
400 R æ n2a ö
high pressure, the pressure correction factor çç 2 ÷÷ can be

ree
3 èV ø
Similarly, p2 = 1 bar, V2 = 3 m 3, T2 = 300 K, n2 =
300 R ignored in comparison to p.

F
Let at equilibrium the new volume of A will be (1+ x ) B. At P ~ 0, gas will behave like an ideal gas, pV = nRT .
So, the new volume of B will be (3- x ) C. CO2 (p = 1 atm, T = 273 K), Z < 1.

or
Now, from the ideal gas equation.
ur D. At very large molar volume, real gas behaves like an
p1V1 pV ideal gas.

f
= 2 2
n1RT1 n2RT2 3
53. Less; E = RT
ks
and at equilibrium (due to conduction of heat) 2
Yo
p1 p2 3
54. 1 : 16, KE = nRT . At same temperature, KE (total) µ n.
oo
=
T1 T2 2
1
eB

V1 V2 55. 0.25 RT because at NTP, 5.6 L = mole.


So, = or V1n2 = V2n1 4
n1 n2
56. Inversely, time.
After putting the values
57. For an ideal gas, Cp - CV = R
r

3 5 (3 - x ) 5
(1+ x ) ´ = (3 - x ) ´ or (1+ x ) =
ou
ad

300 R 400 R 4 3 3
58. At 27°C, E = RT = ´ 2 ´ 300 = 900 cal
11 2 2
Y

or 4 (1+ x ) = 15 - 5x or 4 + 4 x = 15 - 5x or x =
9
59. An ideal gas cannot be liquefied because there exist no
11 20
Hence, new volume of A i.e., (1+ x ) will comes as 1 + = or intermolecular attraction between the molecules of ideal gas.
nd
Re

9 9
2.22. 60. a is the measure of intermolecular force.
Fi

48. Assertion is incorrect because besides amount, pressure also 61. In a close container, gas exert uniform pressure everywhere in
depends on volume. However, reason is correct because both the container.
frequency of collisions and impact are directly proportional to 3
62. KE = RT where, T is absolute temperature (in Kelvin).
root mean square speed which is proportional to square root of 2
absolute temperature
63. (DC) Diffusion coefficient µ l (mean free path) µU mean
49. a is the measure of intermolecular force of attraction. Greater the
intermolecular force of attraction (H-bond in the present case) Thus (DC) µ l Umean
higher the value of a. RT T
But, l= Þ lµ
50. X is a lighter gas than Y, hence X has greater molecular speed. 2 N0 sp p
Due to greater molecular speed of X, it will have smaller mean 8RT
free path and greater collision frequency with the incrt gas and U mean =
pM
molecules. As a result X will take more time to travel a given
distance along a straight line. Hence X and Y will meet at a U mean µ T
distance smaller than one calculated from Graham’s law. (T )3/ 2
\ DC µ
Hence, (d) is the correct choice. p
States of Matter 75

æp ö æ T2 ö
3/ 2
æ p ö æ 4T1 ö
3/ 2 pV 1 ´ 50
(DC)2
(x ) = çç 1 ÷÷ çç ÷÷ = çç 1 ÷÷ çç ÷÷ (c) Z = = = 1.22
(DC)1 RT 0.082 ´ 500
è p2 ø è T1 ø è 2 p1 ø è T1 ø
(d) Q Z > 1, repulsive force is dominating.
æ 1ö
= ç ÷ (8) = 4
è 2ø 3 3
(ii) Ek = k BT = ´ 1.38 ´ 10-23 ´ 1000 J = 2.07 ´ 10-20 J
0 2 2
64. 92 U
238
¾® 82Pb
206
+ 8 2He (g ) + 6- 1b
4

n(gas)[Initial] = 1 (air) 70. In case of negligible molecular volume, b = 0. For 1 mole of gas
n(gas)[Final] = 8 (He) + 1(air) = 9 æ a ö
ç p + 2 ÷ V = RT
Þ At constant temperature and volume; è V ø
p µ n. Þ pV +
a
= RT
pf nf 9 V

w
So, = = =9
ni 1 pV a é pV ù
pi Þ + =1 = Zú
ëê RT
Q
RT VRT û
65. PLAN This problem can be solved by using the concept involved in
a ap
calculation of significant figure. Þ Z+ =1 Þ Z+ =1

Flo
Universal gas constant, R = kN A æ ZRT ö ZR 2T 2
çç ÷÷ RT
where, k = Boltzmann constant è p ø

ree
and N A = Avogadro’s number ZR 2T 2 (1 - Z ) 0.5 (0.082 ´ 273)2 (1- 0.5)
Þ a= =
p 100
\ R = 1.380 ´ 10-23 ´ 6.023 ´ 1023 J/Kmol

F
= 8.31174 =~ 8.312 a = 1.25 atm L2 mol -2
Since, k and N A both have four significant figures, so the value 71. In case of negligible molecular volume, b = 0 and

or
ur
of R is also rounded off upto 4 significant figures.
van der Waals’ equation reduces to

f
[When number is rounded off, the number of significant figure is
æ n2a ö
reduced, the last digit is increased by 1 if following digits ³ 5 ç p+ ÷ V = nRT
ç V 2 ÷ø
ks
and is left as such if following digits is £ 4.] è
Yo
Hence, correct integer is (4). RT a
Þ p= - 2 (n =1 mole)
oo
66. Since, the external pressure is 1.0 atm, the gas pressure is also V V
1.0 atm as piston is movable. Out of this 1.0 atm partial pressure 0.082 ´ 273 3.592
eB

= - = 0.99 atm
due to unknown compound is 0.68 atm. 22.4 (22.4)2
Therefore, partial pressure of He
72. (i) For the same amount of gas being effused
= 1.00 – 0.68 = 0.32 atm.
r

n(He)RT 0.1 ´ 0.082 ´ 273 r1 t2 p1 M2


ou

Þ Volume = = =7L = =
ad

p(He) 0.32 r2 t1 p2 M1
Y

Þ Volume of container = Volume of He. 57 0.8 M2


Þ =
38 1.6 28
67. Given, urms = umps
Þ M 2 = 252 g mol -1
nd
Re

3RT 2RT
Þ = Also, one molecule of unknown xenon-fluoride contain only
M (X ) M (Y )
Fi

one Xe atom [M (Xe) = 131], formula of the unknown gas


3R ´ 400 2R ´ 60 can be considered to be XeFn.
Þ =
40 M (Y ) Þ 131 + 19n = 252; n = 6.3, hence the unknown gas is XeF6.
Þ M (Y ) = 4 (ii) For a fixed amount and volume, p µ T
8RT 3RT 8 1 T
68.
uav
= : = Þ = where, T = Kelvin temperature
urms pM M 3p 1.1 T + 10

3p 3 ´ 3.14 Þ T = 100 K = t + 273


Þ urms = uav = ´ 400 = 434 ms-1 Þ t = - 173° C
8 8
nRT æ 12 ö 0.082 ´ 100
rgas 32 Volume = =ç ÷´ = 0.82 L
69. = 1.33 = p è 120 ø 1
rO 2 Mgas
73. The van der Waals’ equation is
(i) (a) Mgas = 18 g mol -1
æ n2a ö
çp+ ÷ (V - nb) = nRT
(b) Vm =
18
= 50 L mol -1 ç V 2 ÷ø
è
0.36
76 States of Matter

V 2 é nRT ù (4 )2 é 2 ´ 0.082 ´ 300 ù Also, pressure of gas outside the cylinder is 1.0 atm.
Þ a= 2 ê
- pú = 2 ê
- 11ú
n ë V - nb û (2) ë 4 - 2 (0.05) û Þ pV = nRT
= 6.46 atm L2 mol -2 nRT (14.2 - 8.4 ) ´ 103 0.082 ´ 30
Þ V = = ´ L
p 58 1
74. Mass of liquid = 148 - 50 = 98 g
= 2460 L = 2.46 m 3
98
Þ Volume of liquid = = 100 mL = volume of flask
0.98 rHe nHe M CH 4 4 16
79. = = =8
mass of gas = 50.5 - 50 = 0.50 g rCH4 n CH4 M He 1 4
æ wö
Now applying ideal gas equation : pV = ç ÷ RT Initial ratio of rates of effusion gives the initial composition of
èMø mixture effusing out. Therefore, n (He) : n (CH4 ) = 8 : 1
wRT 0.5 ´ 0.082 ´ 300
Þ M = = = 123 g mol -1 2 ´1021
80. Number of moles = = 0.33 ´ 10-2

w
pV 1 ´ 0.1
6 ´ 1023
75. False, ideal gas cannot be liquefied as there is no intermolecular p = 7.57 ´ 103 Nm -2
attraction between the molecules of ideal gas. Hence, there is no
Now, pV = nRT

Flo
point of forming ideal solution by cooling ideal gas mixture.
pV 7.57 ´ 103 ´ 10-3
76. If ‘a’ is the degree of dissociation, then at equilibrium Þ T = = = 276 K
Cl 2 r 2Cl nR 0.33 ´ 10-2 ´ 8.314

ree
Moles 1-a 2a Total = 1 + a 3RT 3 ´ 8.314 ´ 276
Þ urms = = m s-1 = 496 ms-1
From diffusion information
M 28 ´ 10-3

F
r(mix) 84 umps
= 1.16 = Also, = 0.82
r(Kr) M (mix) urms

or
umps = 0.82 ´ urms = 0.82 ´ 496 ms-1 = 407 ms-1
Þ M (mix) = 62.4
ur Þ

f
71 81. First we calculate partial pressure of NO and O2 in the combined
Þ M (mix) = = 62.4
1+ a system when no reaction taken place.
ks
Þ a = 0.14 pV = constant Þ p1V1 = p2V2
Yo
1.053 ´ 250
oo
pV 1 ´ 40 Þ p2 (NO) = = 0.752 atm
77. The total moles of gaseous mixture = = 350
RT 0.082 ´ 400
0.789 ´ 100
eB

= 1.22 p2 (O2 ) = = 0.225 atm


350
Let the mixture contain x mole of ethane. Therefore,
7 Now the reaction stoichiometry can be worked out using partial
C2H6 + O2 ¾® 2CO2 + 3H2O pressure because in a mixture.
r

x 2 pi µ ni
ou
ad

C2H4 + 3O2 ¾® 2CO2 + 2H2O 2NO + O2 ¾® 2NO2 ¾® N2O4


1.22- x
Y

Initial 0.752 atm 0.225 atm 0 0


7 x
Total moles of O2 required = x + 3 (1.22 - x ) = + 3.66 Final 0.302 0 0 0.225 atm
2 2
130 x Now, on cooling to 220 K, N2O4 will solidify and only
nd

Þ = + 3.66
Re

unreacted NO will be remaining in the flask.


32 2
Q p µT
Þ x = 0.805 mole ethane and 0.415 mole ethene.
Fi

p1 T1
0.805 \ =
Þ Mole fraction of ethane = = 0.66 p2 T2
1.22
0.302 300
Mole fraction of ethene = 1 - 0.66 = 0.34 Þ =
p2 220
78. Weight of butane gas in filled cylinder = 29 - 14.8 kg
Þ p2 (NO) = 0.221 atm
= 14.2 kg
Þ During the course of use, weight of cylinder reduces to 23.2 kg pV 6´3
82. Total moles of gas in final mixture = = = 0.731
RT 0.082 ´ 300
Þ Weight of butane gas remaining now
= 23.2 - 14.8 = 8.4 kg Q Mole of H2 in the mixture = 0.70
Also, during use, V (cylinder) and T remains same. \ Mole of unknown gas ( X ) = 0.031
p1 n1
Therefore, = Because both gases have been diffused for same time
p2 n2
r (H2 ) 0.70 M
æn ö æ 8.4 ö é n w ù = =
Þ p2 = çç 2 ÷÷ p1 = ç ÷ ´ 2.5 ê Here, 2 = 2 ú r ( X ) 0.031 2
è 1ø
n è 14.2 ø ë n1 w1 û
Þ M = 1020 g mol -1
= 1.48 atm
States of Matter 77

nRT pV
83. V = 88. Number of moles (n) =
p RT
5 N (Number of molecules)
For acetylene gas, 5 g = mol and n=
26 N A (Avogadro number)
740 æ pV ö
p = 740 mm = atm Þ N = nNA = ç ÷ NA
760 è RT ø
æ 7.6 ´ 10-10 1 ö
T = 50° C = 323 K = çç ´ ÷ ´ 6.023 ´ 1023
760 0.082 ´ 273 ÷
Substituting in ideal gas equation è ø
10
5 0.082 ´ 323 = 2.7 ´ 10 molecules
V = ´ ´ 76 = 5.23 L
26 74 89. From the given information, it can be easily deduced that in the

w
8RT1 final mixture,
84. uav (average velocity ) = partial pressure of A = 1.0 atm
pM
partial pressure of B = 0.5 atm
9 ´ 104 8 ´ 8.314 T1
Þ ms-1 = pV V
nA = A =

Flo
100 3.14 ´ 44 ´ 10-3 Also
RT RT
Þ T1 = 1682.5 K p V 0.5 V
nB = B =

ree
Also, for the same gas RT RT
uav
=
8RT1 2RT2
=
8T1
´
1
=
4T1 nB 1 wB M A 3 æ M A ö
: Þ = = ´ = ´ç ÷
pM p 2T2 pT2 nA 2 M B wA 2 çè M B ÷ø

F
umps M
4T1 Þ M A :M B = 1 : 3
Þ 1=
pT2

or
ur 90. Rate of effusion (r) µ
p
4T1 4 ´ 1682.5 M
Þ T2 = = = 2142 K

f
p 3.14 r (NH3 ) 1 36.5 40 1 36.5
Þ = ´ Þ =
ks
Hence, T1 = 1682.5 K, T2 = 2142 K r (HCl ) 17 p 60 p 17
Yo
3
4 3 4 æ 21ö 3 36.5
oo
85. Volume of balloon = pr = ´ 3.14 ´ ç ÷ cm 3 Þ p= = 2.20 atm
3 3 è 2ø 2 17
eB

= 4847 cm 3 » 4.85 L 3
91. KE = k BT : k B = Boltzmann’s constant
Now, when volume of H2 (g ) in cylinder is converted into NTP 2
3
volume, then = ´ 1.38 ´ 10-23 ´ 300 J = 6.21 ´ 10-21 J/molecule
2
r

p1V1 p2V2
= dp kp
ou

92. Rate of effusion is expressed as - =


ad

T1 T2
dt M
20 ´ 2.82 1 ´ V2
Þ = , V2 = NTP volume k = constant, p = instantaneous pressure
Y

300 273 dp k dt
Þ V2 = 51.324 L Þ - =
p M
nd
Re

Also, the cylinder will not empty completely, it will hold 2.82 L æp ö kt
Integration of above equation gives ln çç 0 ÷÷ =
of H2 (g ) when equilibrium with balloon will be established.
è ø
p M
Fi

Hence, available volume of H2 (g ) for filling into balloon is


æ 2000 ö k 47
51.324 - 2.82 = 48.504 L Using first information : ln ç ÷=
è 1500 ø 32
48.504
Þ Number of balloons that can be filled = = 10 32 æ4ö
4.85 Þ k= ln ç ÷ …(i)
47 è 3ø
3RT 3 ´ 8.314 ´ 293
86. urms = = = 390.2 ms-1 Now in mixture, initially gases are taken in equal mole ratio,
M 48 ´ 10-3 hence they have same initial partial pressure of 2000 mm of Hg
each.
87. (i) NH3 (l ) is highly volatile, a closed bottle of NH3 (l ) contains
large number of molecules in vapour phase maintaining high After 74 min :
pressure inside the bottle. When the bottle is opened, there is æ 2000 ö 74 k
chances of bumping of stopper. To avoid bumping, bottle For O2 ln ç ÷=
ç pO ÷ 32
should be cooled that lowers the pressure inside. è 2 ø
(ii) According to Avogadro’s hypothesis, “Under identical Substituting k from Eq. (i) gives
conditions of pressure and temperature, equal volume of
æ 2000 ö
ideal gases contain equal number of molecules.” ln ç ÷ = 74 ´ 32 ln æç 4 ö÷
ç pO ÷ 32 47 è 3ø
è 2 ø
78 States of Matter

æ 2000 ö 74 æ4ö Q 0.05 mole MCO3 = 4.21 5 g


ln ç ÷=
ç pO ÷ 47 çè 3 ÷ø
ln 4.215
è 2 ø \ 1.0 mole MCO3 = = 84.3 g (molar mass)
0.05
Solving gives p (O2 ) at 74 min = 1271.5 mm Þ 84.3 = MW of M + 12 + 48
æ 2000 ö 74 k Þ Molecular weight of metal = 24.3
For unknown gas : ln ç ÷=
ç pg ÷ 79 Metal is bivalent, equivalent weight
è ø Q
Molecular weight
Substituting k from (i) gives = = 12.15
æ 2000 ö 2
ln ç ÷ = 74 ´ 32 ln æç 4 ö÷
ç pg ÷ 79 47 è 3ø 96. The ideal gas equation :
è ø w
Solving gives : pg = 1500 mm pV = nRT = RT
M
Þ After 74 min, p (O2 ) : p (g ) = 1271.5 : 1500

w
w
Also, in a mixture, partial pressure µ number of moles Þ pM = RT = d RT where, ‘d’ is density.
V
Þ n (O2 ) : n (g ) = 1 : 1.18 pM 5 ´ 17
Þ d= = = 3.42 g L-1.
0.082 ´ 303

Flo
93. First we determine empirical formula as RT
C H
Topic 2 Liquid State

ree
Weight 10.5 1
1. I (CH3OH) : Surface tension decreases as concentration
10.5
Mole = 0.875 1 increases.

F
12 II (KCl) : Surface tension increases with concentration for ionic
Simple ratio 1 1/0.875 = 1.14 salt.

or
III [CH3 (CH2 )11 OSO-3 Na + ] : It is an anionic detergent.
Whole no. 7
ur 8
There is decrease in surface tension before micelle formation,

f
Þ Empirical formula = C7H8 and after CMC (Critical Micelle Concentration) is attained, no
æ wö
ks
From gas equation : pV = ç ÷ RT change in surface tension.
èMø
Yo
oo
wRT 2.8 ´ 0.082 ´ 400 KCl (II)
Surface tension

M = = = 91.84 » 92
pV 1´ 1
eB

CH3OH (I)
Q Molar mass (M ) is same as empirical formula weight. -
CH3(CH2)11OSO3 Na + (III)
Molecular formula = Empirical formula = C7 H8
r

1
94. For same p and V , n µ
ou

Concentration
ad

T
n (gas) T (H2 ) 2. Let us consider, 1.0 L of liquid water is converted into steam .
Þ =
Y

n (H2 ) T (gas) Volume of H2O (l) = 1 L, mass = 1000 g


0.184 1000
n(H2 ) = = 0.092 Þ Volume of 1000 g steam = cm 3
nd

0.0006
Re

2
290 1000
Þ n(gas) = ´ 0.092 = 0.0895 Q Volume of molecules in cm 3 steam = 1000 cm 3
0.0006
Fi

298
Q 0.0895 mole of gas weigh 3.7 g \ Volume of molecules in
1000
3.7 1000 cm 3 steam = ´ 0.0006 ´ 1000 = 0.60 cm 3
\ 1 mole of gas will weigh = 41.32 g 1000
0.0895
3. Critical temperature is directly proportional to intermolecular
95. Moles of CO2 can be calculated using ideal gas equation as : force of attraction. H2O is a polar molecule, has greater
pV æ 700 ö æ 1336 ö 1 intermolecular force of attraction than O2, hence higher critical
n= =ç ÷ç ÷´ = 0.05 temperature.
RT è 760 ø è 1000 ø 0.082 ´ 300
Also, the decomposition reaction is : 4. At liquid-vapour equilibrium at boiling point, molecules in two
phase posses the same kinetic energy.
MCO3 ¾® MO + CO2
0.05 mol 0.05 mol

Download Chapter Test


http://tinyurl.com/y5zm482e or
6
Chemical and Ionic Equilibrium

w
Topic 1 Chemical Equilibrium

Flo
Objective Questions I (Only one correct option) 6. Two solids dissociate as follows:
1. The incorrect match in the following is (2019 Main, 12 April II) A (s ) - B (g ) + C(g ); K p1 = x atm2

ree
(a) DG ° < 0, K > 1 (b) DG ° = 0, K = 1 D ( s ) - C( g ) + E ( g ); K = y atm2
p
(c) DG ° > 0, K < 1 (d) DG ° < 0, K < 1 2
The total pressure when both the solids dissociate

F
2. In which one of the following equilibria, K p ¹ K c ? simultaneously is (2019 Main, 12 Jan I)
(2019 Main, 12 April II)
(a) x + y atm (b) x2 + y2 atm

or
(a) 2C( s) + O2 ( g )
c 2CO( g )
ur
(b) 2HI( g ) c H2 ( g ) + I2 ( g ) (c) ( x + y ) atm (d) 2( x + y ) atm
(c) NO2 ( g ) + SO2 ( g )
c NO( g ) + SO3 ( g )
f
7. Consider the reaction,
ks
(d) 2NO( g ) c N2 ( g ) + O2 ( g ) N 2 ( g ) + 3H 2 ( g ) =2NH (g )
Yo
3
3. For the reaction,
oo
The equilibrium constant of the above reaction is K p . If
2SO2 ( g ) + O2 ( g ) ¾® 2SO3 ( g ), DH = - 57.2 kJ mol -1 and pure ammonia is left to dissociate, the partial pressure
eB

K c = 1.7 ´ 1016 . Which of the following statement is incorrect? of ammonia at equilibrium is given by (Assume that
pNH 3 < < p total at equilibrium) (2019 Main, 11 Jan I)
(2019 Main, 10 April II)
(a) The equilibrium constant decreases as the temperature 33 / 2 K 1p/ 2 P 2 33 / 2 K 1p/ 2 P 2
r

(a) (b)
ou

increases 4 16
ad

(b) The addition of inert gas at constant volume will not affect 1/ 2
K p P2
1/ 2
K p P2
Y

the equilibrium constant (c) (d)


16 4
(c) The equilibrium will shift in forward direction as the pressure
increases 8. 5.1 g NH4 SH is introduced in 3.0 L evacuated flask at
nd
Re

(d) The equilibrium constant is large suggestive of reaction 327°C. 30% of the solid NH4 SH decomposed to NH3
going to completion and so no catalyst is required and H2 S as gases. The K p of the reaction at 327° C is
Fi

(R = 0.082 atm mol -1 K -1 , molar mass of S = 32 g mol -1 ,


4. For the following reactions, equilibrium constants are given :
- SO (g ); K = 10
S( s ) + O2 ( g ) 52 molar mass of N = 14 g mol -1 ) (2019 Main, 10 Jan II)
2 1
-4 2
2S( s ) + 3O ( g ) - 2SO ( g ); K = 10 129 (a) 0. 242 ´ 10 atm (b) 0. 242 atm 2
2 3 2
-3
The equilibrium constant for the reaction, (c) 4 . 9 ´ 10 atm 2
(d) 1 ´ 10-4 atm 2
2SO( g ) + O ( g ) - 2SO ( g ) is
2 3 (2019 Main, 8 April II) Kp
25 77 154 9. The values of for the following reactions at 300 K
(a) 10 (b) 10 (c) 10 (d) 10181 KC
are, respectively (At 300 K, RT = 24.62 dm 3 atm mol -1 )
K
5. In a chemical reaction, A + 2B - 2C + D,
the initial
concentration of B was 1.5 times of the concentration of A, but the N 2 ( g ) + O2 ( g )
=2NO(g )
equilibrium concentrations of A and B were found to be equal. The N 2O4 ( g )
=2NO (g ) 2
equilibrium constant (K ) for the aforesaid chemical reaction is
(2019 Main, 12 Jan I) N ( g ) + 3H ( g ) =2NH ( g )
2 2 3
1 (2019 Main, 10 Jan I)
(a) (b) 16 (c) 1 (d) 4
4
80 Chemical and Ionic Equilibrium

(a) 1, 24.62 dm 3 atm mol -1 , 606.0 dm 6 atm 2 mol -2 16. N2 + 3H2 r 2NH3
(b) 1, 24.62 dm 3 atm mol -1 , 1.65 ´ 10-3 dm -6 atm -2 mol 2 Which is correct statement if N2 is added at equilibrium
condition? (2006, 3M)
(c) 24.62 dm 3 atm mol -1 , 606.0 dm 6 atm -2 mol 2 ,
(a) The equilibrium will shift to forward direction because
1.65 ´ 10-3 dm-6 atm -2 mol 2
according to IInd law of thermodynamics, the entropy
(d) 1, 4.1 ´ 10-2 dm -3 atm -1 mol, 606 dm 6 atm 2 mol -2
must increases in the direction of spontaneous reaction
10 Consider the following reversible chemical reactions, (b) The condition for equilibrium is G(N2) + 3G(H2)
K1
A2 ( g ) + B2 ( g ) -2 AB (g ) …(i) = 2G(NH3) where, G is Gibbs free energy per mole of the
K2 gaseous species measured at that partial pressure. The
6 AB ( g ) - 3 A ( g ) + 3B ( g )
2 2 …(ii) condition of equilibrium is unaffected by the use of

w
The relation between K 1 and K 2 is (2019 Main, 9 Jan II)
catalyst, which increases the rate of both the forward and
backward reactions to the same extent
(a) K 2 = K 13 (b) K 1 K 2 = 3
(c) The catalyst will increase the rate of forward reaction by
1
(c) K 2 = K 1- 3 (d) K 1 K 2 = a and that of backward reaction by b

Flo
3
(d) Catalyst will not alter the rate of either of the reaction
11. An aqueous solution contains 0.10 M H2S and 0.20 M HCl. If
17. Ag + + NH3 s [Ag(NH3 )]+ ; K 1 = 3. 5 ´ 10-3

ree
the equilibrium constants for the formation of HS- from H2S
[Ag (NH3 )]+ + NH3 s [Ag (NH3 )2 ]+ ; K 2 = 1.7 ´10-3
. ´ 10-7 and that of S2- from HS- ions is 12
is 10 . ´ 10-13 then

F
the concentration of S2- ions in aqueous solution is : then the formation constant of [Ag(NH3 )2 ]+ is (2006, 3M)
(a) 6.08 ´ 10-6
(2018 Main)
(b) 6.08 ´ 106
(a) 5 ´ 10-8 (b) 3 ´ 10-20

or
-21
ur (c) 6.08 ´ 10-9 (d) None of these
(c) 6 ´ 10 (d) 5 ´ 10-19

f
18. Consider the following equilibrium in a closed container
12. The equilibrium constant at 298 K for a reaction,
ks
A + B q C + D is 100. If the initial concentrations of all N2 O4 ( g ) r 2NO2 ( g )
Yo
the four species were 1 M each, then equilibrium At a fixed temperature, the volume of the reaction container
oo
concentration of D (in mol L-1 ) will be (2016 Main) is halved. For this change, which of the following statements
eB

(a) 0.818 (b) 1.818 hold true regarding the equilibrium constant ( K p ) and
(c) 1.182 (d) 0.182 degree of dissociation ( a ) ? (2002, 3M)
13. The standard Gibbs energy change at 300 K for the reaction, (a) Neither K p nor a changes
r

2A a B + C is 2494. 2 J. At a given time, the composition


(b) Both K p and a change
ou

1 1
ad

of the reaction mixture is [A]= , [ B ] = 2 and [C ] = . The


2 2 (c) K p changes but a does not change
Y

reaction proceeds in the (d) K p does not change but a changes


(R = 8.314JK / mol, e = 2.718) (2015, Main)
nd

(a) forward direction because Q > K c 19. At constant temperature, the equilibrium constant ( K p ) for
Re

(b) reverse direction because Q > K c the decomposition reaction, N2 O4 r 2NO2 , is expressed
4 x2 p
Fi

(c) forward direction because Q < K c by K p = , where, p = pressure, x = extent of


(d) reverse direction because Q < K c (1 - x2 )
1 decomposition. Which one of the following statement is
14. For the reaction, SO2 ( g ) + O2 ( g ) q SO3 ( g ) true? (2001, 1M)
2
if K p = K C ( RT )x where, the symbols have usual meaning, (a) K p increases with increase of p
(b) K p increases with increase of x
then the value of x is (assuming ideality) (2014 Main)
(c) K p increases with decrease of x
1 1
(a) - 1 (b) - (c) (d) 1 (d) K p remains constant with change in p and x
2 2
20. When two reactants, A and B are mixed to give products, C
15. The species present in solution when CO2 is dissolved in
and D, the reaction quotient, (Q ) at the initial stages of the
water are (2006 Main)
reaction (2000)
(a) CO2 , H2 CO3 , HCO3- , CO32 -
(a) is zero
(b) H2 CO3 , CO32- (b) decreases with time
(c) HCO-3 , CO32 - (c) is independent of time
(d) CO2 , H2 CO3 (d) increases with time
Chemical and Ionic Equilibrium 81

21. For the reversible reaction, Objective Questions II


N2 ( g ) + 3H2 ( g ) r 2NH3 ( g )
(One or more than one correct option)
at 500° C , the value of K p is 1.44 ´ 10–5 when partial
28. For a reaction, A P, the plots of [A] and [P] with time
-
pressure is measured in atmosphere. The corresponding at temperatures T1 and T2 are given below.
value of K c with concentration in mol/L is (2000, S, 1M)

1.44 ´ 10-5 1.44 ´ 10-5 10 10


(a) (b) T1

[A]/(mol L–1)

[P]/(mol L–1)
-2 -2
(0.082 ´ 500) (8.314 ´ 773) T2
5 5
1.44 ´ 10-5 1.44 ´ 10–5
(c) (d) T2
(0.082 ´ 773)2 (0.082 ´ 773)-2 T1

w
22. For the chemical reaction, Time Time
3 X ( g ) + Y ( g ) r X 3Y ( g ) If T2 > T1 , the correct statement(s) is are
the amount of X 3Y at equilibrium is affected by (1999, 2M) (Assume DH s and DS s are independent of temperature and

Flo
(a) temperature and pressure ratio of ln K at T1 to ln K at T2 is greater than T2 / T1 . Here
(b) temperature only H , S , G and K are enthalpy, entropy, Gibbs energy and

ree
(c) pressure only equilibrium constant, respectively.) (2018 Adv.)
(d) temperature, pressure and catalyst (a) DH s < 0, DS s < 0 (b) DGs < 0, DH s > 0

F
23. For the reaction , (c) DGs < 0, DS s < 0 (d) DGs < 0, DS s > 0
CO( g ) + H2 O( g ) r CO2 ( g ) + H2 ( g ) ,
29. The % yield of ammonia as a function of time in the reaction,

or
ur
at a given temperature, the equilibrium amount of CO2 ( g )
N2( g )+ 3H2( g ) w 2 NH3( g ); DH < 0 (2015 adv.)
can be increased by (1998)

f
(a) adding a suitable catalyst at ( p, T1 ) is given below.
ks
(b) adding an inert gas
Yo
(c) decreasing the volume of the container
oo
% yield
T1
(d) increasing the amount of CO( g )
eB

24. One mole of N2 O4 ( g ) at 300 K is kept in a closed container


under one atmosphere. It is heated to 600 K when 20% by
Time
mass of N2 O4 ( g ) decomposes to NO2 (g). The resultant
r

pressure is (1996, 1M) If this reaction is conducted at ( p , T1 ), with T2 > T1 the %


ou
ad

(a) 1.2 atm (b) 2.4 atm (c) 2.0 atm (d) 1.0 atm yield by of ammonia as a function of time is represented by
25. An example of a reversible reaction is
Y

(1985, 1M)
(a) Pb(NO3 )2 ( aq ) + 2NaI ( aq ) = PbI2 ( s ) + 2NaNO3 ( aq ) T2 T1
% yield

% yield

T1 T2
(b) AgNO3 ( aq ) + HCl ( aq ) = AgCl ( s ) + HNO3 ( aq ) (a) (b)
nd
Re

(c) 2Na ( s ) + 2H2 O ( l ) = 2NaOH ( aq ) + H2 ( g )


(d) KNO3 ( aq ) + NaCl ( aq ) = KCl ( aq ) + NaNO3 ( aq ) Time Time
Fi

26. Pure ammonia is placed in a vessel at a temperature where its T2


T1
dissociation constant ( a ) is appreciable. At equilibrium,
% yield

% yield

T2 T1
N2 + 3H2 s 2NH3 (c) (d)
(1984, 1M)
(a) K p does not change significantly with pressure
Time Time
(b) a does not change with pressure
(c) concentration of NH3 does not change with pressure 30. The initial rate of hydrolysis of methyl acetate (1 M) by a
weak acid (HA, 1M) is 1/100th of that of a strong acid
(d) concentration of hydrogen is less than that of nitrogen
(HX, 1M), at 25°C. The K a (HA) is (2013 Adv.)
27. For the reaction, H2 ( g ) + I2 ( g ) r 2HI( g ) (a) 1 ´ 10-4 (b) 1 ´ 10-5
the equilibrium constant K p changes with (1981, 1M) (c) 1 ´ 10-6 (d) 1 ´ 10-3
(a) total pressure
31. The equilibrium 2 Cu I r Cu 0 + Cu II in aqueous medium
(b) catalyst
(c) the amount of H2 and I2 present at 25°C shifts towards the left in the presence of (2011)

(d) temperature (a) NO -3 (b) Cl - (c) SCN- (d) CN-


82 Chemical and Ionic Equilibrium

32. For the reaction, PCl 5 ( g ) r PCl 3 ( g ) + Cl 2 ( g ) the 42. When a liquid and its vapour are at equilibrium and the pressure
forward reaction at constant temperature is favoured by is suddenly decreased, cooling occurs. (1984, 1M)
(1991, 1M)
(a) introducing an inert gas at constant volume Subjective Questions
(b) introducing chlorine gas at constant volume 43. (a) In the following equilibrium N2O4 (g ) r 2NO2 (g )
(c) introducing an inert gas at constant pressure when 5 moles of each are taken, the temperature is kept at 298
(d) increasing the volume of the container K the total pressure was found to be 20 bar. Given that
(e) introducing PCl 5 at constant volume
DG °f (N2 O4 ) = 100 kJ, DG °f (NO2 ) = 50 kJ
33. The equilibrium SO2 Cl 2 ( g ) r SO2 ( g ) + Cl 2 ( g ) is (i) Find DG of the reaction.
attained at 25° C in a closed container and an inert gas, (ii) The direction of the reaction in which the equilibrium shifts.
helium is introduced. Which of the following statements (b) A graph is plotted for a real gas which follows van der Waals’

w
are correct? (1989, 1M) equation with pVm taken on Y-axis and p on X-axis. Find the
(a) Concentration of SO2 ,Cl 2 and SO2 Cl 2 change intercept of the line where Vm is molar volume. (2004, 4M)

(b) More chlorine is formed 44. When 3.06 g of solid NH4 SH is introduced into a two litre

Flo
(c) Concentration of SO2 is reduced evacuated flask at 27° C, 30% of the solid decomposes into
(d) None of the above gaseous ammonia and hydrogen sulphide.
34. When NaNO3 is heated in a closed vessel, oxygen is liberated (i) Calculate K c and K p for the reaction at 27°C.

ree
and NaNO2 is left behind. At equilibrium, (1986, 1M) (ii) What would happen to the equilibrium when more solid
(a) addition of NaNO2 favours reverse reaction NH4SH is introduced into the flask? (1999, 7M)

F
(b) addition of NaNO3 favours forward reaction 45. (a) The degree of dissociation is 0.4 at 400 K and 1.0 atm for the
(c) increasing temperature favours forward reaction gaseous reaction PCl 5 r PCl 3 + Cl 2. Assuming ideal

or
ur
(d) increasing pressure favours reverse reaction behaviour of all the gases, calculate the density of equilibrium
mixture at 400 K and 1.0 atm (relative atomic mass of P = 31.0

f
35 For the gas phase reaction,
and Cl = 35.5).
C2 H4 + H2 r C2 H6 ( DH = -32.7 kcal)
ks
(b) Given, [Ag(NH3 )2+ ] r Ag+ + 2NH3 ,
carried out in a vessel, the equilibrium concentration of
Yo
K c = 6.2 ´ 10-8 and K sp of AgCl
oo
C2 H4 can be increased by (1984, 1M)
= 1.8 ´ 10-10 at 298 K.
(a) increasing the temperature
eB

If ammonia is added to a water solution containing excess of


(b) decreasing the pressure AgCl(s) only. Calculate the concentration of the complex in
(c) removing some H2 1.0 M aqueous ammonia. (1998, 3M+5M)
(d) adding some C2 H6
r

46. The progress of reaction,


ou
ad

A r nB
Fill in the Blanks
with time, is represented in fig. use given below.
Y

36. For a gaseous reaction 2B ¾® A, the equilibrium


constant K p is …… to/than K c .
(Concentration/mol L 1 )

(1997 C, 1M)
nd
Re

37. A ten-fold increase in pressure on the reaction, 0.5


N2 ( g ) + 3H2 ( g ) r 2NH3 ( g ) at equilibrium, results
Fi

in .............. in K p .
(1996, 1M) 0.3
38. For a given reversible reaction at a fixed temperature,
equilibrium constant K p and K c are related by .....
(1994, 1M) 0.1

True/False 1 3 5 7
Time/h
39. The rate of an exothermic reaction increases with Determine :
increasing temperature. (1993, 1M) (i) the value of n
40. Catalyst makes a reaction more exothermic. (1987, 1M) (ii) the equilibrium constant, K and
(iii) the initial rate of conversion of A. (1994, 3M)
41. If equilibrium constant for the reaction,
A2 + B2 r 2 AB, is K, then for the backward reaction 47. 0.15 mole of CO taken in a 2.5 L flask is maintained at 750 K
1 1 1 along with a catalyst so that the following reaction can take place:
AB r A2 + B2 , the equilibrium constant is . CO ( g ) + 2H2 ( g ) r CH3 OH( g )
2 2 K
(1984, 1M) Hydrogen is introduced until the total pressure of the system is
8.5 atm at equilibrium and 0.08 mole of methanol is formed.
Chemical and Ionic Equilibrium 83

Calculate (i) K p and K c and (ii) the final pressure if the 54. One mole of nitrogen is mixed with three moles of hydrogen
same amount of CO and H2 as before are used, but with no in a four litre container. If 0.25 per cent of nitrogen is
catalyst so that the reaction does not take place. (1993, 5M) converted to ammonia by the following reaction
N2 ( g ) + 3H2 ( g ) r 2NH3 ( g ), then
48. For the reaction, CO( g ) + 2H2 ( g ) r CH3 OH( g )
calculate the equilibrium constant, K c in concentration units.
hydrogen gas is introduced into a five litre flask at 327° C, What will be the value of K c for the following equilibrium?
containing 0.2 mole of CO( g ) and a catalyst, until the 1 3
pressure is 4.92 atm. At this point 0.1 mole of CH3 OH( g ) is N2 ( g ) + H2 ( g ) r NH3 ( g ) (1981, 4M)
2 2
formed. Calculate the equilibrium constant, K p and K c .
(1990, 5M) Passage Based Questions
49. The equilibrium constant K p of the reaction, Thermal decomposition of gaseous X 2 to gaseous X at 298 K takes

w
place according to the following equation:
2SO2 ( g ) + O2 ( g ) r 2SO3 ( g )
X 2 (g ) s 2X (g )
is 900 atm at 800 K. A mixture containing SO3 and O2 having
initial pressure of 1 and 2 atm respectively is heated at The standard reaction Gibbs energy, Dr G°, of this reaction is

Flo
constant volume to equilibrate. Calculate the partial pressure positive. At the start of the reaction, there is one mole of X 2 and no
of each gas at 800 K. X . As the reaction proceeds, the number of moles of X formed is
(1989, 3M)
given by b. Thus, bequilibrium is the number of moles of X formed at

ree
50. N2 O4 is 25% dissociated at 37° C and one atmosphere equilibrium. The reaction is carried out at a constant total pressure
pressure. Calculate (i) K p and (ii) the percentage dissociation of 2 bar. Consider the gases to behave ideally.

F
at 0.1 atm and 37° C. (1988, 4M) (Given, R = 0.083 L bar K -1 mol -1 )
51. At a certain temperature, equilibrium constant ( K c ) is 16 for

or
55. The equilibrium constant K p for this reaction at 298 K, in
the reaction;
ur
terms of bequilibrium is (2016 Adv.)

f
SO2 ( g ) + NO2 ( g ) r SO3 ( g ) + NO( g )
8 b2 equilibrium 8b2 equilibrium
ks
If we take one mole each of all the four gases in a one litre (a) (b)
2 - bequilibrium 4 - b2 equilibrium
Yo
container, what would be the equilibrium concentrations of
oo
NO and NO2 ? (1987, 5M) 4 b2 equilibrium 4 b2 equilibrium
(c) (d)
2 - bequilibrium 4 - b2 equilibrium
eB

52. The equilibrium constant of the reaction


A2 ( g ) + B2 ( g ) r 2 AB ( g ) at 100°C is 50. If a one litre 56. The incorrect statement among the following for this
flask containing one mole of A2 is connected to a two litre reaction, is (2016 Adv.)
r

flask containing two moles of B2 , how many moles of AB (a) Decrease in the total pressure will result in the
ou
ad

will be formed at 373 K? (1985, 4M) formation of more moles of gaseous X


(b) At the start of the reaction, dissociation of gaseous X 2
Y

53. One mole of N2 and 3 moles of PCl 5 are placed in a 100 L


vessel heated to 227°C. The equilibrium pressure is 2.05 takes place spontaneously
atm. Assuming ideal behaviour, calculate the degree of (c) bequilibrium= 0.7
nd
Re

dissociation for PCl 5 and K p for the reaction, (d) K C < 1


PCl 5 ( g ) r PCl 3 ( g ) + Cl 2 ( g ) (1984, 6M)
Fi

Topic 2 Ionic Equilibrium


Objective Questions I (Only one correct option) 2. What is the molar solubility of Al(OH)3 in 0.2 M NaOH
-5
. ´ 10 m in water.
1. The molar solubility of Cd (OH)2 is 184 solution? Given that, solubility product of
The expected solubility of Cd(OH)2 in a buffer solution of Al(OH)3 = 2.4 ´ 10-24 (2019 Main, 12 April II)
pH = 12 is (2019 Main, 12 April II) (a) 3 ´ 10-19
2. 49 12 ´ 10-21
. ´ 10-9 M
(a) 184 (b) ´ 10-9 M (b)
184
. (c) 3 ´ 10-22
(c) 6.23 ´ 10-11 M (d) 2.49 ´ 10-10 M (d) 12 ´ 10-23
84 Chemical and Ionic Equilibrium

3. The pH of a 0.02 M NH4 Cl solution will be [Given 12. How many litres of water must be added to 1 L of an aqueous
K b (NH4 OH) = 10 -5
and log 2 = 0.301] solution of HCl with a pH of 1 to create an aqueous solution
(2019 Main, 10 April II)
with pH of 2? (2013 Main)
(a) 4.65 (b) 2.65 (c) 5.35 (d) 4.35
(a) 0.1 L (b) 0.9 L (c) 2.0 L (d) 9.0 L
4. Consider the following statements. 13. Solubility product constant ( K sp ) of salts of types MX , MX 2
I. The pH of a mixture containing 400 mL of 0.1 M H2SO4
and 400 mL of 0.1 M NaOH will be approximately 1.3. and M 3 X at temperature ‘T ’ are 4.0 ´ 10-8 , 3.2 ´ 10-14 and
II. Ionic product of water is temperature dependent. 2.7 ´ 10-15 , respectively. Solubilities (mol dm- 3 ) of the salts
III. A monobasic acid with K a = 10-5 has a pH = 5. The degree at temperature ‘T ’ are in the order (2008, 3M)
of dissociation of this acid is 50%. (a) MX > MX 2 > M 3 X (b) M 3 X > MX 2 > MX
IV. The Le-Chatelier’s principle is not applicable to (c) MX 2 > M 3 X > MX (d) MX > M 3 X > MX 2

w
common-ion effect.
2
The correct statements are (2019 Main, 10 April I) 14. 2.5 mL of M weak monoacidic base (K b = 1 ´ 10- 12 at
(a) I, II and IV (b) II and III 5
2
(c) I and II (d) I, II and III 25°C) is titrated with M HCl in water at 25°C. The

Flo
15
5. If solubility product of Zr3 (PO4 )4 is denoted by K sp and its concentration of H+ at equivalence point is
(K w = 1 ´ 10- 14 at 25°C)

ree
molar solubility is denoted by S , then which of the following (2008, 3M)
relation between S and K sp is correct? (2019 Main, 8 April I)
1/ 6 1/ 7
(a) 3.7 ´ 10- 13 M (b) 3.2 ´ 10- 7 M
æ K sp ö æ K sp ö

F
(a) S = çç ÷ (b) S = çç ÷ (c) 3.2 ´ 10- 2 M (d) 2.7 ´ 10- 2 M
÷ ÷
è 144 ø è 6912 ø
15. CH3 NH2 (0.1 mole, K b = 5 ´ 10- 4 ) is added to 0.08 mole of

or
æ K sp ö
(c) S = çç ÷
1/ 9
ur æ K sp ö
(d) S = çç ÷
1/ 7
HCl and the solution is diluted to one litre, resulting
÷ ÷

f
è 929 ø è 216 ø hydrogen ion concentration is (2005, 1M)

6. If K sp of Ag 2 CO3 is 8 ´ 10- 12 , the molar solubility of (a) 1.6 ´ 10- 11 (b) 8 ´ 10- 11
ks
(c) 5 ´ 10- 5 (d) 8 ´ 10- 2
Yo
Ag 2 CO3 in 0.1 M AgNO3 is (2019 Main, 12 Jan II)
oo
(a) 8 ´ 10- 12 M (b) 8 ´ 10- 13 M 16. HX is a weak acid ( K a = 10-5 ). It forms a salt NaX (0.1M) on
eB

(c) 8 ´ 10- 10 M (d) 8 ´ 10- 11 M reacting with caustic soda. The degree of hydrolysis
of NaX is (2004, 1M)
7. 20 mL of 0.1 M H2 SO4 solution is added to 30 mL of 0.2 M
(a) 0.01% (b) 0.0001%
NH4 OH solution. The pH of the resultant mixture is [pK b of
r

(c) 0.1% (d) 0.5%


ou

NH4 OH = 4.7] (2019 Main, 9 Jan I)


17. A solution which is 10-3 M each in Mn 2+ , Fe2+ , Zn 2+ and
ad

(a) 9.3 (b) 5.0 (c) 9.0 (d) 5.2 Hg 2+ is treated with 10-16 M sulphide ion. If K sp of
Y

8. An aqueous solution contains an unknown concentration of MnS, FeS, ZnS and HgS are 10-15 , 10-23 , 10-20 and 10-54
Ba 2 + . When 50 mL of a 1 M solution of Na 2SO 4 is added, respectively, which one will precipitate first? (2003, 1M)
nd

BaSO4 just begins to precipitate. The final volume is (a) FeS (b) MgS (c) HgS (d) ZnS
Re

500 mL. The solubility product of BaSO4 is 1 ´ 10-10 . What 18. Identify the correct order of solubility of Na 2 S, CuS and ZnS
Fi

is the original concentration of Ba 2+ ? (2018 Main) in aqueous medium. (2002)


(a) 5 ´ 10-9 M (b) 2 ´ 10-9 M (a) CuS > ZnS > Na 2 S (b) ZnS > Na 2 S > CuS
. ´ 10-9 M
(c) 11 . ´ 10-10 M
(d) 10 (c) Na 2 S > CuS > ZnS (d) Na 2 S > ZnS > CuS
9. Which of the following are Lewis acids? (2018 Main) 19. For a sparingly soluble salt A p Bq , the relationship of its
(a) PH3 and BCl 3 (b) AlCl 3 and SiCl 4 solubility product ( Ls ) with its solubility (S) is (2001, 1M)
(c) PH3 and SiCl 4 (d) BCl 3 and AlCl 3 (a) Ls = S p+q
× p ×q
p q
(b) Ls = S p+q
× p ×q
q p

10. Which of the following salts is the most basic in aqueous (c) Ls = S pq
× p ×q
p q
(d) Ls = S pq
× ( p. q )( p + q)
solution? (2018 Main)
(a) Al(CN)3 (b) CH3 COOK 20. The pH of 0.1 M solution of the following salts increases in
(c) FeCl 3 (d) Pb(CH3 COO)2 the order (1999, 2M)

11. pK a of a weak acid (HA) and pK b of a weak base (BOH) are (a) NaCl < NH4 Cl < NaCN < HCl
3.2 and 3.4, respectively. The pH of their salt (AB) solution is (b) HCl < NH4 Cl < NaCl < NaCN
(2017 Main) (c) NaCN < NH4 Cl < NaCl < HCl
(a) 7.2 (b) 6.9 (c) 7.0 (d) 1.0 (d) HCl < NaCl < NaCN < NH4 Cl
Chemical and Ionic Equilibrium 85

21. Which of the following solutions will have pH close to 1.0 ? 30. A certain buffer solution contains equal concentration of X -
(1992, 1M) and HX. The K b for X - is 10-10 . The pH of the buffer is
(a) 100 mL of (M/10) HCl + 100 mL of (M/10) NaOH (1984, 1M)
(b) 55 mL of (M/10) HCl + 45 mL of (M/10) NaOH (a) 4 (b) 7
(c) 10 (d) 14
(c) 10 mL of (M/10) HCl + 90 mL of (M/10) NaOH
(d) 75 mL of (M/5) HCl + 25 mL of (M/5) NaOH 31. The precipitate of CaF2 , ( K sp = 1.7 ´ 10-10 ) is obtained, when
22. Amongst the following hydroxides, the one which has the equal volumes of which of the following are mixed?
lowest value of K sp at ordinary temperature (about 25° C) is (1982, 1M)
-4 2+ -4 -
(1990, 1M) (a) 10 M Ca + 10 MF
(a) Mg(OH)2 (b) Ca(OH)2 (c) Ba(OH)2 (d) Be(OH)2 (b) 10-2 M Ca 2 + + 10-3 M F-
(c) 10-5 M Ca 2 + + 10-3 M F-

w
23. Which of the following is the strongest acid? (1989, 1M)
(a) ClO3 (OH) (b) ClO2 (OH) (d) 10-3 M Ca 2 + + 10-5 M F-
(c) SO(OH)2 (d) SO2 (OH) 2 32. An acidic buffer solution can be prepared by mixing the

Flo
solution of (1981, 1M)
24. When equal volumes of the following solutions are mixed,
-10 (a) acetate and acetic acid
precipitation of AgCl ( K sp = 1.8 ´ 10 ) will occur only

ree
(b) ammonium chloride and ammonium hydroxide
with (1988, 1M) (c) sulphuric acid and sodium sulphate
(a) 10-4 M (Ag + ) and 10-4 M (Cl - ) (d) sodium chloride and sodium hydroxide

F
(b) 10-5 M (Ag + ) and 10-5 M (Cl - ) 33. Of the given anions, the strongest base is (1981, 1M)
-6 -6 - (a) ClO- (b) ClO-2

or
+
(c) 10 M (Ag ) and 10
ur
M (Cl )
(d) 10 -10 +
M (Ag ) and 10 -10
M (Cl - ) (c) ClO-3 (d) ClO-4

25. The pK a of acetyl salicylic acid (aspirin) is 3.5. The pH of


f
34. At 90°C, pure water has [H3 O+ ] as 10-6 mol L-1 . What is the
ks
gastric juice in human stomach is about 2-3 and the pH in the value of K w at 90°C ? (1981, 1M)
Yo
(a) 10-6 (b) 10-12 -14 -8
oo
small intestine is about 8. Aspirin will be (1988, 1M) (c) 10 (d) 10
(a) unionised in the small intestine and in the stomach 35. The pH of 10-8 M solution of HCl in water is (1981, 1M)
eB

(b) completely ionised in the small intestine and in the (a) 8 (b) -8
stomach (c) between 7 and 8 (d) between 6 and 7
(c) ionised in the stomach and almost unionised in the small
r

intestine Objective Questions II


ou
ad

(d) ionised in the small intestine and almost unionised in (One or more than one correct option)
Y

the stomach
36. The K sp of Ag 2 CrO4 is 1.1´ 10-12 at 298 K. The solubility
26. The compound that is not a Lewis acid is (1985, 1M)
(in mol/L) of Ag 2 CrO4 in a 0.1 M AgNO3 solution is
nd

(a) BF3 (b) AlCl 3 (c) BeCl 2 (d) SnCl 4


Re

(a) 1.1 ´ 10-11 (b) 1.1 ´ 10-10


27. The conjugate acid of NH–2 is (c) 1.1 ´ 10 -12
(d) 1.1 ´ 10-9
Fi

(1985, 1M) (2013 Adv.)


(a) NH3 (b) NH2 OH (c) NH+4 (d) N2 H4
37. Aqueous solutions of HNO3 KOH, CH3 COOH and
28. The best indicator for detection of end point in titration of a CH3 COONa of identical concentrations are provided. The
weak acid and a strong base is (1985, 1M) pair(s) of solutions which form a buffer upon mixing is(are)
(a) methyl orange (3 to 4) (a) HNO3 and CH3 COOH (2010)
(b) methyl red (5 to 6) (b) KOH and CH3 COONa
(c) bromothymol blue (6 to 7.5) (c) HNO3 and CH3 COONa
(d) phenolphthalein (8 to 9.6) (d) CH3 COOH and CH3 COONa
29. A certain weak acid has a dissociation constant of 1.0 ´ 10-4 . 38. A buffer solution can be prepared from a mixture of
The equilibrium constant for its reaction with a strong base is (a) sodium acetate and acetic acid in water (1999, 3M)
(1984, 1M) (b) sodium acetate and HCl in water
(a) 1.0 ´ 10-4 (b) 1.0 ´ 10-10 (c) ammonia and ammonium chloride in water
(c) 1.0 ´ 1010 (d) 1.0 ´ 1014 (d) ammonia and sodium hydroxide in water
86 Chemical and Ionic Equilibrium

39. Which of the following statement(s) is (are) correct? Fill in the Blanks
(a) The pH of 1.0 ´ 10-8 M solution of HCl is 8 (1998, 2M) 42. In the reaction, I- + I2 ¾® I3- , the Lewis acid is .............
(b) The conjugate base of H2 PO4- is HPO2-
4 (1997, 1M)
(c) Autoprotolysis constant of water increases with 43. Silver chloride is sparingly soluble in water because its
temperature lattice energy is greater than .............. energy. (1987, 1M)
(d) When a solution of a weak monoprotic acid is titrated
against a strong base, at half-neutralisation point 44. An element which can exist as a positive ion in acidic
æ 1ö solution and also as a negative ion in basic solution is said to
pH = ç ÷ pK a be................... (1984, 1M)
è 2ø
45. The conjugate base of HSO–4 in aqueous solution is ……

w
Numerical Value Based Question (1982, 1M)

40. The solubility of a salt of weak acid (AB) at pH 3 is True/False


-3 -1
Y ´ 10 mol L . The value of Y is__ (Given that the value 46. The following species are in increasing order of their acidic

Flo
of solubility product of AB ( K sp ) = 2 ´ 10 -10
and the value of property : ZnO, Na 2 O2 , P2 O5 , MgO. (1985, 1/2M)
-8 47. Solubility of sodium hydroxide increases with increase in
ionisation constant of HB ( K a ) = 1 ´ 10

ree
(2018 Adv.)
temperature. (1985, 1/2M)

Matrix Match Type 48. Aluminium chloride ( AlCl 3 ) is a Lewis acid because it can

F
41. Dilution processes of different aqueous solutions, with donate electrons. (1982, 1M)

water, are given in List-I. The effects of dilution of the

or
ur
solution on [H + ] are given in List-II. Integer Answer Type Questions

f
Note Degree of dissociation ( a ) of weak acid and weak
49. The molar conductivity of a solution of a weak acid HX (0.01
+
M) is 10 times smaller than the molar conductivity of a
ks
base is << 1; degree of hydrolysis of salt << 1; [H ] represents solution of a weak acid HY (0.10 M). If l0X - » l0Y - , the
Yo
the concentration of H+ ions
difference in their pK a values, pK a (HX ) - pK a (HY ), is
oo

List-I List-II (consider degree of ionisation of both acids to be <<1).


eB

P. (10 mL of 0.1 M NaOH + 1. the value of [H+ ] does not (2015 Adv.)
20 mL of 0.1 M acetic change on dilution 50. In 1 L saturated solution of AgCl [K sp ( AgCl ) = 1.6 ´ 10-10 ],
acid) diluted to 60 mL
0.1 mole of CuCl [K sp ( CuCl )= 1.0 ´ 10-6 ] is added. The
r

Q. (20 mL of 0.1 M NaOH + 2. the value of [H+ ] changes


ou

resultant concentration of Ag + in the solution is 1.6 ´ 10- x .


ad

20 mL of 0.1 M acetic to half of its initial value


acid) diluted to 80 mL on dilution
The value of ‘x’ is (2011)
Y

R. (20 mL of 0.1M HCl + 20 3. the value of [H+ ] changes


mL of 0.1 M ammonia to two times of its initial 51. Amongst the following, the total number of compounds
solution) diluted to 80 mL whose aqueous solution turns red litmus paper blue is
nd

value on dilution.
Re

S. 10 mL saturated solution 4. the value of [H+ ] changes KCN K 2SO4 (NH4 )2 C2O4 NaCl
of Ni(OH)2 in equilibrium 1
Fi

to times of its initial Zn(NO3 )2 FeCl 3 K 2CO3 NH4NO3


with exces solid Ni(OH)2 2 LiCN (2010)
is diluted to 20 mL (solid value on dilution
Ni(OH)2 is still present
after dilution).
Subjective Questions
5. the value of [H+ ] changes 52. The dissociation constant of a substituted benzoic acid at
to 2 times of its initial 25°C is 1.0 ´ 10- 4 . The pH of 0.01 M solution of its sodium
value on dilution salt is (2009)
Match each process given in List-I with one or more effect(s) 53. 0.1 M of HA is titrated with 0.1 M NaOH, calculate the pH at
in List-II. The correct option is (2018 Adv.)
end point. Given, K a (HA) = 5 ´ 10- 6 and a << 1. (2004)
(a) P ® 4; Q ® 2; R ® 3; S ® 1
(b) P ® 4; Q ® 3; R ® 2; S ® 3 54. 500 mL of 0.2 M aqueous solution of acetic acid is mixed
(c) P ® 1; Q ® 4; R ® 5; S ® 3 with 500 mL of 0.2 M HCl at 250°C.
(d) P ® 1; Q ® 5; R ® 4; S ® 1 (i) Calculate the degree of dissociation of acetic acid in the
resulting solution and pH of the solution.
Chemical and Ionic Equilibrium 87

(ii) If 6 g of NaOH is added to the above solution, determine the 64. An aqueous solution of a metal bromide MBr2 (0.05 M) is
final pH (assuming there is no change in volume on mixing,
saturated with H2 S. What is the minimum pH at which MS
K a of acetic acid is 1.75 ´ 10-5 mol/L. (1984, 1M) will precipitate? K sp for MS = 6.0 ´ 10-21 , concentration of
55. The average concentration of SO2 in the atmosphere over a saturated H2 S = 0.1 M, K 1 = 10-7 and K 2 = 1.3 ´ 10-13 , for
city on a certain day is 10 ppm, when the average H2 S. (1993, 3M)
temperature is 298 K. Given that the solubility of SO2 in
65. The pH of blood stream is maintained by a proper balance of
water at 298 K is 1.3653 mol/L and pK a of H2 SO3 is 1.92,
H2 CO3 and NaHCO3 concentrations. What volume of 5 M
estimate the pH of rain on that day. (2000, 5M) NaHCO3 solution should be mixed with a 10 mL sample of
56. The solubility of Pb(OH)2 in water is 6.7 ´ 10-6 M. Calculate blood which is 2 M in H2 CO3 , in order to maintain a pH of
7.4? (K a for H2 CO3 in blood is 7.8 ´ 10-7 )
the solubility of Pb(OH)2 in a buffer solution of pH = 8. (1993, 2M)

w
(1999, 4M) 66. The solubility product ( K sp ) of Ca(OH)2 at 25° C is
57. (a) Find the solubility product of a saturated solution of 4.42 ´ 10-5 . A 500 mL of saturated solution of Ca(OH)2 is
Ag2CrO4 in water at 298 K if the emf of the cell

Flo
mixed with equal volume of 0.4 M NaOH. How much
Ag|Ag+ (saturated. Ag2CrO4 solution.) || Ag+ (0.1 M) | Ag is
0.164 V at 298 K. Ca(OH)2 in milligrams is precipitated? (1992, 4M)
(1998, 6M)

ree
(b) What will be the resultant pH when 200 mL of an aqueous 67. A 40 mL solution of a weak base, BOH is titrated with 0.1N
solution of HCl (pH = 2.0) is mixed with 300 mL of an HCl solution. The pH of the solution is found to be 10.04 and
aqueous solution of NaOH (pH = 12.0) ?

F
(1998, 6M) 9.14 after the addition of 5.0 mL and 20.0 mL of the acid
58. A sample of AgCl was treated with 5.00 mL of 1.5 M respectively. Find out the dissociation constant of the base.

or
(1991, 6M)
ur
Na 2 CO3 solution to give Ag 2 CO3 . The remaining solution
contained 0.0026 g of Cl – ions per litre. Calculate the 68. The solubility product of Ag 2 C2 O4 at 25° C is
solubility product of AgCl. [K sp (Ag 2 CO3 ) = 8.2 ´ 10-12 ]
f
-11 3 -3
1.29 ´ 10 mol L . A solution of K 2 C2 O4 containing
ks
(1997, 5M) 0.1520 mole in 500 mL water is shaken at 25° C with excess
Yo
of Ag 2 CO3 till the following equilibrium is reached
oo
59. An acid type indicator, HIn differs in colour from its
conjugate base (In - ). The human eye is sensitive to colour Ag 2 CO3 + K 2 C2 O4 r Ag 2 C2 O4 + K 2 CO3
eB

differences only when the ratio [ In – ] / [ HIn ] is greater than At equilibrium, the solution contains 0.0358 mole of
10 or smaller than 0.1. What should be the minimum change K 2 CO3 . Assuming the degree of dissociation of K 2 C2 O4 and
in the pH of the solution to observe a complete colour K 2 CO3 to be equal, calculate the solubility product of
r

change? ( K a = 1.0 ´ 10-5 ) Ag 2 CO3 .


ou

(1991, 4M)
ad

(1997, 2M)

60. The ionisation constant of NH+4 in water is 5.6 ´ 10-10 at 69. What is the pH of a 1.0 M solution of acetic acid? To what
Y

volume must one litre of this solution be diluted so that the


25° C. The rate constant for the reaction of NH+4 and OH- to pH of the resulting solution will be twice the original value?
form NH3 and H2 O at 25° C is 3.4 ´ 1010 L/mol/s. Calculate Given, K a = 1.8 ´ 10-5
nd
Re

(1990, 4M)
the rate constant per proton transfer from water to NH3 .
70. Freshly precipitated aluminium and magnesium hydroxides
Fi

(1996, 3M)
are stirred vigorously in a buffer solution containing
61. What is the pH of a 0.50 M aqueous NaCN solution? 0.25 mol/L of NH4 Cl and 0.05 M of ammonium hydroxide.
(pK b of CN- = 4.70). (1996, 2M) Calculate the concentration of aluminium and magnesium
ions in solution.
62. Calculate the pH of an aqueous solution of 1.0 M ammonium
formate assuming complete dissociation. K b [NH4 OH] = 1.8 ´ 10-5
(pK a of formic acid = 3.8 and pK b of ammonia = 4.8) K sp [Mg(OH)2 ] = 8.9 ´ 10-12
(1995, 2M)
K sp [Al(OH)3 ] = 6 ´ 10-32 (1989, 3M)
63. For the reaction, [Ag(CN)2 ]– r Ag + + 2CN-
71. How many gram-mole of HCl will be required to prepare one
The equilibrium constant, at 25° C, is 4.0 ´ 10-19 . Calculate litre of buffer solution (containing NaCN and HCl) of pH 8.5
the silver ion concentration in a solution which was using 0.01 g formula weight of NaCN?
originally 0.10 M in KCN and 0.03 M in AgNO3 . (1994, 3M)
K HCN = 4.1 ´ 10-10 (1988, 4M)
88 Chemical and Ionic Equilibrium

72. What is the pH of the solution when 0.20 mole of HCl is 76. The dissociation constant of a weak acid HA is 4.9 ´ 10-8 .
added to one litre of a solution containing After making the necessary approximations, calculate
(i) 1 M each of acetic acid and acetate ion, (i) pH
(ii) 0.1 M each of acetic acid and acetate ion? (ii) OH- concentration in a decimolar solution of the acid.
Assume the total volume is one litre. (Water has a pH of 7). (1983, 2M)
K a for acetic acid = 1.8 ´ 10-5. (1987, 5M) 77. Give reason for the statement that “the pH of an aqueous
-3
73. The solubility of Mg(OH)2 in pure water is 9.57 ´ 10 g / L. solution of sodium acetate is more than seven”. (1982, 1M)

Calculate its solubility (in g/L) in 0.02 M Mg(NO3 )2 78. 20 mL of 0.2 M sodium hydroxide is added to 50 mL of
solution. (1986, 5M) 0.2 M acetic acid solution to give 70 mL of the solution.
74. The concentration of hydrogen ions in a 0.20 M solution of What is the pH of this solution?

w
formic acid is 6.4 ´ 10-3 mol/L. To this solution, sodium Calculate the additional volume of 0.2 M NaOH required to
formate is added so as to adjust the concentration of sodium make the pH of the solution 4.74.
formate to one mole per litre. (Ionisation constant of CH3 COOH = 1.8 ´ 10-5 ). (1982, 3M)

Flo
What will be the pH of this solution? The dissociation
79. How many moles of sodium propionate should be added to
constant of formic acid is 2.4 ´ 10-4 and the degree of
1 L of an aqueous solution containing 0.020 mole of

ree
dissociation of sodium formate is 0.75. (1985, 3M) propionic acid to obtain a buffer solution of pH 4.75? What
75. A solution contains a mixture of Ag + (0.10 M) and will be pH if 0.010 moles of HCl are dissolved in the above
Hg 2+ (0.10 M) which are to be separated by selective

F
buffer solution? Compare the last pH value with the pH of
precipitation. Calculate the maximum concentration of 0.010 M HCl solution. Dissociation constant of propionic
iodide ion at which one of them gets precipitated almost acid, K a at 25°C is 1.34 ´ 10-5 .

or
ur
completely. What percentage of that metal ion is
(1981, 4M)

f
precipitated? (1984, 4M)
K sp : AgI = 8.5 ´ 10-17 , HgI2 = 2.5 ´ 10-26
ks
Yo
Answers
oo

Topic 1 9. (d) 10. (b) 11. (b) 12. (d)


eB

1. (d) 2. (a) 3. (d) 4. (a) 13. (d) 14. (d) 15. (b) 16. (a)
5. (d) 6. (d) 7. (b) 8. (b) 17. (c) 18. (d) 19. (a) 20. (b)
9. (b) 10. (c) 11. (b) 12. (b) 21. (d) 22. (d) 23. (a) 24. (a)
r

(b) (b) (a) (b) 25. (d) 26. (c) 27. (a) 28. (d)
ou

13. 14. 15. 16.


ad

29. (c) 30. (a) 31. (b) 32. (a)


17. (a) 18. (d) 19. (d) 20. (d)
33. (a) 34. (b) 35. (d) 36. (b)
Y

21. (d) 22. (a) 23. (d) 24. (b) 37. (c, d) 38. (a, b, c) 39. (b, c) 40. (4.47)
25. (d) 26. (a) 27. (d) 28. (a,c) 41. (d) 42. I2 43. hydration
29. (b) 30. (a) 31. (b, c, d) 32. (c, d, e) 44. amphoteric SO 2- F 47. F
nd

45. 4 46.
Re

33. (d) 34. (c, d) 35. (a, b, c, d) 36. smaller 48. F 49. (3) 50. (1.6 ´ 10 -7) 52. (8)
37. no change 38. K p = Kc ( RT ) Dn 39. T (1.2 ´ 10 -3 M)
Fi

53. (9) 55. (4.86) 56.


40. F 41. F 42. T 46. (1.2) 58. (2 ´ 10 -8) 61. (11.5) 62. (6.50) 64. (1)
52. (1.86) 53. (0.33) 55. (b) 56. (c) 65. (80) 67. (1.8 ´ 10 -5) 68. (9.67 ´ 10 -11) 69.
(27.78 ´ 10 3)
Topic 2 71. (0.177) 72. (8.7 ´ 10 -4 gL -1) 74. (4.20)
1. (d) 2. (c) 3. (c) 4. (d)
75. (99.83) 77. (>7)
5. (b) 6. (c) 7. (a) 8. (c)
Hints & Solutions
Topic 1 Chemical Equilibrium
1. The incorrect match is DG ° < 0, K < 1. (c) For the equilibrium,
For an ideal gas DG ° = - RT ln K . Dng = 2 - (2 + 1) = - 1, i.e. (-ve)
DG ° So, increase in pressure will shift the equilibrium in the
\ ln K = - and K = e- DG ° / RT forward direction.
RT
The above equation is helpful in predicting the spontaneity of Thus, statement (c) is correct.
the reaction. e.g. (d) The reaction takes place in the presence of a catalyst which is

w
V2O5 (s) in contact process or NO(g ) in chamber process.
(i) If DG ° < 0, – DG °/ RT = + ve and e- DG ° / RT > 1 and hence,
Thus, statement (d) is incorrect.
K > 1. It means that the reaction occur spontaneously in the
forward direction or products predominate over reactants. 4. S + O2 - SO2 , K 1

Flo
(ii) If DG ° > 0; - DG °/ RT = - ve and 1
\ SO2 - S + O2 , K 1 ¢ =
- DG °/ RT
e < 1 and hence, K < 1. It means that the reaction is K1

ree
non-spontaneous in forward direction (i.e. product side) but 1
or, 2SO2 - 2S + 2O2, K 1 ¢¢ = (K 1 ¢ )2 = … (i)
spontaneous in reverse direction (i.e. reactants predominate K 12

F
over products or the reaction occurs rarely).
Þ 2S + 3O2 - 2SO3 , K 2 … (ii)
(iii) When K = 1, then DG ° = 0. This situation generally occur
Now, [(i) + (ii)] gives
at equilibrium.

or
ur 2SO2 + O2 - 2SO3 , K 3
2. The value of equilibrium constant,
Key Idea The relationship between K p and K c is
f K 3 = K 2 ´ K 1 ¢¢ = K 2 ´ 2
1
ks
Dng K1
K p = K c (RT )
Yo
1
= 10129 - 104 = 1025
oo
where, Dng = nproducts - nreactants = 10129 ´
(1052 )2
If Dng = 0 then K p = K c
eB

If Dng = + ve then K p > K c 5. For the given chemical reaction,


If Dng = - ve then K p < K c A + 2B w 2C + D
At, t = 0 a0 1.5a 0 0 0
r

Consider the following equilibria reactions t = t eq a0 - x 1.5a 0 - 2x 2x x


ou

[x = degree of dissociation]
ad

(a) 2C(s) + O2 (g ) - 2CO(g )


Dng = nproduct - nreactant = 2 - (1) = 1 Given, at equilibrium.
Y

Dng ¹ 0 Þ So, K p ¹ K c [ A] =[ B]
a0 - x = 15. a0 - 2x
(b) 2HI(g ) -H2(g ) + I2 (g )
nd

x = 0.5a0
Re

Dng = nproduct - nreactant = 2 - 2 = 0


\ [A] = a0 - x = a0 - 0.5a0 = 0.5a0
Dng = 0 Þ So, K p = K c
Fi

[ B] = 15
. a0 - 2x = 15 . a0 - 2 ´ 0.5a0 = 0.5a0
(c) NO2 (g ) + SO2 (g ) - NO(g ) + SO3 (g )
[C] = 2x = 2 ´ 0.5a0 = a0
Dng = nproduct - nreactant = 2 - 2 = 0
[D] = x = 0.5a0
Dng = 0 Þ So, K p = K c
[C ]2[ D]
(d) 2NO(g ) - N2(g ) + O2 (g ) Now, K=
[ A][ B]2
Dng = nproduct - nreactant = 2 - 2 = 0
Dng = 0 Þ So, K p = K c Now, substituting the values in above equation, we get
(a0 )2 ´ (0.5a0 )
3. The explanation of given statements are as follows: K= =4
(0.5a0 ) ´ (0.5a0 )
(a) For the given equilibrium, DH is negative, so the equilibrium
constant will decrease with increase in temperature and the 6. The equilibrium reaction for the dissociation of two solids is
equilibrium will shift in the backward direction. given as:
Thus, statement (a) is correct. A(s) e B(g)+ C (g)
p1 p1 + p 2
(b) When inert gas is added at constant volume and constant At equilibrium
temperature, an equilibrium remains undisturbed. K p1 = x = pB × pC = p1( p1 + p2 ) …(i)
Thus, statement (b) is correct. Similarly, D(s) e C (g)+ E(g)
90 Chemical and Ionic Equilibrium

At equilibrium p1 + p2 × p2 9. We know that, the relationship between K p and KC of a


K p 2 = y = pC × pE =( p1 + p2) p2 …(ii) chemical equilibrium state (reaction) is
On adding Eq. (i) and (ii), we get. Dn Kp Dn
K p1 + K p 2 = x + y = p1( p1 + p2)+ p2( p1 + p2) K p = KC (RT ) g Þ = (RT ) g
KC
= ( p1 + p2 )2
where, Dng = SnProducts - SnReactants
or x + y = p1 + p2 …(iii) (i) N2(g ) + O2(g ) c 2NO(g )
Now, total pressure is given as Þ (RT )2 - (1 + 1) = (RT )0 = 1
pT = pB + pC + pE (ii) N2O4 (g ) 2NO2 (g )
c
= p1 + ( p1 + p2)+ p2 Þ (RT )2 - 1 = RT = 24.62 dm3 atmmol-1
= 2 ( p1 + p2) …(iv)
(iii) N2 (g ) + 3H2 (g ) c 2NH3 (g )

w
On substituting the value of p1 + p2 from Eq. (iii) to Eq. (iv), we
get Þ (RT )2- (3 + 1) = (RT )- 2
1
pT = 2 x + y =
(24.62 dm 3 atm mol -1 )2

Flo
7. N 2 (g ) + 3H2 (g ) = 2NH3 (g ) = 1.649 ´ 10-3 dm –6 atm - 2 mol 2
At equilibrium: pN 2 = P, pH 2 = 3P, pNH 3 = 2P [ AB ]2
10. (i) A2(g )+ B2(g ) c 2AB(g ); K1 =

ree
Þ p(total ) = pN 2 + pH 2 + pNH 3 - ~p + p
N2 H2 [ A2 ][ B2 ]
[Q P(total ) >> pNH 3 ]
= p + 3p = 4 p (ii) 6 AB(g ) c 3 A2(g ) + 3 B2 (g );

F
2 [ A2 ]3[ B2 ]3 1 1
p2NH 3 pNH K2 = = = 3,
Now, Kp = 3
= 3
[ AB ]6 æ [ AB ] ö2 3
K
pN 2 ´ pH p ´ (3 p)3 ç ÷
1

or
2
pNH
2
2
pNH
ur Þ K 2 = K 1-3
ç [ A ][ B ]÷
è 2 2 ø

f
= 3
4
= 3
4
[Q P = 4 p]
27 ´ p æ Pö 11. Given [H2S] = 010
. M
27 ´ ç ÷
ks
è4ø [HCl] = 0.20 M So, [H+ ] = 0.20 M
Yo
2
pNH ´ 44 H2S-H +
+ HS- , K 1 = 10
. ´ 10-7
oo
Kp = 3
-
32 ´ 3 ´ P 4 HS - H +
+ S2- , K 2 = 12
. ´ 10-13
eB

It means for,
32 ´ 3 ´ P 4 ´ K p
Þ 2
pNH = H2S - 2H +
+ S2-
3
44
. ´ 10-7 ´ 12
K = K 1 ´ K 2 = 10 . ´ 10-13
3 ´ 3 ´ P 2 ´ K p1/ 2
1/ 2
33/ 2 ´ P 2 ´ K p1/ 2
r

Þ pNH 3 = = . ´ 10-20
= 12
ou

42 16
ad

K ´ [ H2S]
Now [ S2- ] = [according to the final equation]
8. Molar mass of NH4SH = 18 + 33 = 51 g mol -1 [ H+ ]2
Y

Number of moles of NH4SH introduced in the vessel . ´ 10-20 ´ 01


12 . M
=
Weight 5×1 (0.2M)2
= = = 01. mol
nd

. ´ 10-20 ´ 1 ´ 10-1 M
Re

Molar mass 51 12
=
4 ´ 10-2 M
NH4 SH( s ) c NH3 ( g ) + H2 S( g )
Fi

-20
= 3 ´ 10 M
Number of 0.1 0 0
moles at t = 0
At t = t eq . (1 - 0. 03)
01 30% of 30% of 0.1
12. A + B q C + D
01. = 0. 03 = 0. 03 Initially at t = 0 1 1 1 1
Active mass 0. 03 0. 03 At equilibrium 1-x 1-x 1+x 1+x
= 0. 01 = 0. 01
(mol L -1) 3 3 [C ][ D ] (1 + x)(1 + x) (1 + x)2
K eq = = =
[ NH3 ][ H2S ] 0.01 ´ 0.01 [ A ][ B ] (1 - x)(1 - x) (1 - x)2
KC = = = 10-4 (mol L -1 2
)
[ NH4HS(s) ] 1 æ1 + xö
2
1+ x
Dng or 100 = çç ÷÷ or 10 =
Þ K p = KC (RT ) è1- xø 1- x

[where, Dng = Snproduct - Snreactant ] = 2 - 0 = 2 or 10 - 10x = 1 + x


10 - 1 = x + 10x
\ K p = KC (RT )2
9 = 11x
= 10- 4 ´ [0.082 ´ (273 + 327)]2 atm 2 x=
9
= 0.818
11
= 0.242 atm 2
\ [ D ] = 1 + x = 1 + 0.818 = 1818
.
Chemical and Ionic Equilibrium 91

13. Given, DG ° = 2494.2 J 21. N2 (g ) + 3H2 (g ) r 2NH3 (g ) Dn = - 2


1 Dn
2´ K p = K c ( RT )
[ B ][C ] 2 =4
Q= =
[ A ]2 æ ö
1
2 Kp 1.44 ´ 10- 5
ç ÷ Kc = Dn
=
è 2ø (RT ) (0.082 ´ 773)- 2
\ We know, DG = DG °+ RT ln Q 22. Both temperature and pressure will change the equilibrium
= 2494.2 + 8.314 ´ 300 ln 4 amount of X 3Y (g ). Temperature changes the value of equilibrium
constant.
= 28747.27 J (+ ve value)
Q 23. Adding reactant will drive the reaction in forward direction in
Also, we have DG = RT ln
K order to restore equilibrium. Therefore, addition of CO (g) will
If DG is positive, Q > K increase the equilibrium amount of CO2.

w
Therefore, reaction shifts in reverse direction. 24. N2O4 r 2NO2
14. For the given reaction, Dng = nP - nR At 300 K : 1.0 atm 0
At 600 K : 2.0 – 0.40 0.80 Total pressure = 2.40 atm
where, nP = number of moles of products

Flo
n R = number of moles of reactants
25. In reactions (a), (b) and (c), atleast one of the product is either
insoluble precipitate or a gas that drive the reaction
Dng

ree
K p = K c ( RT ) continuously to right and do not allow equilibrium to be
established. Following is the reversible reaction.
1
Dng = - KNO3 (aq) + NaCl (aq) r KCl (aq) + NaNO3 (aq)

F
2
15. When CO2 is dissolved in water, following equilibria are established: 26. Kp for a given reversible reaction depends only on temperature.

or
H2O + CO2 r H2CO3 27. Equilibrium constant of a given reversible reaction depends
H2CO3 r H+ + HCO-3
ur only on temperature.

f
HCO-3 r H+ + CO23- 28. For the reaction, A =P
ks
Therefore, in solution, all of the above mentioned species exist. Given, T1 < T2
Yo
ln K 1 T2
16. At equilibrium, DG = 0 > …(i)
oo
ln K 2 T1
G (reactants) = G (products)
It shows, On increasing the temperature, K decreases so reaction
eB

G (N2 ) + 3G (H2 ) = 2G (NH3 )


is exothermic i.e., DH o < 0
A catalyst does not affect either equilibrium composition or Besides, graph shows K >1
equilibrium constant, it just increases rate of both forward and
So D G º< 0
r

backward reaction but by the same factor.


Now from equation (i)
ou
ad

17. Ag+ + NH3 r [ Ag(NH3 )+ ] K 1 = 3.5 ´ 10- 3 T1 ln K 1 > T2 ln K 2


Y

[Ag(NH3 )+ ] + NH3 r [ Ag(NH3 )+2 ] K 2 = 1.7 ´ 10- 3 - DG º1> - DG º2


Adding : Ag+ + 2NH3 r [ Ag(NH3 )2+ ] Likewise (- DH º+ T1DS º ) > (- DH º+ T2 DS º )
or simply T1DS º> T2DS º
nd

K = K 1 ´ K 2 = 5.95 ´ 10- 6
Re

So, (T2 - T1 ) DS ° < 0


18. N2O4 (g ) r 2NO2 (g ) Total \ DS º< 0
Fi

1- a 2a 1+a In other words, increase of DG with increase in temperature is


1- a 2a 4a 2 possible only when DS ° < 0. Hence, options (a) and (c) are correct.
pi : p p Kp = p
1+ a 1+ a 1 - a2 29. Since, the reaction is exothermic, there will be less ammonia at
equilibrium and higher temperature. However, rate of reaction
At constant temperature, halving the volume will change both p increases with rise in temperature, NH3 will be formed at faster
and a but K p remains constant. rate in the initial stage when the temperature is high.
4 x2 p H+
19. N2O4 r 2NO2, K p = . K p is function of temperature 30. PLAN RCOOR ¢ + H 2O ¾¾® RCOOH + R ¢ OH
1- x 2x 1 - x2
Acid hydrolysis of ester is follows first order kinetics.
only, does not change with either p or x.
For same concentration of ester in each case, rate is dependent
[C ][ D ]
20. A + B r C + D, Q = on [ H+ ] from acid.
[ A ][ B ]
Rate = k [ RCOOR ¢ ]
As time passes, amount of products ‘C’ and ‘D’ increases, hence
Q increases. Also for weak acid, HA r H+ + A -
92 Chemical and Ionic Equilibrium

[ H+ ][ A - ] 41. It is
1
.
Ka =
[ HA ] K
(Rate)HA = k [ H+ ]HA 42. Evaporation is an endothermic process.
(Rate) HX = k [ H+ ]HX 43. (a) N2O4 (g ) r 2NO2 (g )
DG ° = 2 DG f° (NO2 ) - DG f° (N2O4 ) = 0
(Rate) HX = 100(Rate)HA
+ Also DG ° = - RT ln K = 0 , K = 1
\ Also in strong acid, [ H ] = [ HX ] = 1 M
Let the reaction shifts in forward direction.
(Rate) HX [ H + ] HX 1 N2O4 (g ) r 2NO2 (g ) Total
= 100 = =
(Rate) HA [ H + ] HA [ H+ ]HA 5-x 5 + 2x 10 + x
5-x 5 + 2x
1 pi : ´ 20 ´ 20
\ [ H + ] HA = 10 + x 10 + x

w
100
(5 + 2x )2 10 + x
HA r H+ + A - Þ K = ´ ´ 20 = 1
(10 + x )2 5-x
1 0 0
Þ 81x 2 + 405x + 450 = 0

Flo
(1 - x ) x x
x = 0.01 x = - 1.66 and – 3.33
[ H+ ][ A - ] 0.01 ´ 0.01

ree
Both values of x indicates that reaction actually proceeds in
\ Ka = = = 1.01 ´ 10-4 backward direction.
[ HA ] 0.99
æ a ö
(b) ç p + ÷ (Vm - b) = RT

F
31. Cl - , CN - and SCN - forms precipitate with Cu (I), remove
è Vm2 ø
Cu (I) ion from equilibrium and reaction shifts in backward
direction according to Le-Chatelier's principle. æ ap2 ö æ pV ö
çp+ ÷

or
çç - b÷÷ = RT
ur
32. If inert gas is introduced at constant pressure, volume of ç
è ( pV )2 ÷ø è p ø

f
container will have to be increased and this will favour the
Þ [( pV 2 ) p + ap2 ][( pV ) - b ] = p ( pV )2 RT
forward reaction. Also adding PCl 5 (g ) at constant volume will
ks
favour forward reaction because PCl 5 (g ) is a reactant. Þ p × [ pV 2 + ap ] ( pV - bp) = p ( pV 2 ) RT
Yo
33. SO2Cl 2 (g ) r SO2 (g ) + Cl 2 (g ), But p = 0
oo
Intercept = RT Þ ( pV )3 = ( pV )2 RT
Adding inert gas at constant volume will not affect partial
eB

pressure of reactant or products, hence will not affect 44. (i) Mole of solid NH4HS taken initially = 3.06 = 0.06
equilibrium amount of either reactant or products. 51
1 At equilibrium NH4HS (s) r NH3 (g ) + H2S (g )
34. NaNO3 (s) r NaNO2 (s) + O2 (g ) , DH > 0
r

2 0.018 0.018
ou
ad

NaNO3 and NaNO2 are in solid state, changing their amount has 2
æ 0.018 ö -5
no effect on equilibrium. Increasing temperature will favour Kc = ç ÷ = 8.1 ´ 10
Y

forward reaction due to endothermic nature of reaction. Also, è 2 ø


increasing pressure will favour backward reaction in which 0.018 ´ 0.082 ´ 300
p (NH3 ) = = 0.22 atm
some O2 (g ) will combine with NaNO2 (s) forming NaNO3. 2
nd
Re

35. C2H4 + H2 r C2H6, DH = - 32.7 kcal K p = (0.22)2 = 4.84 ´ 10- 2


Fi

The above reaction is exothermic, increasing temperature will (ii) Addition of solid NH4HS will have no effect on equilibrium.
favour backward reaction, will increase the amount of C2H4.
Decreasing pressure will favour reaction in direction containing 45. (a) PCl 5 (g ) r PCl 3 (g ) + Cl 2 (g ) Total moles
more molecules (reactant side in the present case). Therefore, 1- a a a 1+ a
decreasing pressure will increase amount of C2H4. 208.5
Removing H2 , which is a reactant, will favour reaction in Average molar mass = = 148.9
1.4
backward direction, more C2H4 will be formed.
Adding C2H6 will favour backward reaction and some of the pM 1 ´ 148.9
r (density) = = = 4.54 g/L
C2H6 will be dehydrogenated to C2H4. RT 0.082 ´ 400
K
36. Smaller : K p = c (b) AgCl (s) + 2NH3 (aq) r [ Ag(NH3 )2+ ] + Cl -
RT -x 1 - 2x x x
37. changing pressure has no effect on equilibrium constant. 2
K sp æ x ö
K = = 2.9 ´ 10- 3 = çç ÷÷
38. K p = K c (RT )Dn, where, Dn = Sn (products) - Sn (reactants) Kc è 1 - 2x ø
39. Rate of any reaction increases on rising temperature. x = 0.049 M

40. Catalyst has no effect on thermodynamics of reaction.


Chemical and Ionic Equilibrium 93

46. Observing the graph indicates that when 0.20 mole of A is 1


p= atm
reacted, 0.40 mole of product is formed. 87
A r nB 2
Partial pressure of SO2 = 2 p = atm
- 0.20 + 0. 40 Þ n=2 87
At equilibrium, [A] = 0.30 M, [B] = 0.60 M 1 175
Partial pressure of O2 = 2 + p = 2 + = atm
[ B ] 2 0.36 87 87
Kc = = = 1.2
[ A ] 0.30 æ 1 ö 85
Partial pressure of SO3 = 1 - 2 p = 1 - 2 ç ÷ = atm
47. CO (g ) + 2H2 (g ) r CH3OH (g ) è 87 ø 87
0.15 – 0.08 x - 0.16 0.08 50. N2O4 r 2NO2 Total
Total moles at equilibrium = x - 0.01 1- a 2a 1+ a
8.5 ´ 2.5 1- a 2a

w
x - 0.01 = = 0.34 Þ x = 0.35 pi :
1+ a
p
1+ a
p
0.082 ´ 750
0.07 4a 2 4 (0.25)2
(i) Partial pressures : CO = ´ 8.5 Kp = 2
p= = 0.26 atm
0.34 1- a 1 - (0.25)2

Flo
0.18 When p = 0.10 atm
H2 = ´ 8.5
0.34 4 a 2 (0.1)
0.26 = Þ a = 0.62

ree
0.08 1 - a2
CH3OH = ´ 8.5
0.34
51. SO2 (g ) + NO2 (g ) r SO3 (g ) + NO (g )

F
2
0.08 æ 0.34 ö
Kp = ´ ç ÷ = 0.056 1- x 1- x x x
(0.07) (0.18)2 è 8.5 ø
Qc = 1 < K c , i.e. reaction proceed in forward direction to attain

or
(ii) Concentrations : [CH3OH] =
ur
0.08
= 0.032 M equilibrium.
2.5

f
2
0.18 æ x ö
[H2 ] = = 0.072 M 16 = çç ÷÷ Þ x = 0.80
è 1- x ø
ks
2.5
Yo
0.07 [NO] = 0.80 M, [NO2 ] = 0.20 M
[CO] = = 0.028 M
oo
2.5
0.032 52. A2 (g ) + B2 (g ) r 2 AB (g ) Dn = 0
Kc = = 213.33
eB

(0.028) (0.072)2 [ AB ]2 (n )2 (2x )2


K = = AB =
[ A2 ][ B2 ] nA2 × nB 2 (1 - x ) (2 - x )
48. CO (g ) + 2H2 (g ) r CH3OH (g )
r

Mole : 0.2 – 0.10 x - 0.20 0.10 Þ Total moles = x 4 x2


Þ 50 = Þ 23x 2 - 75x + 50 = 0
ou
ad

2
4.92 ´ 5 x - 3x + 2
Þ x= = 0.5
0.082 ´ 600
Y

752 - 4 ´ 23 ´ 50
75 ±
Þ x= = 0.93, 2.32
Þ moles of H2 at equilibrium = x - 0.2 = 0.3 46
0.1 0.3
nd

Partial pressures : CO = p, H2 = 2.32 is not acceptable because x cannot be greater than 1.


Re

p,
0.5 0.5
Mole of AB = 2x = 2 ´ 0.93 = 1.86
0.1
CH3OH =
Fi

p pV 2.05 ´ 100
0.5 53. Total moles of gases at equilibrium = = = 5.0
p RT 0.082 ´ 500
5 25 25 Out of this 5 moles, 1.0 mole is for N2 (g ) and remaining 4 moles
Kp = 2
= 2= = 0.11 atm - 2
æ pö æ 3 ö 9p 9 (4.92)2 for PCl 5 and its dissociation products.
ç ÷ ç p÷ PCl 5 r PCl 3 + Cl 2
è 5ø è 5 ø
3-x x x
0.1 0.3
Concentrations : [CO] = M , [H2] = M, 3+ x = 4 Þ x =1
5 5
0.1 (0.1/ 5) 1
[CH3OH] = M Þ Kc = = 277.77 M -2. Degree of dissociation = = 0.33
5 (0.1/ 5) (0.3 / 5)2 3
54. N2 + 3H2 r 2NH3
49. 2SO2 (g ) + O2 (g ) r SO3 (g )
Initial : 1.0 3.0 0
Initial pi : 0 2 1 Equilibrium 1 – 0.25 3 – 0.75 0.05
Equilibrium pi : 2p 2+ p 1- 2p = 0.75 = 2.25
(1 - 2 p)2 0.75 2.25 0.50
Kp = 900 = [Ignoring p in comparison to 2] [N2 ] = , [H2 ] = , [NH3 ] =
(2 + p) (2 p)2 4 4 4
94 Chemical and Ionic Equilibrium

[NH3 ]2 (0.50)2 Topic 2 Ionic Equilibrium


Kc = 3
= ´ 16
[N2 ][H2 ] (0.75) (2.25)3
= 0.468 L2 mol - 2 Key Idea The concentration of substance in a saturated
1. solution is defined as its solubility (S). Its value depends
1 3 upon the nature of solvent and temperature.
Also for : N2 + H2 r NH3
2 2
Ax By - xA y + + yB x - Ksp = [ A y + ]x [ B x - ] y
K c¢ = K c = 0.68
. ´ 10- 5 M
Solubility of Cd (OH)2 (S ) = 184
55. X 2 (g ) 2 2 X (g ) Given, pH = 12 [for Cd (OH)2 in buffer solution]
At t=0 1 0 So, pOH = 2 (Q pH + pOH = pK w )
æ xö 12 + pOH = 14
At equilibrium ç1 - ÷ x (where, x = b eq )
pOH = 14 - 12 = 2

w
è 2ø
æ xö
ç1 - ÷ \ [ OH- ] = 10- 2 in buffer solution.
æ xö è 2ø
Total moles = ç1 + ÷ and Mole fraction, X 2 (g ) =
è 2ø æ xö For reaction Cd (OH)2 ¾® Cd 2 + + 2 OH-
ç1 + ÷ S S2

Flo
æ ö è 2ø 2+ - 2
ç x ÷ Ksp = [Cd ][ OH ]
X (g ) = ç ÷ and p = 2 bar
çç 1 + x ÷÷ . ´ 10- 5 )3
Ksp = (S )(2S )2 = 4 S 3 = 4 (184

ree
è 2ø æ xö
ç 1- ÷ Ksp = 24.9 ´ 10- 15
Partial pressure, pX 2 = ç 2 ÷. p and p = p × x

F
X
çç 1 + x ÷÷ æ xö
[Cd 2 + ] =
Ksp
ç1 + ÷
è 2ø è 2ø [ OH- ]2

or
24.9 ´ 10- 15
é
ur æ xöù
2
[Cd 2 + ] = = 24.9 ´ 10- 15 ´ 10 + 4
ê px / ç1 + 2 ÷ ú (10- 2 )2
ë è øû

f
\ K p = pX2 / pX 2 =
(1 - x / 2) Þ = 24.9 ´ 10- 11 M
p
ks
æ xö
ç1 + ÷ [Cd 2 + ]Þ 2.49 ´ 10- 10 M
Yo
è 2ø
oo
The expected solubility of Cd(OH)2 in a buffer solution of
4 px 2 8 b 2eq
= = pH = 12 is 2.49 ´ 10- 10 M.
eB

(4 - x 2 ) (4 - b 2eq )
2. Key Idea Concentration of substance in a saturated solution
4 px 2
56. (a) Kp = = px 2 (Q 4 > > > x) is defined as its solubility (S). Its value depends upon the
(4 - x 2 ) nature of solvent and temperature. For reaction,
r

1
ou

+
\ xµ + B-
ad

AB -A
p
K sp = [ A + ][ B - ]
Y

If p decreases, x increases. Equilibrium is shifted in the forward


side. Thus, statement (a) is correct.
Al(OH)3 - Al 3+ + 3OH-
(b) At the start of the reaction, Q = 0 where, Q is the reaction
nd

Initially 1 0 0
Re

quotient DG = DG ° + 2.303RT log Q


At equilibrium 1- S S 3S + 0.2
Since, DG °> 0, thus DG is -ve.
Fi

Hence, dissociation takes place spontaneously. NaOH ¾® Na + + OH -


0. 2 0. 2
Thus, (b) is correct.
4 ´ 2(0.7)2 Ksp of Al(OH)3 = 2.4 ´ 10-24 (Given)
(c) If we use x = 0.7 and p = 2 bar then K p =
[ 4 - (0.7)2 ]
Ksp = [ Al3+ ][ OH- ]3
Thus, (c) is incorrect. = 1.16 > 1
2.4 ´ 10-24 = [ S ][ 3S + 0.2 ]3 [Q 0.2 >> S ]
(d) At equilibrium, DG = 0
2.4 ´ 10-24 = [ S ][ 0.008 ]
\ DG ° = - 2.303RT log K p
Since, DG ° = + ve [ S ] = 3 ´ 10-22
Hence, K p < 1
3. Key Idea NH4Cl is a salt of weak base (NH4OH) and strong
Kp
KC = acid (HCl). On hydrolysis, NH4Cl will produce an acidic
(RT ) solution (pH < 7) and the expression of pH of the solution is
Then KC < 1. Thus, (d) is correct. 1
pH = 7 - (pK b + logC )
2
Chemical and Ionic Equilibrium 95

Given, K b (NH4OH) = 10-5 In statement (IV), Le-Chatelier’s principle is applicable to


-5 common ion effect. Because, in presence of common ion (given)
\ pK b = - log K b = - log(10 ) = 5 -
by strong electrolyte (say, Na + A), the product of the
C = concentration of salt solution = 0.02 M
= 2 ´ 10-2 M concentration terms in RHS increases. For the weaker
electrolyte, HA (say) the equilibrium shifts to the LHS,
1
Now, pH = 7 - (pK b + logC ) HA - HÅ + A s.
2
On substituting the given values in above equation, we get As a result dissociation of HA gets suppressed. Hence, the
option (d) is incorrect.
1
= 7 - [ 5 + log(2 ´ 10-2 )]
2 Key Idea The concentration of a substance in a saturated
5.
1 solution is defined as its solubility(S).
= 7 - [ 5 + log 2 - 2 ]
2

w
For Ax B y - xA y+ + yB x- ; K sp = [A y+ ] x [B x- ] y
1
= 7 - [ 5 + 0.301 - 2 ] = 7 - 165
. = 5.35
2 For, Zr3 (PO4 )4,
4. The explanation of given statements are as follows: Zr3 (PO4 )4 (s) -3Zr 4+ (aq) + 4 PO34- (aq)

Flo
3S M 4S M
In statement (I), millimoles of H+ = 400 ´ 01
. ´ 2 = 80 4+ 3
- K sp = [ Zr ] [ PO34- ]4
Millimoles of OH = 400 ´ 01 . = 40 (Limiting reagent)

ree
1
\ Millimoles of H+ left = 80 - 40 = 40 æ K sp ö 7
7
40 40 1 K sp = (3S ) (4 S ) = 6912 S or S = çç
3 4
÷÷
[ H+ ] = = M = è 6912 ø

F
M
400 + 400 800 20
Thus, the relation between molar solubility(S) and solubility
æ 1ö
Þ pH = - log[ H+ ] = - logç ÷ product (K sp ) will be

or
ur è 20 ø
æ K sp ö
1/ 7

= - log1 + log 2 + log10 S = çç ÷÷

f
= -0 + 0.301 + 1 è 6912 ø
ks
Þ 1.30 6. Let the solubility of Ag2CO3 is S. Now, 0.1 M of AgNO3 is
Yo
Hence, the option (a) is correct. added to this solution after which let the solubility of Ag2CO3
oo
In statement (II), ionic product of H2O is temperature becomes S ¢.
dependent. \ [Ag+ ] = S + 0.1 and [CO2-3 ]= S¢
eB

K w = [ H+ ][ OH- ] » 10-14 (mol / L)2 at 25ºC K sp = (S + 0.1)2 (S ¢ ) ...(i)


+ - 12
With increase in temperature, dissociation of H2O units into H Given, K sp = 8 ´ 10
r

and OH- ions will also increase. As a result, the value of ionic
Q K sp is very small, we neglect S ¢ against S in Eq. (i)
ou

product, [H+ ] ´ [ OH- ] will be increased. e.g.


ad

\ . )2 S ¢
K sp = (01
Y

Temperature K w (mol/L2) or 8 ´ 10- 12 = 0.01 S ¢


-14
5ºC 0186
. ´ 10 or S ¢ = 8 ´10- 12 ´ 102 = 8 ´ 10- 10 M
nd
Re

25ºC 1008
. ´ 10-14 Thus, molar solubility of Ag2CO3 in 0.1 M
45ºC 4.074 ´ 10 -14 AgNO3 is 8 ´ 10- 10 M.
Fi

Hence, the option (b) is correct. 7. The reaction takes place when H2SO4 is added to NH4OH is as
follows :
In statement (III), for a weak monobasic acid HA
H2SO4 + 2NH4OH ¾® (NH4 )2 SO4 + 2H2O
HA - HÅ + A s
Strong acid Weak base Salt of strong acid
(1 - a) C M aC M aC M
+ weak base
Þ pH of the solution is 5, i.e. Millimoles at t = 0 20 ´ 0.1 = 2 30 ´ 0.2 = 6 0
[H+ ] = 10-5 M = aC Millimoles at t = t 0 2 2
So, the resulting solution is a basic buffer
aC ´ aC 10-5 ´ a
Þ Ka = = [NH4OH + (NH4 )2 SO4 ].
(1 - a )C 1- a
According to the Henderson’s equation,
10-5 ´ a [(NH4 )2SO4 ]
Þ 10-5 = pOH = pK b + log
1- a [NH4OH]
Þ a = 0.5 2
Þ a % = 50 = 4.7 + log = 4.7
2
Hence, the option (c) is correct. Þ pH = 14 - pOH = 14 - 4.7 = 9.3
96 Chemical and Ionic Equilibrium

8. Its given that the final volume is 500 mL and this final volume SiCl 4 - Although this compound does not have incomplete octet
was arrived when 50 mL of 1 M Na 2SO4 was added to unknown but it shows the tendency to accept lone pair of electrons in its
Ba 2+ solution. vacant d-orbital. This tendency of SiCl 4 is visible in following
reaction.
So, we can interpret the volume of unknown Ba 2+ solution as
450 mL i.e. Cl Cl
450mL + 50mL ¾® 500mL H
Ba 2+ Na 2SO 4 BaSO 4 Si + H2O Si O
solution solution solution
Cl Cl Cl Cl H
From this we can calculate the concentration of SO2- Cl Cl
4 ion in the
solution via Lone pair acceptance
M 1V1 = M 2V2 in d-orbital
1 ´ 50 = M 2 ´ 500 Cl

w
(as 1M Na 2SO4 is taken into consideration) Si OH + HCl
1 Cl
M2 = = 01
. M Cl
10
Thus, option (b) and (d) both appear as correct but most suitable

Flo
Now for just precipitation,
answer is (d) as the condition of a proper Lewis acid is more well
Ionic product = Solubility product (K sp )
defined in BCl 3 and AlCl 3.
i.e. [Ba 2+ ][SO24- ] = K sp of BaSO4

ree
10. Among the given salts
Given K sp of BaSO4 = 1 ´ 10-10 FeCl 3 is acidic in nature i.e., have acidic solution as it is the salt

F
So, [Ba 2+ ][0.1] = 1 ´ 10-10 of weak base and strong acid.
Al(CN) 3 and Pb(CH 3COO) 2 are the salts of weak acid and
or [Ba 2+ ] = 1 ´ 10-9 M weak base.

or
ur
Remember This is the concentration of Ba 2+ ions in final CH 3COOK is the salt of strong base and weak acid.
solution. Hence, for calculating the [Ba 2+ ] in original solution

f
Hence, the solution of CH3COOK will be most basic because of
we have to use the following reaction.
ks
M 1V1 = M 2V2 CH3COOK + H2O - CH3COOH + KOH
Yo
as M 1 ´ 450 = 10-9 ´ 500 (Weak acid) (Strong base)
oo
so, M 1 = 1.1 ´ 10-9 M 11. For a salt of weak acid and weak base,
eB

1 1
9. Key Idea Lewis acids are defined as, pH = 7 + pKa - pKb
2 2
‘‘Electron deficient compounds which have the ability to
accept atleast one lone pair.’’ Given, pK a (HA ) = 3.2, pK a ( BOH) = 3.4
r

The compound given are 1 1


ou

\ pH = 7 + (3.2) - (3.4)
ad

PH 3-Octet complete although P has vacant 3d-orbital but does 2 2


Y

not have the tendency to accept lone pair in it. Hence, it cannot
= 7 + 1.6 - 1.7 = 6. 9
be considered as Lewis acid.
BCl 3-Incomplete octet with following orbital picture. 12. pH = 1 \ [ H+ ] = 10-1 = 0.1 M
nd
Re

1s 2s 2p Vacant pH = 2 \ [ H+ ] = 10-2 = 0.01 M


p- orbital
For dilution of HCl, M 1V1 = M 2V2
Fi

B-
0.1 ´ 1 = 0.01 ´ V2
Used in bond V2 = 10 L
formation with
Cl having one electron Volume of water to be added = 10 - 1 = 9 L
each from B and Cl
13. MX : K sp = S 2 = 4 ´ 10- 8
Hence, vacant p-orbital of B can accept one lone pair thus it can
be considered as Lewis acid. Þ S = 2 ´ 10- 4
AlCl 3-Similar condition is visible in AlCl3 as well i.e. MX 2 : K sp = 4 S 3 = 3.2 ´ 10- 14 Þ S = 2 ´ 10- 5
Al ( Valence orbital only) =
M3X : K sp = 27S 4 = 2.7 ´ 10- 15 Þ S = 10- 4
Vacant p-orbital
3s 3p 3d vacant Order of solubility is MX > M 3 X > MX 2
2
14. mmol of base = 2.5 ´ = 1
Used in bond 5
formation with Cl
mmol of acid required to reach the end point = 1
Hence this compound can also be considered as Lewis acid.
Chemical and Ionic Equilibrium 97

15 After neutralisation, 10 mmol HCl will be remaining in 100 mL


Volume of acid required to reach the end point = mL
2 of solution.
15 10
Total volume at the end point = + 2.5 = 10 mL Molarity of HCl in the final solution = = 0.10
2 100
1 pH = - log [H+ ] = - log (0.10) = 1
Molarity of salt at the end point = = 0.10
10
22. In case of hydroxides of Group II A, solubility increases down
B+ + H2O r BOH + H+ the group. Therefore, Be(OH)2 is least soluble, has lowest value
C (1 - a ) Ca Ca
of K sp .
Kw
Kh = = 10-2 23. HClO4 is the strongest acid among these.
Kb
Ca 2 0.1 a 2 24. For precipitation to occur, K sp < Qsp .
K h = 10- 2 = =

w
1- a 1- a æ 10- 4 ö æ 10- 4 ö -9
2
Qsp = çç ÷
÷
ç ÷
ç 2 ÷ = 2.5 ´ 10 > K sp
Þ 10 a + a - 1 = 0 è 2 ø è ø
- 1 + 1 + 40 Hence, precipitate will be formed in this case. In all other case,
Þ a= = 0.27

Flo
20 Qsp < K sp and no precipitation will occur.
Þ [H+ ] = Ca = 0.1 ´ 0.27 = 0.027 M
25. In stomach, pH is 2-3, i.e. strongly acidic and aspirin will be

ree
15. CH3NH2 + HCl ¾® CH3NH+3 + Cl - almost unionised here due to common ion effect. However, pH
Initial : 0.10 0.08 0 0 in small intestine is 8, basic, aspirin will be neutralised here.

F
Final : 0.02 0 0.08 0.08 26. BeCl 2 exist in polymeric forms and has no electron deficiency,
[CH3NH3+
] not a Lewis acid.
pOH = pK b + log

or
Cl Cl Cl
ur
[CH3NH2 ]
0.08 Be Be Be
-4
= - log (5 ´ 10 ) + log = 3.9

f
0.02 Cl Cl Cl
ks
pH = 14 – pOH = 10.1 27. NH-2 + H2O r NH3 + OH-
[H+ ] = 8 ´ 10-11
Yo
Base Conjugate
oo
acid

K w 10- 14 Kh 10- 9
16. K h ( X - ) = = - 5 = 10- 9 Þ a = = = 10- 4 28. When a weak acid (HX) is titrated against a strong base
eB

Ka 10 C 0.10 NaOH, basic salt (NaX) is present at the end point which
% hydrolysis = 100 a = 0.01 makes end point slightly basic with pH around 8. Hence,
phenolphthalein, that changes its colour in this pH range,
17. Minimum S2- concentration would be required for precipitation
r

would be the best choice of indicator to detect the end point.


ou

of least soluble HgS.


ad

For HgS, S2- required for precipitation is 29. The reaction of HA with strong base is
Y

K sp 10-54 HA + OH- r H2O + A -


[S2- ] = = = 10-51 M
[Hg2+ ] 10-3 [ A- ] [H+ ] K a 10- 4
K = ´ = = = 1010
nd

[HA ][OH- ] [H+ ] K w 10- 14


Re

18. Alkali metal salts are usually more soluble than the salts of
transition metals. Also, CuS is less soluble than ZnS because of Kw
= 10- 4
Fi

3d 9 configuration of Cu 2+ . Therefore, solubility order is 30. K a (HX ) =


Kb
Na 2S > ZnS > CuS
[X -]
pH = pK a + log
19. Ap Bq r pA + qB [HX ]
pS qS
Þ pK a = 4 [Q [ X - ] = [HX ]]
K sp = ( pS ) (qS )q = S (p + q) × pp × qq
p

31. For precipitation reaction, QIP > K sp .


20. NaCN is basic salt, has highest pH while HCl has lowest pH. 2
NaCl is neutral salt has pH = 7 while NH4Cl is acidic salt, has pH æ 10- 2 ö æ 10- 3 ö
less than 7. QIP = [Ca 2+ ][F- ]2 = çç ÷ ´ç
÷ ç
÷
÷
è 2 ø è 2 ø
pH : HCl < NH4Cl < NaCl < NaCN
M = 1.25 ´ 10- 9 > K sp , precipitate will be formed.
21. 75 mL HCl = 15 mmol HCl
5
32. Acidic buffer is prepared by mixing weak acid with salt of its
M conjugate base. Therefore, acetic acid and sodium acetate can be
25 mL NaOH = 5 mmol NaOH
5 used to prepare acidic buffer.
98 Chemical and Ionic Equilibrium

33. The order of acidic strength of conjugate acids is K a = 1´10-8 Þ Ksp = 2 ´10-10
HOCl < HClO2 < HClO3 < HClO4 after putting the values in above formula
Reverse is the order of basic strength of their conjugate base, i.e. æ 10-3 ö
Solubility = 2 ´ 10-10 çç -8 + 1÷÷ » 2 ´10-5 = 4.47 ´ 10-3M
ClO- is the strongest base.
è 10 ø
34. K w = [H3O+ ][OH- ] = 10- 6 ´ 10- 6 = 10- 12 Hence, the value of y= 4.47
35. No matter, what is the concentration of HCl, its pH will always 41. For P, i.e. (10 mL of 0.1 M NaOH + 20 mL of 0.1 M acetic
be less than 7 at 25°C. In the present case, the solution is very acid) is diluted to 60 mL
dilute, pH will be between 6 and 7. The correct match is 1, i.e. the value of [H+] does not change on
36. PLAN In presence of common ion (in this case Ag + ion) solubility of dilution due to the formation of following buffer.
sparingly soluble salt is decreased. - +
NaOH + CH 3COOH - CH COO Na + H 2O

w
3
Let solubility of Ag2CrO4 in presence of 0.1 M
Initial millimol 1 2
AgNO 3 = x
Final millimol 1 1
Ag2CrO4 a 2 Ag+ + CrO24- Final volume – 30 mL (20 + 10) in which millimoles of

Flo
2x x
CH3COOH and CH3COO -Na + are counted.
AgNO3 a Ag+ + NO3-
0.1 0.1 For Q, i.e. (20 mL of 0.1 M NaOH + 20 mL of 0.1 M

ree
+
Total [ Ag ] = (2x + 0.1) M » 0.1 M CH3COOH) is diluted to 80 mL
as x <<< 0.1 M The correct match is 5, i.e. the value of [H+] changes to

F
[CrO 2-
4 ]= xM 2 times of its initial value on dilution.
Thus, [Ag+ ]2 [CrO24- ] = K sp As per the condition given in Q the resultant solution before
dilution contain 2 millimoles of CH3COO -Na+ in 40 mL

or
ur
(0.1)2 (x ) = 1.1 ´ 10-12
solution. Hence, it is the salt of weak acid and strong base. So,

f
Q x = 1.1 ´ 10-10 M KW K a
[H + ]initial =
ks
37. In HNO3 and CH3COONa combination, if HNO3 is present in C
Yo
limiting amount, it will be neutralised completely, leaving C
After dilution to 80 mL, the new ‘C ’ becomes , So,
oo
behind some excess of CH3COONa. 2
K wK a
eB

CH3COONa + HNO3 ¾® CH3COOH + NaNO3 [H ]new =


+
or [H ] initial ´ 2
+
C /2
Buffer combination For R, i.e. (20 mL of 0.1 M HCl + 20 mL of 0.1 M NH 3) is
r

38. CH3COOH + CH3COONa = Buffer solution diluted to 80 mL


ou

1
ad

CH3COONa + HCl ¾® CH3COOH + NaCl The correct match is 4, i.e. the value of[H+] changes to times
2
Y

If HCl is taken in limited quantity, final solution will have both of its initial value of dilution.
CH3COOH and CH3COONa needed for buffer solution.
As per the condition given in R the resultant solution before
Ammonia and ammonium chloride forms basic buffer. dilution contains 2 millimoles of NH4Cl in 40 mL of solution.
nd
Re

Hence, a salt of strong acid and weak base is formed.


39. pH of 10- 8 M solution will be between 6 and 7 but never 8. The
For this,
Fi

conjugate base of an acid is formed by removing a proton (H+ )


- Kw ´ C
from acid. Therefore, HPO2-
4 is a conjugate base of H 2PO 4 . [H+]initial =
Kb
H2O r H+ + OH- DH > 0 Now on dilution upto 80 mL new conc. becomes C /2.
Increasing temperature will increase equilibrium constant of the C
Kw ´
above endothermic reaction. 2
So, [H ]new =
+

At the mid-point of titration pH = pK a Kb


1
40. Key Idea Solubility of salt of weak acid (AB) in presence or [H+]new = [H + ]initial ´
+
of H ions from buffer solution can be calculated with the 2
help of following formula. For S , i.e. 10 mL saturated solution of Ni(OH)2 in
é[H+ ] ù equilibrium with excess solid Ni(OH)2 is diluted to 20 mL
Solubility = K sp ê + 1ú and solid Ni(OH)2 is still present after dilution.
ë ka û
The correct match is 1.
Given, pH = 3, so [H+ ] = 10-3 Ni(OH)2 (s) - Ni 2+
+ 2OH-
Chemical and Ionic Equilibrium 99

as per the condition given it is a sparingly soluble salt. Hence, on 53. At the end-point, [ A - ] = 0.05
dilution the concentration of OH- ions remains constant in
saturated solution. K b ( A - ) = K w / K a = 2 ´ 10- 9
So for this solution,
[OH- ] = K bC = 2 ´ 10- 9 ´ 0.05 = 10- 5
[H+]new =[H+]initial
pOH = 5 and pH = 9
42. I2 : I- ¾® I2 = I3- -
54. (i) CH3COOH r CH3COO + H+
43. Hydration energy facilitate solubility. C (1 - a ) Ca Ca
44. Amphoteric If no HCl is present,
0.2
45. SO2–
4 Conjugate base is formed by removing a proton from acid. [HCl] = = 0.10 M
2

w
46. P2O5 is strongest acid and MgO is strongest base. [CH3COOH] = 0.10 M
The major contributor of H+ in solution is HCl.
47. NaOH + H2O ¾® NaOH (aq); DH < 0
C a (0.1)
Ka = = 1.75 ´ 10- 5

Flo
48. Lewis acid accept lone pair of electron. C (1- a )
^m a = 1.75 ´ 10- 4
49. Degree of ionisation (a) =

ree
6
X (ii) mmol of NaOH added = ´ 1000 = 150
Let ^ m (HY ) = x Þ ^ m (HX ) = 40
10

F
mmol of HCl = 500 ´ 0.2 = 100
^ m (HX ) 1 a (HX )
Þ = = [Q ^ ¥ (HX ) = ^ ¥ (HY )] mmol of CH3COOH = 500 ´ 0.2 = 100
^ m ( XY ) 10 a (HY )

or
After neutralisation, mmol of CH3COOH = 50
Also : K a(HX ) = (0.01) [ a( HX )]2
ur ...(i) mmol of CH3COONa = 50

f
K a (HY ) = (010
. ) [ a( HY )] 2
pH = pK a = 4.75
ks
= 010
. [10 a (HX )]2 = 10 [ a (HX )]2 ... (ii) 55. Partial pressure of SO2 in air = 10- 5 atm
Yo
oo
K a (HX ) 0.01 1 [SO2 ]aq = 1.3653 ´ 10- 5 mol L- 1
Þ = =
K a (HY ) 10 1000
Q pK a = 1.92 and concentration of H2SO3 is very low, it is
eB

Þ log K a (HX ) - log K a (HY ) = - 3 almost completely ionised as


Þ - log K a (HX ) - [ - log K a (HY )] = 3 H2SO3 r H+ + HSO-3
Þ pK a (HX ) - pK a (HY ) = 3
r

[H+ ] = 1.3653 ´ 10- 5 M


ou
ad

50. It is a case of simultaneous solubility of salts with a common ion. pH = – log (1.3653 ´ 10- 5 ) = 4.86
Here, solubility product of CuCl is much greater than that of
Y

AgCl, it can be assumed that Cl– in solution comes mainly from 56. In water, K sp = 4 S 3 = 4 (6.7 ´ 10- 6 )3
CuCl.
= 1.2 ´ 10- 15
nd
Re

[ Cl – ] = K sp (CuCl) = 10–3 M
In buffer of pH = 8, pOH = 6, [OH- ] = 10-6
–10 + –
Now, for AgCl, K sp = 1.6 × 10 = [Ag ] [Cl ]
Fi

K sp = S [OH- ] 2
+ –3
= [Ag ] × 10
1.2 ´ 10- 15
Þ [Ag+] = 1.6 × 10–7 S = = 1.2 ´ 10- 3 M
10- 12
51. Basic salts solution will have pH > 7, will change colour of
[Ag+ ] anode
litmus paper red to blue 57. (a) E = 0.164 = – 0.059 log
0.10
KCN, K2CO3 and LiCN are the only basic salts among these.
+ -4
[Ag ] anode = 1.66 ´ 10 M
52. The hydrolysis reaction is
A - + H2O r AH + OH- [Ag+ ]
[CrO24- ] = = 8.3 ´ 10- 5 M
2
Kw
Kh = = 10- 10 K sp = [Ag+ ]2 [CrO24- ]
Ka
[OH- ] = K hC = 10- 6 = (1.66 ´ 10- 4 )2 (8.3 ´ 10- 5 )

pOH = 6 and pH = 8 = 2.3 ´ 10- 12


100 Chemical and Ionic Equilibrium

(b) pH of HCl = 2 61. CN- + H2O r HCN + OH-


\ [ H+ ] = 10-2 M K h = 2 ´ 10- 5
+ -2
Moles of H ions in 200 mL of 10 M HCl solution
[OH- ] = K hC = 2 ´ 10- 5 ´ 0.5 = 10- 5
10-2
= ´ 200 = 2 ´ 10-3 pOH = 2.5 and pH = 11.5
1000
62. For salts of weak acid and weak base .
Similarly, pH of NaOH = 12 1 1
pH = 7 + (pK a - pK b ) = 7 + (3.8 - 4.8) = 6.50
\ [ H+ ] = 10-12 M 2 2
or [OH- ] = 10-2 M [Q [H+ ][ OH- ] = 10-14 m] 63. Ag+ + 2CN- r Ag(CN)-2
Moles of OH- ion in 300 mL of 10-2 M NaOH solution Initial : 0.03 0.10 0

w
Equilibrium : x 0.10 – 0.06 0.03
10-2
= ´ 300 = 3 ´ 10-3 1
1000 K = = 2.5 ´ 1018
4 ´10- 19
Total volume of solution after mixing = 500 mL

Flo
0.03
Moles of OH- ion left in 500 mL of solution Þ K = 2.5 ´ 1018 =
(0.04)2 x
= (3 ´ 10-3 ) - (2 ´ 10-3 ) = 10-3
x = 7.50 ´ 10- 18 M Ag+

ree
Molar concentration of OH- ions in the resulting
10-3 64. For H2S, H2S r 2H+ + S2-
solution = ´ 1000 = 2 ´ 10-3 M

F
500 K = K 1 ´ K 2 = 1.3 ´ 10- 20
pOH = - log (2 ´ 10-3 ) Minimum [S2- ] required to begin precipitation of

or
= - log 2 + 3 log10
ur 6 ´ 10- 21
~ 103 = 2.699
= - 0.3 - MS = = 1.2 ´ 10- 19

f
0.05
\ pH = 14 - 2.699 = 11.301
[H+ ]2 [S2- ] (1.2 ´ 10- 19 )
ks
K = 1.3 ´ 10- 20 = = [H+ ] 2
58. 2AgCl (s) + CO23- r Ag2CO3 (s) + 2Cl - [H2S] 0.10
Yo
oo
[Cl - ]2 [Cl - ]2 [Ag+ ]2 [ K sp (AgCl)]
2 [H+ ] = 0.10 M Þ pH = 1
K = = ´ =
[CO32- ] [CO23- ] [Ag+ ]2 K sp (Ag2CO3 )
eB

65. Mixing H2CO3 with NaHCO3 results in buffer solution.


0.0026 [NaHCO3 ] n (NaHCO3 )
[Cl - ] = M = 7.3 ´ 10- 5 M pH = pK a + log = pK a + log
35.5 [H2CO3 ] n (H2CO3 )
r

The above concentration of Cl - indicates that [CO2-


3 ] remains
ou

x
7.4 = – log (7.8 ´ 10- 7 ) + log
ad

Þ
almost unchanged. 20
Y

2
7.3 ´ 10- 5 [K sp (AgCl)] Þ x = 400 mmol
=
1.5 8.2 ´ 10- 12 NaHCO3 = 5 ´ V Þ V = 80 mL
nd

K sp (AgCl) = 2 ´ 10- 8
Re

3 -5
66. K sp = 4 S = 4.42 ´ 10
-
[In ] S = 0.022 M
Fi

59. pH = pK In + log 10 = pK In + 1 When = 10


[HIn]
mmol of Ca(OH)2 in 500 mL saturated solution = 11
[In - ]
= pK In + log (0.1) = pK In -1 When = 0.1 mmol of NaOH in 500 mL 0.40 M solution = 200
[HIn]
Total mmol of OH- = 200 + 2 ´ 11 = 222
pH range is pK In -1 to pK In + 1.

60. K a (NH+4 ) = 5.6 ´ 10- 10 [OH- ] = 0.222 M


K sp
10- 14 Solubility in presence of NaOH =
K b (NH3 ) = K w / K a = = 1.8 ´ 10- 5 [OH- ]2
5.6 ´ 10- 10
4.42 ´ 10- 5
k1
- = = 9 ´ 10- 4 M
i.e. NH3 + H2O r NH+4 + OH (0.222)2
k2
k1 mmol of Ca 2+ remaining in solution = 0.9
K = = 1.8 ´ 10- 5
k2 mmol of Ca(OH)2 precipitated = 10.1
-5 10 5 mg of Ca(OH)2 precipitated = 10.1 ´ 7.4 = 747.4 mg
k 1 = Kk 2 = 1.8 ´ 10 ´ 3.4 ´ 10 = 6.12 ´ 10
Chemical and Ionic Equilibrium 101

67. Let 40 mL of base contain x mmol of BOH. 1.8 ´ 10- 5 a


K a = 1.8 ´ 10- 5 =
BOH + HCl ¾® BCl + H2O 1- a
x - 0.5 0.5 When 5 mL acid is added a = 0.5 and x = 3.6 ´ 10- 5 M
x- 2 2.0 When 20 mL of acid is added Volume (final) = 1/3.6 ´ 10- 5 = 27.78 ´ 103 L.
When pH is 10.04, pOH = 3.96 and when pH is 9.14, pOH is
[NH+4 ]
4.86. Therefore, 70. pOH of buffer solution = pK b + log
[NH4OH]
0.50
3.96 = pK b + log …(i)
x - 0.5 0.25
= - log (1.8 ´ 10- 5 ) + log = 5.44
2.0 0.05
3.96 = pK b + log …(ii)
x- 2 [OH- ] = 3.6 ´ 10- 6 M
6 ´ 10- 32

w
Subtracting Eq. (i) from Eq. (ii) gives K sp
[Al 3+ ] = - 3
= = 1.28 ´ 10- 15 M
æ 2 x - 0.5 ö [OH ] (3.6 ´ 10- 6 )3
0.90 = log çç ´ ÷
èx - 2 0.5 ÷ø K sp 8.9 ´ 10- 12
[Mg2+ ] = = = 0.68 M

Flo
4 (x - 0.5) [OH- ]2 (3.6 ´ 10- 6 )2
Þ 28 =
x-2
71. HCN for buffer will be formed by the reaction

ree
Þ x = 3.5, substituting in equation (i) gives NaCN + HCl ¾® NaCl + HCN
0.5 0.01
3.96 = pK b + log mmol of NaCN present initially = ´ 1000 = 0.2

F
3 49
K b = 1.8 ´ 10- 5
Let x mmol of HCl is added so that x mmol of NaCN will be

or
68. Initial concentration of K2C2O4 =
ur
0.152
= 0.304 M, neutralised forming x mmol of HCN.
0.50 [NaCN]

f
pH = pK a + log
Also for the following equilibrium: [HCN]
ks
Ag2CO3 (s) + K2C2O4 (aq) r Ag2C2O4 (s) + K2CO3
0.2 - x
8.5 = - log (4.1 ´ 10- 10 ) + log
Yo
0.304 - x x
oo
x
[CO23- ] [Ag+ ]2 K sp (Ag2CO3 )
K = ´ = x = 0.177 mmol
[C2O42- ] [Ag+ ]2 K sp (Ag2C2O4 )
eB

72. (i) 0.20 mole HCl will neutralise 0.20 mole CH3COONa,
Given, 0.304 – x = 0.0358 producing 0.20 mol CH3COOH. Therefore, in the solution
Þ x = 0.2682 moles of CH3COOH = 1.20
r

0.2682 Moles of CH3COONa = 0.80


Þ K = = 7.5
ou
ad

0.0358 [Salt]
pH = pK a + log
K sp (Ag2CO3 ) = K ´ K sp (Ag2C2O4 ) [Acid]
Y

= 7.5 ´ 1.29 ´ 10- 11 (0.80)


= - log (1.8 ´ 10- 5 ) + log = 4.56
= 9.675 ´ 10- 11 (1.20)
nd
Re

(ii) CH3COONa + HCl ¾® CH3COOH + NaCl


69. CH3COOH r CH3COO- + H+
Initial 0.10 0.20 0 0
Fi

When concentration of CH3COOH is 1.0 M, ‘a’ is negligible, Final 0 0.10 0.10 0.10
Now, the solution has 0.2 mole acetic acid and 0.1 mole HCl.
[H+ ] = K aC = 4.24 ´ 10- 3 M
Due to presence of HCl, ionisation of CH3COOH can be
pH = - log (4.24 ´ 10- 3 ) = 2.37 ignored (common ion effect) and H+ in solution is mainly
due to HCl.
Now, let us assume that solution is diluted to a volume where
concentration of CH3COOH (without considering ionisation) [H+ ] = 0.10
is x. pH = – log (0.10) = 1.0
CH3COOH r CH3COO- + H+ 9.57
x (1 - a ) xa xa 73. In pure water, solubility = ´ 10- 3 M
58
xa2
Ka = = 1.65 ´ 10- 4 M
1- a
K sp = 4 S 3 = 4 (1.65 ´ 10- 4 )3 = 1.8 ´ 10- 11
Also, desired pH = 2 ´ 2.37 = 4.74
[H+ ] = 1.8 ´ 10- 5 = x a In 0.02 M Mg(NO3 )2;
102 Chemical and Ionic Equilibrium

K sp 1 pH = 4.15
solubility of Mg(OH)2 = ´
[Mg2+ ] 2 Kw 10- 14
[OH- ] = =
[H+ ] 7 ´ 10- 5
= 1.5 ´ 10- 5 mol L- 1
= 1.5 ´ 10- 5 ´ 58 g L- 1 = 1.43 ´ 10- 10 M

= 8.7 ´ 10- 4 g L- 1 77. Sodium acetate (CH3COONa) is a basic salt (salt of strong base
and weak acid) therefore, its aqueous solution has pH > 7.
74. HCOOH r H+ + HCOO-
78. mmol of NaOH = 20 ´ 0.2 = 4
HCOONa r Na + + HCOO-
1 - 0.75 0.75 mmol of acetic acid = 50 ´ 0.2 = 10
After neutralisation, buffer solution is formed which contain 6
In the above buffer solution, the significant source of formate

w
ion (HCOO- ) is HCOONa. Hence, mmol CH3COOH and 4 mmol CH3COONa.
[CH3COONa]
K a = 2.4 ´ 10- 4 pH = pK a + log
[CH3COOH]
[H+ ] (0.75)

Flo
4
= = - log (1.8 ´ 10- 5 ) + log = 4.56
[HCOOH] 6
2.4 ´ 10- 4 ´ 0.20 Now, let x mmol of NaOH is further added so that pH of the

ree
[H+ ] = = 6.4 ´ 10- 5 resulting buffer solution is 4.74.
0.75
pH = – log (6.4 ´ 10-5 ) = 4.20 Now, the buffer solution contains (4 + x ) mmol CH3COONa

F
and (6 - x ) mmol of CH3COOH.
76. K sp (AgI) = 8.5 ´ 10- 17 = [Ag+ ][I- ] 4+x
4.74 = - log (1.8 ´ 10- 5 ) + log

or
ur
[I- ] required to start precipitation of AgI
6-x

f
8.5 ´ 10- 17 4+x
= = 8.5 ´ 10- 16 M Þ =1
6-x
ks
0.10
(HgI2 ) = 2.5 ´ 10- 26 = [Hg2+ ][I- ]2 Þ x = 1.0 mmol = 0.2 ´ V
Yo
K sp
oo
-
[I ] required to start precipitation of HgI2 Þ V = 5.0 mmol NaOH.
eB

2.5 ´ 10- 26 79. For acidic buffer, the Henderson’s equation is


= = 5 ´ 10- 13 M
0.10 (mole of salt)
pH = pK a + log
- (mole of acid)
The above calculation indicates that lower [I ] is required for
r

precipitation of AgI. When [I- ] reaches to 5 ´ 10- 13 , AgI gets x


ou

4.75 = – log (1.34 ´ 10- 5 ) + log


ad

precipitated almost completely. 0.02


Þ x = 0.015 mole of sodium propionate.
Y

When HgI2 starts precipitating,


8.5 ´ 10- 17 Addition of 0.01 mole HCl will increase moles of propionic acid
[Ag+ ] = = 1.70 ´ 10- 4 M
5 ´ 10- 13 by 0.01 and moles of sodium propionate will decrease by same
nd
Re

amount.
1.70 ´ 10- 4 ´ 100
% Ag+ remaining = = 0.17 New moles of acid = 0.02 + 0.01 = 0.03
Fi

0.10
% Ag+ precipitated = 100 – 0.17 = 99.83 New moles of salt = 0.015 – 0.01 = 0.005
æ 0.005 ö
76. Molarity (C ) = 0.10 pH = – log (1.34 ´ 10- 5) + log ç ÷ = 4.09
è 0.030 ø
[H+ ] = K a × C = 7 ´ 10- 5 M (a is negligible) pH of 0.01 HCl = 2, just half of the pH of final buffer solution.

Download Chapter Test


http://tinyurl.com/y3x893rn or
7
Thermodynamics and
Thermochemistry

w
Flo
Topic 1 Thermodynamics

ree
Objective Questions I (Only one correct option) (a) Cyclic process : q =- W
(b) Adiabatic process : DU = - W

F
1. An ideal gas is allowed to expand from 1 L to 10 L against a
constant external pressure of 1 bar. The work done in kJ is (c) Isochoric process : DU = q
(2019 Main, 12 April I) (d) Isothermal process : q = - W

or
(a) - 9.0 (b) + 10.0
ur
(c) - 0.9 (d) - 2.0 8. For silver, C p ( J K -1 mol -1 ) = 23 + 0.01 T. If the

f
2. The difference between DH and DU ( DH - DU ), when the temperature (T) of 3 moles of silver is raised from 300 K to
combustion of one mole of heptane (l) is carried out at a 1000 K at 1 atm pressure, the value of DH will be close to
ks
temperature T, is equal to (2019 Main, 8 April I)
Yo
(2019 Main, 10 April II)
(a) - 4 RT (b) 3 RT (d) - 3 RT (a) 62 kJ (b) 16 kJ
oo
(c) 4 RT
(c) 21 kJ (d) 13 kJ
3. A process will be spontaneous at all temperature if
eB

(2019 Main, 10 April I) 9. For a diatomic ideal gas in a closed system, which of the
(a) DH > 0 and DS < 0 (b) DH < 0 and DS > 0 following plots does not correctly describe the relation
(c) DH < 0 and DS < 0 (d) DH > 0 and DS > 0 between various thermodynamic quantities?
r

(2019 Main, 12 Jan I)


4. During compression of a spring the work done is 10 kJ and 2
ou
ad

kJ escaped to the surroundings as heat. The change in


internal energy, DU (in kJ) is Cp Cv
Y

(2019 Main, 9 April II) (a) (b)


(a) 8 (b) -12 (c) 12 (d) -8
5. Among the following the set of parameters that represents p T
nd
Re

path functions, is (2019 Main, 9 April I)


(A) q + W (B) q (C) W (D) H - TS
Fi

(c) U (d) Cv
(a) (A) and (D) (b) (A), (B) and (C)
(c) (B), (C) and (D) (d) (B) and (C) T V
6. 5 moles of an ideal gas at 100 K are allowed to undergo -
O
reversible compression till its temperature becomes 200 K. 10. The standard electrode potential E and its temperature
-
æ dE O ö
If CV = 28 JK -1 mol -1 , calculate DU and DpV for this coefficient ç ÷ for a cell are 2V and - 5 ´ 10-4 VK -1 at
ç dT ÷
process. ( R = 8.0 JK -1 mol-1 ) (2019 Main, 8 April II) è ø
(a) DU = 2.8 kJ; D( pV ) = 0.8 kJ 300 K respectively. The cell reaction is
(b) DU = 14 J; D( pV ) = 0.8 J Zn( s ) + Cu2 + ( aq ) ® Zn2 + ( aq ) + Cu( s )
(c) DU = 14 kJ; D( pV ) = 4 kJ -
The standard reaction enthalpy (Dr H O ) at 300 K in kJ mol -1
(d) DU = 14 kJ; D( pV ) = 18 kJ
is, [Use, R = 8 JK -1 mol-1 and F = 96,000 C mol-1 ]
7. Which one of the following equations does not correctly
(2019 Main, 12 Jan I)
represent the first law of thermodynamics for the given
(a) - 412.8 (b) - 384.0
processes involving an ideal gas ? (Assume non-expansion
work is zero) (2019 Main, 8 April I)
(c) 206.4 (d) 192.0
104 Thermodynamics and Thermochemistry

11. The reaction, MgO( s ) + C( s ) ® Mg( s ) + CO( g ), 18 The entropy change associated with the conversion of 1 kg
for which Dr H º = + 491.1 kJ mol- 1 and of ice at 273 K to water vapours at 383 K is
-1 -1
Dr S º = 198.0 JK mol , is not feasible at 298 K. (Specific heat of water liquid and water vapour are 4.2 kJ
K -1kg-1 and 2.0 kJK -1 kg-1; heat of liquid fusion and
Temperature above which reaction will be feasible is
(a) 2040.5 K (b) 1890.0 K vapourisation of water are 334 kJ kg-1 and 2491 kJkg-1
(c) 2380.5 K (d) 2480.3 K respectively). (log 273 = 2.436, log 373 = 2.572,
12. The standard reaction Gibbs energy for a chemical reaction log 383 = 2.583 ) (2019 Main, 9 Jan II)
-1 -1
at an absolute temperature T is given by, Dr G º = A - BT (a) 9.26 kJ kg K (b) 8.49 kJ kg-1K -1
Where A and B are non-zero constants. (c) 7.90 kJ kg-1K -1 (d) 2.64 kJ kg-1K -1
Which of the following is true about this reaction?
19 Consider the reversible isothermal expansion of an ideal gas

w
(2019 Main, 11 Jan II)
in a closed system at two different temperatures T1 and T2
(a) Endothermic if, A < 0 and B > 0
(T1 < T2 ). The correct graphical depiction of the dependence
(b) Exothermic if, B < 0
of work done (W) on the final volume (V) is
(c) Exothermic if, A > 0 and B < 0

Flo
(2019 Main, 9 Jan I)
(d) Endothermic if, A > 0 T2 T2
|W| |W|
13. For the chemical reaction, X - Y , the standard reaction T1

ree
T1
Gibbs energy depends on temperature T (in K) as
3 (a) (b)
DrG ° (in kJ mol–1 ) = 120 - T

F
8
The major component of the reaction mixture at T is O ln V O ln V

or
ur (2019 Main, 11 Jan I)
(a) Y if T = 280 K (b) X if T = 350 K |W| T2 |W|

f
T2
(c) X if T = 315 K (d) Y if T = 300 K
T1
ks
T1
14. Two blocks of the same metal having same mass and at
Yo
temperature T1 and T2 respectively, are brought in contact (c) (d)
oo
with each other and allowed to attain thermal equilibrium at
constant pressure. The change in entropy, DS , for this O
eB

ln V O ln V
process is (2019 Main, 11 Jan I)
é (T + T2 )1/ 2 ù
éT1 + T2 ù 20. The combustion of benzene ( l ) gives CO2 ( g ) and H2 O( l ).
(a) 2 C p lnê 1 2
ú (b) C p lnê ú
êë T T úû ë 4T1T2 û Given that heat of combustion of benzene at constant volume
r

1 2
is -3263.9 kJ mol -1 at 25° C; heat of combustion (in kJ
ou
ad

é (T + T2 )2 ù éT + T2 ù
(c) C p lnê 1 ú (d) 2 C p lnê 1 ú mol -1 ) of benzene at constant pressure will be
êë 4T1T2 úû ë 2T1T2 û (R = 8.314 JK -1 mol -1 )
Y

(2018 Main)
15. The process with negative entropy change is (a) 4152.6 (b) -452.46
(2019 Main, 10 Jan II) (c) 3260 (d) -3267.6
nd
Re

(a) synthesis of ammonia from N 2 and H 2


21. DU is equal to (2017 Main)
(b) dissociation of CaSO4(s) to CaO(s) and SO3 ( g )
Fi

(a) isochoric work (b) isobaric work


(c) dissolution of iodine in water
(c) adiabatic work (d) isothermal work
(d) sublimation of dry ice
22. The standard state Gibbs free energies of formation of
16. An ideal gas undergoes isothermal compression from 5 m 3
3 -2 C(graphite) and C(diamond) at T = 298 K are
to 1 m against a constant external pressure of 4 Nm . Heat
released in this process is used to increase the temperature D f G °[C(graphite)] = 0 kJ mol -1
of 1 mole of Al. If molar heat capacity of Al is 24 J mol-1 K -1 ,
D f G °[C(diamond)] = 2.9 kJ mol -1
the temperature of Al increases by (2019 Main, 10 Jan II)
3 2 The standard state means that the pressure should be 1 bar,
(a) K (b) 1 K (c) 2 K (d) K
2 3 and substance should be pure at a given temperature. The
conversion of graphite [C(graphite)] to diamond
17 A process has DH = 200 J mol -1 and DS = 40 JK -1 mol -1 .
[C(diamond)] reduces its volume by 2 ´ 10-6 m 3 mol -1 . If
Out of the values given below, choose the minimum
C(graphite) is converted to C(diamond) isothermally at
temperature above which the process will be spontaneous
(2019 Main, 10 Jan I) T = 298K, the pressure at which C(graphite) is in
(a) 20 K (b) 4 K (c) 5 K (d) 12 K equilibrium with C(diamond), is (2017 Adv.)
Thermodynamics and Thermochemistry 105

[Useful information : 1 J = 1 kg m 2 s -2 , Given that DS ( A ® C ) = 50 eu


1 Pa = 1 kg m -1 s -2 ; 1 bar = 105 Pa] DS (C ® D ) = 30 eu
(a) 58001 bar (b) 1450 bar DS (D ® B ) = –20 eu where, eu is entropy unit
(c) 14501 bar (d) 29001 bar Then, DS ( A ® B ) is (2006, 3M)

23. One mole of an ideal gas at 300 K in thermal contact with (a) + 100 eu (b) +60 eu
surroundings expands isothermally from 1.0 L to 2.0 L (c) –100 eu (d) –60 eu
against a constant pressure of 3.0 atm. 30. A monoatomic ideal gas undergoes a process in which the
In this process, the change in entropy of surroundings ratio of p to V at any instant is constant and equals to 1. What
( DS surr ) in JK -1 is (1 L atm = 101.3 J) (2016 Adv.) is the molar heat capacity of the gas ? (2006, 3M)
(a) 5.763 (b) 1.013 (c) - 1.013 (d) - 5.763 4R 3R 5R
(a) (b) (c) (d) 0
2 2 2

w
24. The following reaction is performed at 298K
31. One mole of monoatomic ideal gas expands adiabatically at
2NO( g ) + O2 ( g ) r 2NO2 ( g )
initial temperature T against a constant external pressure of
The standard free energy of formation of NO(g) is
1 atm from 1 L to 2 L. Find out the final temperature

Flo
86.6 kJ/mol at 298 K. What is the standard free energy of
(R = 0.0821 L atm K - 1 mol - 1 ) (2005, 1M)
formation of NO2 ( g ) at 298 K? ( K p = 1.6 ´ 1012 ) (2015 Main) T
(a) T (b)

ree
(a) R( 298 ) ln (1. 6 ´ 1012 ) – 86600 ( 2 )5 / 3 - 1
(b) 86600 + R( 298 ) ln (1. 6 ´ 1012 ) 2 2
(c) T - (d) T +

F
ln (1. 6 ´ 1012 ) 3 ´ 0.082 3 ´ 0.082
(c) 86600 –
R ( 298 ) 32. 2 moles of an ideal gas expanded isothermally and

or
ur 12
(d) 0. 5 [ 2 ´ 86600 – R ( 298 ) ln (1. 6 ´ 10 )] reversibly from 1 L to 10 L at 300 K. What is the enthalpy
change? (2004, 1M)

f
25. For the process, H2 O ( l ) ¾® H2 O ( g )
(a) 4.98 kJ (b) 11.47 kJ (c) –11.47 kJ (d) 0 kJ
at T = 100° C and 1 atmosphere pressure, the correct choice is
ks
33. Spontaneous adsorption of a gas on solid surface is an
(a) DS system > 0 and DS surrounding > 0
Yo
(2014 Adv.)
exothermic process because (2004, 1M)
oo
(b) DS system > 0 and DS surrounding < 0 (a) DH increases for system (b) DS increases for gas
(c) DS system < 0 and DS surrounding > 0 (c) DS decreases for gas (d) DG increases for gas
eB

(d) DS system < 0 and DS surrounding < 0


34. One mole of a non-ideal gas undergoes a change of state
26. A piston filled with 0.04 mole of an ideal gas expands (2.0 atm, 3.0 L, 95 K) ¾® (4.0 atm, 5.0 L, 245 K) with a
reversibly from 50.0 mL to 375 mL at a constant temperature change in internal energy, DE = 30.0 L-atm. The change in
r

of 37.0°C. As it does so, it absorbs 208 J of heat. The values enthalpy ( DH ) of the process in L-atm is
ou

(2002, 3M)
ad

of q and W for the process will be (a) 40.0 (b) 42.0


(R = 8.314 J / mol K, ln 7.5 = 2.01)
Y

(2013 Main)
(c) 44.0
(a) q = + 208 J, W = - 208 J
(d) not defined, because pressure is not constant
(b) q = - 208 J, W = - 208 J
35. Which of the following statements is false?
nd

(c) q = - 208 J, W = + 208 J


Re

(2001, 1M)
(d) q = + 208 J, W = + 208 J (a) Work is a state function
(b) Temperature is a state function
Fi

27. For the process H2 O( l ) (1 bar, 373 K) ® H2 O( g )


(c) Change in the state is completely defined when the
(1 bar, 373 K), the correct set of thermodynamic
initial and final states are specified
parameters is (2007, 3M)
(a) DG = 0, DS = + ve (b) DG = 0, DS = - ve (d) Work appears at the boundary of the system
(c) DG = + ve, DS = 0 (d) DG = - ve, DS = + ve 36. In thermodynamics, a process is called reversible when
(2001, 1M)
28. The value of log 10 K for a reaction A r B is (a) surroundings and system change into each other
-1
(Given : Dr H °298 K = - 54.07 kJ mol , (b) there is no boundary between system and surroundings
Dr S °298 K = 10 JK -1 mol -1 and R = 8.314 JK-1 mol-1 ; (c) the surroundings are always in equilibrium with the system
2. 303 ´ 8. 314 ´ 298 = 5705) (2007, 3M)
(d) the system changes into the surroundings spontaneously
(a) 5 (b) 10 (c) 95 (d) 100 37. For an endothermic reaction, where DH represents the
29. The direct conversion of A to B is difficult, hence it is carried enthalpy of the reaction in kJ/mol, the minimum value for
out by the following shown path the energy of activation will be (1992, 1M)
C ¾¾¾® D (a) less than DH (b) zero
­ ¯ (c) more than DH (d) equal to DH
A B
106 Thermodynamics and Thermochemistry

38. The difference between heats of reaction at constant pressure (c) With increase in temperature, the value of K for
and constant volume for the reaction endothermic reaction increases because the entropy change
2C6 H6 ( l ) + 15O2 ¾® 12CO2 ( g ) + 6H2 O( l ) of the system is negative
(d) With increase in temperature, the value of K for exothermic
at 25° C in kJ is (1991, 1M) reaction decreases because the entropy change of the system
(a) - 7.43 (b) + 3.72 is positive
(c) - 3.72 (d) + 7.43 42. An ideal gas in thermally insulated vessel at internal
pressure = p1 , volume =V1 and absolute temperature = T1
Objective Questions II expands irreversibly against zero external pressure, as
(One or more than one correct option) shown in the diagram. The final internal pressure,
volume and absolute temperature of the gas are p2 , V2
39. A reversible cyclic process for an ideal gas is shown below.
and T2 , respectively. For this expansion

w
Here, p , V and T are pressure, volume and temperature,
respectively. The thermodynamic parameters q , w , H and U
pext=0
are heat, work, enthalpy and internal energy, respectively.
(2018 Adv.) pext=0 Irreversible

Flo
A(p1, V1, T1) C(p2, V1, T2)

ree
Volume (V)

p1,V1,T1 p2,V2,T2

F
Thermal insulation
B(p2, V2, T1) (2014 Adv.)

or
ur (a) q = 0 (b) T2 = T1
Temperature (T) (c) p2V2 = p1V1 (d) p2V2g = p1V1g

f
ks
The correct options is (are) 43. Benzene and naphthalene form an ideal solution at room
(a) q AC = DU BC and w AB = p2 (V2 - V1 ) temperature. For this process, the true statement(s) is (are)
Yo
oo
(2013 Adv.)
(b) wBC = p2 (V2 - V1 ) and qBC = DH AC (a) DG is positive (b) DS system is positive
(c) DHCA < DUCA and q AC = DU BC (c) DS surroundings = 0 (d) DH = 0
eB

(d) qBC = DH AC and DHCA > DUCA


44. The reversible expansion ob an ideal gas under adiabatic
40. An ideal gas is expanded form ( p1 , V1 , T1 ) to ( p2 , V2 , T2 ) under and isothermal conditions is shown in the figure. Which of
r

different conditions. The correct statement(s) among the the following statement(s) is (are) correct? (2012)
ou
ad

following is (are) (2017 Adv.)


(a) The work done by the gas is less when it is expanded reversibly (p1,V1,T1)
Y

from V1 to V2 under adiabatic conditions as compared to that Iso


when expanded reversibly form V1 to V2 under isothermal the
p rma
l
nd

conditions.
Ad

ia
Re

ba (p2,V2,T2)
(b) The change in internal energy of the gas is (i) zero, if it is tic
(p3,V3,T3)
expanded reversibly with T1 = T2, and (ii) positive, if it is
Fi

expanded reversibly under adiabatic conditions with T1 ¹ T2 V

(c) If the expansion is carried out freely, it is simultaneously both


(a) T1 = T2 (b) T3 > T1
isothermal as well as adiabatic
(c) Wisothermal > Wadiabatic (d) DU isothermal > DU adiabatic
(d) The work done on the gas is maximum when it is compressed 45. For an ideal gas, consider only P -V work in going from
irrversibly from ( p2 , V2 ) to ( p1, V1 ) against constant pressure p1 initial state X to the final state Z. The final state Z can be
41. For a reaction taking place in a container in equilibrium with reached by either of the two paths shown in the figure.
its surroundings, the effect of temperature on its equilibrium
p(atmosphere)

X Y
constant K in terms of change in entropy is described by
(2017 Adv.)
(a) With increase in temperature, the value of K for endothermic Z
reaction increases because unfavourable change in entropy of the
surroundings decreases
(b) With increase in temperature, the value of K for exothermic V(litre)
reaction decreases because favourable change in entropy of the [Take DS as change in entropy and W as work done].
surrounding decreases
Which of the following choice(s) is (are) correct? (2012)
Thermodynamics and Thermochemistry 107

(a) DS X ® Z = DS X ® Y + DS Y ® Z 52. The pair of isochoric processes among the transformation of


(b) W X ® Z = W X ® Y + WY ® Z states is
(c) WX ® Y ® Z = WX ® Y (a) K to L and L to M
(d) DS X ® Y ® Z = DS X ® Y (b) L to M and N to K
(c) L to M and M to N
46. Among the following, extensive property is
(d) M to N and N to K
(properties are) (2010)
(a) molar conductivity (b) electromotive force 53. The succeeding operations that enable this transformation of
(c) resistance (d) heat capacity states are
(a) heating, cooling, heating, cooling
47. Among the following, the state function(s) is(are)
(b) cooling, heating, cooling, heating
(a) internal energy (2009)
(c) heating, cooling, cooling, heating
(b) irreversible expansion work

w
(d) cooling, heating, heating, cooling
(c) reversible expansion work
(d) molar enthalpy
Match the Columns
48. Identify the intensive quantities from the following.
54. Match the thermodynamic processes given under Column I with

Flo
(1993, 1M)
the expressions given under Column II.
(a) enthalpy (b) temperature

ree
(c) volume (d) refractive index Column I Column II
A. Freezing of water at 273 K and 1 p. q=0
Assertion and Reason atm

F
Read the following questions and answer as per the B. Expansion of 1 mole of an ideal q. W =0
direction given below : gas into a vacuum under isolated

or
ur
(a) Statement I is true; Statement II is true; Statement conditions
II is the correct explanation of Statement I

f
C. Mixing of equal volumes of two r. DS sys < 0
(b) Statement I is true; Statement II is true; Statement
ideal gases at constant
ks
II is not the correct explanation of Statement I
temperature and pressure in an
Yo
(c) Statement I is true; Statement II is false
isolated container
oo
(d) Statement I is false; Statement II is true
D. Reversible heating of H2 ( g ) at 1 s. DU = 0
49. Statement I There is a natural asymmetry between
eB

atm from 300 K to 600 K,


converting work to heat and converting heat to work. followed by reversible cooling to
Statement II No process is possible in which the 300 K at 1 atm
sole result is the absorption of heat from a reservoir DG = 0
r

t.
ou

and its complete conversion into work. (2008, 3M)


ad

55. Match the transformations in Column I with appropriate options in


50. Statement I For every chemical reaction at Column II. (2011)
Y

equilibrium, standard Gibbs energy of reaction is zero.


Statement II At constant temperature and pressure, Column I Column II
nd

chemical reactions are spontaneous in the direction A. CO2 ( s ) ¾® CO2 ( g ) p. Phase transition
Re

of decreasing Gibbs energy. (2008, 3M) B. CaCO3 ( s ) ¾® CaO( s ) + CO2 ( g ) q. Allotropic change
Fi

51. Statement I The heat absorbed during the C. 2H· ¾® H2 ( g ) r. DH is positive


isothermal expansion of an ideal gas against
D. P(white, solid) ® P(red, solid) s. DS is positive
vacuum is zero.
Statement II The volume occupied by the t. DS is negative
molecules of an ideal gas is zero. (2000, S, 1M)
Fill in the Blanks
Passage Based Questions 56. Enthalpy is an ............... property. (1997, 1M)
A fixed mass m of a gas is K L 57. When Fe(s) is dissolved in aqueous hydrochloric acid in a closed
subjected to transformation vessel, the work done is ................ . (1997)
of states from K to L to M to
Pressure 58. The heat content of the products is more than that of the reactants in
N and back to K as shown
in the figure. (2013 Adv.) an .............. reaction. (1993, 1M)
N M 59. A system is said to be ........ if it can neither exchange matter nor
Volume energy with the surroundings. (1993, 1M)
108 Thermodynamics and Thermochemistry

60. C p - CV for an ideal gas is ................. (b) a reversible isochoric change of state from (1.0 atm,
(1984, 1M)
40.0 L) to (0.5 atm, 40.0 L)
61. The total energy of one mole of an ideal monatomic gas at (c) a reversible isothermal compression from (0.5 atm,
27° C is ..............cal. (1984, 1M) 40.0 L) to (1.0 atm, 20.0 L)
(i) Sketch with labels each of the processes on the same
True/False p-V diagram.
62. First law of thermodynamics is not adequate in predicting (ii) Calculate the total work (W) and the total heat change
the direction of a process. (1982, 1M) (Q) involved in the above processes.
63. Heat capacity of a diatomic gas is higher than that of a (iii) What will be the values of DU, DH and DS for the
monoatomic gas. (1985, 1/2 M) overall process? (2002, 5M)

w
Integer Answer Type Questions 69. When 1-pentyne (A) is treated with 4 N alcoholic KOH at
175° C, it is converted slowly into an equilibrium mixture of
64. One mole of an ideal gas is taken from a to b along two paths
denoted by the solid and the dashed lines as shown in the 1.3% 1-pentyne (A), 95.2% 2-pentyne (B) and 3.5% of

Flo
graph below. If the work done along the solid line path is Ws 1, 2-pentadiene (C). The equilibrium was maintained at
and that along the dotted line path is Wd , then the integer 175° C. Calculate DG° for the following equilibria.
closest to the ratio Wd / Ws is (2010)

ree
B r A, DG°1 = ?
4.5
4.0 a B r C, DG°2 = ?

F
3.5 From the calculated value of DG°1 and DG°2 indicate the
3.0
p 2.5 order of stability of (A), (B) and (C). Write a reasonable

or
(atm)
2.0
ur reaction mechanism showing all intermediates leading to
1.5 (A), (B) and (C).

f
(2001, 10M)
1.0
1
O2 ( g ) ¾® CO2 ( g ) at
ks
0.5 b 70. Show that the reaction, CO( g ) +
0.0 2
Yo
0.0 0.5 1.0 1.5 2.0 2.5 3.0 3.5 4.0 4.5 5.0 5.5 6.0 300 K, is spontaneous and exothermic, when the standard
oo
V (L)
entropy change is -0.094 kJ mol -1 K -1 . The standard
eB

Subjective Questions Gibbs’ free energies of formation for CO2 and CO are
–394.4 and –137.2 kJ mol -1 , respectively. (2000, 3M)
65. For the reaction, 2CO + O2 ® 2CO2 ; DH = - 560 kJ. Two
moles of CO and one mole of O2 are taken in a container of 71. A sample of argon gas at 1 atm pressure and 27° C expands
r

reversibly and adiabatically from 1.25 dm3 to 2.50 dm3 .


ou

volume 1 L. They completely form two moles of CO2 , the


ad

gases deviate appreciably from ideal behaviour. If the Calculate the enthalpy change in this process CV m for argon
is 12.49 JK -1 mol -1 .
Y

pressure in the vessel changes from 70 to 40 atm, find the (2000, 4M)
magnitude (absolute value) of DU at 500 K. (1 L-atm = 0.1 kJ) 72. A gas mixture of 3.67 L of ethylene and methane on
(2006, 3M)
complete combustion at 25° C produces 6.11 L of CO2 . Find
nd
Re

66. 100 mL of a liquid contained in an isolated container at a out the amount of heat evolved on burning 1 L of the gas
pressure of 1 bar. The pressure is steeply increased to mixture. The heat of combustion of ethylene and methane
Fi

100 bar. The volume of the liquid is decreased by 1 mL at are - 1423 and - 891 kJ mol -1 at 25°C. (1991, 5M)
this constant pressure. Find the DH and DU . (2004, 2M)
3R 3R 73. An athlete is given 100 g of glucose (C6 H12 O6 ) of energy
67. CV value of He is always but CV value of H2 is at low
2 2 equivalent to 1560 kJ. He utilizes 50 per cent of this gained
5R energy in the event. In order to avoid storage of energy in the
temperature and at moderate temperature and more than
2 body, calculate the weight of water he would need to
5R perspire. The enthalpy of evaporation of water is 44 kJ/mol.
at higher temperature. Explain in two or three lines.
2 (1989, 2M)
(2003, 2M)
68. Two moles of a perfect gas undergo the following 74. Following statement is true only under some specific
processes : conditions. Write the conditions for that in not more than
two sentences
(a) a reversible isobaric expansion from (1.0 atm, 20.0 L)
“The heat energy q, absorbed by a gas is DH.’’ (1984, 1M)
to (1.0 atm, 40.0 L)
Topic 2 Thermochemistry
Objective Questions I (Only one correct option) 6. The standard enthalpies of formation of CO2 ( g ), H2 O( l ) and
1. Enthalpy of sublimation of iodine is glucose(s) at 25°C are - 400 kJ/mol, - 300 kJ/mol and
24 cal g -1 at 200°C. If specific heat of I2( s) and I2 (vap.) are - 1300 kJ/mol, respectively. The standard enthalpy of
combustion per gram of glucose at 25°C is (2013 Adv.)
0.055 and 0.031 cal g -1 K -1 respectively, then enthalpy of
(a) + 2900 kJ (b) - 2900 kJ
sublimation of iodine at 250°C in cal g -1 is
(c) - 16. 11 kJ (d) + 16.11 kJ
(2019 Main, 12 April I)
(a) 2.85 (b) 5.7 (c) 22.8 (d) 11.4 7. Using the data provided, calculate the multiple bond energy
(kJ mol -1 ) of a Cºº C bond C2 H2 . That energy is (take the
2. Given :

w
bond energy of a C ¾ H bond as 350 kJ mol -1 )
(i) C(graphite) + O2 ( g ) ® CO2 ( g ); Dr H È = x kJ mol- 1
(2012)
2C( s ) + H2 ( g ) ¾® C2 H2 ( g ) ; DH = 225 kJ mol -1
1
(ii) C(graphite) + O2 ( g ) ¾® CO2 ( g ); 2C( s ) ¾® 2C( g ) ; DH = 1410 kJ mol -1

Flo
2
H2 ( g ) ¾® 2H( g ) ; DH = 330 kJ mol -1
Dr H È = y kJ mol- 1
(a) 1165 (b) 837 (c) 865 (d) 815
1

ree
(iii) CO ( g ) + O2 ( g ) ¾® CO2 ( g ); Dr H È = z kJ mol- 1 8. The species which by definition has zero standard molar
2
enthalpy of formation at 298 K is (2010)
Based on the above thermochemical equations, find out

F
(a) Br2 ( g ) (b) Cl 2 ( g )
which one of the following algebraic relationships is correct?
(2019 Main, 12 Jan II) (c) H2 O( g ) (d) CH4 ( g )

or
(a) y = 2 z - x (b) x = y - z 9. The bond energy (in kcal mol -1 ) of C— C single bond is
ur
(c) z = x + y (d) x = y + z approximately (2010)
3. Given, C (graphite) + O2 ( g ) ¾® CO2 ( g );
f
(a) 1 (b) 10
ks
(c) 100 (d) 1000
Dr H ° = - 393.5 kJ mol - 1
Yo
10. DH vap = 30 kJ/mol and DS vap = 75 Jmol –1 K –1 . Find the
oo
1
H2 ( g ) + O2 ( g ) ¾® H2O( l);
2 temperature of vapour, at one atmosphere (2004, 1M)
eB

Dr H ° = - 285.8 kJ mol - 1 (a) 400 K (b) 350 K (c) 298 K (d) 250 K
CO2 ( g ) + 2 H2O( l) ¾® CH 4( g )+2O2 ( g ); 11. Which of the following reactions defines DH f° ?
Dr H ° = + 890.3 kJ mol - 1 (2003, 1M)
(a) C(diamond) + O2 ( g ) ¾® CO2 ( g )
r
ou

Based on the above thermochemical equations, the value of


ad

1 1
Dr H ° at 298 K for the reaction, (2017 Main) (b) H2 ( g ) + F2 ( g ) ¾® HF( g )
2 2
Y

C (graphite) + 2 H2 ( g ) ¾® CH 4( g ) will be
(c) N2 ( g ) + 3H2 ( g ) ¾® 2NH3 ( g )
(a) + 78 .8 kJ mol - 1 (b) + 144.0 kJ mol - 1 1
(d) CO ( g ) + O2 ( g ) ¾® CO2 ( g )
nd
Re

(c) - 74 .8 kJ mol - 1 (d) - 144.0 kJ mol - 1 2


4. The heats of combustion of carbon and carbon monoxide are 12. The DH f° for CO2 ( g ),CO( g ) and H2 O ( g ) are
Fi

-1
- 393.5 and - 283.5 kJ mol , respectively. The heat of - 393.5, - 110.5 and -241.8 kJ mol -1 respectively. The
formation (in kJ) of carbon monoxide per mole is
standard enthalpy change (in kJ mol -1 ) for the reaction
(2016 Main)
(a) 676.5 (b) -676.5 CO2 ( g ) + H2 ( g ) ¾® CO( g ) + H2 O( g ) is (2000, 1M)
(c) -110.5 (d) 110.5 (a) 524.1 (b) + 41.2
5. For the complete combustion of ethanol, (c) -262.5 (d) -41.2
C2 H 5 OH( l ) + 3O2 ( g ) ¾® 2CO2 ( g ) + 3H2 O( l ), the
amount of heat produced as measured in bomb Objective Question II
calorimeter, is 1364.47 kJ mol -1 at 25°C. Assuming (One or more than one correct option)
ideality the enthalpy of combustion, DC H, for the reaction 13. The following is/are endothermic reaction(s) (1999, 3M)
will be (R = 8.314 J K–1 mol –1 ) (2014 Main) (a) Combustion of methane
(a) - 1366. 95 kJ mol -1 (b) - 1361. 95 kJ mol -1 (b) Decomposition of water
(c) - 1460. 50 kJ mol -1 (d) - 1350. 50 kJ mol -1 (c) Dehydrogenation of ethane to ethylene
(d) Conversion of graphite to diamond
110 Thermodynamics and Thermochemistry

14. The thermal dissociation of equilibrium of CaCO3 ( s ) is Use these data to estimate the magnitude of the resonance
studied under different conditions. (2013 Adv.) energy of benzene. (1996, 2M)
CaCO3 ( s ) r CaO( s ) + CO2 ( g ) 21. The polymerisation of ethylene to linear polyethylene is
For this equilibrium, the correct statement(s) is/are represented by the reaction,
(a) DH is dependent on T n [CH2 == CH2 ] ¾® ¾[ CH2 — CH2¾
]n
(b) K is independent of the initial amount of CaCO3
(c) K is dependent on the pressure of CO2 at a given T where, n has large integral value. Given that the average
(d) DH is independent of the catalyst, if any enthalpies of bond dissociation for C == C and C ¾ C at
298 K are +590 and +311 kJ/mol respectively, calculate the
Subjective Questions enthalpy of polymerization per mole of ethylene at 298 K.
(1994, 2M)
15. In a constant volume calorimeter, 3.5 g of a gas with

w
molecular weight 28 was burnt in excess oxygen at 298.0 K. 22. In order to get maximum calorific output, a burner should
The temperature of the calorimeter was found to increases have an optimum fuel to oxygen ratio which corresponds
from 298.0 K to 298.45 K due to the combustion process. to 3 times as much oxygen as is required theoretically
for complete combustion of the fuel. A burner which has been
Given that the heat capacity of the calorimeter is 2.5 kJ K - 1 ,

Flo
adjusted for methane as fuel (with x litre/hour of CH4 and
the numerical value for the enthalpy of combustion of the
6x litre/hour of O2 ) is to be readjusted for butane, C4 H10 .
gas in kJ mol - 1 is (2009)

ree
In order to get the same calorific output, what should be the
16. Diborane is a potential rocket fuel which undergoes rate of supply of butane and oxygen? Assume that losses due
combustion according to the reaction to incomplete combustion etc., are the same for both fuels

F
B2 H6 ( g ) + 3O2 ( g ) ¾® B2 O3 ( s ) + 3H2 O ( g ) and that the gases behave ideally. Heats of combustions:
CH4 = - 809 kJ/mol, C4 H10 = - 2878 kJ/mol

or
From the following data, calculate the enthalpy change for
the combustion of diborane.
ur (1993, 3M)

(2000, 2M) 23. Determine the enthalpy of the reaction,

f
C3 H8 ( g ) + H2 ( g ) ® C2 H6 ( g ) + CH4 ( g ), at 25°C, using the
3
2B ( s ) + O2 ( g ) ¾® B2 O3 ( s ) ; DH = - 1273 kJ mol -1
ks
2 given heat of combustion values under standard conditions.
Yo
1 Compound : H2 ( g ) CH4 ( g ) C2 H6 ( g ) C(graphite)
H2 ( g ) + O2 ( g ) ¾® H2 O ( l ) ; DH = - 286 kJ mol –1
oo
2 DH ° (kJ/mol): -285.8 - 890.0 -1560.0 -393.0
H2 O ( l ) ¾® H2 O ( g ) ; DH = 44 kJ mol -1 The standard heat of formation of C3 H8 ( g ) is -103 kJ/mol.
eB

2B ( s ) + 3H2 ( g ) ¾® B2 H6 ( g ) ; DH = 36 kJ mol -1 (1992, 3M)


24. Using the data (all values are in kilocalories per mol at 25° C)
17. Estimate the average S–F bond energy in SF6 . The values of given below, calculate the bond energy of C ¾ C and C ¾ H
r
ou

standard enthalpy of formation of SF6 (g), S( g ) and F( g ) bonds.


ad

are : – 1100, 275 and 80 kJ mol –1 respectively. (1999, 3M) C( s ) ¾® C( g ); DH = 172


Y

18. From the following data, calculate the enthalpy change for H2 ( g ) ¾® 2H ( g ); DH = 104
1
the combustion of cyclopropane at 298 K. The enthalpy of H2 ( g ) + O2 ( g ) ¾® H2 O( l ); DH = - 68.0
2
nd

formation of CO2 ( g ), H2 O ( l ) and propane (g) are –393.5,


Re

-285.8 and 20.42 kJ mol -1 respectively. The enthalpy of C( s ) + O2 ( g ) ¾® CO2 ( g ); DH = - 94.0


isomerisation of cyclopropane to propene is -33.0 kJ mol -1 . Heat of combustion of C2 H6 = -372.0
Fi

(1998, 5M) Heat of combustion of C3 H8 = -530.0 (1990, 5M)

19. Compute the heat of formation of liquid methyl alcohol in 25. The standard enthalpy of combustion at 25° C of hydrogen,
kJ mol -1 , using the following data. Heat of vaporisation of cyclohexene (C6 H10 ) and cyclohexane (C6 H12 ) are - 241,
liquid methyl alcohol = 38 kJ/mol. Heat of formation of - 3800 and - 3920 kJ/mol respectively. Calculate the heat of
gaseous atoms from the elements in their standard states : hydrogenation of cyclohexene. (1989, 2M)
H = 218 kJ/mol, C = 715 kJ/mol, O = 249 kJ/mol. 26. An intimate mixture of ferric oxide, Fe2 O3 , and aluminium,
Average bond energies: (1997, 5M) Al, is used in solid fuel rockets. Calculate the fuel value per
C— H = 415 kJ/mol, C— O = 356 kJ/mol, gram and fuel value per cc of the mixture. Heats of
O— H = 463 kJ/mol formation and densities are as follows:
20. The standard molar enthalpies of formation of H f (Al 2 O3 ) = - 399 kcal/mol
cyclohexane (l) and benzene (l) at 25° C are -156 and H f (Fe2 O3 ) = - 199 kcal/mol
+ 49 kJ mol -1 respectively. The standard enthalpy of Density of Fe2 O3 = 5.2 g/cc, Density of Al = 2.7 g/cc
hydrogenation of cyclohexene (l) at 25° C is -119 kJ mol -1 . (1989, 2M)
Thermodynamics and Thermochemistry 111

27. The standard molar heat of formation of ethane, carbon 1 1


(i) H2 ( g ) + O2 ( g ) ¾® OH( g ) DH ° = -10.06 kcal
dioxide and liquid water are -21.1, -94.1 and - 68.3 kcal 2 2
respectively. Calculate the standard molar heat of (ii) H2 ( g ) ¾® 2H( g ) DH ° = 104.18 kcal
combustion of ethane. (1986, 2M)
(iii) O2 ( g ) ¾® 2O( g ) DH ° = 118.32 kcal
28. The bond dissociation energies of gaseous H2 ,Cl 2 and HCl
Calculate the O — H bond energy in the hydroxyl radical.
are 104, 58 and 103 kcal/mol respectively. Calculate the (1981, 2M)
enthalpy of formation of HCl gas. (1985, 2M)
29. Given the following standard heats of reactions Passage Based Questions
(i) heat of formation of water = - 68.3 kcal When 100 mL of 1.0 M HCl was mixed with 100 mL of 1.0 M
(ii) heat of combustion of acetylene = - 310.6 kcal NaOH in an insulated beaker at constant pressure, a temperature
(iii) heat of combustion of ethylene = - 337.2 kcal

w
increase of 5.7°C was measured for the beaker and its contents
Calculate the heat of reaction for the hydrogenation of (Expt. 1). Because the enthalpy of neutralisation of a strong acid
acetylene at constant volume (25° C). with a strong base is a constant ( -57.0 kJ mol-1 ), this experiment
(1984, 4M)
could be used to measure the calorimeter constant. In a second

Flo
30. The molar heats of combustion of C2 H2 ( g ), C (graphite) and
experiment (Expt. 2), 100 mL of 2.0 M acetic acid
H2 ( g ) are 310.62 kcal, 94.05 kcal and 68.32 kcal ( K a = 2.0 ´ 10-5 ) was mixed with 100 mL of 1.0 M NaOH (under

ree
respectively. Calculate the standard heat of formation of
identical conditions to Expt. 1) where a temperature rise of 5.6°C
C2 H2 ( g ) . (1983, 2M)
was measured.
31. The standard heats of formation of CCl 4 ( g ), H2 O( g ),

F
33. Enthalpy of dissociation (in kJ mol-1 ) of acetic acid
CO2 ( g ) and HCl( g ) at 298 K are -25.5, -57.8 , -94.1 and obtained from the Expt. 2 is
-22.1 kcal/mol respectively. Calculate DH ° (298 K) for the

or
(a) 1.0 (b) 10.0 (c) 24.5 (d) 51.4
reaction
ur
34. The pH of the solution after Expt. 2 is

f
CCl 4 ( g ) + 2H2 O ( g ) ¾® CO2 ( g ) + 4 HCl ( g ) (1982, 2M)
(a) 2.8 (b) 4.7 (c) 5.0 (d) 7.0
32. The enthalpy for the following reactions ( DH ° ) at 25°C are
ks
Yo
given below
oo
eB

Answers
Topic 1 Thermodynamics 56. extensive 57. zero 58. exothermic reaction
1. (c) 2. (a) 3. (b) 4. (a)
r

59. isolated 60. R 61. 900 62. T


ou

5. (d) 6. (c) 7. (b) 8. (a)


ad

63. T 64. (2) 65. (-563 kJ) 69. (12.3 kJ)


9. (a) 10. (a) 11. (d) 12. (d) 70. (-285.4 kJ)71. (- 116.4 J) 72. (49.82 kJ) 73. (318.96 g)
Y

13. (c) 14. (c) 15. (a) 16. (d)


Topic 2 Thermochemistry
17. (c) 18. (a) 19. (c) 20. (d)
1. (c) 2. (d) 3. (c) 4. (c)
nd

21. (c) 22. (c) 23. (c) 24. (d)


Re

5. (a) 6. (c) 7. (d) 8. (b)


25. (b) 26. (a) 27. (a) 28. (b)
9. (c) 10. (a) 11. (b) 12. (b)
Fi

29. (b) 30. (a) 31. (c) 32. (d)


13. (b,c,d) 14. (a, b, c, d) 15. (9 kJ) 16. (-2035 kJ)
33. (c) 34. (c) 35. (a) 36. (c)
17. (309.16 kJ) 18. (-2091.32 kJ) 19. (-116.4 kJ)
37. (c) 38. (a) 39. (b,c) 40. (a, c, d)
20. (-152 kJ/mol) 21. (-32 kJ/mol)
41. (a, b) 42. (a,b,c) 43. (b,c,d) 44. (a,c,d) 22. (5.46 xL/h) 23. (-55 kJ)
45. (a,c) 46. (c,d) 47. (a,c,d) 48. (b, d) 25. (-121 kJ/mol) 27. (-372 kcal/mol)
49. (b) 50. (d) 51. (b) 52. (b) 28. (-22 kcal/mol) 29. (-41.7 kcal)
53. (c) 54. A ® r, t; B ® p, q, s; C ® p, q, s; D ® p,q, s, t 30. (54.2 kcal) 31. (-41.4 kcal)
55. A ® p, r, s; B ® r, s; C ® t; D ® p, q, t 32. (121.31 kcal) 33. (1 kJ/mol) 34. (4.7)
Hints & Solutions
Topic 1 Thermodynamics 5. q (heat) and W (work) represents path functions. These variables
1. Key Idea Work done during isothermal expansion of an are path dependent and their values depends upon the path
ideal gas is given by the equation. followed by the system in attaining that state. They are inexact
differentials whose integration gives a total quantity depending
W = - pext (V2 - V1 )
upon the path.
According to the given conditions, the expansion is against Option (a), i.e. q + W and option (d), i.e. H–TS are state
constant external pressure. So, the work done is given by functions. The value of state functions is independent to the way
following formula; in which the state is attained. All the state functions are exact
differentials and cyclic integration involving a state functions is

w
W = - pext (V2 - V1 )
zero.
= - 1bar (10L - 1L) = - 9 L bar (Q 1L bar =100J)
6. Given,
= -9 ´ 100 J = -0.9 kJ n = 5 mol, T2 = 200 K, T1 = 100 K

Flo
2. CV = 28 JK -1mol-1
Key Idea The relation between DH and DU is
DH = DU + DngRT DU = nCV DT

ree
where, Dng = Snp - SnR = nCV (T2 - T1 )
= number of moles of gaseous products - number of = 5 mol ´ 28 JK -1mol-1 ´ (200 - 100) K

F
moles of gaseous reactants. = 14 ,000 J = 14 kJ
The general combustion reaction of a hydrocarbon is as follows : DpV = nRDT

or
æ yö
ur y = nR (T2 - T1 )
CxH y + ç x + ÷ O2 ¾® xCO2 + H2O = 5 mol ´ 8 JK -1mol-1 ´ (200 - 100) K

f
è 4ø 2
For heptane, x = 7, y = 16 = 4000 J = 4 kJ
ks
Þ C7H16 (l ) + 11O2(g ) ¾® 7CO2(g ) + 8H2O(l )
Yo
7. From the 1st law of thermodynamics,
oo
\ Dng = 7 - 11 = - 4 DU = q + W
Now, from the principle of thermochemistry, where, DU = change in internal energy
eB

DH = DU + DngRT q = heat
Þ DH - DU = DngRT = - 4RT W = work done
The above equation can be represented for the given processes
3. A process will be spontaneous when its free energy (Gibb’s
r

involving ideal gas as follows:


energy) change will be negative, i.e. DG < 0.
ou
ad

(a) Cyclic process For cyclic process, DU = 0


Spontaneity of a process is decided by the value of DG, which
can be predicted from the Gibb’s equation, DG = DH - TDS for \ q = -W
Y

positive/negative signs of DH and DS at any/higher/lower Thus, option (a) is correct.


temperature as: (b) Adiabatic process For adiabatic process,
nd

q=0
Re

DH DS Comment on DG Comment on the \ DU = W


Fi

temperature process Thus, option (b) is incorrect.


(T)
(c) Isochoric process For isochoric process, DV = 0.
<0 >0 at any temp. <0 spontaneous Thus, W =0 (QW = pDV ).
>0 <0 at any temp. >0 non-spontaneous \ DV = q
<0 <0 at lower temp. <0 spontaneous Thus, option (c) is correct.
>0 >0 at higher temp. <0 spontaneous (d) Isothermal process For isothermal process, DU = 0
\ q = -W
4. In the given system, during the compression of a spring the Thus, option (d) is correct.
workdone is 10 kJ and 2 kJ of heat is escaped to the
surroundings. So, q = - 2 kJ and W = 10 kJ 8. According to Kirchoff’s relation,
T2
According to the first law of thermodynamics,
DH = n ò C p dT …(i)
DU = q + W = - 2 kJ + 10 kJ
T1
DU = 8 kJ
where, DH = Change in enthalpy.
The change in internal energy, DU (in kJ) is 8 kJ. C p = Heat capacity at constant pressure.
Thermodynamics and Thermochemistry 113

Given, n = 3 moles, T1 = 300 K, T2 = 1000 K, C p = 23 + 0.01 T or DS = 2 ´ 96000 C mol-1 ´ (-5 ´ 10-4 VK -1 )


On substituting the given values in Eq. (i), we get = - 96 JK -1 mol -1
1000 1000
Thus, on substituting the values of DG and DS in Eq. (i), we get
DH = 3 ò (23 + 0.01 T )dT = 3 ò 23dT + 0.01T dT
300 300 - 384000 J mol-1
1000
é 0.01 T 2 ù = DH - 300 K ´ (-96 JK -1mol-1 )
= 3 ê 23T + ú
ë 2 û 300 DH = - 384000 - 28800 Jmol-1
é 0.01 ù = - 412800J mol -1
= 3 ê 23 (1000 - 300) + (10002 - 3002 )ú
ë 2 û = - 412.800 kJ mol -1
= 3 [16100 + 4550 ] = 61950 J » 62 kJ 11. According to Gibbs-Helmholtz equation,

w
9. For diatomic ideal gases, D rG ° = D r H ° - TD r S °
f æ f ö For a reaction to be feasible (spontaneous)
CV = R and C p = ç + 1÷ R
2 è2 ø Dr G° < 0
Dr H °-TDr S ° <0
where, f = degree of freedom

Flo
f = translational degree of freedom + rotational degree of Given, Dr H ° = + 491.1 kJ mol - 1 ,
freedom
Dr S ° = 198 JK - 1 mol - 1

ree
= 3 + 2 = 5 [at normal temperature]
The explanation of various plots are as follows. \ 491.1 ´ 103 -T ´ 198 < 0

F
(a) We know that, C p is heat capacity at constant pressure. Thus, 491.1 ´ 103
it does not vary with the variation in pressure. Hence, plot T> = 2480.3 K
198
given in option (a) is incorrect.

or
ur
(b) In this plot, CV first increases slightly with increase in
\Above 2480.3 K reaction will become spontaneous.

f
temperature and then increases sharply with temperature. 12. According to Gibb’s Helmholtz equation,
The sharp increase is due to increase in degree of freedom. D rG ° = D r H ° - TD r S °
ks
Thus, plot given in option (b) is correct. Given, D rG ° = A - BT
Yo
(c) For ideal gases, On comparing above two equations, we get,
oo
Internal energy (U ) µ T A = DH ° and DS ° = B
eB

Thus, as temperature increases internal energy also We know that, if DH ° is negative, reaction is exothermic and
increases. As temperature increases further degree of when it is positive, reaction is endothermic.
freedom also increases thus, there is slight variation in the \ If A > 0, i.e. positive, reaction is endothermic.
graph. First translational degree of freedom is present
r

followed by rotational and vibrational degree of freedom. 13. For a given value of T ,
ou
ad

Hence, plot given in option (c) is also correct. (i) If D rG ° becomes < 0, the forward direction will be spontaneous
(d) CV is heat capacity at constant volume. Thus, it does not vary and then the major and minor components will be Y and X
Y

with variation in volume. Hence, plot given in option (d) is respectively.


correct. (ii) IfD rG ° becomes > 0, the forward direction will be
nd

non-spontaneous and then the major and minor components


Re

10. Given, will be X and Y respectively.


æ dE ° ö -4 -1 3
Fi

E° = 2V, ç ÷ = - 5 ´ 10 VK (a) D rG ° = 120 - ´ 280 = 15


è dT ø 8
T = 300K, R = 8 JK -1mol -1, i.e. D rG ° > O 0, major component = X ;
F = 96000 Cmol -1 3
(b) D rG ° = 120 - ´ 350 = - 1125 .
According to Gibbs-Helmholtz equation, 8
D G = D H - TD S …(i) i.e. D rG ° < 0, major component = Y
3
Also, DG = - nFE °cell …(ii) (c) D rG ° = 120 - ´ 315 = 1875 .
8
On substituting the given values in equation (ii), we get
i.e. D rG ° > 0, major component = X
DG = - 2 ´ 96000 C mol-1 ´ 2 V
3
[Q n = 2 for the given reaction] (d) D rG ° = 120 - ´ 300 = 7.5
8
= - 4 ´ 96000J mol -1
i.e. D rG ° > 0, major component = X
= - 384000J mol -1
14. At the thermal equilibrium,
æ dE ° ö T1 + T2
Now, DS = nF ç ÷
è dT ø final temperature T f =
2
114 Thermodynamics and Thermochemistry

Tf 18. The conversion of 1 kg of ice at 273 K into water vapours at 383


Þ for the 1st block, DS I = C p ln
T1 K takes place as follows:
Tf DS1 DS2
Þ for the 2nd block, DS II = C p ln H2O(s) H2O(l) H2O(l)
T2 273K 273K 373K
When brought in contact with each other, DS3
Tf Tf
D S = DS I + DS II = C p ln + C p ln H2O(g) H2O(g)
T1 T2 DS4
383K 373K
æ Tf Tf ö é Tf 2 ù
-1
= C p ln çç ´ ÷÷ = C p ln ê ú DH Fusion 334 kJ kg
è 1
T T2ø êë 1T2 úû
T DS 1 = = . kJ kg -1K -1
= 122
DTFusion 273 K
é æ T1 + T2 ö 2 ù

w
êç ÷ ú T2 æ 373 K ö
é (T + T2 )2 ù DS 2 = C ln = 4.2 kJ K -1kg -1 ln çç ÷
= C p ln ê è
2 ø ú ÷
= C p ln ê 1 ú T1 è 273 K ø
ê T1T2 ú ë 4T1T2 û
ê ú = 4.2 ´ 2.303 (log 373 - log 273) kJ K -1kg -1
ë û

Flo
. kJ K -1kg -1
= 4.2 ´ 2.303 (2.572 - 2.436) = 131
15. The explanation of all the options are as follows :
DH vap. 2491 kJ kg -1
(a) N2(g ) + 3H2(g ) ¾® 2NH3 (g ), DS 3 = =

ree
DTvap. 373 K
Dng = 2 - (1 + 3) = - 2
So, DS is also negative (entropy decreases) = 6.67 kJ kg -1K -1

F
D T2 æ 383 K ö
(b) CaSO4 (s) ¾® CaO(s) + SO3 (g ), DS 4 = Cln = 2 kJ K -1kg -1 ln ç ÷
ç 373 K ÷
T1 è ø

or
Dng = (1 + 0) - 0 = + 1
ur = 2 ´ 2.303 (log 383 - log 373) kJ K -1kg -1
So, DS = + ve

f
(c) In dissolution, DS = + ve because molecules/ions of the = 2 ´ 2.303 (2.583 - 2.572) kJ K -1kg -1
ks
solid solute (here, iodine) become free to move in = 0.05 kJ K -1kg -1
Yo
solvated/dissolved state of the solution,
DS Total = DS 1 + DS 2 + DS 3 + DS 4
oo
I2 (s) ¾Water
¾¾® I2 (aq) = 122
. + 131. + 6.67 + 0.05
(KI)
= 9.26 kJ kg -1K -1
eB

(d) In sublimation process, molecules of solid becomes quite


free when they become gas,
19. For isothermal reversible expansion,
CO2(s) ¾® CO2 (g )
Vf V
| W | = nRT ln = nRT ln
r

Dry ice
ou

So, DS will be positive. Vi Vi


ad

where, V = final volume, Vi = initial final.


16. It is an irreversible isothermal compression of an ideal gas.
Y

or | W | = nRT ln V - nRT ln Vi
(i) dE = dq + p(V f - Vi )
On comparing with equation of straight line, y = mx + c, we get
where, dE = Internal energy change
slope = m = + nRT
nd
Re

dq = amount of heat released


intercept = - nRT ln Vi
Þ 0 = dq + p (V f - Vi )
Thus, plot of |W | with lnV will give straight line in which slope
Fi

[Q dE = 0 for an isothermal process] of 2(T2 ) is greater than slope of 1(T1 ) which is given in all
Þ dq = - 4 (1 - 5) = 16 J options.
(ii) dq = n ´ C ´ DT (for Al) Now, if Vi < 1 then y intercept (- nRT Vi ) becomes positive and
Þ 16 J = 1 mol ´ 24 J mol - 1 K -1 ´ DT if it is positive for one case then it is positive for other case also.
Thus, it is not possible that one y-intercept goes above and other
16 2
Þ DT = K= K y-intercept goes below. Thus, option (b) and (d) are incorrect.
24 3 If we extent plot given in option (a) it seems to be merging
17. DG = DH - TDS which is not possible because if they are merging they give same
+ve y-intercept. But they cannot give same y-intercept because
The process will be spontaneous, when value of T is different.
DG = - ve, i.e. |TDS | > | D H |
Now, if we extent the line of T1 and T2 given in option (c) it
Given : DH = 200 Jmol-1 and DS = 40 JK -1mol-1 seems to be touching the origin. If they touch the origin then
| D H | 200 y-intercept becomes zero which is not possible. Thus, it is not
Þ T > = =5K
| DS | 40 the exactly correct answer but among the given options it is the
most appropriate one.
So, the minimum temperature for spontaneity of the process is
5 K.
Thermodynamics and Thermochemistry 115

20. Key idea Calculate the heat of combustion with the help of 24. For the given reaction,
following formula 2NO(g ) + O2 (g ) s 2NO2 (g )
DH p = DU + DngRT °
Given, DG (NO) = 86.6 kJ/mol
where, DH p = Heat of combustion at constant pressure f

DU = Heat at constant volume (It is also called DE) DG °f (NO2 ) = ?


Dng = Change in number of moles (In gaseous state).
K p = 1.6 ´ 1012
R = Gas constant; T = Temperature.
Now, we have,
From the equation, DG ° = 2DG °
f f (NO ) - [ 2DG °f (NO) + DG °f (O ) ]
15 2 2
C6H6( l ) + O2( g ) ¾® 6CO2( g ) + 3H2O( l )
2 = - RT lnK p = 2DG °f (NO ) - [ 2 ´ 86,600 + 0 ]
2
Change in the number of gaseous moles i.e.

w
1
15 3 DG ° f (NO ) = [ 2 ´ 86600 - R ´ 298 ln (1.6 ´ 1012 )]
Dng = 6 - = - or -15. 2 2
2 2
DG °f (NO = 0.5 [ 2 ´ 86,600 - R ´ (298)ln (1.6 ´ 1012 )]
Now we have Dng and other values given in the question are 2)

Flo
DU = - 3263.9 kJ/mol 25. PLAN This problem is based on assumption that total entropy change
T = 25° C = 273 + 25 = 298 K of universe is zero.

ree
R = 8.314 JK-1mol -1 At 100°C and 1 atmosphere pressure,
-3 H2O (l ) r H2O(g ) is at equilibrium.
So, DH p = (-3263.9) + (-15
. ) ´ 8.314 ´ 10 ´ 298
DS total = 0

F
-1
For equilibrium,
= - 3267.6 kJ mol and DS system + DS surrounding = 0
21. According to first law of thermodynamics, As we know during conversion of liquid to gas entropy of

or
D U = q + W = q - pD V
ur system increases, in a similar manner entropy of surrounding
decreases.

f
In isochoric process (DV = 0), DU = q
\ DS system > 0 and DS surrounding < 0
In isobaric process (Dp = 0), DU = q
ks
In adiabatic process (q = 0), DU = W 26. The process is isothermal expansion, hence
Yo
In isothermal process (DT = 0) and DU = 0 q= -W
oo

\ DU is equal to adiabatic work. DE = 0


eB

22. G = H - TS = U + pV - TS V2
W = – 2.303 nRT log
V1
Þ dG = dU + pdV + Vdp - TdS - SdT = Vdp - SdT
335
[Q dU + pdV = dq = TdS ] = - 2.303 ´ 0.04 ´ 8.314 ´ 310 ´ log
r

50
Þ dG = Vdp if isothermal process (dT = 0)
ou
ad

Þ DG = VDp = - 208 J
q = + 208 J
Y

Now taking initial state as standard state


Ggr - Ggr ° = Vgr Dp …(i) W = - 208 J (expansion work)
G d - G d ° = Vd Dp …(ii) 27. At transition point (373 K, 1.0 bar), liquid remains in
nd
Re

Now (ii)-(i) gives, equilibrium with vapour phase, therefore DG = 0. As


(Vd - Vgr )Dp = Gd - Ggr ° - G°
+ (Ggr vaporisation occur, degree of randomness increases, hence
Fi

d
DS > 0.
At equilibrium, Gd = Ggr
28. DG ° = DH ° - TDS ° = – 54.07 ´ 10 3 J – 298 × 10 J
Þ (Vgr - Vd )Dp = Gd ° - Ggr ° = 2.9 ´ 103 J
= – 57.05 × 103 J
2.9 ´ 103 29 29000 DG ° = - 2.303 RT log K
Þ Dp = Pa = ´ 108 Pa = bar Also,
2 ´ 10-6 2 2 - DG °
Þ log K =
29000 29000 2.303 RT
p = p0 + =1+ = 14501bar
2 2 57.05 ´ 103
= = 10
23. By first law, DE = Q + W 5705
For isothermal expansion, DE = 0
29. Entropy is a state function hence,
\ Q = -W
- Qirrev = W irrev = pDV = 3(2 - 1) = 3 L atm DS A ® B = DS A ® C + DSC ® D + DS D ® B
Q (- 3 ´ 101.3) J 303.9 = 50 eu + 30 eu + (– 20 eu)
Also, DS surr = irrev = =- = - 1.013 JK-1
T 300 K 300 = 60 eu
116 Thermodynamics and Thermochemistry

30. Given,
p
=1 Þ p=V ...(i) However, W AB ¹ p2 (V2 - V1 ) instead
V æV ö
W AB = nRT1 ln çç 2 ÷÷
Also from first law : dq = CV dT + pdV è V1 ø
For one mole of an ideal gas : pV = RT So, this option is incorrect.
Þ pdV + Vdp = RdT ...(ii) (ii) For option (b)
From (i) pdV = Vdp
qBC = DH AC = nC p (T2 - T1 )
Substituting in Eq. (ii) gives
where, C p = specific heat capacity at constant pressure
R
2pdV = RdT Þ pdV = dT Likewise,
2
R W BC = - p2 (V1 - V2 )
Þ dq = CV dT + dT
2 Hence, this option is correct.

w
dq R 3 R (iii) For option (c)
Þ ò dT = CV + 2 = 2 R + 2 = 2R as nC p (T2 - T1 ) < nCV (T2 - T1 )
31. For an irreversible, adiabatic process; so DHCA < DUCA

Flo
0 = CV (T2 - T1 ) + pe (V2 - V1 ) and qAC = DU BC
Substituting the values Hence, this option is also correct.

ree
CV (T - T2 ) = 1(2 - 1)atm L (iv) For option (d)
1 2 2
Þ T - T2 = = Þ T2 = T - Although qBC = DH AC
CV 3R 3 ´ 0.082

F
but DHCA >/ DUCA
32. In case of reversible thermodynamic process, Hence, this option is incorrect.
DH = nC p DT

or
ur
\ Process is isothermal, DT = 0 Þ DH = 0
40.
(a)

f
33. For a spontaneous process D G < 0 P1 P1
ks
Also; DG = DH - TDS –Wirr
Yo
P2 P2 P2 –Wrev P2
For adsorption of gas on solid surface, DS < 0. Therefore, in
oo
order to be DG < 0, DH must be negative.
eB

34. DH = DU + D ( pV ) = 30 + 2(5 - 3) + 5(4 – 2) = 44 L atm. V1 V2 V1 V2


Irreversible compression Reversible compression
35. Work is not a state-function, it depends on path followed.
36. In a reversible thermodynamic process, system always remains Maximum work is done on the system when compression
r

occur irreversibly and minimum work is done is reversible


ou

in equilibrium with surroundings.


ad

compression.
37.
(b)
Y

P1 A
nd

DH > 0
Re

E B
C
Fi

Reaction coordinates
V1 V2
Minimum value of activation energy must be greater than DH .
AB is isothermal and AC is adiabatic path. Work
38. DH = DE + DngRT Þ DH - DE = DngRT = - 3RT done is area under the curve. Hence, less work is
= - 3 ´ 8.314 ´ 298 = - 7433 J = – 7.43 kJ obtained in adiabatic process than in isothermal
39. In the given curve AC represents isochoric process as volume (c) It is incorrect. In adiabatic expansion cooling is observed,
at both the points is same i.e., V1 hence DU = nCvDT < 0.
Similarly, AB represents isothermal process (as both the points (d) q = 0 (adiabatic), W = 0 (Free expansion)
are at T1 temperature) and BC represents isobaric process as Hence, DU = 0, DT = 0 (Isothermal)
both the points are at p2 pressure. - DH
Now (i) for option (a) 41. DS surr =
Tsurr
qAC = DU BC = nCV (T2 - T1 )
where, n= number of moles For endothermic reaction, if Tsurr increases, DSsurr will increase.
For exothermic reaction, if Tsurr increases, DSsurr will decrease.
Cv = specific heat capacity at constant volume
Thermodynamics and Thermochemistry 117

42. PLAN This problem includes concept of isothermal adiabatic Therefore, W X ® Y ® Z = W X ® Y. Also, work is the area under the
irreversible expansion.
curve on p-V diagram.
Process is adiabatic because of the use of thermal insolution
therefore, q = 0 X Y X Y
Q pext = 0
WX ® Y
w = pext × DV = 0 ´ DV = 0 = WX®Y®Z
Internal energy can be written as WX®Y
Z Z
DU = q + W = 0 p p

The change in internal energy of an ideal gas depends only on


temperature and change in internal energy (DU ) = 0 therefore,
V V
DT = 0 hence, process is isothermal and
As shown above W X ® Y + WY ® Z = W X ® Y = W X ® Y® Z but not

w
T2 = T1
equal to W X ® Z.
and p2V2 = p1V1
46. Resistance and heat capacity are mass dependent properties,
(d) p2V2g = p1V1g is incorrect, it is valid for adiabatic
hence they are extensive.

Flo
reversible process.
47. Internal energy, molar enthalpy are state function. Also,
Hence, only (a), (b) and (c) are correct choices. reversible expansion work is a state function because between

ree
43. PLAN When an ideal solution is formed process is spontaneous thus given initial and final states, there can be only one reversible path.
According to Raoult’s law, for an ideal solution 48. Intensive properties are those property which do not depends

F
DH = 0, DVmin = 0 on amount of sample. Both temperature and refractive index
From the relation are intensive properties while enthalpy and volumes are
D G = D H - TD S extensive properties as they depends on amount of sample.

or
Since, DH = 0
ur 49. Statement I is true, it is statement of first law of thermodynamics.
\ DG = - ve

f
Statement II is true, it is statement of second law of
i.e. less than zero. and DS surroundings = 0 thermodynamics. However, Statement II is not the correct
ks
Therefore, DS sys = + ve explanation of statement I.
Yo
i.e. more than zero.
oo
50. Statement I is false. At equilibrium
44. (a) Since, change of state ( p1 , V1 , T1 ) to ( p2 , V2 , T2 ) is isothermal DG = 0, G ¹ 0.
eB

therefore, T1 = T2.
Statement II is true, spontaneous direction of reaction is towards
(b) Since, change of state ( p1 , V1 , T1 ) to ( p3 , V3 , T3 ) is an adiabatic
lower Gibbs free energy.
expansion it brings about cooling of gas, therefore, T3 < T1.
r

(c) Work done is the area under the curve of p-V diagram. As 51. Statement I is true.
ou

dq = dE + pext dV = 0
ad

obvious from the given diagram, magnitude of area under the


isothermal curve is greater than the same under adiabatic DT = 0
Y

curve, hence Wisothermal > Wadiabatic \ dE = 0 ; pext = 0


(d) DU = nCv DT \ pext dV = 0
In isothermal process, DU = 0 as DT = 0
nd
Re

Statement II is true. According to kinetic theory of gases,


In adiabatic process, DU = nC v (T3 - T1 ) < 0 as T3 < T1 .
volume occupied by molecules of ideal gas is zero.
Þ DU isothermal > DU adiabatic
Fi

However, Statement II is not the correct explanation of Statement I.


NOTE Here only magnitudes of work is being considered
otherwise both works have negative sign. 52. L ® M At constant V — isochoric,
45. (a) Entropy is a state function, change in entropy in a cyclic N®K
process is zero. 1
53. PLAN By Boyle’s law at constant temperature, p µ
Therefore, DS X ®Y + DS Y ®Z + DS Z ®X =0 V
Þ - DS Z ® X = DS X ® Y + DSY ® Z By Charles’ law at constant pressure, V µ T
= DS X ® Z Process taking place at
Constant temperature — isothermal
Analysis of options (b) and (c)
Constant pressure — isobaric
Work is a non-stable function, it does depends on the path Constant volume — isochoric
followed. WY ® Z = 0 as DV = 0.
Constant heat — adiabatic
118 Thermodynamics and Thermochemistry

K ¾® L At constant p, volume thus, heating 60. R : For an ideal gas, C p - CV = R


increases
3 3
L ¾® M At constant V, pressure thus, cooling 61. 900 cal : E = RT = ´ 2 ´ 300 cal
2 2
decreases
62. True First law deals with conservation of energy while second
M ¾® N At constant p, volume thus, cooling law deals with direction of spontaneous change.
decreases
63. True Diatomic gases have more degree of freedom than a
N ¾® K At constant V, pressure thus, heating monatomic gas.
increases
64. Work done along dashed path |- W | = SpDV
54. (A) ® r, t ; (B) ® p, q, s ; (C) ® p, q, s ; (D) ® q, s, t 2
= 4 ´ 1.5 + 1 ´ 1 + ´ 2.5 = 8.65 L atm
(A) H2O(l ) s
0°C
H2O(s) 3

w
1 atm V2 V
Work done along solid path - W = nRT ln = p1V1 ln 2
q< 0, W < 0 (expansion) V1 V1
DS sys < 0 (solid state is more ordered than liquid state) 5.5
DU < 0 ; DG = 0 (At equilibrium) = 2 ´ 2.3 log = 2 ´ 2.3 log 11 = 4.79

Flo
0.5
(B) q = 0 (isolated), W = 0 (pext = 0) W d 8.65
D Ssys > 0 Q V2 > V1 Þ = = 1.80 » 2
W s 4.79

ree
DU = 0 Q q = W = 0
DG < 0 Q p2 < p1 65. DH = DU + D ( pV ) = DU + VDp

F
(C) q = 0 (isothermal mixing of ideal gases at constant p) Þ DU = DH - VDp
W = 0Q DU = 0; q = 0, DS sys > 0 = – 560 – 1 ´ 30 ´ 0.1 = – 563 kJ

or
Q V2 > V1, DU = 0
ur 66. DU = q + W
QD T = 0

f
For adiabatic process, q = 0, hence DU = W
D G < 0Q mixing is spontaneous.
ks
(D) q = 0 (returning to same state and by same path) W = - p (DV ) = - p (V2 - V1 )
Yo
W =0 Þ DU = - 100 (99 - 100) = 100 bar mL
oo
DS sys = 0 (same initial and final states) DH = DU + D ( pV )
DU = 0
eB

where, DpV = p2V2 - p1V1


Q Ti = T f , D G = 0
67. He is monatomic, so it has only three degree of freedom
55. (A) CO2 (s) ¾
¾® CO2 (g ) (translational only) at all temperature hence, CV value is
3
r

It is just a phase transition (sublimation) as no chemical always R.


ou

2
ad

change has occurred. Sublimation is always endothermic.


Hydrogen molecule is diatomic, has three translational, two
Product is gas, more disordered, hence DS is positive.
Y

rotational and one vibrational degree of freedom. The energy


(B) CaCO3 (s) ¾ ¾® CaO(s) + CO2 (g ) spacing between adjacent levels are in the order of :
It is a chemical decomposition, not a phase change. Thermal translational < rotational < vibrational
nd
Re

decomposition occur at the expense of energy, hence At lower temperature only translational degree of freedom
endothermic. Product contain a gaseous species, hence, DS > 0. contribute to heat capacity while at higher temperature
Fi

(C) 2H ¾ ¾® H2 (g ) rotational and vibrational degree of freedom starts contributing


A new H—H covalent bond is being formed, hence, DH < 0. to heat capacity.
Also, product is less disordered than reactant, DS < 0. 68. (i) 1
(D) Allotropes are considered as different phase, hence 1
P(white, solid) ¾® P(red, solid) is a phase transition as well as
2
allotropic change. p
Also, red phosphorus is more ordered than white 0.5 3
phosphorus, DS < 0.
56. Extensive : Enthalpy is an extensive property while molar
20 L 40 L
enthalpy is an intensive property. V
57. Zero : - W = pDV = 0Q DV = 0
(ii) - W 1 = p DV = 20 L atm
58. Exothermic reaction. W 2 = 0 Q DV = 0
59. Isolated This system neither exchange matter nor energy with 40
W 3 = nRT ln = 20 ln 2
surroundings. 20
Thermodynamics and Thermochemistry 119

Total work done = W1 + W2 + W3 72. Let the mixture contain x litre of CH4 and 3.67 - x litre of
= - 20 L atm + 0 + 20 ln 2 ethylene.
= – 6.14 atm CH4 + O2 ¾® CO2
From first law : q = DE + (- W ) = - W x x
(Q DE = 0 for cyclic process) C2H4 + O2 ¾® 2CO2
3.67 - x 2 (3.67 - x)
Þ q = 6.14 L atm = 622.53 J
(iii) All the states function, DU , DH and DS are zero for cyclic Given : x + 2 (3.67 - x ) = 6.11 L
process. Þ x = 1.23 L
Volume of ethylene = 2.44 L
69. At equilibrium : B r A
pV 1 ´1
95.2% 1.3% Total moles of gases in l litre = = = 0.04
RT 0.082 ´ 298

w
13
Þ K1 = Also, CH4 and ethylene are in 1 : 2 volume (or mole) ratio, moles
952
0.04 2 ´ 0.04
B r C of CH4 = and moles of ethylene =
3 3

Flo
95.2% 3.5% 0.04
35 Þ Heat evolved due to methane = ´ 891 = 11.88 kJ
Þ K2 = 3
952 2 ´ 0.04

ree
Heat evolved due to ethylene = ´ 1423 = 37.94 kJ
Þ DG1° = - RT ln K 1 3
13 Þ Total heat evolved on combustion of 1.0L gaseous mixture
= – 8.314 ´ 448 ´ 2.303 log

F
= 16 kJ at 25°C is 11.88 + 37.94 = 49.82 kJ
952
1560
DG2° = - RT ln K 2 73. Moles of H2O needs to perspire = = 17.72

or
2 ´ 44
ur 35
= – 8.314 ´ 448 ´ 2.303 log
Weight of water needs to perspire = 17.72 ´ 18 = 318.96 g

f
952
= 12.3 kJ 74. At constant pressure, q = DH .
ks
70. D rG ° = D f G ° (products) – D f G ° (reactants)
Yo
oo
= – 394.4 – (– 137.2) = – 257.2 kJ < 0
Topic 2 Thermochemistry
eB

The above negative value of DG indicates that the process is Key Idea When q is the amount of heat involved
1.
spontaneous.
in a system then at constant pressure
Also, DG ° = DH ° - TDS ° q = qp and C p DT = DH
r

Þ D H ° = D G ° + TD S °
ou

Given reaction :
ad

= – 257.2 + 300 (– 0.094)


I2 (s) ¾® I2 (g )
= – 285.4 kJ < 0
Y

Specific heat of I2 (s) = 0.055 cal g -1 K -1.


71. Given : CV = 12.49 Þ C p = 20.8 Specific heat of I2 (vap) = 0.031 cal g –1K –1.
nd
Re

Cp
Þ = g = 1.66 Enthalpy (H 1 ) of sublimation of iodine = 24 cal g –1
CV
If q is the amount of heat involved in a system then at constant
Fi

In case of reversible adiabatic expansion : pressure q = qp and


TV g- 1= constant DH = C p DT
g-1 0.66 H 2 - H 1 = C p (T2 - T1 )
T2 æ V1 ö æV ö
Þ =ç ÷ = çç 1 ÷÷ H 2 = H 1 + DC p (T2 - T1 )
T1 çè V2 ÷ø è V2 ø
H 2 = 24 + (0.031 - 0.055) (250 - 200)
0.66
æV ö H 2 = 24 + (-0.024 ) (50) = 24 - 12
. = 22. 8 cal/g
Þ T2 = T1 çç 1 ÷÷
è V2 ø Thus, the enthalpy of sublimation of iodine at 250°C is
0.66
22.8 cal/g.
æ 1ö
= 300 ç ÷ = 189.86 K 2. Second equation given in this question is wrong. Hence, No
è 2ø
answer in correct.
Þ DH = nCp DT
If corrected second equation is given,
1 ´ 1.25 1
= ´ 20.8 ´ (189.86 - 300) J i.e. C(graphite) + O2 (g ) ¾® CO (g )
0.082 ´ 300 2
= – 116.4 J
120 Thermodynamics and Thermochemistry

and if we take the above reaction in consideration then x = y + z D cH ° (glucose) = 6[ D f H ° (CO2 ) + D f H °(H2O )]
will be the answer as:
1 - [ D f H ° (C6H12O6 ) + 6D f H ° (O 2 )]
(ii) C(graphite) + O2 (g ) ¾® CO(g ), D r H ° = y kJ/mol
2 = 6[ -400 - 300 ] - [ -1300 + 6 ´ 0 ]
1 = - 2900 kJ mol -1
(iii) CO(g ) + O2 (g ) ¾® CO2 (g ), D r H ° = z kJ/mol
2 Molar mass of C 6H12O6 = 180 g mol -1
Summing up both the equation you will get equation (i):
Thus, standard heat of combustion of glucose per gram
C(graphite) + O2 (g ) ¾® CO2 (g ), D r H ° = x kJ/mol
-2900
Hence, x, y and z are related as: = = - 16.11 kJ g-1
180
x =y+ z
To solve such problem, students are advised to keep much
3. Based on given D rH ° importance in unit conversion. As here, value of R

w
D f H ° = H CO
° = - 393.5 kJ mol - 1 …(i) (8.314 J K-1 mol -1) in JK-1 mol -1 must be converted into kJ by
2
dividing the unit by 1000.
D f H ° = H °H 2 O = - 285.8 kJ mol - 1 …(ii)
7. For calculation of C ºº C bond energy, we must first calculate
D f H ° = H O° 2 = 0.00 (elements)

Flo
…(iii) dissociation energy of C2H2 as
Required thermal reaction is for D f H ° of CH4 C2H2 (g ) ¾® 2C(g ) + 2H(g ) …(i)

ree
Thus, from III Using the given bond energies and enthalpies :
890.3 = [ D f H ° (CH4 ) + 2 D f H ° (O2 )] C2H2 (g ) ¾® 2C (g ) + 2H (g ) ; DH = - 225 kJ …(ii)
2C( s) ¾® 2C(g ); DH = 1410 kJ …(iii)

F
- [ D f H ° (CO2 ) + 2 D f H ° (H2O)] H2 (g ) ¾® 2H(g ) ; DH = 330 kJ …(iv)
= D f H ° (CH4 ) + 0] - [ - 393.5 - 2 ´ 285.5] Adding Eqs. (ii), (iii) and (iv) gives Eq. (i).

or
ur Þ C2H2 (g ) ¾® 2C(g ) + 2H(g ) ; DH = 1515 kJ
\ D f H ° (CH4 ) = - 74.8 kJ / mol

f
Þ 1515 kJ = 2 ´ (C ¾ H) BE + (C ºº C) BE
4. C(s)+ O2 (g ) ¾® CO2 (g ) ; DH = - 393.5 kJ mol -1 …(i) = 2 ´ 350 + (C ºº C) BE
ks
1 Þ (C ºº C) BE = 1515 - 700 = 815 kJ / mol
CO+ O2 ¾® CO2( g ); DH = - 283.5 kJ mol -1 …(ii)
Yo
2
oo
On subtracting Eq. (ii) from Eq. (i), we get
8. Elements in its standard state have zero enthalpy of formation.
Cl 2 is gas at room temperature, therefore DH °f of Cl 2 (g ) is zero.
eB

1
C (s) + O2 (g ) ¾® CO(g );
2 9. C—C bond energy is approximately 100 kcal.
DH = (- 393.5 + 283.5)kJ mol -1 DH vap 30,000
10. T = = = 400 K
r

= - 110 kJ mol-1(approx.) DS vap 75


ou
ad

5. C2H5OH ( l ) + 3O2 (g ) ¾® 2CO2 (g ) + 3H2O(l ) 1 1


11. H2 (g ) + F2 (g ) ¾® HF(g )
Y

DU = - 1364.47 kJ/mol 2 2
DH = DU + DngRT Here DH ° = Standard molar enthalpy of formation of HF(g).
Dng = - 1
nd

12. CO2 (g ) + H2 (g ) ¾® CO(g ) + H2O(g )


Re

- 1 ´ 8.314 ´ 298
DH = - 1364.47 + DH = SD f H ° (products) - SD f H ° (reactants)
1000
Fi

= – 110.5 – 241.8 – (– 393.5) = + 41.20 kJ


[Here, value of R in unit of J must be converted into kJ]
= - 1364.47 - 2. 4776 = - 1366.9476 kJ/mol 1
13. H2O ¾® H2 + O2, DH > 0
or = - 1366.95 kJ/mol 2
It is reverse of combustion of H2 (g ), hence endothermic.
6. PLAN D c H ° (Standard heat of combustion) is the standard enthalpy C2H6 ¾® C2H4 + H2; DH > 0
change when one mole of the substance is completely oxidised.
Here, more stable (saturated) hydrocarbon is being transformed
Also standard heat of formation (D f H ° ) can be taken as the
to less stable (unsaturated) hydrocarbon, hence endothermic.
standard of that substance.
C(gr) ¾® C(d) , DH > 0
H °CO 2 = D f H ° (CO2 ) = - 400 kJ mol -1
More stable allotrope is being converted to less stable allotrope.
H H° 2 O = D f H ° (H2O ) = - 300 kJ mol -1
14. PLAN Heat of reaction is dependent on temperature (Kirchhoff’s
°
Hglucose = D f H ° (glucose) = - 1300 kJ mol -1 equation) in heterogeneous system, equilibrium constant is
independent on the molar concentration of solid species.
H O° 2 = D f H ° (O2 ) = 0.00 Heat of reaction is not affected by catalyst. It lowers activation
energy.
C6H12O6 (s) + 6 O2 (g ) ¾® 6 CO2 (g ) + 6H2O(l )
CaCO3 (s) r CaO(s) + CO2 (g )
Thermodynamics and Thermochemistry 121

By Kirchhoff’s equation, 2H2 (g ) ¾® 4H (g );


DH °2 (at T2 ) = DH °1 ( at T1 ) + DC p (T2 - T1 ) DH = 2 ´ 2 ´ 218 = 872 kJ
DH ° varies with temperature. Thus, (a) is correct. 1
O2 (g ) ¾® O (g ); DH = 249 kJ
K = pCO2 2
K is dependent on pressure of CO2 but independent of molar 1
Adding : C (gr) + 2H2 (g ) + O2 (g ) ¾® CH3OH (l )
concentration of CaCO3. Thus, (b) and (c) are correct. At a given 2
temperature, addition of catalysis lowers activation energy, DH DH = - 266 kJ/mol
remaining constant. Thus, (d) is also correct. 1 ´ 1.25
= ´ 20.8 ´ (189.86 - 300) J
0.082 ´ 300
Ea = -116.4 J

w
CaO + CO2 20.
T
E'a + H2 ; DH = –119
DH°

Flo
CaCO3 E'a <E a
Þ + 3H 2 ;

ree
Ea = Activation energy in absence of catalyst
DH = –119 × 3 = – 357 kJ (Theoretical)
Ea¢ = Activation energy in presence of catalyst

F
– 357 kJ = DH °f (cyclohexane) - DH °f (C6H6 )
15. Temperature rise = T2 - T1 = 298.45 – 298 = 0.45 K
q = heat capacity ´ DT = 2.5 ´ 0.45 = 1.125 kJ Þ DH °f (C6H6 )Theoretical = – 156 + 357 = 201 kJ

or
ur
1.125
Þ Heat produced per mole = ´ 28 = 9 kJ Þ Resonance energy = DH °f (exp.) - DH °f (Theoretical)

f
3.5
16. DH r° = DH f° (B2O3 ) + 3DH °f (H2O) – DH °f (B2H6 ) = 49 – 201 = – 152 kJ/mol
ks
Yo
DH °f (H2O)(g ) = DH °f (H2O)(l ) + 44 = – 242 kJ 21. Per mole of ethylene polymerized, one C == C bond is broken
oo
and two C—C bonds are formed.
Þ DH r° = – 1273 – 3 ´ 242 – 36 DH ° (Polym.) = 590 - 2 ´ 311
eB

= – 2035 kJ = – 32 kJ/mol
17. SF6 (g ) ¾® S (g ) + 6F (g ) 22. At same temperature and pressure, equal volumes contain equal
r

DH = S DH °f (products) - S DH °f (reactants) moles of gases.


ou
ad

Let 1.0 L of CH4 contain ‘n’ mol


= 275 + 6 ´ 80 + 1100 = 1855 kJ
Þ x L of CH4 contain nx mol
Y

1855
Þ Average S—F bond energy = = 309.16 kJ/mol
6 Þ Heat evolved in combustion by x L CH4 = 809 nx kJ
æ1 ö
18. Given : Cyclopropane ¾®Propene (C3H6 ); DH = - 33 kJ
nd

Now, 2878 kJ energy is evolved from 1 mole ç L ÷ C4H10.


Re

èn ø
9
Propene (C3H6 ) + O2 ¾® 3CO2 (g ) + 3H2O (l ); 809 nx
Fi

2 Þ 809 nx kJ energy will be evolved from L of C4H10


DH = – 3 (393.5 + 285.8) – 20.42 = – 2058.32 kJ 2878 n
Adding : = 0.28 x L of C4H10
9 Also, the combustion reaction of butane is
Cyclopropane + O2 (g ) ¾® 3CO2 (g ) + 3H2 (g ) ;
2 13
C4 H10 + O2 ¾® 4CO2 + 5H2O
DH = H 1 + H 2 2
= -33 + (-2058.32)kJ 13
DH = - 2091.32 kJ Þ Rate of supply of oxygen = ´ 0.28 x ´ 3
2
19. Given : CH3OH (g ) ¾® CH3OH (l ); DH = - 38 kJ = 5.46 x L/h
C (g ) + 4H (g ) + O (g ) ¾® CH3OH (g ); 23. First we need to determine heat of combustion of C3H8 .
DH = - (3 ´ 415 + 356 + 463)
Q H = H 1 + H 2 = - 2064 kJ 3C(gr) + 4 H2 (g ) ¾® C3H8 (g ) DH °f = - 103 kJ
C(g ) ¾® C(g ); DH = 715 kJ Þ °
– 103 kJ = – 3 ´ 393 – 4 ´ 285.80 – D H comb (C3H8 )
122 Thermodynamics and Thermochemistry

° 1 1
Þ D H comb (C3H8 ) = – 2219.20 kJ 28. H2 (g ) + Cl 2 (g ) ¾® HCl (g ); DH °f
2 2
Þ DH r° = S D H comb
° °
(reactants) - S D H comb (products)
DH °f = S BE (reactants) - S BE (products)
= – 2219.20 – 285.80 + 1560 + 890
1
= – 55 kJ = (104 + 58) - 103 = – 22 kcal/mol
2
24. Let x kcal be the C—C bond energy and y kcal be the C—H bond
energy per mole. 29. C2H2 + H2 ¾® C2H4
Þ 2C(gr) + 3H2 (g ) ¾® C2H6 (g ) ; DH ° = S D H comb
° °
(reactants) - S D H comb (products)
DH ° = – 2 ´ 94 – 3 ´ 68 + 372 = – 310.6 – 68.3 – (– 337.2)
= – 20 kcal = – 41.7 kcal

w
Þ – 20 kcal = 2 ´ 172 + 3 ´ 104 - BE (C2H6 )
30. The standard state formation reaction of C2H2 (g ) is :
Þ BE (C2H6 ) = 676 kcal
2C(g ) + H2 (g ) ¾® C2H2 (g ); DH °f
Similarly, 3C(gr) + 4 H2 (g ) ¾® C3H8 (g ) ;
DH r° = S D H comb
° °
(reactants) - S D H comb (products)

Flo
DH ° = – 3´ 94 – 4 ´ 68 + 530
= – 24 kcal = – 2 ´ 94.05 – 68.32 – (– 310.62)
= 54.2 kcal = DH f° (C2H2 )

ree
Þ – 24 kcal = 3 ´ 172 + 4 ´ 104 - BE (C3H8 )
Þ BE (C3H8 ) = 956 kcal 31. DH r° = S D f H ° (products) - S D f H ° (reactants)

F
Also, BE (C2H6 ) = 676 kcal = x + 6 y …(i) = – 94.1 + 4 (– 22.1) – (– 25.5 – 2 ´ 57.8)
BE (C3H8 ) = 956 kcal = 2x + 8 y …(ii) = – 41.4 kcal

or
Solving Eqs. (i) and (ii) gives
ur 32. DH ° = S BE (reactants) - S BE (products)
y = 99 kcal (C—H) BE

f
1 1
Þ - 10.06 =
(104.18) + (118.32) - BE (O ¾ H)
x =82 kcal (C—C) BE 2 2
ks
BE (O—H) = 121.31 kcal
Yo
33. Let C JK-1 be the heat capacity of calorimeter.
oo
25. + H2
Mass of solution = 200 mL ´ 1 g mL-1 = 200 g
eB

°
DH = S D H comb °
(reactants) - S D H comb (products) Heat evolved in Expt.1
= 57 ´ 1000 ´ 0. 1(mol ) = 5700 J
= – 3800 – 241 – (– 3920)
Þ 5700 J = (200 ´ 4.2 + C ) ´ 5.7
r

= – 121 kJ/mol
Þ 1000 = 200 ´ 4.2 + C …(i)
ou
ad

26. Fe2O3 (s) + 2Al (s) ¾® Al 2O3 (s) + 2Fe(s) Let x kJ/mol is heat evolved in neutralisation of acetic acid.
Y

Þ x ´ 1000 ´ 010 . = (200 ´ 4.2 + C ) ´ 5.6


DH r° = DH f° (products) - DH °f (reactants)
x ´ 100
Þ = 200 ´ 4.2 + C …(ii)
= – 399 – (– 199) 5.6
nd
Re

= – 200 kcal From (i) and (ii) : x = 56 kJ/mol


Mass of reactants = 56 ´ 2 + 16 ´ 3 + 27 ´ 2 = 214 g Þ Enthalpy of ionisation of acetic acid
Fi

= - 56 - (- 57) = 1 kJ/mol
200
Þ Fuel value/gram = = 0.93 kcal/g
214 34. CH3COOH + NaOH ® CH3COONa + H2O
160 54 200 mmol 100 mol 0 0
Volume of reactants = cc + cc = 50.77 cc 100 mmol 0 100 mmol
5.2 2.7
A buffer is now formed.
200
Þ Fuel value/cc = = 3.94 kcal/cc [H+ ] [ CH3 COO- ]
50.77 Ka = = [ H+ ]
[CH3 COOH]
27. DH = S DH °f (products) - S DH °f (reactants) [Q [ CH3 COOH] = [ CH3 COO- ]]
= – 2 ´ 94.1 – 3 ´ 68.3 – (– 21.1) Þ pH = pK a = - log ( 2 ´ 10-5 ) = 5 - log 2 = 4.7
= – 372 kcal/mol

Download Chapter Test


http://tinyurl.com/y6ylao38 or
8
Solid State

w
Objective Questions I (Only one correct option) 6. The radius of the largest sphere which fits properly at the

Flo
1. The ratio of number of atoms present in a simple cubic, body centre of the edge of a body centred cubic unit cell is
centered cubic and face centered cubic structure are, (Edge length is represented by ‘a’) (2019 Main, 11 Jan II)
(a) 0.134 a (b) 0.027 a (c) 0.047 a (d) 0.067 a

ree
respectively. (2019 Main, 12 April II)
(a) 8 : 1 : 6 (b) 1 : 2 : 4 7. A solid having density of 9 ´ 103 kgm -3 forms face centred
(c) 4 : 2 : 1 (d) 4 : 2 : 3

F
cubic crystals of edge length 200 2 pm. What is the molar
2. An element has a face-centred cubic (fcc) structure with a cell mass of the solid?
edge of a. The distance between the centres of two nearest [Avogadro constant = 6 ´ 1023 mol-1 , p = 3]

or
tetrahedral voids in the lattice is
ur (2019 Main, 12 April I) (2019 Main, 11 Jan I)
(a) 0.03050 kg mol-1 (b) 0.4320 kg mol-1

f
a 3
(a) 2a (b) a (c) (d) a
2 2 (c) 0.0432 kg mol-1 (d) 0.0216 kg mol-1
ks
3. Consider the bcc unit cells of the solids 1 and 2 with the
Yo
8. A compound of formula A2B3 has the hcp lattice. Which
oo
position of atoms as shown below. The radius of atom B is atom forms the hcp lattice and what fraction of tetrahedral
twice that of atom A. The unit cell edge length is 50% more is voids is occupied by the other atoms ? (2019 Main, 10 Jan II)
eB

solid 2 than in 1. What is the approximate packing efficiency 2


(a) hcp lattice- A, tetrahedral voids-B
in solid 2? (2019 Main, 8 April II) 3
1
A A (b) hcp lattice-A, tetrahedral voids-B
r

A A
3
ou

A 1
ad

A A A (c) hcp lattice-B, tetrahedral voids-A


B 3
Y

A 2
A A A (d) hcp lattice-B, tetrahedral voids-A
A 3
nd
Re

A A A A 9. Which primitive unit cell has unequal edge lengths


Solid 1 Solid 2 ( a ¹ b ¹ c ) and all axial angles different from 90°?
Fi

(2019 Main, 10 Jan I)


(a) 65% (b) 90% (a) Hexagonal (b) Monoclinic
(c) 75% (d) 45% (c) Tetragonal (d) Triclinic
4. The statement that is incorrect about the interstitial 10. At 100°C, copper (Cu) has FCC unit cell structure with cell
compounds is (2019 Main, 8 April II)
edge length of x Å. What is the approximate density of Cu (in g
(a) they are very hard
cm -3 ) at this temperature?
(b) they have metallic conductivity
[Atomic mass of Cu = 63.55 u] (2019 Main, 9 Jan II)
(c) they have high melting points
211 205
(d) they are chemically reactive (a) 3 (b) 3
x x
5. Element ‘B ’ forms ccp structure and ‘A ’ occupies half of the 105 422
octahedral voids, while oxygen atoms occupy all the (c) 3 (d) 3
x x
tetrahedral voids. The structure of bimetallic oxide is
(2019 Main, 8 April I) 11. The one that is extensively used as a piezoelectric material
(a) A2BO4 (b) AB2O4 is (2019 Main, 9 Jan I)
(c) A2B2O (d) A4B2 O (a) quartz (b) tridymite
(c) amorphous silica (d) mica
124 Solid State

12. Which type of ‘defect’ has the presence of cations in the 20. The packing efficiency of the two-dimensional square unit
interstitial sites? (2018 Main) cell shown below is (2010)
(a) Schottky defect (b) Vacancy defect
(c) Frenkel defect (d) Metal deficiency defect
13. A metal crystallises in a face centred cubic structure. If the
edge length of its unit cell is ‘a’, the closest approach between
two atoms in metallic crystal will be (2017 Main) l
(a) 2 a (b) 2 2 a (a) 39.27% (b) 68.02% (c) 74.05% (d) 78.54%
a 21. Which of the following fcc structure contains cations in
(c) 2 a (d)
2 alternate tetrahedral voids? (2005, 1M)
(a) NaCl (b) ZnS (c) Na 2 O (d) CaF2

w
14. Sodium metal crystallises in a body centred cubic lattice with
a unit cell edge of 4.29Å. The radius of sodium atom is 22. A substance Ax B y crystallises in a face centred cubic (fcc)
approximately (2015 Main) lattice in which atoms A occupy each corner of the cube and
(a) 1.86Å (b) 3.22Å atoms B occupy the centres of each face of the cube. Identify

Flo
(c) 5.72Å (d) 0.93Å the correct composition of the substance Ax B y (2002, 1M)
15. CsCl crystallises in body centred cubic lattice. If ‘a’ its edge (a) AB3

ree
length, then which of the following expressions is correct? (b) A4 B3
(2014 Main) (c) A3 B
3a
(a) r + rCl - = 3a (b) r + rCl - = (d) composition cannot be specified

F
Cs + Cs + 2
23. In a solid AB having the NaCl structure, A atoms occupy the
3
(c) r + rCl - = a (d) r + rCl - = 3a corners of the cubic unit cell. If all the face centred atoms

or
Cs + 2
ur Cs +
along one of the axes are removed, then the resultant
16. The arrangement of X - ions around A + ion in solid AX is stoichiometry of the solid is

f
(2001, S, 1M)
given in the figure (not drawn to scale). If the radius of X - is (a) AB2 (b) A2 B (c) A4 B3 (d) A3 B4
ks
250 pm, the radius of A + is (2013 Adv.) 24. The coordination number of a metal crystallising in a
Yo
hexagonal close-packed structure is (1999, 2M)
oo

X (a) 12 (b) 4 (c) 8 (d) 6
A+
eB

Objective Questions II
(One or more than one correct option)
r

(a) 104 pm (b) 125 pm 25. The correct statement(s) for cubic close packed (ccp) three
ou
ad

(c) 183 pm (d) 57 pm dimensional structure is (are) (2016 Adv.)


17. Experimentally it was found that a metal oxide has formula (a) The number of the nearest neighbours of an atom present
Y

M 0.98O. Metal M, present as M 2+ and M 3 + in its oxide. in the topmost layer is 12


Fraction of the metal which exists as M 3+ would be (b) The packing efficiency of atom is 74%
nd

(a) 7.01% (b) 4.08% (c) The number of octahedral and tetrahedral voids per atom
Re

(2013 Main)
(c) 6.05% (d) 5.08% are 1 and 2, respectively
Fi

18. Which of the following exists as covalent crystals in the solid (d) The unit cell edge length is 2 2 times the radius of the atom
state? (2013 Main) 26. If the unit cell of a mineral has cubic close packed (ccp) array
(a) Iodine (b) Silicon of oxygen atoms with m fraction of octahedral holes
(c) Sulphur (d) Phosphorus occupied by aluminium ions and n fraction of tetrahedral
holes occupied by magnesium ions, m and n respectively, are
19. A compound M p X q has cubic close packing (ccp) (2015 Adv.)
arrangement of X . Its unit cell structure is shown below. The 1 1 1 1 1 1 1
(a) , (b) 1 , (c) , (d) ,
empirical formula of the compound, is (2012) 2 8 4 2 2 4 8
27. The correct statement(s) regarding defects in solids is/are
(a) Frenkel defect is usually favoured by a very small
M
difference in the sizes of cation and anion (1999)
X
(b) Frenkel defect is a dislocation defect
(c) Trapping of an electron in the lattice leads to the formation
of F-centre
(a) MX (b) MX 2 (c) M 2 X (d) M 5 X 14 (d) Schottky defects have no effect on the physical properties
of solids
Solid State 125

28. Which of the following statement(s) is/are correct? 31. The number of atoms in one of this hcp unit cell is
(a) The coordination number of each type of ion in CsCl crystal (2008, 3 ´ 4M = 12M)
is 8 (1998, 2M) (a) 4 (b) 6 (c) 12 (d) 17
(b) A metal that crystallises in bcc structure has a coordination 32. The volume of this hcp unit cell is
number of 12 64 r3
(a) 24 2 r3 (b) 16 2 r3 (c) 12 2 r3 (d)
(c) A unit cell of an ionic crystal shares some of its ions with 3 3
other unit cells 33. The empty space in this hcp unit cell is
(d) The length of the unit cell in NaCl is 552 pm. (a) 74 % (b) 47.6 % (c) 32 % (d) 26 %
(rNa + = 95 pm; rCl - =181 pm)
Match the Columns
Numerical Value 34. Match the crystal system / unit cells mentioned in Column I

w
29. Consider an ionic solid MX with NaCl structure. Construct a with their characteristic features mentioned in Column II.
new structure (Z) whose unit cell is constructed from the unit (2007, 6M)
cell of MX following the sequential instruction given below. Column I Column II
Neglect the charge balance. (2018 Adv.)

Flo
A. Simple cubic and face p. have these cell
(a) Remove all the anions (X ) except the central one centred cubic parameters a = b = c
(b) Replace all the face centered cations (M ) by anions (X ) and a = b = g = 90°

ree
(c) Remove all the corner cations (M ) B. Cubic and q. are two crystal
(d) Replace the central anion (X ) with cation (M ) rhombohedral systems

F
æ Number of anions ö C. Cubic and tetragonal r. have only two
The value of ç ÷ in Z is ___
è Number of cations ø crystallographic
angles of 90°

or
Assertion and Reason
ur D. Hexagonal and s. belong to same crystal

f
monoclinic system
Read the following questions and answer as per the direction
ks
given below :
(a) Statement I is correct Statement II is correct Statement II is
Integer Answer Type Questions
Yo
oo
the correct explanation of Statement I 35. A crystalline solid of a pure substance has a face-centred
(b) Statement I is correct Statement II is correct Statement II is cubic structure with a cell edge of 400 pm. If the density of
the substance in the crystal is 8 g cm -3 , then the number of
eB

not the correct explanation of Statement I


(c) Statement I is correct Statement II is incorrect atoms present in 256 g of the crystal is N ´ 1024 . The value of
(d) Statement I is incorrect Statement II is correct N is (2017 Adv.)
r

30. Statement I In any ionic soid (MX ) with Schottky defects, 36. The number of hexagonal faces that are present in a truncated
ou
ad

the number of positive and negative ions are same. octahedron is (2011)
37. Silver (atomic weight = 108 g mol -1 ) has a density of
Y

Statement II Equal numbers of cation and anion vacancies


are present. 10.5 g cm-3 . The number of silver atoms on a surface of area
(2001, 1M)
10-12 m2 can be expressed in scientific notation as y ´ 10 x.
nd

The value of x is
Re

(2010)
Passage Based Questions
Subjective Questions
Fi

Passage
38. The edge length of unit cell of a metal having molecular
In hexagonal systems of crystals, a frequently encountered
weight 75 g/mol is 5 Å which crystallises in cubic lattice. If
arrangement of atoms is described as a hexagonal prism. Here, the
the density is 2 g/cc then find the radius of metal atom.
top and bottom of the cell are regular hexagons and three atoms are
(N A = 6 ´ 1023 ). Give the answer in pm. (2006, 3M)
sandwiched in between them. A space-filling model of this
structure, called hexagonal close-packed (hcp), is constituted of a 39. An element crystallises in fcc lattice having edge length
sphere on a flat surface surrounded in the same plane by six 400 pm. Calculate the maximum diameter of atom which can
identical spheres as closely as possible. Three spheres are then be placed in interstitial site without distorting the structure.
placed over the first layer so that they touch each other and (2005, 2M)
represent the second layer. Each one of these three spheres touches 40. The crystal AB (rock salt structure) has molecular weight
three spheres of the bottom layer. 6.023 y u. Where, y is an arbitrary number in u. If the
Finally, the second layer is covered with a third layer that is minimum distance betweeen cation and anion is y1 / 3 nm and
identical to the bottom layer in relative position. Assume radius of the observed density is 20 kg / m3 . Find the (i) density in
every sphere to be ‘r ’. kg / m3 and (ii) type of defect. (2004, 2M)
126 Solid State

41. (i) Marbles of diameter 10 mm are to be put in a square area 43. A metal crystallises into two cubic phases, face centred cubic
of side 40 mm so that their centres are within this area. (fcc) and body centred cubic (bcc), whose unit cell lengths
(ii) Find the maximum number of marbles per unit area and are 3.5 and 3.0 Å, respectively. Calculate the ratio of
deduce an expression for calculating it. (2003, 4M)
densities of fcc and bcc. (1999, 3M)
44. Chromium metal crystallises with a body centred cubic
42. The figures given below show the location of atoms in three
lattice. The length of the unit edge is found to be 287 pm.
crystallographic planes in a fcc lattice. Draw the unit cell for Calculate the atomic radius . What would be the density of
the corresponding structures and identify these planes in chromium in g/cm 3 ? (1997, 3M)
your diagram. (2000)
45. A metallic element crystallises into a lattice containing a
sequence of layers of ABABAB…… Any packing of layers
leaves out voids in the lattice. What percentage of this lattice

w
is empty space? (1996, 3M)
46. Sodium crystallises in a bcc cubic lattice with the cell edge,
a = 4.29 Å. What is the radius of sodium atom? (1994, 2M)

Flo
Answers

ree
1. (b) 2. (c) 3. (b) 4. (d) 5. (b) 6. (d) 7. (a) 8. (c) 9. (d)

F
10. (d) 11. (a) 12. (c) 13. (d) 14. (a) 15. (c) 16. (a) 17. (b) 18. (b)
19. (b) 20. (d) 21. (b) 22. (a) 23. (d) 24. (a) 25. (b,c,d) 26. (a) 27. (b,c)

or
34. A ® p, s; B ® q; C ® q; D ® q, r
28. (a,c,d) 29. (3)
ur
30. (a) 31. (b) 32. (a) 33. (d)
35. (2) 36. (8) 37. (7) 38. (217 pm) 39. (117 pm) 43. (1.26) 44. (7.3 g/cm 3)
45. (0.74) 46. (1.86 Å)
f
ks
Yo
oo

Hints & Solutions


eB

1. The ratio of number of atoms present in simple cubic, body é aù a a


these two is ê a - ú - = .
centred cubic and face centered cubic structure are 1 : 2 : 4 ë 4û 4 2
respectively.
r
ou
ad

2. In fcc unit cell, two tetrahedral voids are formed on each of the 3. Key Idea Packing efficiency
four non-parallel body diagonals of the cube at a distance of
Y

Volume occupied by sphere


3a / 4 from every corner along the body diagonal. = ´ 100
Volume of cube
One of the body diagonal
Given,
nd

of cubic unit cell


Re

A
rB = 2rA
1
— AB 50
Fi

4
Ö3 a Position of 2 TVs a2 = a1 + a1 = 15
. a1

4
A¢ 1 on one of the 100
—(AB)
2 body diagonal For bcc lattice
of the unit cell
4 rA = 3 a1
B¢ 1 3 a1
—(AB)
4 rA =
B 4
Ö3 a 4 rA

4 a1 =
3
The angle between body diagonal and an edge is cos-1 (1/ 3 ). æ 4r ö
\ a2 = 15
. ç A÷
So, the projection of the line on an edge is a/ 4. Similarly, other è 3ø
tetrahedral void also will be a/4 away. So, the distance between 3 æ 4 rA ö
= ç ÷
2 è 3ø
Solid State 127

a2 = 2 3 rA a 3 2a - 3a
r= - a=
4 3 4 2 4 4
prA ´ zA + prB3 ´ zB
3 3 a(2 - 3 )
Packing efficiency = r=
a23 4
1 r = 0.067a
[As the atoms A are present at the edges only zA = ´ 8 = 1,
8 7. Density of a crystal
atom B is present only at the body centre zB = 1 ]
M ´Z d ´ N A ´ a3
æ4 3 ö æ4 3 ö d= 3
ÞM =
ç prA ´ 1÷ + ç prB ´ 1÷ NA ´a Z
PE 2 = è ø è3 ø
3
\ Given, d = 9 ´ 103 kg m -3
a23
M = Molar mass of the solid

w
4 3 4 4 3 Z = 4 (for fcc crystal)
prA + p (2rA )3 prA ´ 9
p
= 3 3
3
= 3 = N A = Avogadro’s constant = 6 ´ 1023 mol -1
(2 3 rA ) 8 ´ 3 3 rA3 2 3
a = Edge length of the unit cell
= 90.72% » 90%

Flo
= 200 2 pm = 200 2 ´ 10-12 m
4. Interstitial compounds are formed when a neutral atom with a On substituting all the given values, we get
small radius occupies in an interstitial hole (tetrahedral or

ree
(9 ´ 103 ) kg m -3 ´ (6 ´ 1023 ) mol -1 ´ (200 2 ´ 10-12 )3 m 3
octahedral voids) in a transition metal’s hcp or ccp lattices =
(host lattice). Examples of small atoms (guest atom) are H, B, C 4
= 0.0305 kg mol -1

F
and N.
Interstitial compounds are non-stoichiometric (Birtholide) in
composition. They are very hard with very high melting points.
8. Total effective number of atoms in hcp unit lattice = Number of

or
ur
The electrical conductivity of interstitial compounds are
octahedral voids in hcp = 6
comparable to that of the pure metal. These are chemically \Number of tetrahedral voids (TV) in hcp
unreactive in nature.
f = 2 ´ Number of atoms in hcp lattice
ks
5. The number of element ‘B ’ in the crystal structure = 4 N = 2 ´ 6 = 12
Yo
As, formula of the lattice is A2 B3 .
oo
Number of tetrahedral voids = 2N
Suppose, A B
Number of octahedral voids = N
æ1 ö
eB

N 4 ç ´ TV ÷ (hcp)
\Number of ‘A’ in the crystal = = =2 è3 ø
2 2
1
Number of oxygen (O) atoms = 2N = 2 ´ 4 = 8 Þ ´ 12 6
3
r

\The structure of bimetallic oxide = A2 B4 O8 = AB2 O4


ou

2
ad

Þ 1
6. For body centred cubic bcc structure, 3
Y

radius (R ) =
3
a ...(i) Þ 2 3
4 1
So, A = tetrahedral voids, B = hcp lattice
Where, a = edge length 3
nd
Re

According to question, the structure of cubic unit cell can be 9. Triclinic primitive unit cell has dimensions as, a ¹ b ¹ c and
shown as follows:
Fi

a ¹ b ¹ g ¹ 90° .
R 2r R
Among the seven basic or primitive crystalline systems, the
triclinic system is most unsymmetrical. In other cases, edge
length and axial angles are given as follows :
Hexagonal : a = b ¹ c and a = b = 90°, g = 120°
Monoclinic : a ¹ b ¹ c and a = g = 90°, b ¹ 90°
Tetragonal : a ¹ b ¹ c and a = b = g = 90°

10. For fcc, rank of the unit cell (Z ) = 4


a Mass of one Cu-atom, M = 63.55 u
Avogadro’s number, N A = 6.023 ´ 1023 atom
\ a = 2(R + r) ...(ii)
On substituting the value of R from Eq. (i) to Eq. (ii), we get Edge length, a = x Å = x ´ 10-8 cm
a 3 Z ´M
= a+ r density (d ) =
2 4 N A ´ a3
128 Solid State

4 ´ 63.55 422.048
= 23 -8 3
= g cm-3 16. PLAN Given arrangement represents octahedral void and for this
6.023 ´ 10 ´ (x ´ 10 ) x3 r+ (cation)
= 0.414
r- (anion)
11. Piezoelectric materials are those materials that produce an
r( A + )
electric current when they are placed under mechanical stress. = 0.414
Crystalline solids can be used as piezoelectric material hence, r( X - )
quartz is a correct answer. r( A + ) = 0.414 ´ r( X - ) = 0.414 ´ 250pm
12. It is the ‘‘Frenkel defect’’ in which cations leave their original = 103.5 pm » 104 pm
site and occupy interstitial site as shown below.
17. From the valency of M 2+ and M 3+ , it is clear that three
+ – + – + – + – Original M 2+ ions will be replaced by M 3+ causing a loss of one
– + – + – + – + vacant site
+ – + – + – + – of cation M 3+ ion. Total loss of them from one molecule of
MO = 1 - 0.98 = 0.02

w
– + – – + – + Cation in
+ – + – + – + –
– + –
+
+ – + – +
interstitial site Total M 3+ present in one molecule of
+ – + – + – + – MO = 2 ´ 0.02 = 0.04
– + – + – + – + That M 2+ and M 3+ = 0.98

Flo
0.04 ´ 100
13. For fcc arrangement, 4 r = 2a Thus, % of M 3+ = = 4.08%
0.98
where, r = radius and a = edge length

ree
18. Silicon exists as covalent crystal in solid state. (Network like
2a a structure, as seen in diamond).
\ Closest distance = 2 r = =

F
2 2 19. Contribution of atom from the edge centre is 1/4. Therefore,
number of
14. For bcc unit cell, 3 a = 4a 1

or
M = ´ 4 (from edge centre) + 1 (from body centre) = 2
3
ur 4
r= a 1 1

f
4 Number of X = ´ 8 (from corners) + ´ 6
8 2
ks
3
= ´ 4.29Å = 1.85Å (from face centre) = 4
Yo
4 Þ Empirical formula = M 2 X 4 = MX 2
oo
r = 1.85Å » 1.86Å 1
20. Contribution of circle from corner of square =
15. In CsCl, Cl - lies at corners of simple cube and Cs+ at the body 4
eB

centre. Hence, along the body diagonal, Cs+ and Cl - touch each Þ Effective number of circle per square
1
other so rCs+ + rCl - = 2r = ´ 4 + 1 (at centre) = 2
4
Calculation of r
r

Þ Area occupied by circle = 2pr2, r = radius.


In DEDF,
ou
ad

Also, diagonal of square 4 r = 2 L, where L = side of square.


G B Area occupied by circles
Y

Þ Packing fraction =
H r A Area of square
2pr2 2pr2 p
= 2 = = = 0.785
nd
2r

a
Re

c
r L 8r2 4
F C
b Þ % packing efficiency = 78.5%.
Fi

a
E a D 21. In ZnS, S2- (sulphide ions) are present at fcc positions giving
four sulphide ions per unit cell. To comply with 1 : 1
Body centred cubic unit cell stoichiometry, four Zn 2+ ions must be present in four alternate
tetrahedral voids out of eight tetrahedral voids present.
FD = b = a2 + a2 = 2 a
In NaCl, Na + ions are present in octahedral voids while in Na 2O,
In DAFD, Na + ions are present in all its tetrahedral voids giving the desired
c2 = a2 + b2 = a2 + ( 2 a)2 = a2 + 2 a2 2 : 1 stoichiometry. In CaF2 , Ca 2+ ions occupies fcc positions
c2 = 3 a2 Þ c = 3 a and all the tetrahedral voids are occupied by fluoride ions.
As D AFD is an equilateral triangle. 1
22. In cubic system, a corner contribute th part of atom to one unit
8
\ 3a = 4r [QC = 3r + r + r]
1
cell and a face centre contribute part of atom to one unit cell.
3a 2
Þ r=
4 Therefore,
3 3 1
Hence, rCs + + rCl- = 2 r = 2 ´ a= a Number of A per unit cell = ´ 8 = 1
4 2 8
Solid State 129

Number of B per unit cell =


1
´6= 3 æ4 ö
4 ç pr3 ÷
2 Volume of 4 spheres è 3 ø ´ 100
= =
Þ Formula = AB3 Volume of unit cell [ 2( 2r)]3
23. In NaCl, Na + occupies body centre and edge centres while Cl - ~ 74%
= 74.05% -
occupies corners and face centres, giving four Na + and four Cl - 26. Oxide ions are at ccp positions, hence 4O2– ions. Also, there
per unit cell. In the present case A represent Cl - and B represents are four octahedral voids and eight tetrahedral voids. Since ‘m’
Na + . Two face centres lies on one axis. fraction of octahedral voids contain Al 3+ and ‘n’ fraction of
1
Þ Number of A removed = 2 ´ = 1 tetrahedral voids contain Mg2+ ions, to maintain
2
Number of B is removed because it is not present on face centres. etectroneutrality 2(2Al 3+ = + 6charge) and
Þ A remaining = 4 - 1 = 3, B remaining = 4, (Mg2+ = + 2charge), will make unit cell neutral
Formula = A3 B4 2 1 1

w
Hence: m = = , n=
4 2 8
24. Three consecutive layers of atoms in hexagonal close packed
lattice is shown below: 27. (a) Wrong statement. A small difference in sizes of cation and
anion favour Schottky defect while Frenkel defect is

Flo
A favoured by large difference in sizes of cation and anion.
(b) Correct statement. In Frenkel defect the smaller atom or ion
gets dislocated from its normal lattice positions and occupies

ree
the interstitial space.
(c) Correct Statement In F-centre defect, some anions leave

F
X B
the lattice and the vacant sites hold the electrons trapped in it
maintaining the overall electroneutrality of solid.

or
ur (d) Wrong statement : In Schottky defect, some of the atoms or
ions remaining absent from their normal lattice points

f
A
without distorting the original unit cell dimension. This
lowers the density of solid.
ks
Atom X is in contact of 12 like atoms, 6 from layer B and 3 from
Yo
28. (a) The unit cell of CsCl has bcc arrangement of ions in which
oo
top and bottom layers A each. each ion has eight oppositely charged ions around it in the
25. (a) Nearest neighbour in the topmost layer of ccp structure is 9 nearest neighbours as shown below :
eB

thus, incorrect.
(b) Packing efficiency is 74% thus, correct.
(c) Tetrahedral voids = 2 Cs+
r

Octahedral voids = 1 per atom thus, correct. Cl–


ou
ad

4
(d) Edge length, a = r = 2 2r
2 Unit cell of CsCl
Y

A
thus, correct (b) In bcc, coordination number of atom is 8.
Explanation Edge length = a (c) In an unit cell, a corner is shared in eight unit cells and a face
nd

a centre is shared between two adjacent unit cells.


Re

Radius = r
(d) In NaCl unit cell; 2(rNa+ + rCl - ) = a
AC 2 = AB 2 + BC 2
Fi

a C Þ a = 2 (95 + 181) = 552 pm


(4 r)2 = a2 + a2 = 2a2 B
Hence, a, c, d are correct.
4 r = 2a 29. The unit cell of initial structure of ionic solid MX looks like
2 a Cl– (at face centre)
Þ r= a= Þ a=2 2 r
4 2 2
Cl– (at corner)
In ccp structure, number of spheres is 4.
æ4 ö Na+ (at face)
Hence, volume of 4 spheres = 4 ç pr3 ÷
è 3 ø
Total volume of unit cell = a3 = (2 2r)3 Na+ (at corner edge)
% of packing efficiency
In NaCl type of solids cations (Na + ) occupy the octahedral
voids while anions (Cl - ) occupy the face centre positions.
130 Solid State

However, as per the demand of problem the position of cations = 24 2 r3


and anions are swapped.
Volume occupied by atoms
We also know that (for 1 unit cell) 33. Packing fraction =
Volume of unit cell
(A) Total number of atoms at FCC = 4 4 1
(B) Total number of octahedral voids = 4 = 6 ´ pr3 ´ = 0.74
3 24 2 r3
(as no. of atoms at FCC = No. of octahedral voids)
Þ Fraction of empty space = 1 - 0.74 = 0.26 = 26%
Now taking the conditions one by one
(i) If we remove all the anions except the central one than
number of left anions. 34. A. Simple cubic and face centred cubic both have cell
= 4 -3 = 1 parameters a = b = c and a = b = g = 90°. Also both of them
belongs to same, cubic, crystal system.
(ii) If we replace all the face centred cations by anions than
B. The cubic and rhombohedral crystal system belongs to

w
effective number of cations will be = 4 - 3 = 1
different crystal system.
Likewise effective number of anions will be = 1+ 3 = 4
C. Cubic and tetragonal are two different types of crystal
(iii) If we remove all the corner cations then effective number of
systems having different cell parameters.
cations will be 1- 1= 0

Flo
D. Hexagonal and monoclinic are two different crystal system
(iv) If we replace central anion with cation then effective
number of cations will be 0 + 1= 1 and both have two of their crystallographic angles of 90°.

ree
Likewise effective number of anions will be 4 - 1= 3 4 ´M
35. Density (r) = 8 =
Thus, as the final outcome, total number of cations present in N A (4 ´ 10-8 cm )3

F
Z after fulfilling all the four sequential instructions = 1 256
Þ M = 128 ´ 10-24 N A Þ No of atoms = ´NA
Likewise, total number of anions = 3 M
256

or
Number of anions 3
Hence, the value of
ur
Number of cations 1
= =3 =
128 ´ 10-24 N A
´ N A = 2 ´ 1024

f
30. In ionic solid MX (1 : 1 solid) same number of M n+ and X n- ions 36. The truncated octahedron is the 14-faced Archimedean solid,
ks
are lost in Schottky defect to maintain electroneutrality of solid. with 14 total faces : 6 squares and 8 regular hexagons.
Yo
The truncated octahedron is formed by removing the six right
oo
square pyramids one from each point of a regular octahedron as :
A
eB

31. B
Q
r

C M
ou
ad

P
R S N
Y

Truncated octahedron
A hcp unit cell
Contribution of atoms from corner = 1/ 6 Truncated octahedron unfolded in two-dimension
nd

Contribution from face centre = 1 / 2


Re

37. Ag crystallises in fcc unit cell with 4 atoms per unit cell.
1 1
Þ Total number of atoms per unit cell = 12 ´ + 2 ´ + 3 = 6 4 ´ 108
r= = 10.5 g cm -3.
Fi

6 2
6.023 ´ 1023 ´ a3
32. In close packed arrangement, side of the base = 2r
Þ a3 (Volume of unit cell) = 6.83 ´ 10-23 cm 3
Þ RS = r
Also MNR is equilateral triangle, Ð PRS = 30° Þ a = 4 ´ 10-8 cm = 4 ´ 10-10 m
RS 3
In triangle PRS , cos 30° = = Þ Surface area of unit cell = a2 = 1.6 ´ 10-19 m 2
PR 2
2 2 Þ Number of unit cells on 10-12 m 2 surface
Þ PR = RS = r
3 3 10-12
= = 6.25 ´ 106
In right angle triangle PQR : PQ = QR 2 - PR 2 = 2
2
r 1.6 ´ 10-19
3
Q There are two atoms (effectively) on one face of unit cell
2
Þ Height of hexagon = 2PQ = 4 r Number of atoms on 10-12 m 2 surface = 2 ´ number of unit cell
3
3 2 = 1.25 ´ 107 . [Q y ´10x ]
Þ Volume = Area of base ´ height = 6 (2r)2 ´ 4 r
4 3 Þ x = 7 Þ y = 125
.
Solid State 131

38. From the given information, the number of atoms per unit cell centres within the square = 5 (shown in diagram)
and therefore, type of unit cell can be known as Maximum number of marbles per unit area = 5 ´ 5 = 25
NM (ii) If x mm is the side of square and d is diameter of marble then
r=
NA a3 maximum number of marbles on square area with centres
within square area can be known by the following general
r N A a3 2 ´ 6 ´ 1023 ´ (5 ´ 10-8 cm )3
Þ N = = = 2 ( bcc) formula :
M 75 2
æx ö
Þ In bcc, 4 r = 3a N = ç + 1÷
èd ø
3 3
Þ r= a= ´ 5 ´ 10-10 m
4 4
= 2.17 ´ 10-10 m = 217 pm

w
39. In a cubic crystal system, there are two types of voids known as 42.
octahedral and tetrahedral voids. If r1 is the radius of void and r2
is the radius of atom creating these voids then

Flo
æ r1 ö ær ö 43. Density µ
N
çç ÷÷ = 0.414 and çç 1 ÷÷ = 0.225 a3
è 2 ø octa
r è r2 ø tetra 3 3
d1 N 1 æ a2 ö
çç ÷÷ = æç
4 3 ö

ree
The above radius ratio values indicate that octahedral void has Þ = ÷ = 1.26
larger radius, hence for maximum diameter of atom to be present d2 N 2 è a1 ø 2 è 3.5 ø
in interstitial space :

F
44. In bcc unit cell, 4 r = 3a
r1 = 0.414 r2
3a 3
Also in fcc, 4 r2 = 2a Þ r (Cr) = = ´ 287 pm = 124.3 pm

or
ur
Þ Diameter required (2r1 ) = (2r2 ) ´ 0.414
4
NM
4

Density of solid =

f
a
= ´ 0.414 N A × a3
2
ks
400 ´ 0.414 N = Number of atoms per unit cell, M = Molar mass
Yo
= = 117 pm 3
a = Volume of cubic unit cell, N A = Avogadro’s number
oo
2
3
40. (i) In rock salt like crystal AB, there are four AB units per unit 2 ´ 52 g æ 1 ö
eB

cell. Therefore, density (d ) is = 23


´ çç -8
÷ = 7.3 g / cm 3
÷
6.023 ´ 10 è 2.87 ´ 10 cm ø
4 ´ 6.023 y
d=
6.023 ´ 1023 ´ 8 y ´ 10-27 45. The given arrangement : ABABAB...... represents hexagonal
r

close-packed unit cell in which there are six atoms per unit cell.
[Q a = 2 y1/ 3 nm = 2 y1/ 3 ´ 10-9 m ]
ou
ad

Also, volume of unit cell = 24 2r3.


= 5 ´ 103 g/m 3 = 5 kg/m 3
Volume occupied by atoms
Y

(ii) Since, observed density is greater than expected, theoretical Þ Packing fraction =
Volume of unit cell
density, there must be some excess metal occupying 4 1
= 6 ´ pr3 ´ = 0.74
nd

interstitial spaces. This type of defect is known as metal


Re

3 24 2 r3
excess defect.
Þ Percent empty space = 100 (1 - 0.74) = 26%
Fi

41. (i) Side of square = 40 mm 10 mm 46. In bcc arrangement of atoms : 4 r = 3a, atoms on body
Diameter of marble = 10 mm diagonal remain in contact
Number of marble spheres along 3a 3 ´ 4.29
Þ r= = = 1.86 Å
an edge of square with their 4 4

40 mm

Download Chapter Test


http://tinyurl.com/y6yuk3b2 or
9
Solution and
Colligative Properties

w
Flo
ree
Topic 1 Solution and Vapour Pressure of Liquid Solutions
Objective Questions I (Only one correct option)

F
Partial pressure

Partial pressure
1. The mole fraction of a solvent in aqueous solution of a solute z
-1
is 0.8. The molality (in mol kg ) of the aqueous solution is

or
(a) (b)
ur (2019 Main, 12 April I)
z
y x
y

f
(a) 13.88 ´ 10-2 (b) 13.88 ´ 10-1 x w w
ks
-3
(c) 13.88 (d) 13.88 ´ 10 (0, 0) Mole fraction (0, 0) Mole fraction
Yo
of water of water
2. What would be the molality of 20% (mass/mass) aqueous
oo
z
solution of KI? (Molar mass of KI = 166 g mol -1 ) z y
Partial pressure

Partial pressure
eB

(2019 Main, 9 April I)


y xw
(a) 1.48 (b) 1.51 (c) (d)
(c) 1.35 (d) 1.08
x
r

3. Liquid M and liquid N form an ideal solution. The vapour


ou

w
ad

pressures of pure liquids M and N are 450 and 700 mmHg,


(0, 0) Mole fraction (0, 0) Mole fraction
respectively, at the same temperature. Then correct statement
Y

of water of water
is (2019 Main, 9 April I)
x M = mole fraction of M in solution; 5. The vapour pressures of pure liquids A and B are 400 and 600
nd

x N = mole fraction of N in solution; mmHg, respectively at 298 K. On mixing the two liquids, the
Re

y M = mole fraction of M in vapour phase; sum of their initial volumes is equal to the volume of the final
mixture. The mole fraction of liquid B is 0.5 in the mixture.
Fi

y N = mole fraction of N in vapour phase


The vapour pressure of the final solution, the mole fractions
x y
(a) M > M of components A and B in vapour phase, respectively are
xN yN (2019 Main, 8 April I)
xM y (a) 450 mmHg, 0.4, 0.6 (b) 500 mmHg, 0.5, 0.5
(b) = M
xN yN (c) 450 mmHg, 0.5,0.5 (d) 500 mmHg, 0.4,0.6
x y 6. Liquids A and B form an ideal solution in the entire
(c) M < M
xN yN composition range. At 350 K, the vapour pressures of pure A
(d) ( x M - y M ) < ( x N - y N ) and pure B are 7 ´ 103 Pa and 12 ´ 103 Pa, respectively. The
composition of the vapour in equilibrium with a solution
4. For the solution of the gases w , x, y and z in water at 298 K,
containing 40 mole percent of A at this temperature is
the Henry’s law constants ( K H ) are 0.5, 2, 35 and 40 K bar, (2019 Main, 10 Jan I)
respectively. The correct plot for the given data is
(a) x A = 0.76; xB = 0.24 (b) x A = 0.28; xB = 0.72
(2019 Main, 8 April II)
(c) x A = 0.4; xB = 0.6 (d) x A = 0.37; xB = 0.63
Solution and Colligative Properties 133

7. Which one of the following statements regarding Henry’s solution. The correct statement(s) applicable to this system is
law is not correct? (2019 Main, 8 Jan I) (are) (2017 Adv.)
(a) Different gases have different K H (Henry’s law
constant) values at the same temperature
Z
(b) Higher the value of K H at a given pressure, higher is the
solubility of the gas in the liquids pL
(c) The value of K H increases with increase of temperature
and K H is function of the nature of the gas
(d) The partial pressure of the gas in vapour phase is
1 XM 0
proportional to the mole fraction of the gas in the solution
8. 18 g of glucose (C6 H12 O6 ) is added to 178.2 g water. The (a) The point Z represents vapour pressure of pure liquid M

w
vapour pressure of water (in torr) for this aqueous solution is and Raoult’s law is obeyed from xL = 0 to xL = 1
(2016 Main) (b) Attractive intermolecular interactions between L - L in
(a) 76.0 (b) 752.4 (c) 759.0 (d) 7.6 pure liquid L and M - M in pure liquid M are stronger

Flo
than those between L - M when mixed in solution
9. The vapour pressure of acetone at 20°C is 185 torr.
When 1.2 g of a non-volatile substance was dissolved in (c) The point Z represents vapour pressure of pure liquid M
and Raoult’s law is obeyed when xL ® 0

ree
100 g of acetone at 20°C, its vapour pressure was 183 Torr.
The molar mass of the substance is (2015, 1M) (d) The point Z represents vapour pressure of pure liquid L
and Raoult’s law is obeyed when xL ® 1

F
(a) 32 (b) 64 (a) 128 (b) 488
10. The Henry’s law constant for the solubility of N2 gas in 15. Mixture(s) showing positive deviation from Raoult’s law at
water at 298 K is 1.0 ´ 105 atm. The mole fraction of N2 in air 35°C is (are) (2016 Adv.)

or
ur
is 0.8. The number of moles of N2 from air dissolved in (a) carbon tetrachloride + methanol

f
10 moles of water of 298 K and 5 atm pressure is (2009) (b) carbon disulphide + acetone
-4 -5 (c) benzene + toluene
(a) 4.0 ´ 10 (b) 4.0 ´ 10
ks
(d) phenol + aniline
(c) 5.0 ´ 10- 4 (d) 4.0 ´ 10- 6
Yo
oo
11. A molal solution is one that contains one mole of a solute in Numerical Value Based Question
eB

(1986, 1M) 16. Liquids A and B form ideal solution over the entire range of
(a) 1000 g of the solvent (b) 1 L of the solvent composition. At temperature T, equimolar binary solution of
(c) 1 L of the solution (d) 22.4 L of the solution liquids A and B has vapour pressure 45 torr. At the same
temperature, a new solution of A and B having mole fractions
r

12. For a dilute solution, Raoult’s law states that (1985, 1M)
x A and xB , respectively, has vapour pressure of 22.5 torr. The
ou
ad

(a) the lowering of vapour pressure is equal to the mole value of x A / xB in the new solution is ____.
fraction of solute
Y

(Given that the vapour pressure of pure liquid A is 20 Torr at


(b) the relative lowering of vapour pressure is equal to the temperature T) (2018 Adv. Paper-1)
mole fraction of solute
nd
Re

(c) the relative lowering of vapour pressure is proportional True/False


to the amount of solute in solution 17. Following statement is true only under some specific
Fi

(d) the vapour pressure of the solution is equal to the mole conditions. Write the condition for it.
fraction of solvent “Two volatile and miscible liquids can be separated by
13. An azeotropic solution of two liquids has boiling point lower fractional distillation into pure components.” (1994)
than either of them when it (1981, 1M)
(a) shows negative deviation from Raoult’s law
Subjective Questions
(b) shows no deviation from Raoult’s law 18. The vapour pressure of two miscible liquids A and B are 300
(c) shows positive deviation from Raoult’s law and 500 mm of Hg respectively. In a flask 10 moles of A is
mixed with 12 moles of B. However, as soon as B is added, A
(d) is saturated
starts polymerising into a completely insoluble solid. The
polymerisation follows first-order kinetics. After 100 min,
Objective Questions II 0.525 mole of a solute is dissolved which arrests the
(One or more than one correct option) polymerisation completely. The final vapour pressure of the
14. For a solution formed by mixing liquids L and M , the vapour solution is 400 mm of Hg. Estimate the rate constant of the
pressure of L plotted against the mole fraction of M in polymerisation reaction. Assume negligible volume change
solution is shown in the following figure. Here xL and x M on mixing and polymerisation and ideal behaviour for the
final solution. (2001, 4M)
represent mole fractions of L and M , respectively, in the
134 Solution and Colligative Properties

19. The molar volume of liquid benzene (density = 0.877 g/mL) same temperature by the mixing 60 g of ethanol with 40 g of
increases by a factor of 2750 as it vaporises at 20° C and that methanol. Calculate the total vapour pressure of the solution
of liquid toluene (density = 0.867 g mL-1 ) increases by a and the mole fraction of methanol in the vapour. (1986, 4M)
factor of 7720 at 20° C. A solution of benzene and toluene at 25. An organic compound (Cx H2 y O y ) was burnt with twice the
20° C has a vapour pressure of 45.0 torr. Find the mole amount of oxygen needed for complete combustion to CO2
fraction of benzene in the vapour above the solution. and H2 O. The hot gases when cooled to 0°C and 1 atm
(1996, 3M) pressure, measured 2.24 L. The water collected during
20. What weight of the non-volatile solute urea cooling weight 0.9 g. The vapour pressure of pure water at
(NH2 — CO —NH2 ) needs to be dissolved in 100 g of water, 20°C is 17.5 mm Hg and is lowered by 0.104 mm when 50 g
in order to decrease the vapour pressure of water by 25%? of the organic compound are dissolved in 1000 g of water.
What will be the molality of the solution? (1993, 3M) Give the molecular formula of the organic compound.

w
(1983, 5M)
21. The degree of dissociation of Ca(NO3 )2 in a dilute aqueous
solution, containing 7.0 g of the salt per 100 g of water at
26. Two liquids A and B form ideal solution. At 300 K, the
vapour pressure of a solution containing 1 mole of A and
100° C is 70%. If the vapour-pressure of water at 100° C is

Flo
3 moles of B is 550 mm of Hg. At the same temperature, if
760mm, calculate the vapour pressure of the solution.
one more mole of B is added to this solution, the vapour
(1991, 4M)
pressure of the solution increases by 10 mm of Hg.

ree
22. The vapour pressure of pure benzene at a certain temperature Determine the vapour pressure of A and B in their pure
is 640 mm Hg. A non-volatile, non-electrolyte solid states. (1982, 4M)

F
weighing 2.175 g is added to 39.0 g of benzene. The vapour
pressure of the solution is 600 mm Hg. What is the molecular 27. The vapour pressure of pure benzene is 639.70 mm of Hg and
the vapour pressure of solution of a solute in benzene at the
weight of the solid substance?

or
(1990, 3M)
ur same temperature is 631.9 mm of Hg. Calculate the molality
23. The vapour pressure of a dilute aqueous solution of glucose of the solution.

f
(1981, 3M)
(C6 H12 O6 ) is 750 mm of mercury at 373 K.
28. What is the molarity and molality of a 13% solution
ks
Calculate (i) molality and (ii) mole fraction of the solute.
(by weight) of sulphuric acid with a density of 1.02 g/mL ?
Yo
(1989, 3M)
To what volume should 100 mL of this solution be diluted in
oo
24. The vapour pressure of ethanol and methanol are 44.5 and order to prepare a 1.5 N solution ? (1978, 2M)
88.7 mm Hg respectively. An ideal solution is formed at the
eB

Topic 2 Colligative Properties


r
ou
ad

Objective Questions I (Only one correct option) (a) 0.027 mmHg


(b) 0.031 mmHg
Y

1. A solution is prepared by dissolving 0.6 g of urea (molar


mass = 60 g mol -1 ) and 1.8 g of glucose (molar mass = 180 g (c) 0.017 mmHg
(d) 0.028 mmHg
mol -1 ) in 100 mL of water at 27°C. The osmotic pressure of
nd
Re

the solution is ( R = 0.08206 L atm K -1 mol -1 ) 4. Molal depression constant for a solvent is 4.0 K kg mol -1 .
The depression in the freezing point of the solvent for 0.03
Fi

(2019 Main, 12 April II)


mol kg -1 solution of K 2SO4 is
(a) 8.2 atm (b) 2.46 atm (c) 4.92 atm (d) 1.64 atm
(Assume complete dissociation of the electrolyte)
2. 1 g of a non-volatile, non-electrolyte solute is dissolved in (2019 Main, 9 April II)
100 g of two different solvents A and B, whose ebullisocopic
constants are in the ratio of 1 : 5. The ratio of the elevation in (a) 0.18 K (b) 0.36 K
DT ( A ) (c) 0.12 K (d) 0.24 K
their boiling points, b , is
DTb ( B ) (2019 Main, 10 April II) 5. The osmotic pressure of a dilute solution of an ionic
(a) 5 : 1 (b) 10 : 1 compound XY in water is four times that of a solution of 0.01
(c) 1 : 5 (d) 1 : 0.2 M BaCl2 in water. Assuming complete dissociation of the
given ionic compounds in water, the concentration of XY (in
3. At room temperature, a dilute solution of urea is prepared by
dissolving 0.60 g of urea in 360 g of water. If the vapour mol L-1 ) in solution is (2019 Main, 9 April I)

pressure of pure water at this temperature is 35 mm Hg, (a) 4 ´ 10-2 (b) 16 ´ 10-4
lowering of vapour pressure will be (c) 4 ´ 10-4 (d) 6 ´ 10-2
(Molar mass of urea = 60 g mol -1 ) (2019 Main, 10 April I)
Solution and Colligative Properties 135

6. Molecules of benzoic acid (C6H5COOH) dimerise in plots of vapour pressure (V.P.) versus temperature (T ).
benzene. ‘w’ g of the acid dissolved in 30 g of benzene [Molecular weight of ethanol is 46 g mol -1 ] (2017 Adv.)
shows a depression in freezing point equal to 2 K. If the Among the following, the option representing change in the
percentage association of the acid to form dimer in the freezing point is
solution is 80, then w is
(Given that K f = 5 K kg mol- 1 , molar mass of benzoic Water Water

V.P./bar

V.P./bar
-1 (a) 1 (b) 1
acid = 122 g mol ) (2019 Main, 12 Jan II) Ice Ice
Water+Ethanol Water+Ethanol
(a) 1.8 g (b) 1.0 g (c) 2.4 g (d) 1.5 g
7. Freezing point of a 4% aqueous solution of X is equal to 270 273 T/K 271 273 T/K
freezing point of 12% aqueous solution of Y . If molecular

w
weight of X is A, then molecular weight of Y is
(2019 Main, 12 Jan I) Ice Water Ice Water

V.P./bar

V.P./bar
(c) 1 (d) 1
(a) 4A (b) 2A (c) 3A (d) A

Flo
8. K 2HgI4 is 40% ionised in aqueous solution. The value of its Water+Ethanol Water+Ethanol

van’t Hoff factor (i) is (2019 Main, 11 Jan II) 270 273 T/K 271 273 T/K

ree
(a) 1.6 (b) 1.8 (c) 2.2 (d) 2.0 15. Consider separate solution of 0.500 M C2 H5 OH (aq),
9. The freezing point of a diluted milk sample is found to be 0.100 M Mg 3 (PO4 )2 (aq), 0.250 M KBr (aq) and 0.125 M

F
-0.2° C, while it should have been -0.5°C for pure milk. Na 3 PO4 (aq) at 25°C. Which statement is true about these
How much water has been added to pure milk to make the solution, assuming all salts to be strong electrolytes?

or
diluted sample? (a) They all have the same osmotic pressure
ur (2019 Main, 11 Jan I) (2014 Main)
(a) 2 cups of water to 3 cups of pure milk (b) 0.100 M Mg 3 (PO4 )2 (aq) has the highest osmotic

f
(b) 1 cup of water to 3 cups of pure milk pressure
ks
(c) 3 cups of water to 2 cups of pure milk (c) 0.125 M Na 3 PO4 (aq) has the highest osmotic pressure
Yo
(d) 1 cup of water to 2 cups of pure milk (d) 0.500 M C2 H5 OH (aq) has the highest osmotic pressure
oo
10. Elevation in the boiling point for 1 molal solution of 16. For a dilute solution containing 2.5 g of a non-volatile
glucose is 2 K . The depression in the freezing point for 2 non-electrolyte solute in 100 g of water, the elevation in
eB

molal solution of glucose in the same solvent is 2 K. The boiling point at 1 atm pressure is 2°C. Assuming concentration
relation between K b and K f is (2019 Main, 10 Jan II) of solute is much lower than the concentration of solvent, the
(a) K b = 15
. Kf (b) K b = 0.5 K f vapour pressure (mm of Hg) of the solution is (take K b = 0.76
r

(c) K b = K f (d) K b = 2K f K kg mol -1 ).


ou

(2012)
ad

11. A solution contain 62 g of ethylene glycol in 250 g of water (a) 724 (b) 740
Y

is cooled upto –10º C. If K f for water is 1.86 K kg mol -1 , (c) 736 (d) 718
then amount of water (in g) separated as ice is 17. The freezing point (in°C) of solution containing 0.1 g of
nd

K 3 [Fe(CN)6 ] (mol. wt. 329) in 100 g of water


Re

(2019 Main, 9 Jan II)


(a) 32 (b) 48 (c) 64 (d) 16 ( K f = 1.86 K kg mol -1 ) is (2011)
Fi

-2 -2
12. For 1 molal aqueous solution of the following compounds, (a) - 2.3 ´ 10 (b) -5.7 ´ 10
which one will show the highest freezing point?(2018 Main) (c) -5.7 ´ 10-3 (d) -1.2 ´ 10-2
(a) [Co(H2 O)6 ]Cl 3 (b) [Co(H2 O)5 Cl]Cl 2 × H2 O
(c) [Co(H2 O)4 Cl 2 ]Cl × 2H2 O (d) [Co(H2 O)3 Cl 3 ] × 3H2 O 18. When 20 g of naphthoic acid (C11 H8 O2 ) is dissolved in 50 g of
benzene (K f = 1.72 K kg mol - 1 ), a freezing point depression
13. The freezing point of benzene decreases by 0.45°C when
0.2 g of acetic acid is added to 20 g of benzene. If acetic of 2 K is observed. The van’t Hoff factor ( i ) is (2007, 3M)
acid associates to form a dimer in benzene, percentage (a) 0.5 (b) 1
association of acetic acid in benzene will be (c) 2 (d) 3
. K kg mol - 1)
(K f for benzene = 512 (2017 Main) 19. The elevation in boiling point, when 13.44 g of freshly
(a) 64.6 % (b) 80.4 % (c) 74.6 % (d) 94.6 % prepared CuCl 2 are added to one kilogram of water, is. [Some
14. Pure water freezes at 273 K and 1 bar. The addition of 34.5 useful data, K b = 0.52 K kg mol - 1 , molecular weight of
g of ethanol to 500 g of water changes the freezing point of CuCl 2 =134.4 g]. (2005, 1M)
the solution. Use the freezing point depression constant of (a) 0.05 (b) 0.1
water as 2 K kg mol -1 . The figures shown below represent (c) 0.16 (d) 0.21
136 Solution and Colligative Properties

20. 0.004 M Na 2 SO4 is isotonic with 0.01 M glucose. Degree of 1 2 3 4


dissociation of Na 2 SO4 is (2004, S, 1M)
(a) 75% (b) 50% 760
(c) 25% (d) 85%

Pressure (mmHg)
21. During depression of freezing point in a solution the 1. Solvent X
following are in equilibrium (2003) 2. Solution of NaCl in solvent X
3. Solvent Y
(a) liquid solvent, solid solvent 4. Solution of NaCl in solvent Y
(b) liquid solvent, solid solute
(c) liquid solute, solid solute
(d) liquid solute, solid solvent
22. The molecular weight of benzoic acid in benzene as

w
360

362

367
368
determined by depression in freezing point method
Temperature (K)
corresponds to
(a) ionisation of benzoic acid (1996, 1M) On addition of equal number of moles of a non-volatile solute

Flo
(b) dimerisation of benzoic acid S in equal amount (in kg) of these solvents, the elevation of
(c) trimerisation of benzoic acid boiling point of solvent X is three times that of solvent Y.
(d) solvation of benzoic acid

ree
Solute S is known to undergo dimerisation in these solvents.
23. The freezing point of equimolal aqueous solutions will be If the degree of dimerisation is 0.7 in solvent Y , the degree of
highest for (1990, 1M) dimerisation in solvent X is ____. (2018 Adv.)

F
(a) C6 H5 NH3 Cl (aniline hydrochloride)
(b) Ca(NO3 )2 Subjective Questions

or
(c) La(NO3 )3
ur 28. 75.2 g of C6 H 5 OH (phenol) is dissolved in a solvent of
(d) C6 H12 O6 (glucose)

f
K f = 14. If the depression in freezing point is 7 K, then find
24. Which of the following 0.1 M aqueous solution will have
the percentage of phenol that dimerises.
ks
(2006, 2M)
the lowest freezing point? (1989, 1M)
Yo
(a) Potassium sulphate (b) Sodium chloride 29. 1.22 g C6 H5 COOH is added into two solvents and data of DTb
oo
(c) Urea (d) Glucose and K b are given as :
(i) In 100 g CH3COCH3 DTb = 0.17, K b = 1.7 K kg/mol
eB

25. When mercuric iodide is added to the aqueous solution of


potassium iodide (1987, 2M) (ii) In 100 g benzene, DTb = 0.13 and K b = 2.6 K kg/mol
(a) freezing point is raised Find out the molecular weight of C6 H5 COOH in both the
cases and interpret the result.
r

(b) freezing point is lowered (2004, 2M)


ou

(c) freezing point does not change


ad

30. Consider the three solvents of identical molar masses. Match


(d) boiling point does not change their boiling point with their K b values
Y

Objective Questions II Solvents Boiling point Kb values


(One or more than one correct option) X 100°C 0.92
nd
Re

26. In the depression of freezing point experiment, it is found Y 27°C 0.63


that the (1999, 3M) Z 283°C 0.53
Fi

(a) vapour pressure of the solution is less than that of pure (2003)
solvent 3 –3
31. To 500 cm of water, 3.0 ´ 10 kg of acetic acid is added. If
(b) vapour pressure of the solution is more than that of
23% of acetic acid is dissociated, what will be the depression
pure solvent
in freezing point? K f and density of water are
(c) only solute molecules solidify at the freezing point
1.86 K kg –1 mol –1 and 0.997 g cm-3 , respectively.
(d) only solvent molecules solidify at the freezing point
(2000, 3M)
Numerical Value Based Question 32. Nitrobenzene is formed as the major product along with a
27. The plot given below shows p -T curves (where p is the minor product in the reaction of benzene with a hot mixture of
pressure and T is the temperature) for two solvents X and Y nitric acid and sulphuric acid. The minor product consists of
and isomolal solution of NaCl in these solvents. NaCl carbon : 42.86%, hydrogen : 2.40%, nitrogen : 16.67% and
completely dissociates in both the solvents. oxygen : 38.07%,
(i) Calculate the empirical formula of the minor product.
(ii) When 5.5 g of the minor product is dissolved in 45 g of
benzene, the boiling point of the solution is 1.84°C higher
Solution and Colligative Properties 137

than that of pure benzene. Calculate the molar mass of the Standard boiling point of ethanol = 351.5 K
minor product then determine its molecular and structural Vapour pressure of pure water = 32.8 mm Hg
formula. (Molal boiling point elevation constant of benzene Vapour pressure of pure ethanol = 40 mm Hg
is 2.53 K kg mol - 1 ). (1999) Molecular weight of water = 18 g mol -1
33. A solution of a non-volatile solute in water freezes at Molecular weight of ethanol = 46 g mol -1
- 0.30° C. The vapour pressure of pure water at 298 K is
In answering the following questions, consider the solutions
23.51 mm Hg and K f for water is 1.86 K kg mol -1 . Calculate
to be ideal dilute solutions and solutes to be non-volatile and
the vapour pressure of this solution at 298 K. (1998, 4M)
non-dissociative. (2008, 3 ´ 4M = 12M)
34. Addition of 0.643 g of a compound to 50 mL of benzene
35. The freezing point of the solution M is
(density : 0.879 g/mL) lowers the freezing point from 5.51° C
(a) 268.7 K (b) 268.5 K (c) 234.2 K (d) 150.9 K
to 5.03° C. If K f for benzene is 5.12, calculate the molecular

w
weight of the compound. (1992, 2M) 36. The vapour pressure of the solution M is
(a) 39.3 mm Hg (b) 36.0 mm Hg
Passage Based Questions (c) 29.5 mm Hg (d) 28.8 mm Hg

Flo
Passage 1 37. Water is added to the solution M such that the mole fraction
of water in the solution becomes 0.9. The boiling point of this
Properties such as boiling point, freezing point and vapour pressure
solution is

ree
of a pure solvent change when solute molecules are added to get
(a) 380.4 K (b) 376.2 K (c) 375.5 K (d) 354.7 K
homogeneous solution. These are called colligative properties.
Applications of colligative properties are very useful in day-to-day

F
life.
Fill in the Blank
One of its examples is the use of ethylene glycol and water mixture 38. Given that DT f is the depression in freezing point of the

or
ur
as anti-freezing liquid in the radiator of automobiles. solvent in a solution of a non-volatile solute of molality, m,
A solution M is prepared by mixing ethanol and water. The mole the quantity lim ( DT f / m ) is equal to ...... (1994, 1M)

f
m ®0
fraction of ethanol in the mixture is 0.9.
ks
Given, freezing point depression constant of water Integer Answer Type Question
) = 1.86 K kg mol -1
Yo
( K water
f 39. If the freezing point of a 0.01 molal aqueous solution of a
oo

Freezing point depression constant of ethanol cobalt (III) chloride-ammonia complex (which behaves as a
) = 2.0 K kg mol -1 strong electrolyte) is - 0.0558°C, the number of chloride(s)
eB

( K ethanol
f
in the coordination sphere of the complex is
Boiling point elevation constant of water . K kg mol -1 ]
[ K f of water = 186 (2015 Adv.)
( K bwater ) = 0.52 K kg mol -1
r

40. MX 2 dissociates into M 2+ and X - ions in an aqueous


ou

Boiling point elevation constant of ethanol


ad

solution, with a degree of dissociation ( a ) of 0.5. The ratio of


( K bethanol ) = 1.2 K kg mol -1
the observed depression of freezing point of the aqueous
Y

Standard freezing point of water = 273 K solution to the value of the depression of freezing point in the
Standard freezing point of ethanol = 155.7 K absence of ionic dissociation is (2014 Adv.)
nd
Re

Standard boiling point of water = 373 K


Fi

Answers
Topic 1 Topic 2
1. (c) 2. (b) 3. (a) 4. (a) 1. (c) 2. (c) 3. (c) 4. (b)
5. (d) 6. (b) 7. (b) 8. (b) 5. (d) 6. (c) 7. (c) 8. (b)
9. (b) 10. (a) 11. (b) 12. (b) 9. (c) 10. (d) 11. (c) 12. (d)
13. (c) 14. (b, d) 15. (a, b) 16. (19) 13. (d) 14. (b) 15. (a) 16. (a)
17. T 19. (0.72) 20. (18.5) 21. (746.32 mm) 17. (a) 18. (a) 19. (c) 20. (a)
22. (65.25) 23. (0.75) 24. (0.657) 27. (0.158) 21. (a) 22. (b) 23. (d) 24. (a)
25. (a) 26. (a, d) 27. (0.05) 28. (75%)
28. (180.40 mL)
30. (0.23°C) 33. (23.44 mm) 34. (156 g/mol) 35. (d)
36. (a) 37. (b) 38. (K f ) 40. (2)
Hints & Solutions
Topic 1 Solution and Vapour Pressure pgas = partial pressure of the gas above its solution with a liquid
(solvent) say water.
of Liquid Solutions
cgas = mole fraction of the gas (solute) in the solution.
c H 2 O = mole fraction of water (solvent).
1. Mass of solute (w2 ) ´ 1000
Key Idea Molality ( m) =
Molar mass of solute (M 2 ) ´
mass of solvent (w1 )
w2 1000 pgas pgas
m= ´ KH
M2 w1 KH

w
1000
and also, m = n2 ´
n1 ´ M1
cH2O=0 cH2O=0

Flo
X solvent = 0.8 (Given) It means that nsolvent (n1 ) = 0.8 and
cgas=1 cgas=0
nsolute (n2 ) = 0.2
1000 1000
= 13.88 mol kg -1

ree
Using formula m = n2 ´ = 0.2 ´
n1 ´ M 1 0.8 ´ 18 [ i.e. pgas = K H ] Higher the value of K H, higher
will be the partial pressure of the gas ( pgas ), at a given

F
2. Key Idea Molality is defined as number of moles of solute temperature. The plot of pgas vs c H2O gives a (-ve) slope.
per kg of solvent. pgas = K H - K H ´ c H2O
w2 1000 Comparing the above equation with the equation of straight line

or
m = ´
Mw2 w1
ur y = mx + c

f
w2 = mass of solute, Mw2 = molecular mass of solute Slope = - K H , intercept = K H
w1 = mass of solvent. So, (i) Higher the value of K H, more (-ve) will be the slope and
ks
it is for z (K H = 40 K bar )
Yo
The molality of 20% (mass/mass) aqueous solution of KI (ii) Higher the value of K H, higher with the value of intercept,
oo
can be calculated by following formula. i.e. partial pressure and it is also for z.
w ´ 1000
m= 2
eB

5. (d) According to Dalton’s law of partial pressure


Mw2 ´ w1
ptotal = pA + pB
20% aqueous solution of KI means that 20 gm of KI is present in
= pA° c A + pB° c B …(i)
80 gm solvent.
r

20 1000 Given, pAº = 400 mm Hg, pBº = 600 mm Hg


ou

m= ´ = 1. 506 » 1. 51mol/kg
ad

166 80 c B = 0.5, c A + c B = 1
Y

\ c A = 0.5
3. Key Idea For a solution of volatile liquids the partial vapour
pressure of each component of the solution is directly On substituting the given values in Eq. (i). We get,
proportional to its mole fraction present in solution. This is ptotal = 400 ´ 0.5 + 600 ´ 0.5 = 500 mm Hg
nd
Re

known as Raoult’s law. Mole fraction of A in vapour phase,


pAº c A 0.5 ´ 400
Fi

Liquid M and N form an ideal solution. Vapour pressures of p


YA = A = = = 0.4
pure liquids M and N are 450 and 700 mm Hg respectively. ptotal ptotal 500
\ pºN > pºM Mole of B in vapour phase,
So, by using Raoult’s law YA + YB = 1
yN > xN …(i) YB = 1 - 0. 4 = 0.6
and xM > yM …(ii)
6. For ideal solution,
Multiplying (i) and (ii) we get
p = x ¢A p°A + x ¢B p°B
yN xM > yM xN
xM yM Q x ¢A = 0.4, x ¢B = 0.6
\ > p°A = 7 ´ 103 Pa, p°B = 12 ´ 103 Pa
xN yN
Thus, correct relation is (a). On substituting the given values in Eq. (i),
we get
4. According to Henry’s law (at constant temperature) p = 0.4 ´ 7 ´ 103 + 0.6 ´ 12 ´ 103
pgas = K H ´ cgas (solute) = K H ´ [1 - c H 2 O (solvent) ]
= 10 ´ 103 Pa = 1 ´ 104 Pa
pgas = K H - K H c H 2 O
Solution and Colligative Properties 139

In vapour phase, 4 = 105 ´ c N 2


p x ¢ p° 0.4 ´ 7 ´ 103 c N 2 = 4 ´ 10-5
xA = A = A A = = 0.28
p p 1 ´ 104
nN 2
\ xB = 1 - 0.28 = 0.72 [Q xA + xB = 1] = 4 ´ 10-5
nN 2 + nH 2 O
7. At constant temperature, solubility of a gas (S ) varies inversely nN 2
with Henry’s law constant (K H ) = 4 ´ 10-5
nN 2 + 10
Pressure P
KH = =
Solubility of a gas in a liquid S nN 2 = 4 ´ 10-4
Thus, higher the value of K H at a given pressure, the lower is the
11. Molality = moles of solute present in 1.0 kg of solvent.
solubility of the gas in the liquid.

w
12. The relative lowering of vapour pressure :
8. Key Idea Vapour pressure of water ( p° ) = 760 torr
- Dp
Mass (g) = c2 (mole fraction of solute)
Number of moles of glucose = p°
Molecular mass (g mol -1 )

Flo
18 g 13. In case of positive deviation from Raoult's law, the observed
= = 0.1 mol vapour pressure is greater than the ideal vapour pressure and
180 gmol -1
boiling point of azeotrope becomes lower than either of pure

ree
Molar mass of water = 18 g/mol liquid.
Mass of water (given) = 178.2g 14. The graph shown indicates that there is positive deviation

F
Number of moles of water because the observed vapour pressure of L is greater than the
Mass of water ideal pressure
=
Molar mass of water

or
=
178. 2g
= 9.9 mol
ur Z

f
18 g / mol pL
Real
Total number of moles = (0.1 + 9.9) moles = 10 moles
ks
Now, mole fraction of glucose in solution = Change in pressure al
Ide
Yo
with respect to initial pressure
oo

Dp 01 .
i.e. = 1 xM 0
eB

p° 10
Since, deviation is positive, the intermolecular force between L
or Dp = 0.01p° = 0.01 ´ 760 = 7.6 torr
and M is smaller than the same in pure L and pure M.
\ Vapour pressure of solution = (760 - 7.6) torr = 752.4 torr
Also as xL ® 1, xM ® 0, the real curve approaching ideal curve
r

9. Given, p° = 185 Torr at 20°C where Raoult’s law will be obeyed.


ou
ad

ps = 183 Torr at 20°C 15. When intermolecular attraction between two components A and
Y

Mass of non-volatile substance, m = 1.2 g B in the mixture is same as between A and A or B and B, hence it
Mass of acetone taken = 100g is a case of ideal solution.
M =? When intermolecular attraction between A and B in a mixture is
nd
Re

p° - ps n smaller than that between A and A or B and B, then mixture is


As, we have = more vaporised, bp is lowered. It is a case of positive deviation
ps N
Fi

from Raoult’s law.


Putting the values, we get, When intermolecular attraction between A and B is higher than
1.2 that between A and A or B and B, then mixture is less vaporised,
185 - 183 M 2 1.2 ´ 58 bp is increased. It is a case of negative deviation.
= Þ =
183 100 183 100 ´ M (a) Methanol molecules (CH3OH) are hydrogen bonded. In a
58 mixture of CCl 4 and CH3OH, extent of H-bonding is
183 ´ 1. 2 ´ 58 decreased. Mixture is more vaporised thus, positive
\ M =
2 ´ 100 deviation from Raoult’s law.
M = 63.684 = 64 g/mol (b) Acetone molecules have higher intermolecular attraction
due to dipole-dipole interaction. With CS2, this interaction is
10. Give, K H = 1 ´ 105 atm, c N 2 = 0.8 decreased thus, positive deviation.
nH 2 O = 10 moles, ptotal = 5 atm (c) Mixture of benzene and toluene forms ideal solution.
pN 2 = ptotal ´ c N 2 = 5 ´ 0.8 = 4 atm (d) Phenol and aniline have higher interaction due to
According to Henry’s law, intermolecular H-bonding. Hence, negative deviation.
pN 2 = K H ´ c N 2
140 Solution and Colligative Properties

16. Key Idea Use the formula 0.082 ´ 293


Þ VP of pure toluene = ´ 760 mm = 22.3mm
pTotal = pA° ´ c A + pB° ´ c B 819.2
Now, let mole fraction of benzene in the liquid phase = c
1
and for equimolar solutions c A = c B = Þ 4.65 c + 22.3 (1 – c ) = 45
2
Given, pTotal = 45 torr for equimolar solution Þ c = 0.43
pA° = 20 torr Þ Mole fraction of benzene in vapour phase
Partial vapour pressure of benzene
So,
1 1 1
45 = pA° ´ + pB° ´ = ( pA° + pB° ) =
2 2 2 Total vapour pressure
74.65 ´ 0.43
or pA° + pB° = 90 torr …(i) = = 0.72
45
But we know pA° = 20 torr
20. Vapour pressure of solution = 0.75 ´ VP of water

w
so, pB° = 90 - 20 = 70 torr (From Eq. (i)) Þ 75 = 100 c 1 : c 1 = mole fraction of solute
Now, for the new solution from the same formula 3 1
Þ c 1 = and c 2 = 1 - c 1 =
Given, pTotal = 22 . 5 torr 4 4

Flo
So, 22 . 5 = 20c A + 70 (1- c A ) (As c A + c B =1) c 2 n2 1
Þ = =
or 22.5 = 70 - 50c A c 1 n1 3

ree
70 - 22 . 5 n 100
So, cA = = 0.95 Þ n2 = 1 = = 1.85
50 3 18 ´ 3

F
Thus c B = 1 - 0.95 = 0.05 (as c A + c B =1) Þ Weight of urea = 1.85 ´ 60 = 111 g
Hence, the ratio n 1000
c A 0.95 Molality = 2 ´

or
=
c B 0.05
= 19
ur n1 M1
1 1000

f
= ´ = 18.5
17. It will be true only if boiling points of two liquids are 3 18
ks
significantly different.
21. Ca(NO3 )2 r Ca 2+ + 2NO3-
Yo
18. Let after 100 min, x moles of A are remaining unpolymerised 1- a a 2a
oo
moles of B = 12 i = 1 + 2 a where, a = 0.7
Þ i = 1 + 2 ´ 0.7= 2.4
eB

Moles of non-volatile solute = 0.525


c n1
Þ Mole fraction of A = Þ Mole fraction of solvent =
c + 12 + 0.525 n1 + in2
æ 100 ö
r

Mole fraction of B =
12 ç ÷
è 18 ø
ou

c + 12 + 0.525 =
ad

= 0.982
æ 100 ö 7
æ c ö æ 12 ö ç ÷ + 2.4 ´
Y

Þ 400 = çç ÷÷ ´ 300 + çç ÷÷ ´ 500 è 18 ø 164


è c + 12.525 ø è c + 12.525 ø Þ p = p0 c1 = 760 ´ 0.982
Þ c = 9.9 (VP of H2O at 100°C = 760 mm of Hg)
nd
Re

Þ Moles of A polymerised in 100 min = 10 – 9.9 = 0.10 = 746.32 mm


1 10 1 10
Þ k = ln = min -1
Fi

ln 22. According to Raoult’s law :


t 9.9 100 9.9
p = p0 c1
= 1.005 ´ 10-4 min -1
æ n1 ö
78 Þ 600 = 640 çç ÷÷
19. Volume of 1.0 mole liquid benzene = mL = 88.94 mL è n1 + n2 ø
0.877
n2 64 1
Þ Molar volume of benzene vapour at 20°C Þ = -1=
n1 60 15
88.94 ´ 2750
= L = 244.58 L 39 1
1000 Þ n2 = ´ = 0.033
0.082 ´ 293 78 15
Þ VP of pure benzene at 20°C = ´ 760 mm 2.175
244.58 Þ = 0.033
M
= 74.65 mm
Þ M = 65.25
Similarly; molar volume of toluene vapour
92 7720 23. At 373 K (bp) of H2O, Vapour pressure = 760 mm
= ´ L = 819.2L
0.867 1000 VP of solution at 373 K = 750 mm
Þ p = p0c 1
Solution and Colligative Properties 141

or 750 = 760 c 1 26. When 1.0 mole of A is mixed with 3 moles of B.


75 550 = 0.25 pA° + 0.75 pB° …(i)
Þ c1 = = mole fraction of H2O
76
75 1 When 1.0 mole of A is mixed with 4 moles of B.
Þ c2 = 1 - = = mole fraction of solute 560 = 0.20 pA° + 0.80 pB° …(ii)
76 76
n2 1
Now = Now, solving (i) and (ii) pA° = 400 mm
n1 + n2 76
n1 pB° = 600 mm.
Þ = 75
n2 27. According to Raoult’s law :
n2 1000 p = p0c 1 Þ 631.9 = 639.7 c 1
Þ Molality = ´ 1000 = = 0.74 molal
n1M 1 75 ´ 18 Þ c 1 = 0.9878 Þ c 2 = 0.0122

w
60 0.0122
24. Moles of ethanol = = 1.3 Þ Molality = ´ 1000 = 0.158
46 0.9878 ´ 78
40
Moles of methanol = = 1.25 28. Let us consider 1.0 L of solution.
32

Flo
1.3 Weight of solution = 1000 ´ 1.02 = 1020 g
Þ Mole fraction of ethanol = = 0.51 13
1.3 + 1.25 Weight of H2SO4 = 1020 ´ = 132.60 g

ree
Þ Vapour pressure of solution = pethanol + pmethanol 100
= 0.51 ´ 44.5 + 0.49 ´ 88.7 Weight of H2O = 1020 – 132.60 = 887.40 g
= 66.16 mm 132.60

F
Þ Molarity = = 1.353 M
Mole fraction of methanol in vapour phase 98
pmethanol 132.60 1000
Molality = ´ = 1.525 m

or
=
ur
Total vapour pressure 98 887.40
Normality = 2 ´ M = 2.706

f
43.463
= = 0.657 Þ 2.706 ´ 100 = 1.5 V
66.16
ks
Þ V = 180.40 mL
Yo
25. From lowering of vapour pressure information :
oo
0.104 n2
= c2 = Topic 2 Colligative Properties
17.5 n1 + n2
eB

n1 1. Key Idea Osmotic pressure is proportional to the molarity


Þ + 1 = 168.27
n2 (C) of the solution at a given temperature (T).
n1 Thus, p µ C, p = CRT (for dilute solution)
r

Þ = 167.27 n
p=
ou

n2 RT
ad

V
1000 M
Þ ´ = 167.27 n
Y

18 50 For the relation, p = CRT = RT


V
Þ M = 150 g/mol
Given, mass of urea = 0.6 g
nd

Also, the combustion reaction is :


Re

Molar mass of urea = 60 g mol- 1


CxH2 yO y + xO2 ¾® xCO2 + yH2O
Q 18 y g of H2O is produced from 1.0 mole of compound. Mass of glucose = 1.8 g
Fi

0.9 1 Molar mass of glucose = 180 g mol- 1


\0.9 g of H2O will be produced from = mol
18 y 20 y [ n (urea ) + n2 (glucose)]
p= 2 RT
x V
Þ At the end, moles of O2 left =
20 y æ 0.6 18 . ö
ç + ÷
x = è 60 180 ø
´ 1000 ´ 0.0821 ´ 300
moles of CO2 formed = 100
20 y
2x 2.24 = (0.01 + 0.01) ´ 10 ´ 0.0821 ´ 300
Þ Total moles of gases at STP = = p = 4 . 92 atm
20 y 22.4
Þ x= y 2. The expression of elevation of boiling point,
Þ Molar mass; 150 = 12x + 2x + 16x = 30x DTb = K b ´ m ´ i
150 w ´ 1000
Þ x= =5 = kb ´ 2 ´i
30 M 2 ´ w1
Þ Formula = C5H10O5
142 Solution and Colligative Properties

where, m = molality Using formula


i = van’t Hoff factor = 1 (for DT f = iK f ´ m
non-electrolyte/non-associable) DT f = 3 ´ 4 ´ 0.03 = 0.36 K
w2 = mass of solute in g = 1 g (present in both of
the solutions) 5. Key Idea Osmotic pressure is proportional to the molarity (C)
M 2 = molar mass of solute in g mol -1 (same of the solution at a given temperature, p = CRT
solute in both of the solutions) Concentration of BaCl2 = 0.01M (Given)
w1 = mass of solvent in g = 100 g (for both of the p XY = 4 p BaCl 2 (Given)
solvents A and B) i ´ CRT = 4 ´ i ´ CRT …(i)
K b = ebullioscopic constant For the calculation of i,
So, the expression becomes, XY ¾® X + + Y - (Here, i = 2)

w
DTb µ K b 2+ -
BaCl2 ¾® Ba + 2Cl (Here, i = 3)
DTb ( A ) K b ( A ) 1 é K b ( A) 1 ù
Þ = = ê Given = ú Putting the values of i in (i)
DTb ( B ) K b ( B ) 5 ë K b ( B) 5 û
2 ´ [ XY ] = 4 ´ 3 ´ [ BaCl2 ]

Flo
3. Key Idea For dilute solution, lowering of vapour pressure 2 ´ [ XY ] = 12 ´ 0.01
Dp
(Dp) = p0 - p and relative lowering of vapour pressure = 0 12 ´ 0.01

ree
p [ XY ] =
2
which is a colligative property of solutions.
So, the concentration of XY = 0.06 mol L-1
Dp

F
= c B ´ i Þ Dp = c B ´ i ´ p0 = 6 ´ 10-2 mol L-1
p0
where, p0 = vapour pressure of pure solvent 6. Molecules of benzoic acid dimerise in benzene as:

or
i = van’t Hoff factor
ur 2(C6H5COOH) - (C6H5COOH)2

f
c B = mole fraction of solute Now, we know that depression in freezing point (DT f ) is given
by following equation:
ks
Given, i ´ K f ´ wsolute ´ 1000
DT f = i ´ K f ´ m =
Yo
...(i)
p° = vapour pressure of pure water of 25º C Mwsolute ´ wsolvent
oo
= 35 mm Hg
Given, wsolute (benzoic acid) = w g
c B = mole fraction of solute (urea)
eB

wsolvent (benzene) = 30 g
0.60
nB 0.01 Mw Solute (benzoic acid) = 122 g mol- 1, DT f = 2 K
= = 60 =
nA + nB 360 + 0.60 20 + 0.01 K f = 5 Kkg mol- 1, %a = 80 or a = 0.8
r

18 60
ou
ad

2(C6H5COOH) - (C6H5COOH)2
0.01
= = 0.0005 Initial 1 0
Y

20.01 Final 1-a a/2


i = van’t Hoff factor = 1 (for urea) = 1 - 0. 8 = 0. 2 0. 8 / 2 = 0. 4
Now, according to Raoult’s law Total number of moles at equilibrium = 0.2 + 0.4 = 0.6
nd
Re

Dp = c B ´ i ´ pº Number of moles at equilibrium


i=
On substituting the above given values, we get Number of moles present initially
Fi

Dp = 0.0005 ´ 1 ´ 35 = 0.0175 mm Hg 0.6


i= = 0.6
1
4. Key Idea Depression in freezing point (D Tf ) is given by
On substituting all the given values in Eq. (i), we get
DTf = iK f m
0.6 ´ 5 ´ w ´ 1000
i = vant Hoff factor 2= , w = 2.44 g
K f = molal depression constant 122 ´ 30
m = molality Thus, weight of acid (w) is 2.4 g.

K f = 4.0 K kg mol -1 (Given) 7. Given, Freezing point of 4% aqueous solution of X .


-1 = Freezing point of 12% aqueous solution of Y
m = 0.03 mol kg (Given)
or (DT f )X = (DT f )Y [Q DT f = T f° - T f ]
DT f = ?
For K 2SO4, i = 3 K f ´ mX = K f mY
It can be verified by the following equation : where, mX and mY are molality of X and Y, respectively.
K 2SO4 - 2K + + SO2- or mX = mY
4
Solution and Colligative Properties 143

Now, molality =
Number of moles of solute (n) For the glucose solution (van’t Hoff factor, i = 1),
Mass of solvent (in kg) DTb1m = DT f2m = 2K
Weight
n= So, Kb ´ 1 ´ 1 = K f ´ 2 ´ 1 Þ K b = 2K f
Molecular mass
wX wY 11. Considering the expression of the depression in freezing point of
= a solution,
M X ´ (wsolvent )1 MY ´ (wsolvent )2
DT f = K f ´ m ´ i
Given, wX = 4 and w(solvent )1 =96 wB ´ 1000
wY = 12 and w(solvent )2 = 88 T f° - Tf = K f ´ ´i …(i)
M B ´ wA (in g )
MX = A
4 ´ 1000 12 ´ 1000 Here, T f° = 0°C, T f = - 10°C
\ =
M X ´ 96 MY ´ 88 wB = mass of ethylene glycol = 62 g

w
12 ´ 1000 ´ M X ´ 96 M B = molar mass of ethylene glycol
Thus, MY = æ ö
4 ´ 1000 ´ 88 ç CH2 — CH2 ÷
ç ÷
96 ´ 12 ç ½ ½ ÷
= ´ A = 3.27A » 3A

Flo
ç ÷
4 ´ 88 ç OH OH ÷
è ø
-1
8. The ionisation of K2HgI4 in aqueous solution is as follows: = 62 g mol

ree
K 2 [ HgI4 ] - 2K + + [ HgI4 ]2 - wA = mass of water in g as liquid solvent,
i = van’t-Hoff factor = 1 (for ethylene glycol in water)
van’t Hoff factor (i) for ionisation reaction is given as,
K f = 1.86 K kg mol -1

F
i = 1 + a ( n - 1)
where, On substituting in Eq. (i), we get

or
62 ´ 1000
n = number of ions,
ur 0 - (- 10) = 186
. ´ ´1
a = degree of ionisation or dissociation 62 ´ wA

f
From above equation, it is clear that n = 3 . ´ 62 ´ 1000
186
Þ wA = = 186 g
ks
i = 1 + 0.4 (3 - 1) 10 ´ 62
Yo
[Given, % a = 40% or a = 0.4] So, amount of water separated as ice (solid solvent)
oo
= 1.8 = 250 - wA = (250 - 186)g = 64 g
eB

9. We know that,
12. Key idea ‘‘Addition of solute particles to a pure solvent results
Depression in freezing points (DT f ) to depression in its freezing point.’’
T ° f - Tf = K f ´ m ´ i
All the compounds given in question are ionic in nature so,
K f = molal depression constant
r

where, consider their van’t Hoff factor (i ) to reach at final conclusion.


ou

wsolute ´ 1000
ad

m = molality = The solution with maximum freezing point must have minimum
M solute ´ wsolvent (in g)
Y

number of solute particles. This generalisation can be done with


i = van’t Hoff factor the help of van’t Hoff factor (i ) i.e.
For diluted milk Number of solute particles µ van’t Hoff factor (i )
nd
Re

DT f1 = K f ´ m1 ´ i Thus, we can say directly


wmilk ´ 1000 Solution with maximum freezing point will be the one in which
Þ 0 - (0.2) Þ 0.2 = K f ´ ´1
Fi

M milk ´ w1 (H2O) solute with minimum van’t Hoff factor is present


For pure milk Now, for Co(H2 O)6 ]Cl 3
- [Co(H2 O)6 ]3+ + 3Cl -
DT f2 = K f ´ m2 ´ i van’t Hoff factor ( i ) is 4. Similarly for,
wmilk ´ 1000 2+ -
[Co(H O) Cl]Cl × H O -[Co(H O) Cl] + 2Cl ‘i’ is 3
Þ 0 - (-0.5) = 0.5 = K f ´ ´1 2 5 2 2 2 5
M milk ´ w2 (H2O) [Co(H O) Cl ]Cl × 2H O -[Co(H O) Cl ] + Cl + -
‘i’ is 2
2 4 2 2 2 4 2
0.2 K f wmilk ´ 1000 M ´ w2 (H2O) w2 (H 2O)
So, = ´ ´ milk = and for [Co(H2O)3 Cl 3 ]× 3H2O, ‘i’ is 1 as it does not show
0.5 K f M milk ´ w1 (H2O) wmilk ´1000 w1 (H2O) ionisation. Hence, [Co(H2O)3 Cl 3 ]× 3H2O have minimum
w2 (H2O) (in pure milk) 2 number of particles in the solution.
Þ =
w1 (H2O) (in diluted milk) 5 So, freezing point of its solution will be maximum.
i.e. 3 cups of water has to be added to 2 cups of pure milk. 13. Let the degree of association of acetic acid (CH3COOH) in
10. Elevation in boiling point (DTb ) = K b ´ m ´ i benzene is a, then
2CH3 COOH q (CH3 COOH)2
Depression is freezing point (DT f ) = K f ´ m ´ i Initial moles 1 0
where, m = molality Moles at equilibrium 1-a
a
2
144 Solution and Colligative Properties

a a a Also, from Raoult’s law of lowering of vapour pressure :


\ Total moles = 1 - a + =1- or i = 1 -
2 2 2 - Dp n2 n
= x2 = » 2 [Q n1 >> n2 ]
Now, depression in freezing point (DT f ) is given as p° n1 + n2 n1
DT f = i K f m K (i)
5 18
where, K f = molal depression constant or Þ - Dp = 760 ´ ´ = 36 mm of Hg
cryoscopic constant. 19 100
m = molality Þ p = 760 - 36 = 724 mm of Hg

number of moles of solute 0.2 1000 17. van’t Hoff factor (i) = 4 {3K++ [Fe(CN)6 ]3
Molality = = ´
weight of solvent (in kg) 60 20 0.1 1000 1
Molality = ´ =
329 100 329
Putting the values in Eq. (i)
Þ – DT f = iK f . m
é aù é 0.2 1000 ù
\ 0.45 = ê1 - ú (512 ´

w
. )ê 1
ë 2û ë 60 20 úû = 4 ´ 1.86 ´ = 2.3 ´ 10–2
329
a 0.45 ´ 60 ´ 20 Þ T f = –2 .3 ´ 10–2 ° C
1- =
. ´ 0.2 ´ 1000
2 512

Flo
(As % freezing point of water is 0ºC)
a a æ 20 ö 1000
Þ 1 - = 0.527 Þ = 1 - 0.527 18. Molality = ç ÷´ = 2.325 m
2 2 è 172 ø 50

ree
\ a = 0.946 Þ - DT f = 2 = iK f × m
Thus, percentage of association = 94.6% 2

F
Þ i= = 0.5
1.72 ´ 2.325
14. -DT f = ik f m 2
13.44

or
19. Molality =
=1´2 ´
34.5
´ 1000 = 3
ur 134.1
= 0.1
46 ´ 500

f
i=3
Vapour pressure curves shown in (b) is in agreement with the
Þ DTb = iK b × m = 3 ´ 0.52 ´ 0.1 = 0.156
ks
calculated value of -DT f . (a) is wrong, vapour pressure
Yo
decreases on cooling. 20. For isotonic solutions, they must have same concentrations of
oo
ions, Therefore,
15. PLAN This problem includes concept of colligative properties
(osmotic pressure here) and van’t Hoff factor. Calculate the 0.004 i (Na 2SO4) = 0.01
eB

effective molarity of each solution. 0.01


Þ i= = 2.5
i.e. effective molarity = van’t Hoff factor ´ molarity 0.004
0.5 M C2H5OH (aq) i=1 Also Na 2SO4 r 2Na + + SO24- i
r

Effective molarity = 0 .5 1- a 2a a 1 + 2a
ou
ad

0.25 M KBr (aq) i=2 Þ i = 1 + 2a = 2.5


Effective molarity = 0 .5 M a = 0.75 = 75%
Y

0.1 M Mg3 (PO4 )2(aq) i=5


21. During freezing, liquid solvent solidify and solid solvent
Effective molarity = 0 . 5 M remains in equilibrium with liquid solvent.
nd
Re

0.125 M Na 3PO4 (aq) i=4


Effective molarity = 0. 5 M 22. In benzene, benzoic acid dimerises as :
Fi

1
Molarity is same hence, all colligative properties are also same. C6H5COOH r (C H COOH)2
2 6 5
NOTE This question is solved by assuming that the examiner 23. C6 H5 NH3 Cl : i = 2;
has taken Mg 3(PO 4 )2 to be completely soluble. However, in real it
is insoluble (sparingly soluble). Ca(NO3 )2 : i=3
16. The elevation in boiling point is La(NO3 )3 : i = 4;
n2 C6H12O6 : i=1
DTb = K b × m : m = molality = ´ 1000
w1 Lower the value of i, smaller will be the depression in freezing
[n2 = Number of moles of solute, w1 = Weight of point, higher will be the freezing temperature, if molalities are
solvent in gram] equal. Hence, glucose solution will have highest freezing
temperature.
n2
Þ 2 = 0.76 ´ ´ 1000 24. K2SO4 : i=3
100
5 NaCl : i = 2
Þ n2 =
19 Urea : i = 1
Glucose : i = 1
Solution and Colligative Properties 145

Greater the value of i, greater the lowering in freezing point, Actual molar mass of phenol = 94 g/mol
lower will be the freezing temperature, if molarity in all cases are Calculated molar mass
Now, van’t Hoff factor, i =
same. Therefore, K2SO4 solution has the lowest freezing point. Observed molar mass
94
25. Addition of HgI2 to KI solution establishes the following \ i= = 0.625
150.4
equilibrium :
Dimerisation of phenol can be shown as :
HgI2 + 2KI r K2[HgI4 ]
2C6H5OH r (C6 H5OH)2
The above equilibrium decreases the number of ions (4 ions on Initial 1 0
left side of reactions becomes three ions on right side), hence a
At equilibrium 1–a
2
rises the freezing point.
a
Total number of moles at equilibrium, i = 1 - a +
26. In depression of freezing point experiment, vapour pressure of 2

w
solution is less than that of pure solvent as well as only solvent a
molecules solidify at freezing point. i =1-
2
27. From the graph we can note a
But i = 0.625, thus, 0.625 = 1 -

Flo
DTb for solution X i.e., 2
DTb (X) = 362 - 360 = 2 a
= 1 - 0.625
Likewise, DTb for solution Y i.e., DTb (Y) = 368 - 367 = 1 2

ree
Now by using the formula a = 0.75
DTb = i ´ molality of solution× K b Thus, the percentage of phenol that dimerises is 75%.

F
For solution X 29. (i) DTb = K b × m2
2 = i ´ mNaCl ´ K b (X) …(i) 1.22 1000
Þ 0.17 = 1.7 ´ ´ Þ M = 122

or
Similarly for solution y
ur M 100
1= i ´ mNaCl ´ K b (Y) ……(ii) 1.22 1000

f
(ii) 0.13 = 2.6 ´ ´ Þ M = 244
from Eq. (i) and (ii) above M 100
ks
K b (X) 2 The above molar masses suggests thapt benzoic acid is
= or 2 or K b (X) = 2K b (Y)
Yo
K b (Y) 1 monomeric in acetone while dimeric in benzene.
oo

For solute S 30. Higher the value of K b of a solvent suggest that there is larger
eB

polarity of solvent molecules, which in turn implies higher


2 S ¾® S 2 (given due to dimerisation)
boiling point due to dipole-dipole interaction.
Initial a 0
a Therefore, the correct order of K b values of the three given
Final (1 - a)
2 solvents is
r

æ aö
ou
ad

So, here i = ç1- ÷ Solvents Boiling point Kb


è 2ø
X 100°C 0.63
Y

æ a ö
DTb [ X ](s) = ç1 - 1 ÷ K b (X) Y 27°C 0.53
è 2 ø Z 283°C 0.92
æ a2ö
nd
Re

DTb [Y] (s) = ç1- ÷ K b (Y) 31. Mass of water = 500 ´ 0.997 g = 498.5 g
è 2 ø
Fi

Given, DTb (X)(s) = 3DTb (Y)(s) Also CH3COOH r CH3COO- + H+


æ a1 ö æ a ö 1- a a a
ç1- ÷ K b (X) = 3 ´ ç1- 2 ÷ ´ K b (Y)
è 2 ø è 2 ø Þ i = 1 + a = 1.23
æ a ö æ a ö Þ - DT f = iK f × m = 1.23 ´ 1.86 ´
3 1000
´ = 0.23°C
or 2 ç1- 1 ÷ = 3 ç1- 2 ÷ [Q K b (X) = 2K b (Y) ]
è 2 ø è 2 ø 60 498.5
æ a ö æ 0.7 ö 32. (i) Empirical formula determination
or 2 ç1- 1 ÷ = 3 ç1- ÷ (as given, a 2 = 0.7)
è 2 ø è 2 ø Elements C H N O
or 4 - 2a 1 = 6 - 2.1 or 2a 1 = 01 . Weight % 42.86 2.40 16.67 38.07
so, a 1 = 0.05 Moles 3.57 2.40 1.19 2.38
1000 ´ K f ´ W B Simplest ratio 3 2 1 2
28. Molar mass of solute (M B ) =
W A ´ DT f
Þ Empirical formula = C3H2NO2
1000 ´ 14 ´ 75.2 5.5 1000
Þ MB = (ii) DTb = 1.84 = 2.53 ´ ´
1000 ´ 7 M 45
M B = 150.4 g per mol Þ M = 168
146 Solution and Colligative Properties

Q Empirical formula weight (84) is half of molar mass, 36. Vapour pressure = p (H2O) + p(ethanol )
molecular formula is C6H4N2O4 a dinitrobenzene :
= 32.8 ´ 0.1 + 40 ´ 0.9
NO2 = 3.28 + 36
= 39.28 mm
37. Now ethanol is solute.
NO2 0.1
(C6H4N2O4)
Molality of solute = ´ 1000 = 6.17
0.9 ´ 18
33. - DT f = K f × m2 Þ DTb = 6.17 ´ 0.52 = 3.20
Þ Tb = 373 + 3.2
0.3
Þ m2 = = 0.1613 = 376.2 K
1.86

w
n 1000 æ DT f ö
Also, m2 = 2 ´ = 0.1613 38. lim çç ÷÷ = K f (Cryoscopic constant)
n1 M1 m ®0è m ø

n2 0.1613 ´ 18 39. 1+ DT f = iK f m

Flo
Þ = = 2.9 ´ 10-3
n1 1000
DT f = 0 – (–0 .0558° C)
n2 n + n1 = 0.0558° C

ree
Þ + 1= 2 = 2.9 ´ 10-3 + 1
n1 n1 0.0558
Þ i (vant Hoff’s factor) = =3
n1 1 . ´ 0.01
186

F
Þ = c1 = = 0.997
n1 + n2 1 + 2.9 ´ 10-3 This indicates that complex upon ionisation produces three
ions as:
Þ p = p0c 1 = 23.51 ´ 0.997 = 23.44 mm

or
ur [Co(NH3 )5 Cl]Cl 2 ® [Co(NH3 )5 Cl]2+ (aq) + 2Cl - (aq)
34. - DT f = 5.51 - 5.03 = 0.48

f
Thus, only one Cl is inside the coordination sphere.
Þ - DT f = 0.48 = K f × m
40. MX 2 ¾® M 2+ + 2 X -
ks
0.643 1000
Þ 0.48 = 5.12 ´ ´
Yo
M 50 ´ 0.879 van’t Hoff factor for any salt can be calculated by using equation
oo
i = 1 + a (n - 1)
Þ M = 156 g/mol
where, n = number of constituent ions
eB

35. In the given solution ‘M’, H2O is solute. \ i (MX 2 ) = 1 + a (3 - 1) = 1 + 2a


0.1 (DT f )observed
Therefore, molality of H2O = ´ 1000 = 2.4 = i = 1 + 2a
0.9 ´ 46
r

(DT f )theoretical
ou

Þ - DT f = K ethanol ´ 2.4 = 2 ´ 2.4 = 4.8


ad

f
\ i = 1 + 2 ´ 0.5 Þ i=2
Þ T f = 155.7 – 4.8 = 150.9 K
Y
nd
Re

Download Chapter Test


Fi

http://tinyurl.com/yxr5edmo or
10
Electrochemistry

w
Topic 1 Electrochemical Cells 5. Consider the following reduction processes:
Zn 2+ + 2e- ¾
¾® Zn (s); E ° = - 0.76 V

Flo
Objective Questions I (Only one correct option)
1. Given, Ca 2+ + 2e- ¾
¾® Ca (s); E ° = - 2.87 V

ree
Co 3 + + e- ¾® Co 2 + ; E ° = + 181
. V Mg2+ + 2e- ¾
¾® Mg(s); E ° = - 2.36 V

Pb 4 + + 2e- ¾® Pb 2 + ; E ° = + 167
. V Ni2+ + 2e- ¾
¾® Ni(s); E ° = - 0.25 V

F
Ce4 + + e- ¾® Ce3 + ; E ° = + 161
. V The reducing power of the metals increases in the order
3+ - (2019 Main, 10 Jan I)
Bi + 3e ¾® Bi; E ° = + 0.20 V

or
(a) Zn < Mg < Ni < Ca (b) Ni < Zn < Mg < Ca
ur
Oxidising power of the species will increase in the order (c) Ca < Zn < Mg < Ni (d) Ca < Mg < Zn < Ni

f
(2019 Main, 12 April I)
4+ 4+ 3+
6. The anodic half-cell of lead-acid battery is recharged using
ks
3+
(a) Ce < Pb < Bi < Co electricity of 0.05 Faraday. The amount of PbSO4
Bi 3 + < Ce4 + < Pb 4 + < Co 3 +
Yo
(b) electrolysed in g during the process is (Molar mass of
oo
(c) Co 3 + < Ce4 + < Bi 3 + < Pb 4 + PbSO4 = 303g mol -1 ) (2019 Main, 9 Jan I)
Co 3 + < Pb 4 + < Ce4 + < Bi 3 + (a) 11.4 (b) 7.6 (c) 15.2 (d) 22.8
eB

(d)
2. A solution of Ni(NO3 )2 is electrolysed between platinum 7. How long (approximate) should water be electrolysed by
electrodes using 0.1 Faraday electricity. How many mole of passing through 100 amperes current so that the oxygen
r

Ni will be deposited at the cathode? (2019 Main, 9 April II) released can completely burn 27.66 g of diborane?
ou

(Atomic weight of B = 10.8 m)


ad

(2018 Main)
(a) 0.20 (b) 0.10 (c) 0.15 (d) 0.05
(a) 6.4 hours (b) 0.8 hours (c) 3.2 hours (d) 1.6 hours
Y

3. Calculate the standard cell potential (in V) of the cell in


which following reaction takes place °
8. Given, ECl -
° 3+
= 1.36 V, ECr = - 0.74 V
2 /Cl /Cr
Fe2+ ( aq ) + Ag+ ( aq ) ¾® Fe3 + ( aq ) + Ag( s ) ° 2 - 3 + = 1.33 V, E ° -
nd

ECr = 1.51 V
Re

O /Cr
2 7 MnO / Mn 2 + 4
Given that,
Among the following, the strongest reducing agent is
Fi

E °Ag + / Ag = x V (2017 Main)


E °Fe 2+ / Fe = y V (a) Cr (b) Mn 2+

E °Fe3 + / Fe = z V (2019 Main, 8 April II)


(c) Cr 3+
(d) Cl -
(a) x + 2 y - 3 z (b) x - y 9. For the following electrochemical cell at 298 K,
(c) x + y - z (d) x - z
Pt ( s ) | H2 ( g , 1bar ) | H+ ( aq , 1 M)
4. Given, that EOs2 / H 2O = +1.23V;
| | M 4 + ( aq ), M 2 + ( aq ) | Pt ( s )
= 2.05V; 2+
E sS O 2- / SO 2- [ M ( aq )]
2 8 4 Ecell = 0.092 V when = 10x
[ M 4 + ( aq )]
s
EBr / Br s
= +1.09V,
RT
Given : E °M 4+ / M 2+ = 0.151 V; 2.303
2
= 0.059 V
3+ / Au = +1.4V
s
EAu F
The value of x is (2016 Adv.)
The strongest oxidising agent is (2019 Main, 8 April I)
(a) - 2 (b) - 1
(a) Au 3+ (b) O2 (c) S2 O2-
8 (d) Br2 (c) 1 (d) 2
148 Electrochemistry

10. Two Faraday of electricity is passed through a solution of 17. The standard reduction potential values of three metallic
CuSO4 . The mass of copper deposited at the cathode is cations, X, Y, Z are 0.52, - 3.03 and - 1.18 V respectively.
(at. mass of Cu = 63.5 u) (2015 Main) The order of reducing power of the corresponding metals is
(a) 0 g (b) 63.5 g (c) 2 g (d) 127 g (a) Y > Z > X (b) X > Y > Z (1998, 2M)

° 3+ = - 0.74 V; E ° -
11. Given, ECr = 1.51 V (c) Z > Y > X (d) Z > X > Y
/Cr MnO /Mn 2+4
18. The standard reduction potentials E°, for the half reactions
° 2- 3+ = 1.33 V; E ° - = 1.36 V
ECr O /Cr
2 7 Cl /Cl are as
Based on the data given above strongest oxidising agent will Zn = Zn 2+ + 2e- , E° = + 0.76 V
be (2013 Main) Fe = Fe2+ + 2e- , E° = 0.41 V
(a) Cl (b) Cr 3+ (c) Mn 2+ (d) MnO -4 The emf for the cell reaction,
12. Electrolysis of dilute aqueous NaCl solution was carried out Fe2+ + Zn ® Zn 2+ + Fe is

w
(1989, 1M)
by passing 10 mA current. The time required to liberate (a) – 0.35 V (b) + 0.35 V (c) + 1.17 V (d) - 1.17 V
0.01 mole of H2 gas at the cathode is (1 F = 96500 C mol -1 )
19. When a lead storage battery is discharged (1986, 1M)
(a) 9.65 ´ 104 s (b) 19.3 ´ 104 s

Flo
(2008, 3M) (a) SO2 is evolved
(c) 28.95 ´ 104 s (d) 38.6 ´ 104 s (b) lead is formed
13. In the electrolytic cell, flow of electrons is from (c) lead sulphate is consumed

ree
(2003, 1M)
(a) cathode to anode in solution (d) sulphuric acid is consumed
(b) cathode to anode through external supply 20. The reaction,

F
(c) cathode to anode through internal supply 1
H2 ( g ) + AgCl( s ) r H+ ( aq ) + Cl - ( aq ) + Ag ( s )
(d) anode to cathode through internal supply 2

or
14. Standard
ur
electrode potential data are useful for occurs in the galvanic cell (1985, 1M)
understanding the suitability of an oxidant in a redox (a) Ag |AgCl ( s )|KCl(soln) | AgNO3 |Ag
titration. Some half-cell reactions and their standard
f
(b) Pt|H2 ( g )|HCl(soln) | AgNO3 (soln)|Ag
ks
potentials are given below : (c) Pt |H2 ( g )HCl (soln) | AgCl ( s )|Ag
Yo
MnO-4 ( aq) + 8H+ ( aq ) + 5e- ¾® Mn 2+ ( aq ) + 4H2 O( l ), (d) Pt | H2 ( g ) | KCl (soln) | AgCl( s )|Ag
oo
E° = 1.51 V
2- + - 3+ 21. The electric charge for electrode deposition of one gram
Cr2 O7 ( aq ) + 14 H ( aq ) + 6e ¾® 2Cr ( aq ) + 7H2 O( l ) ,
eB

equivalent of a substance is (1984, 1M)


E° = 1.38 V (a) one ampere per second
Fe3+ ( aq ) + e- ¾® Fe2+ ( aq ) E° = 0.77 V (b) 96,500 coulombs per second
r

Cl 2 ( g ) + 2e- ¾® 2Cl - ( aq ) E° = 1.40 V (c) one ampere for one hour


ou
ad

Identify the incorrect statement regarding the quantitative (d) charge on one mole of electrons
estimation of aqueous Fe(NO3 )2 22. A solution containing one mole per litre of each Cu (NO3 )2 ,
Y

(2002, 3M)
(a) MnO-4 can be used in aqueous HCl AgNO3 ,Hg 2 (NO3 )2 and Mg(NO3 )2 is being electrolysed by
(b) Cr2 O2- using inert electrodes. The values of standard electrode
7 can be used in aqueous HCl
nd

potentials in volts (reduction potential) are


Re

(c) MnO-4 can be used in aqueous H2 SO4


Ag + / Ag = + 0.80, Hg 2+
2 / 2Hg = + 0.79
(d) Cr2 O2-
7 can be used in aqueous H2 SO4
Fi

Cu 2+ / Cu = + 0.34, Mg 2+ / Mg = - 2.37
15. Saturated solution of KNO3 is used to make ‘salt-bridge’ With increasing voltage, the sequence of deposition of
because (2001, 1M) metals on the cathode will be (1984, 1M)
(a) velocity of K + is greater than that of NO–3 (a) Ag, Hg, Cu, Mg (b) Mg, Cu, Hg, Ag
(b) velocity of NO-3 is greater than that of K + (c) Ag, Hg, Cu (d) Cu, Hg, Ag
(c) velocities of both K + and NO-3 are nearly the same 23. Faraday’s laws of electrolysis are related to the (1983, 1M)
(d) KNO3 is highly soluble in water (a) atomic number of the cation
16. The gas X at 1 atm is bubbled through a solution containing a (b) atomic number of the anion
mixture of 1 M Y - and 1 M Z - at 25° C . If the order of (c) equivalent weight of the electrolyte
reduction potential is Z > Y > X , then (1999, 2M) (d) speed of the cation
(a) Y will oxidise X and not Z 24. The standard reduction potentials at 298K for the following
(b) Y will oxidise Z and not X half cells are given :
(c) Y will oxidise both X and Z Zn 2 + ( aq ) + 2e- r Zn ( s ) ; E° = -0.762 V
(d) Y will reduce both X and Z
Cr 3 + ( aq ) + 3e- r Cr ( s ) ; E° = -0.740 V
Electrochemistry 149

2H+ ( aq ) + 2e- r H2 ( g ); E° = 0.000 V in diverse areas such as analytical chemistry, biochemistry,


3+ - 2+ electrochemistry and radiochemistry. The following
Fe ( aq ) + e r Fe ( aq ); E° = 0.770 V
example illustrates a typical case, involving
Which is the strongest reducing agent? (1981, 1M)
chemical/electrochemical reaction, which requires a clear
(a) Zn( s ) (b) Cr( s ) (c) H2 ( g ) (d) Fe2+ ( aq ) understanding of the mole concept. A 4.0 M aqueous solution
of NaCl is prepared and 500 mL of this solution is
Objective Questions II electrolysed. This leads to the evolution of chlorine gas at
(One or more than one correct option) one of the electrodes
(atomic mass : Na = 23, Hg = 200 , 1 F = 96500 C).
25. In a galvanic cell, the salt-bridge (2014 Adv.)
(2007, 3 ´ 4M = 12M)
(a) does not participate chemically in the cell reaction
(b) stops the diffusion of ions from one electrode to 29. The total number of moles of chlorine gas evolved is
(a) 0.5 (b) 1.0

w
another
(c) is necessary for the occurrence of the cell reaction (c) 2.0 (d) 3.0
(d) ensures mixing of the two electrolytic solutions 30. If the cathode is a Hg electrode, the maximum weight
NO–3 (in gram) of amalgam formed from this solution is

Flo
26. For the reduction of ion in an aqueous solution E° is
(a) 200 (b) 225
+ 0.96 V. Values of E° for some metal ions are given below
(c) 400 (d) 446
V2+ ( aq ) + 2e– ¾® V; E° = - 1.19 V

ree
Fe3+ ( aq ) + 3e– ¾® Fe; E° = -0.04V 31. The total charge (coulombs) required for complete
Au 3+ ( aq ) + 3e– ¾® Au; E° = + 1.40 V electrolysis is

F
Hg 2+ ( aq ) + 2e– ¾® Hg ; E° = + 0.86V (a) 24125 (b) 48250
(c) 96500 (d) 193000
The pair(s) of metals that is/are oxidised by NO–3
in aqueous

or
solution is (are)
ur (2009)
(a) V and Hg (b) Hg and Fe (c) Fe and Au (d) Fe and V
Subjective Questions

f
32. The following electrochemical cell has been set-up :
Numerical Value Based Question Pt (1) | Fe3+ , Fe2+ ( a = 1) | Ce4+ , Ce3+ ( a = 1) | Pt (2)
ks
E° (Fe3+ , Fe2+ ) = 0.77 V
Yo
27. For the electrochemical cell,
oo
Mg( s )|Mg 2 + ( aq, 1 M) || Cu 2 + ( aq , 1 M) | Cu( s ) and E° (Ce4+ , Ce3+ ) = 1.61 V
If an ammeter is connected between the two platinum
eB

The standard emf of the cell is 2.70 V at 300 K. When the


concentration of Mg 2+ is changed to x M, the cell potential electrodes, predict the direction of flow of current, will the
current increases or decreases with time? (2000, 2M)
changes to 2.67 V at 300 K. The value of x is _____.
r

F 33. Copper sulphate solution (250 mL) was electrolysed using a


(Given, = 11500 K V-1 , where F is the Faraday constant
ou

platinum anode and a copper cathode. A constant current of


ad

R
2 mA was passed for 16 min. It was found that after
and R is the gas contant, In (10) = 2.30) (2018 Adv.)
Y

electrolysis the absorbance of the solution was reduced to


28. Consider an electrochemical cell : 50% of its original value. Calculate the concentration of
A( s ) | A n + ( aq , 2M )|| B 2 n + ( aq ,1M)| B( s ). The value of DH s copper sulphate in the solution to begin with.
nd

(2000, 3M)
Re

for the cell reaction is twice of DGs at 300 K. If the emf of 34. A cell, Ag | Ag + || Cu 2+ | Cu , initially contains 1 M Ag + and
the cell is zero, the DS s (in J K -1 mol -1 ) of the cell reaction
Fi

1 M Cu 2+ ions. Calculate the change in the cell potential


per mole of B formed at 300 K is …… . after the passage of 9.65 A of current for 1 h. (1999, 6M)
(Given : ln( 2 ) = 0.7, R (universal gas constant) = 8.3 J K -1 35. How many grams of silver could be plated out on a serving
mol -1 . H , S and G are enthalpy, entropy and Gibbs energy, tray by electrolysis of a solution containing silver in +1
respectively.) (2018 Adv.) oxidation state for a period of 8.0 h at a current of 8.46 A?
What is the area of the tray, if the thickness of the silver
Passage Based Questions plating is 0.00254 cm? Density of silver is 10.5 g/cm3 .
(1997, 3M)
Passage 36. The Edison storage cell is represented as:
Chemical reactions involve interaction of atoms and Fe ( s ) / FeO( s) / KOH( aq ) /Ni 2 O3 ( s ) / Ni( s )
molecules. A large number of atoms/molecules The half-cell reactions are :
(approximately 6.023 ´ 1023 ) are present in a few grams of Ni 2 O3 ( s ) + H2 O ( l ) + 2e- r 2NiO( s ) + 2OH- ,
any chemical compound varying with their atomic/molecular E°= + 0.40 V
masses. To handle such large numbers conveniently, the FeO ( s ) + H2 O( l ) + 2e- r Fe( s ) + 2OH- ,
mole concept was introduced. This concept has implications E° = -0.87 V
150 Electrochemistry

(i) What is the cell reaction? 44. In a fuel cell hydrogen and oxygen react to produces
(ii) What is the cell emf ? How does it depend on the electricity. In the process hydrogen gas is oxidised at the
concentration of KOH? anode and oxygen at the cathode. If 67.2 L of H2 at STP react
(iii) What is the maximum amount of electrical energy that can in 15 min, what is the average current produced? If the entire
be obtained from one mole of Ni 2O3? (1994, 4M) current is used for electro deposition of copper from copper
37. The standard reduction-potential for the half-cell (II) solution, how many grams of copper will be deposited?
NO-3 ( aq ) + 2H+ + e- ¾® NO2 ( g ) + H2 O is 0.78 V Anode reaction : H2 + 2OH- ¾® 2H2 O + 2e-
+
(i) Calculate the reduction-potential in 8M H . Cathode reaction : O2 + 2H2 O + 2e- ¾® 4OH- (1988, 4M)
(ii) What will be the reduction-potential of the half-cell in a
neutral solution? Assume all the other species to be at unit 45. A cell contains two hydrogen electrodes. The negative
concentration. (1993, 2M) electrode is in contact with a solution of 10-6 M hydrogen

w
38. Chromium metal can be plated out from an acidic solution ions. The emf of the cell is 0.118 V at 25° C . Calculate the
containing Cr O3 according to the following equation. concentration of hydrogen ions at the positive electrode.
(1988, 2M)
Cr O3 ( aq ) + 6H+ ( aq ) + 6e- ¾® Cr ( s ) + 3H2 O

Flo
46. A 100 watt, 110 V incandecent lamp is connected in series
Calculate (i) How many grams of chromium will be plated with an electrolyte cell containing cadmium sulphate
out by 24,000 C and (ii) How long will it take to plate out

ree
solution. What weight of cadmium will be deposited by the
1.5 g of chromium by using 12.5 A current? (1993, 2M) current flowing for 10 h? (1987, 5M)
39. An aqueous solution of NaCl on electrolysis gives

F
47. During the discharge of a lead storage battery, the density of
H2 ( g ), Cl 2 ( g ) and NaOH according to the reaction. sulphuric acid fell from 1.294 to 1.139 g/mL. Sulphuric acid
2Cl – ( aq ) + 2H2 O r 2OH- ( aq ) + H2 ( g ) + Cl 2 ( g ) of density 1.294 g/mL is 39% H2 SO4 by weight and that of

or
ur
A direct current of 25 A with a current efficiency of 62% is density 1.139 g/mL is 20% H2 SO4 by weight. The battery

f
passed through 20 L of NaCl solution (20% by weight). holds 3.5 L of the acid and the volume remained practically
Write down the reactions taking place at the anode and constant during the discharge.
ks
cathode. How long will it take to produce 1kg of Cl 2 ? What Calculate the number of ampere-hours for which the battery
Yo
will be the molarity of the solution with respect to hydroxide
oo
must have been used. The charging and discharging reactions
ion? (Assume no loss due to evaporation) (1992, 3M) are
eB

40. For the galvanic cell, Pb + SO24 - = PbSO4 + 2e- (charging)


Ag | AgCl( s ) , KCl (0.2 M) || KBr (0.001 M), AgBr (s) | Ag PbO2 + 4H+ + SO2- -
4 + 2e
Calculate the emf generated and assign correct polarity to = PbSO4 + 2H2 O (discharging)
r

(1986, 5M)
each electrode for a spontaneous process after taking into
ou
ad

account the cell reaction at 25° C. 48. How long a current of 3 A has to be passed through a
[ K sp (AgCl) = 2.8 ´ 10-10 , K sp (AgBr) = 3.3 ´ 10-13] (1992, 4M) solution of silver nitrate to coat a metal surface of 80 cm2
Y

with a 0.005 mm thick layer?


41. A current of 1.70 A is passed through 300.0 mL of 0.160M
solution of a ZnSO4 for 230 s with a current efficiency of Density of silver is 10.5 g/cm3 . (1985, 3M)
nd
Re

90%. Find out the molarity of Zn 2+ after the deposition Zn. 49. In an electrolysis experiment current was passed for 5 h
Assume the volume of the solution to remain constant during through two cells connected in series. The first cell contains a
Fi

the electrolysis. (1991, 4M) solution of gold and the second contains copper sulphate
42. Calculate the quantity of electricity that would be required to solution. 9.85 g of gold was deposited in the first cell. If the
reduce 12.3 g of nitrobenzene to aniline, if the current oxidation number of gold is +3, find the amount of copper
efficiency for the process is 50%. If the potential drop across deposited on the cathode of the second cell. Also calculate
the cell is 3.0 V, how much energy will be consumed? the magnitude of the current in ampere.
(1990, 3M) (Atomic weight of Au = 197 and atomic weight of
43. An acidic solution of Cu 2+ salt containing 0.4 g of Cu 2+ is Cu = 63.5) (1983, 3M)
electrolysed until all the copper is deposited. The electrolysis
50. A current of 3.7 A is passed for 6 h between nickel electrodes
is continued for seven more minutes with the volume of
in 0.5 L of a 2.0 M solution of Ni(NO3 )2 . What will be the
solution kept at 100 mL and the current at 1.2 A. Calculate
molarity of solution at the end of electrolysis? (1978, 2M)
the volume of gases evolved at NTP during the entire
electrolysis. (1989, 5M)
Topic 2 Conductivity of Electrolytic Solutions and their
Measurement and Nernst Equation
Objective Questions I (Only one correct option) 6. Given the equilibrium constant ( K C ) of the reaction :
1. The decreasing order of electrical conductivity of the Cu( s ) + 2Ag+ ( aq ) ¾® Cu2 + ( aq ) + 2Ag( s )
following aqueous solution is
°
is 10 ´ 1015 , calculate the Ecell of this reaction at 298 K.
0.1 M formic acid (A),
0.1 M acetic acid (B), é RT ù
0.1 M benzoic acid (C). (2019 Main, 12 April II) êë2.303 F at 298 K = 0.059 V úû
(2019 Main, 11 Jan II)
(a) A > C > B (b) C > B > A

w
(a) 0.4736 V (b) 0.04736 mV
(c) A > B > C (d) C > A > B (c) 0.4736 mV (d) 0.04736 V
2. Which one of the following graphs between molar
7. For the cell, Zn( s)|Zn2+ ( aq )||M x+ ( aq )|M ( s), different
conductivity (Lm ) versus C is correct?

Flo
half cells and their standard electrode potentials are given
(2019 Main, 10 April II)
below.
Na

ree
Au 3+ (aq)/ Ag + (aq)/ Fe 3+ (aq)/ Fe 2+ (aq)/
Cl

KC M x+ (aq)/M(s)
l Au(s) Ag(s) Fe 2+ (aq) Fe(s)
(a) L m (b) L m
E ° M x + / M /V
Na

F
1.40 0.80 0.77 - 0.44
KC

Cl
l

If E°Zn 2+ / Zn = - 0.76 V, which cathode will give a maximum

or
ÖC
ur ÖC value of E°cell per electron transferred? (2019 Main, 11 Jan I)
KC

f
Ag+ Fe2+ Au3+ Fe3 +
l

(a) (b) (c) (d)


Ag Fe Au Fe2+
ks
(c) L m (d) L m
Yo
Na 8. In the cell,
Na

KC

oo
Cl
Cl

Pt(s) H 2 ( g , 1 bar) HCl( aq )|AgCl( s ) Ag( s ) Pt( s ) the cell


potential is 0.92 V when a 10-6 molal HCl solution is used.
eB

ÖC ÖC
The standard electrode potential of ( AgCl/ Ag,Cl- )
3. Consider the statements S1 and S 2 :
ì 2.303RT ü
S 1 : Conductivity always increases with decrease in the electrode is í Given, = 0.06 V at 298 K ý
r

î F þ
concentration of electrolyte.
ou
ad

(a) 0.40 V (b) 0.20 V (c) 0.94 V (d) 0.76 V


S 2 : Molar conductivity always increases with decrease in
9. If the standard electrode potential for a cell is 2V at 300 K,
Y

the concentration of electrolyte.


the equilibrium constant (K) for the reaction,
The correct option among the following is
Zn( s ) + Cu2+ ( aq ) - Zn 2+
( aq ) + Cu( s )
nd

(2019 Main, 10 April I)


Re

(a) S1 is correct and S2 is wrong at 300 K is approximately


(b) S1 is wrong and S2 is correct
( R = 8 JK -1 mol-1 , F = 96000 C mol-1 )
Fi

(c) Both S1 and S2 are wrong


(2019 Main, 9 Jan II)
(d) Both S1 and S2 are correct -160 160 -80
(a) e (b) e (c) e (d) e320
4. The standard Gibbs energy for the given cell reaction in kJ
mol-1 at 298 K is 10. For the following cell,
Zn( s ) + Cu2 + ( aq ) ¾® Zn2 + ( aq ) + Cu( s ), Zn ( s )| ZnSO4 ( aq )||CuSO4 ( aq )|Cu( s )
E° = 2V at 298 K when the concentration of Zn2+ is 10 times the concentration
(Faraday’s constant, F = 96000 C mol-1 ) of Cu2+ , the expression for DG (in J mol -1 ) is
(2019 Main, 9 April I) [F is Faraday constant; R is gas constant;
(a) 384 (b) 192 (c) -384 (d) -192 T is temperature; E° (cell) = 11
. V] (2017 Adv.)
5. L°m for NaCl, HCl and NaA are 126.4, 425.9 and (a) 2.303 RT +11
. F (b) 1.1 F
100.5 S cm 2 mol- 1 , respectively. If the conductivity of (c) 2.303 RT - 2.2 F (d) -2.2 F
0.001 M HA is 5 ´ 10- 5 S cm - 1 , degree of dissociation of 11. Galvanisation is applying a coating of (2016 Main)

HA is (2019 Main, 12 Jan II)


(a) Cr (b) Cu
(c) Zn (d) Pb
(a) 0.25 (b) 0.50 (c) 0.75 (d) 0.125
152 Electrochemistry

12. Given below are the half-cell reactions (2014 Main) Column I Column II
Mn 2 + + 2e- ¾® Mn ; E° = - 1.18 eV P. (C2 H5 )3 N + CH3 COOH 1. Conductivity decreases
2 (Mn 3+ + e- ¾® Mn 2+ ) ; E° = + 1.51 eV and then increases
X Y
The E ° for 3Mn 2 + ® Mn + 2Mn 3+ will be
Q. KI(0.1 M) + AgNO3 (0.01 M) 2. Conductivity decreases
(a) - 2. 69 V; the reaction will not occur
X Y and then does not
(b) - 2. 69 V; the reaction will occur
change much
(c) - 0. 33 V; the reaction will not occur
(d) - 0. 33 V; the reaction will occur R. CH3 COOH + KOH 3. Conductivity increases
13. The equivalent conductance of NaCl at concentration C and X Y and then does not
at infinite dilution are lC and l¥ , respectively. The correct change much
relationship between lC and l¥ is given as (where, the S. NaOH + HI 4. Conductivity does not

w
constant B is positive) (2014 Main) change much and then
X Y
(a) lC = l¥ + ( B ) C (b) lC = l¥ - ( B ) C increases
(c) lC = l¥ - ( B ) C (d) lC = l¥ + ( B ) C

Flo
Codes
14. Resistance of 0.2 M solution of an electrolyte is 50 W . The P Q R S P Q R S
specific conductance of the solution of 0.5 M solution of same (a) 3 4 2 1 (b) 4 3 2 1

ree
electrolyte is 1.4 S m-1 and resistance of same solution of the (c) 2 3 4 1 (d) 1 4 3 2
same electrolyte is 280 W . The molar conductivity of 0.5 M 17. Consider the following cell reaction,
solution of the electrolyte in Sm 2mol-1 is

F
(2014 Main) 2Fe( s ) + O2 ( g ) + 4H+ ( aq ) ¾® 2Fe2+ ( aq ) + 2H2 O ( l ),
-4 -3
(a) 5 ´ 10 (b) 5 ´ 10 E ° = 1.67 V

or
(c) 5 ´ 10 3
ur 2
(d) 5 ´ 10
At [Fe2+ ] = 10-3 M, P(O 2 ) = 0.1 atm and pH = 3, the cell

f
15. The standard reduction potential data at 25°C is given below. potential at 25° C is (2011)
E° (Fe3 + / Fe2 + ) = + 0.77 V; E° (Fe2 / Fe) = -0.44 V; (a) 1.47 V (b) 1.77 V
ks
E° (Cu 2 + / Cu) = + 0.34 V; E° (Cu + / Cu) = + 0.52 V; (c) 1.87 V (d) 1.57 V
Yo
oo
E ° ( O2 ( g ) + 4H+ + 4 e- ) ¾® 2H2 O) = + 1.23 V; 18. AgNO3 (aqueous) was added to an aqueous KCl solution
- gradually and the conductivity of the solution was measured.
E ° ( O2 ( g ) + 2H2 O + 4 e ) ¾® 4OH) = + 0.40 V
eB

The plot of conductance (L) versus the volume of AgNO3 is


E° (Cr 3 + / Cr) = -0.74 V; E° (Cr 2 + / Cr) = +0.91 V
× ×
L ×L L L ×
××

Match E° of the rebox pair in Column I with the values given ×


× ××
×
r

× × ×× × ××× ×
××
×

in Column II and select the correct answer using the code ×


×

×
ou

×
ad

given below the lists. (2013 Adv.) Volume Volume Volume Volume
Y

(P) (Q) (R) (S)


Column I Column II (2011)

P. E° (Fe 3+
/ Fe) 1. - 0.18 V (a) (P) (b) (Q) (c) (R) (d) (S)
nd
Re

19. The half cell reactions for rusting of iron are :


Q. E° ( 4H2 O r 4H+ + 4OH- ) 2. – 0.4 V 1
2H+ + 2e- + O2 ¾® H2 O( l ); E° = + 1.23V
Fi

2+ +
R. E° ( Cu + Cu ¾® 2 Cu ) 3. – 0.04 V 2
Fe2 + + 2e- ¾® Fe( s ); E° = - 0.44 V
S. E° (Cr 3 + , Cr 2 + ) 4. – 0.83 V
DG° (in kJ) for the reaction is (2005, 1M)
Codes (a) – 76 (b) – 322
P Q R S (c) – 122 (d) – 176
(a) 4 1 2 3 20. Zn | Zn 2+ ( a = 0.1M ) || Fe2+ ( a = 0.01M) | Fe.
(b) 2 3 4 1 The emf of the above cell is 0.2905 V. Equilibrium constant
(c) 1 2 3 4 for the cell reaction is (2004, 1M)
(d) 3 4 1 2 (a) 100.32/ 0.059 (b) 100.32/ 0.0295
16. An aqueous solution of X is added slowly to an aqueous (c) 100.26/ 0.0295 (d) 100.32/ 0.295
solution of Y as shown in Column I. The variation in
21. The correct order of equivalent conductance at infinite
conductivity of these reactions is given in Column II. Match
dilution of LiCl, NaCl and KCl is (2001, 1M)
Column I with Column II and select the correct answer using
the codes given below the Columns. (2013 Adv.) (a) LiCl > NaCl > KCl (b) KCl > NaCl > LiCl
(c) NaCl > KCl > LiCl (d) LiCl > KCl > NaCl
Electrochemistry 153

22. For the electrochemical cell, ( M |M + )||( X - | X ), Passage II


E° ( M + / M ) = 0.44 V and E° ( X / X - ) = 0.33 V. The concentration of potassium ions inside a biological cell
From this data one can deduce that (2000, 1M) is at least twenty times higher than the outside. The resulting
(a) M + X ¾® M + + X - is the spontaneous reaction potential difference across the cell is important in several
(b) M + + X – ¾® M + X is the spontaneous reaction processes such as transmission of nerve impulses and
(c) Ecell = 0.77 V maintaining the ion balance. A simple model for such a
concentration cell involving a metal M is :
(d) Ecell = - 0.77 V
M ( s ) | M + ( aq; 0.05 molar) || M + ( aq; 1 molar) | M ( s )
23. The standard reduction potentials of Cu 2+ /Cu and
Cu 2+ / Cu + are 0.337 V and 0.153 V respectively. The For the above electrolytic cell the magnitude of the cell
potential | Ecell | = 70 mV. (2010)
standard electrode potential of Cu + /Cu half-cell is

w
(1997, 1M) 27. For the above cell
(a) 0.184 V (b) 0.827 V (c) 0. 521 V (d) 0.490 V (a) Ecell < 0; DG > 0 (b) Ecell > 0; DG < 0
(c) Ecell < 0; DG° > 0 (d) Ecell > 0; DG° < 0
Numerical Value Based Question 28. If the 0.05 molar solution of M + is replaced by a 0.0025

Flo
24. The surface of copper gets tarnished by the formation of molar M + solution, then the magnitude of the cell potential
copper oxide. N 2 gas was passed to prevent the oxide would be

ree
formation during heating of copper at 1250 K. However, the (a) 35 mV (b) 70 mV
N 2 gas contains 1 mole % of water vapour as impurity. The (c) 140 mV (d) 700 mV

F
water vapour oxidises copper as per the reaction given below
Passage III
2Cu( g )+ H 2O( g ) ¾® Cu 2O( s)+ H 2( g )
Redox reaction play a pivotal role in chemistry and biology.

or
ur
pH 2 is the minimum partial pressure of H 2 (in bar) needed to The values of standard redox potential ( E° ) of two half-cell
prevent the oxidation at 1250 K. The value of ln( pH 2 ) is reactions decide which way the reaction is expected to
…… .
f
proceed. A simple example is a Daniell cell in which zinc
ks
(Given : total pressure = 1 bar, R (universal gas constant) goes into solution and copper gets deposited. Given below
Yo
= 8 J K -1 mol -1 , ln(10 ) = 2.30 Cu ( s ) and Cu 2O ( s ) are are a set of half-cell reactions (acidic medium) along with
oo
their E °(V with respect to normal hydrogen electrode)
mutually immiscible.)
values. Using this data obtain the correct explanations to
eB

At 1250 K : 2Cu( s)+ 1 / 2 O 2 ( g ) ¾® Cu 2O( s); Questions 17–19. (2007, 4 ´ 3M = 12M)


DGs = - 78,000 J mol -1
I2 + 2e- ® 2I- E° = 0.54
1
H 2( g )+ O2 ( g ) ¾® H 2O ( g ); Cl 2 + 2e- ® 2Cl - E° = 1.36
r

2
ou
ad

3+ - 2+
Mn + e ® Mn E° = 1.50
DGs = - 1,78,000 J mol -1 ; G is the Gibbs energy (2018 Adv.)
Fe3+ + e- ® Fe2+ E° = 0.77
Y

O2 + 4H+ + 4e- ® 2H2O E° = 1.23


Passage Based Questions
29. Among the following, identify the correct statement.
nd
Re

Passage I (a) Chloride ion is oxidised by O2


The electrochemical cell shown below is a concentration (b) Fe2+ is oxidised by iodine
Fi

cell. M | M 2+ (saturated solution of a sparingly soluble salt, (c) Iodide ion is oxidised by chlorine
MX 2 )||M 2+ (0.001 mol dm -3 )|M . The emf of the cell (d) Mn 2+ is oxidised by chlorine
depends on the difference in concetration of M 2+ ions at the 30. While Fe3+ is stable, Mn 3+ is not stable in acid solution
two electrodes. The emf of the cell at 298K is 0.059 V. because
(2012) (a) O2 oxidises Mn 2+ to Mn 3+
25. The solubility product (K sp : mol 3 dm -9 ) of MX 2 at 298 (b) O2 oxidises both Mn 2+ to Mn 3+ and Fe2+ to Fe3+
based on the information available the given concentration (c) Fe3+ oxidises H2 O to O2
cell is (take 2.303 ´ R ´ 298/ F = 0.059 V) (d) Mn 3+ oxidises H2 O to O2
(a) 1 ´ 10-15 (b) 4 ´ 10-15
31. Sodium fusion extract, obtained from aniline, on treatment
(c) 1 ´ 10-12 (d) 4 ´ 10-12 with iron (II) sulphate and H2 SO4 in the presence of air gives
26. The value of DG (kJ mol -1 ) for the given cell is a Prussian blue precipitate. The blue colour is due to the
(take 1 F = 96500 C mol -1 ) formation of
(a) Fe4 [Fe (CN)6 ] 3 (b) Fe3 [Fe(CN)6 ] 2
(a) - 5.7 (b) 5.7 (c) 11.4 (d) –11.4
(c) Fe4 [Fe(CN) 6 ] 2 (d) Fe3 [Fe(CN)6 ] 3
154 Electrochemistry

Passage IV Ag+ + e- ¾® Ag; E° = 0.80V


2+ -
Tollen’s reagent is used for the detection of aldehydes. When Zn + 2e ¾® Zn; E° = -76V
a solution of AgNO3 is added to glucose with NH4 OH, then Also find how many moles of Ag will be formed?
gluconic acid is formed.
37. Find the equilibrium constant for the reaction
Ag + + e- ¾® Ag; E °red = 0.80 V
Cu 2+ + In 2 + r Cu + + In 3+
C6 H12 O6 + H2 O ¾® C6 H12 O7 + 2H+ + 2e- ;
Gluconic acid ° Given that E°Cu 2+ /Cu + = 0.15V,
Eoxi = - 0.05 V
E°In 2+ / In + = - 0.4V,
Ag(NH3 )+2 + e- ¾® Ag (s) + 2NH3; °
Ered = 0.337 V
E° = -0.42 V
RT F In 3+ / In + (2004, 4M)
[Use 2.303 ´ = 0.0592 and = 38.92 at 298 K]
F RT 38. (a) Will pH value of water be same at temperature 25°C and
(2006, 3 ´ 4M = 12M)

w
4°C? Justify in not more than 2 or 3 sentences.
32. 2Ag + + C6 H12 O6 + H2 O ¾® 2Ag ( s ) + C6 H12 O7 + 2H+ (b) Two students use same stock solution of ZnSO4 and a
Find ln K of this reaction. solution of CuSO4. The emf of one cell is 0.03 V higher
(a) 66.13 (b) 58.38 (c) 28.30 (d) 46.29 than the other. The concentration of CuSO4 in the cell with

Flo
33. When ammonia is added to the solution, pH is raised to 11. higher emf value is 0.5 M. Find out the concentration of
Which half-cell reaction is affected by pH and by how much? CuSO4 in the other cell. Given : 2.303 RT /F = 0.06V .

ree
° (2003, 2M)
(a) Eoxi will increase by a factor of 0.65 from Eoxi
°
(b) Eoxi will decrease by a factor of 0.65 from Eoxi 39. The standard potential of the following cell is 0.23 V at 15° C
°
(c) E red will increase by a factor of 0.65 from E red and 0.21 V at 35° C.

F
(d) E red will decrease by a factor of 0.65 from E °red Pt | H2 ( g ) | HCl( aq ) | AgCl ( s ) | Ag ( s )
34. Ammonia is always added in this reaction. Which of the (i) Write the cell reaction.

or
following must be incorrect?
ur (ii) Calculate DH ° and DS ° for the cell reaction by assuming
(a) NH3 combines with Ag + to form a complex

f
that these quantities remain unchanged in the range 15° C
(b) Ag(NH3 )2+ is a stronger oxidising reagent than Ag + to 35° C.
ks
(c) In the absence of NH3 silver salt of gluconic acid is (iii) Calculate the solubility of AgCl in water at 25° C.
Yo
formed
oo
Given, the standard reduction potential of the
(d) NH3 has affected the standard reduction potential of (Ag+ (aq)/Ag (s) is 0.80 V at 25° C.
glucose/gluconic acid electrode (2001, 10M)
eB

40. Find the solubility product of a saturated solution of


Subjective Questions Ag 2 CrO4 in water at 298 K, if the emf of the cell Ag | Ag +
35. We have taken a saturated solution of AgBr, K sp is (Saturated Ag 2 CrO4 solution. ) ||Ag + (0.1 M) | Ag is 0.164 V
r

12 ´ 10- 14 . If 10- 7 M of AgNO3 are added to 1 L of this


ou
ad

at 298 K. (1998, 6M)


solution, find conductivity (specific conductance) of this
solution in terms of 10- 7 Sm- 1 units. (2006, 6M) 41. Calculate the equilibrium constant for the reaction,
Y

° -3 -1
Given, 2
l(Ag + ) = 6 ´ 10 Sm mol , 2Fe3+ + 3I- r 2Fe2+ + I-3 . The standard reduction
l°(Br - ) = 8 ´ 10- 3 Sm2 mol -1 , potentials in acidic conditions are 0.77 V and 0.54 V
nd
Re

respectively for Fe3+ / Fe2+ and I-3 / I- couples. (1998, 3M)


l°(NO – ) = 7 ´ 10- 3 Sm2 mol -1 .
Fi

3 42. Calculate the equilibrium constant for the reaction


36. Calculate DGr° of the following reaction: Fe2+ + Ce4+ r Fe3+ + Ce3+
+ -
(a) Ag (aq) + Cl (aq) ¾® AgCl (s)
Given, E° ( Ce4+ /Ce3 + ) = 1.44 V, E° ( Fe3+ /Fe2 + ) =0.68 V
Given (1997, 2M)
DG ° (AgCl )
f
-109 kJ /mol 2+
43. The standard reduction potential for Cu /Cu is +0.34 V.
DG ° (Cl )-
f
-129kJ /mol Calculate the reduction potential at pH = 14 for the above
couple. K sp of Cu (OH)2 is 1.0 ´ 10-19 .
DG ° (Ag+ ) 77kJ /mol (1996, 3M)
f
44. An excess of liquid mercury is added to an acidified solution
Represent the above reaction in form of a cell. Calculate
of 1.0 ´ 10-3 M Fe3+ . It is found that 5 % of Fe3+ remains at
E° of the cell. Find log10 K sp of AgCl. (2005, 6M)
(b) 6.539 ´ 10-2 g of metallic Zn(u = 65.39) was added to
equilibrium at 25° C. Calculate E° (Hg 2+ / Hg ) assuming that
100 mL of saturated solution of AgCl. Calculate the only reaction that occurs is
[ Zn 2+ ] 2Hg + 2Fe3+ ¾® Hg 2+ 2 + 2Fe
2+
log10 . Given that
[ Ag+ ]2 Given, E° (Fe3+ / Fe2+ ) = 0.77 V (1995, 4M)
Electrochemistry 155

45. The standard reduction potential of the Ag + /Ag electrode at 50. Give reasons in one or two sentences.
-17
298 K is 0.799 V. Given that for AgI, K sp = 8.7 ´ 10 , “Anhydrous HCl is a bad conductor of electricity but
aqueous HCl is a good conductor.”
evaluate the potential of the Ag + / Ag electrode in a saturated (1985, 1M)

solution of AgI. Also calculate the standard reduction 51. Consider the cell,
potential of the I- / AgI/Ag electrode. (1994, 3M) Zn | Zn 2 + ( aq ) (1.0 M ) || Cu 2 + ( aq ) (1.0 M ) | Cu
46. Zinc granules are added in excess to a 500 mL of 1.0 M The standard reduction potentials are 0.350 V for
nickel nitrate solution at 25° C until the equilibrium is Cu 2 + ( aq ) + 2e- ¾® Cu
reached. If the standard reduction potential of Zn 2+ / Zn and and -0.763 V for Zn 2 + ( aq ) + 2e- ¾® Zn
Ni 2+ / Ni are – 0.75 V and – 0.24 V respectively. Find out the
(i) Write down the cell reaction.
concentration of Ni 2+ in solution at equilibrium. (1991, 2M) (ii) Calculate the emf of the cell.

w
47. The standard reduction potential of Cu 2+ / Cu and Ag + / Ag (iii) Is the cell reaction spontaneous or not? (1982, 2M)
electrodes are 0.337 and 0.799 V respectively. Construct a
galvanic cell using these electrodes so that its standard emf is Integer Type Questions

Flo
positive. For what concentration of Ag + will the emf of the 52. The conductance of a 0.0015 M aqueous solution of a weak
cell, at 25° C, be zero if the concentration of Cu 2+
is 0.01 M? monobasic acid was determined by using a conductivity cell

ree
(1990, 3M) consisting of platinised Pt electrodes. The distance between
the electrodes is 120 cm with an area of cross section of 1 cm
48. The standard reduction potential at 25° C of the reaction, 2
. The conductance of this solution was found to be 5 ´ 10-7

F
2H2 O + 2e- r H2 + 2OH- , is -0.8277 V. Calculate the
S. The pH of the solution is 4. The value of limiting molar
equilibrium constant for the reaction,
conductivity ( L°m ) of this weak monobasic acid in aqueous

or
2H2 O r H3 O+ + OH- at 25° C.
ur (1989, 3M)
solution is Z ´ 102 S cm -1 mol -1 . The value of Z is

f
49. The emf of a cell corresponding to the reaction. (2017 Adv.)
ks
Zn( s ) + 2H+ ( aq ) ¾® Zn 2+ (0.1M) + H2 , ( g ,1atm) 53. All the energy released from the reaction
X ® Y , Dr G ° = - 193 kJmol -1 is used for oxidising M + as
Yo
is 0.28 V at 25° C.
oo
Write the half-cell reactions and calculate the pH of the M + ® M 3 + + 2e- , E ° = - 0.25 V.
solution at the hydrogen electrode. Under standard conditions, the number of moles of M +
eB

E° ( Zn 2+ / Zn ) = -0.76 V E ° + =0
H /H2
oxidised when one mole of X is converted to Y is
(1986, 4M)
[F = 96500 C mol - ] (2015 Adv.)
r
ou
ad

Answers
Y

Topic 1 Topic 2
nd

1. (b) 2. (d) 3. (a) 4. (c) 1. (a) 2. (c) 3. (b) 4. (c)


Re

5. (b) 6. (b) 7. (c) 8. (a) 5. (d) 6. (a) 7. (a) 8. (b)


Fi

9. (d) 10. (b) 11. (d) 12. (b) 9. (b) 10. (c) 11. (c) 12. (a)
13. (c) 14. (a) 15. (c) 16. (a) 13. (c) 14. (a) 15. (d) 16. (a)
17. (a) 18. (b) 19. (d) 20. (c) 17. (d) 18. (d) 19. (b) 20. (b)
21. (d) 22. (c) 23. (c) 24. (a) 21. (b) 22. (b) 23. (c) 24. (–14.16)
25. (a, b) 26. (a,b,d) 27. (10) 25. (b) 26. (d) 27. (b) 28. (c)
28. (–11.62 JK mol -1) 29. (b) 30. (d) 29. (c) 30. (d) 31. (a) 32. (b)
-5
31. (d) 33. (8 ´ 10 M) 34. (0001 V) 35. (300 cm ) 2
33. (c) 34. (d) 35. (55) 37. (1010)
-12
36. (-245.11 kJ) 39. (1.4085 M) 40. (-0.037 V) 41. (0.154 M) 38. (0.05 M) 40. (2.45 ´ 10 ) 41. (5.89 ´ 10 7)
-4
42. (347.40 kJ) 44. (190.50 g) 45. (10 M) 46. (19.1 g) 42. (6.88 ´ 10 )
12
43. (- 0.222 V) 44. (0.7926 V) 46. (1.7 ´ 1017)
47. (265 Ah) 48. (125 s) 49. (0.80 A) 50. (1.172 M) 47. (1.57 ´ 10 -9) 48. (1.04 ´ 10 -14 ) 49. (8.6)
52. (6 ´ 10 2 S cm -1 mol -1) 53. (4 mol)
Hints & Solutions
Topic 1 Electrochemical Cells 5. Reducing power of an element
Key Idea Negative E ° means that redox couple is weaker 1
1. µ
oxidising agent than H+ /H2 couple. Positive E° means that Standard reduction potential
redox couple is a stronger oxidising agent than H+ / H2 Here, E ° values of the given metals are as,
M2 + / M
couple Metals Ni Zn Mg Ca
3+ - 2+
Given, Co + e ¾¾® Co ; E ° = + 1.81 V E°( V ) - 0.25 - 0.76 - 2.36 - 2.87
Pb4+ + 2e- ¾¾® Pb2+ ; E ° = + 1.67 V ¾¾¾¾¾¾¾¾¾¾¾¾¾¾®

w
Reducing power
4+ - 3+
Ce + e ¾¾® Ce ; E ° = + 1.61 V Thus, the correct order of increasing reducing power of the
given metal is,
Bi3+ + 3e- ¾¾® Bi; E ° = + 0.20 V
Ni < Zn < Mg < Ca.

Flo
Oxidising power of the species increases in the
order of Bi3 + < Ce4+ < Pb4+ < Co3+ . 6. Key Idea This question is based upon Faraday’s first law
Higher the emf value, stronger the oxidising power. The which states that “Mass of any substance deposited or

ree
maximum value of emf is possessed by Co3+ . Hence, it has liberated at any electrode is directly proportional to the
maximum oxidising power. Whereas Bi3+ possess the lowest quantity of electricity passed.”

F
emf value. Hence, it has minimum oxidising power. During charging:
2. A solution of Ni(NO3 )2 is electrolysed between platinum Pb + SO24 - ¾® PbSO4 + 2e-

or
ur
electrodes using 0.1 Faraday electricity. It means that 0.1 Þ 1 F º 1 g-equiv. of PbSO4
equivalent of Ni2+ will be discharged. 1 303
= mol of PbSO 4 Þ

f
g PbSO4
Electrolysis of Ni(NO3 )2 gives 2 2
Ni2+ + 2e- ¾® Ni (Atomic mass of Ni = 58.7)
ks
303
\ 0.05 F º ´ 0.05 g of PbSO4
Yo
Number of equivalents = Number of moles ´ number of electrons. 2
oo
. = Number of moles ´ 2
01 = 7.575 g of PbSO4
01
. 7. Given that, i = 100 amp. also, 27.66 g of diborane (B2 H6 )
\Number of moles of Ni = = 0.05
eB

2
Molecular mass of B2 H6 = 10.8 ´ 2 + 6 = 27.6
3. Fe2+ (aq) + Ag + (aq) ¾® Fe3 + (aq) + Ag(s) Given mass 27.66
Number of moles of B2 H6 in 27.66 g = = »1
º
\ Ecell = Eº - Eº = xV - E º Molar mass 27.6
r

… (i)
Ag + /Ag Fe3+ / Fe2 + Fe3+ / Fe2 +
ou

Now consider the equation


ad

Now, for two half-cells B2H6 + 3O2 ¾® B2O3 + 3H2O


(i) Fe2+ + 2e- ¾® Fe; EFe
º
= E1º = yV DG2º = - 2FE1º
Y

2+
/ Fe From the equation we can interpret that 3 moles of oxygen is
required to burn 1 mole (i.e. 27.6 g) B2 H6 completely.
(ii) Fe3 + + 3e- ¾® Fe ; EFe
º
3+ = E2º = zV DG2º = - 3FE2º
/ Fe Also consider the electrolysis reaction of water i.e.
nd
Re

So, Fe3 + + e– ¾® Fe2+ ; E º = E3º = ? H2O s 2H+ + O--


Fe3+ / Fe2 +
; DG3º = - 1 ´ FE3º +2e -
2H+ ¾¾® 2H ¾® H2 ­
Fi

Cathode
Again, DG3º = DG2º - DG1º 2 such
-- Anode
O ¾¾® O ¾¾® O2 ­
Þ - FE3º = - 3FE2º - (- 2FE1º ) -
-2e atoms

Þ - E3º = 2E1º - 3E2º Þ E3º = 3E2º - 2E1º From the above equation it can be easily interpreted that in
Þ E º 3+ 2 + = (3z - 2 y) V electrolysis of water for the production of 1 mole of oxygen
Fe / Fe from 1 mole of H2O at anode 4 moles electrons are required.
So, from equation (i) Likewise for the production of 3 moles of O2 12(3 ´ 4 ) moles of
º
Ecell = xV - (3z - 2 y) V = (x - 3z + 2 y) V electrons will be needed.
So, the total amount of charge required to produce 3 moles of
4. Higher the standard reduction potential (E ºMn+ / M ), better is
oxygen will be 12 ´ F or 12 ´ 96500
oxidising agent. Among the given, E °S2O 28- / SO 24- is highest, hence We know Q = it
S2O82- is the strongest oxidising agent. So, 12 ´ 96500 = 100 ´ t in seconds
12 ´ 96500
The decreasing order of oxidising agent among the given option or = tin hours = 3.2 hours
is as follows: 100 ´ 3600
S2O82- > Au 3+ > O2 > Br2
Electrochemistry 157

8. The substances which have lower reduction potentials are 15. One of the requirement for electrolyte used in salt-bridge is, both
° 3 + / Cr = - 0.74 V) is
stronger reducing agents. Therefore, Cr (ECr cation and anion must have comparable size so that they migrate
towards electrodes of opposite polarity at comparable speeds.
the strongest reducing agent among all the other given options.
16. Higher the value of reduction potential, stronger the oxidising
9. Oxidation at anode agent.
° = 0.00 V
H2 (g ) ¾® 2H+ (aq)+ 2e-; ESHE Q E° : Z > Y > X
Reduction at cathode Þ Y will oxidise X but not Z.
M 4+ (aq)+ 2e- ¾® M 2+ (aq); EM° 4+ / M 2+ = 0.151 V
17. Lower the value of E°, stronger the reducing agent.
Net: M 4+ (aq) + H2 (g ) ¾® M 2+ (aq) + 2H+ (aq); Reducing power:
[ M 2+ ] [ H+ ] 2 [ M 2+ ] Y (E ° = - 3.03 V) > Z (E ° = - 1.18 V) > X (E ° = 0.52 V).
K= ° = 0.151 V ) =
(Ecell
4+
[ M 4+ ]

w
[ M ] pH 2 18. Fe2+ + 2e- ¾® Fe ; E° = - 0.41V
0.059 -
° -
Ecell = Ecell log K Zn ¾® Zn 2+
+ 2e ; E° = + 0.76V
2 2+ 2+
0.059 [ M 2+ ] Þ Fe + Zn ¾® Zn + Fe ; E° = + 0.35V

Flo
0.092 = 0151. - log 4+
2 [M ] 19. In a lead storage battery, sulphuric acid is consumed as :
0.059 Pb + PbO2 + 2H2SO4 ¾® 2PbSO4 + 2H2O
0.059 = log10 x

ree
2
\ log10 = 2
x 20. In a galvanic cell, oxidation occur in the left hand electrode
chamber and reduction in right hand electrode chamber. In the
\ x=2

F
following cell.
10. Given, Q = 2F
Pt | H2 (g ) | HCl (l )||AgCl (s) | Ag(s)
Atomic mass of Cu = 63.5u

or
Valency of the metal Z = 2
ur The cell reactions are :
1
H (g ) ¾® H+ + e- At anode

f
We have, CuSO4 ¾® Cu 2+ + SO24- 2 2
- -
AgCl(s) + e ¾® Ag + Cl At cathode
ks
Cu 2+ + 2 e- ¾® Cu
Yo
1mol 2mol 1mol =63.5g 1
2F Net : H2 (g ) + AgCl (s) ¾® H+ + Ag(s) + Cl -
oo
2
Alternatively.
E 2 ´ 63.5 21. One gram equivalent of an electrolyte required 1.0 mole of
eB

W = ZQ = × 2F = 2E = = 63.5
F 2 electronic charge for discharging.
11. Higher the standard reduction potential, better is oxidising 22. In aqueous solution, only those ions who are less electropositive
°
agent. Among the given EMnO - is highest, hence MnO -4 is than hydrogen (E° > 0) would be deposited.
r

/ Mn 2+
4
ou

the strongest oxidising agent. Therefore, in the present case, only Ag, Hg and Cu would be
ad

deposited on passing electricity through aqueous solution of


12. 0.01 mol of H2 = 0.02 g equivalent
Y

these ions, Mg will not be deposited.


Þ Coulombs required = 0.02 ´ 96500 = 1930 C
Also, higher the value of E°, easier will be their reduction,
Þ Q = It = 1930 C
therefore, the sequence in which ions will be deposited on
nd
Re

1930
Þ t= = 19.3 ´ 104 s increasing voltage across the electrodes is :
10 ´ 10- 3
Fi

Ag, Hg, Cu.


13. In electrolytic cell electrolysis occur at the cost of electricity :
23. Faraday’s law of electrolysis is related to equivalent weight of
At cathode : M n+ + ne ¾® M
electrolytes as “the number of Faraday’s passed is equal to the
(electron gone in solution) number of gram equivalent of electrolytes discharged.”
At anode : X n- ¾® X + ne-
24. Lower the value of E°, stronger the reducing agent.
(electron supplied to anode)
Therefore, electron is moving from cathode to anode via internal 25. PLAN This problem is based on characteristics of salt-bridge.
circuit. Functions of salt-bridge are
14. MnO-4 cannot be used for oxidation of Fe2+ in HCl medium (i) It connects the two half-cells and completes the cell circuit.
because the following reaction is spontaneous : (ii) It keeps the solutions of two half-cells and complete the cell
circuit but does not participate chemically in the cell
MnO4- + Cl - ¾® Mn 2+ + Cl 2; E° = 1.51 - 1.40 = 0.11 V reaction.
In all other cases, the redox process between oxidising agent and (iii) It maintains the diffusion of ions from one electrode to
medium (HCl or H2SO4) are non-spontaneous, would not another electrode.
interfere oxidation of Fe2+ . (iv) A cell reaction may also occur in the absence of salt-bridge.
Sometimes, both the electrodes dip in the same electrolyte
158 Electrochemistry

solution and in such cases we do not require a salt-bridge.” Now by using the Nernst equation
So, option (c) is incorrect. RT [Product]
Ecell = E°cell - ln
(v) This prevent mixing of two electrolytic solutions hence, nF [Reactant]
option (d) is incorrect choice. After putting the values
Hence, correct choices are (a), (b). RT [ A n+ ]2
0 = Ecell
° - ln
26. Metals with E° value less than 0.96 V will be able to reduce NO-3 2nF [ B 2n+ ]
in aqueous solution.Therefore, metals V (E° = - 1.19 V), RT [ 2 ]2 RT
or E° = ln = ln 4 …(i)
Fe (E° = – 0.04 V), Hg (E° = 0.86 V) will all reduce NO-3 but Au 2 nF [1] 2nF
(E° = 1.40 V) cannot reduce NO-3 in aqueous solution. Further from the formula,
27. (10) Equation of cell reaction according to the cell notation DG ° = - nFE ° Þ DG ° = - 2nFE °
Now putting the value of E° from eq. (i)

w
given, is
RT
Reduction DG ° = - 2nF ´ ln 4 …(ii)
2nF
Mg(s) + Cu2+(aq) Mg2+(aq) + Cu(s) DG ° = - RT ln 4

Flo
Finally, using the formula
Oxidation D G ° = D H ° - TD S °
DG ° = 2DG ° - TDS ° (as DH ° = 2DG °, given)

ree
° = 2.70 V, T = 300 K
Given, Ecell
D G ° = TD S °
with [Mg 2+ (aq)]=1 M and [Cu 2+ (aq)]=1 M
DG ° - RT ln 4
or DS ° = =

F
and n= 2 T T
Further, Ecell = 2 . 67 V (from eq. (ii), DG ° = - RT ln 4)

or
with [Cu 2+(aq) ]=1M = - R ln 4 = - 8.3 ´ 2 ´ 0.7
ur (as all values given)
and [Mg 2+ (aq)]= xM

f
= - 1162
. J/K-mol
F
= 11500 KV -1
ks
and 500
R 29. Moles of NaCl electrolysed = 4 ´ = 2.0
Yo
1000
where F = Faraday constant, R = gas constant
oo
From the formula, Þ moles of Cl 2 produced = 1.0
2+ 2Cl - ¾® Cl 2 + 2e-
eB

RT [Mg (aq)]
° -
Ecell = Ecell ln 2+
nF [Cu (aq)] Hg
30. At cathode Na + + e- ¾¾® Na(Hg)
After putting the given values Amalgam
r

RT x Two moles of Na formed during electrolysis would produce two


2.67 = 2.70 -
ou

ln
ad

2F 1 moles of Na(Hg) amalgam.


R ´ 300 Þ mass of amalgam = 2 ´ (23 + 200) = 446 g
2.67 = 2.70 - ´ ln x
Y

or
2F 31. Two Faraday of electric charge would be required for
- R ´ 300
-0.03 = ´ ln x electrolysis of 2.0 moles of NaCl.
nd

2F
Re

Þ total coulombs = 2 ´ 96500 = 193000 C


0.03 ´ 2 F
or ln x = ´ ° . Also, left
32. Since, activities of all the ions are unity, Ecell = Ecell
Fi

300 R
0.03 ´ 2 ´ 11500 hand electrode is at lower reduction potential, it act as anode and
= = 2.30
300 E ° = E ° (Ce4+ , Ce3+ ) - E ° (Fe3+ , Fe2+ ) = 0.84
So, ln x = 2.30
i.e. electrons will flow from left to right hand electrode and
or x =10 (as given ln (10) = 2.30)
current from right hand electrode [Pt (2)] to left hand electrode
28. Given, [Pt(1)].
A (s)| A n+ (aq, 2 M ) || B 2n+ (aq, 1 M ) | B (s) [Fe3+ ][Ce3+ ]
Also, E = E ° - 0.0592 log
So, reactions at respective electrode will be [Fe2+ ][Ce4+ ]
Anode A (s)¾® A n+ + ne- ´ 2 As electrolysis proceeds, E will decrease and therefore, current.
Cathode B 2n+ + 2 ne- ¾® B (s) 2 ´ 10- 3 ´ 16 ´ 60
33. The number of Faraday’s passed =
Overall reaction 96500
2 A (s) + B 2n+ (aq) ¾® 2 A n+ (aq) + B (s) = 1.99 ´ 10- 5
Further, Þ number of gram equivalent of Cu 2+ deposited
DH ° = 2DG ° and Ecell = 0 is also given = 1.99 ´ 10- 5
Electrochemistry 159

1.99 1.5 9
Þ number of moles of Cu 2+ deposited = ´10- 5 » 10- 5 (ii) Number of gram equivalent of Cr = ´6=
2 52 52
Absorbance is directly proportional to [Cu 2+ ]. Therefore, 9
Þ Coulombs required for 1.5 g Cr = ´ 96500 = It
if ‘C’ be the initial molarity, 0.5 C will be the final molarity. 52
Þ 0.5 C ´ 0.25 = 10- 5 Þ C = 8 ´ 10- 5 M 9 ´ 96500
Þ t= s = 22.27 min
9.65 ´ 60 ´ 60 52 ´ 12.5
34. The number of Faraday’s passed = = 0.36 F
96500 39. At anode 2Cl - ¾® Cl 2 + 2e-
After electrolysis : [Ag+ ] = 1.36 M At cathode 2H2O + 2e- ¾® H2 + 2OH-
0.36 1000
[Cu 2+ ] = 1 – = 0.82 M 1 kg Cl 2 = equivalent of Cl 2 = 28.17 equivalent
2 35.5

w
E1 (before electrolysis) = E ° Þ Theoretical electricity requirement = 28.17 F
0.0592 [Ag+ ]2 Q Efficiency is only 62%
E2 (after electrolysis) = E ° - log \ Electricity requirement (experimental)
2 [Cu 2+ ]
28.17 ´ 100

Flo
0.0592 (1.36)2 = F = 45.44 F
Þ E1 - E2 = log = 0.01 V (decreased) 62
2 0.82
Þ 45.44 ´ 96500 = 25 t (in second)

ree
35. Coulombs passed = 8.46 ´ 8 ´ 60 ´ 60 = 243648 C Þ t = 48.72 h
Number of Faraday’s passed =
243648
= 2.52 Also, gram equivalent of HO- produced = 28.17

F
96500 28.17
63.5 Þ Molarity of HO- = = 1.4085 M
Weight of Cu plated = 2.52 ´ g = 80.01 g 20

or
2
80.01
ur 40. [Ag+ ] in left hand electrode chamber =
2.8 ´ 10- 10
Volume of Cu plated = = 7.62 cm 3 0.2

f
10.5 = 1.4 ´ 10- 9 M
7.62
ks
Þ Area plated out = = 3000 cm 2 3.3 ´ 10- 13
0.00254 [Ag+ ] in right hand electrode chamber =
Yo
0.001
oo
36. Given, FeO (s)/Fe (s) E° = – 0.87 V = 3.3 ´ 10- 10 M
and Ni 2O3 /NiO (s) E° = + 0.40 V
eB

[Ag+ ] anode
Electrode at lower reduction potential act as anode and that at emf = 0 – 0.0592 log
[Ag+ ] cathode
higher reduction potential act as cathode.
(i) Electrodes reaction : 1.4 ´ 10- 9
r

= – 0.0592 log = – 0.037 V


Fe(s) + 2OH- ¾® FeO (s) + H2O (l ) 3.3 ´ 10- 10
ou
ad

E° = + 0.87 V Therefore, the cell as written is non-spontaneous and its reverse


Y

Ni 2O3 (s) + H2O (l ) + 2e- ¾® 2NiO (s) + 2OH- E° = 0.40 V will be spontaneous with emf = 0.037 V.
Net : Fe (s) + Ni 2O3 (s) ¾® 2NiO (s) + FeO (s) 1.7 ´ 230
41. Faraday’s passed = = 4.052 ´ 10- 3 F
nd

E° = 1.27 V 96500
Re

(ii) Emf is independent of concentration of KOH. Faradays used for reduction of Zn 2+ = 4.052 ´ 10- 3 ´ 0.9
(iii) Maximum amount of energy that can be obtained = DG °
Fi

= 3.65 ´ 10- 3
Þ DG ° = - nE ° F = – 2 ´ 1.27 ´ 96500 J = – 245.11 kJ
Þ Meq. of Zn 2+ reduced = 3.65
i.e. 245.11 kJ is the maximum amount of obtainable energy.
Initial meq. of Zn 2+ = 300 ´ 0.16 ´ 2 = 96
1
37. (i) E = 0.78 – 0.0592 log 2 = 0.887 V Þ Meq. of Zn 2+ remaining = 96 - 3.65 = 92.35
8
1 92.35 1
(ii) E = 0.78 – 0.0592 log = – 0.0488 V Þ Molarity of Zn 2+ = ´ = 0.154 M
(10- 7 )2 2 300

38. Molar mass of Cr = 52 g 42. NO2 NH2


52
Equivalent mass of Cr = g
6
(i) Mass of Cr deposited on passing 24000 Coulombs
24000 52
= ´ g = 2.15 g
96500 6 Change in oxidation number at nitrogen = 4 - (- 2) = 6
123
Equivalent weight of nitrobenzene = g
6
160 Electrochemistry

12.3 ´ 6 Þ Total H2SO4 used up = 276.86 ´ 3.5 = 969.01 g


Þ gram equivalent of nitrobenzene = = 0.60
123 969.01
= mol = 9.888 mol
Þ Theoretical requirement = 0.60 ´ 96500 C = 57900 C 98
Þ Actual requirement of electricity = 2 ´ 57900 = 115800 C Q 1 mole of H2SO4 is associated with transfer of 1.0 mole of
Q V ´C = J electrons, total of 9.888 moles of electron transfer has occurred.
Þ Energy consumed = 115800 ´ 3 J = 347.40 kJ Coulomb produced = 9.888 ´ 96500
9.888 ´ 96500
43. If the salt is CuSO4 Ampere-hour = = 265 Ah
3600
During deposition of Cu at cathode, O2 (g ) will evolve at anode 0.005
0.4 ´ 2 48. Volume of Ag coating = 80 cm 2 ´ cm = 0.04 cm 3
gram-equivalent of Cu deposited = = 0.0126 10
63.5
Þ mass of Ag coating = 0.04 ´ 10.5 g = 0.42 g

w
Volume of O2 liberated at NTP at anode 0.42
= 0.0126 ´ 5600 mL = 70.56 mL Þ gram equivalent of Ag = = number of Faraday’s
108
In the next 7 min, H2 at cathode and O2 at anode would be 0.42
produced. Þ ´ 96500 C = 3 ´ t Þ t = 125 s

Flo
108
1.2 ´ 7 ´ 60
Faraday’s passed = = 5.22 ´ 10- 3 9.85
96500 49. Moles of Au deposited = = 0.05

ree
197
Þ Volume of H2 (at NTP) = 5.22 ´ 10- 3 ´ 11200 mL Þ gram equivalent of Au deposited = 0.05 ´ 3 = 0.15
= 58.46 mL Now, according to Faraday’s law of electrolysis, if same

F
Volume of O2 (at NTP) = 5.22 ´ 10- 3 ´ 5600 mL = 29.23 mL quantity of electricity is passed through different cells
Therefore, O2 (g) at NTP = 70.56 + 29.23 = 99.79 mL connected in series, same number of gram equivalents of

or
ur
H2 (g ) at NTP = 58.46 mL electrolytes are discharged at respective electrodes.
67.2 Þ gram equivalent of Cu deposited = 0.15

f
44. Total number of gram equivalent of H2 used = =6 63.5
11.2 Þ amount of Cu deposited = 0.15 ´ = 4.7625 g
ks
Þ 6 ´ 96500 = 15 ´ 60 ´ I Þ I = 643.33A 2
Also, Coulombs passed = 0.15 ´ 96500 = I ´ 5 ´ 60 ´ 60
Yo
63.5
Mass of Cu deposited = 6 ´ g = 190.50 g 0.15 ´ 96500
oo
2 Þ I= = 0.80 A
5 ´ 3600
45. Q Emf = 0.118 V > 0, it is galvanic cell and anode is negative
eB

electrode : 50. During electrolysis, Ni 2+ will be reduced at cathode and H2O


+ -6 -
At anode : H2 (g ) ¾® 2H (10 M ) + 2e will be oxidised at anode.
At cathode : 2H+ (x ) + 2e- ¾® H2 3.7 ´ 6 ´ 60 ´ 60
r

-6
Number of Faraday’s passed = = 0.828
ou

+ +
Cell reaction : H (x ) ¾® H (10 M) 96500
ad

10- 6 Þ 0.828 g equivalent of Ni 2+ will be deposited at cathode.


Emf = 0.118 V = 0 – 0.0592 log Þ x = 10- 4 M
Y

x Initial moles of Ni 2+ ion = 2 ´ 0.5 = 1.0


46. 100 W lamp will produce 100 Js - 1. 0.828
Moles of Ni 2+ ion remaining after electrolysis = 1.0 –
nd
Re

10 2
Þ 100 J = 110 ´ C Þ C = Coulombs
11 = 0.586
Fi

Therefore, total Coulomb passed in 10 h 2+ 0.586


Þ Molarity of Ni in final solution = = 1.172 M
10 0.50
= ´ 10 ´ 60 ´ 60 = 32727.27 C
11
Number of gram equivalent of Cd 2+ deposited
Topic 2 Conductivity of Electrolytic
32727.27 Solutions and their Measurement
= = 0.34
96500 and Nernst Equation
112.4 1. Electrical conductivity of the given aqueous solutions depends
Weight of Cd deposited = 0.34 ´ g = 19.1 g
2 on the degree of ionisation. Degree of ionisation is directly
47. For 1.0 L H2SO4 : proportional to the acidic strength. Electron withdrawing groups
(EWGs) increases the stability of the carboxylate ion by
39 dispersing the negative charge through resonance effect on the
Initial mass of H2SO4 = 1294 ´ = 504.66 g
100 conjugate while electron donating groups (EDGs) decreases the
20 stability of the carboxylate ion by intensifying the negative
Final mass of H2SO4 = 1139 ´ = 227.80 g
100 charge.
Þ H2SO4 consumed/litre = 504.66 – 227.80 = 276.86 g
Electrochemistry 161

O O So, in a conductivity cell


EWG C s EDG C s 1
O O lm µ
Acidity of carboxylic
M
Acidity of carboxylic
acids increases due acids decreases due i.e. molar conductivity increases with decrease in the
to the presence of electron to the presence of concentration (M) of electrolyte.
withdrawing groups (EWGs) electron donating groups
Thus, statement S 2 is correct.
The correct order of acidic strength and electrical conductivity
is as follows: 4. Key Idea Gibbs energy of the reaction is related to E°cell by
HCOOH > PhCOOH > CH3COOH the following formula
A C B DGº = - nFE °cell
2. NaCl and KCl are strong electrolytes. So, the study of their DG º = Gibbs energy of cell
molar conductances (l m ) can be experimentally verified by nF = amount of charge passed

w
Debye-Huckel Onsagar equation, E = EMF of a cell
Lcm = L0m - B C
Given reaction is,
Lcm = molar conductance at concentration.
Zn + Cu 2+ ¾® Zn 2+ + Cu

Flo
L0m = molar conductance at infinite dilution. i.e. C ® 0 Eºcell = 2.0 V
B = Debye-Huckel Onsagar constant. F = 96000 C

ree
For (both NaCl and KCl) a strong binary electrolyte like AB, the n=2
nature of the plot of Lm vs C will be To find the value of DG º (kJ mol), we use the formula
DG º = - nFE ºcell

F
DG º = -2 ´ 96000 ´ 2 = -384000 J/mol
-384000
In terms of kJ/mol, DG º = = -384 kJ/mol

or
Lm
ur 1000

f
5. According to Kohlrausch’s law, the molar conductivity of HA at
ÖC
infinite dilution is given as,
ks
Size of Na Å is being smaller than K Å and Na Å will remain in L°m (HA ) = [ L°m (H+ ) + L°m (Cl- ) ] + [ L°m (Na + ) + L°m ( A - ) ]
Yo
more hydrated state, i.e. larger sized in aqueous solution. As a
- [ L°m (Na + ) + L°m (Cl- ) ]
oo
result, ionic mobility as well as ionic conductance of Na Å
(or NaCl as ClÈ is common to NaCl and KCl) will be lower than = 425.9 + 100.5 - 126.4 = 400 S cm 2 mol - 1
eB

K Å (or KCl). Thus, the plot of Lm vs C for NaCl and KCl is as Also, molar conductivity at given concentration is given as,
follows : 1000 ´ k
Lm =
M
r

Given, k = conductivity Þ 5 ´10- 5 S cm - 1


ou
ad

KC
l
Na M = Molarity Þ 0.001 M
Lm Cl
Y

1000 ´ 5 ´ 10- 5 S cm- 1


\ Lm = = 50 S cm 2 mol - 1
ÖC 10- 3 M
nd
Re

Therefore, degree of dissociation (a), of HA is,


3. The explanation of statements (S 1 and S 2) are as follows :
L 50 S cm2mol- 1
In conductivity cell, conductivity (k ) is equal to the sum of ionic a= m = = 0.125
Fi

L°m 400 S cm2mol- 1


conductances (c), of an electrolytic solution present is unit
volume of the solution enclosed by two electrodes of unit area 6. According to Nernst equation,
(a ¹ 1) separated by a unit length (l = 1). 2.303 RT
l Ecell = E °cell - log Q
k = c´ Þ k = cwhen l = 1, a = 1 nF
a 2.303 RT
Given, = 0.059 V
So, with decrease in the concentration of electrolyte, number of F
ions in the given unit volume also decreases, i.e. k 0.059
[conductivity] also decreases. \ Ecell = E °cell - log Q
n
Thus, statement S 1 is wrong. S 2 : Molar conductivity (l m ) is At equilibrium, Ecell = 0
defined as the conducting power of all the ions present in a 0.059
solution containing 1 mole of an electrolyte. E °cell = log KC
n
1000
l m = k ´ VmL = k ´ For the given reaction, n = 2
M
Also, KC = 10 ´ 1015 [given]
where, VmL = volume in mL containing 1 mole of electrolyte
0.059
m = molar concentration (mol/L) \ E°cell = 15
log (10 ´ 10 ) = 0.472V » 0.473V
2
162 Electrochemistry

7. Cell E °cell (SRP) = E °C - E ° A E°cell free e transfer


Anode (A) Cathode (C)
Zn Ag
156
.
1. [Zn + 2Ag+ ® Zn2+ + 2Ag] . V for 2e-
0.80 - (-0.76) = + 156 + = + 0.78 V
2

Zn Fe 0.32
2. [Zn + Fe 2+ ® Zn2+ + Fe] - 0.44 - (- 0.76) = + 0.32 V for 2e- + = + 016
. V
2

Zn Au
216
.
3. [ 3Zn + 2Au 3+ ® 3Zn 2+ + 2Au] 1.40 - (-0.76) = + 2.4 V for 6e- + = + 0.36 V
6

w
Zn Fe
153
.
4. [ 3Zn + 2Fe3+ ® Zn 2+ + 2Fe2+ ] . V for 2e-
0.77 - (-0.76) = + 153 + = + 0.765 V
2

Flo
8. It is an electrochemical cell. The overall cell reaction can be æ 2.303 RT ö
Also, DG = - nEF = -2F ç11
. - ÷
written, as è 2F ø

ree
H 2 ( g ) + 2AgCl( s ) ¾® 2HCl( aq ) + 2Ag(s) = - 2.2F + 2.303RT
(1 bar) (10-6 M) = 2.303RT - 2. 2F

F
(i) According to Nernst equation,
2.303 ´ RT [HCl]2 [ Ag]2
11. Zinc metal is the most stable metal to cover iron surfaces. The
° °
Ecell = ( Ecathode - Eanode )- log process of coating the iron surface by zinc is called

or
n´ F
ur pH [ AgCl]2 2 galvanisation.

f
Here, (i) Ec° = EAgCl/
°
Ag, Cl -
°
= Ecathode 12. Standard electrode potential of reaction [ E° ] can be calculated
ks
as
° °
(ii)Eanode = E2H = 0.00 V o =E - E
Yo
+
/H
Ecell R P
2
oo
(Standard hydrogen electrode) where, ER = SRP of reactant , EP = SRP of product
-6 2
(10 ) ´ 1 2 If Ecell
o = +ve, then reaction is spontaneous otherwise
eB

Þ 0.92 = ( Ec° - 0 ) - 0.06 ´ log


1 ´ 12 non-spontaneous.
E ° = 1.51 V
= Ec° + 0.06 ´ 6 ´ 2 Mn 3+ ¾ ¾ ¾® Mn 2+
1 ¾¾

E ° = - 1.18 V
r

Þ Ec° = 0.92 - 0.72 = 0.20 V Mn 2+ ¾ ¾


2
¾ ¾ ¾® Mn
ou
ad

Note 10 -6 molal HCl is a very dilute solution. \ For Mn 2+ disproportionation,


So, 10 -6 m ~
- 10 -6 M
Y

E° = - 1. 51 V - 1.18 V = - 2. 69 V < 0
9. The relationship between standard electrode potential (E° ) and Thus, all reaction will not occur.
equilibrium constant (K ) of the cell reaction,
nd

13. According to Debye Huckel Onsager equation,


Re

Zn(s)+ Cu 2+(aq) c Zn 2+(aq)+ Cu(s) lC = l ¥ - B C


Fi

can be expressed as,


where, lC = limiting equivalent conductivity at concentration C
RT
E° = ln K Þ K = enFE °/ RT l ¥ = limiting equivalent conductivity at infinite dilution
nF
C = concentration
Given, n = 2, F = 96000 C mol -1
14. In order to solve the problem, calculate the value of cell constant
E° = 2 V, R = 8 JK -1mol -1
of the first solution and then use this value of cell constant to
T = 300 K calculate the value of k of second solution. Afterwards, finally
2 ´ 96000 ´ 2
calculate molar conductivity using value of k and m.
8 ´ 300
\ K =e = e160 For first solution,
10. The redox reaction is : Zn(s) + Cu 2+ ¾® Zn 2+ + Cu k = 1.4 Sm -1, R = 50 W, M = 0.2
2.303 RT 1 l
The Nernst equation is E = E ° - log10 Specific conductance (k ) = ´
2F R A
1 l
2.303RT 1.4 Sm -1 = ´
= 11
. - 50 A
2F
l
Þ = 50 ´ 1.4 m -1
A
Electrochemistry 163

For second solution,


l
= 50 ´ 1.4 m -1
R = 280, (S) Cr 3+ ¾® Cr 2+
A is obtained from n E°
k=
1
´ 1.4 ´ 50 =
1 Cr 3+ + 3e- ¾® Cr 3 - 0.74 V
280 4 Cr ¾® Cr 2+ + 2e- 2 + 0.91 V
Now, molar conductivity 3+ - 2+
k 1/ 4 1 Cr + e ¾® Cr 1 ?
lm = = =
1000 ´ m 1000 ´ 0 .5 2000
- 0.74 ´ 3 + 2 ´ 0.91
= 5 ´ 10-4 S m 2 mol -1 E3 ° = = -0.4 V
1
15. PLAN When different number of electrons are involved in a redox Thus, S = (2)
reaction P — (3), Q — (4), R — (1), S — (2)
° = DG °1 + DG °2
DGnet 16. The variation is conductivities in general can be seen as :

w
- n3FE °3 = - n1FE °1 - n2FE °2
n E ° + n2E °2 In burette
\ E °3 = 1 1 In flask base Curve
acid
n3
I. Strong Strong Conductance first

Flo
(P) E3° Fe3+ / Fe (HI) (NaOH) decreases due to
Net reaction Fe3+ ¾® Fe formation of H2O and then
increases due to addition

ree
is obtained from n E° of strong electrolyte.
Fe3+ + e- ¾® Fe2+ n1 = 1 E1°d = 0.77 V

F
Fe2+ + 2e- ¾® Fe n2 = 2 E2° = - 0.44 V

Conductance
Q Fe3+ + 3e- ¾® Fe n3 = 3 E3° = ?

or
ur
n E ° + n2E °2 0.77 + 2(-0.44) -0.11
E °3 = 1 1 = = = -0.04 V

f
n3 3 3
Volume of acid added
ks
Thus, P — (3)
Strong Weak
Yo
Net reaction II. Conductance increases
slightly as NH+4 (salt) is
oo
(CH 3COOH) (KOH)
4 H2O r 4 H+ + 4 OH-
hydrolysed forming HCl.
eB

is obtained from n E° After neutral point, it acid


2H2O ¾® O 2 + 4 H+ + 4 e- n1 = 4 -1.23 V increases rapidly due to
2H2O + O 2 + 4 e- ¾® 4OH- n2 = 4 + 0.40 V addition of strong
r

+ -
4H2O ¾® 4H + 4 e n3 = 4 ?
ou
ad

Conductance

n1E °1 + n2E °2
E °3 = = E °1 + E °2
Y

n3
= - 1.23 + 0.40 = -0.83 V
nd
Re

Thus, Q — (4)
(R) Cu 2+ + Cu ¾® 2Cu + Volume of acid added
For thus E° of Cu 2+ ¾® Cu +
Fi

is also required. n E° III. Weak Strong Conductivity decreases


Cu 2+ + 2e- ¾® Cu 2 0.34 V (CH 3COOH) (KOH) due to neutralisation of
conducting strong base
Cu ¾® Cu + + e- 1 - 0.52 V
and then remains constant
2+ - +
Cu + e ¾® Cu E°3 ? due to addition of weak
n E ° + n2E2 ° 2 ´ 0.34 + 1 ´ (-0.52) acid.
E3 ° = 1 1 = = 0.16 V
n3 1
Conductance

Also, n E°
Cu ¾® Cu + + e- n1 = 1, - 0.52 V
Cu 2+ + e- ¾® Cu + n2 = 1 0.10 V
2+ +
Cu + Cu ¾® 2Cu
Weak acid added
E° = - 0.52 + 0.16 = - 0.36 V to strong base
Thus, (R) — (1)
164 Electrochemistry

In burette 23. E° is an intensive property :


In flask base Curve
acid E° DG ° = - nE ° F
IV. Weak Weak Conductivity increases (i) Cu 2+ + 2e- ¾® Cu 0.337 V – 0.674 F
[(C 2H 5) 3N] (CH 3COOH) due to formation of ions (ii) Cu 2+ + e- ¾® Cu + 0.153 V – 0.153 F
and then remains constant
Subtracting (ii) from (i) gives :
due to addition of weak
Cu + + e- ¾® Cu DG ° = - 0.521 F = - nE ° F
base.
Þ E° = 0.521 V
V. KX AgNO3 Insoluble salt AgX is
Q n=1
formed, hence
conductance remains 24. Given
constant. It increases due 1
(i) 2Cu(s)+ O 2 (g ) ¾® Cu 2O(s); DG º= -78000 J mol -1
to addition of KX. 2

w
=-78 kJ mol -1
17. The half reactions are Fe( s ) ¾® Fe2+ ( aq ) + 2e– ´ 2 1
(ii) H2 (g ) + O 2 (g ) ¾® H2O(g ); DG º= -178000 J mol -1
+ – 2
O2 (g ) + 4H + 4 e ¾® 2H2O

Flo
2Fe(s) + O2 (g ) + 4H ¾® 2Fe2+ (aq) + 2H2O(l ) ;
+ = -178 kJ mol -1
0.059 (10–3 )2 So, net reaction is (By (i)-(ii))
= 1.57V

ree
E = E º– log 2Cu(s)+H2O(g) ¾® Cu 2O(s)+H2(g);
4 (10–3 )4 (0.1)
DG =100000 J/ mol or 105 J/mol = 100 kJ mol -1
18. As AgNO3 is added to solution, KCl will be displaced according

F
Now , for the above reaction
to following reaction.
é pH 2 ù
AgNO3 (aq) + KCl(aq) ¾
¾® AgCl(s) + KNO3 (aq) DG = DG º+ RT ln ê ú

or
ur
For every mole of KCl displaced from solution, one mole of êë pH 2O úû
KNO3 comes in solution resulting in almost constant and to prevent above reaction,
conductivity. As the end point is reached, added AgNO3 remain
f DG ³ 0
ks
in solution increasing ionic concentration, hence conductivity So,
Yo
increases. é pH 2 ù
oo
DG º + RT ln ê ú³0
19. The net reaction is êë pH 2O úû
1
eB

2H+ + O2 + Fe ¾® H2O + Fe2+ ; E° = 1.67 V After putting the values,


2
é pH 2 ù
2 ´ 1.67 ´ 96500 105 + 8 ´ 1250 ln ê ú³0
DG ° = - nE ° F = - kJ = – 322.31 kJ êë pH 2O úû
1000
r

é pH 2 ù
ou
ad

20. The cell reaction is : or 105 + 104 ln ê ú³0


Zn + Fe2+ r Zn 2+ + Fe ; Ecell = 0.2905 V êë pH 2O úû
Y

0.059 [Zn 2+ ] or 104 (ln pH 2 - ln pH 2O) ³ -105


Þ E = E° - log
2 [Fe2+ ] or ln pH 2 ³ -10 + ln pH 2O
nd
Re

0.059 0.1 or ln pH 2 ³ -10 + 2.3 log (0.01) (as pH 2O = 1%)


Þ E° = 0.2905 + log = 0.32 V
2 0.01 ³ -10 - 4.6
Fi

0.059 so ln pH 2 ³ -14.6
Also E° = log K
n
2E ° 0.32
25. For the given concentration cell, the cell reaction are
Þ log K = = M ¾® M 2 + at left hand electrode.
0.059 0.0295
Þ K = (10)0.32/ 0.0295 M 2+ ¾® M at right hand electrode
21. In LiCl, NaCl and KCl, anions are same. Þ M 2+ (RHS electrode) ¾® M 2+ (LHS electrode)
Cations have same charge but different size. Smaller cations are E° = 0
more heavily hydrated in aqueous solution giving larger Applying Nernst equation
hydrated radius and thus smaller ionic speeds and equivalent 0.059 [ M 2 + ] at LHS electrode
Ecell = 0.059 = 0 - log
conductance. 2 0.001
Þ Equivalent conductance : KCl > NaCl > LiCl [ M 2 + ] at LHS electrode
Þ log =-2
22. The spontaneous cell reaction is 0.001
X - + M + ¾® M + X ; E° = 0.11V Þ [ M 2+ ] at LHS electrode = 10-2 ´ 0.001 = 10-5 M
Electrochemistry 165

The solubility equilibrium for MX 2 is Solving for S gives : S = 3 ´ 10-7 M


MX 2 (s) r M 2+ (aq) + 2 X - (aq)
Þ [Br - ] = 3 ´ 10-7 M,
Solubility product, K sp = ][ M 2+ ][ X - ]2
[Ag+ ] = 4 ´ 10-7 M,
= 10-5 ´ (2 ´ 10-5 )2 = 4 ´ 10-15
[NO-3 ] = 10- 7 M
[Q In saturated solution of MX 2 , [ X - ] = 2 [ M 2+ ]]
Þ k (sp. conductance) = k Br - + k Ag + + k NO -
2 ´ 0.059 ´ 96500 3
26. DG = - nEF = - kJ = - 11.4 kJ = [8 ´ 10-3 ´ 3 ´ 10-7 + 6 ´ 10-3 ´ 4 ´ 10-7
1000
27. M (s) + M + (aq, 1 M ) ¾® M + (aq, 0.05 M ) + M (s) + 7 ´ 10-3 ´ 10-7 ] 1000

Ecell = 0 -
2.303 RT
log
0.05
>0 = 24 ´ 10-7 + 24 ´ 10-7 + 7 ´ 10-7
F 1
= 55 ´ 10-7 S m -1
Hence, | Ecell | = Ecell = 0.70 V and DG < 0 for spontaneity of

w
reaction. = 55 (in terms of 10- 7 S m -1 )
0.0538
28. Ecell = E ° - log 0.0025 = 0.139 V » 140 mV 36. (a) DG ° = S DG °f (products) - S DG °f (reactants)
1

Flo
= – 109 – (– 129 + 77) kJ = – 57 kJ
° >0
29. For spontaneous redox reaction : Ecell
Cell : Ag | AgCl, Cl - || Ag+ | Ag
For 2I- + Cl 2 ¾® 2Cl - + I2

ree
E° = 1.36 – 0.54 = 0.82 V > 0 For K sp ; reaction is AgCl (s) r Ag+ + Cl -
i.e. Cl 2 will spontaneously oxidise I- . DG ° = + 57 kJ

F
° < 0, they are non-spontaneous.
In other cases Ecell Þ DG ° = - RT ln K sp
DG ° 57 ´ 1000
Þ log K sp = - =- = - 10

or
30. For the reaction :
(i) 4Fe3+ 2+
ur +
+ 2H2O ¾® 4Fe + 4H + O2; E° = - 0.46 V
2.3 RT 2.3 ´ 8.314 ´ 298
E° of Ag+ + Cl - r AgCl

f
Now,
(ii) 4Mn 3+ + 2H2O ® 4Mn 2+ + 4H+ + O2;E° = + 0.27 V
DG ° 57000
ks
As evidenced above, reaction (i) is non-spontaneous, therefore, E° = - = = 0.59 V
Fe3+ is stable in acid solution. However, reaction (ii) is nF 96500
Yo
oo
spontaneous Mn 3+ oxidises H2O to O2 and itself reduced to (b) The cell reaction is :
Mn 2+ in acidic medium. Zn + 2Ag+ r Zn 2+ + 2Ag; E ° = 1.56 V
eB

31. Sodium fusion extract from aniline produces NaCN which 0.059 [Zn 2+ ]
2+ 4- Þ 0 = E° - log
reacts with Fe to form [Fe(CN)6 ] . The complex ion then 2 [Ag+ ] 2
reacts with Fe3+ to give blue precipitate of prussian blue. [Zn 2+ ] 2E ° 2 ´ 1.56
Þ = = = 52.88
r

Fe3+ + [Fe(CN)6 ]4- r Fe4[Fe(CN)6 ]3 log


[Ag+ ] 2 0.059 0.059
ou
ad

Prussian blue
6.539 ´ 10- 2
32. E° for 2Ag+ + C6H12O6 + H2O r 2Ag(s) Moles of Zn added = = 10- 3
Y

65.39
+ C6H12O7 + 2H+ is 0.75 V
Þ Moles of Ag formed = 2 ´ 10- 3.
0.0592 2E °
Also E ° = log K Þ log K = = 25.33
nd
Re

2 0.0592 37. Given,


Þ ln K = 2.303 log K = 58.35
In 2+ + e- ¾® In + E° = – 0.40 …(i)
Fi

+
33. On increasing concentration of NH3 , the concentration of H ion
Þ DG ° = 0.40 F
decreases, therefore,
In 3+ + 2e- ¾® In + E° = – 0.42 …(ii)
° - 0.0592 log [ H+ ]2 = 0 - 0.0592 ´ 2 log 10- 11
Ered = Ered
2 2 Þ DG ° = 0.84 F
= 0.65 V Subtracting (i) from (ii)
i.e. Ered increases by 0.65 V. In 3+ + e ¾® In 2+ ; DG ° = 0.44 F = – E ° F
34. NH3 has no effect on the E° of glucose/gluconic acid electrode. Þ E° = - 0.44 V
Now, for : Cu 2+ + In 2+ ¾® Cu + + In 3+
35. The solubility of AgBr in 10- 7 M AgNO3 solution is determined as
E ° = E ° (Cu 2+ / Cu + ) - E ° (In 3+ / In 2+ )
AgBr r Ag+ + Br -
S + 10- 7 S = 0.15 – (– 0.44) = 0.59 V
AgNO3 ¾® Ag +
+ NO-3 Also E° = 0.0590 log K
S + 10- 7 10-7 E°
Þ log K = = 10 Þ K = 1010
K sp = 14 ´ 10-14 = S (S + 10-7 ) 0.059
166 Electrochemistry

38. (a) pH = – log [H+ ] In saturated Ag2CrO4 solution present in anode chamber :
+ Ag2CrO4 (s) r 2Ag+ + CrO42-
In pure water, [H ] depends on value of K w which is
1.7 ´ 10- 4 M 1.7 ´ 10- 4 M
K w = [H+ ][OH- ] 2
Q K w is a function of temperature, [H+ ] will change K sp = [Ag+ ]2 [CrO24- ]
with temperature. æ 1.7 ö
(b) Let the emf of first cell be X volt. = (1.7 ´10- 4 )2 ç ´ 10- 4 ÷
è 2 ø
Þ emf of 2nd cell = (X + 0.03) volt = 2.45 ´ 10- 12
[Cu 2+ ] in 2nd cell = 0.50 M
[Cu 2+ ] in 1st cell = ?
41. For 2 Fe3+ + 3I- r 2Fe2+ + I-3
E ° = E ° (Fe3+ / Fe2+ ) - E ° (I-3 / I- )
2.303 RT [Zn 2+ ]
E1 = E1° - log = 0.77 – 0.54 = 0.23 V

w
2F [Cu 2+ ] 0.0592
Q E° = log K (n = 2)
2.303 RT [Zn 2+ ] 2
E2 = E1° - log 2E ° 2 ´ 0.23
2F [Cu 2+ ]2 log K = = = 7.77

Flo
0.0592 0.0592
2.303 RT é [Cu 2+ ]2 ù Þ K = 5.89 ´ 10 7
Þ E2 - E1 = ê log ú
2F ë [Cu 2+ ]1 û

ree
42. Fe2+ + Ce4+ r Fe3+ + Ce3+
0.50
Þ 0.03 = 0.03 log E° = E° (Ce4+ / Ce3+ ) – E° (Fe3+ / Fe2+ )
[Cu 2+ ]1

F
= 1.44 – 0.68 = 0.76 V
0.50 E° = 0.0592 log K
Þ = 10 Þ [Cu 2+ ]1 = 0.05 M Q
[Cu 2+ ]1 E°

or
0.76
ur Þ log K = =
0.0592 0.0592
= 12.83
1
H2 ¾® H+ + e- ; E° = 0

f
39. At anode Þ K = 6.88 ´ 1012
2
ks
- -
At cathode AgCl (s) + e ¾® Ag + Cl ; E° = ? 43. pH = 14
Yo
1
(i) Cell reaction : H2 + AgCl (s) ¾® Ag + H+ + Cl - Þ
oo
pOH = 0
2
Þ [OH- ] = 1.0 M
(ii) DG ° = - nE ° F = DH ° - T DS °
eB

At 15°C : – 0.23 ´ 96500 = DH ° - 288 DS ° …(i) K sp = 10- 19 = [Cu 2+ ][OH- ]2


At 35°C : – 0.21 ´ 96500 = DH ° - 308 DS ° …(ii) 10- 19
Þ 96500 (0.23 - 0.21) = - 20 DS ° Þ [Cu 2+ ] = = 10- 19
r

[OH- ]2
ou

96500 ´ 0.02
ad

Þ DS ° = - = - 96.5 J For reaction : Cu 2+ + 2e- ¾® Cu; E° = 0.34 V


20
Y

Substituting value of DS ° in (i) E = E° -


0.0592
log
1
DH ° = 288 ´ (- 96.5) - 0.23 ´ 96500 = – 49.987 kJ 2 [Cu 2+ ]
nd

(iii) At 25°C 0.0592


Re

= 0.34 – log 1019 = – 0.222 V


- E ° ´ 96500 = - 49987 - 298 (- 96.5) 2
Fi

Þ E° = 0.22 V
44. For reaction,
Þ AgCl (s) + e ¾® Ag + Cl - ; E° = 0.22 V
-
2Hg + 2Fe3+ r Hg2+ 2+
2 + 2Fe
+ -
Ag ¾® Ag + e ; E° = – 0.80 V Initial : 10 - 3 M 0 0
Adding : AgCl (s) ¾® Ag + Cl ; + -
E° = – 0.58 V Equilibrium : 5 ´ 10 - 5 4.75 ´ 10 - 4 9.5 ´ 10 - 4

Þ E° = 0.0592 log K sp [Fe2+ ]2 [Hg2+


2 ]
K =
- 0.58 [Fe ]3+ 2
Þ log K sp = = - 9.79
0.0592 (9.5 ´ 10- 4 )2 (4.75 ´ 10- 4 )
Þ K sp = 1.6 ´ 10-10 = = 0.17
(5 ´ 10- 5 )2
0.0592 [Ag + ]anode 0.0592
40. E = 0 - log Q E° = log K = - 0.0226 V
1 [Ag + ]cathode 2
[Ag+ ]anode = E ° (Fe3+ / Fe2+ ) - E ° (Hg22+ / Hg)
Þ 0.164 = - 0.0592 log
0.10 Þ E° (Hg2+
2 / Hg) = 0.77 + 0.0226 = 0.7926 V
Þ [Ag+ ]anode = 1.7 ´ 10- 4 M
Electrochemistry 167

45. In a saturated AgI solution; 0.0592 [Zn 2+ ]


E = E° - log
+
[Ag ] = 8.7 ´ 10 - 17
M 2 [H+ ]2
= 9.32 ´ 10-9
M 2 (E - E ° ) 1
Þ = - log [Zn 2+ ] - 2 log +
1 0.0592 [H ]
Þ EAg +/ Ag = E ° - 0.0592 log
[Ag+ ] Þ - 16.2 = – log (0.1) – 2 pH
1 1 + 16.2
= 0.799 – 0.0592 log Þ pH = = 8.6
9.32 ´ 10- 9 2
= 0.324 V 50. For conductivity, the charge carriers are required. In
anhydrous state, HCl is not ionised and no charge carrier
Also, for AgI r Ag+ + I- ; E° = 0.0592 log K sp
ions are available, hence bad conductor. However, in
= – 0.95 V

w
aqueous solution, HCl is fully ionised producing H+ and Cl -
Ag ¾® Ag+ + e- ; E° = – 0.799 V and conducts electricity.
AgI + e- ¾® Ag + I- E° = x
51. (i) The cell reaction is

Flo
-
Adding : +
AgI ¾® Ag + I ; E° = – 0.95 V Zn + Cu 2+ ¾® Zn 2+ + Cu
= x – 0.799 ° = Ecathode
(ii) Ecell ° ° = 0.350 – (– 0.763) = 1.113 V
- Eanode
Þ x = - 0.151 V

ree
Q Both Zn 2+ and Cu 2+ are at unit concentrations,
46. The redox reaction is E = E ° = 1.113 V
Zn + Ni 2+ r Zn 2+ + Ni E° = + 0.51 V Ecell = 1.113 V > 0

F
(iii) Q
0.0592 DG = - nEF < 0
Þ E° = log K Therefore, the cell reaction is spontaneous.
2

or
Þ log K =
0.51 ´ 2
= 17.23
ur 52. pH = Ca = 10-4

f
0.0592
17 10-4
Þ K = 1.7 ´ 10 Þ a=
ks
0.0015
Such a high value of equilibrium constant indicates that the æ Aö
Yo
reaction is almost complete. Therefore, concentration of Also, conductance (G ) = k ç ÷
oo
è lø
Zn 2+ in solution will be equal to initial concentration of Ni 2+ æ lö 120
Þ k = G ç ÷ = 5 ´ 10-7 ´ = 6 ´ 10-5
eB

ion, i.e. 1.0 M.


è Aø 1
47. The galvanic cell is : Cu | Cu 2+ || Ag + | Ag k ´ 1000
Þ Lc =
Cell reaction : Cu + 2Ag+ r Cu 2+ + 2Ag; E° = 0.462 V C
r

0.0592 (0.01) 6 ´ 10-5 ´ 1000


ou

E = 0 = 0.462 - =
ad

log
2 [Ag+ ]2 0.0015
[Ag+ ] = 1.57 ´ 10- 9 M L ´ 10-5 ´ 1000 0.0015
Y

c
Þ 6
Þ L¥ = = ´
1 a 0.0015 10-4
48. H2 O + e- r H2 + HO- ; E° = – 0.8277 V = 600 = 6 ´ 10 S cm mol -1
2 -1
nd

2
Re

1
H2 + H2O r H3O+ + e- ; E° = 0 V 53. Energy obtained as one mole X is converted into Y is 193 kJ.
2
Fi

2H2O r H3O+ + HO- E° = – 0.8277 V 3s 3p 3d


H(1s1) Second excited
state of H-atom
E° = 0.0592 log K Ground state
- 0.8277 All are degenerate
Þ log K = = - 13.98 degeneracy = 9
0.0592
Þ K = 1.04 ´ 10- 14. Energy consumed in converting one mole of M + to M 3+
96500
= - nE ° F = 2 ´ 96500 ´ 0.25 J = J
49. At anode Zn ¾® Zn 2+ + 2e- E° = 0.76 V 2
At cathode 2H+ + 2e- ¾® H2 (g ) E° = 0.00 V Þ 193 ´ 103 = n ç
æ 96500 ö
÷ Þ n = 4 mol
Þ For Zn + 2H+ ¾® Zn 2+ + H2 (g ) E° = 0.76 V è 2 ø

Download Chapter Test


http://tinyurl.com/y6pveqhz or
11
Chemical Kinetics

w
Objective Questions I (Only one correct option) 5. Consider the given plot of enthalpy of the following reaction
between A and B. A + B ¾® C+D

Flo
1. NO2 required for a reaction is produced by the
decomposition of N2 O5 in CCl 4 as per the equation, Identify the incorrect statement. (2019 Main, 9 April II)

2N2 O5 ( g ) ¾® 4NO2 ( g ) + O2 ( g )

ree
20
The initial concentration of N2 O5 is 3.00 mol L-1 and it is
Enthalpy 15
2.75 mol L -1 after 30 minutes. The rate of formation of NO2

F
(kJ mol–1)10
D
is (2019 Main, 12 April II)
-3 -1 -1 -2 -1 -1 5
´ 10 ´ 10

or
(a) 4167
. mol L min (b) 1667
. mol L min
(c) 8.333 ´ 10 -3
mol L -1
min -1
ur
(d) 2.083 ´ 10 -3
mol L -1
min -1
A+B
C

f
Reaction
coordinate
2. In the following reaction; xA ® yB
ks
é d[ A ]ù é d[B ]ù (a) D is kinetically stable product.
Yo
log 10 ê- ú = log 10 ê ú + 0.3010 (b) Formation of A and B from C has highest enthalpy of activation.
oo
ë dt û ë dt û
(c) C is the thermodynamically stable product.
A and B respectively can be (2019 Main, 12 April I)
Activation enthalpy to form C is 5 kJ mol -1 less than that to
eB

(d)
(a) n-butane and iso-butane (b) C2H2 and C6 H6
form D.
(c) C2H4 and C4 H8 (d) N2O4 and NO2
6. The given plots represent the variation of the concentration
3. For the reaction of H 2 with I2 , the rate constant is
of a reaction R with time for two different reactions (i) and
r

2.5 ´ 10- 4 dm3 mol- 1 s- 1 at 327ºC and 10. dm 3 mol- 1 s- 1


ou

(ii). The respective orders of the reactions are


ad

at 527ºC. The activation energy for the reaction, in (2019 Main, 9 April I)
Y

kJ mol- 1 is (R = 8.314 JK - 1 mol- 1 ) (2019 Main, 10 April II) (i) (ii)


(a) 59 (b) 72 (c) 150 (d) 166
In [R] [R]
nd

4. A bacterial infection in an internal wound grows as


Re

N ¢ ( t ) = N0 exp ( t ), where the time t is in hours. A dose of


Fi

antibiotic, taken orally, needs 1 hour to reach the wound. time time
Once it reaches there, the bacterial population goes down as
dN N (a) 1, 1 (b) 0, 2
= - 5N 2 . What will be the plot of 0 vs t after 1 hour ? (c) 0, 1 (d) 1, 0
dt N
(2019 Main, 10 April I) k1 k2
7. For a reaction scheme, A ¾® B ¾® C , if the rate of
formation of B is set to be zero then the concentration of B is
N0 N given by (2019 Main, 8 April II)
(a) (b) 0
N N æk ö
(a) k1k2[ A ] (b) çç 1 ÷÷[ A ]
t(h) t(h) è k2 ø
(c) (k1 - k2 )[ A ] (d) (k1 + k2 )[ A ]
N0 N0 8. For the reaction, 2 A + B ® C , the values of initial rate at
(c) (d)
N N different reactant concentrations are given in the table below.
The rate law for the reaction is (2019 Main, 8 April I)
t(h) t(h)
Chemical Kinetics 169

[A](mol L-1 ) [B](mol L-1 ) Initial rate Choose the correct option.
(mol L-1s-1 ) (a) Both I and II are wrong
(b) Both I and II are correct
0.05 0.05 0.045
(c) I is wrong but II is right
0.10 0.05 0.090 (d) I is right but II is wrong
0.20 0.10 0.72
15 For the reaction, 2A + B ¾® products
(a) rate = k [A ][B ]2 (b) rate = k [A ]2[B ]2
When concentration of both (A and B) becomes double, then
(c) rate = k [A ][B ] (d) rate = k [A ]2[B ]
rate of reaction increases from 0.3 mol L -1 s -1 to
9. For a reaction, consider the plot of ln k versus 1/ T given in 2.4 mol L -1 s -1 .
the figure. If the rate constant of this reaction at 400 K is When concentration of only A is doubled, the rate of reaction
10- 5 s - 1 , then the rate constant at 500 K is increases from 0.3 mol L -1 s -1 to 0.6 mol L -1 s -1 .

w
(2019 Main, 12 Jan II) Which of the following is true? (2019 Main, 9 Jan II)
(a) The whole reaction is of 4th order
(b) The order of reaction w.r.t. B is one

Flo
Slope = –4606 (c) The order of reaction w.r.t. B is 2
ln k
(d) The order of reaction w.r.t. A is 2
16 The following results were obtained during kinetic studies of

ree
1/ T the reaction; (2019 Main, 9 Jan I)

(a) 4 ´ 10 -4 -1 -6 -1 2A + B ¾® Products

F
s (b) 10 s
-4 -1
(c) 10 s (d) 2 ´ 10- 4 s- 1 Initial rate of
[A] [B]
Experiment reaction
10 Decomposition of X exhibits a rate constant of 0.05 mg/year. (in mol L-1 ) (in mol L-1)

or
ur
How many years are required for the decomposition of 5 mg
(in mol L-1 min -1)
I. 0.10 0.20 6.93 ´ 10-3

f
of X into 2.5 mg? (2019 Main, 12 Jan I)
(a) 20 (b) 25 (c) 40 (d) 50 II. 0.10 0.25 6.93 ´ 10-3
ks
11. The reaction, 2 X ® B is a zeroth order reaction. If the initial III. 0.20 0.30 1386
. ´ 10-2
Yo
oo
concentration of X is 0.2 M, the half-life is 6 h. When the
The time (in minutes) required to consume half of A is
initial concentration of X is 0.5 M, the time required to reach
(a) 5 (b) 10
eB

its final concentration of 0.2 M will be (2019 Main, 11 Jan II) (c) 100 (d) 1
(a) 7.2 h (b) 18.0 h (c) 12.0 h (d) 9.0 h
17. Which of the following lines correctly show the temperature
12. If a reaction follows the Arrhenius equation, the plot lnk vs dependence of equilibrium constant, K , for an exothermic
r

1/(RT) gives straight line with a gradient (- y) unit. reaction?


ou

(2018 Main)
ad

The energy required to activate the reactant is In K A


(2019 Main, 11 Jan I)
B
Y

y 1
(a) unit (b) - y unit (c) yR unit (d) y unit (0, 0) T(K)
R
××
nd

××
13. For an elementary chemical reaction, ×× C
Re

××
××
k1 ××
D
A2 = 2A , the expression for d[dtA ] is
Fi

k -1 (a) A and B (b) B and C


(2019 Main, 10 Shift II) (c) C and D (d) A and D
(a) 2k1[ A2 ] - k-1[ A ]2 (b) k1[ A2 ] - k-1 [ A ]2 18. At 518°C, the rate of decomposition of a sample of gaseous
(c) 2k1[ A2 ] - 2k-1 [ A ]2 (d) k1[ A2 ] + k-1[ A ]2 acetaldehyde, initially at a pressure of 363 Torr, was 1.00
14. Consider the given plots for a reaction obeying Arrhenius Torr s -1 when 5% had reacted and 0.5 Torr s -1 when 33%
equation (0°C < T < 300°C) : ( k and E a are rate constant and had reacted. The order of the reaction is : (2018 Main)
activation energy, respectively) (2019 Main, 10 Jan I)
(a) 2 (b) 3
(c) 1 (d) 0
19. Two reactions R1 and R2 have identical pre- exponential
k k
factors. Activation energy of R1 exceeds that of R2 by 10 kJ
mol- 1 . If k1 and k2 are rate constants for reactions R1 and R2 ,
æk ö
respectively at 300 K, then ln çç 2 ÷÷ is equal to
Ea T(°C)
( R = 8.314 J mol - 1K - 1 ) è k1 ø (2017 Main)
I II
(a) 8 (b) 12 (c) 6 (d) 4
170 Chemical Kinetics

20. Decomposition of H2 O2 follows a first order reaction. In 50 26. Plots showing the variation of the rate constant ( k ) with
min, the concentration of H2 O2 decreases from 0.5 to 0.125 M temperature (T ) are given below. The plot that follows
in one such decomposition. When the concentration of H2 O2 Arrhenius equation is (2010)
reaches 0.05 M, the rate of formation of O2 will be (2016 Main)
(a) 6.93 ´ 10-4 mol min -1 (b) 2.66 L min -1 at STP
k k
(c) 1.34 ´ 10-2 mol min -1 (d) 6.93 ´ 10-2 mol min -1 (a) (b)
21. Higher order (>3) reactions are rare due to (2015 Main)
(a) low probability of simultaneous collision of all the reacting
T T
species
(b) increase in entropy and activation energy as more molecules are
involved k k
(c) (d)

w
(c) shifting of equilibrium towards reactants due to elastic collisions
(d) loss of active species on collision
22. For the elementary reaction, M ¾® N , the rate of T T

Flo
disappearance of M increases by a factor of 8 upon doubling the
concentration of M . The order of the reaction with respect to M 27. For a first order reaction, A ® P, the temperature (T )
is (2014 Adv.) dependent rate constant ( k ) was found to follow the

ree
(a) 4 (b) 3 equation :
(c) 2 (d) 1 2000
log k = + 6.0
23. For the non-stoichiometric reaction, 2 A + B ® C + D, the

F
T
following kinetic data were obtained in three separate the pre-exponential factor A and the activation energy Ea ,
experiments, all at 298 K. (2014 Main) respectively, are

or
ur (a) 1.0 ´ 106 s- 1 and 9.2 kJ mol - 1
(2009)

Initial Initial Initial rate of

f
(b) 6.0 s- 1 and 16.6 kJ mol - 1
concentration concentration formation of C
(mol L -1s -1)
ks
[A] [B]
(c) 1.0 ´ 106 s- 1 and 16.6 kJ mol - 1
1. 2 ´ 10-3
Yo
(i) 0.1 M 0.1 M
(d) 1.0 ´ 106 s- 1 and 38.3 kJ mol - 1
oo
(ii) 0.1 M 0.2 M 1. 2 ´ 10-3
28. Under the same reaction conditions, initial concentration
eB

(iii) 0.2 M 0.1 M 2. 4 ´ 10-3


of 1.386 mol dm -3 of a substance becomes half in 40 s and
The rate law for the formation of C is 20 s through first order and zero order kinetics
dC dC
= k[ A ] 2[ B ] æk ö
r

(a) = k[ A ][ B ] (b) respectively. Ratio çç 1 ÷÷ of the rate constants for first


ou

dt dt
è k0 ø
ad

dC dC
(c) = k[ A ][ B ] 2 (d) = k[ A ] order ( k1 ) and zero order ( k0 ) of the reaction is
dt dt
Y

(2008, 3M)
24. In the reaction, P + Q ¾® R + S , the time taken for 75% (a) 0.5 mol -1 dm 3 (b) 1.0 mol dm -3
reaction of P is twice the time taken for 50% reaction of P. The (c) 1.5 mol dm -3 (d) 2.0 mol -1 dm 3
nd
Re

concentration of Q varies with reaction time as shown in the


figure. The overall order of the reaction is (2013 Adv.) 29. Consider a reaction, aG + bH ® products. When
Fi

concentration of both the reactants G and H is doubled, the


rate increases by eight times. However, when
[Q]0
concentration of G is doubled keeping the concentration of
H fixed, the rate is doubled. The overall order of the
[Q]
reaction is (2007, 3M)
Time (a) 0 (b) 1
(a) 2 (b) 3 (c) 0 (d) 1 (c) 2 (d) 3

25. The rate of a reaction doubles when its temperature changes 30. Which one of the following statement(s) is incorrect about
from 300 K to 310 K. Activation energy of such a reaction will order of reaction? (2005, 1M)
(a) Order of reaction is determined experimentally
be ( R = 8.314 JK -1 mol -1 and log 2 = 0.301)
(b) Order of reaction is equal to sum of the power of
(a) 53.6 kJ mol -1 (b) 48.6 kJ mol -1 (2013 Main)
concentration terms in differential rate law
(c) 58.5 kJ mol -1 (d) 60.5 kJ mol -1
(c) It is not affected with stoichiometric coefficient of the
reactants
(d) Order cannot be fractional
Chemical Kinetics 171

31. (A) follows first order reaction, ( A ) ® product. 39. The rate constant of a reaction depends on (1981, 1M)
Concentration of A, changes from 0.1 M to 0.025 M in (a) temperature
40 min. Find the rate of reaction of A when concentration of (b) initial concentration of the reactants
A is 0.01 M. (c) time of reaction
(2004, 1M)
(d) extent of reaction
(a) 3.47 ´ 10–4 M min –1 (b) 3.47 ´ 10-5 M min –1
(c) 1.73 ´ 10-4 M min –1 (d) 1.73 ´ 10-5 M min –1 Objective Questions II
32. In a first order reaction the concentration of reactant (One or more than one correct option)
decreases from 800 mol/dm3 to 50 mol/dm3 in 2 ´ 104 s. The 40. For a first order reaction A( g ) ¾® 2B( g )+ C ( g ) at
rate constant of reaction in s -1 is (2003, 1M) constant volume and 300 K, the total pressure at the
(a) 2 ´ 104 (b) 3.45 ´ 10-5 beginning ( t = 0 ) and at time t are p0 and pt , respectively.

w
(c) 1.386 ´ 10-4 (d) 2 ´ 10-4 Initially, only A is present with concentration [ A ]0 , and t1 / 3
33. Consider the chemical reaction, is the time required for the partial pressure of A to reach 1/3 rd
N2 ( g ) + 3H2 ( g ) ¾® 2NH3 ( g ) of its initial value. The correct option(s) is (are) (Assume that
all these gases behave as ideal gases)

Flo
(2018 Adv.)
The rate of this reaction can be expressed in terms of time
derivatives of concentration of N2 ( g ), H2 ( g ) or NH3 ( g ).

In(3p0–pt)

ree
Identify the correct relationship amongst the rate expressions

t1/3
(a) (b)
d [N2 ] 1 d [H2 ] 1 d[NH3 ]
(a) Rate = - =- =
dt 3 dt 2 dt (2002, 3M)

F
d [N2 ] d [H2 ] d [NH3 ]
(b) Rate = - = -3 =2 Time [A]0
dt dt dt

Rate constant
or
d [N2 ] 1 d [H2 ] 1 d [NH3 ]
(c) Rate =
dt
=
3 dt
=
2
ur
dt (c)
In(p0–pt)
(d)

f
d [N2 ] d [H2 ] d [NH3 ]
(d) Rate = - =- =
dt dt dt
ks
34. If I is the intensity of absorbed light and C is the
Yo
Time [A]0
oo
concentration of AB for the photochemical process.
AB + hn ® AB* , the rate of formation of AB* is directly 41. In a bimolecular reaction, the steric factor P was
eB

proportional to (2001, 1M)


experimentally determined to be 4.5. the correct option(s)
among the following is(are) (2017 Adv.)
(a) C (b) I
(a) The activation energy of the reaction is unaffected by the value
(c) I 2 (d) C × I
of the steric factor
r

35. The rate constant for the reaction, 2N2 O5 ¾® 4NO2 + O2


ou

(b) Experimentally determined value of frequency factor is higher


ad

than that predicted by Arrhenius equation


is 3.0 ´ 10-5 s -1 . If the rate is 2.40 ´ 10-5 mol L - 1 s - 1 , then
Y

(c) The value of frequency factor predicted by Arrhenius equation


the concentration of N2 O5 (in mol L - 1 ) is (2000, 1M) is higher than that determined experimentally
(a) 1.4 (b) 1.2 (d) Since P = 4.5, the reaction will not proceed unless an effective
nd
Re

(c) 0.04 (d) 0.8 catalyst is used


36. The half-life period of a radioactive element is 140 days. 42. According to the Arrhenius equation, (2016 Adv.)
Fi

After 650 days, one gram of the element will reduce to (a) a high activation energy usually implies a fast reaction
(1986) (b) rate constant increases with increase in temperature. This is
1 1 1 1
(a) g (b) g (c) g (d) g due to a greater number of collisions whose energy exceeds
2 4 8 16 the activation energy
37. A catalyst is a substance which (1983, 1M) (c) higher the magnitude of activation energy, stronger is the
(a) increases the equilibrium concentration of the product temperature dependence of the rate constant
(b) changes the equilibrium constant of the reaction (d) the pre-exponential factor is a measure of the rate at which
(c) shortens the time to reach equilibrium collisions occur, irrespective of their energy
(d) supplies energy to the reaction 43. For the first order reaction,
38. The specific rate constant of a first order reaction depends on 2N2 O5 ( g ) ¾® 4NO2 ( g ) + O2 ( g ) (2011)
the (1983, 1M) (a) the concentration of the reactant decreases exponentially with time
(a) concentration of the reactant (b) the half-life of the reaction decreases with increasing temperature
(b) concentration of the product (c) the half-life of the reaction depends on the initial concentration
(c) time of the reactant
(d) temperature (d) the reaction proceeds of 99.6% completion in eight half-life
duration
172 Chemical Kinetics

44. The following statement (s) is are correct (1999, 3M) The comparison of the b- activity of the dead matter with that of the
1 carbon still in circulation enables measurement of the period of the
(a) A plot of log K p vs is linear
T isolation of the material from the living cycle. The method
(b) A plot of log [X] vs time is linear for a first order reaction, however, ceases to be accurate over periods longer than 30,000 yr.
x®p The proportion of 14 C to 12 C in living matter is 1 : 1012.
1
(c) A plot of log p vs is linear at constant volume (2006, 3 ´ 4M = 12M)
T
1 48. Which of the following option is correct?
(d) A plot of p vs is linear at constant temperature
V (a) In living organisms, circulation of 14 C from atmosphere is high
so the carbon content is constant in organism
45. For the first order reaction, (1998, 2M)
(b) Carbon dating can be used to find out the age of earth crust and
(a) the degree of dissociation is equal to (1 - e- kt ) rocks

w
(b) a plot of reciprocal concentration of the reactant vs time gives a (c) Radioactive absorption due to cosmic radiation is equal to the
straight line rate of radioactive decay, hence the carbons content remains
(c) the time taken for the completion of 75% reaction is thrice the constant in living organisms
1 (d) Carbon dating cannot be used to determine concentration of
of the reaction

Flo
2 14
C in dead beings
(d) the pre-exponential factor in the Arrhenius equation has the
49. What should be the age of fossil for meaningful
dimension of time, T -1

ree
determination of its age?
46. A catalyst (1984, 1M) (a) 6 yr
(a) increases the average kinetic energy of reacting molecules (b) 6000 yr

F
(b) decreases the activation energy (c) 60,000 yr
(c) alters the reaction mechanism (d) It can be used to calculate any age

or
(d)
ur
increases the frequency of collisions of reacting species 50. A nuclear explosion has taken place leading to increase in
concentration of C14 in nearby areas. C14 concentration is C1

f
Numerical Value in nearby areas and C 2 in areas far away. If the age of the
ks
fossil is determined to be T1 and T2 at the places respectively
47. Consider the following reversible reaction,
then
Yo
A( g )+ B( g ) - AB( g )
oo
(a) the age of fossil will increase at the place where explosion has
The activation energy of the backward reaction exceeds that 1 C
taken place and T1 - T2 = ln 1
of the forward reaction by 2RT (in J mol -1 ). If the
eB

l C2
pre-exponential factor of the forward reaction is 4 times that (b) the age of fossil will decrease at the place where explosion has
of the reverse reaction, the absolute value of DGs (in J 1 C
taken place and T1 - T2 = ln 1
mol -1 ) for the reaction at 300 K is ……… . l C2
r
ou
ad

(Given ; ln( 2 ) = 0.7 RT = 2500 J mol -1 at 300 K and G is the (c) the age of fossil will be determined to be the same
T C
Gibbs energy) (d) 1 = 1
Y

(2018 Adv.)
T2 C 2
Passage Based Questions
Fill in the Blanks
nd
Re

Passage 51. In Arrhenius equation, k = A exp (- E a / RT )


. A may be termed
Carbon-14 is used to determine the age of organic material. The
Fi

as the rate constant at ......... (1997, 1M)


procedure is based on the formation of 14 C by neutron capture in the
upper atmosphere. 52. For the reaction : N2 ( g ) + 3H2 ( g ) ¾® 2NH3 ( g )
14 1 14 1
7 N + 0n ¾® 6 C + 1p
Under certain conditions of temperature and partial pressure
14 14 of the reactants, the rate of formation of NH3 is 0.001 kg/h-1.
C is absorbed by living organisms during photosynthesis. The C
content is constant in living organism once the plant or animal dies, The rate of conversion of H2 under the same condition is ....
the uptake of carbon dioxide by it ceases and the level of 14 C in the kg /h -1 . (1994, 1M)
dead being, falls due to the decay which C-14 underoges 53. The hydrolysis of ethyl acetate in ........... medium is a ..........
14
6 C ¾® 14
7 N + b- order reaction. (1986, 1M)
14
The half-life period of C is 5770 yr. 54. The rate of chemical change is directly proportional to
The decay constant ( l ) can be calculated by using the following ............. (1985, 1M)
0.693
formula l = .
t1 / 2
Chemical Kinetics 173

True/False 63. The rate constant for an isomerisation reaction, A ® B is


55. For a first order reaction, the rate of the reaction doubles as the 4.5 ´ 10-3 min. If the initial concentration of A is 1 M,
concentration of the reaction (s) doubles. (1986, 1M) calculate the rate of the reaction after 1 h. (1999, 4M)

Integer Answer Type Questions 64. (i) The rate constant of a reaction is 1.5 ´ 107 s-1 at 50° C and
56. An organic compound undergoes first order decomposition. 4.5 ´ 107 s-1 at 100° C. Evaluate the Arrhenius parameters
The time taken for its decomposition to 1/8 and 1/10 of its A and Ea . (1998, 5M)
initial concentration are t1 / 8 and t1 / 10 respectively. What is the 1
(ii) For the reaction, N2O5 (g ) ¾® 2NO2 (g ) + O2 (g ),
[t ] 2
value of 1 / 8 ´ 10 ? (log 10 2 = 0.3)
[ t1 / 10 ] calculate the mole fraction N2O5 (g ) decomposed at a
(2012)
constant volume and temperature, if the initial pressure is
57. The concentration of R in the reaction R ¾® P was

w
600 mm Hg and the pressure at any time is 960 mm Hg.
measured as a function of time and the following data is Assume ideal gas behaviour.
obtained :
65. The rate constant for the first order decomposition of a
[ R ] (molar) 1.0 0.75 0.40 0.10
certain reaction is described by the equation

Flo
t (min) 0.0 0.05 0.12 0.18
1.25 ´ 104 K
The order of the reaction is (2010) log k (s -1 ) = 14.34 -
T

ree
Subjective Questions (i) What is the energy of activation for the reaction?
58. 2 X ( g ) ¾® 3Y ( g ) + 2 Z ( g ) (ii) At what temperature will its half-life period be 256 min?

F
(1997, 5M)
Time 0 100 200
66. One of the hazards of nuclear explosion is the generation of
(in min)

or
Sr 90 and its subsequent incorporation in bones. This
Partial pressure of 800
ur 400 200 nucleide has a half-life of 28.1 yr. Suppose one microgram

f
X (in mm of Hg)
was absorbed by a new-born child, how much Sr 90 will
ks
Assuming ideal gas condition. Calculate remain in his bones after 20 yr. (1995, 2M)
Yo
(a) order of reaction
67. At 380° C, the half-life period for the first order
oo
(b) rate constant
(c) time taken for 75% completion of reaction decomposition of H2 O2 is 360 min. The energy of activation
of the reaction is 200 kJ mol -1 . Calculate the time required
eB

(d) total pressure when px = 700 mm (2005, 4M)

59. For the given reaction, A + B ¾® Products for 75% decomposition at 450° C. (1995, 4M)

Following data are given 68. From the following data for the reaction between A and B
r
ou

[A], (mol/L) [B], (mol/L) Initial rate (mol L–1s–1) at


ad

Initial conc. Initial conc. Initial rate


(m/L) (m/L) [mL–1s –1 ]
300 K 320 K
Y

[ A ]0 [ B ]0 -4 -5 -4
2. 5 ´ 10 3. 0 ´ 10 5. 0 ´ 10 2.0 ´ 10-3
0.1 0.1 0.05
nd

0.2 0.1 0.1 5.0 ´ 10-4 6.0 ´ 10-5 4.0 ´ 10-3


Re


0.1 0.2 0.05
1. 0 ´ 10-3 6. 0 ´ 10-5 1. 6 ´ 10-2 —
Fi

(a) Write the rate equation.


(b) Calculate the rate constant. (2004, 2M) Calculate
60. 64
Cu (half-life = 12.8 h) decays by b emission (38%), b+ (i) the order of the reaction with respect to A and with
emission (19%) and electron capture (43%). Write the decay respect to B.
products and calculate partial half-lives for each of the decay (ii) the rate constant at 300 K.
processes. (2002) (iii) the pre-exponential factor. (1994, 5M)
61. The rate of first order reaction is 0.04 mol L–1s –1 at 10 min and
69. The gas phase decomposition of dimethyl ether follows first
0.03 mol L–1s –1 at 20 min after initiation. Find the half-life of the order kinetics
reaction. (2001, 5M)
CH3 — O— CH3 ( g ) ¾® CH4 ( g ) + H2 ( g ) + CO ( g )
62. A hydrogenation reaction is carried out at 500 K .If the same The reaction is carried out in a constant volume container at
reaction is carried out in the presence of a catalyst at the same 500° C and has a half-life of 14.5 min. Initially only
rate, the temperature required is 400 K. Calculate the dimethyl ether is present at a pressure of 0.40 atm. What is
activation energy of the reaction if the catalyst lowers the the total pressure of the system after 12 min? Assume ideal
activation barrier by 20 kJ mol -1 . (2000, 3M) gas behaviour. (1993, 4M)
174 Chemical Kinetics

70. A first order reaction, A ® B, requires activation energy of period of 42 Mo


99
, which is a beta emitter, is 66.6 h. Find
-1
70 kJ mol . When a 20% solution of A was kept at 25° C for the minimum amount of 42 Mo
99
required to carry out the
20 min, 25% decomposition took place. What will be the experiment in 6.909 h. (1989, 5M)
percentage decomposition in the same time in a 30% solution -6
maintained at 40°C ? Assume that activation energy remains 76. A first order gas reaction has k = 1.5 ´ 10 per second at
constant in this range of temperature. (1993, 4M)
200° C. If the reaction is allowed to run for 10 h, what
percentage of the initial concentration would have change in
71. Two reactions (i) A ® products (ii) B ® products, follow the product? What is the half-life of this reaction?
first order kinetics.The rate of the reaction (i) is doubled (1987, 5M)

when the temperature is raised from 300 K to 310 K. 77. While studying the decomposition of gaseous N2 O5 , it is
The half-life for this reaction at 310 K is 30 min. At the same observed that a plot of logarithm of its partial pressure versus

w
temperature B decomposes twice as fast as A. If the energy of time is linear. What kinetic parameters can be obtained from
activation for the reaction (ii) is half that of reaction (i), this observation? (1985, 2M)
calculate the rate constant of the reaction (ii) at 300 K.
78. Radioactive decay is a first order process. Radioactive

Flo
(1992, 3M)
3 1
carbon in wood sample decays with a half-life of 5770 yr.
72. The nucleidic ratio, 1H to 1H in a sample of water is What is the rate constant (in yr - 1 ) for the decay? What
8.0 ´ 10- 18 : 1. Tritium undergoes decay with a half-life

ree
fraction would remain after 11540 yr? (1984, 3M)
period of 12.3 yr. How many tritium atoms would 10.0 g of 79. A first order reaction is 20% complete in 10 min. Calculate
such a sample contain 40 yr after the original sample is

F
(i) the specific rate constant of the reaction, and
collected. (1992, 4M) (ii) the time taken for the reaction to go to 75% completion.
73. The decomposition of N2 O5 according to the equation,

or
(1983, 2M)
ur
2N2 O5 ( g ) ¾® 4NO2 ( g ) + O2 ( g ) 80. Rate of reaction, A + B ® products is given below as a

f
is a first order reaction. After 30 min from the start of the function of different initial concentrations of A and B
ks
decomposition in a closed vessel, the total pressure
[A] mol/L [B] (mol/L) Initial rate
Yo
developed is found to be 284.5 mm of Hg. On complete
(mol L–1 min –1 )
oo
decomposition, the total pressure is 584.5 mm of Hg.
Calculate the rate constant of the reaction. (1991, 6M)
0.01 0.01 0.005
eB

0.02 0.01 0.010


74. In Arrhenius equation for a certain reaction, the value of A and
13 -1 –1 0.01 0.02 0.005
Ea (activation energy) are 4 ´ 10 s and 98.6 kJ mol
respectively. If the reaction is of first order, at what Determine the order of the reaction with respect to A and B.
r

What is the half-life of A in the reaction ?


ou

temperature will its half-life period be 10 min? (1982, 4M)


ad

(1990, 3M)
75. An experiment requires minimum beta activity produced at
Y

the rate of 346 beta particles per minute. The half-life


nd

Answers
Re

51. T = ¥
Fi

1. (b) 2. (c) 3. (d) 4. (a) 48. (c) 49. (b) 50. (a)
5. (d) 6. (d) 7. (b) 8. (a) 52. 0.0015 53. acidic first or basic, second
9. (c) 10. (d) 11. (b) 12. (d) 54. concentration of reactant(s) at that instant
13. (c) 14. (b) 15. (c) 16. (b) 55. T 56. (9) 57. (0)
17. (a) 18. (a) 19. (d) 20. (a) 58. (960 mm Hg) 61. (25 min) 62. (100 kJ mol -1)
-3 -1 -1
21. (a) 22. (b) 23. (d) 24. (d) 63. (3.26 ´ 10 mol L min ) 66. (6.1 ´ 10 -7 g)
25. (a) 26. (a) 27. (d) 28. (a)
67. (20.74 min) 69. (0.75 atm) 70. (67 %)
29. (d) 30. (d) 31. (a) 32. (c)
71. (3.26 ´ 10 -2 min -1
) 72. (5.6 ´ 10 5)
33. (a) 34. (d) 35. (d) 36. (d)
73. (5.2 ´ 10 -3 min -1
) 74. (311.34 K)
37. (c) 38. (d) 39. (a) 40. (a,d) -16
75. (3.56 ´ 10 g) 78. (0.25)
41. (a,c) 42. (b,c,d) 43. (a,b,d) 44. (a,b,d)
80. (1.386 min)
45. (a,d) 46. (b,c) 47. (+8500J/ mol)
Hints & Solutions
1. For the reaction, H2 + I2 ¾® 2HI
Key Idea The rate of a chemical reaction means the speed Given k 1 = 2.5 ´ 10-4 dm3mol-1s -1
with which the reaction takes place.
For R ¾® P T1 = (273 + 327) K = 600 K
Rate of disappearance of R k 2 = 1 dm 3mol -1 s -1 at T2 = (273 + 527) K = 800 K
Decrease in conc.of R D[ R]
= =- k2 Ea æ T2 - T1 ö
Time taken Dt Now, log = ç ÷
k 1 2.303R çè T1T2 ÷ø
Rate of appearance of P
Increase in conc. of P D[ P] 1 Ea æ 800 - 600 ö
= =+ Þ log = çç ÷÷
Dt 2.5 ´ 10-4 2.303 ´ 8.314 ´ 10-3

w
Time taken è 600 ´ 800 ø
Given, [N2O5 ]initial = 3.00 mol L- 1 (10 ´ 103 ) E 200
Þ log = a ´
2.5 0.019 48 ´ 104
After 30 min, [N2O5 ] = 2 .75 mol L- 1

Flo
Þ log 4 + 3 log10 -~ E ´ 0.022
2N2O5 (g ) ¾® 4NO2 (g ) + O2 (g ) a
2 ´ log 2 + 3
t= 0 3.0 M Þ Ea =

ree
t = 30 2.75 M
0.022
3.6 ~
From the equation, it can be concluded that = – 163.6kJ mol -1
0.022

F
1 - D[N2O5 ] 1 D[NO2 ]
´ = ´ 4. The expression for bacterial growth is
2 Dt 4 Dt
N = N 0et

or
-1
=
- D[N2O5 ] - (2.75 - 3.00) mol L
=
ur Þ
0.25
N0
Dt 30 30 = e- t

f
N
D[NO2 ] D (N2O5 ) D[NO2 ] 0.25
and =-2 Þ =- 2 ´ From 0 to 1 hour N ¢ (t ) = N 0et
ks
Dt Dt Dt 30
Yo
= - 1667
. ´ 10- 2 mol L- 1 min - 1 From 1 hour onwards,
dN
= -5N 2
oo
dt
2. In the given reaction; x A ¾¾® y B On differentiating the above equation from N ¢ to N we get.
eB

é - d[ A ]ù é d[ B ]ù N t
log10 = log10 + 0.3010
òN dN = -5ò dt
-2
êë dt úû êë dt úû [Q At 1 hour, N ¢= eN 0]
eN0 1
Value of log 2 = 0.3010
r

é 1 1 ù
ê N - eN ú = 5(t - 1)
Substituting 0.3010 by log2
ou
ad

é d[ A ]ù é d[ B ]ù ë 0û
log10 - = log10 + log 2
êë dt úû êë dt úû Multiply both sides by N 0, we get
Y

N0 1 N 1
Using logarithm rules, - = 5N 0 (t - 1) or, 0 = 5N 0 (t - 1) +
N e N e
nd

é - d[ A ]ù é d[ B ]ù 1 é d [ A ]ù é d[ B ]ù
Re

= 2´ Þ- = …(i) N0 é1 ù
ëê dt ûú ëê dt ûú 2 ëê dt ûú êë dt ûú = 5N 0t + - 5N 0
N ëê e ûú
Fi

Using the rate equation (i) to determine the reaction involved is On comparing the above equation with equation of straight line,
2A ¾¾® B y = mx + c
Option that fits correct in the above reaction is (c). 1
We get m = 5N 0, c = - 5N 0
2C2H4 ¾¾® C4H8 . e
N0
\ Plot of vs t is shown aside.
3. N
Key Idea The Arrhenius equation for rate constants at two
different temperatures is
k Ea é T2 - T1 ù
log 2 = [where, T2 > T1]
k 1 2.303R êë T1T2 úû
N0
where, k 1 and k 2 are rate constants at temperatures T1 and T2, —
N
respectively.
R = Gas constant, Ea = Activation energy t(h)
176 Chemical Kinetics

5. Only statement (d) is incorrect. Corrected statement is 7.


K1
A ¾® B ¾® C
K2
-1
‘‘Activation enthalpy to form C is 15 kg mol more than 5 kg
Rate of formation of B is
mol -1 that is required to form D.’’ It can be easily explained by
d[ B ]
following graph. = k 1[ A ] - k 2[ B ]
dt
é d[ B ] ù
20 Þ 0 = k 1[ A ] - k 2[ B ] Q Given, =0
êë dt úû
15
Enthalpy Activation Þ k 2[ B ] = k 1[ A ]
(kJ mol ) 10
–1
D enthalpy k1
Þ Concentration of B, [ B ] = [A]
5 k2
A+B

Reaction C 8. Let the rate equation be k [ A ]x [ B ]y

w
coordinate
From Ist values,
Activation enthalpy (or energy) is the extra energy required by 0.045 = k [ 0.05 ]x [ 0.05 ]y …(i)
the reactant molecules that result into effective collision
From 2nd values,

Flo
between them to form the products.
0.090 = k [ 010
. ]x [ 0.05 ]y …(ii)
6. In first order reaction, the rate expression depends on the
From 3rd values,

ree
concentration of one species only having power equal to unity.
nR ¾® products 0.72 = k [ 0.20 ]x [ 010
. ]y …(iii)
- d[ R ] On dividing equations (i) by (ii), we get

F
= k [R ] x
dt 0.045 é 0.05 ù
=
On integration, - ln[ R ] = kt - ln[ R0 ] 0.09 êë 010
. ûú

or
or ln(R ) = ln (R0 ) - kt
ur é 0.05 ù
1
é 0.05 ù
x
=

f
y = c + mx êë 010
. úû êë 010
. úû
m = slope = -k (negative) \ x =1
ks
c = intercept = ln (r0 )
Yo
Similarly on dividing Eq. (ii) by (iii) we get
oo
x y
The graph for first order reactions is 0.09 é 01. ù é 0.05 ù
=
0.72 êë 0.2 úû êë 010
. úû
eB

y
. é 0.05 ù
0.01 01
=
ln (R) 0.08 0.2 êë 01
. úû
r

y
é 0.05 ù
0.25 =
ou

êë 010
. úû
ad

t
0.25 = [ 0.5 ]y
Y

In zero order reaction,


[ 0.5 ]2 = [ 0.5 ]y
[ R ] ¾® product
\ y=2
nd

- d[ R ]t
Re

\ = k or - d[ R ]t = kdt Hence, the rate law for the reaction


dt
Rate = k [ A ][ B ]2
Fi

On integrating, -[ R ]t = kt + c
If t = 0, [ R ]t = [ R ]0 9. The temperature dependence of a chemical reaction is expressed
by Arrhenius equation,
\ -[ R ]t = kt - [ R ]0
k = Ae- E a$ / RT …(i)
[ R ]t = [ R ]0 - kt
Thus, the graph plotted between [ r ]t and t gives a straight line Taking natural logarithm on both sides, the Arrhenius equation
with negative slope (-k ) and intercept equal to [ R ]0. becomes,
E
The graph for zero order reaction is ln k = ln A - a
RT
E
where, - a is the slope of the plot and ln A gives the intercept.
R
[R] Eq. (i) at two different temperatures for a reaction becomes,
k E æ1 1ö
ln 2 = a çç - ÷÷ …(ii)
k1 R è T1 T2 ø

t
Chemical Kinetics 177

Þ In the given problem, Taking log on both sides of the Eq. (i), the equation
T1 = 400K, T2 = 500 K E
becomes ln k = ln A - a
k 1 = 10- 5 s - 1, k 2 = ? RT
E
- a (Slope) = - 4606
R
On substituting all the given values in Eq. (ii), we get ln/k
k æ 1 1 ö
ln -2 5 = 4606 ç - ÷
10 è 400 500 ø
k 1/RT
ln -2 5 = 2.303
10 On comparing with equation of straight line
k2 1
= 10 Þ k 2 = 10- 4 s - 1 ( y = mx + c), the nature of the plot of lnk vs

w
will be:
10- 5 RT
Therefore, rate constant for the reaction at (i) Intercept = C = ln A
500 K is 10- 4s - 1. (ii) Slope/gradient = m = - Ea = - y Þ Ea = y

Flo
So, the energy required to activate the reactant, (activation
10. Given, rate constant (k) = 0.05 mg/year energy of the reaction, Ea is = y )
Thus, from the unit of k, it is clear that the reaction is zero order. k1

ree
Now, we know that 13. The elementary reaction, A2 c 2A
k -1
a
half-life (t1/ 2 ) for zero order reaction = o follows opposing or reversible kinetics,

F
2k
where, ao = initial concentration, (i) Rate of the reaction,
k = rate constant r = rforward - rbackward

or
5 mg
ur = k 1[ A2 ] - k - 1[ A ]2 … (i)
t1/ 2 = = 50 years

f
2 ´ 0.05 mg / year (ii) Again, rate of the reaction can be expressed as,
Thus, 50 years are required for the decomposition of 5 mg of X d [ A2 ] 1 d[ A ]
ks
r=- =+
into 2.5 mg. dt 2 dt
Yo
So, the rate of appearance of A, i.e.
oo
11. For zero order reaction,
[ A0 ] - [ At ] = kt d[ A ]
...(i) = 2r = 2k 1[ A2 ] - 2 k - 1 [ A ]2 [from Eq. (i)]
eB

where, [ A0 ] = initial concentration dt


[ At ] = final concentration at time ‘t’ 14. The Arrhenius equation is,
k = rate constant k = A. e- E a / RT
r

[A ]
ou

Also, for zero order reaction, t1/ 2 = 0 where, k = rate constant,


ad

2k A = Arrhenius constant, Ea = activation energy,


Given, t1/ 2 = 6 h and [ A0 ] = 0.2 M
Y

and T = temperature in K
0.2
\ 6= From the equation, it is clear that k decreases exponentially with
2k Ea . So, the plot-I is correct.
nd
Re

0.2 1 In the plot-II, k is plotted with temperature (in °C but not in K).
or, k= =
2 ´ 6 60 So, at 0°C, k ¹ 0 and k will increase exponentially with
Fi

Now, from Eq. (i) temperature upto 300°C. Therefore, the plot-II is also correct.
[ A0 ] - [ At ] = kt 15. For the reaction, 2A + B ¾® products.
Given, [ A0 ] = 0.5 M, [ At ] = 0.2 M Let, the rate expression is
1 é 1ù r µ [ A ]a [ B ]b
\ 0.5 - 0.2 = ´t Qk =
60 êë 60 úû a b
r2 æ 2 A ö æ 2 B ö
1 Expt 1 =ç ÷ ç ÷
0.3 = ´t r1 è A ø è B ø
60
2.4
t = 0.3 ´ 60 = 18 h Þ = 2a ´ 2b Þ 23 = 2a + `b
0.3
12. The temperature dependence of rate of a chemical reaction is Þ 3=a+ b … (i)
expressed by Arrhenius equation as, k = Ae- E a / RT …(i) a b
r2 æ 2 A ö æ B ö
where, A = Arrhenius factor or frequency factor or Expt 2 =ç ÷ ç ÷
pre-exponential factor r1 è A ø è B ø
R = Gas constant, Ea = Activation energy 0.6
Þ = 2a ´ 1 Þ 21 = 2a Þ a = 1 …(ii)
0.3
178 Chemical Kinetics

\ From Eq. (i), 1 + b = 3 Þ b = 2 18. For the reaction,


Þ Order of the reaction (n) = a + b = 1 + 2 = 3 Decomposes
CH3CHO( g ) ¾¾¾¾® CH4 + CO
Þ Order of the reaction wrt. A = 1
Þ Order of the reaction wrt. B = 2 Let order of reaction with respect to CH3CHO is m.
16. Let, the rate expression is r µ [ A ] [ B ] . a b Its given, r1 = 1 torr/sec. when CH3CHO is 5% reacted i.e. 95%
unreacted. Similarly, r2 = 0.5 torr/sec when CH3CHO is 33%
From experiment I, reacted i.e., 67% unreacted.
a b
r2 æ 0.1 ö æ 0.25 ö Use the formula, r µ (a - x )m
=ç ÷ ´ç ÷
r1 è 0.1 ø è 0.20 ø where (a - x ) = amount unreacted
-3 b m
6.93 ´ 10 æ 5ö r1 (a - x1 )m r1 é a - x1 ù
Þ =1 ´ ç ÷ so, = or =
6.93 ´ 10- 3 è4ø r2 (a - x2 )m r2 êë a - x2 úû

w
b 0 b
æ 5ö æ 5ö æ 5ö Now putting the given values
Þ 1 = ç ÷ Þ ç ÷ = ç ÷ Þ b =0 m
è4ø è4ø è4ø 1 æ 0.95 ö
=ç ÷ Þ 2 = (141
. )m or m = 2
0.5 è 0.67 ø

Flo
a b
r3 æ 0.2 ö æ 0.30 ö
From experiment II, =ç ÷ ´ç ÷
r1 è 0.1 ø è 0.20 ø 19. According to Arrhenius equation
-2
k = Ae- E a / RT

ree
1.386 ´ 10
Þ = (2)a ´ (1.5)0
0.693 ´ 10- 2 where, A = collision number or pre-exponential factor.
Þ 2 = 2a ´ 1 Þ 21 = 2a Þ a =1

F
R = gas constant
Þ So, 1 0
r µ[ A] [ B] Þ r µ[ A] T = absolute temperature
Ea = energy of activation

or
Order of the reaction (n) = 1
ur - E a1 / RT
Þ Now, let for the 1st experiment, For reaction R1, k 1 = Ae …(i)
r1 = k × [ A ]

f
For reaction R2, k 2 = Ae
- E a2 / RT
…(ii)
ks
r1 6.93 ´ 10- 3
Þ k= = = 6.93 ´ 10- 2 s- 1
Yo
[A] 0.1 On dividing Eq. (ii) by Eq. (i), we get
oo
(E a - Ea )
0.693 0.693 k2 - 2 1
Þ t50 = = = 10 s =e RT …(iii)
6.93 ´ 10- 2
eB

k k1
17. From thermodynamics, [Q Pre-exponential factor ‘A’ is same for both
-DH ° DS ° reactions]
ln k = + …(i)
r

RT R Taking ln on both the sides of Eq. (iii), we get


ou

æ k ö E a1- E a 2
ad

Mathematically, the equation of straight line is


ln çç 2 ÷÷ =
y = c + mx …(ii) è k1 ø RT
Y

After comparing Eq. (ii) with (i) we get, Given, E a 1 = E a 2 + 10 kJ mol - 1 = Ea 2 + 10,000 J mol - 1
- DH ° DS °
slope = and intercept = 10,000 J mol - 1
nd

k2
Re

R R \ ln = =4
Now, we know for exothermic reaction DH is negative (-)ve. k 1 8.314 J mol - 1K- 1 ´ 300 K
Fi

But here, 2.303 a


-DH ° 20. For first order reaction, k = log
Slope = is positive t a-x
R
Given, t = 50 min, a = 0.5 M, a - x = 0125
. M
So, lines A and B in the graph represent temperature dependence
2.303 0.5
of equilibrium constant K for an exothermic reaction as shown \ k= log = 0.0277 min -1
below 50 0125
.
Now, as per reaction
2H2O2 ¾® 2H2O + O2
ln K A
1 d [H2O2 ] 1 d [H2O] d [O2 ]
- = =
B 2 dt 2 dt dt
1 d [H2O2 ]
Rate of reaction, - = k [H2O2 ]
(0, 0) T(K) dt
d [O2 ] 1 d [H2O2 ] 1
\ =- = k [H2O2 ] …(i)
dt 2 dt 2
Chemical Kinetics 179

When the concentration of H2O2 reaches 0.05 M, k2 - Ea æ 1 1ö


25. From Arrhenius equation, log = ç - ÷
d [O2 ] 1 k1 2.303 R çT ÷
= ´ 0.0277 ´ 0.05 [from Eq. (i)] è 2 T1 ø
dt 2 k2
d [O2 ] Given, = 2 T2 = 310 K
or = 6.93 ´ 10-4 mol min -1 k1
dt
T1 = 300 K
Alternative Method
On putting values,
In fifty minutes, the concentration of H2O2 decreases from 0.5 to
0.125 M or in one half-life, concentration of H2O2 decreases - Ea æ 1 1 ö
Þ log 2 = ç - ÷
from 0.5 to 0.25 M. In two half-lives, concentration of H2O2 2.303 ´ 8.314 è 310 300 ø
decreases from 0.5 to 0.125 M or 2 t1/ 2 = 50 min Þ Ea = 53.603 kJ/mol
t1/ 2 = 25 min
26. According to Arrehnius equation, rate constant increases

w
æ 0.693 ö -1
\ k =ç ÷ min exponentially with temperature :
è 25 ø k = Ae- E a / RT
d [O2 ] 1 d [H2O2 ] k [H2O2 ]
or =- = = 6.93 ´ 10-4 mol min -1
dt 2 dt 2 27. The logarithmic form of Arrhenius equation is

Flo
Ea
21. The main conditions for the occurrence of a reaction is log k = log A -
proper orientation and effective collision of the reactants. 2.303 RT

ree
Since, the chances of simultaneous collision with proper 2000
Given : log k = 6 -
orientation between two species in high order reactions are very T

F
rare, so reaction with order greater than 3 are rare. Comparing the above two equations :
22. For the elementary reaction, M ¾® N log A = 6 Þ A = 106

or
Rate law can be written as
Rate µ [ M ] n
ur and
Ea
2.303 R
= 2000

Rate = k [ M ] n …(i) Þ

f Ea = 2000 ´ 2.303 ´ 8.314 J


ks
When we double the concentration of [ M ], = 38.3 kJ mol -1
Yo
rate becomes 8 times, hence new rate law can be written as ln 2
oo
28. For first order reaction t1/ 2 = = 40 s …(i)
8 ´ Rate = k [ 2M ] n …(ii) k1
eB

Rate k [M ]n 1 1 [ A ]0
= Þ = For zero order reaction t1/ 2 = = 20 s …(ii)
8 ´ Rate k [ 2M ] n 8 [2]n 2k 0
Þ [2]n = 8 = [2]3 Þ n=3 1 [ A ]0 k
Þ Eq. (ii)/(i) = = ´ 1
r

2 2k 0 ln 2
ou

23. This problem can be solved by determining the order of reaction


ad

w.r.t. each reactant and then writing rate law equation of the k 1 ln 2 0.693
Þ = = = 0.5
given equation accordingly as k 0 [ A ]0 1.386
Y

dC
R= = k [ A ]x [ B ] y 29. Rate µ [G ] m [ H ] n
dt
nd
Re

where, x = order of reaction w.r.t A Q Rate is double on doubling the concentration of G and
y = order of reaction w.r.t B maintaining H constant, m = 1, i.e. R µ [G ].
1. 2 ´ 10-3 = k (0 .1)x (0 .1) y
Fi

Also, when both concentration of G and H are doubled, rate


1. 2 ´ 10-3 = k (01
. )x (0.2) y increases by a factor of 8. Here rate is increasing by a factor of 2
2.4 ´ 10-3 = k (0.2)x (01
. )y due to G (first order in G), therefore, factor due to H is 4.
Þ R µ [ H ]2
R = k [ A ]1[ B ]0
As shown above, rate of reaction remains constant as the Þ Overall order = m + n = 1 + 2 = 3
concentration of reactant (B) changes from 0.1 M to 0.2 M and 30. Order of a reaction can take any real value, i.e. negative, integer,
becomes double when concentration of A change from 0.1 to 0.2, fraction etc.
(i.e. doubled).
31. For first order reaction,
24. PLAN Time of 75% reaction is twice the time taken for 50% reaction if
2.303 a 2.303 0.1
it is first order reaction w.r.t. P. From graph, [Q ] decreases k= log = log = 3.46 ´ 10-2
linearly with time, thus it is zeroth order reaction w.r.t. Q t a-x 40 0.025
dx
= bk[ P ]a[Q ]b Rate = [ k ] A = 3.46 ´ 10-2 ´ 0.01 = 3.46 ´ 10-4
dt
Order w.r.t P = a=1 [ A ]0
Order w.r.t Q =b=0
32. For a first order reaction, kt = ln
[A]
Thus, overall order of the reaction = 1 + 0 = 1
180 Chemical Kinetics

1 [ A ]0 1 800 4 ln 2 - 1 2 p0
Þ k= ln = ln = s or kt = ln
t [ A ] 2 ´ 104 50 2 ´ 104 ( 3 p0 - pt )
= 1.386 ´ 10- 4 s- 1 or kt = ln 2 p0 - ln ( 3 p0 - pt )
33. For any general reaction, or ln ( 3 p0 - pt ) = - kt + ln 2 p0
aA + bB ¾® cC + dD It indicates graph between ln (3p0 - pt ) vs ‘t’ will be a straight
1 d [A] 1 d[ B ] line with negative slope , so option (a) is correct
Rate = - =-
a dt b dt 1 p0 1
1 d [C ] 1 d [ D ] t1/3 = ln = ln 3
= = k p0 / 3 k
c dt d dt
Þ For N2 + 3H2 ¾® 2NH3 It indicates t1/ 3 is independent of initial concentration so, option
(b) is incorrect.

w
d [ N2 ] 1 d [ H2 ] 1 d [ NH3 ]
Rate = - =- = Likewise, rate constant also does not show its dependence over
dt 3 dt 2 dt
initial concentration. Thus, graph between rate constant and
34. Rate will be directly proportional to both concentration and [ A ]0 will be a straight line parallel to X-axis.
intensity, i.e. rate of formation of AB * µ C × I .

Flo
41. If steric factor is considered, the corrected Arrhenius equation
35. The unit of rate constant (t - 1) indicating that the decomposition will be
reaction following first order kinetics. -Ea

ree
Þ Rate = k[ N2O5 ] k = pAe RT where A = frequency factor by Arrhenius.
Rate 2.40 ´ 10-5 Q p > 1, pA > A hence, (a) is correct.
[ N2O5 ] = = = 0. 8 M

F
k 3 ´ 10-5 Activation energy is not related to steric factor.

36. 560 days =


560
= 4 half-lives. 42. Rate constant, k = Ae- E a / RT

or
140
ur where, Ea = activation energy and A = pre-exponential factor
Amount of reactant remaining after n-half-lives

f
n 4
(a) If Ea is high, it means lower value of k hence, slow reaction.
æ 1ö æ 1ö 1 Thus, incorrect.
ks
= ç ÷ ´ initial amount = ç ÷ ´ 1.0 g = g
è 2ø è 2ø 16 (b ) On increasing temperature, molecules are raised to higher
Yo
oo
energy (greater than Ea ), hence number of collisions
37. A catalyst increases the rate of reaction but by the same factor to
increases. Thus, correct.
both forward and backward directions. Hence, a catalyst shorten
eB

the time required to reach the equilibrium. E d (log k ) E


(c) log k = log A - a Þ = a2
RT dT RT
38. Specific rate constant of reaction depends on temperature.
Thus, when Ea is high, stronger is the temperature
r

39. The rate constant (k ) of all chemical reactions depends on dependence of the rate constant. Thus, correct.
ou

temperature.
ad

(d) Pre-exponential factor (A) is a measure of rate at which


k = Ae- E a / RT collisions occur. Thus, correct.
Y

where, A = pre-exponential factor, Ea = activation energy. 43. (a) For a first order reaction, the
concentration of reactant remaining
40. Given for the reaction (at T= 300 K and constant volume = V) after time t is given by [A]
nd
Re

A ( g ) ¾® 2B ( g ) + C ( g ) [ A ] = [ A ] 0 e– kt
at t = 0 p0 – –
Therefore, concentration of reactant
Fi

at t = t p0 - x 2x x t
decreases exponentially with time.
é 2 p0 ù p0 4 p0 2 p0
at t = t1/ 3 p
ê 0 - =
ë 3 úû 3 3 3 (b) Rise in temperature increases rate constant (k) and therefore
decreases half-life (t1/ 2) as
We can calculate,
ln 2
pt = p0 - x + 2x + x = p0 + 2x t1/ 2 =
p - p0 k
or 2x = pt - p0 or x = t (c) Half-life of first order reaction is independent of initial
2
concentration.
Now for first order reaction,
1 p0 (d) For a first order reaction, if 100 moles of reactant is taken
t = ln initially, after n half-lives, reactant remaining is given by
k ( p0 - x) n 8
Putting the value of x in the equation, æ 1ö æ 1ö
% A = 100 ç ÷ = 100 ç ÷ = 0.3906
1 p0 1 2 p0 è 2ø è 2ø
t = ln = ln Þ A reacted = 100 – 0.3906 = 99.6%
k æ pt - p0 ö k 2 p0 - pt + p0
p0 - ç ÷
è 2 ø
Chemical Kinetics 181

44. Equilibrium constant is related to temperature Now, DG ° = - RT ln K eq = - 2500 ln(4 e2 )


DH
log K p = constant - = - 2500 (ln 4 + ln e2 ) = - 2500 (1.4 + 2)
2.3 RT
Þ Plot of log K p vs 1/ T will be a straight line. = - 2500 ´ 3.4 = - 8500 J/mol
For the first order reaction X ® P Absolute value = 8500 J/mol
[ X ]0 kt kt 48. Living plants maintain an equilibrium between the absorption of
log = Þ log [ X ] = log [ X 0 ] - ,
[ X ] 2.3 2.3 C14 (produced due to cosmic radiation) and the rate of decay of
C14 present inside the plant. This gives a constant amount of C14
i.e. log [ X ] vs ‘t’ will give a straight line.
per gram of carbon in a living plant.
Also at constant temperature, pV = constant
49. Fossil whose age is closest to half-life of C-14 (5770 yr) will
1
Þ Plot of p vs will give a straight line. yield the most accurate age by C-14 dating.
V N
50. lT = ln 0

w
45. For a first order reaction : N
1 where N 0 = Number of C14 in the living matter and
kt = ln where, a = degree of dissociation. N = Number of C14 in fossil. Due to nuclear explosion,
1- a

Flo
amount of C14 in the near by area increases. This will increase
Þ 1 - a = e- kt Þ a = 1 - e- kt N 0 because living plants are still taking C-14 from atmosphere,
ekt during photosynthesis, but N will not change because fossil will

ree
1
Also = , i.e. plot of reciprocal of concentration of not be doing photosynthesis.
[ A ] [ A ]0
Þ T (age) determined in the area where nuclear explosion has
reactant vs time will be exponential.

F
occurred will be greater than the same determined in normal
1 100 2 ln 2 area.
Time for 75% = ln = = 2 (t1/ 2 )
k 100 - 75 k C C 1 C

or
Also, lT1 = ln 1 Þ lT2 = ln 2 Þ T1 - T2 = = ln 1
The Arrhenius equation is :
ur C C l C2

f
ln k = ln A -
Ea C = Concentration of C-14 in fossil.
RT
ks
The dimensions of k and A must be same. For first order 51. k = A e- Ea / RT : At T = ¥ , k = A
Yo
reaction, dimensions of k is t - 1. 1 d [ H2 ] 1 d [ NH3 ]
oo
52. - =
46. A catalyst lowers the activation energy by enabling the reaction 3 dt 2 dt
- d [ H2 ] 3 d [ NH3 ] 3
eB

to continue through an alternative path, i.e. catalyst changes the Þ = = ´ 0.001 = 0.0015 kg h - 1 .
reaction mechanism. However, catalyst does not affect either dt 2 dt 2
average kinetic energies of reactants or the collision frequency.
53. acidic, first or basic, second.
r

47. For the reaction, 54. Rate is directly proportional to concentration of reactants.
ou
ad

A (g ) + B (g ) - AB(g )
Given Ea b = Ea f + 2RT or Ea b - Ea f = 2RT 55. R µ [Reactant]
Y

Further On doubling the concentration of reactant, rate would be double.


Af [ A ]0
A f = 4 Ab or =4 56. For a first order process kt = ln
nd

[A]
Re

Ab
Now, rate constant for forward reaction, where, [ A ]0 = initial concentration.
Fi

- E a f / RT [ A ] = concentration of reactant remaining at time “t”.


k f = Af e
[ A ]0
Likewise, rate constant for backward reaction, Þ kt1/ 8 = ln = ln 8 …(i)
[ A ]0 / 8
- E ab / RT
k b = Ab e [ A ]0
and kt1/ 10 = ln = ln 10 …(ii)
At equilibrium, [ A ]0 /10
Rate of forward reaction = Rate of backward reaction t1/ 8 ln 8
Therefore, = = log 8 = 3 log 2 = 3 ´ 0.3 = 0.9
kf t1/ 10 ln 10
i.e., k f = k b or = k eq
kb t1/ 8
Þ ´ 10 = 0.9 ´ 10 = 9
- E a f / RT t1/ 10
Af e Af - (E a f - E ab ) / RT
so k eq = - E ab / RT
= e
Ab e Ab 57. Rate of reaction is constant with time.
After putting the given values 58. (a) Partial pressure becomes half of initial in every 100 min,
Af therefore, order = 1.
k eq = 4 e2 (as Ea b - Ea f = 2RT and = 4) 800
Ab (b) k ´ 100 = ln = ln 2 Þ k = 6.93 ´ 10-3 min - 1
400
182 Chemical Kinetics

(c) For 75% reaction; time required = 2 ´ half-life = 200 min 62. k 500 = A e- E1 / RT1
(d) 2 X (g ) ¾® 3Y (g ) + 2 Z (g ) k 400 = A e- E 2 / RT2
3
800 - x x x
2 Q k 500 = k 400
3 E1 E E2 T2 400 4
Total pressure = 800 + x Þ = 2 Þ = = =
2 RT1 RT2 E1 T1 500 5
Also 800 - x = 700 Þ x = 100 Also E1 = E2 + 20000 J
3 E1 - 20,000 4
Þ Total pressure = 800 + ´ 100 = 950 mm Hg Þ = Þ E1 = 100,000 J = 100 kJ mol -1
2 E1 5
a
(Rate)1 0.05 1 æ 1 ö [ A ]0
59. = = = ç ÷ Þ a = 1; order w.r.t A. 63. kt = ln
(Rate)2 0.10 2 è 2 ø [A]

w
Order w.r.t B = 0 1
Þ 4.5 ´ 10- 3 ´ 60 = ln Þ [ A ] = 0.76 M
(a) Rate = k [ A ] [A]
Rate 0.05 Þ Rate = k [ A ] = 4.5 ´ 10- 3 ´ 0.76
(b) k = = = 0.5 s- 1

Flo
[ A ] 0.10 = 3.42 ´ 10-3 mol L-1 min -1

60. 29 Cu
64
¾®
38%
- 1b
0
+ 30Zn
64 k 2 Ea æ T2 - T1 ö
= ç ÷

ree
64. (i) ln
R çè T1T2 ÷ø
k1
k1
64 19% 0 64
29 Cu ¾® + 1b + 28 Ni æ 4.5 ´ 107 ö Ea æ 50 ö
Þ ln çç ÷= çç ÷÷

F
k2

64 0 43% 64
è 1.5 ´ 10 ø 8.314 è 323 ´ 373 ø
29 Cu + - 1e ¾® 28 Ni
k3 Þ Ea = 22 kJ

or
ur
Above are the parallel reactions occurring from Cu . 64
Also
Ea
ln k = ln A -

f
k 1 38 T k 1 38 T3 RT
= = 2 = 2 and = = 7 22 ´ 1000
k 2 19 k 3 43 T1 At 50°C : ln A = ln (1.5 ´ 10 ) - = 8.33
ks
T1
8.314 ´ 323
Yo
T1 , T2 and T3 are the corresponding partial half-lives.
Þ A = 4.15 ´ 103 s- 1
oo
Also k = k1 + k2 + k3
1
ln 2 ln 2 ln 2 ln 3 (ii) N2O5 (g ) ¾® 2NO2 (g ) + O2 (g )
eB

Þ = + + 2
T T1 T2 T3
600 - p 2p p/ 2
1 1 1 1 1 1 43
Þ = + + = + + 3
T T1 T2 T3 T1 2T1 38T1 Total pressure = 960 = 600 + p Þ p = 240 mm
r

2
ou

1 æ 1 43 ö Þ Partial pressure of N2O5 (g ) remaining = 600 – 240


ad

= ç1 + + ÷
T1 è 2 38 ø = 360 mm
Y

1 æ 38 + 19 + 43 ö 100 360
= ç ÷= Þ Mole fraction = = 0.375
T1 è 38 ø 38T1 960
nd
Re

100T 100 65. (i) The Arrhenius equation is


Þ T1 = = ´ 12.8 = 33.68 h Ea
38 38 log k = log A -
Fi

2.303 RT
T2 = 2T1 = 67.36 h
38T1 38 ´ 33.68 Comparing with the given equation :
T3 = = = 29.76 h Ea
43 43 1.25 ´ 104 = Þ Ea = 239.33 kJ mol -1
2.303 R
61. R = k [A] (ii) When half-life = 256 min,
Þ R1 = k [ A ]1 and R2 = k [ A ]2 ln 2 0.693 - 1
k= = s = 4.5 ´ 10- 5 s- 1
Þ
R1 4 [ A ]1
= = t1/ 2 256 ´ 60
R2 3 [ A ]2 1.25 ´ 104
Þ = 14.34 – log 4.5 ´ 10- 5 = 16.68
[ A ]1 4 T
Also k (t2 - t1 ) = ln = ln
[ A ]2 3 1.25 ´ 104
Þ T = = 669 K
ln 2 4 16.68
Þ ´ 10 = ln
t1/ 2 3 w
66. k t = ln 0
10 log 3 3 w
Þ t1/ 2 = = = 25 min
log 4 - log 3 0.6 - 0.48 ln 2 10- 6 g
Þ ´ 20 = ln Þ w = 6.1 ´ 10- 7 g
28.1 w
Chemical Kinetics 183

67. For 1st order reaction : ì k (40° C) ü Ea æ 15 ö


1
70. ln í ý= çç ÷÷
kµ î k (25 ° C ) þ R è 298 ´ 313 ø
t1/ 2 70 ´ 1000 15
= ´ = 1.35
ì k (450° C) ü ì t1/ 2 (380° C) ü Ea æ 450 - 380 ö 8.314 298 ´ 313
Þ ln í ý = ln í ý= çç ÷÷
î k (380° C) þ î t1/ 2 (450° C) þ R è 727 ´ 653 ø k (40° C)
Þ = 3.87
ì 360 ü 200 ´ 103 70 k (25° C)
Þ ln í ý= ´ = 3.54
î t (450 ° C) þ 8.314 727 ´ 653 1 100 1 4
1/ 2 Also k (25°C) = ln = ln
Þ t1/ 2 (450°C) =10.37 min 20 75 20 3
Þ Time for 75% reaction at 450°C Þ k (40° C) = 3.87 ´ k (25°C)
1 4
= 2 ´ t1/ 2 = 2 ´ 10.37 = 20.74 min = 3.87 ´ ln = 55.66 ´ 10- 3 min - 1

w
20 3
68. Comparing the data of experiment number 2 and 3 : 100
Now k (40°C) ´ 20 = ln
R3 1.6 ´ 10- 2 æ 1.0 ´ 10- 3 ö
m
100 - x
= = çç ÷
-4 ÷ 100
R2 4 ´ 10- 3 è 5 ´ 10 ø Þ -3
55.66 ´ 10 ´ 20 = ln Þ x = 67%

Flo
100 - x
Þ m = 2, order w.r.t. A
k
Now comparing the data of experiment number 1 and 2 : 71. (i) A ¾®
A
Product

ree
2 n
R2 4 ´ 10-3 æ 5 ´10- 4 ö æ 6.0 ´ 10- 5 ö kB
(ii) B ¾® Product
= =ç ÷ ç ÷
R1 5 ´ 10- 4 çè 2.5 ´ 10- 4 ÷ø ç 3.0 ´ 10-5 ÷
è ø

F
Ea æ 10 ö
Þ 8 = (2)2 (2)n Þ n = 1, order w.r.t. B. For (i) çç ÷÷ = ln 2
R è 300 ´ 310 ø
(i) Order with respect to A = 2, order with respect to B = 1.

or
ur Þ Ea (i) = 9300 R ln 2 = 53.6 kJ
(ii) At 300 K, R = k [ A ]2 [ B ] E (i )

f
Þ Ea (ii) = a = 26.8 kJ
R 5.0 ´ 10- 4 2
Þ k= =
ks
[ A ] [ B ] (2.5 ´ 10- 4 )2 (3.0 ´ 10- 5 )
2 At 310 K t1/ 2 (i) = 30 min
Yo
Q Rate of (ii) = 2 rate of (i)
= 2.66 ´ 108 s- 1 L2 mol -2
oo
Þ t1/ 2 (ii) = 15 min
(iii) From first experiment :
Now for reaction (ii) :
eB

Rate (320 K) = k (320 K) (2.5 ´ 10- 4 )2 (3.0 ´ 10- 5 )


ì k (310) ü ì t1/ 2 (300) ü Ea (ii ) æ 10 ö
ln í B ý = ln í ý= çç ÷
Þ k (320 K) =
2 ´ 10-3
î k B (300) þ î t1/ 2 (310) þ R è 300 ´ 310 ÷ø
(2.5 ´ 10- 4 )2 (3.0 ´ 10- 5 )
r

ì t (300) ü ln 2
Þ ln í 1/ 2 ý= Þ t1/ 2 (300) = 21.2 min
ou

= 1.066 ´ 109 s- 1 L2 mol - 2.


ad

î 15 þ 2
ì k (320 K) ü Ea æ T2 - T1 ö ln 2 0.693
Þ k B (300) = = = 3.26 ´ 10- 2 min - 1
Y

Þ ln í ý= çç ÷÷
î k (300 K) þ R è T1T2 ø t1/ 2 21.2

æ 1.066 ´ 109 ö Ea æ 20 ö 72. Initially :


nd

Þ ln çç ÷= çç ÷÷
Re

8 ÷
10 20
è 2.66 ´ 10 ø 8.314 è 300 ´ 320 ø N (1 H3 + 1H1 ) = ´ 2 ´ 6 ´ 1023 = ´ 1023
8 3
Ea = 55.42 kJ mol -1
Fi

Þ N (1 H1 ) 20 ´ 1023
Þ 1+ =
Ea N (1 H3 ) 3N (1 H3 )
Now ln k = ln A -
RT 1 20 ´1023
3 Þ 1+ = » 1.25 ´ 1017
55.42 ´ 10 8 ´ 10- 18 3N (1 H3 )
At 300 K : ln (2.66 ´ 108 ) = ln A -
8.314 ´ 300 20 ´ 1023
18 Þ N (1 H 3 ) = = 5.33 ´ 106
Solving : ln A = 41.62 Þ A = 1.2 ´ 10 3 ´ 1.25 ´ 1017
69. CH3 ¾ O ¾ CH3 (g ) ¾® CH4 (g ) + H2 (g ) + CO (g ) N0 ln 2 5.33 ´ 106
Þ kt = ln Þ ´ 40 = ln
At 12 min : 0.40 - p p p p
N 12.3 N
Total pressure = 0.4 + 2p Þ N = 5.6 ´ 105
0.40 ln 2 73. For the reaction :
Also k ´ 12 = ln = ´ 12 = 1.77 Þ p = 0.175
0.40 - p 14.5 2N2O5 ¾® 4NO2 + O2
Þ Total pressure = 0.4 + 2p = 0.4 + 2 ´ 0.175 = 0.75 atm If p0 is the initial pressure, the total pressure after completion of
5
reaction would be p0.
2
184 Chemical Kinetics

5 100
Þ 584.5 = p0 Þ p0 = 233.8 mm Þ = 1.055
2 100 - x
Let the pressure of N2O5 decreases by ‘p’ amount after 30 min. Þ x = 5.25% reactant is converted into product.
Therefore, ln 2 0.693
Half-life = = = 462000 s = 128.33 h
2N2O5 ¾® 4NO2 + O2 k 1.5 ´ 10- 6
p
At 30 min : p0 - p 2p
2 [ A ]0
77. For a first order process : ln = kt Þ ln [ A ] = ln [ A ]0 - kt
3 [A]
Total pressure = p0 + p = 284.5
2 If the reactant is in gaseous state
2 [ A ]0
Þ p = (284.5 - 233.8) = 33.8 and kt1/ 10 - ln = ln 10 …(ii)
3 [ A ]0 / 10
p0

w
Now, kt = ln t ln 8
p0 - p Therefore, 1/ 8 = = log 8 = 3 log 2 = 3 ´ 0.3 = 0.9
t1/ 10 ln 10
1 233.8
Þ k= ln min - 1 = 5.2 ´ 10- 3 min - 1 t1/ 8
30 233.8 - 33.8 Þ ´ 10 = 0.9 ´ 10 = 9

Flo
t1/ 10
74. Arrhenius equation is : ln p = ln p0 - kt …(i)
Ea where p is the partial pressure of reactant remaining unreacted at
log k = log A -

ree
2.303 RT instant ‘t’ and p0 is its initial partial pressure.
ln 2 0.693 Also, from equation (i), ln p vs t would give a straight line.
when t1/ 2 = 10 min, k = = = 1.115 ´ 10- 3 s- 1

F
t1/ 2 10 ´ 60 Therefore, decomposition of N2O5 following first order kinetics.
ln 2 0.693 - 1
Þ
Ea
= log A - log k 78. k = = yr = 1.2 ´ 10- 4 yr - 1

or
2.303 RT
ur t1/ 2 5770
1 ln 2 1
4 ´ 1013 Also kt = ln = ´ 11540 = ln 4 Þ f = = 0.25

f
A
= log = log = 16.54 f 5770 4
k 1.115 ´10- 3
ks
Ea 98.6 ´ 1000 79. For a first order reaction,
Þ T = =
Yo
2.303 R ´ 16.54 2.303 ´ 16.54 ´ 8.314 [ A ]0
oo
kt = ln
= 311.34 K [A]
eB

75. The minimum rate of decay required after 6.909 h is where [ A ]0 = Initial concentration of reactant
346 particles min . -1 [ A ] = Concentration of reactant remaining
unreacted at time t.
Þ Rate = kN
r

1 [ A ]0 1 100 1 5
Rate 346 ´ 66.6 ´ 60 (i) Þ k = ln = ln = ln
ou

Þ N = = = 1.995 ´ 106 atoms [ A ] 10 100 - 20 10


ad

t 4
k 0.693
N ln 2 N 2.303 (log 5 - 2 log 2) -1
= min = 0.023 min –1
Y

Þ kt = ln 0 Þ ´ 6.909 = ln 0 = 0.0715
N 66.6 N 10
N0 1 100 2 ln 2 2 ´ 0.693
Þ = 1.074 (ii) t = ln = = = 60 min
nd

25 0.023
Re

N k k
Þ N 0 = 1.074 ´ N = 1.074 ´ 1.995 ´ 106 80. Looking at the rate data of experiment number 1 and 2 indicates
Fi

6
= 2.14 ´ 10 atoms of Mo that rate is doubled on doubling concentration of A while
concentration of B is constant. Therefore, order with respect to A
2.14 ´ 106 is 1. Similarly, comparing data of experiment number 1 and 3,
Þ Mass of Mo required = ´ 99 = 3.56 ´ 10- 16 g
6.023 ´ 1023 doubling concentration of B, while concentration of A is
constant, has no effect on rate.
76. k = 1.5 ´ 10- 6 s- 1 Therefore, order with respect to B is zero.
100 Þ Rate = k [ A ]
kt = ln
100 - x 0.005 0.693
Þ k= = 0.5 min - 1 =
100 0.010 t1/ 2
Þ ln = 1.5 ´ 10- 6 s- 1 ´ 10 ´ 60 ´ 60 s = 0.0054
100 - x 0.693
Þ t1/ 2 = = 1.386 min
0.5

Download Chapter Test


http://tinyurl.com/yyxaf3dv or
12
Nuclear Chemistry

w
Objective Questions I (Only one correct option) Objective Questions II
1. Bombardment of aluminium by a-particle leads to its artificial (One or more than one correct option)

Flo
disintegration in two ways, (i) and (ii) as shown. Products X , Y 9. A plot of the number of neutrons (n) against the number of
and Z respectively, are (2011)
protons (p) of stable nuclei exhibits upward deviation from
30 linearity for atomic number, Z > 20. For an unstable nucleus

ree
27 (ii)
13Al 15P +Y
having n/p ratio less than 1, the possible mode(s) of decay is
(i) (are) (2016 Adv.)

F
(a) b - - decay (b - emission) (b) orbital or K-electron capture
30 30 (c) neutron emission (d) b + -decay (positron emission)
14Si +X 14Si +Z

or
ur 10. In the nuclear transmutation, 94 Be + X ¾¾® 84 Be + Y
(a) proton, neutron, positron (b) neutron, positron, proton X and Y are

f
(2013 Adv.)
(c) proton, positron, neutron (d) positron, proton, neutron (a) (g , n) (b) ( p, D) (c) (n, D) (d) (g , p)
ks
23
2. A positron is emitted from The ratio of the atomic mass
11 Na. 11. Decrease in atomic number is observed during (1998, 2M)
Yo
and atomic number of the resulting nuclide is (2007, 3M) (a) alpha emission (b) beta emission
oo
(a) 22/10 (b) 22/11 (c) 23/10 (d) 23/12 (c) positron emission (d) electron capture
23
eB

3. Na is the more stable isotope of Na. Find out the process by 12. The nuclear reactions accompanied with emission of neutron(s)
which 24
11 Na can undergo radioactive decay. (2003, 1M) are (1988, 1 M)
(a) b - -emission (b) a-emission (a) 27 4
¾® 30
(b) 12 1
¾® 13
13 Al + 2He 15P 6 C + 1H 7N
(c) b + -emission
r

(d) K-electron capture 30 30 0 241 4 244


(c) 15 P ¾® 14Si + (d) 96 Cm + 2He ¾® 97Bk + 01e
ou

1e
ad

139
4. The number of neutrons accompanying the formation of 54 Xe
and 94
38 Sr from the absorption of a slow neutron by
235
92 U,
Y

Assertion and Reason


followed by nuclear fission is (1999, 2M)
(a) 0 (b) 2 (c) 1 (d) 3 Read the following questions and answer as per the direction
nd

given below:
Re

27 29
5. 13 Al is a stable isotope. 13 Al is expected to decay by
(a) Statement I is true; Statement II is true; Statement II is
(a) a-emission (b) b-emission (1996, 1M)
Fi

the correct explanation of Statement I


(c) positron emission (d) proton emission
(b) Statement I is true; Statement II is true; Statement II is
6. The radiation from a naturally occurring radioactive substance, not the correct explanation of Statement I
as seen after deflection by a magnet in one direction, are
(c) Statement I is true; Statement II is false
(1984, 1M)
(a) definitely alpha rays (b) definitely beta rays (d) Statement I is false; Statement II is true
(c) both alpha and beta rays (d) either alpha rays or beta rays 13. Statement I The plot of atomic number ( y-axis) versus number
of neutrons (x-axis) for stable nuclei shows a curvature towards
7. An isotope of Ge76
32 is (1984, 1M) x-axis from the line of 45° slope as the atomic number is
(a) Ge77
32 (b) As77
33 (c) Se77
34 (d) Se78
34 increased.
8. If uranium (mass number 238 and atomic number 92) emits an Statement II Proton-proton electrostatic repulsions begin to
a-particle, the product has mass number and atomic number overcome attractive forces involving protons and neutrons and
(1981, 1 M) neutrons in heavier nuclides. (2008)

(a) 236 and 92 (b) 234 and 90 14. Statement I Nuclide 30 40


13 Al is less stable than 20 Ca.
(c) 238 and 90 (d) 236 and 90 Statement II Nuclides having odd number of protons and
neutrons are generally unstable. (1998)
186 Nuclear Chemistry

235
Fill in the Blanks 21. The number of neutrons emitted when 92 U undergoes
142 90
15. (a) 235 1
+ 0n ¾® 137
+ 97
+ ...... controlled nuclear fission to 54 Xe and 38 Sr is
92 U 52 A 40 B
(2010)
82 0
(b) 34 Se ¾® 2 - 1e + ...... (2005, 1M ´ 2 = 2M) Subjective Questions
16. A radioactive nucleus decays by emitting one alpha and two beta 22. The total number of a and b particles emitted in the nuclear
particles, the daughter nucleus is.... of the parent. (1989, 1M) reaction 238
¾® 214
is ..... .
92 U 82Pb (2009)
14
17. The number of neutrons in the parent nucleus which gives N -7a
234
on beta emission is ........... (1985, 1M) 23. 92 X ¾¾® Y. Find out atomic number, mass number of Y
-6b
18. Elements of the same mass number but different atomic number and identify it. (2004)
are known as ……… (1983, 1M) 238
is radioactive and it emits a and b particles to form
24. 92 U

w
19. An element Z M A undergoes an a-emission followed by two 82 Pb
206
. Calculate the number of a and b particles emitted in this
successive b -emissions. The element formed is …… conversion.
(1982, 1M) 238 238 206
An ore of 92 U is found to contain 92 U and 82 Pb in the

Flo
238
Integer Answer Type Questions weight ratio of 1 : 0.1. The half-life period of 92 U is 4.5 ´ 109
yr. Calculate the age of the ore. (2000)

ree
20. The periodic table consists of 18 groups. An isotope of copper, 14
25. Write a balanced equation for the reaction of N with
on bombardment with protons, undergoes a nuclear reaction
a-particle.

F
yielding element X as shown below. To which group, element 234
26. 90 Th disintegrates to give 82 Pb206 as the final product. How
X belongs in the periodic table? (2012)
63
many alpha and beta particles are emitted during this process?
29 Cu + 11 H ¾® 610 n + 42a + 211 H + X

or
ur (1986, 2M)

Answers
f
ks
A-4
1. (a) 2. (c) 3. (a) 4. (b) 16. isotope 17. eight 18. isobars 19. ZM
Yo
5. (b) 6. (c) 7. (a) 8. (b) 20. (8) 21. 3 22. (8) 23. 206
84 PO
oo
9. (b, d) 10. (a, b) 11. (a, c, d) 12. (a, d)
24. (7.12 ´ 10 8yr) 26. (13)
82
15. 2 0n1,
eB

13. (a) 14. (b) 36 Kr

Hints & Solutions


r
ou
ad

27
1. (i) 13 Al + 42He (a) ¾® 30
14Si + 11X 6. Both a-rays and b - rays are deflected by magnetic field.
1
Y

X is proton 1H. 7. Isotopes have same atomic number (Z ) but different mass
27
(ii) 13 Al + 42He (a ) ¾® 30
15P + 10Y number ( A ). Therefore, 32 Ge
76
and 32 Ge
77
are isotopes.
1
nd

Y is neutron, 0 n.
Re

8. The nuclear reaction is


30 30
15 P ¾® 14Si + 10Z 238
¾® 2He4 (a ) + 234
92 U 90Th
Fi

0
Z is positron , +1e. 9. For the elements with atomic number (Z ) larger than 20,
2. The required nuclear reaction is Neutrons (n) > Protons (p); Thus, n/ p > 1
23 0 23
11 Na ¾® +1e + 10Na Thus, there is upward deviation from linearity.
3. In stable isotope of Na, there are 11 protons and 12 neutrons. In If n < p, Thus n/ p < 1, then
the given radioactive isotope of sodium (Na 24 ), there are 13 (a) By b - - decay, 10 n ¾® 11 p + -01e neutron changes to
neutrons, one neutron more than that required for stability. A proton. Thus, (n/ p) ratio further decreases below 1.
neutron rich isotope always decay by b-emission as Thus, this decay is not allowed.
1 0
0n ¾® -1b + 1H1 (b) By orbital or K- electron capture, 11 p + 0
-1e ¾® 10n
4. The balanced nuclear reaction is proton changes to neutron, hence, (n/ p) ratio increases.
1 235 139 94 Thus stability increases. Thus correct.
20 n + 92U ¾® 54Xe + 38 Sr
(c) Neutron emission further decreases n/p ratio.
29
5. 13 Al is neutron rich isotope, will decay by b-emission (d) By b + -emission, 11 p ¾® 10n + +01e
converting some of its neutron into proton as
1 0 1 proton changes to neutron.
0 n ¾® -1b + 1H
Hence, n/ p ratio increases. Thus correct.
Nuclear Chemistry 187

electrostatic repulsion, curve of stability incline towards


120 neutron axis.
Neutron-rich
100 nuclei 1.5 14. Upto atomic number of 20, stable nuclei possess neutron to
Number of neutrons

proton ratio (n/ p) = 1.


80 1.4 n 17
(13 Al 30 ) = = 1.3 > 1, unstable, b-emitters.
p 13
60 1.2 n 20
( Ca 40 ) = = 1, stable.
40 p 20 20
1.1 ratio Also, nuclei with both neutrons and protons odd are usually
20
Neutron poor nuclei unstable but it does not explain the assertion appropriately.
235
0 15. (a) 92 U + 0n1 ¾® 137
+ 97
+ 2 0n1

w
52 A 40 B
20 40 60 80 82
Number of protons (b) 34 Se ¾® 2 -1e0 + 36Kr
82

Plot of the number of neutrons against the number of protons in 16. Isotope : Z X A ¾® 2He4 + 2 -1e0 + ZY A - 4

Flo
stable nuclei (shown by dots). 17. 8 : 6 C14 ¾® 7 N14 + 0
-1e
10. PLAN 9
4 Be + b
aX ¾® 4 Be + c Y
8 d
18. Isobars have same mass number but different atomic number.

ree
A-4
Atomic number same 19. ZM : Z M A ¾® 2He4 + 2 -1e0 + ZM A - 4
4 + a=4 + c

F
20. Balancing the given nuclear reaction in terms of atomic number
9+ b=8+ d (charge) and mass number:
a=0

or
63
If X = 0
0g + 1 H1 ¾® 60 n1 + 2 He4 (a ) + 21 H1 + 26 X 52
b=0
ur 29 Cu

The atomic number 26 corresponds to transition metal Fe which

f
c=0
Y = 10 n belongs to 8th group of modern periodic table.
d =1
ks
235 142 90
21. 92 U ¾® 54Xe + 38 Sr + 30 n1
a =1
Yo
If X = 11 p 238
¾® 214
+ 6 2He4 + 2 0
b =1 22. 92 U 82Pb
oo
-1e

c =1 Þ Number of (a + b ) = 6 + 2 = 8
Y = 12 D
eB

d =2 23. 92 X
234
¾® 7 2He4 + 6 -1e0 + 84Y
206

9 206
4 Be + 00 g ¾® 84 Be + 10n Y is 84 Po .
9 1 8 2
+ ¾® +
r

238 206
4 Be 1p 4 Be 1D 24. 92 U ¾® 82Pb + 8 2He4 + 6-1 e0
ou
ad

11. In the following nuclear reactions, there occur decrease in Present : N 0 - N N


atomic number (Z ) w (U) 1
Y

A- 4
Given, = = 10
ZX
A
¾® 2He4 + Z - 2Y , a - emission w (Pb) 0.1
A
¾® 0
+ A
, positron emission N (U) 10 206 N 0 - N
ZX +1e Y Þ = ´ =
nd

Z-1
Re

0 N (Pb) 238 1 N
ZX
A
+ -1e ¾® A
Z - 1Y , electron capture
N 238 N0 238 2298
In beta emission, increase in atomic number is observed. Þ = Þ =1+ =
Fi

0
N 0 - N 2060 N0 - N 2060 2060
ZX
A
¾® -1e + Z + 1Y , b -emission
A
Now, applying first order rate law
12. If sum of mass number of product nuclides is less than the sum æ ln 2 ö æ N0 ö (t ) æ N0 ö
çç ÷÷ t = ln çç ÷÷ Þ t = 1/ 2 log çç ÷÷
of parent nuclides, then neutron emission will occur. In both (a) -
è 1/ 2 ø
t è 0
N N ø log 2 èN0 - N ø
and (d), sum of mass number of product nuclides is one unit less
than the sum of parent nuclides, neutron emission will balance 4.5 ´ 109 2298
= log = 7.12 ´ 108 yr
the mass number. 0.3 2060
14
13. After atomic number 20, proton-proton repulsion increases 25. 7N + 2He4 ¾® 9F
18

immensely, more neutrons are required to shield this 234


¾® 206
+ 7 2He4 + 6 -1e0
26. 90 Th 82Pb

Download Chapter Test


http://tinyurl.com/y3qvab9f or
13
Surface Chemistry

w
Objective Questions I (Only one correct option) (a) (A)-(ii), (B)-(iii), (C)-(i), (D)-(iv)

Flo
1. Among the following, the incorrect statement about colloids (b) (A)-(iv), (B)-(iii), (C)-(ii), (D)-(i)
is (2019 Main, 12 April II) (c) (A)-(iii), (B)-(i), (C)-(ii), (D)-(iv)

ree
(a) They can scatter light (d) (A)-(iii), (B)-(iv), (C)-(i), (D)-(ii)
(b) They are larger than small molecules and have high molar mass 6. The number of water molecule(s) not coordinated to copper
(c) The osmotic pressure of a colloidal solution is of higher order ion directly in CuSO4 × 5H 2O, is

F
(2019 Main, 9 April I)
than the true solution at the same concentration (a) 2 (b) 3 (c) 1 (d) 4
(d) The range of diameters of colloidal particles is between 1 and
7. The aerosol is a kind of colloid in which (2019 Main, 9 April I)

or
1000 nm
ur (a) gas is dispersed in liquid (b) gas is dispersed in solid
2. Peptisation is a (2019 Main, 12 April I) (c) liquid is dispersed in water (d) solid is dispersed in gas

f
(a) process of bringing colloidal molecule into solution
8. Adsorption of a gas follows Freundlich adsorption isotherm.
ks
(b) process of converting precipitate into colloidal solution
x is the mass of the gas adsorbed on mass m of the adsorbent.
Yo
(c) process of converting a colloidal solution into precipitate
x x
oo
(d) process of converting soluble particles to form colloidal solution The plot of log versus log p is shown in the given graph.
m m
3. The correct option among the following is is proportional to
eB

(2019 Main, 8 April I)


(2019 Main, 10 April II)
(a) colloidal medicines are more effective, because they have
small surface area.
r

(b) brownian motion in colloidal solution is faster if the viscosity x


log —
ou

2
ad

of the solution is very high. m


(c) addition of alum to water makes it unfit for drinking. 3
Y

(d) colloidal particles in lyophobic sols can be precipitated by


electrophoresis. log p
4. A gas undergoes physical adsorption on a surface and
nd

2/3 3/ 2
(a) p (b) p (c) p3 (d) p2
Re

follows the given Freundlich adsorption isotherm equation


x 9. Among the following, the false statement is
Fi

= Kp 0. 5 (2019 Main, 12 Jan II)


m (a) Tyndall effect can be used to distinguish between a colloidal
Adsorption of the gas increases with (2019 Main, 10 April I) solution and a true solution
(a) increase in p and increase in T (b) It is possible to cause artificial rain by throwing electrified
(b) increase in p and decrease in T sand carrying charge opposite to the one on clouds from an
(c) decrease in p and decrease in T aeroplane
(d) decrease in p and increase in T (c) Lyophilic sol can be coagulated by adding an electrolyte
5. Match the catalysts Column I with products Column II. (d) Latex is a colloidal solution of rubber particles which are
(2019 Main, 9 April I) positively charged
10. Given, Gas : H2 , CH4 , CO2 , SO2
Column I (Catalyst) Column II (Product)
(A) V2O5 (i) Polyethlyene Critical temperature/K 33 190 304 630
On the basis of data given above, predict which of the
(B) TiCl 4 / Al(Me)3 (ii) Ethanal
following gases shows least adsorption on a definite amount
(C) PbCl 2 (iii) H2SO4 of charcoal? (2019 Main, 12 Jan I)
(D) Iron oxide (iv) NH3 (a) CH4 (b) SO2 (c) CO2 (d) H2
Surface Chemistry 189

11. Among the colloids cheese (C), milk (M) and smoke (S), the 2. The diameter of the dispersed particle is not much
correct combination of the dispersed phase and dispersion smaller than the wavelength of the light used.
medium, respectively is (2019 Main, 11 Jan II) 3. The refractive indices of the dispersed phase and
(a) C : liquid in solid; M : liquid in liquid; S : solid in gas dispersion medium are almost similar in magnitude.
(b) C : solid in liquid; M : liquid in liquid; S : gas in solid 4. The refractive indices of the dispersed phase and
(c) C : liquid in solid; M : liquid in solid; S : solid in gas dispersion medium differ greatly in magnitude.
(d) C : solid in liquid; M : solid in liquid; S : solid in gas (a) 1 and 4 (b) 2 and 4 (c) 1 and 3 (d) 2 and 3
12. An example of solid sol is (2019 Main, 11 Jan I) 19. For a linear plot of log ( x / m ) versus log p in a Freundlich
(a) gem stones (b) hair cream
adsorption isotherm, which of the following statements is
(c) butter (d) paint
correct? (k and n are constants) (2016 Main)
13. Haemoglobin and gold sol are examples of (a) 1/n appears as the intercept

w
(2019 Main, 10 Jan II)
(a) negatively and positively charged sols, respectively (b) Only 1/n appears as the slope
(b) negatively charged sols æ1ö
(c) log ç ÷ appears as the intercept
(c) positively charged sols ènø

Flo
(d) positively and negatively charged sols, respectively (d) Both k and 1/n appear in the slope term
14. Which of the following is not an example of heterogeneous 20. Methylene blue, from its aqueous solution, is adsorbed on

ree
catalytic reaction? (2019 Main, 10 Jan I) activated charcoal at 25°C. For this process, the correct
(a) Haber’s process statement is (2013 Adv.)
(a) the adsorption requires activation at 25°C

F
(b) Combustion of coal
(c) Hydrogenation of vegetable oils (b) the adsorption is accompanied by a decreases in enthalpy
(d) Ostwald’s process (c) the adsorption increases with increase of temperature

or
ur
15. The correct match between item-I and Item-II is (d) the adsorption is irreversible
21. The coagulating power of electrolytes having ions Na + , Al 3+

f
(2019 Main, 9 Jan II)
A. Benzaldehyde P. Dynamic phase and Ba 2+ for arsenic sulphide sol increases in the order
ks
(2013 Main)
B. Alumina Q. Adsorbent
(a) Al 3+ < Ba 2+ < Na +
Yo
(b) Na + < Ba 2+ < Al 3+
C. Acetonitrile R. Adsorbate
oo
(c) Ba 2+ < Na 2+ < Al 3+ (d) Al 3+ < Na + < Ba 2+
(a) (A) ® (R) ; (B) ® (Q); (C) ® (P)
(b) (A) ® (P); (B) ® (R); (C) ® (Q) 22. Among the electrolytes Na 2 SO4 , CaCl 2 , Al 2 (SO4 )3 and
eB

(c) (A) ® (Q); (B) ® (P); (C) ® (R) NH4 Cl, the most effective coagulating agent for Sb 2 S3 sol is
(d) (A) ® (Q); (B) ® (R); (C) ® (P) (2009, 1M)
(a) Na 2SO4 (b) CaCl 2
r

16. Which of the salt-solution is most effective for coagulation


(c) Al 2 (SO4 )3 (d) NH4Cl
ou

of arsenious sulphide?
ad

(2019 Main, 9 Jan II)


(a) BaCl 2 (b) AlCl 3 23. Among the following, the surfactant that will form micelles
in aqueous solution at the lowest molar concentration at
Y

(c) Na 3PO4 (d) NaCl


ambient conditions, is (2008, 3M)
17. Adsorption of a gas follows Freundlich adsorption isotherm. (a) CH3 (CH2 )15 N+ (CH3 )3 Br - (b) CH3 (CH2 )11 OSO-3 Na +
In the given plot,x is the mass of the gas adsorbed on mass m
nd
Re

x (c) CH3 (CH2 )6 COO- Na + (d) CH3 (CH2 )11 N+ (CH3 )3 Br -


of the adsorbent at pressure p × is proportional to
m 24. Lyophilic sols are (2005, 1M)
Fi

(2019 Main, 9 Jan I) (a) irreversible sols


(b) prepared from inorganic compounds
(c) coagulated by adding electrolytes
2 unit (d) self-stabilising
x
log m
4 unit 25. Spontaneous adsorption of a gas on solid surface is an
exothermic process, because (2004, 1M)
(a) DH increases for system (b) DS increases for gas
log p (c) DS decreases for gas (d) DG increases for gas
26. Rate of physisorption increases with (2003, 1M)
(a) p2 (b) p1/ 4 (c) p1/ 2 (d) p
(a) decrease in temperature (b) increase in temperature
18. The Tyndall effect is observed only when following (c) decrease in pressure (d) decrease in surface area
conditions are satisfied (2017 Main) 27. When the temperature is increased, surface tension of water
1. The diameter of the dispersed particles is much smaller (2002, 1M)
than the wavelength of the light used. (a) increases (b) decreases
(c) remains constant (d) shows irregular behaviour
190 Surface Chemistry

(a) I is physisorption and II is chemisorption


Objective Questions II
(b) I is physisorption and III is chemisorption
(One or more than one correct option) (c) IV is chemisorption and II is chemisorption
28. The correct statement(s) about surface properties is(are) (d) IV is chemisorption and III is chemisorption
(2017 Adv.)
31. Choose the correct reason(s) for the stability of the
(a) The critical temperatures of ethane and nitrogen are 563 K and
lyophobic colloidal particles. (2012)
126 K, respectively. The adsorption of ethane will be more than
(a) Preferential adsorption of ions on their surface from the
that of nitrogen of same amount of activated charcoal at a given
solution
temperature
(b) Preferential adsorption of solvent on their surface from the
(b) Cloud is an emulsion type of colloid in which liquid is dispersed
solution
phase and gas is dispersion medium
(c) Attraction between different particles having opposite
(c) Adsorption is accompanied by decrease in enthalpy and decrease

w
charges on their surface
in entropy of the system
(d) Potential difference between the fixed layer and the
(d) Brownian motion of colloidal particles does not depend on the
diffused layer of opposite charges around the colloidal
size of the particles but depends on viscosity of the solution
particles

Flo
29. When O2 is adsorbed on a metallic surface, electron transfer
32. The correct statement(s) pertaining to the adsorption of a
occurs from the metal to O 2 . The true statement(s) regarding gas on a solid surface is (are) (2011)

ree
this adsorption is (are) (2015 Adv.) (a) Adsorption is always exothermic
(a) O2 is physisorbed (b) Physisorption may transform into chemisorption at high

F
(b) heat is released temperature
(c) occupancy of p * 2 p of O2 is increased (c) Physisorption increases with increasing temperature but

or
(d) bond length of O2 is increased
ur chemisorption decreases with increasing temperature
(d) Chemisorption is more exothermic than physisorption,

f
30. The given graph/data I, II, III and IV represent general trends however it is very slow due to higher energy of activation
observed for different physisorption and chemisorption
ks
processes under mild conditions of temperature and pressure. Assertion and Reason
Yo
Which of the following choice(s) about I, II, III and IV is (are)
oo
correct? (2012)
Read the following questions and answer as per the
direction given below:
eB

p constant
p constant (a) Statement I is true; Statement II is true; Statement II is a
Amount of gas

correct explanation of Statement I.


Amount of gas
absorbed

absorbed

(I) (b) Statement I is true; Statement II is true; Statement II is


(II)
r

not the correct explanation of Statement I.


ou
ad

(c) Statement I is true; Statement II is false.


T T (d) Statement I is false; Statement II is true.
Y

33. Statement I Micelles are formed by surfactant molecules


Potential energy

above the critical micelle concentration (CMC).


nd
Amount of gas

200 K
Re absorbed

250 K Eads Statement II The conductivity of a solution having


(III) (IV) 0
Distance of molecule surfactant molecules decreases sharply at the CMC.(2007)
Fi

from the surface

p Dhads = 150 kJ mol–1

Answers
1. (c) 2. (b) 3. (d) 4. (b) 21. (b) 22. (c) 23. (a) 24. (d)
5. (c) 6. (c) 7. (d) 8. (a) 25. (c) 26. (a) 27. (b) 28. (a, c)
9. (d) 10. (d) 11. (a) 12. (a) 29. (b, c, d) 30. (a, c) 31. (a, d) 32. (a, b, d)
13. (d) 14. (b) 15. (a) 16. (b) 33. (b)
17. (c) 18. (b) 19. (b) 20. (b)
Hints & Solutions
1. Statement (c) is incorrect about colloids. Colligative Adsorption isobar (Pressure, p = constant)
properties such as relative lowering of vapour pressure,
elevation in boiling point, depression in freezing point and
osmotic pressure of a colloidal solution is of low order than
x
the true solution at the same concentration. —
m
2. Peptisation is a process of converting precipitate into
colloidal solution. This process involves the shaking of
precipitate with the dispersion medium in the presence of T
small amount of electrolyte. The electrolyte added is called

w
peptising agent. So, the rate of physical adsorption of the gas, increases with p
(when, T is constant) and decreases with T (when p is constant).
During peptisation, the precipitate adsorbs one of the ions of
the electrolyte on its surface. This causes the development of 5. Correct match is

Flo
positive or negative charge on precipitates, which ultimately (A) ® (iii); (B) ® (i); (C) ® (ii); (D) ® (iv)
breakup into smaller particles of the size of a colloid. (i) TiCl4 + AlCl3 (Ziegler- Natta catalyst) is used to prepare

ree
3. The explanation of the given statements are as follows : polyethylene from ethene.
(a) Colloidal medicines are more effective because they Zieglar-Natta
catalyst
(dispersed phase) have larger surface area.

F
n CH2 CH2 CH2 CH2 n
Thus, option (a) is incorrect. Ethene Polyethylene

(b) Brownian motion of dispersed phase particles in colloidal (ii) V2O5 (Vanadium pentoxide) is used as catalyst to prepare

or
ur
solution is faster if the viscosity of the solution is very H2SO4 from contact process. Reaction involved is

f
low. Thus, option (b) is incorrect. V 2O 5
2SO2 ( g ) + O2 ( g ) ¾¾® 2SO3 ( g )
(c) Addition of alum(K 2SO4 × Al2(SO4 )3 × 24H2O), an
ks
It is the key step in the manufacture of H2SO4.
electrolyte to water makes it fit for drinking purposes
Yo
because alum coagulates mud particles from water. (iii) Fe (Iron) is used as catalyst in Haber’s process for the
oo
Thus, option (c) is incorrect. manufacture of ammonia.
Fe( s )
eB

(d) Precipitation of lyophobic solution particles by N2 (g ) + 3H2 (g ) ¾¾® 2NH3 (g )


electrophoresis is called cottrell precipitation.
(iv) Pd (Palladium) is used to prepare ethanal. Reaction involved is
Thus, option (d) is correct.
PdCl 2/CuCl 2
H 2C == CH 2 + O2 ¾¾¾¾® CH 3CHO
r

4. For physisorption or physical adsorption, H 2O


ou
ad

Adsorption isotherm (Temperature, T = constant) is shown


This reaction is also known as Wacker’s process.
below:
Y

6. In CuSO4 × 5H2O, one molecule of water is indirectly connected to


Cu. In this molecule, four water molecules form coordinate bond
x with Cu 2+ ion while one water molecule is associated with H-bond
nd

— x
Re

m Moderate pressure zone, — µ p1/n


m with SO2-
4 .
Structure of CuSO4 × 5H2 O
Fi

p
H H
where, x = amount of adsorbate, m = amount of adsorbent,
d– d+
x O O H H O– O
= degree of adsorption H
m Cu2+ O S
H d– d+
1 1 O O H H O– O
= order of the reaction, where, 0 < < 1 and so,
n n
1< n < ¥ H H

x
1
[Cu(H 2O)4 ] SO4 × H 2O
Here, = Kp 2,
m 7. The aerosol is a kind of colloid in which solid is dispersed in gas.
x
1 e.g. smoke, dust.
i.e. µ p2
m
192 Surface Chemistry

8. (liquid in liquid). Butter is a colloidal solution of liquid in solid.


Key Idea According to Freundlich,
Paint is also sol (solid in liquid).
x
= Kp1/n [ n > 1]
m 13. Haemoglobin and gold sol both are colloids and always carry an
where, m = mass of adsorbent, x = mass of the gas electric charge. Haemoglobin is a positively charged sol,
x because in haemoglobin, Fe2+ ion is the central metal ion of the
adsorbed, = amount of gas adsorbed per unit mass of
m octahedral complex.
solid adsorbent, p = pressure, K and n = constants. All metal sols like, Au-sol, Ag-sol etc; are negatively charged
sols.
The logarithm equation of Freundlich adsorption isotherm is
14. In heterogeneous catalytic reactions, physical state of reactants
x 1
log = log K + log p and that of catalyst(s) used are different.
m n Haber’s process, hydrogenation of vegetable oils and

w
On comparing the above equation with straight line equation, Ostwald’s process all are heterogeneous process. Combustion of
( y = mx + c) coal is not a heterogeneous catalytic reaction.
we get ●
In Haber’s process
1 Fe( s), Mo( s)
m = slope = and c = log K

Flo
n N 2 ( g ) + 3H 2( g ) ¾¾¾¾® 2NH 3 ( g )
From the given plot, ●
Hydrogenation of vegetable oils,
y - y1 1 2

ree
m= 2 = = [(Ph P) Rh] Cl
Vegetable oil (l ) ¾¾¾¾¾¾¾
3 3
® Vanaspati (s)
x2 - x1 n 3
(Unsaturated) or Ni ( s)
x

F
\ = Kp2 / 3 ●
Ostwald’s process,
m
Pt( s)
9. Statement given as statement (d) is incorrect. Latex is a stable 4NH3 (g ) + 5O 2 (g ) ¾¾¾® 4NO(g ) + 6H2O(g )

or
ur
dispersion, i.e. emulsion of polymer microparticles in an
V 2 O5 (s)

No catalyst is used in combustion of coal. The reaction is

f

aqueous medium.
highly spontaneous in nature.
These microparticles belong to rubber and are negatively
ks
charged in nature. Natural latex contains some amount of sugar, C + O2 ¾® CO2
Yo
(Coal)
resin, protein and ash as well.
oo
15. Using the principle of adsorption chromatography, qualitative
The closest synthetic latex that can be associated with the and quantitative analysis of benzaldehyde can be done from its
properties of natural latex is SBR, i.e. Styro Butane Rubber.
eB

mixture with acetonitrile. Here, a mobile phase moves over a


Rest of all the statements are correct. stationary phase (adsorbent). Adsorbents used are alumina
(Al 2O 3 ) and silica gel. The sample solution of benzaldehyde and
10. Same adsorbant (charcoal in this case) at same temperature will acetonitrile when comes in contact with the adsorbent,
r

adsorb different gases to different extent. The extent to which benzaldehyde gets adsorbed on the surface of the adsorbent. So,
ou
ad

gases are adsorbed is proportional to the critical temperature of benzaldehyde acts as absorbate whereas acetonitrile starts
gas. moving as mobile phase over the stationary phase of the
Y

8a adsorbate. Hence, act as dynamic phase.


Q Tc =
27Rb 16. Arsenious sulphide sol is a negative colloid, As2S3.(S2- ). So, it
nd

will be coagulated by the cation of an electrolyte.


Re

where, a is the magnitude of intermolecular forces between


gaseous molecules. According to the Hardy-Schulze rule, the higher the charge of
the ion, the more effective it is in bringing about coagulation.
Fi

Thus, higher the cirtical temperature more is the gas adsorbed.


Among the given gases, H2 has the minimum critical Here, the cations available are Al 3+ (from AlCl 3), Ba 2+ (from
temperature, i.e. 33K thus, it shows least adsorption on a definite BaCl 2) and Na + (from Na 3PO 4 and NaCl). So, their power to
amount of charcoal. coagulate As2S3. (S2- ) will follow the order as

11. Dispersed Dispersion Type of Examples Al 3+ > Ba 2+ > Na+


phase medium colloid 17. According to Freundlich adsorption isotherm,
Liquid Solid Gel Cheese (C), butter, x x
µ p1/ n Þ = Kp1/ n
jellies m m
Liquid Liquid Emulsion Milk (M), hair cream On taking log on both sides, we get
Solid Gas Aerosol Smoke (S), dust æxö 1
log ç ÷ = log K + log p
Thus, C : liquid in solid, M : liquid in liquid and S : solid in gas. è mø n
On comparing with equation of straight line, y = mx + c, plot of
12. Solid sol consists of solid as both dispersed phase and dispersion x
medium. In gemstones, metal crystals (salt and oxides of metals) log vs log p gives,
are dispersed in solid (stone) medium. Hair cream is an emulsion m
Surface Chemistry 193

( y2 - y1 ) 1 2 1 27. As temperature increases surface tension of liquid decreases.


Slope = Þ =
( x2 - x1 ) n 4 2 28. (a) Higher the critical temperature, greater the extent of
x 1/ 2 adsorption.
\ µp
m (c) P (s) + Q (g ) ¾® PQ (s)
Adsorbent Adsorbate
18. Colloidal solutions show Tyndall effect due to scattering of light As gaseous adsorbate is adsorbed on solid surface, entropy
by colloidal particles in all directions in space. It is observed decreases, DS < 0. Also formation of bond between P and Q
only under the following conditions. results in release of energy, hence DH < 0.
(i) The diameter of the colloids should not be much smaller than
the wavelength of light used. 29. Since, adsorption involves electron transfer from metal to O2, it
(ii) The refractive indices of the dispersed phase and dispersion is chemical adsorption not physical adsorption, hence (a) is
incorrect. Adsorption is spontaneous which involves some
medium should differ greatly in magnitude.
bonding between adsorbent and adsorbate, hence exothermic.

w
19. According to Freundlich adsorption isotherm,
x
= kp1/ n The last occupied molecular orbital in O2 is p* 2p. Hence,
m electron transfer from metal to oxygen will increase occupancy
On taking logarithm of both sides, we get of p* 2p molecular orbitals. Also increase in occupancy of p* 2p
x

Flo
log = log k + log p1 / n orbitals will decrease bond order and hence increase bond length
m Slope = 1 of O2.
x 1 q n
log x/m

or log = log k + log p 30. Graph-I represents physisorption as in physisorption, absorbents

ree
m n are bonded to adsorbate through weak van der Waals’ force.
y = c + mx Increasing temperature increases kinetic energy of adsorbed
log k

F
x particles increasing the rate of desorption, hence amount of
y = log , adsorption decreases.
m
log p Graph-II represents chemisorption as it is simple activation
c = intercept = log k

or
1
ur energy diagram of a chemical reaction.
m = slope = and x = log p Graph-III also represents physical adsorption as extent of

f
n
adsorption increasing with pressure.
ks
20. Physical adsorption takes place with decrease in enthalpy thus Graph-IV represents chemisorption as it represents the potential
exothermic change. It is physical adsorption and does not
Yo
energy diagram for the formation of a typical covalent bond.
require activation. Thus, (a) is incorrect.
oo
31. Lyophobic sol, which is otherwise unstable, gets stabilised by
Being physical adsorption DH < 0 thus, (b) is correct. Exothermic preferential adsorption of ions on their surface, thus developing
eB

reaction is favoured at low temperature thus (c) is incorrect. a potential difference between the fixed layer and the diffused
Physical adsorption is always reversible, thus (d) is incorrect. layer. Thus, option (a) and (d) are correct.
21. According to Hardy Schulze rule, greater the charge on
32. (a) In the process of adsorption, a bond is formed between
oppositely charged ion, greater is its coagulating power. Since
r

adsorbate and adsorbent, hence always exothermic.


arsenic sulphide is a negatively charged sol, thus, the order of
ou
ad

(b) Physisorption require very low activation energy while


coagulating power is Na + < Ba 2+ < Al 3+ .
chemisorption require high activation energy. Therefore a
Y

22. Sb2S3 is a negative (anionic) sol. According to Hardy Schulze physisorption may transform into chemisorption but only at
rule, greater the valency of cationic coagulating agent, higher its high temperature.
coagulating power. Therefore, Al 2 (SO4 )3 will be the most (c) It is wrong statement as at higher temperature, physically
nd
Re

effective coagulating agent in the present case. adsorbed substance starts desorbing.
23. Larger the hydrophobic fragment of surfactant, easier will be the (d) In physical adsorption, van der Waals' force hold the
Fi

micellisation, smaller the crticial micelle concentration. adsorbate and adsorbent together which is a weak
Therefore, CH3 (CH2 )15 N+ (CH3 )3 Br - will have the lowest electrostatic attraction. In chemisorption, strong chemical
crticial micelle concentration. bond binds the adsorbate to the adsorbent. Therefore,
24. Lyophilic sols are reversible, not easily coagulated because it is chemisorption is more exothermic than physical adsorption.
self-stabilising. 33. Both statements are independently correct but Statement II does
25. DG = DH - TDS not explain Statement I. Critical micelle concentration is the
As gas is adsorbed on surface of solid, entropy decreases, i.e. minimum concentration of surfactant at which micelle
DS < 0. Therefore, for DG < 0, DH must be negative. formation commences first. At critical micelle concentration,
several molecules of surfactant coalesce together to form one
26. It is an exothermic process, according to Le-Chatelier’s principle, single micelle molecule. This decreases the apparent number of
lowering temperature drive the process in forward direction. molecule suddenly lowering conductivity sharply.

Download Chapter Test


http://tinyurl.com/y6lc4mog or
14
s-Block Elements

w
I. They activate many enzymes.
Topic 1 Group I Elements II. They participate in the oxidation of glucose to produce
ATP.
Objective Questions I (Only one correct option)

Flo
III. Along with sodium ions, they are responsible for the
1. The temporary hardness of a water sample is due to transmission of nerve signals.
compound X . Boiling this sample converts X to compound

ree
(a) I, and III only (b) I, II and III
Y . X and Y , respectively, are (2019 Main, 12 April II) (c) I and II only (d) III only
(a) Mg(HCO3 )2 and Mg(OH)2
8. A metal on combustion in excess air forms X . X upon

F
(b) Ca(HCO3 )2 and Ca(OH)2
(c) Mg(HCO3 )2 and MgCO3 hydrolysis with water yields H2 O2 and O2 along with another
(d) Ca(HCO3 )2 and CaO product. The metal is (2019 Main, 12 Jan I)

or
2. The incorrect statement is
ur (2019 Main, 12 April II)
(a) Li (b) Mg (c) Rb (d) Na
9. The hardness of a water sample (in terms of equivalents of

f
(a) lithium is the strongest reducing agent among the alkali metals.
(b) lithium is least reactive with water among the alkali metals. CaCO3 ) containing is
ks
(c) LiNO3 decomposes on heating to give LiNO2 and O2. (Molar mass of CaSO4 = 136 g mol -1 ) (2019 Main, 12 Jan I)
Yo
(d) LiCl crystallise from aqueous solution as LiCl × 2H2O. (a) 100 ppm (b) 10 ppm (c) 50 ppm (d) 90 ppm
oo

3. The metal that gives hydrogen gas upon treatment with both 10. The hydride that is not electron deficient is
eB

acid as well as base is (2019 Main, 12 April I) (2019 Main, 11 Jan II)
(a) magnesium (b) mercury (a) AlH3 (b) B2H6 (c) SiH4 (d) GaH3
(c) zinc (d) iron 11. The correct statements among (a) to (d) regarding H2 as a
r

4. The correct statements among (a) to (d) are: fuel are : (2019 Main, 11 Jan I)
ou

(2019 Main, 8 April II)


ad

1. Saline hydrides produce H2 gas when reacted with H2O. I. It produces less pollutants than petrol.
Y

2. Reaction of LiAlH4 with BF3 leads to B2H6. II. A cylinder of compressed dihydrogen weights ~ 30 times
3. PH3 and CH4 are electron rich and electron precise more than a petrol tank producing the same amount of
hydrides, respectively. energy.
nd
Re

4. HF and CH4 are called as molecular hydrides. III. Dihydrogen is stored in tanks of metal alloys like NaNi 5 .
(a) (1), (2), (3) and (4) (b) (1), (2) and (3) only IV. On combustion, values of energy released per gram of
Fi

(c) (3) and (4) only (d) (1), (3) and (4) only liquid dihydrogen and LPG are 50 and 142 kJ,
5. The strength of 11.2 volume solution of H 2O2 is [Given that respectively.
(a) I, II and III only (b) II, III and IV only
molar mass of (c) II and IV only (d) I and III only
H = 1 g mol -1 and O = 16 g mol -1 ] (2019 Main, 8 April II) 12. NaH is an example of (2019 Main, 11 Jan I)
(a) 1.7% (b) 34% (c) 13.6% (d) 3.4% (a) metallic hydride (b) electron-rich hydride
6. The correct order of hydration enthalpies of alkali metal ions (c) saline hydride (d) molecular hydride
is (2019 Main, 8 April I) 13. Sodium metal on dissolution in liquid ammonia gives a deep
(a) Li + > Na + > K + > Cs+ > Rb+ blue solution due to the formation of (2019 Main, 10 Jan II)
(b) Na + > Li + > K + > Rb+ > Cs+ (a) sodium ammonia complex (b) sodium ion-ammonia complex
(c) sodamide (d) ammoniated electrons
(c) Na + > Li + > K + > Cs+ > Rb+
(d) Li + > Na + > K + > Rb+ > Cs+ 14. The total number of isotopes of hydrogen and number of
radioactive isotopes among them, respectively, are
7. The correct statement(s) among I to III with respect to (2019 Main, 10 Jan I)
potassium ions that are abundant within the cell fluids is/are (a) 2 and 1 (b) 3 and 2 (c) 2 and 0 (d) 3 and 1
(2019 Main, 12 Jan II)
s-Block Elements 195

15. The chemical nature of hydrogen peroxide is 25. Hydrogen peroxide in its reaction with KIO4 and NH2 OH
(2019 Main, 10 Jan I)
respectively, is acting as a (2014 Adv.)
(a) oxidising and reducing agent in both acidic and basic medium (a) reducing agent, oxidising agent
(b) oxidising and reducing agent in acidic medium, but not in (b) reducing agent, reducing agent
basic medium (c) oxidising agent, oxidising agent
(c) reducing agent in basic medium, but not in acidic medium (d) oxidising agent, reducing agent
(d) oxidising agent in acidic medium, but not in basic medium 26. In which of the following reactions H2 O2 acts as a reducing
16. The metal that forms nitride by reacting directly with N 2 of agent? (2014 Main)
+ -
air, is (2019 Main, 9 Jan II) I. H2O2 + 2H + 2e ¾® 2H2O
(a) Rb (b) K (c) Cs (d) Li II. H2O2 - 2e- ¾® O2 + 2H+
17. What is reason of temporary hardness of water? III. H2O2 + 2e- ¾® 2OH-

w
(2019 Main, 9 Jan II)
(a) Na 2SO 4 (b) CaCl 2 (c) NaCl (d) Ca(HCO 3) 2 IV. H2O2 + 2OH- - 2e- ¾® O2 + 2H2O
(a) I and II (b) III and IV (c) I and III (d) II and IV
18. The isotopes of hydrogen are (2019 Main, 9 Jan I)
(a) deuterium and tritium only 27. A sodium salt of an unknown anion when treated with MgCl 2

Flo
(b) protium and deuterium only gives white precipitate only on boiling. The anion is
(c) protium, deuterium and tritium (2004, 1M)

ree
(d) tritium and protium only (a) SO2- (b) HCO-3 (c) CO2- (d) NO-3
4 3
19. Hydrogen peroxide oxidises [Fe(CN)6 ]4- to [Fe(CN)6 ]3- in 28. A dilute aqueous solution of Na 2 SO4 is electrolysed using
acidic medium but reduces [Fe(CN)6 ]3- to [Fe(CN)6 ]4- in

F
platinum electrodes. The products at the anode and cathode
alkaline medium. The other products formed are, are respectively (1996, 1M)
respectively. (a) O2 , H2 (b) S2O2–
8 , Na (c) O2 , Na (d) S2O2–
8 , H2

or
(2018 Main)
(a) (H2O + O2 ) and H2O
ur
(b) (H2O + O2 ) and (H2O + OH- ) 29. Hydrolysis of one mole of peroxodisulphuric acid produces

f
(c) H2O and (H2O + O2 ) (d) H2O and (H2O + OH- ) (a) two moles of sulphuric acid (1996, 1M)
ks
(b) two moles of peroxomono sulphuric acid
20. Both lithium and magnesium display several similar
(c) one mole of sulphuric acid and one mole of peroxomono
Yo
properties due to the diagonal relationship; however, the one
sulphuric acid
oo
which is incorrect is (2017 Main)
(d) one mole of sulphuric acid, one mole of peroxomono sulphuric
(a) Both form basic carbonates
eB

acid and one mole of hydrogen peroxide


(b) Both form soluble bicarbonates
(c) Both form nitrides 30. The species that do not contain peroxide ions, is (1992, 1M)
(d) nitrates of both Li and Mg yield NO2 and O2 on heating (a) PbO2 (b) H2O2 (c) SrO2 (d) BaO2
r

21. The hottest region of Bunsen flame shown in the figure given 31. The metallic lustre exhibited by sodium metal is explained
ou
ad

below is (2016 Main) by (1987, 1M)


Region 4 (a) diffusion of sodium ions
Y

Region 3 (b) oscillation of loose electron


Region 2 (c) excitation of free protons
Region 1 (d) existence of body centred cubic lattice
nd
Re

32. A solution of sodium sulphate in water is electrolysed using


inert electrodes. The products at cathode and anode are
Fi

(a) region 2 (b) region 3 (c) region 4 (d) region 1 respectively (1987, 1M)
22. Which one of the following statements about water is false? (a) H2 , O2 (b) O2 , H2 (c) O2 , Na (d) O2 , SO2
(2016 Main) 33. Nitrogen dioxide cannot be obtained by heating (1985, 1M)
(a) Water can act both as an acid and as a base (a) KNO3 (b) Pb(NO3 )2 (c) Cu(NO3 )2 (d)AgNO3
(b) There is extensive intramolecular hydrogen bonding in the
34. The oxide that gives H2 O2 on treatment with a dilute acid is
condensed phase
(a) PbO2 (b) Na 2O2 (1985, 1M)
(c) Ice formed by heavy water sinks in normal water
(c) MnO2 (d) TiO2
(d) Water is oxidised to oxygen during photosynthesis
35. Molecular formula of Glauber’s salt is (1985, 1M)
23. The main oxides formed on combustion of Li, Na and K in (a) MgSO4 × 7H2O (b) CuSO4 × 5H2O
excess of air respectively are (2016 Main)
(c) FeSO4 × 7H2O (d) Na 2SO4 × 10H2O
(a) LiO2, Na 2O2 and K 2O (b) Li 2O2, Na 2O2 and KO2
(c) Li 2O, Na 2O2 and KO2 (d) Li 2O , Na 2O and KO2 36. Heavy water is (1983, 1M)
(a) H2O18
24. Which of the following atoms has the highest first ionisation (b) water obtained by repeated distillation
energy? (2016 Main) (c) D2O
(a) Na (b) K (c) Sc (d) Rb (d) water at 4°C
196 s-Block Elements

37. A solution of sodium metal in liquid ammonia is strongly 46. Statement I LiCl is predominantly a covalent compound.
reducing due to the presence of (1981, 1M) Statement II Electronegativity difference between Li and
(a) sodium atoms (b) sodium hydride Cl is too small. (1998, 2M)
(c) sodium amide (d) solvated electrons
47. Statement I The alkali metals can form ionic hydrides
38. The temporary hardness of water is due to calcium which contain the hydride ion, H- .
bicarbonate can be removed by adding (1979, 1M)
Statement II The alkali metals have low electronegativity,
(a) CaCO3 (b) Ca(OH)2 (c) CaCl 2 (d) HCl
their hydrides conduct electricity when fused and liberate
hydrogen gas at the anode. (1994, 2M)
Objective Questions II
(One or more than one correct option) Fill in the Blanks
39. The pair(s) of reagents that yield paramagnetic species is/are 48. Hydrogen gas is liberated by the action of aluminium with

w
(a) Na and excess of NH3 (b) K and excess of O2 concentrated solution of ……… . (1987, 1M)
(c) Cu and dilute HNO3 (d) O2 and 2-ethylanthraquinol
49. Sodium dissolved in liquid ammonia conducts electricity
40. The compound(s) formed upon combustion of sodium metal because of …… (1985, 1M)
in excess air is (are) (2007, 2M)

Flo
50. The adsorption of hydrogen by palladium is commonly
(a) Na 2O2 (b) Na 2O (c) NaO2 (d) NaOH
known as …… (1983, 1M)
41. Sodium nitrate decomposes above » 800°C to give

ree
51. Iodine reacts with hot NaOH solution. The products are NaI
(a) N2 (b) O2 (1998, 2M)
and …… (1980, 1M)
(c) NO2 (d) Na 2 O

F
42. Highly pure dilute solution of sodium in liquid ammonia True/False
(a) shows blue colour (1998, 2M)
52. Sodium when burnt in excess of oxygen gives sodium oxide.
(b) exhibits electrical conductivity

or
(c) produces sodium amide
ur (1987, 1M)

(d) produces hydrogen gas Subjective Questions


43. When zeolite, which is hydrated sodium aluminium silicate,
f
53. A white solid is either Na 2 O or Na 2 O2 . A piece of red litmus
ks
is treated with hard water, the sodium ions are exchanged paper turns white when it is dipped into a freshly made
Yo
with (1990, 1M) aqueous solution of the white solid.
oo
(a) H+ ions (b) SO2–
4 ions (c) Mg2+ ions (d) OH- ions (i) Identify the substance and explain with balanced
eB

44. Sodium sulphate is soluble in water, whereas barium equation.


sulphate is sparingly soluble because (1989, 1M) (ii) Explain what would happen to the red litmus if the
(a) the hydration energy of sodium sulphate is more than its white solid were the other compound. (1999, 4M)
lattice energy
r

54. Hydrogen peroxide acts both as an oxidising and as a


ou

(b) the lattice energy of barium sulphate is more than its


ad

reducing agent in alkaline solution towards certain first row


hydration energy transition metal ions. Illustrate both these properties of H2 O2
Y

(c) the lattice energy has no role to play in solubility using chemical equations. (1998, 4M)
(d) the hydration energy of sodium sulphate is less than its
55. Element A burns in nitrogen to give an ionic compound B.
lattice energy
nd

Compound B reacts with water to give C and D. A solution of


Re

Assertion and Reason C becomes ‘milky’ on bubbling carbon dioxide gas. Identify
A, B, C and D. (1997, 3M)
Fi

Read the following questions and answer as per the direction given
below : 56. Complete and balance the following chemical reaction.
(a) Statement I is correct; Statement II is correct; Statement Anhydrous potassium nitrate is heated with excess of
II is the correct explanation of Statement I metallic potassium
(b) Statement I is correct; Statement II is correct; Statement KNO3 ( s ) + K ( s ) ¾® K + K (1992, 1M)
II is not the correct explanation of Statement I 57. Give reasons in one or two sentences for the following:
(c) Statement I is correct; Statement II is incorrect
(d) Statement I is incorrect; Statement II is correct “H2 O2 is a better oxidising agent than H2 O.” (1986, 1M)

45. Statement I Alkali metals dissolve in liquid ammonia to give 58. Sodium carbonate is prepared by Solvay process but the
blue solution. same process is not extended to the manufacture of
Statement II Alkali metals in liquid ammonia give solvated potassium carbonate, explain. (1981, 1M)

species of the type [ M ( NH 3 )n ]+ (M = alkali metals). 59. Water is a liquid, while H2 S is a gas at ordinary temperature.
(2007, 3M) Explain . (1978, 1M)
s-Block Elements 197

Topic 2 Group II Elements


Objective Questions I (Only one correct option) 11. The alkaline earth metal nitrate that does not crystallise with
1. In comparison to boron, berylium has (2019 Main, 12 April II) water molecules, is (2019 Main, 9 Jan I)
(a) Ca(NO3 )2 (b) Sr(NO3 )2
(a) lesser nuclear charge and lesser first ionisation enthalpy
(c) Ba(NO3 )2 (d) Mg(NO3 )2
(b) greater nuclear charge and lesser first ionisation enthalpy
(c) greater nuclear charge and greater first ionisation enthalpy 12. Which one of the following alkaline earth metal sulphates
(d) lesser nuclear charge and greater first ionisation enthalpy has its hydration enthalpy greater than its lattice enthalpy?
(a) CaSO 4 (b) BeSO 4 (2015 Main)
2. The correct sequence of thermal stability of the following (c) BaSO 4 (d) SrSO 4
carbonates is (2019 Main, 12 April I)
BaCO3 < CaCO3 < SrCO3 < MgCO3 13. The following compounds have been arranged in order of their

w
(a)
(b) MgCO3 < CaCO3 < SrCO3 < BaCO3 increasing thermal stabilities. Identify the correct order.
(c) MgCO3 < SrCO3 < CaCO3 < BaCO3 K 2 CO3 (I), MgCO3 (II), CaCO3 (III), BeCO3 (IV)
(d) BaCO3 < SrCO3 < CaCO3 < MgCO3 (a) I < II < III < IV (b) IV < II < III < I (1996, 1M)

Flo
3. A hydrated solid X on heating initially gives a (c) IV < II < I < III (d) II < IV < III < I
monohydrated compound Y . 14. The oxidation state of the most electronegative element in
Y upon heating above 373 K leads to an anhydrous white the products of the reaction, BaO2 with dil. H2 SO4 are

ree
powder Z. X and Z, respectively, are (2019 Main, 10 April II) (a) 0 and –1 (b) –1 and –2 (1991, 1M)
(a) baking soda and soda ash (c) –2 and 0 (d) –2 and –1

F
(b) washing soda and soda ash
15. Calcium is obtained by (1980, 1M)
(c) baking soda and dead burnt plaster
(d) washing soda and dead burnt plaster
(a) electrolysis of molten CaCl 2

or
ur
4. The alloy used in the construction of aircrafts is
(b) electrolysis of solution of CaCl 2 in water
(c) reduction of CaCl 2 with carbon

f
(2019 Main, 10 April I)
(d) roasting of limestone
(a) Mg-Zn (b) Mg-Mn (c) Mg-Sn (d) Mg-Al
ks
5. The structures of beryllium chloride in the solid state and Objective Questions II
Yo
vapour phase, respectively are (One or more than one correct option)
oo
(2019 Main, 9 April II)
(a) dimeric and dimeric (b) chain and chain
(c) dimeric and chain (d) chain and dimeric 16. The reagent(s) used for softening the temporary hardness of
eB

water is(are) (2010)


6. Magnesium powder burns in air to give (2019 Main, 9 April I) (a) Ca 3 (PO4 )2 (b) Ca(OH)2
(a) MgO and Mg 3N2 (b) Mg (NO3 )2 and Mg 3N2 (c) Na 2CO3 (d) NaOCl
r

(c) MgO only (d) MgO and Mg (NO3 )2 17. MgSO 4 on reaction with NH4 OH and Na 2 HPO4 forms a
ou
ad

7. The covalent alkaline earth metal halide ( X = Cl, Br, I) is white crystalline precipitate. What is its formula? (2006, 3M)
(a) Mg(NH4 )PO4 (b) Mg3 (PO4 )3
Y

(2019 Main, 8 April II)


(a) SrX 2 (b) CaX 2 (c) MgX 2 (d) BeX 2 (c) MgCl 2 × MgSO4 (d) MgSO4
8. Match the following items in Column I with the corresponding 18. The material used in solar cells contains (1993, 1M)
nd
Re

items in Column II. (2019 Main, 11 Jan II) (a) Cs (b) Si (c) Sn (d) Ti

Fill in the Blank


Fi

Column I Column II
(i) Na 2CO3 × 10H2O A. Portland cement 19. Anhydrous MgCl 2 is obtained by heating the hydrated salt
ingredient with …… (1980 , 1M)
(ii) Mg(HCO3 )2 B. Castner-Kellner process
(iii) NaOH C. Solvay process True/False
(iv) Ca 3Al 2O6 D. Temporary hardness 20. MgCl 2 × 6H2 O on heating gives anhydrous MgCl 2 . (1982, 1M)

(a) (i) - (D); (ii) - (A); (iii) - (B); (iv) - (C) Subjective Questions
(b) (i) - (B); (ii) - (C); (iii) - (A); (iv) - (D)
(c) (i) - (C); (ii) - (B); (iii) - (D); (iv) - (A) 21. Identify (X) in the following synthetic scheme and write their
(d) (i) - (C); (ii) - (D); (iii) - (B); (iv) - (A) structures.
9. The amphoteric hydroxide is (2019 Main, 11 Jan I) *
Ba CO3 + H2 SO4 ¾® X (gas) (C* denotes C14 ) (2001, 1M)
(a) Be(OH)2 (b) Ca(OH)2 (c) Sr(OH)2 (d) Mg(OH)2
10. The metal used for making X-ray tube window is 22. Give reasons for the following in one or two sentences only :
(2019 Main, 10 Jan I) “BeCl 2 can be easily hydrolysed.” (1999, 2M)
(a) Na (b) Be (c) Mg (d) Ca
198 s-Block Elements

23. The crystalline salts of alkaline earth metals contain more alkaline solution. The solution on exposure to air produces a
water of crystallisation than the corresponding alkali metal thin solid layer of B on the surface. Identify the compounds A
salts. Why? (1997, 2M) and B. (1996, 3M)

24. Calcium burns in nitrogen to produce a white powder which 25. Arrange the following in increasing order of basic strength :
dissolves in sufficient water to produce a gas A and an MgO, SrO, K 2 O, NiO, Cs2 O (1991, 1M)

Answers
Topic 1 41. (a, b, d) 42. (a, b) 43. (a, d) 44. (a, b)
1. (a) 2. (c) 3. (c) 4. (a) 45. (b) 46. (c) 47. (a) 48. NaOH
5. (d) 6. (d) 7. (b) 8. (c) 49. solvated electrons 50. occlusion 51. NaIO 3

w
9. (a) 10. (c) 11. (a) 12. (c) 52. F
13. (d) 14. (d) 15. (a) 16. (d)
17. (d) 18. (c) 19. (c) 20. (a)
Topic 2

Flo
1. (d) 2. (b) 3. (b) 4. (d)
21. (a) 22. (b) 23. (c) 24. (c)
5. (c) 6. (a) 7. (d) 8. (d)
25. (a) 26. (d) 27. (b) 28. (a)

ree
9. (a) 10. (b) 11. (c) 12. (b)
29. (c) 30. (a) 31. (b) 32. (a)
13. (b) 14. (d) 15. (a) 16. (b, c, d)
33. (a) 34. (b) 35. (d) 36. (c)
17. (a) 18. (b)

F
19. dry HCl 20. F
37. (d) 38. (b) 39. (a, b, c) 40. (a, b)

or
ur
Hints & Solutions
Topic 1 Group I Elements
f
ks
Yo
1. The temporary hardness of a water sample is due to compound 3. PH3 and CH4 are covalent hydrides and in both of the
oo
X [i.e. Mg(HCO3 )2]. Boiling of this sample converts X [i.e. hydrides, octet of P and C have been satisfied. But P in PH3
Mg(HCO3 )2] to compound Y[i.e. Mg(OH)2 ]. Generally, has one lone pair of electrons and C in CH4 does not have so
eB

temporary hardness is due to presence of magnesium and PH3 (group 15) and CH4 (group 14) are electron rich and
calcium hydrogen carbonates. It can be removed by boiling. electron precise hydrides, respectively.
During boiling, the soluble Mg(HCO3 )2 is converted into Thus, statement (3) is correct.
insoluble Mg(OH)2 and Ca(HCO3 )2 changed to insoluble
r

4. HF and CH4 are called as molecular hydrides because of


CaCO3 . These precipitates can be removed by filteration.
ou
ad

their discrete and sterically symmetrical structure.


Heating
Mg(HCO3 )2 ¾ ¾ ¾
¾® Mg(OH)2 ¯ + 2CO2 ­
Thus, statement (4) is also correct.
Y

Heating
Ca(HCO3 )2 ¾¾¾® CaCO3 ¯ + H2O + CO2 ­ 5. 11.2 volume of H2O2 means that 1 mL of this H2O2 will give
2. Statement (c) is incorrect. LiNO3(Lithium nitrate) on heating 11.2 mL of oxygen at STP.
nd
Re

gives a mixture of Li 2O, NO2 and O2. 2H2O2 (l ) ¾® O2 (g ) + 2H2O(l )


D 2 ´ 34 g 22.4 L at STP
Fi

4LiNO3 ¾® 2Li 2O + 4NO2 ­ + O2 ­


22.4 L of O2 at STP is produced from H2O2 = 68 g
Among the alkali metals, lithium is the strongest reducing agent.
\11.2 L of O2 at STP is produced from
3. Metal that gives hydrogen gas upon treatment with both acid as
68
well as base is zinc. Hence, it is amphoteric in nature. H2O2 = ´ 112
. = 34 g
Reactions involved are as follows: 22.4
Zn + Dil. NaOH ¾® Na 2 ZnO2 + H2 ­ \34 g of H2O2 is present in 1000 g of solution
Zn + 2HCl(dil.) ¾® ZnCl 2 + H2 ­ 34
\% w/w = ´ 100 = 3.4%
1000
4. The explanation of given statements are as follows :
1. Saline or ionic hydrides produce H2 with H2O. Key Idea The amount of energy released when one mole of
Å È
6.
gaseous ions combine with water to form hydrated ions is
M H + H2O ¾® H2 ­ + MOH
called hydration enthalpy.
Thus, statement (1) is correct.
Ether The correct order of hydration enthalpies of alkali metal ions is
2. 3LiAlH4 + 4BF3 ¾® 2B2H6 + 3LiF + 3AlF3
(Diborane) Li + > Na + > K+ > Rb+ > Cs+
Thus, statement (2) is correct.
s-Block Elements 199

Li+ possesses the maximum degree of hydration due to its small 11. (I) H2 is a 100% pollution free fuel. So, statement (I) is correct.
size. As a consequence of hydration enthalpy, their mobility (II) Molecular weight of H2(2u).
also get affected. Cs+ has highest and Li + has lowest mobility in 1
aqueous solution. = ´ molecular weight of butane,
29
7. All the statements are correct. K+ being metallic unipositive C 4H10 (LPG) [58u].
ions work as enzyme activators. These also participate in many So, compressed H2 weighs ~30 times more than a petrol tank
reactions of glycolysis and Kreb’s cycle to produce ATP from and statement (II) is correct.
glucose. (III) NaNi 5, Ti - TiH2 etc. are used for storage of H2 in small
Being unipositive these are also equally responsible for nerve quantities. Thus, statement (III) is correct.
signal transmission along with Na + . (Na + ion-pump theory) (IV) On combustion values of energy released per gram of liquid
8. Metal (A) is rubidium (Rb). In excess of air, it forms RbO2( X ). dihydrogen (H2 ) : 142 kJ g -1, and for LPG : 50 kJ g -1. So,

w
X is a superoxide that have O-2 ion. It is due to the stabilisation staement (IV) is incorrect.
r s
of large anion by large cations through lattice energy effects. 12. Na H is an example of ionic or saline hydride. These hydrides
RbO2 (X ) gets easily hydrolysed by water to form the are formed when hydrogen combines with metals having less
hydroxide, H2O2 and O2.

Flo
electronegativity and more electropositive character with
The reaction involved are as follows: respect to hydrogen.
Rb + O2 ¾® RbO2 (superoxide) Except Be and Mg, all s-block metals form saline hydrides.

ree
(X)
Hydrides of p-block elements are covalent in nature, viz,
2RbO2 + 2H2O ¾® 2RbOH + H2O2 + O2 electron deficient hydrides (by group-13 elements),

F
(X) electron-precise hydrides (by group-14 elements), and
electron-rich hydrides (by group 15-17 elements). Hydrides of
9. Hardness of water sample can be calculated in terms of ppm
d , f -block metals are called interstitial or metallic hydrides.

or
concentration of CaCO3.
ur 13. Sodium metal on dissolution in liquid ammonia gives a deep
Given, molarity = 10-3M

f
blue solution due to the formation of ammoniated electrons. The
i.e. 1000 mL of solution contains 10-3 mole of CaCO3. reaction is represented as follows:
ks
\Hardness of water = ppm of CaCO3 Na (s) + (x + y) NH3 (l ) ¾® [Na(NH)x ]+ + [ e(NH 3 ) y ]
Yo
oo
10-3 ´ 1000 [Ammoniated Na + [Ammoniated
= ´ 106 or expanded Na] electrons]
1000
eB

Ammoniated (solvated) electrons show electronic transition in


= 100 ppm
visible region and the solution becomes deep blue coloured.
10. GaH3, AlH3 and B2H6 are the hydrides of group-13 (ns2np1 ), This deep blue solution also shows the following properties due
to the presence of ammoniated electrons.
r

whereas SiH4 is an hydride of group 14.


ou

(i) It is strongly reducing in nature.


ad

H H (ii) It is paramagnetic.
Y

sp2 sp2 (iii) It is a good conductor of electricity.


Ga Al
H H H H
14. Hydrogen has three isotopes:
6e – around 6e – around
nd
Re

Ga 3+ <8e – (octet) Al 3+ <8e – (octet) 1 2 3


1H 1H 1 H
(GaH3) (AlH3)
Fi

3c–2e – bridge bond Protium (P) Deuterium (D) Tritium (T)


H 3
H sp H p 1 1 1
B B
H H n 0 1 2
H
B is sp3, but cannot satisfy octet due n 0 1 2
to the attachment of two 3c-2e –
bridge bonds (B–H–B) p
n
So, B2H6, AlH3 and GaH3 are electron deficient hydrides. Only tritium (T) is radioactive, because of its very high value,
p
But, SiH4 is an electron precise hydride of group-14 (ns2np2 ),
æn ö
i.e. these hydrides can have the required number of electrons to çç = 2÷÷ .
write their conventional Lewis structures. èp ø
H 15. H2O2 can act as both oxidising and reducing agents in both
8e– around Si (Octet gets satisfied)
Si acidic and basic medium.
H H2 O2 as oxidising agent
H H

In acidic medium: H2 O2 + 2H+ + 2e- ¾® 2H2 O
200 s-Block Elements

- 22. There is extensive intermolecular H-bonding in the condensed



In basic medium : H2 O2 + 2O H + 2e- ¾® 2H2 O + 2O2 -
phase.
H2 O2 as reducing agent 1
23. 2Li + O2 (g ) ¾® Li 2O

In acidic medium : H2 O2 ¾® O2 + 2H+ + 2e- 2
(Excess)
In basic medium :
2Na + O2 (g ) ¾® Na 2O2; K + O2 (g ) ¾® KO2
H2O2 + 2OH- ¾® O2 + 2H2O + 2e- ( Excess) (Excess)

16. Among the group-1 metals, only Li is able to form its nitride, 24. Order of first ionisation energy is Sc > Na > K > Rb.
Li 3N. [All alkaline earth metals of group-2 form their nitride, Due to poor shielding effect, removal of one electron from 4s
M 3N2] orbital is difficult as compared to 3s-orbital.
6Li + N 2 ¾® 2Li 3N (Ruby red solid)
(Air) 25. PLAN This problem can be solved by using concept of oxidant and

w
reductant.
I II
[ 3M + N 2 ¾® M 3N 2 ] Oxidant Oxidant increases the oxidation number of the
Å Å species with which it is reacted.
Li is the smallest metal ion of group-1. Smaller size of Li and
larger size of nitride ion, N 3- , enable Li Å to polarise the Reductant Reductant decreases the oxidation number of the

Flo
spherical electron cloud of N 3- and it gives higher stability to species with which it is reacted.
Li 3N. H2O2 reacts with KIO4 in the following manner:

ree
17. Temporary hardness of water is due to presence of soluble +7 +5
Ca(HCO3 )2 or Mg(HCO3 )2. KIO4 + H2O2 ¾® KIO3 + H2O + O2

F
Permanent hardness of water is due to the presence of CaCl 2 or On reaction of KIO4 with H2O2, oxidation state of I varies from
CaSO4 or MgCl 2 or MgSO4. +7 to +5, i.e. decreases. Thus, KIO4 gets reduced hence, H2O2 is
Temporary hardness of water is also called carbonate hardness a reducing agent here.

or
ur
which can be easily removed by boiling or by treatment with With NH2OH, it given following reaction:

f
Ca(OH)2 (Clark’s method). -1 +3
N H2OH + H2O2 ¾® N 2 O3 + H2O
18. There are three known isotopes of hydrogen, each possessing an
ks
atomic number 1 and atomic masses 1, 2 and 3 respectively. In the above reaction, oxidation state of N varies from –1 to +3.
Yo
These are named as protium (1 H), deuterium Here, oxidation number increases, hence H2O2 is acting as an
oo
(2 H or D) and tritium (3 H or T) oxidising agent here.
Hence, (a) is the correct choice.
eB

The most common isotope is the ordinary hydrogen usually


called protium. It consists of one proton in the nucleus and an 26. Release of electron is known as reduction. So, H2O2 acts as
electron revolving around it. reducing agent when it releases electrons.
The second isotope of hydrogen is called heavy hydrogen or
r

Here, in reaction (II) and (IV), H2O2 releases two electron,


deuterium. It consists of one proton and one neutron in the
ou
ad

hence reaction (II) and (IV) is known as reduction.


nucleus and an electron revolving around it. The third isotope of
hydrogen is called tritium. It consists of one proton and two In reaction (I) and (III), two electrons are being added, so (I)
Y

neutrons in the nucleus and an electron revolving around it. and (III) represents oxidation.
19. Both reactions in their complete format are written below 27. Mg(HCO3 )2 on boiling decomposes to give white precipitate of
nd
Re

(i) In acidic medium, MgCO3 as:


-1 -2
2+ 4- + 3+ 3-
[Fe (CN)6 ] + H2 O2 + 2H ¾® [Fe (CN)6 ] + 2H2 O Heat
Fi

Mg(HCO3 )2 (aq) ¾¾® MgCO3 ¯ + H2O + CO2 ­


(ii) In alkaline medium,
-1 28. Electrolysis of aqueous Na 2SO4 gives H2 (g ) at cathode and
[Fe3+ (CN)6 ]3- + H2 O2 + 2OH- ¾®
O2 (g ) at anode.
[Fe2+ (CN)6 ]4- + O2 + 2H2O
29. Peroxodisulphuric acid (H2S2O8 ) on complete hydrolysis gives
Hence, H2O (for reaction (i)) and O2 + H2O
(for reaction (ii)) are produced as by product. two moles of H2SO4 and one mole of H2O2 as
O O
20. Mg can form basic carbonate while Li cannot. ½½ ½½
5 Mg2 + + 6 CO2–
3 + 7H 2O ¾® 4MgCO 3 × Mg(OH)2 HO ¾ S -¾¾ O ¾ O ¾¾- S ¾ OH ® 2H2SO4 + H2O2
×5H2O + 2 HCO-3 ½½ HO ¾ H H ¾ OH ½½
O O
21. Region 1 (Pre-heating zone)
Region 2 (Primary combustion zone, hottest zone) On partial hydrolysis, it gives one mole of H2SO4 and one mole
Region 3 (Internal zone) of peroxomonosulphuric acid as
Region 4 (Secondary reaction zone)
s-Block Elements 201

O O 40. When sodium metal is burnt in excess of air, mainly sodium


½½ ½½ peroxide (Na 2O2 ) with little sodium oxide (Na 2O) are formed.
HO ¾ S ¾ O ¾ O ¾¾ S ¾ OH ¾® H2SO4
½½ H ¾ OH ½½ 41. NaNO3 when heated, it decomposes in two stages as:
O O
O T < 500°C 1
NaNO3 ¾¾¾¾® NaNO2 + O2
½½ 2
+ HO ¾ S ¾ O ¾ O ¾ H NaNO2
T > 800°C
¾¾¾¾® Na 2O + N2 + O2
½½
O 42. In dilute solution of Na in liquid ammonia, solvated electrons
Peroxomonosulphuric acid
are present whose emission spectrum gives blue colouration to
30. In PbO2 , Pb is in +4 oxidation state and oxygen is in –2 oxidation solution.
state. In all other case, peroxide ion (O2-
2 ) is present. Na + NH3 ¾® Na + + NH3 (e- )

w
Solvated electron
31. Metallic lustre of any metal is due to oscillation of free electrons
present in the metal. Also, presence of solvated electrons and solvated Na + ion
32. H2O is reduced as well as oxidised giving H2 (g ) at cathode and makes solution highly conducting.
43. Zeolite acts as ion exchange resin and its Na + is exchanged with

Flo
O2 (g ) at anode.
33. KNO3 and other nitrates of alkli metals (except LiNO3) are H+ and Mg2+ ions present in hard water.
thermally stable.
44. Solubility of a salt is influenced by two major factors, lattice

ree
34. Sodium peroxide on treatment with dilute acid gives H2O2 energy and hydration energy. For greater solubility, there
Na 2O2 + H2SO4 ¾® Na 2SO4 + H2O2 should be smaller lattice energy and greater hydration energy.

F
35. Glauber’s salt is Na 2SO4 × 10H2O. 45. Both statements are correct but blue colour is due to presence of
36. D2O is commonly known as heavy water. solvated electron NH3 (e- ).

or
ur
37. Presence of solvated electrons makes solution of alkali metal in
liquid ammonia makes them strongly reducing agent.
46. Statement I is correct. Small size of Li + makes it highly
polarising, introduces predominant covalency in LiCl.

f
38. Lime treatment remove bicarbonate hardness by forming Statement II is incorrect, there is very large difference in
insoluble CaCO3 as
ks
electronegativity of Li and Cl.
Ca(HCO3 )2 + Ca(OH)2 ¾® 2CaCO3 ¯ + 2H2O
Yo
47. Alkali metal forms MH in which hydrogen is in –1 oxidation
oo
39. PLAN Paramagnetic character of species can be easily explained on state. Both statements are correct and statement –2 is correct
the basis of presence of unpaired electrons, i.e. compounds explanation of statement I.
eB

containing unpaired electron(s) is/are paramagnetic.


Reaction of alkali metals with ammonia depends upon the 48. Al + conc. NaOH ¾® NaAlO2 + H2 ­
physical state of ammonia whether it is in gaseous state or liquid 49. Na in liquid ammonia contain NH3 (e- ) which possesses charge
state. If ammonia is considered as a gas then reaction will be
r

and conduct electricity.


1
ou
ad

(a) Na + NH3 ¾® NaNH2 + H2 50. Occlusion is a phenomena in which particles are physically
(Excess) 2
trapped in voids.
Y

(NaNH2 + 1/2 H2 are diamagnetic)


51. I2 disproportionate in alkali giving NaI and NaIO3.
If ammonia is considered as a liquid then reaction will be
M + (x + y)NH3 ¾® [ M (NH3 )x ]+ + [ e(NH 3 ) y ]- 52. Sodium when burnt in excess of oxygen, gives sodium peroxide
nd
Re

· Ammoniated electron as major product.


· Blue colour D
Na + O2 ¾®
Fi

· Paramagnetic Na 2O2 + Na 2O
· Very strong reducing agent Major Minor

(b) K + O2 ¾® KO2 (K , O-2 )


+
53. The substance is Na 2O2. When Na 2O2 is dissolved in water, it
(Excess) Potassium superoxide
paramagnetic
forms NaOH and H2O2. In this case, NaOH is a strong base
while H2O2 is a weak acid.
(c) 3Cu + 8HNO3 ¾® 3Cu(NO3 )2 + 2NO + 4H2O
Paramagnetic Paramagnetic (i) Na 2O2 + 2H2O ¾® 2NaOH + H2O2
OH O H2O2 decolourises red litmus paper due to its bleaching
Et Et action which is due to its oxidising character.
(d) O2 H2O2 ¾® H2O + [O]
+ H2O2 Bleaches colour
of red litmus

OH O (ii) If the compound is Na 2O, it will hydrolyse to form NaOH.


2-ethylanthraquinol 2-ethylanthraquinone
Na 2O + H2O ¾® 2NaOH
Hence, option (a), (b) and (c) are correct choices. NaOH solution formed above will change colour of red
litmus paper into blue.
202 s-Block Elements

54. KMnO4 + H2O2 ¾® MnO2 + KOH + O2 4. Names of magnesium alloys are given by two letters followed by
OA RA two numbers. The common alloying elements are A (Aluminium),
FeSO4 + H2O2 ¾® Fe3+ + H2O Z (zinc), T (tin), M (manganese) etc. Numbers indicate
RA OA respective nominal compositions of main alloying elements, e.g.
55. 3M + N2 ¾® M 3 N2 ‘AZ 91’ implies the composition of the alloy as : Al = 9%, Zn =
A B 1% and Mg = 100 – (9 + 1) = 90%
M 3N2 + 6H2O ¾® 3M (OH)2 + 2NH3 Among the alloys given, Mg – Al (Magnalium ; Mg = 5%, Al
B C D =95%) is being light, tough and strong, hence it is used in
aircrafts.
M (OH)2 + CO2 ¾® MCO3
C Milkyness 5. The structures of beryllium chloride in the solid state and vapour
phase, respectively are dimeric and chain. In vapour phase at
M can be either Ca or Ba but essentially not Mg because
above 900°C, BeCl2 is monomeric having a linear structure Cl

w
Mg(OH)2 is very sparingly soluble in water.
¾Be¾Cl. The bonding in BeCl2 is covalent and Be atom
56. 2KNO3 (s) + 10K(s) ¾® 6K2O (s) + N2 (g ) accommodates 2 + 2 = 4 electrons in the two sp-hybrid orbitals.
57. In H2O2 , the peroxide ion (O2-
2 ) is unstable, has tendency to pass
Below 900°C, beryllium chloride in vapour phase exists as a

Flo
into stable oxide state (O2- ). Hence, H2O2 is a good oxidising mixture of monomer BeCl2 and dimer Be2Cl4.
agent while H2O is stable. 6. Magnesium powder burns in air to give MgO and Mg 3N2. MgO

ree
58. In Solvay process, NaHCO3 is extracted from the solution by does not combine with excess oxygen to give any superoxide.
fractional crystallisation, which is then heated to convert it into Mg reacts with nitrogen to form magnesium nitride (Mg 3N2 ).
Na 2CO3 × KHCO3 , being more soluble than NaHCO3 , cannot be
Mg + O2 ¾® MgO

F
extracted by fractional crystallisation. Hence, Solvay process
fails in production of K2CO3. 3Mg + N2 ¾® Mg3N2
7.

or
59. Water forms stronger intermolecular H-bonds, therefore it is
ur
liquid at room temperature while H2S cannot form such strong
Key Idea According to Fajan’s rule, degree of covalency
(ionic potential), f µ-polarisation power of the

f
intermolecular bonds, gas at room temperature.
cation µ charge on the cation
1
ks
Topic 2 Group II Elements µ .
size of the cation
Yo
1. In comparison to boron, beryllium has lesser nuclear charge and
oo
greater first ionisation enthalpy. Alkaline earth metals contains bipositive (H2+ ) ions in their
compounds.
Electronic configuration of Be(4 ) = 1s2 , 2s2.
eB

So, here
It possess completely filled s-orbitals. Hence, high amount of
(i) Charge on cation, i.e. + 2 is constant.
energy is required to pull the electron from the gaseous atom.
(ii) Halide present (X - ) is also constant.
r

Beryllium (4) lies left to the boron (5) and on moving from left
ou

to right an electron is added due to which nuclear charge So, the covalent character depends on the size of alkaline earth
ad

increases from Be to B. metal. As we move down the group, size of metal ion increases.
Y

2. The correct sequence of thermal stability of carbonates is Be2+ < Mg2+ < Ca 2+ < Sr 2+ < Ba 2+
MgCO3 < CaCO3 < SrCO3 < BaCO3 So, Be2+ readily forms covalent compounds like BeX 2, because
nd

On moving down the group, i.e. from Mg to Ba, atomic radius of very high positive charge density over its small size, so that it
Re

generally increases. It is due to the addition of shell. As a result, readily polarises anionic spherical electron cloud.
the atomic size increases. CO2- 3 is a large anion. Hence, more
Fi

8. (i) Washing soda (Na 2CO3 × 10H2O) is manufactured in Solvay


stabilised by Ba 2+ (large cation) and less stabilised by Mg2+ . process. In this method, CO2 gas is passed through a
Therefore, BaCO3 has highest thermal stability followed by conc. solution of NaCl saturated with NH3. It gives ammonium
SrCO3, CaCO3 and MgCO3. carbonate followed by ammonium hydrogen carbonate.
3. Baking soda (NaHCO3 ) is not a hydrated solid. Thus, ( X ) is not The obtained NH4HCO3 is treated with solution of NaCl which
result in the formation of NaHCO3. The crystal obtained are
baking soda. Thus, option (a) and (c) are incorrect. Dead burnt
heated to obtain Na 2CO3.
plaster (CaSO4 ) is obtained from gypsum via the formation of
plaster of Paris. NaCl + NH3 + CO 2 + H2O ¾® NaHCO3 + NH4Cl
1 D Crystallisation
380-393 K > 393K
CaSO4 ×2H2O ¾¾¾® CaSO4 × H2O ¾¾¾® CaSO4 2NaHCO3 ¾¾¾¾®Na 2CO3 ¾¾¾¾¾® Na 2CO3 × 10H2O
Gypsum -1/ 2 H 2 O 2 1
- H 2 O Dead burnt plaster - H2O, CO2
Plaster of Paris 2
(anhydrous) (ii) Mg(HCO3 )2 and Ca(HCO3 )2 cause temporary hardness to
Therefore, the reaction takes place as follows : water that can be easily removed by boiling.
< 373K > 373K
Na 2CO3 × 10H2O ¾¾® Na 2CO3 × H2O ¾¾® Na 2CO3 (iii) NaOH is manufactured by Castner-Kellner process.
-9H 2 O -H 2 O
Washing soda Monohydrate Anhydrous white powder
(X ) (Y ) (soda ash) (Z ) In this reaction, Na amalgam flows out and treated with
water to give NaOH and H2 gas. During electrolysis,
s-Block Elements 203

hydrogen is evolved at cathode and chlorine is evolved at in the group has lesser size and not comparable with the size of
anode, which are the by product of this process. sulphates. Hence, BeSO 4 is the right response.

Electrolysis
Carbon anode 2Cl –2e Cl2 13. Thermal stability of salts with common anion depends on
2NaCl(aq) + polarising power of cation. Greater the polarising power, lower
Hg cathode 2Na +2e
be their thermal stability. Hence,
BeCO3 (IV) < MgCO3 (II) < CaCO3 (III) < K2CO3 (I)
2Na (Na/Hg)
H 2O 14. The reaction involved is
2NaOH(aq) +H2
BaO2 + H2SO4 ¾® BaSO4 + H2O2
The most electronegative atom, oxygen, in BaSO4 and H2O2 has
(iv) Portland cement constitutes, tricalcium aluminosilicate, -2 and -1 oxidation state respectively.
3CaO ×Al 2O3. SiO2, i.e. Ca 3Al 2O6 × SiO2.
9. For group-2 metal hydroxides, basicity increases down the 15. Electrolysis of molten CaCl 2 gives calcium at cathode

w
group, as: Ca 2+ + 2e- ¾® Ca (at cathode)
Be(OH)2 < Mg(OH)2 < Ca(OH)2 < Sr(OH)2 < Ba(OH)2 In case of electrolysis in aqueous medium, less electropositive
This is because as the size of metal atom increases, M—OH H+ is reduced at cathode rather than Ca 2+.

Flo
bond length increases or M—OH bond become weaker thus
16. Ca(OH)2 + Ca(HCO3 )2 ¾® 2CaCO3 ¯ + 2H2O
readily breaks to release OH - ions which are responsible for the (Clark’s method)
basicity of these solutions.

ree
But Be(OH)2 shows amphoteric (basic as well as acidic) NaOCl + H2 ¾® NaOH + HOCl
character as it reacts with acid and alkali both which is shown in HO- + HCO-3 ¾® CO23- + H2O
the following reactions. Be(OH)2 as a base :

F
Be(OH)2 + 2HCl ¾® BeCl 2 + 2H2O Ca(HCO3 )2 + Na 2CO3 ¾® CaCO3 ¯ + 2NaHCO3
Be(OH)2 as an acid : 17. Magnesium ammonium phosphate is precipitated out.

or
ur
Be(OH)2 + 2NaOH ¾® Na 2[Be(OH)4 ] MgSO4 + NH4OH + Na 2HPO4 ¾® Mg(NH4 )PO4 ¯ + Na 2SO4

f
10. Among the four elements given, Na, Be, Mg and Ca, Be has 18. Si is used in solar cells, because of its semi-conductor properties.
highest IE value because of its smallest size and 2s2 valence shell
ks
configuration.
19. Anhydrous MgCl 2 is obtained by heating hydrated salt in stream
Yo
So, X-ray cannot cause ionisation from the material used, i.e. Be of dry HCl.
oo

in the tube window, which may cause interference in the study. 20. Heating MgCl 2 × 6H2O brings about partial dehydration as
eB

I
11. A saturated aqueous solution of M (NO3 )2 on crystallisation D
MgCl 2 × 6H2O ¾® Mg(OHCl) + HCl + 5H2O
II
will produce hydrated crystal, M (NO3 )2 × nH2O only when * * *
21. BaCO 3 + H 2SO 4 ¾® BaSO 4 + H 2O + CO 2 (C = C14)
hydration enthalpy (DH °hyd ) of M 2 + ion will be appreciably
r

22. Be in BeCl 2 is electron deficient, short of two lone pair of


ou
ad

more negative. electrons from stable octet. H2O has lone pair of electrons, reacts
Hydration of an ion depends on its size. Smaller the size of an with BeCl 2.
Y

ion, higher will be its charge density and as a result it will remain
more solvated (hydrated) through ion dipole interaction.
23. Alkaline earth metal salts have M 2+ ions which has very high
polarising power compared to polarising power of monovalent
nd

Size of group-2 metal ions increases on going down the group.


Re

metal ion (M + ) of alkali metal. Due to high polarising power of


So, their ability to form hydrated crystals follows the order:
Be2 + >> Mg 2 + >> Ca 2 + >> Sr 2 + >> Ba 2 + M 2+ , it associate more water than M + .
Fi

Thus, Ba(NO3 )2 is slightly or almost insoluble in water. 24. A = NH3 , B = CaCO3.


12. As we move down the group, size of metal increases. Be has Reactions involved are :
lower size while SO2-
4 has bigger size, that’s why BeSO 4 breaks Heat
3Ca + N2 ¾¾® Ca 3N2
easily and lattice energy becomes smaller but due to lower size Ca 3N2 + 6H2O ¾® 3Ca(OH)2 + 2NH3
of Be, water molecules are gathered around and hence hydration A
energy increases. On the other hand, rest of the metals, i.e. Ca, Ca(OH)2 + CO2 ¾® CaCO3 + H2O
Ba, Sr have bigger size and that’s why lattice energy is greater (From air) B
than hydration energy. 25. Basic strength (i) decreases from left to right in period and
Time saving technique In the question of finding hydration (ii) increases from top to bottom in group. Therefore,
energy only check the size of atom. Smaller sized atom has more NiO < MgO < SrO < K2O < Cs2O Basic strength
hydration energy. Thus, in this question Be is placed upper most

Download Chapter Test


http://tinyurl.com/y4mfo7t4 or
15
p-Block Elements-I

w
Topic 1 Group 13 Elements

Flo
Objective Questions I 8. B(OH)3 + NaOH w NaBO2 + Na[B(OH)4 ] + H2 O
(Only one correct option)

ree
How can this reaction is made to proceed in forward
1. The correct statements among I to III regarding group 13 direction? (2006, 3M)
element oxides are: (a) Addition of cis 1, 2-diol

F
(2019 Main, 9 April II)
(b) Addition of borax
I. Boron trioxide is acidic.
(c) Addition of trans 1, 2-diol
II. Oxides of aluminium and gallium are amphoteric.

or
ur
III. Oxides of indium and thallium are basic.
(d) Addition of Na 2HPO4
9. H3 BO3 is

f
(2003, 1M)
(a) I, II and III (b) I and III only
(c) I and II only (d) II and III only (a) monobasic acid and weak Lewis acid
ks
(b) monobasic and weak Bronsted acid
2. Diborane (B2 H6 ) reacts independently with O2 and H2 O to
Yo
(c) monobasic and strong Lewis acid
oo
produce, respectively. (2019 Main, 8 April I) (d) tribasic and weak Bronsted acid
(a) B2O3 and H3BO3 (b) B2O3 and [BH4 ]-
eB

10. In compounds of type ECl 3 , where E = B , P, As or Bi, the


(c) H3BO3 and B2O3 (d) HBO2 and H3BO3
angles Cl ¾ E ¾ Cl for different E are in the order
3. The relative stability of + 1 oxidation state of group 13 (1999, 2M)
r

elements follows the order (2019 Main, 11 Jan II)


(a) B > P = As = Bi (b) B > P > As > Bi
ou
ad

(a) Al < Ga < Tl < In (b) Al < Ga < In < Tl (c) B < P = As = Bi (d) B < P < As < Bi
(c) Tl < In < Ga < Al (d) Ga < Al < In < Tl
11. Moderate electrical conductivity is shown by
Y

(1982, 1M)
4. The number of 2-centre-2-electron and 3-centre-2-electron (a) silica (b) graphite
bonds in B2 H6 , respectively, are (2019 Main, 10 Jan II)
(c) diamond (d) None of these
nd

(a) 4 and 2 (b) 2 and 4


Re

(c) 2 and 2 (d) 2 and 1


Objective Questions II
Fi

5. The electronegativity of aluminium is similar to


(2019 Main, 10 Jan I)
(One or more than one correct option)
(a) lithium (b) carbon 12. Among the following, the correct statement(s) is(are)
(c) beryllium (d) boron (2017 Adv.)
(a) Al (CH3 )3 has the three-centre two-electron bonds in its dimeric
6. Aluminium is usually found in +3 oxidation state. In contrast,
structure
thallium exists in +1 and +3 oxidation states. This is due to
(2019 Main, 9 Jan I)
(b) The Lewis acidity of BCl 3 is greater than that of AlCl 3
(a) lattice effect (b) lanthanoid contraction (c) AlCl 3 has the three-centre two-electron bonds in its dimeric
(c) inert pair effect (d) diagonal relationship structure
(d) BH3 has the three-centre two-electron bonds in its dimeric
7. The increasing order of atomic radii of the following Group structure
13 elements is (2016 Adv.)
13. The crystalline form of borax has (2016 Adv.)
(a) Al < Ga < In < Tl
(a) tetranuclear [ B4O5 (OH)4 ]2 - unit
(b) Ga < Al < In < Tl
(c) Al < In < Ga < Tl (b) all boron atoms in the same plane
(d) Al < Ga < Tl < In (c) equal number of sp2 and sp3 hybridised boron atoms
(d) one terminal hydroxide per boron atom
p-Block Elements-I 205

14. The correct statement(s) for orthoboric acid is/are(2014 Adv.) Fill in the Blank
(a) It behaves as a weak acid in water due to self ionisation
(b) Acidity of its aqueous solution increases upon addition of
20. The two types of bonds present in B2 H6 are covalent and
ethylene glycol ……… (1994, 1M)
(c) It has a three-dimensional structure due to hydrogen bonding True/False
(d) It is a weak electrolyte in water
21. The basic nature of hydroxide of group 13 (group IIIA)
15. In the reaction, 2 X + B2 H6 ¾® [BH2 ( X )2 ] +[BH4 ] - decreases progressively down the group. (1993, 1M)
the amine(s) X is/are (2009)
22. All the Al ¾ Cl bonds in Al 2 Cl 6 are equivalent. (1989, 1M)
(a) NH3 (b) CH3NH2
(c) (CH3 )2 NH (d) (CH3 )3 N
Integer Answer Type Question

w
Assertion and Reason 23. Three moles of B2 H6 are completely reacted with methanol.
Read the following questions and answer as per the direction The number of moles of boron containing product formed is
given below : (2015 Adv.)

Flo
(a) Statement I is correct; Statement II is correct Statement II is the 24. The value of n in the molecular formula Ben Al2 Si 6O18 is
correct explanation of Statement I
(2010)
(b) Statement I is correct; Statement II is correct Statement II is

ree
not the correct explanation of Statement I Subjective Questions
(c) Statement I is correct; Statement II is incorrect
25. AlF3 is insoluble in anhydrous HF but when little KF is added
(d) Statement I is incorrect; Statement II is correct

F
to the compound it becomes soluble. On addition of BF3,
16. Statement I Boron always forms covalent bond. AlF3 is precipitated. Write the balanced chemical equations.
(2004, 2M)
Statement II The small size of B3+ favours formation of

or
covalent bond.
ur (2007, 3M) 26. (i) How is boron obtained from borax? Give chemical

f
equations with reaction conditions.
17. Statement I In water, orthoboric acid behaves as a weak
(ii) Write the structure of B2H6 and its reaction with HCl.
ks
monobasic acid.
(2002)
Yo
Statement II In water, orthoboric acid acts as a proton
oo
donor. (2007, 3M) 27. Compound X on reduction with LiAlH4 gives a hydride Y
18. Statement I Al(OH)3 is amphoteric in nature. containing 21.72% hydrogen alongwith other products. The
eB

compound Y reacts with air explosively resulting in boron


Statement II Al ¾ O and O ¾ H bonds can be broken with trioxide. Identify X and Y. Give balanced reactions involved
equal ease in Al (OH)3 . (1998 , 2M) in the formation of Y and its reaction with air.
r

Draw the structure of Y. (2001, 5M)


ou

Match the Column


ad

28. Aluminium sulphide gives a foul odour when it becomes


19. Match the following. (2006, 6M)
Y

damp. Write a balanced chemical equation for the reaction.


Column I Column II (1997, 2M)

A. 3+
® (BiO) +
p. Heat 29. Anhydrous AlCl 3 is covalent. From the data given below,
nd

Bi
Re

predict whether it would remain covalent or become ionic in


B. [AlO2 ]- ® Al(OH)3 q. Hydrolysis aqueous solution.
Fi

C. SiO44 - ® Si 2O67 - r. Acidification (Ionisation energy for Al = 5137 kJ mol –1


D. (B4O27 - ) ® [B(OH)3 ] s. Dilution by water DH hydration for Al 3+ = – 4665 kJ mol –1
DH hydration for Cl - = - 381 kJ mol -1
(1997, 2M)

Topic 2 Group 14 Elements


Objective Questions I (Only one correct option) (a) (SiO3 )2- (b) SiO2 (c) (SiO4 )4-
1. The C ¾ C bond length is maximum in (2019 Main, 12 April II) R
(a) graphite (b) C70 ½
(d) ¾( Si ¾ O )¾
n (R = Me)
(c) C60 (d) diamond ½
2. The basic structural unit of feldspar, zeolites, mica and R
asbestos is (2019 Main, 12 April I)
206 p-Block Elements-I

3. The correct order of catenation is (2019 Main, 10 April I) Objective Question II


(a) C > Sn > Si » Ge (b) Si > Sn > C > Ge
(c) C > Si > Ge » Sn (d) Ge > Sn > Si > C
(One or more than one correct option)
4. The amorphous form of silica is (2019 Main, 9 April II)
15. With respect to graphite and diamond, which of the
statement(s) given below is/are correct? (2012)
(a) tridymite (b) kieselguhr
(c) cristobalite (d) quartz (a) Graphite is harder than diamond
(b) Graphite has higher electrical conductivity than diamond.
5. C60 an allotrope of carbon contains (2019 Main, 9 April I) (c) Graphite has higher thermal conductivity than diamond.
(a) 16 hexagons and 16 pentagons (d) Graphite has higher C¾ C bond order than diamond
(b) 20 hexagons and 12 pentagons
(c) 12 hexagons and 20 pentagons Assertion and Reason
(d) 18 hexagons and 14 pentagons

w
(a) Statement I is correc;t Statement II is correct Statement II is
6. The element that does not show catenation is the correct explanation of Statement I
(2019 Main, 12 Jan II)
(b) Statement I is correct; Statement II is correct Statement II is
(a) Ge (b) Sn (c) Si (d) Pb
not the correct explanation of Statement I

Flo
7. The element that shows greater ability to form pp- pp multiple (c) Statement I is correct; Statement II is incorrect
bonds, is (2019 Main, 12 Jan II) (d) Statement I is incorrect; Statement II is correct

ree
(a) Ge (b) Si (c) Sn (d) C
16. Statement I Pb 4+ compounds are stronger oxidising
8. The chloride that cannot get hydrolysed is (2019 Main, 11 Jan I) agents than Sn 2+ compounds.
(a) SnCl 4 (b) CCl 4

F
(c) PbCl 4 (d) SiCl 4 Statement II The higher oxidation states for the group
14 elements are more stable for the heavier members of the
9. Correct statements among (I) to (IV) regarding silicones are:

or
group due to ‘inert pair effect’.
ur (2019 Main, 9 Jan I)
(2008, 3M)

I. They are polymers with hydrophobic character. 17. Statement I Between SiCl 4 and CCl 4 , only SiCl 4 reacts
II. They are biocompatible.
f
with water.
ks
III. In general, they have high thermal stability and low Statement II SiCl 4 is ionic and CCl 4 is covalent.
Yo
dielectric strength. (2001, S, 1M)
oo
IV. Usually, they are resistant to oxidation and used as greases.
(a) I and II only (b) I, II, III only Fill in the Blanks
eB

(c) I, II, III and IV (d) I, II and IV only


18. A liquid which is permanently supercooled is frequently
10. Name the structure of silicates in which three oxygen atoms of called ……… . (1997, 1M)
[ SiO4 ] 4- are shared is
r

(2005, 1M)
19. The recently discovered allotrope of carbon (e.g. C60 ) is
ou

(a) pyrosilicate (b) sheet silicate


ad

(c) linear chain silicate (d) three-dimensional silicate commonly known as ……… . (1994, 1M)
Y

11. Me2 SiCl 2 on hydrolysis will produce (2003, 1M)


20. The hydrolysis of trialkyl chlorosilane R3 SiCl, yields
(a) (Me)2 Si(OH)2 (b) (Me)2 Si ==O ……… . (1994, 1M)
(c) [ ¾ O ¾ (Me)2 Si ¾ O ¾]n (d) Me2SiCl(OH)
nd

21. The hydrolysis of alkyl substituted chlorosilanes gives


Re

12. Identify the correct order of acidic strength of ……… . (1991, 1M)
Fi

CO2 , CuO, CaO, H2 O. (2002, 3M)


(a) CaO < CuO < H2O < CO2 True/False
(b) H2O < CuO < CaO < CO2
22. The tendency for catenation is much higher for C than for
(c) CaO < H2O < CuO < CO2 Si. (1993, 1M)
(d) H2O < CO2 < CaO < CuO
23. Diamond is harder than graphite. (1993, 1M)
13. Which one of the following oxides is neutral ? (1996, 1M)
(a) CO (b) SnO2 (c) ZnO (d) SiO2 24. Graphite is a better lubricant on the moon than on the earth.
(1987, 1M)
14. Which of the following halides is least stable and has doubtful
existence? (1996, 1M) 25. Carbon tetrachloride burns in air when lighted to give
(a) CCl 4 (b) GeI4 (c) SnI4 (d) PbI4 phosgene gas. (1983, 1M)
p-Block Elements-I 207

Subjective Questions X Y Z

26. Starting from SiCl 4 , prepare the following in steps not Yeast Fermentation Ethanol
exceeding the number given in parenthesis (give reactions Mica Graphite Abrasive
only). Superphosphate Crystalline cubic Insulator
(i) Silicon Carbon fibres Layer structure Fertiliser
(ii) Linear silicon containing methyl group only Rock salt Diamond structure Reinforced
(iii) Na 2SiO3 (2001, 5M) plastics
27. Draw the structure of a cyclic silicate, (Si 3 O9 )6– with proper Carborundum Bone ash Preservative
(1989, 3M)
labelling. (1998, 4M)
30. Give reasons for the following in one or two sentences :
28. Write the balanced equation for the preparation of crystalline

w
silicon from SiCl 4 . (1990, 1M) “Graphite is used as a solid lubricant.” (1985, 1M)

29. Each entry in column X is in some way related to the entries 31. Give reason for the following in one or two sentences :
in columns Y and Z. Match the appropriate entries. “Solid carbon dioxide is known as dry ice.” (1983, 1M)

Flo
32. Carbon acts as an abrasive and also as a lubricant, explain.
(1981, 1M)

ree
Answers

F
Topic 1 Topic 2
1. (a) 2. (a) 3. (b) 4. (a) 1. (d) 2. (c) 3. (c) 4. (b)

or
5. (c) 6. (c)
ur
7. (b) 8. (a) 5. (b) 6. (d) 7. (d) 8. (b)
9. (a) 10. (b) 11. (b) 12. (a, b, c) 9. (d) 10. (a) 11. (c) 12. (a)
13. (a, c, d) 14. (b, d) 15. (a, b, c) 16. (a) 13. (a)
f 14. (d) 15. (b, d) 16. (a)
ks
17. (a) 18. (a) 17. (c) 18. glass 19. Buckminster
Yo
19. A ® q; B ® r; C ® p ; D ® q, r 20. ( R3SiO) 2 21. silicones 22. T 23. T
oo

20. Three centre two electron bond or banana bond. 24. T 25. F
eB

21. F 22. F 23. (3) 24. (3)


r
ou

Hints & Solutions


ad
Y

Topic 1 Group 13 Elements It gets hydrolysed readily to give boric acid.


1. All the given statements are correct. For group 13 elements, the B2H6 + 6H2O ¾¾® 2H3BO3 + 6H2 ­
nd
Re

acidic nature of oxides decreases and the basic nature of oxides Borane Orthoboric acid Dihydrogen

increases on moving from B to Tl. This is because as we move 3. The stability order of + 3 and + 1 oxidation states of group 13
Fi

down the group, the atomic size of elements goes on increasing,


elements will be:
whereas the ionisation energy decreases, due to which the
strength of metal oxide (MO) bond goes on decreasing. Thus, B3 + > Al 3 + > Ga 3 + > In 3 + >>Tl 3+
boron trioxide or boron oxide is acidic and reacts with basic (order of + 3 oxidation state)
oxides to give metal borates. Aluminium and gallium oxides are B+ <<Al + < Ga + < In + < Tl +
amphoteric while oxides of indium and thallium are basic in (order of + 1 oxidation state)
nature. The presence of two oxidation states in p-block elements is due
2. Diborane (B2H6 ) reacts independently with O2 and H2O to to the inert pair effect.
produce B2O3 and H3BO3 respectively. Diborane is a Because of the presence of poor shielding d and f -orbitals, as
colourless, highly toxic gas, having boiling point 180 K. we move from Ga to Tl, effective nuclear charge of these
Because of its inflammable nature, it catches fire spontaneously elements increases so as to hold the valence ns2 electrons
when exposed to air and burns in oxygen releasing an enormous tightly. It causes difficulty to the ionisation of ns2-electrons and
amount of energy as: it remains inert, only np1-electron ionises to give + 1 oxidation
B2H6 + 3O2 ¾¾® B2O3 + 3H2O + 1976 kJ/mol state.
208 p-Block Elements-I

4. The structure of B2H6 can be shown as : 10. In BCl 3, bond angle = 120°.
H H H In PCl 3 , AsCl 3 and BiCl 3 , central atom is sp3 hybridised.
H
H eH Since P, As and Bi are from the same group, bond angle
–2
2c 3
B B Þ B
3c–3e
B sp decreases down the group. Hence, overall order of bond
H angle is :
H
H H H B > P > As > Bi
H
In B2H6, four 2-centre-2-electron (2c - 2e) bonds are present in the 11. Graphite has layered structure and conducted electricity
same plane and two 3-centre-2-electron (3c - 2e) bonds are present in moderately. Silica and diamond have 3-dimensional
another plane. network structures and non-conducting.
5. Let, us consider the electronegativity values of the given elements, 12. (a) Three centre two
Group-1 Group-2 Group-13 Group-19 H H H electron bond

w
CH3
Period 2 Þ Li Be B C C CH3
(1 . 0) (1 . 5) (2 . 0) (2 . 5)
Al Al
Period 3 Þ Al CH3 C CH3
(1. 5) *

Flo
Z H H H
Be and Al show diagonal relationship which is based on their same
r
(b) BCl 3 is stronger Lewis acid than AlCl 3 due to greater
value (Z* is effective nuclear charge, r = atomic radius). So, they have

ree
extent of pp - pp back bonding in AlCl 3.
similar electronegativity.
(c) Three centre four
6. Due to inert pair effect, group-13 elements (ns2np1 ) show + 3 and + 1

F
electron bond
oxidation states in their compounds. Stability order of these oxidation Cl Cl Cl
states will be as,
Al Al

or

+ 3 oxidation states
ur Cl Cl Cl
B3+ > Al3+ > Ga 3+ > In3+ > Tl3+

f
(d) Three centre two
B3+ does not exist in free states. All B(III) compounds are covalent. electron bond
ks

+ 1 oxidation states H H H
Yo
B+ < Al+ < Ga + < In + < Tl+ B B
oo
H H H
B+ does not exist in ionic as well covalent compounds.
eB

7. Due to poor shielding of d-orbital in Ga, atomic radius of Ga is smaller 13. Na 2B4O7 × 10H2O (borax) is actually made of two
than that of Al. Thus, Ga < Al < In < Tl. tetrahedral and two triangular units, and is actually written
8. Orthoboric acid is a very weak acid, direct neutralisation does not asNa 2[B4O5 (OH)4 ]× 5H2O.
r

complete. However, addition of cis-diol allow the reaction to go to OH


ou
ad

completion by forming a stable complex with [B(OH)4 ]- as: s sp3


O—B O sp2
CH2 ¾ OH
Y

HO OH HO—B O B—OH
B + 2½ ¾® O
HO OH CH2 ¾ OH sp2
O—B
s
nd
Re

H2C ¾ O O ¾ CH2 sp3 OH


½ B ½ + 2H2O
H2C ¾ O O ¾ CH2 (a) Thus, correct.
Fi

(b) Boron atoms are in different planes thus, incorrect.


9. Orthoboric acid is a weak, monobasic, Lewis acid. (c) Two sp2 and two sp3-hybridised B atoms thus,
OH has deficiency of a lone-pair correct.
(Lewis acid) (d) Each boron has one ¾ OH group thus, correct.
HO—B
14. (a) It does not undergo self ionisation in water but accepts
OH an electron pair from water, so it behaves as weak
monobasic acid.
pp - pp backbonding between ‘B’ and ‘O’ decreases acid strength
greatly : H3BO3 + H2O r B(OH)-4 + H+
pp–p Hence, (a) is incorrect.
p
(b) When treated with 1, 2-dihydroxy or polyhydroxy
HO—B—OH compounds, they form chelate (ring complex) which
effectively remove [ B(OH)4 ]- species from solution
OH
and thereby produce maximum number of H3O+ or H+
ions, i.e. results in increased acidity.
p-Block Elements-I 209

(c) Boric acid crystallises in a layer structure in which planar Al(OH)3 + NaOH ¾® Na[Al(OH)4 ]
triangular BO3-
3 ` ions are bonded together through hydrogen
Acid
bonds. Therefore, both statements are correct and statement II is a
correct explanation of statement I.
One trigonal
planar B(OH) 3 unit 19. (A) Bi 3+ hydrolysis to (BiO)+ ¾ q.
H H
(B) [AlO2 ]- exist in basic medium, on acidification gives
H
H
H Al(OH)3 ¾ r.
H
B (C) Orthosilicate (SiO4-
4 ) on heating changes into pyrosilicate
H
H
H Si 2O6-
7 ¾ p.
B
H H
B H (D) Tetraborate ion [B4O72- ] on treatment with dil. acid
H H H hydrolysis gradually to orthoboric acid ¾ q, r.

w
H
H H H 20. Three centred two electron bonds.
B B
H H
H H H H
B B B Covalent bonds
H

Flo
H H
H H H H H
H H

ree
Three centred two electron
(d) In water the pK a value of H3BO3 is 9.25. bridged B—H—B bonds

H3BO3 + H2O r B(OH)-4 + H+ ; pKa = 9.25 12. The basic nature of hydroxide of group-13 increases from top to

F
So, it is a weak electrolyte in water. bottom due to increase in electropositive character.
15. Diborane (B2H6 ) undergoes unsymmetric cleavage with NH3 , 22. In Al 2Cl6 , Al–Cl bonds are not equivalent :

or
ur
primary and secondary amine while tertiary amine brings about Cl Cl Cl

f
symmetrical cleavage of B2H6 as : Al Al The bridged Al—Cl bonds
are different from terminal
H H H Cl Cl Cl
ks
Al—Cl bonds.
B B +
Yo
NH3 or 1° amine
or 2° amine 23. B2H6 + 6 CH3OH ¾® 2[B(OCH3 )3 ] + 6H2
oo
H H H
Unsymmetric
Therefore, from 3 moles of B2H6 , 6 moles of B(OCH3 )3 will be
X
eB

cleavage 24. BenAl 2Si6O18 , 2n + 6 + 24 - 36 = 0 Þ n = 3


+ –
[BH2(X)2] [BH4]
H H H
25. 3KF + AlF3 ¾® B2H5Cl + H2
+ 2R3N K3AlF6 + 3BF3 ¾® AlF3 ¯ + 3KBF4
r

B B 2H3B(R3N)
ppt
ou
ad

H H H
Symmetrical 26. (i) Na 2B4O7 + HCl ¾® NaCl + H3BO3
Y

cleavage
D
H3BO3 + HCl ¾® BCl 3 + H2O
16. Small size and high charge on B3+ makes it highly polarising.
Therefore, in most of its compounds, boron forms covalent D
BCl 3 + Al ¾®
nd

B + AlCl 3
Re

bonds.
H H H
Hence, both statement I and statement II are correct and (ii) B2H6 : B B
Fi

statement II is a correct explanation of statement I. H H H


17. Orthoboric acid is a weak, monobasic, Lewis acid and the poor It has 4 terminal B—H bonds. There are two B—H—B, three
acidic character is due to pp - pp backbondings as: centred two electron bridged bonds.
pp–pp B2H6 + HCl ¾® B2H5Cl + H2
Backbonding decreases electron
HO—B—OH deficiency at boron, decreases LiAlH4
27. Compound X ¾¾¾® Y a hydride + other compound.
its Lewis acid strength. Hydride Y contains 21.72% hydrogen.
OH
D
Y + O2 ¾® B2O3 + H2O
18. Due to small size and high charge on Al in Al(OH)3 the fission
Therefore, Y is a hydride of boron and it is obtained by reduction
ability of Al—O and O—H bonds become comparable and of X with LiAlH4. So, X is either BCl 3 or BF3.
compound can give both H+ and HO- under appropriate
4BCl 3 + LiAlH4 ¾® B2H6 + 3AlCl 3 + 3LiCl
reaction conditions as: X
1442443
Y Other products
Al(OH)3 + 3HCl ¾® AlCl 3 + 3H2O
Base Molar mass of B2H6 = 2 ´ 11 + 6 = 28
210 p-Block Elements-I

6 In SiO4-
4 ion, each Si atom is bonded to four oxygen atoms
% of H in B2H6 = ´ 100 = 21.5 » 21.72
28 tetrahedrally.
B2H6 + 3O2 ¾® B2O3 + 3H2O + Heat
Y 3. Catenation property is an unique property of group 14 elements.
Down the group 14, catenation power decreases as:
Structure of Y (B2H6 )
C > Si > Ge » Sn
H
Å Pb does not show catenation.
3
H 1.3 9ÅH
97° 1.1 4. Silica occurs in nature in several amorphous and crystalline
B B 122° forms. Kieselguhr is the amorphous form of silica. Quartz,
tridymite and cristobalite are crystalline forms of silica.
H H
5. C60 is aromatic allotrope of carbon containing 12 pentagons and
H 20 hexagons. It is a fullerene having a shape like soccer ball and

w
1.77 Å called Buckminster fullerene.

(a) There are 4 terminal B—H bonds. 6. The property of self-linking of atoms of an element through
(b) There are two 3-centre-2-electron B—H—B bridged covalent bonds to form straight or branched chains and rings of

Flo
bonds. different sizes is called catenation. Down the group, catenation
(c) Terminal H—B—H planes are perpendicular to bridged tendency decreases due to decrease in element bond strength.
B—H—B bonds. Carbon (C), silicon (Si), germanium (Ge), tin (Sn), lead (Pb) are

ree
group-14 elements.
28. Al 2S3 + 6H2O ¾® 2Al(OH)3 ¯ + 3H2S(g ) ­
Foul odour Catenation tendency is highest in carbon while silicon has second

F
Foul odour on damping of Al 2S3 is due to the formation of H2S highest tendency of catenation among all elements of family due
gas as shown above. to higher bond energy. The decreasing tendency of catenation

or
among group 14 elements is as follows:
29. The total hydration energy of AlCl 3
ur C >> Si > Ge » Sn
= Hydration energy of Al 3+ + 3 ´ Hydration energy of Cl -

f
However, Pb does not show catenation.
= - 4665 + 3 (- 381) kJ/mol
ks
= - 5808 kJ/mol 7. Carbon (C) has greatest ability to form stable pp-pp multiple
Yo
The above hydration energy is more than the energy required for bonds. 2p-orbitals of this element participate in the process. The
oo
ionisation of AlCl 3 into Al 3+ and 3Cl - . stability of multiple bonds of C is attributed to their closeness
with C-nucleus. Thus, the smaller size of C plays a significant
eB

Due to this reason, AlCl 3 becomes ionic in aqueous solution. In


aqueous solution, it is ionised completely as role in the process.
AlCl 3 + 6H2O ¾® [Al(H2O)6 ]3+ + 3Cl - 8. The compounds given are the tetrahalides
(MCl 4) of group 14 elements. For the hydrolysis, (nucleophilic
r
ou

substitution) of MCl 4 the nature of the M—Cl bond should be as:


ad

Topic 2 Groups 14 Elements


d+ d–
1. The C ¾ C bond length is maximum in diamond having value
Y

M——Cl
154 pm. Here, each carbon atom undergoes sp3 hybridisation
and linked to four other carbon atoms by using hybridised It must expand its covalency beyond 4 by the use
of its vacant d-orbital which will accommodate the
nd

orbitals in tetrahedral fashion. It has a rigid three-dimensional


Re

lone pair of electrons of H 2O (the–nucleophile).


network of carbon atoms.
sp3d
C ¾ C bond length within the layers of graphite is 141.5 pm. In
Fi

sp3
C60, C ¾ C distances between single and double bonds are 143.5 Cl3 M——Cl Cl3 M——Cl
pm and 138.3 pm respectively. d + d – OH2
d0
O
H H
2. The basic structural unit of feldspar, –HCl
Transition state
4-
zeolites, mica and asbestos is (SiO4 ) . These all are silicates.
sp3
All silicates involve two types of Si ¾ O bonds. M(OH)4
3H2O
Cl3 M—(OH)
–3HCl
(i) Terminal Si¾ O bonds in which oxygen is bonded to a
silicon and not other atom. Here, M can be Si, Sn and Pb because they have vacant
(ii) Bridging Si ¾ O ¾ Si bonds in which oxygen is bonded to nd-orbital. But, carbon is a member of second period (n = 2,
two silicon atoms. l = 0, 1),
O– s it does not have d-orbital (l = 2). So, CCl 4 will not be hydrolysed
and correct option is (b).
Si O–
9. Silicones are polysiloxanes with general chemical formula,
O– O– s s [R2SiO]n, where R is an organic group such as:
¾ CH3 , ¾ C2 H5 , ¾ C6 H5 etc.
p-Block Elements-I 211

Silicones have many useful properties: 14. PbI4 is least stable, has doubtful existence. It is due to inert pair
(i) They repel water and form watertight seals. effect, the stable oxidation state of lead is + 2.
(ii) They are heat resistant because of constancy of properties
over a wide range of temperature (- 100° to 250° C). 15. Diamond has a three-dimensional network structure, a hard
substance where graphite is soft due to layered structure.
(iii) Silicones are non-toxic.
In graphite, only three valence electrons are involved in bonding
(iv) Silicones are biocompatible because these do not support and one electron remain free giving electrical conductivity. In
microbiological growth and these have high gas diamond, all the four valence electrons are covalently bonded
permeability at room temperature. hence, insulator.
(v) They are resistant to O2, O3 and UV-radiation.
Diamond is better thermal conductor than graphite. Electrical
(vi) Silicones are formulated to be electrically insulative.
conductivity is due to availability of free electrons, thermal
(vii) Silicone grease is typically used as a lubricant for brake conduction is due to transfer of thermal vibrational energy from
components in automobiles, since it is stable at high one atom to another atom. A compact and precisely aligned

w
temperature, is not water soluble and is a odourless viscous crystals like diamond thus facilitate better movement of heat.
liquid.
In graphite C ¾ C bond acquire some double bond character,
10. In sheet silicates, three out of four oxygen of SiO4-
4 unit are hence, higher bond order than in diamond.

Flo
shared as shown below :
16. In group 13, 14, 15 as we descend down in group, the higher
– – oxidation state becomes less tenable due to inert pair effect.
Therefore, lead show +2 as stable oxidation state. Hence, Pb4+

ree

act as a strong oxidising agent, itself reduced to Pb2+ very easily.
– Both statement I and statement II are correct and statement II is a

F
– – – – correct explanation of statement I.
17. SiCl 4 reacts with water due to vacant d-orbitals available with

or
– – – –
ur Three oxygens of every
tetrahedra are
Si as:

f
shared with others
H2O SiCl4
ks
– – –
Yo

oo
– No such vacant d-orbitals are available with carbon, hence CCl 4

eB

– does not react wtih water. Otherwise, both SiCl 4 and CCl 4 are
covalent.
– – Statement I is correct but statement II is incorrect.
In pyrosilicates, there is only one shared oxygen, in linear chain
r

silicates, two oxygen per tetrahedra are shared while in 18. Glass is commonly known as supercooled liquid.
ou
ad

three-dimensional silicates, all four oxygens are shared. 19. Buckminster fullerene is the name of recently discovered
11. Me2SiCl 2 on hydrolysis yields a linear chain silicone as : allotrope of carbon.
Y

CH3 CH3 20. After dimerisation, no reactive function group remains.


½ ½
nd

Cl ¾ Si ¾ Cl + 2H2O ¾® HO ¾ Si ¾ OH + 2HCl R
Re

½ ½ ½
CH3 CH3 R2SiCl + H2O ¾¾® R ¾ Si ¾ OH
- HCl
Fi

½
R
CH3 CH3 R R
½ Polymerisation ½ ½ ½
nHO ¾ Si ¾ OH ¾¾¾¾® ¾
[ O ¾ Si ¾ O ¾]n ¾® R ¾ Si ¾ O ¾ Si ¾ R
½ ½ ½ ½
CH3 CH3 R R
Dimeric silicone
12. CO2 is acidic oxide, H2O is neutral, CaO is strongly basic and
CuO is weakly basic. Therefore, order of acid strength is : 21. Silicones are organosilicon polymers, obtained by hydrolysis of
alkyl substituted chlorosilanes.
CaO < CuO < H2O < CO2
22. Due to smaller size of carbon than silicon, C—C bond is
13. Carbon monoxide is a neutral oxide, all others are amphoteric : stronger than Si—Si bond, hence former is more likely to
SnO2 + 4HCl ¾® SnCl 4 + 2H2O extend than later.
ZnO + 2HCl ¾® ZnCl 2 + H2O
23. Graphite has a layered structure of hexagonal carbon rings
SiO2 + 2NaOH ¾® Na 2SiO3 + H2O stacked one over other which makes it slippery.
SnO2 and ZnO also react with NaOH. SiO2 is also attacked by On the other hand, in diamond, each carbon is tetrahedrally bond
H3PO4. to other four carbons extended in three dimensional space,
212 p-Block Elements-I

giving a giant, network structure. Due to this reason, diamond O–


is harder than graphite. 27. O = Oxygen
24. Graphite is better lubricant on moon than on earth because of = Silicon

absence of gravitational pull on the moon.
O O
25. Phosgene gas is obtained by treatment of CCl 4 with Si3O9
6–
superheated steam :
CCl 4 + H2O (vapour) ¾® COCl 2 + 2HCl O
O O
26. D 3Si + 4AlCl ;
(i) 3SiCl 4 + 4Al ¾® D
3 28. 3SiCl 4 + 4Al ¾® 4AlCl 3 + 3Si
Mg or Zn can also be used. Vapour Molten Volatilizes Crystalline

(ii) SiCl 4 + 2CH3MgCl ¾® (CH3 )2 SiCl 2 + 2MgCl 2 29.

w
X Y Z
OH Yeast Fermentation Ethanol
- HCl ½ Mica Layered structure Insulator
(CH3 )2 SiCl 2 + H2O ¾¾® CH3 ¾ Si ¾ CH3

Flo
½ Superphosphate Bone ash Fertiliser
OH Carbon fibres Graphite Reinforced plastics
CH3 CH3 CH3

ree
Rock salt Crystalline cubic Preservative
½ ½ ½
¾® ¾ O ¾ Si ¾ O ¾¾ Si ¾ O ¾¾ Si ¾ O Carborundum Diamond structure Abrasive
½ ½ ½

F
CH3 CH3 CH3 30. Graphite has layered structure and the adjacent layers are weakly
(iii) SiCl 4 + 4H2O ¾® Si(OH)4 + 4HCl associated giving slippery nature, used as solid lubricant.

or
ur
Unstable 31. Carbon dioxide solidifies at very low temperature, hence solid
CO2 is very cold, commonly known as dry ice. Also solid carbon

f
heat
Si(OH)4 ¾¾® SiO2 + 2H2O dioxide sublime, without passing through liquid state.
ks
D 32. The two common allotropes of carbon are diamond and graphite.
SiO2 + Na 2CO3 ¾¾® Na 2SiO3 + CO2
Yo
Diamond is the hardest, natural, substance, used as an abrasive
oo
while graphite is soft, used as a lubricant.
eB

Download Chapter Test


http://tinyurl.com/y3ctmvau or
r
ou
ad
Y
nd
Re
Fi
16
p-Block Elements-II

w
Flo
Topic 1 Elements and Compounds of Group 15 and 16
Objective Questions I (Only one correct option)

ree
1. The correct statement among the following is 8. The compound that does not produce nitrogen gas by the
(2019 Main, 12 April I) thermal decomposition is

F
(2018 Main)
(a) (SiH3 )3 N is planar and less basic than (CH3 )3 N. (a) Ba(N3 )2 (b) (NH4 )2 Cr2O7
(b) (SiH3 )3 N is pyramidal and more basic than (CH3 )3N. (c) NH4NO2 (d) (NH4 )2 SO4

or
ur
(c) (SiH3 )3 N is pyramidal and less basic than (CH3 )3 N. 9. The order of the oxidation state of the phosphorus atom in

f
(d) (SiH3 )3 N is planar and more basic than (CH3 )3 N. H3 PO2 , H3 PO4 , H3 PO3 and H4 P2 O6 is (2017 Adv.)

2. The number of pentagons in C60 and trigons (triangles) in (a) H3PO4 > H3PO2 > H3PO3 > H4P2O6
ks
white phosphorus, respectively, are (b) H3PO4 > H4P2O6 > H3PO3 > H3PO2
Yo
(2019 Main, 10 April II)
oo
(a) 20 and 3 (b) 12 and 4 (c) H3PO2 > H3PO3 > H4P2O6 > H3PO4
(c) 20 and 4 (d) 12 and 3 (d) H3PO3 > H3PO2 > H3PO4 > H4P2O6
eB

3. The oxoacid of sulphur that does not contain bond between 10. The species in which the N-atom is in a state of
sulphur atoms is (2019 Main 10 April I) sp hybridisation is (2016 Main)
(a) H2S2O3 (b) H2S2O4 (a) NO-2 (b) NO-3
r

(c) H2S2O7 (d) H2S4O6 (d) NO+2


ou

(c) NO2
ad

4. The correct order of the oxidation states of nitrogen in NO, 11. The pair in which phosphorus atoms have a formal oxidation
Y

NO2 , NO2 and N2 O3 is (2019 Main, 9 April I)


state of +3 is (2016 Main)
(a) NO2 < NO < N2O3 < N2O (b) N2O < NO < N2O3 < NO2 (a) pyrophosphorous and hypophosphoric acids
(c) O2 < N2O3 < NO < N2O (d) N2O < N2O3 < NO < NO2
nd

(b) orthophosphorous and hypophosphoric acids


Re

5. The pair that contains two P¾ H bonds in each of the (c) pyrophosphorous and pyrophosphoric acids
Fi

oxoacids is (2019 Main, 10 Jan II) (d) orthophosphorous and pyrophosphorous acids
(a) H4P2O5 and H4P2O6 (b) H3PO3 and H3PO2 12. The product formed in the reaction of SOCl 2 with white
(c) H4P2O5 and H3PO3 (d) H3PO2 and H4P2O5
phosphorus is (2014 Adv.)
6. When the first electron gain enthalpy (De g H ) of oxygen is (a) PCl 3 (b) SO2Cl 2
(c) SCl 2 (d) POCl 3
- 141 kJ/ mol, its second electron gain enthalpy is
(2019 Main, 9 Jan II) 13. Which of the following properties is not shown by NO?
(a) a positive value (a) It is paramagnetic in liquid state (2014 Main)
(b) a more negative value than the first (b) It is a neutral oxide
(c) almost the same as that of the first (c) It combines with oxygen to form nitrogen dioxide
(d) negative, but less negative than the first (d) Its bond order is 2.5
7. Good reducing nature of H3 PO2 is attributed to the presence 14. Concentrated nitric acid upon long standing, turns
of (2019 Main, 9 Jan II) yellow-brown due to the formation of (2013 Main)
(a) two P¾ H bonds (b) one P¾ H bond (a) NO (b) NO2
(c) two P¾ OH bonds (d) one P ¾ OH bond (c) N 2O (d) N 2O4
214 p-Block Elements-II

15. Which of the following is the wrong statement? (2013 Main) 27. Polyphosphates are used as water softening agents because
(a) ONCl and ONO - are not isoelectronic they (2002, 3M)
(b) O3 molecule is bent (a) form soluble complexes with anionic species
(c) Ozone is violet-black in solid state (b) precipitate anionic species
(d) Ozone is diamagnetic gas (c) form soluble complexes with cationic species
(d) precipitate cationic species
16. The reaction of white phosphorus with aqueous NaOH gives
phosphine alongwith another phosphorus containing 28. The number of S ¾ S bonds in sulphur trioxide trimer,
compound. The reaction type, the oxidation states of (S3 O9 ) is (2001, 1M)
phosphorus in phosphine and the other product respectively (a) three (b) two
are (2012) (c) one (d) zero
(a) redox reaction, - 3 and - 5 29. Ammonia can be dried by (2000, 1M)
redox reaction, 3 and + 5

w
(b) (a) conc. H2SO4 (b) P4O10
(c) disproportionation reaction, - 3 and + 5 (c) CaO (d) anhydrous CaCl 2
(d) disproportionation reaction, - 3 and + 3
30. Amongst H2 O, H2 S, H2 Se and H2 Te, the one with the
17. Which ordering of compounds is according to the

Flo
highest boiling point is (2000, 1M)
decreasing order of the oxidation state of nitrogen? (2012)
(a) H2O because of hydrogen bonding
(a) HNO3 , NO, NH4Cl, N2 (b) HNO3 , NO, N2 , NH4Cl
(b) H2Te because of higher molecular weight

ree
(c) HNO3 , NH4Cl, NO, N2 (d) NO, HNO3 , NH4Cl, N2
(c) H2S because of hydrogen bonding
18. Extra pure N 2 can be obtained by heating (2011) (d) H2Se because of lower molecular weight

F
(a) NH3 with CuO (b) NH4NO3
31. The correct order of acidic strength is (2000, 1M)
(c) (NH4 )2 Cr2O7 (d) Ba(N3 )2 (a) Cl 2O7 > SO2 > P4O10 (b) CO2 > N2O5 > SO3

or
ur
19. The reaction of P4 with X leads selectively to P4 O6 . The X, is (c) Na 2O > MgO > Al 2O3 (d) K 2O > CaO > MgO
(a) dry O2 (2009) 32. The number of P ¾ O ¾ P bonds in cyclic metaphosphoric

f
(b) a mixture of O2 and N2 acid is (2000, 1M)
ks
(c) moist O2 (a) zero (b) two (c) three (d) four
Yo
(d) O2 in the presence of aqueous NaOH 33. One mole of calcium phosphide on reaction with excess
oo
water gives (1999, 2M)
20. The percentage of p-character in the orbitals forming
(a) one mole of phosphine
eB

P—P bonds in P4 is (2007, 3M)


(b) two moles of phosphoric acid
(a) 25 (b) 33 (c) 50 (d) 75
(c) two moles of phosphine
21. Which of the following is not oxidised by O3 ? (2005, 1M) (d) one mole of phosphorus pentaoxide
r

(a) KI (b) FeSO4


34. Sodium thiosulphate is prepared by
ou

(1996, 1M)
ad

(c) KMnO4 (d) K 2MnO4


(a) reducing Na 2SO4 solution with H2S
22. Which gas is evolved when PbO2 is treated with (b) boiling Na 2SO3 solution with S in alkaline medium
Y

concentrated HNO3? (2005) (c) neutralising H2S2O3 solution with NaOH


(a) NO2 (b) O2 (d) boiling Na 2SO3 solution with S in acidic medium
nd

(c) N2 (d) N2O


Re

35. There is no S ¾ S bond in (1991, 1M)


23. A pale blue liquid obtained by equimolar mixture of two (a) S 2O2– (b) S 2O2– (c) S2O2– (d) S2O2–
Fi

4 5 3 7
gases at – 30°C is (2005, 1M)
(a) N2O (b) N2O3 36. Which one of the following is the strongest base? (1989, 2M)
(c) N2O4 (d) N2O5 (a) AsH3 (b) NH3 (c) PH3 (d) SbH3
24. Which of the following isomers of phosphorus is 37. Amongst the trihalides of nitrogen, which one is least basic?
thermodynamically most stable? (2005, 1M) (a) NF3 (b) NCl 3 (1987, 1M)
(a) Red (b) White (c) NBr3 (d) NI3
(c) Black (d) Yellow 38. Which of the following oxides of nitrogen is a coloured gas ?
25. Which of the following has —O—O— linkage? (2004, 3M) (a) N2O (b) NO (1987,1M)
(a) H2S2O6 (b) H2S2O8 (c) N2O4 (d) NO2
(c) H2S2O3 (d) H2S4O6
39. The bonds present in N2 O5 are (1986, 1M)
26. For H3PO3 and H3PO4, the correct choice is (2003, 1M) (a) only ionic (b) covalent and coordinate
(a) H3PO3 is dibasic and reducing (c) only covalent (d) covalent and ionic
(b) H3PO3 is dibasic and non-reducing
(c) H3PO4 is tribasic and reducing 40. A gas that cannot be collected over water is (1985, 1M)

(d) H3PO3 is tribasic and non-reducing (a) N2 (b) O2 (c) SO2 (d) PH3
p-Block Elements-II 215

41. Ammonia gas can be dried by (1978, 1M) Numerical Value


(a) conc H2SO4 (b) P2O5
(c) CaCl 2 (d) quicklime 51. The total number of compounds having at least one bridging
oxo group among the molecules given below is ……… .
42. Which of the following is incorrect statement? (1978, 1M)
(a) NO is heavier than O2 N 2O 3, N 2O 5, P4O 6, P4O7 , H 4 P2O 5 , H 5P3O10, H 2S 2O 3,
(b) The formula of heavy water is D2O H 2S 2O 5
(c) N2 diffuses faster than oxygen through an orifice (2018 Adv.)
(d) NH3 can be used as a refrigerant
Assertion and Reason
Objective Questions II Read the following questions and answer as per the direction
(One or more than one correct option) given below:

w
43. The compound(s) which generate (s) N2 gas upon thermal (a) Statement I is correct, Statement II is correct, Statement II
is the correct explanation of Statement I
decomposition below 300°C is (are) (2018 Adv.)
(b) Statement I is correct, Statement II is correct, Statement II
(a) NH4NO3 (b) (NH4 )2 Cr2O7
is not the correct explanation of Statement I

Flo
(c) Ba(N3 )2 (d) Mg3N2
(c) Statement I is correct, Statement II is incorrect
44. Based on the compounds of group 15 elements, the correct (d) Statement I is incorrect, Statement II is correct

ree
statement(s) is (are) (2018 Adv.)
52. Statement I Nitrogen and oxygen are the main components
(a) Bi 2O5 is more basic than N2O5
in the atmosphere but these do not react to form oxides of

F
(b) NF3 is more covalent than BiF3
nitrogen.
(c) PH3 boils at lower temperature than NH3
Statement II The reaction between nitrogen and oxygen
(d) The N—N single bond is stronger than the P—P single bond

or
ur requires high temperature. (1998, 2M)
45. The nitrogen containing compound produced in the reaction 53. Statement I The electronic structure of O3 is

f
of HNO 3 with P4 O10 (2016 Adv.) Å
ks
(a) can also be prepared by reaction of P4 and HNO 3 O
(b) is diamagnetic O Os
Yo
oo
(c) contains one N¾N bond
Statement II The following structure is not allowed
(d) reacts with Na metal producing a brown gas
because octet around O cannot be expanded.
eB

46. The correct statement(s) about O 3 is/are (2013 Adv.) O


(a) O — O bond lengths are equal O O (1998, 2M)
(b) thermal decomposition of O3 is endothermic
54. Statement I HNO3 is a stronger acid than HNO2 .
r

(c) O3 is diamagnetic in nature


ou
ad

(d) O3 has a bent structure Statement II In HNO3 , there are two nitrogen to oxygen
bonds whereas in HNO2 there is only one.
Y

47. The nitrogen oxide(s) that contain(s) N—N bond(s) is/are (1998, 2M)
(2009) 55. Statement I Although PF5 , PCl 5 and PBr5 are known, the
(a) N2O (b) N2O3
pentahalides of nitrogen have not been observed.
nd
Re

(c) N2 O4 (d) N 2O5


Statement II Phosphorus has lower electronegativity than
48. Ammonia, on reaction with hypochlorite anion, can form nitrogen.
Fi

(1994, 2M)
(1999, 3M)
(a) NO (b) NH4Cl Passage Based Questions
(c) N2H4 (d) HNO2
Passage
49. White phosphorus (P4 ) has (1998, 2M) Upon heating KClO 3 in presence of catalytic amount of
(a) six P ¾ P single bonds MnO 2 , a gas W is formed. Excess amount of W reacts with
(b) four P ¾ P single bonds white phosphorus to give X . The reaction of X with pure
(c) four lone pairs of electrons HNO 3 gives Y and Z. (2017 Adv.)
(d) P¾P¾P angle of 60°
56. Y and Z are, respectively
50. Nitrogen (I) oxide is produced by (1989, 1M) (a) N2O4 and HPO3 (b) N2O4 and H3PO3
(a) thermal decomposition of NH4NO3 (c) N2O3 and H3PO4 (d) N2O5 and HPO3
(b) disproportionation of N2O4
(c) thermal decomposition of NH4NO2
57. W and X are, respectively
(a) O2 and P4O10 (b) O2 and P4O6
(d) interaction of hydroxylamine and nitrous acid
(c) O3 and P4O6 (d) O3 and P4O10
216 p-Block Elements-II

Passage Codes
P Q R S P Q R S
There are some deposits of nitrates and phosphates in earth’s
(a) 4 2 3 1 (b) 3 2 1 4
crust. Nitrates are more soluble in water. Nitrates are
(c) 1 4 2 3 (d) 3 4 2 1
difficult to reduce under the laboratory conditions but
microbes do it easily. Ammonia forms large number of
complexes with transition metal ions. Hybridisation easily Fill in the Blanks
explains the ease of sigma donation capability of NH3 and 62. The lead chamber process involves oxidation of SO2 by
PH3 . Phosphine is a flammable gas and is prepared from atomic oxygen under the influence of ………as catalyst.
white phosphorus. (2008, 3 ´ 4M = 12M) (1992, 1M)
58. Among the following, the correct statement is 63. In P4 O10 , the number of oxygen atoms bonded to each
(a) Phosphates have no biological significance in humans phosphorus atom is ……… . (1992, 1M)

w
(b) Between nitrates and phosphates, phosphates are less abundant
in earth’s crust 64. The basicity of phosphorus acid (H3 PO3 ) is ………
(c) Between nitrates and phosphates, nitrates are less abundant in (1990, 1M)
earth’s crust 65. ……… phosphorus is reactive because of its highly strained

Flo
(d) Oxidation of nitrates is possible in soil tetrahedral structure. (1987, 1M)
59. Among the following, the correct statement is

ree
(a) Between NH3 and PH3 , NH3 is a better electron donor True/False
because the lone pair of electrons occupies spherical ‘s’ orbital 66. Nitric oxide, though an odd electron molecule, is

F
and is less directional diamagnetic in liquid state. (1991, 1M)
(b) Between NH3 and PH3 , PH3 is a better electron donor
because the lone pair of electrons occupies sp3-orbital and is
67. The H ¾ N ¾ H bond angle in NH3 is greater than the

or
more directional
ur H ¾ As ¾ H bond angle in AsH3 . (1984, 1M)

f
(c) Between NH3 and PH3 , NH3 is a better electron donor 68. In aqueous solution, chlorine is a stronger oxidising agent
because the lone pair of electrons occupies sp3-orbital and is than fluorine.
ks
(1984, 1M)
more directional 2+
Yo
69. Dilute HCl oxidises metallic Fe to Fe . (1983, 1M)
oo
(d) Between NH3 and PH3 , PH3 is a better electron donor
because the lone pair of electrons occupies spherical ‘s’ orbital
Integer Answer Type Question
eB

and is less directional


70. The total number of lone pair of electrons in N2O3 is
60. White phosphorus on reaction with NaOH gives PH3 as one
(2015 Adv.)
of the products. This is a
r

(a) dimerisation reaction (b) disproportionation reaction 71. Among the following, the number of compounds that can
ou

react with PCl 5 to give POCl 3 is O2 , CO2 , SO2 , H2 O,


ad

(c) condensation reaction (d) precipitation reaction


H2 SO4 , P4 O10 . (2011)
Y

Match the Columns 72. The total number of diprotic acids among the following is
61. The unbalanced chemical reactions given in Column I show H3PO4 H2SO4 H3PO3
nd
Re

missing reagent or condition (?) which are provided in H2CO3 H2S2O7 H3BO3
Column II. Match Column I with Column II and select the H3PO2 H2CrO4 H2SO3 (2010)
Fi

correct answer using the codes given below the Columns.


(2013 Adv.) Subjective Questions
Column I Column II 73. Draw the structure of P4O10. (2005)

P. 1. NO 74. Arrange the following oxides in the increasing order of


?
PbO2 + H 2SO4 ¾® PbSO4 + O2 + Bronsted basicity.
other product Cl2O7, BaO, SO3, CO2, B2O3 (2004)

? NaHSO + 2. I2 75. Identify the compounds A, B, C, D


Q. Na2S2O3 + H 2O ¾® 4
SO Na 2CO3 Elemental S I2
other product Na 2 CO3 ¾®
2
A ¾¾® B ¾¾¾¾® C ¾® D
D
? N + other product 3. Warm and give oxidation state of sulphur in each compounds.
R. N 2H 4 ¾® 2
(2003, 4M)
? Xe + other product 4. Cl 2 76. Write the balanced equations for the reactions of the
S. XeF2 ¾®
following compounds with water:
(i) Al 4C3 (ii) CaNCN (iii) BF3 (iv) NCl 3 (v) XeF4 (2002, 5M)
p-Block Elements-II 217

77. Give reason(s), why elemental nitrogen exists as a diatomic 84. Draw the structure of P4 O10 and identify the number of
molecule whereas elemental phosphorus is a tetra atomic single and double P ¾ O bonds. (1996, 3M)
molecule? (2000, 2M)
85. Account for the following. Write the answers in four or five
78. The Haber’s process can be represented by the following sentences only.
scheme.
(i) The experimentally determined N ¾ F bond lengths in NF3
CaCO3 is greater than the sum of the single bond covalent radii of N
and F.
CaO + CO2 (ii) Mg 3 N2 when reacted with water gives of NH3 but HCl is
H2O NH3.H2O not obtained from MgCl 2 on reaction with water at room
temperature.
B

w
(iii) (SiH3 )3 N is a weaker base than (CH3 )3 N.
NaHCO3 (1995, 2M ´ 3 = 6M)
NaCl
86. Complete and balance the following reactions. (1994, 1M)
+D
Heat

Flo
C + H 2O Ca 5 (PO4 )3 F + H2SO4 + H2O ¾¾®
NH3 + H2O + E K + 5CaSO4 × 2H2 O + K
A

ree
Identify A, B, C, D and E . (1999, 5M)
87. In the following reaction, identify the compounds A and B
PCl 5 + SO2 ¾® A + B (1994, 1M)
79 (a) In the following equation

F
A + 2B + H2 O ¾® C + 2D 88. Complete and balance the following reaction.
(A = HNO2 , B = H2SO3 , C = NH2OH). Red phosphorus is reacted with iodine in the presence of
water.

or
ur
Identify D. Draw the structures of A, B, C and D.
P + I2 + H2 O ¾® K + K (1992, 1M)
(b) In the contact process for industrial manufacture of

f
sulphuric acid, some amount of sulphuric acid is used as a 89. Give reasons in two or three sentences only.Sulphur dioxide
is a more powerful reducing agent in the alkaline medium
ks
starting material. Explain briefly. What is the catalyst used
than in acidic medium.
Yo
in the oxidation of SO2? (1999, 10M) (1992, 2M)
oo
80. Complete and balance the following chemical equations. 90. Draw the two resonance structures of ozone which satisfy
the octet rule. (1991, 1M)
(i) P4O10 + PCl 5 ¾®
eB

91. Give reasons in one or two sentences.


(ii) SnCl 4 + C2H5Cl + Na ¾® (1998, 1 M ´ 2 = 2M)
Ammonium chloride is acidic in liquid ammonia solvent.
81. (a) Thionyl chloride can be synthesised by chlorinating SO2 (1991, 1M)
r

using PCl5. Thionyl chloride is used to prepare


ou

92. Write the balanced chemical equations for the following.


ad

anhydrous ferric chloride starting from its hexahydrated


salt. Alternatively, the anhydrous ferric chloride can also (i) Sodium nitrite is produced by absorbing the oxides of
Y

be prepared from its hexahydrated salt by treating with nitrogen in aqueous solution of washing soda.
2,2-dimethoxypropane. Discuss all this using balanced (ii) Nitrogen is obtained in the reaction of aqueous ammonia
chemical equations.
nd

with potassium permanganate.


Re

(b) Reaction of phosphoric acid with Ca3(PO4)2 yields a (iii) Elemental phosphorus reacts with concentrated HNO3 to
fertiliser “triple superphosphate” represent the same
Fi

give phosphoric acid.


through balanced chemical equation. (1998, 5M)
(iv) Sulphur is precipitated in the reaction of hydrogen
82. A soluble compound of a poisonous element M, when sulphide with sodium bisulphite solution.
heated with Zn / H2 SO4 , gives a colourless and extremely (v) Carbon dioxide is passed through a suspension of
poisonous gaseous compound N, which on passing through limestone in water. (1991, 1 ´ 5 = 5M)
a heated tube gives a silvery mirror of element M. Identify
M and N. (1997, 2M) 93. Write the balanced chemical equation for the following
reactions.
83. Write balanced equations for the following.
(i) Aqueous solution of sodium nitrate is heated with zinc
(i) Phosphorus is treated with concentrated nitric acid. dust and caustic soda solution.
(ii) Oxidation of hydrogen peroxide with potassium (ii) Sodium iodate is added to a solution of sodium bisulphite
permanganate in acidic medium. (1990, 2M)
(iii) Manufacture of phosphoric acid from phosphorus. 94. Write the two resonance structures of N2 O that satisfy the
(iv) Reaction of aluminium with aqueous sodium hydroxide. octet rule. (1990, 2M)
(1997, 1M ´ 4 = 4M)
218 p-Block Elements-II

95. Draw balanced equations for 104. Write down the balanced equation for the reactions when
(i) the preparation of phosphine from CaO and white (i) calcium phosphate is heated with a mixture of sand and
phosphorus. carbon.
(ii) the preparation of ammonium sulphate from gypsum,
(ii) ammonium sulphate is heated with a mixture of nitric
ammonia and carbon dioxide. (1990, 2M)
oxide and nitrogen dioxide. (1985, 2M)
96. Explain the following (1989, 2M)
105. Draw the resonance structures of nitrous oxide.
(1985, 90, 2M)
(i) H3 PO3 is a dibasic acid.
(ii) Phosphine has lower boiling point than ammonia. 106. Show with balanced chemical reaction what happens when
following are mixed?
97. Write the balanced chemical equations for the following.
(i) Hypophosphorous acid is heated. Aqueous solution of ferric sulphate and potassium iodide.
(1984, 1M)
(ii) Sodium chlorate reacts with sulphur dioxide in dilute

w
sulphuric acid medium. 107. Write the matched set (of three) for each entry in Column A
98. Arrange the following as indicated. CO2 , N2 O5 , SiO2 , SO3 A B C
in the order of increasing acidic character. Asbestos Paramagnetic Air pollutant

Flo
99. Give balanced equations for the following. Lithium metal Silicates of Ca and Mg Electron donor
(i) Phosphorus reacts with nitric acid to give equimolar ratio

ree
Nitric oxide Reducing agent
of nitric oxide and nitrogen dioxide.
(ii) Carbon dioxide is passed through a concentrated aqueous (1984, 2M)
solution of sodium chloride saturated with ammonia. 108. Complete and balance the following reactions.

F
(1988, 3M)
(i) HNO3 + HCl ¾® NO + Cl 2
100. Give reason for “valency of oxygen is generally two, whereas

or
(ii) Ce3+ +S2O2– 2–
8 ¾® SO4 + Ce
4+
ur
sulphur shows valency of two, four and six.” (1988, 1M)
(iii) Cl 2 + OH– ¾® Cl – + ClO–

f
101. Explain the following in one or two sentences. (1983, 3M)
(i) Magnesium oxide is used for the lining of steel making 109. Explain, “orthophosphoric acid, H3 PO4 is tribasic but
ks
furnace.
Yo
phosphorous acid, H3 PO3 is dibasic”. (1982, 1M)
(ii) The mixture of hydrazine and hydrogen peroxide with a
oo
copper (II) catalyst is used as a rocket fuel. 110. Give structural formula for the following.
(iii) Orthophosphorous acid is not tribasic acid.
eB

(i) Phosphorous acid, H3 PO3


(iv) The molecule of magnesium chloride is linear, whereas (ii) Pyrophosphoric acid, H4 P2 O7 (1981, 2M)
that of stannous chloride is angular. (1987, 4M)
111. Sulphur melts to a clear mobile liquid at 119°C, but on
r

102. Write balanced equations for the following. (1987, 2M)


further heating above 160° C, it becomes viscous, explain.
ou
ad

(i) Phosphorus is reacted with boiling aqueous solution of (1981, 1M)


sodium hydroxide in an inert atmosphere.
Y

(ii) Dilute nitric acid is slowly reacted with metallic tin. 112. Explain the following in not more than two sentences .
103. Complete and balance the following reactions. (i) Conc. HNO3 turns yellow in sunlight.
nd

(ii) Bleaching powder loses its bleaching properties when it


Re

(i) S + OH– ¾® S2– + S2 O32– + ......


is kept in an open bottle for a long time. (1980, 2M)
(ii) ClO–3 + I– + H2 SO4 ¾® Cl – + HSO–4 + ...... + ......
Fi

(1986, 2M)

Topic 2 Elements and Compounds of Group 17 and 18


Objective Questions I (Only one correct option) 3. Iodine reacts with concentrated HNO3 to yield Y along with
1. The noble gas that does not occur in the atmosphere is other products. The oxidation state of iodine in Y , is
(2019 Main, 10 April II) (2019 Main, 12 Jan I)
(a) Ra (b) Kr (a) 1 (b) 3
(c) He (d) Ne (c) 7 (d) 5
2. Chlorine on reaction with hot and concentrated sodium 4. Among the following reactions of hydrogen with halogens,
hydroxide gives (2019 Main, 12 Jan II) the one that requires a catalyst is (2019 Main, 10 Jan II)
(a) Cl - and ClO- (b) Cl - and ClO3- (a) H2 + Cl 2 ¾® 2HCl (b) H2 + I2 ¾® 2HI
(c) H2 + F2 ¾® 2HF (d) H2 + Br2 ¾® 2HBr
(c) ClO3- and ClO-2 (d) Cl - and ClO-2
p-Block Elements-II 219

5. The type of hybridisation and number of lone pair(s) of 15. The set with correct order of acidic strength is (2001, 1M)
electrons of Xe in XeOF 4 , respectively, are (a) HClO < HClO2 < HClO3 < HClO4
(2019 Main, 10 Jan I) (b) HClO4 < HClO3 < HClO2 < HClO
(a) sp3d 2 and 1 (b) sp3d and 2 (c) HClO < HClO4 < HClO3 < HClO2
(c) sp3d and 1 (d) sp3d 2 and 2 (d) HClO4 < HClO2 < HClO3 < HClO

6. Which of the following reactions is an example of a redox 16. Which one of the following species is not a pseudo halide?
reaction? (2017 Main) (1997, 1M)
(a) XeF4 + O2F2 ¾® XeF6 + O2 (a) CNO– (b) RCOO- (c) OCN- (d) NNN-

(b) XeF2 + PF5 ¾® [XeF] + PF6- 17. The following acids have been arranged in the order of
decreasing acidic strength. Identify the correct order.
(c) XeF6 + H2O ¾® XeOF4 + 2HF
ClOH (I), BrOH (II), IOH (III) (1996, 1M)
(d) XeF6 + 2H2O ¾® XeO2F2 + 4HF

w
(a) I > II > III (b) II > I > III
(c) III > II > I (d) I > III > II
7. The products obtained when chlorine gas reacts with cold and
dilute aqueous NaOH are (2017 Main) 18. KF combines with HF to form KHF2 . The compound
contains the species

Flo
-
(a) ClO and ClO-3 (b) ClO-2and ClO-3 (1996, 1M)
(a) K + , F - and H+ (b) K + , F – and HF
(c) Cl - and ClO -
(d) Cl -
and ClO-2
(c) K + and [HF2 ] – (d) [KHF]+ and F –

ree
8. Which among the following is the most reactive?
(2015 Main) 19. Bromine can be liberated from potassium bromide solution
(a) Cl 2 (b) Br2 (c) I2 (d) ICl by the action of

F
(1987, 1M)
(a) iodine solution (b) chlorine water
9. Which one has highest boiling point? (2015 Main)
(c) sodium chloride (d) potassium iodide
(a) He (b) Ne (c) Kr (d) Xe

or
ur
10. Under ambient conditions, the total number of gases released 20. Chlorine acts as a bleaching agent only in the presence of

f
(1983, 1M)
as products in the final step of the reaction scheme shown (a) dry air (b) moisture
below is
ks
(2014 Adv.)
(c) sunlight (d) pure oxygen
Complete
Yo
hydrolysis 21. HBr and HI reduce sulphuric acid, HCl can reduce KMnO4
oo
XeF6 P + Other product
and HF can reduce (1981, 1M)
HO–/H2O
eB

(a) H2SO4 (b) KMnO4


Q (c) K 2Cr2O7 (d) None of these
Slow disproportionation
in HO–/H2O Objective Questions II
r
ou

(One or more than one correct option)


ad

Products
22. The correct statement(s) about the oxoacids, HClO4 and
Y

(a) 0 (b) 1 (c) 2 (d) 3


HClO, is (are) (2017 Adv.)
11. Among the following oxoacids, the correct decreasing order (a) The central atom in both HCl O4 and HClO is sp3-hybridised
nd

of acidic strength is (2014 Main)


Re

(a) HOCl > HClO2 > HClO3 > HClO4 (b) HCl O4 is formed in the reaction between Cl 2 and H2O
(b) HClO4 > HOCl > HClO2 > HClO3 (c) The conjugate base of HCl O4 is weaker base than H2O
Fi

(c) HClO4 > HClO3 > HClO2 > HOCl (d) HCl O4 is more acidic than HClO because of the resonance
(d) HClO2 > HClO4 > HClO3 > HOCl stabilisation of its anion

12. The shape of XeO2 F2 molecule is 23. The colour of the X 2 molecules of group 17 elements
(2012)
(a) trigonal bipyramidal (b) square planar changes gradually from yellow to violet down the group.
(c) tetrahedral (d) see-saw This is due to (2017 Adv.)
(a) decrease in p * - s * gap down the group
13. Aqueous solution of Na 2 S2 O3 on reaction with Cl 2 gives (b) decrease in ionisation energy down the group
(2008, 3M)
(c) the physical state of X 2 at room temperature changes from
(a) Na 2S4O6 (b) NaHSO4
gas to solid down the group
(c) NaCl (d) NaOH
(d) decreases in HOMO-LUMO gap down the group
14. When I- is oxidised by KMnO4 in alkaline medium, I- 24. The compound(s) with two lone pairs of electrons on the
converts into (2004, 1M) central atom is (are) (2016 Adv.)
(a) IO-3 (b) I2 (a) BrF5 (b) ClF3
(c) IO-4 (d) IO- (c) XeF4 (d) SF4
220 p-Block Elements-II

25. The correct statement(s) regarding, 30. Argon is used in arc welding because of its
(i) HClO, (ii) HClO2 , (iii) HClO3 and (iv) HClO4 is (are) (a) low reactivity with metal
(b) ability to lower the melting point of metal
(a) the number of Cl == O bonds in (ii) and (iii) together is two
(c) flammability
(b) the number of lone pair of electrons on Cl in (ii) and (iii)
(d) high calorific value
together is three
(c) the hybridisation of Cl in (iv) is sp3 31. The structure of XeO3 is
(d) amongst (i) to (iv), the strongest acid is (i) (a) linear (b) planar
(c) pyramidal (d) T-shaped
Passage Based Questions 32. XeF4 and XeF6 are expected to be
(a) oxidising (b) reducing
Passage 1
(c) unreactive (d) strongly basic

w
The reactions of Cl 2 gas with cold-dilute and hot-concentrated
NaOH in water give sodium salts of two (different) oxoacids
Match the Columns
of chlorine, P and Q, respectively. The Cl 2 gas reacts with
SO2 gas in the presence of charcoal, to give a product R. R 33. All the compounds listed in Column I react with water.

Flo
reacts with white phosphorus to give a compound S. On Match the result of the respective reactions with the
hydrolysis, S gives an oxoacid of phosphorus T. (2013 Adv.) appropriate options listed in Column II. (2010)

ree
26. P and Q respectively, are the sodium salts of Column I Column II
(a) hypochlorous and chloric acids
A. p. Hydrogen halide formation

F
(b) hypochlorous and chlorous acids (CH3 )2 SiCl 2
(c) chloric and perchloric acids B. XeF4 q. Redox reaction
(d) chloric and hypochlorous acids

or
27. R, S and T, respectively, are
ur C. Cl 2 r. Reacts with glass
D. VCl5 s. Polymerisation

f
(a) SO2Cl 2 , PCl 5 and H3PO4 (b) SO2Cl 2 , PCl 3 and H3PO3
(c) SOCl 2 , PCl 3 and H3PO2 (d) SOCl 2 , PCl 5 and H3PO4 t. O2 formation
ks
Yo
Passage 2 Fill in the Blanks
oo

Bleaching powder and bleach solution are produced on a 34. The increase in solubility of iodine in aqueous solution of KI
eB

large scale and used in several household products. The is due to the formation of ….. (1982, 94, 1M)
effectiveness of bleach solution is often measured by
iodometry. (2012) True/False
r

28. 25 mL of household bleach solution was mixed with 30 mL 35. HBr is a stronger acid than HI because of hydrogen
ou
ad

of 0.50 M KI and 10 mL of 4 N acetic acid. In the titration of bonding. (1993, 1M)


the liberated iodine, 48 mL of 0.25 N Na 2 S2 O3 was used to
Y

reach the end point. The molarity of the household bleach Integer Answer Type Questions
solution is 36. Reaction of Br2 with Na 2 CO3 in aqueous solution gives
(a) 0.48 M (b) 0.96 M
nd
Re

(c) 0.24 M (d) 0.024 M sodium bromide and sodium bromate with evolution of CO2
gas. The number of sodium bromide molecules involved in
Fi

29. Bleaching powder contains a salt of an oxoacid as one of its the balanced chemical equation is (2011)
components. The anhydride of that oxoacid is
(a) Cl 2O (b) Cl 2O7 (c) ClO2 (d) Cl 2O6 Subjective Questions
Passage 3 37. Write the balanced equation for the reaction of the following
compound with water.
The noble gases have closed-shell electronic configuration
and are monoatomic gases under normal conditions. The XeF4 (2002, 5M)
low boiling points of the lighter noble gases are due to weak 38. Draw molecular structures of XeF2 , XeF4 and XeO2 F2 ,
dispersion forces between the atoms and the absence of indicating the locations of lone pair(s) of electrons.
other interatomic interactions. (2000, 3M)
The direct reaction of xenon with fluorine leads to a series of 39. Give an example of oxidation of one halide by another
compounds with oxidation numbers + 2, + 4 and + 6. XeF4 halogen. Explain the feasibility of the reaction. (2000, 2M)
reacts violently with water to give XeO3 . The compounds of
xenon exhibit rich stereochemistry and their geometries can
40. Work out the following using chemical equations
be deduced considering the total number of electron pairs in “Chlorination of calcium hydroxide produces bleaching
the valence shell. (2007, 3 ´ 4M = 12 M) powder.” (1998, 2M)
p-Block Elements-II 221

41. Complete the following chemical equations: (ii) increasing oxidation number of iodine
(i) KI + Cl 2 ¾® (ii) KClO3 + I2 ¾® (1996, 2M) I2 , HI, HIO4 , ICl (1986, 2M)

42. Give reasons in two or three sentences only for 49. Give reason in one or two sentences.
(i) Bond dissociation energy of F2 is less than that of Cl 2 . Fluorine cannot be prepared from fluorides by chemical
reduction method. (1985, 1M)
(ii) Sulphur dioxide is a more powerful reducing agent in the
alkaline medium than in acidic medium. (1992, 2M) 50. Complete and balance the following reaction.
Cl 2 + OH– ¾® Cl – + ClO– (1983, 3M)
43. Write the balanced chemical equation for the following:
Sodium bromate reacts with fluorine in the presence of alkali. 51. Explain the following in not more than two sentences.
44. Arrange the following as indicated. HOCl, HOClO2 , Bleaching powder loses its bleaching properties when it is
HOClO3 , HOClO in increasing order of thermal stability kept in an open bottle for a long time. (1980, 2M)

w
(1988, 2M) 52. Give reasons for the following in one or two sentences.
45. Give balanced equation for the following: (i) Hydrogen bromide cannot be prepared by the action of
Iodate ion reacts with bisulphite ion to liberate iodine. (1988, 3M) conc. sulphuric acid on sodium bromide.

Flo
46. Mention the products formed in the following (ii) When a blue litmus paper is dipped into a solution of
hypochlorous acid, it first turns red and then later gets
“Chlorine gas is bubbled through a solution of ferrous
decolourised. (1979, 2M)

ree
bromide.” (1986, 2M)
53. Write the balanced equations involved in the preparation of
47. Complete and balance the following reaction:
ClO–3 + I– + H2SO4 ¾® Cl – + HSO–4 + ......+ ......
(i) bleaching powder from slaked lime (1979, 10M)

F
(1986, 2M)
(ii) nitric oxide from nitric acid
48. Arrange the following in the order of (iii) chlorine from sodium chloride

or
(i)
ur
increasing bond strength HCl, HBr, HF, HI (iv) anhydrous aluminium chloride from alumina

Answers
f
ks
Yo
Topic 1
oo
1. (a) 2. (b) 3. (c) 4. (b) 61. (d) 62. NO2 63. Four 64. Two
eB

5. (d) 6. (a) 7. (a) 8. (d) 65. white 66. T 67. T 68. F


9. (b) 10. (d) 11. (d) 12. (a) 69. T 70. (8) 71. 4 72. 6
13. (a) 14. (b) 15. (c) 16. (c)
Topic 2
r

17. (b) 18. (d) 19. (b) 20. (d)


ou

1. (a) 2. (b) 3. (d) 4. (b)


ad

21. (c) 22. (b) 23. (b) 24. (c)


5. (a) 6. (a) 7. (c) 8. (d)
25. (b) 26. (a) 27. (c) 28. (d)
Y

9. (d) 10. (c) 11. (c) 12. (a)


29. (c) 30. (a) 31. (a) 32. (c)
13. (a) 14. (a) 15. (a) 16. (b)
33. (c) 34. (b) 35. (d) 36. (b)
nd
Re

17. (a) 18. (c) 19. (b) 20. (b)


37. (a) 38. (d) 39. (b) 40. (c)
21. (d) 22. (a, c, d) 23. (b, c) 24. (b, c)
41. (d) 42. (a) 43. (b,c) 44. (a,b,c)
Fi

25. (b, c) 26. (a) 27. (a) 28. (c)


45. (b, d) 46. (a, c, d) 47. (a, b, c) 48. (c)
29. (a) 30. (a) 31. (c) 32. (a)
49. (a, c, d) 50. (a, d) 51. (6) 52. (a)
33. A ® p, s B ® p, q, r, t C ® p, q, t D® p
53. (a) 54. (a) 55. (b) 56. (a)
34. KI3 35. F 36. 5
57. (b) 58. (c) 59. (c) 60. (b)
Hints & Solutions
Topic 1 Elements and Compounds of H2 S 2 O3 (thiosulphuric acid), H2 S 2 O4 (hyposulphurous or
Group 15 and 16 dithionous acid) and H2 S 4 O6 (tetrathionic acid) contains S¾S
bonds.
1. The correct statement is that (SiH3 )3 N is planar and less basic O
than (CH3 )3 N. The compounds trimethylamine (CH3 )3 N and
trisilylamine (SiH3 )3 N have similar formulae, but have totally H2S2O3 Þ HO—S—OH
different structures. In trimethylamine the arrangement of
S
electrons is as follows :
O OH
1s 2s 2p H2S2O4 Þ

w
Electronic structure of S——S
nitrogen atom HO O
(ground state) O O
Three unpaired electrons
form bonds with CH3 groups
H2S4O6 Þ HO—S—S—S—S—OH

Flo
tetrahedral arrangements of
three bond pairs and one lone pair

In trisilylamine, three sp2 orbitals are used for O O

ree
s -bonding, giving a plane triangular structure. 4. The correct increasing order of oxidation state of nitrogen for
nitrogen oxides is
H3Si SiH3 +1 +2 +3 +4

F
N 2 O< N O < N 2 O3 < N O2
N N
H 3C CH3 ●
Oxidation state of N in N2O is

or
CH3
ur SiH3 2(x ) - 2 = 0
N(CH3)3 N(SiH3)3 2

f
molecule x = + = +1
molecule 2
ks

Oxidation state of N in NO is
Yo
2. In C60 (Buckminster fullerene) twenty hexagons and twelve x-2= 0
oo
pentagons are present which are interlocked resulting a shape x = +2
of soccer ball. Every ring in this structure is aromatic.
eB


Oxidation state of N in N2O3 is
2x + 3(-2) = 0
6
x= =3
r

2
ou
ad


Oxidation state of N in NO2 is
x + 2(-2) = 0
Y

x-4 = 0
x = +4
nd
Re

5. Let us consider the structure of the phosphorus oxyacids,


Phosphorus has large atomic size and less electronegativity, so
it forms single bond instead of pp-pp multiple bond. So, it
Fi

consists of discrete tetrahedral P4 molecule as shown below : P P


H OH HO H
P H OH
H3PO2 H3PO3
P P Hypophosphorus Orthophosphorus
acid acid
P (P ¾ H bonds=2) (P ¾ H bond=1)

\ Number of trigons (triangles) = 4


P P P P
3. S ¾ S bond is not present in H2 S 2 O7 (pyrosulphuric acid or H O H O OH
HO OH OH OH
oleum).
H4P2O5 H4P2O6
O O Pyrophosphorus Hypophousphoric
acid acid
H2S2O7 Þ HO—S—O—S—OH (P ¾ H bonds=2) (P ¾ H bond=0)

O O 6. As given, the first electron gain enthalpy of oxygen can be shown


as,
While the other given oxoacids of sulphur, i.e.
O(g )+ e- ¾® O - (g ), DegH 1 = - 141kJ/mol
p-Block Elements-II 223

The expression of second electron gain enthalpy of oxygen will be, 12. PLAN This problem is based on chemical properties of
O - (g )+ e- ¾® O 2- (g,) DegH 2 = + ve phosphorus.

DegH 2 of oxygen is positive, i.e. endothermic, because a strong White phosphorus on reaction with thionyl chloride (SOCl 2 )
electrostatic repulsion will be observed between highy negative O - produces phosphorus trichloride.
and the incoming electron (e- ). A very high amount of energy will P4 (s) + 8SOCl 2 (l ) ¾® 4PCl 3 (l ) + 4SO2 (g ) + 2S2Cl 2 (g )
be consumed (endothermic) by the system to overcome the But if amount of thionyl chloride (SOCl 2 ) is in excess then it
electrostatic repulsion. produces phosphorus pentachloride.
7. The structure of H3PO2 (hypophosphorous) acid is P4 + 10SOCl 2 (l ) ¾® 4PCl 5 + 10SO2
O 13. NO is paramagnetic in gaseous state because in gaseous state,
P it has one unpaired electron.
H
Total number of electrons present = 7 + 8 = 15 e -

w
HO H
Due to the presence of two P ¾ H bonds, H3 PO 2 acts a strong Hence, there must be the presence of unpaired electron in
reducing agent. e.g. gaseous state while in liquid state, it dimerises due to
+1 +1 0 +5 unpaired electron.

Flo
4 Ag NO 3 + H3PO 2 + 2H2O ¾® 4 Ag ¯ + H3 PO 4 + 4 HNO 3
8. The thermal decomposition of given compounds is shown below 14. NO 2 is a brown coloured gas and imparts this colour to
concentrated HNO 3 during long standing.

ree
D
(NH4 )2 Cr2O7 ¾® N2 + 4H2O + Cr2O3 4 HNO 3 ¾® 2H2O + 2NO 2 + 3O 2
-
D 15. (a) ONCl = 8 + 7 + 17 = 32 e

F
NH4NO2 ¾® N2 + 2H2O

D
ONO- = 8 + 7 + 8 + 1 = 24 e- (correct)
(NH4 )2 SO4 ¾® 2NH3 +H2SO4

or
ur (b)
O
Ba(N3 )2 ¾® Ba + 3N2

f
O O
Thus, only (NH4 )2 SO4 does not gives N2 on heating (It give NH3 ).
Central O-atom is sp 2 -hybridised with 1 lone pair, so
ks
While rest of the given compounds gives N2 on their thermal
decomposition. bent shape (correct).
Yo
oo
+5 +4 +3 +1
9. H3 P O 4 > H4 P2 O6 > H3 P O 3 > H3 P O 2 (c) In solid state, ozone is violet-black. Ozone does not
exist in solid state, thus incorrect.
eB

10. Species Hybridisation (d) O 3 has no unpaired electrons, so diamagnetic (correct).


Hence, (c) is the correct.
2
N
r

sp 16. The reaction of white phosphorus with aqueous alkali is



ou

O O
ad

O P4 + 3NaOH + 3H2O ¾® PH3 + NaH2PO2


In the above reaction, phosphorus is simultaneously
Y

N
– sp2 +1
O O oxidised [P4 (0) ¾® NaH2 P O2 ] as well as reduced
-3
nd
Re

N 2 [P4 (0) ¾® P H3 ]. Therefore, this is an example of


sp
O O disproportionation reaction. Oxidation number of
Fi

+
O N O sp phosphorus in PH3 is - 3 and in NaH2PO2 is + 1. However,
+ 1 oxidation number is not given in any option, one might
O
think that NaH2PO2 has gone to further decomposition on
½½
11. Orthophosphorous acid, H3PO3 : HO ¾ P ¾ OH heating.
½ D +5
H 2NaH2PO2 ¾® Na 2H P O4 + PH3
x
H3 PO3 = 3 + x + 3( -2 ) = 0 or x = + 3 17. Let oxidation number of N be x.
Pyrophosphorous acid, H 4 P2 O 5 : In HNO3, + 1 + x + 3 (- 2) = 0 Þ x = + 5
O O In NO, x-2=0 Þ x=+ 2
½½ ½½ In N2, x=0
HO ¾ P ¾ O ¾ P ¾ OH
In NH4Cl, x + 4 -1= 0 Þ x = - 3
½ ½
H H 18. Ba(N3 )2 ¾Heat
¾¾® Ba(s) + 3N2 (g )
x
H4 P2 O5 = 4 + 2x + 5 ( - 2) = 0 Azide salt of barium can be obtained in purest form as well as
the decomposition product contain solid Ba as by product
4 + 2x - 10 = 0, x = + 3
224 p-Block Elements-II

alongwith gaseous nitrogen, hence no additional step of 28. The structure of S3O9 is
separation is required. O O
Other reactions are S
Heat
NH4NO3 ¾¾® N2O + 2H2O O O
O O
Heat
2NH3 + 3CuO ¾¾® 3Cu + 3H2O + N2 S S
O O O
Heat
(NH4 )2 Cr2O7 ¾¾® Cr2O3 + 4H2O + N2 It has no S—S linkage.
19. In limited supply of oxygen, phosphorus is oxidised to its lower 29. CaO, a basic oxide, is most suitable for drying of basic ammonia.
oxide P4O6 while excess of oxygen gives P4O10. A mixture of O2 30. H2O, due to its ability to form intermolecular H-bonds.
and N2 is used for controlled oxidation of phosphorus into P4O6.
31. Corresponding acids are HClO4, H2SO3 and H3PO4. Hence, the

w
20. In P4, all phosphorus are sp3-hybridised and has 75%
order of acidic strength is
p-character.
Cl 2O7 > SO2 > P4O10
P
sp3 32. The structure of cyclic metaphosphate is

Flo

P P O O
P

ree
O O
P O O
– P P –

F
21. In KMnO4, Mn is already in its highest oxidation state (+7), O O
O
cannot be oxidised by any oxidising agent.
There is three P—O—P bonds.

or
22. PbO2 + HNO3 ¾® Pb(NO3 )2 + H2O + O2
ur 33. Ca 3P2 + 6H2O ¾® 3Ca(OH)2 + 2PH3

f
23. Equimolar amounts of NO and NO2 at –30°C gives N2O3 (l ) -
OH
which is a blue liquid. 34. Na 2SO3 + S ¾¾® Na 2S2O3
ks
D
-30 °C
Yo
NO(g ) + NO2 (g ) ¾¾® N2O3 (l ) 35. S2O27 - has no S—S linkage.
oo
( Blue) O O
½½ ½½
eB

24. Black phosphorus is thermodynamically most stable allotrope -


O— S — O— S — O-
of phosphorus. ½½ ½½
It is due to three dimensional, network structure of polymeric O O
black phosphorus.
r

All others have atleast one S—S linkage.


ou
ad

25. H2S2O8 is a peroxy acid, has—O—O—linkage 36. Amongst XH3 where ‘X ’ is group-15 elements, basic strength
O O decreases from top to bottom. Hence, NH3 is strongest base.
Y

½½ ½½ 37. The electron withdrawing inductive effect of halogen decreases


HO— S — O— O— S — OH electron density on nitrogen, lowers basic strength. Since,
½½ ½½
nd

fluorine is most electronegative, NF3 is least basic.


Re

O O
Peroxodisulphuric acid 38. NO2 (g ) is deep brown coloured.
Fi

26. H3PO3 is a dibasic, reducing acid. H3PO4 is tribasic, 39. In N2O5, there are s (sigma) covalent bonds, p (pi) bonds and
non-reducing acid. coordinate covalent bonds as
O O O O
½½ ½½ N N
H— P— OH HO— P— OH O
½ ½ O O
OH OH
Dibasic, reducing Tribasic, non -reducing 40. SO2 cannot be collected over water because it reacts with water
27. Polyphosphates are used as water softening agents because they forming H2SO3.
form soluble complexes with cationic species of hard water. SO2 + H2O ¾® H2SO3
Na 2[Na 4 (PO3 )6 ]+ CaSO4 ¾® Na 2[(Ca 2 (PO3 )6 ] + Na 2SO4 41. Quicklime (CaO) is used for drying NH3 gas because both are
Soluble complex
basic, do not react. On the other hand, H2SO4 and P2O5 are
acidic, reacts with ammonia forming salts. CaCl 2 forms
complex with ammonia.
p-Block Elements-II 225

42. NO is lighter than O2. (b) N2O5 has no unpaired electron and is thus, diamagnetic
thus, (b) is correct.
D2O is commonly known as heavy water.
(c)
N2 is lighter than O2, effuse at faster rate under identical O O
experimental conditions. NH3 liquefies at very low temperature.
N O N
Therefore, liquid NH3 is used as a refrigerant.
O O
43. Among the given compounds, those which generate N2 on thermal There is no N—N bond, thus, (c) is incorrect.
decomposition below 300°C are ammonium dichromate i.e., (d) N2O5 + Na ¾® NaNO3 + NO2
(NH4 )2Cr2 O7 and barium azide or nitride i.e., Ba(N3 )2 . Reactions N2O5 vapours are of brownish colour. Thus, (d) is
of their thermal decomposition are given below correct.
D
(i) (NH4 )2 Cr2O7 ¾¾¾¾® N2 ­ + Cr2O3 + 4H2O 46. Plan Due to resonance, bond lengths between two atoms are
Below 300 °C equal. Species is said to be diamagnetic if all electrons are

w
It is an exothemic reaction with paired.
DH = - 429.1 ± 3 kcal/ mol. Process is endothermic if it takes place with absorption of
D heat.
(ii) Ba(N3 )2 ¾¾¾¾¾¾¾¾® Ba + 3N2 ­

Flo
Around 160 ° and above
O 218 O
r
O
r
Ammonium nitrate (NH4NO 3) on heating below 300°C gives pm s
116.80°
O ®® s
N 2O as O O O

ree
O O
D
NH4NO3 ¾¾¾¾® N2O + 2H2O bent molecule all electrons paired thus, diamagnetic
below 300 °C
2 O 3 ¾® 3 O 2 DH ° = - 142 kJ mol -1

F
However, on rapid heating or explosion
Exothermic
(i.e. above 300°C) it gives off nitrogen as
Thus, (b) is incorrect. (a, c, d) are correct.

or
Rapid heating
¾® 2N2 + O2 + 4H2O
2NH4NO3 ¾ ¾ ¾ ¾ ¾
or explosion
ur 47. The structures of these oxides are

f
Magnesium nitride (Mg3N2 ) does not decompose at lower O O
ks
temperatures being comparatively more stable. Its thermal
N N ¾® O N N
decomposition requires a minimum temperature of 700°C and
Yo
proceeds as
oo

¾®
Mg3N2 ¾700¾-¾¾
1500°C
® 3Mg + N2 ­
(a) (b) O
eB

44. Statement wise explanation is


O O O O
(i) Statement (a) Bi 2O5 is a metallic oxide while N2O5 is a
non-metallic oxide. N N N N
r

Metallic oxides being ionic are basic in nature while non O


ou

¾
ad
®

®
metallic oxides being covalent are acidic in nature. This O O O O
confirms more basic nature of Bi 2O5 in comparison to N2O5.
Y

(c) (d)
Hence, this is a correct statement.
(ii) Statement (b) The electronegativity difference between N(3) and (a), (b), (c) have N—N bonds.
nd

F(4) is less as compared to the electronegativity difference


Re

between Bi (1.7) and F(4). More electronegativity difference


48. 2NH3 + OCl - ¾® H2N— NH2 + H2O + Cl -
leads to ionic compounds. Thus, NF3 must be more covalent in
Fi

49. The structure of P4 is


nature as compared to BiF3. Hence, this statement is also correct.
(iii) Statement (c) In NH3 intermolecular hydrogen bonding is P
present, which is altogether absent in PH3. Thus, PH3 boils at
lower temperature than NH3. P P
Hence, this is also a correct statement.
(iv) Statement (d) Due to smaller size of N the lone pair-lone pair
repulsion is more in N—N single bond as compared to O—P P
single bond. This results to weaker N—N single bond as
It has six P—P single bonds.
compared to P—P single bond. Hence, this statement is incorrect.
There are four lone pairs on four phosphorus. P—P—P
45. P4O10 is a dehydrating agent and converts HNO3 into N2O5 bond angles are of 60°.
2HNO3 ¾® N2O5 + H2O Heat
50. NH4NO3 ¾¾® N2O + 2H2O
P4O10 + 6H2O ¾® 4H3PO4
(a) P4 + 20HNO3 ¾® 4H3PO4 + 20NO2 + 4H2O NH2OH × HCl + NaNO2 ¾® NaCl + 2H2O + N2O
Thus, (a) is incorrect. However, NH4NO2 on heating gives N2.
226 p-Block Elements-II

51. The structures of various molecules given in problem are 8. H2S 2 O5


discussed below— O O
1. N 2 O3 It is the tautomeric mixture of following two
S S
structures— HO OH
O N N O
O N N O O O
O Conclusion This compound also does not contain any
Bridging bridging oxo group.
oxo group
Conclusion 1 bridging oxo group is present in the compound.
52. Both Statement I and Statement II are true and Statement II is
correct explanation of Statement I.
2. N 2 O5 It has following structure.
O O O 53. Both Statement I and Statement II are true and Statement II is
N N correct explanation of Statement I.

w
O O
Bridging oxo 54. Both Statement I and Statement II are true and Statement II
group
explains the Statement I appropriately. Nitrate ion (NO-3 ) is
Conclusion 1 bridging oxo group is present in the compound. more stable than nitrite ion :

Flo
3. P4 O6 -
O O
P ½½ ½

ree
O O -
O— N® O ¬¾® O == N® O (Resonance structure)
O ·· ··
P P
N ¨ N (Resonance structure)

F
O
-
O O O O O O-
P

or
ur
Conclusion 6 bridging oxo groups are present in the
55. Both Statement I and Statement II are independently correct
but reason is not the correct explanation of Statement I.

f
compound. Nitrogen does not has any vacant d-orbitals, it cannot expand
4. P4 O7 its valence shell beyond eight electrons, i.e. it cannot violate
ks
octet. Therefore, nitrogen forms only trihalides
Yo
P
O (NX 3 with eight electrons in valence shell of N).
oo
O
Phosphorus has vacant 3d-orbitals, it can expand its valence
O
shell beyond eight electrons, its both trihalides and
eB

P P O
O pentahalides exist.
O O Passage
P
r

MnO 2
Conclusion 6 bridging oxo groups are present in the KClO 3 ¾¾® KCl + O 2
ou
ad

D
compound. W

5. H4 P2 O5 D HNO 3
Y

O O O 2 + P4 ¾® P4O10 ¾¾® N 2O 5 + HPO 3


X Y Z

P P 56. (a)
nd
Re

H O H
OH OH 57. (b)
Conclusion 1 bridging oxo group is present in the compound.
Fi

Passage
6. H5 P3O10 58. Due to greater solubility in water and prone to microbial
O O O attack, nitrates are less abundant in earth’s crust.
59. NH3 is stronger Lewis base than PH3. In a group of hydrides,
P P P
HO O O OH basic strength decreases down the group.
OH OH OH 60. White phosphorus undergo disproportionation in alkaline
Conclusion 2 bridging oxo groups are present in the medium.
compound. P4 + NaOH ¾® PH3 + NaH2PO2
7. H2S 2 O3 Warm (3)
S 61. (P) 2PbO2 + 2H2SO4 ¾¾¾® 2PbSO4 + O2 + 2H2O
S Cl 2 ( 4 )
HO (Q) Na 2S2O3 + H2O ¾¾® NaHSO4 + HCl
OH
O I2 (2 )
(R) N2H4 ¾¾® N2 + Hl
Conclusion This compound does not contain any bridging
oxo group.
p-Block Elements-II 227

NO(1) 72. Diprotic acids = 6


(S) XeF2 ¾¾® Xe + NOF
O O O
Thus, P—(3), Q—(4), R—(2), S—(1)

— —


Oxides of N2
HO—S—OH P——OH HO—C—OH
62. NO2 : 2SO2 (g ) + O2 (g ) ¾¾¾¾® 2SO3 (g )
(NO2 ) O H OH
63. O O O O O


— —
— —
— —
O P O HO—S—O—S—OH HO—Cr—OH HO—S—OH
O O O
O
O Others are
O P P O
O O

w —

P HO—B—OH P
P
O O
HO OH OH OH H H OH
O

Flo
Triprotic Monobasic Monoprotic
Here four oxygen atoms are bonded to each phosphorus Lewis acid
atom. 73. O

ree
64. H3PO3 [O == PH(OH)2 ] is a dibasic acid. P
O O
65. White phosphorus has highly strained, tetrahedral

F
structure, therefore highly reactive. O
O
66. In liquid state, nitric oxide (NO) dimerises into (NO)2 O P P O

or
ur
and odd electrons disappear giving diamagnetic property.
¾® O == N— N== O (l )

f
2NO P
Paramagnetic Diamagnetic O O
O
ks
67. Both ‘N’ and ‘As’ in corresponding hydrides are (P4O10)
Yo
sp3-hybridised. If central atoms are from same group,
oo
bond angle decreases from top to bottom if all other 74. Cl 2O7 < SO3 < CO2 < B2O3 < BaO
things are similar. Hence, H—N—H bond angle in NH3 +4 +4 +2
eB

+2.5
is greater than H—As—H bond angle in AsH3. 75. A = NaH SO3 ; B = Na 2 SO3 ; C = Na 2 S2O3; D = Na 2 S4 O6
68. Halogens are all good oxidising agent and their oxidising 76. (i) Al 4C3 + 12H2O ¾® 4Al(OH)3 + 3CH4
power decreases from top to bottom (F2 to I2) in group.
r

(ii) CaNCN + 5H2O ¾® CaCO3 + 2NH4OH


Any halogen above in group oxidises halides down in
ou
ad

group from their aqueous solution. Hence, Cl 2 can (iii) 4BF3 + 3H2O ¾® H3BO3 + 3HBF4
oxidise Br - to Br2, I- to I2 but cannot oxidise F- to F2 (iv) NCl 3 + 3H2O ¾® NH3 + 3HOCl
Y

rather F2 can oxidise Cl - to Cl 2. (v) 2XeF4 + 3H2O ¾® Xe + XeO3 + F2 + 6HF


77. Nitrogen in N2 are bonded by one sigma and two pi bonds. Phosphorus
69. Fe is more electropositive than hydrogen, displaces H+
nd
Re

ions from acid solution as : and other elements of this period, due to larger size, are very less likely
to form pi bonds, hence P4 is formed in which there is no pi bonds.
Fi

Fe + 2HCl ¾® FeCl 2 + H2 ­ 78. In given scheme : A = Ca (OH)2


70. N2O3 has two proposed structures. B = NH4HCO3, C = Na 2CO3
·· D = NH4Cl and E = CaCl 2

· ··
·· ·· ·· ·· ·· ·· 79. (a) HNO2 + 2H2SO3 + H2O ¾® NH2OH + 2H2SO4
O == N ¾ O ¾ N == O and ·· N¾ N A B C D
·· ·· ·· O
·· ·· (b) In SO3 + H2O ¾® H2SO4, sulphuric acid is obtained in misty
O
·· form and the reaction is explosive. By adding H2SO4, above
In both cases, number of lone pair of electrons are eight. reaction is prevented :
71. PCl5 produces POCl3 with the following reagents H2SO4 + SO3 ¾® H2S2O7 (oleum)
PCl 5 + SO2 ¾® POCl 3 + SOCl 2 H2S2O7 + H2O ¾® 2H2SO4
PCl 5 + H2O ¾® POCl 3 + 2HCl In the contact process, V2O5 is used as catalyst.
2PCl 5 + H2SO4 ¾® SO2Cl 2 + 2POCl 3 + 2HCl 80. (i) P4O10 + 6PCl 5 ¾® 10 POCl 3
6PCl 5 + P4O10 ¾® 10POCl 3 (ii) SnCl 4 + 2C2H5Cl + 2Na ¾® Na 2SnCl 4 + C4H10
228 p-Block Elements-II

81. (a) PCl 5 + SO2 ¾® POCl 3 + SOCl 2 SO2 + 2OH - ¾® SO24- + 2H + + 2e-
(b) Ca 3 (PO4 )2 + 4H3PO4 ¾® 3Ca(H2PO4 )2 Hence, the above reaction proceeds in forward direction on
triple superphosphate increasing concentration of HO- ion. H+ is on product side,
adding H+ retards the reaction by sending it in backward
82. The poisonous element M may be As. On the basis of given
information direction.
Zn /HCl 90. + +
AsCl 3 + 6H ¾¾® AsH3 + 3HCl O O
N
– –
D O O O O
2AsH3 ¾® 2As + 3H2
M
83. (i) P4 + 20HNO3 ¾® 4H3PO4 + 20NO2 + 4H2O 91. Ammonia, in liquid state undergo self-ionisation as :
2NH3 q NH4+ + NH2-

w
(ii) 3KMnO4 + 5H2O2 + 3H2SO4 ¾® K2SO4
+ 2MnSO4 + 5O2 + 8H2O
Thus, addition of NH4Cl to liquid ammonia increases
(iii) P4 + 20HNO3 ¾® 4H3PO4 + 20NO2 + 4H2O
concentration of NH4+ in solution and NH4Cl act as acid.

Flo
(iv) 2Al + 2NaOH + 2H2O ¾® 2NaAlO2 + 3H2
84. 92. (i) Na 2CO3 + NO + NO2 ¾® 2NaNO2 + CO2
O
(ii) 2KMnO4 + 2NH3 ¾® 2MnO2 + 2KOH + 2H2O + N2

ree
O P O
(iii) P4 + 20HNO3 ¾® 4H3PO4 + 20NO2 + 4H2O
4 P==O bonds

F
O (iv) 2H2S + NaHSO3 + H+ ¾® 3S + 3H2O + Na +
O P O P O (v) CaCO3 + CO2 + H2O ¾® Ca(HCO3 )2

or
ur 93. (i) NaNO2 + Zn + NaOH ¾® 3Na 2ZnO2 + NH3 + H2O

f
P (ii) 2NaIO3 + 5NaHSO3 ¾® 3NaHSO4 + 2Na 2SO4
O O
O + I2 + H2O
ks
+ - - +
Yo
94. N ºº N— O ¬¾® N == N == O
oo
85. (i) The size of both nitrogen and fluorine are very small as well
D
as they have very high electron density. Thus in NF3, N and 95. (i) 15CaO + 4P4 ¾® 5Ca 3P2 + 3P2O5 ­
eB

F repel each other stretching the N—F bond. Hence, in NF3,


[Ca 3P2 + 6H2O ¾® 3Ca(OH)2 + 2PH3 ­ ] ´ 5
N—F bond lengths are greater than the sum of their single
bond covalent radii. 15CaO + 4P4 + 30H2O ¾® 15Ca(OH)2
(ii) Mg3N2 + 6H2O ¾® 3Mg(OH)2 + 2NH3 + 3P2O5 ­ + 10PH3 ­
r

MgCl 2 is a salt of strong acid HCl and strong base Mg(OH)2


ou

(ii) 2NH3 + CO2 + H2O ¾® (NH4 )2 CO3


ad

and therefore, not hydrolysed in aqueous solution.


(iii) In (SiH3 )3 N, the lone pair of nitrogen is involved in pp - dp CaSO4 + (NH4 )2 CO3 ¾® CaCO3 ¯ + (NH4 )2 SO4
Y

bonding, less available on nitrogen for donation to a Lewis gypsum


acid, a weaker Lewis base CaSO4 + 2NH3 + CO2 + H2O ¾® CaCO3 ¯ + (NH4 )2 SO4
nd
Re

pp-dp
96. (i) In H3PO3, there is only two replaceable H, hence dibasic
H3Si—N——SiH3
Fi

O
½½
SiH3 H— P — OH H—of OH are acidic, dibasic.
Carbon does not have any vacant d-orbitals, no such pp -dp ½
bonding occur in trimethyl amine, lone pair of nitrogen is OH
available for donation to Lewis acid, hence a stronger Lewis (ii) NH3 molecules are associated by intermolecular
base. H—bonds.
Heat D
86. Ca 5 (PO4 )3 F + 5H2SO4 + 10H2O ¾® 3H3PO4 97. (i) 2H3PO2 ¾® PH3 + H3PO4 (Disproportionation)
+ 5CaSO4 × 2H2O + HF hypophosphorus
acid
87. PCl 5 + SO2 ¾® POCl 3 + SOCl 2 +
A B (ii) NaClO3 + SO2 ¾10¾H¾® NaCl + S + 5H2O
88. Red phosphorus reacts with iodine in the presence of water to 98. SiO2 < CO2 < N2O5 < SO3
form H3PO3 and HI as–
2P + 3I2 + 6H2O ¾® 2H3PO3 + 6HI 99. (i) 4P + 10HNO3 + H2O ¾® 5NO + 5NO2 + 4H3PO4
89. SO2 acts as reducing agent on account of following reaction : (ii) NaCl + NH4OH + CO2 ¾® NH4Cl + NaHCO3
p-Block Elements-II 229

100. Oxygen lacks empty d-orbitals in its valence shell, cannot O O


violate octet rule, hence in most of its compound it show only ½½ ½½
divalency. On the other hand, sulphur has vacant 3d-orbitals in HO— P — OH O— P — OH
its valence shell, can violate octet rule, show di, tetra and hexa ½ ½
OH OH
valency. orthophosphoric acid orthophosphorus acid
(it has three acidic H) (only two acidic H, H directly
101. (i) MgO is used for the lining of steel making furnace because bonded to P is not acidic)
it forms slag with impurities, and thus helps in removing
them from iron. 110. O O O
(i) (ii)
(ii) The mixture of N2H4 and H2O2 (in presence of Cu(II) P P P
catalyst) is used as a rocket propellant because the reaction H OH HO O OH
OH HO OH
is highly exothermic and large volumes of gases is evolved. phosphorus acid (pyrosphosphoric acid)
N2H4 (l ) + 2H2O2 (l ) ¾® N2 (g ) + 4H2O (g ) (P is sp3-hybridised)

w
(iii) In orthophosphorus acid (H3PO3 ) only two of the three H
111. Rhombic sulphur has a eight membered puckered ring structure.
are replaceable as
On heating ring tends to break and linear chain sulphur is
O
formed. When sulphur melts, the S8 rings slip and roll over

Flo
½½
one another very easily. It gives rise to a clear mobile liquid.
H— P — OH
½ When liquid sulphur is further heated to higher temperature,

ree
OH rings are broken giving long chain sulphur molecules. This
long chain molecules of sulphur gets entangled into one
(Only H of —OH are acidic)
another increasing viscosity of melt.

F
(iv) In MgCl 2, Mg is sp-hybridised while in SnCl 2, Sn is
sp2-hybridised with a lone pair at Sn. Hence, MgCl 2 is 112. (i) In the presence of sunlight, concentrated nitric acid
decomposes partially as

or
linear while SnCl 2 is angular.
ur hn
Conc. HNO3 ¾® NO2 + H+ + O2
Inert atm.

f
102. (i) P4 + 3NaOH + 3H2O ¾¾¾® 3NaH2PO2 + PH3
(phosphine)
It is the NO2 which impart yellow colouration to nitric acid.
ks
(ii) 4Sn + 10HNO3 ¾® 4Sn(NO3 )2 + NH4NO3 + 3H2O (ii) The bleaching action of bleaching powder is due to
Yo
dil presence of available chlorine, but in contact of moisture,
oo
it releases chlorine decreasing the amount of available
103. (i) 4S + 6OH- ¾® 2S2 - + S2O23- + 3H2O chlorine. Hence, bleaching property decreases gradually as
eB

(ii) ClO3– + 6I– + 6H2SO4 ¾® Cl – + 6HSO–4 +3I2 + 3H2O


bleaching powder is kept in open container for long time.

D Topic 2 Element and Compound of


104. (i) 2Ca 3 (PO4 )2 + 6SiO2 + 10C ¾® 6CaSiO3
r

+ 10CO + P4 Group 17 and 18


ou
ad

(white) 1. Radium (Ra) is a radioactive element. Ra belongs to group 2


(ii) (NH4 )2 SO4 + NO + NO2 ¾® 2N2 + 3H2O + H2SO4 (alkaline earth metals), it is not a noble gas.
Y

Note In question noble gas which does not exist in the


+ - - + atmosphere is asked and answer is Ra. But Ra (radium) is an
105. N ºº N— O ¬¾® N== N ==O
nd
Re

alkaline earth metal and not noble gas. It can be Rn (radon) and
106. Fe2 (SO4 )3 + 2KI ¾® 2FeSO4 + K2SO4 + I2 is misprint in JEE Main Paper.
Fi

In the above reaction, strong reducing agent, iodide, reducing 2. Halogens form halates and halides with hot and concentrated
ferric salt into ferrous salt. solution of NaOH as :
3 X 2 + 6NaOH ¾® 5NaX + NaXO3 + 3H2 O
107. A B C So, Cl2 will also give Cl- (as NaCl) and ClO3-
Asbestos Silicates of Ca and Mg Donar (as NaClO3 ) in the above reaction.
Lithium metal Reducing agent Electron donor Thus, option (b) is correct.
Note When halogens react with cold and dilute solution of
Nitric oxide Paramagnetic Air pollutant
NaOH, hypohalites and halides are produced as:
108. (i) 2HNO3 + 6HCl ¾® 2NO + 3Cl 2 + 4H2O X 2 + 2NaOH ¾® NaX + NaXO + H 2O

(ii) 2Ce3 + + S2O28- ¾® 2SO24- + 2Ce4 + 3. Iodine reacts with concentrated HNO 3to yield HIO 3 along
(iii) Cl 2 + 2OH - -
¾® Cl + ClO + H2O - with NO 2 and H2 O. The reaction involved in as follows :
I2 + 10HNO3 ¾® 2HIO3 + 10NO2 + 4H2O
109. Orthophosphoric acid (H3PO4 ) has three replaceable (acidic) The oxidation state of ‘I’ in HIO 3 is + 5 as calculated below :
hydrogen while orthophosphorus acid (H3PO3 ) has only two 1 + x + 3(-2) = 0
replaceable hydrogen.
x - 5 = 0, x = + 5
230 p-Block Elements-II

4. Chemical reactivity of halogens decreases down the group. Complete sequence of reaction can be shown as
XeF6+3H2O XeO3+ 3H2F2
The chemical reactivity follows the order.

\ F2 > Cl2 > Br2 > I2. OH

The highest reactivity of fluorine is attributed to two factors: HXeO–4


(i) The low dissociation energy of F ¾ F bond (which results in
OH–/H2O (disproportionation)
low attraction energy for the reaction).
(ii) Very strong bonds which are formed. Both properties arise
XeO64-(s) + Xe(g) + H2O(l ) + O2(g)
from, small size of fluorine. I2 is being the least reactive
halogen, it requires a catalyst for the reaction. Thus, (c) is the correct answer.
H2 + I2 ¾® 2HI 11. Decreasing order of strength of oxoacids
5. In XeOF4 , Xe is sp3d 2-hybridised. Geometry of the molecule is HClO4 > HClO3 > HClO2 > HOCl

w
octahedral, but shape of the molecule is square pyramidal. Reason Consider the structures of conjugate bases of each
According to VSEPR, theory it has one p bond. Remaining six oxyacids of chlorine.
electron pairs form an octahedron with one position occupied by a O

Flo
lone pair.
Cl Cl
O Cl –
F F O O
– O O O Cl—O –
O–

ree
O
Number of bp = 5 O
Xe
Number of lp = 1
Negative charge is more delocalised on ClO4- due to

F
F F resonance, hence, ClO-4 is more stable (and less basic).
Hence, we can say as the number of oxygen atom(s) around
Here, Xe contains one lone pair of electrons.

or
ur Cl-atom increases as oxidation number of Cl-atom increases and
6. The reaction in which oxidation and reduction occur thus, the ability of loose the H+ increases.

f
simultaneously are termed as redox reaction.
12. In XeO2F2, the bonding arrangement around the central atom Xe is
ks
+4 + 1 + 6 0
XeF4 + O2 ( F2 ) ¾® X eF6 + O 2 O Xe O
Yo
oo
Since, Xe undergoes oxidation while O undergoes reduction. F F
So, it is an example of redox reaction. 4s bonds + 1.0 l p = 5
eB

7. Cl 2, Br2 and I2 form a mixture of halide and hypohalites when Hybridisation of Xe = sp3d
react with cold dilute alkalies while a mixture of halides and
sp3d-hybridisation corresponds to trigonal bipyramidal geometry.
haloate when react with concentrated cold alkalies.
Also, in trigonal bipyramidal geometry, lone pairs remain
r

Cl 2 + 2NaOH ¾® NaCl + NaClO + H2O


present on equatorial positions in order to give less electronic
ou
ad

Cold and dilute


repulsion.
\ Cl and ClO- are obtained as products when chlorine gas
-
F F
Y

reacts with cold and dilute aqueous NaOH. O O


Remove
8. Interhalogen compounds are generally more reactive than Xe Xe
lone-pair
nd

halogens (except fluorine).


Re

O O
9. Xe has highest boiling point. F F
See-saw shape
Fi

10. PLAN This problem can be solved by using concept involved in


NOTE According to Bent’s rule, the more electronegative atoms must
chemical properties of xenon oxide and xenon fluoride.
be present on axial position. Hence, F are kept on axial
XeF6 on complete hydrolysis produces XeO3. positions.
XeO3 on reaction with OH- produces HXeO-4 which on further 13. Sodium thiosulphate, Na 2S2O3 gets oxidised by chlorine water
treatment with OH- undergo slow disproportionation reaction as Na 2S2O3 + 4Cl 2 + 5H2O ¾® 2NaHSO4 + 8HCl
and produces XeO64- along with Xe(g ), H2O(l ) and O2 (g ) as a FeCl 3 oxidises Na 2S2O3 to Na 2S4O6.
by-product.
14. I– is oxidised by MnO–4 in alkaline medium to form IO–3
Oxidation half-cell in basic aqueous solution
2KMnO4 + KI + H2O ¾® 2KOH + 2MnO2 + KIO3
HXeO- + 5OH- ¾® XeO4- + 3H O + 2e–
4 6 2
15. Amongst oxyacids of a given halogen, higher the oxidation
Reduction half-cell in basic aqueous solution number of halogen, stronger the acid. Hence,
HXeO- + 3H O + 6e- ¾® Xe + 7OH-
4 2 HOCl < HClO2 < HClO3 < HClO4.
Balanced overall disproportionation reaction is 16. Pseudo halides must contain atleast one nitrogen atom.
4HXeO-4 + 8OH- ¾® 3XeO64- + Xe + 6H2O
14 4244 3
2 products
p-Block Elements-II 231

17. Among oxyacids of halogens, if there are same number of (b) Number of Lone Pair on Cl in (ii) and (iii) together is three.
oxygens bonded to central atom, higher the electronegativity of Hence, correct.
halogen, stronger the acid. Hence, (c) In (iv), Cl is sp3-hybridised. Hence, correct.
IOH < BrOH < ClOH (d) Amongst (i) to (iv), the strongest acid is (iv). Hence, wrong.
18. All others has at least one S-S linkage. Passage 1 Q. Nos. (26-27)
KF + HF ¾® K+ + HF2- Cold
2NaOH + Cl 2 ¾¾® NaCl + NaOCl + H2O
19. Among halogens, oxidising power decreases from top to bottom. P
Hence, the upper halogen oxidises lower halides from aqueous hot
6NaOH + 3Cl 2 ¾¾® 5NaCl + NaClO3 + 3H2O
solution. Chlorine will oxidise bromide into bromine. Q
20. Moist chlorine gives nascent oxygen, act as oxidising agent : NaOH
HOCl ¾¾® NaOCl
Cl 2 + H2O ¾® HCl + HOCl hypochlorous

w
P
HOCl ¾® HCl + [O] acid
nascent oxygen NaOH
(bleaching action) HClO3 ¾¾® NaClO3
21. Fluorine, being the most electronegative, its size is very small. chloric acid Q

Flo
Therefore, it does not have a tendency to loose electrons. Hence, Cl 2 + SO2 ¾® SO2Cl 2
HF does not act as a reducing agent. R
10 SO2Cl 2 + P4 ¾® 4PCl 5 + 10 SO2

ree
22. (a) ClO-4 is more stable than ClO .-
R S
(b) Incorrect : Cl 2 + H2O ¾® HCl + HOCl PCl 5 + 4H2O ¾® H3PO4 + 5 HCl

F
(c) O
T
Passage 2 Q.Nos. (28-29)
HO Cl O HO Cl

or
28. The involved redox reactions are :
O
sp3
ur sp3
2H+ + OCl - + 2I- ¾® Cl - + I2 + H2O …(i)

f
(d) HClO4 is stronger acid than H2 O. -
I2 + 2S2O2-
3 ¾® 2I + S4O62- …(ii)
ks
23. Colour of halogen arises due to transition from HOMO to LUMO
in the visible region. On moving down a group, the difference in Also the n-factor of S2O2-
3 is one as
Yo
2S2O3 ¾® S4O62- + 2e-
2-
oo
energy between HOMO and LUMO decreases electronic
transition occur more easily and colour intensity increases.
[one ‘e’ is produced per unit of S2O2-
3 ]
eB

24.
Þ Molarity of Na 2S2O3 = 0.25 N ´ 1 = 0.25 M
Lone pair on
Compounds Hydridisation Structures
central atom Þ m mol of Na 2S2O3 used up = 0.25 ´ 48 = 12
Now from stoichiometry of reaction (ii)
r

F F
ou

12 m mol of S2O2-
3 would have reduced 6 m mol of I2.
ad

BrF5 3 2
sp d Br 1
F F From stoichiometry of reaction (i)
Y

F m mol of OCl - reduced = m mol in I2 produced = 6


F
6
F Þ Molarity of household bleach solution = = 0.24 M
nd
Re

ClF3 sp3d Cl 2 25
Shortcut Method
Fi

F
Milliequivalent of Na 2S2O3 = milliequivalent of OCl -
F F
XeF4 sp3d 2 = 0.25 ´ 48 = 12
Xe 2
F F Also n-factor of OCl - = 2 [Cl + ¾® Cl - , gain of 2e- ]
12
SF4 sp3d 1 Þ m mol of OCl - = = 6 m mol. Remaining part is solved in
2
25. the same manner.
H ¾ O ¾ Cl H ¾ O ¾ Cl == O
(i) (ii) 29. Bleaching powder is Ca(OCl)Cl. Therefore, the oxoacid whose
O O salt is present in bleaching powder is HOCl. Anhydride of HOCl
is Cl 2O as
==

==

H ¾ O ¾ Cl == O H ¾ O ¾ Cl == O
2 HOCl ¾® Cl 2O + H2O
==

O NOTE The oxidation number of element in anhydride and oxoacid


(iii) (iv) remains the same.

(a) Number of Cl - == O bonds in (ii) and Passage 3 Q.Nos. (30-32)


(iii) together is three. Hence, wrong.
232 p-Block Elements-II

30. Ar, being inert, provide inert atmosphere in arc welding, and (ii) Sulphur dioxide is a more powerful reducing agent in
prevent from undesired oxidation. alkaline medium because nascent hydrogen is produced in
·· the presence of moisture
31. O == Xe == O
i.e. SO2 + 2H2 O ¾® H2 SO4 + 2H
½½
O And alkaline solution neutralises the acid i.e. H2 SO4 and
3 shift the equilibrium in the forward direction producing
Xe is sp -hybridised with one lone pair. Hence, molecule of more nascent hydrogen. But in acidic medium the
XeO3 has pyramidal shape. equilibrium will suppressed resulting in a lesser amount
32. Both XeF4 and XeF6 are strong oxidising agent. of nascent hydrogen.

33. (CH3 )2 SiCl 2 + H2O ¾® (CH3 )2 Si(OH)2 + 2HCl 43. NaBrO3 + 3F2 ¾® 3F2O + NaBr

w
CH3 CH3 Cl 44. HOCl < HOClO < HOClO2 < HOClO3
Polymerisation
—Si—O—Si—O—Si—O 45. 2IO-3 + 5HSO-3 ¾® I2 + H2O + 3HSO-4 + 2SO24-
CH3 CH3 CH3 46. Cl 2 + FeBr2 ¾® FeCl 3 + Br2

Flo
silicone
3 47. ClO-3 + 6I- + 6H2SO4 ¾® Cl - + 6HSO-4 + 3I2 + 3H2O
3XeF4 + 6H2O ¾® XeO3 + 2Xe + 12HF + O2

ree
2 48. (i) Bond strength is inversely related to bond length. Hence,
1 bond energy : HI < HBr < HCl < HF
Cl 2 + H2O ¾® HCl + HOCl ¾® HCl + O2

F
2 (ii) HI(– 1) < I2 (0) < ICl(+1) < HIO4 (+7)
VCl 5 + H2O ¾® VOCl 3 + 2HCl
49. F2 itself, is the strongest oxidising agent. Therefore, chemical

or
34. KI + I2 ¾® KI3
ur reagent cannot oxidise fluoride to fluorine.

f
35. Among HX, acidic strength increases from HF to HI.
50. Complete and balance the following reactions
ks
36. Br2 is disproportionated in basic medium as Cl 2 + 2OH- ¾® Cl - + ClO- + H2O
3Br2 + 3Na 2CO3 ¾¾® 5NaBr + NaBrO3 + 3CO2
Yo
oo
51. The bleaching action of bleaching powder is due to presence of
37. 2XeF4 + 3H2O ¾® Xe + XeO3 + F2 + 6HF available chlorine, but in contact of moisture, it releases chlorine
eB

38. F F decreasing the amount of available chlorine. Hence, bleaching


F F O property decreases gradually as bleaching powder is kept in open
container for long time.
Xe Xe Xe 52. (i) HBr is a stronger reducing agent, reduces cencentrated
r
ou

H2 SO4 to SO2 . Hence, HBr cannot be prepared by heating


ad

F F
O bromide salts with concentrated H2 SO4 .
F F
Y

linear square planar see-saw shaped (ii) Hypochlorous acid is acidic in nature, therefore it turns
39. Halogen above in the group oxidises halide below to it from blue litmus paper into red. However, HOCl is also an
nd

their aqueous solution, e.g. oxidising acid (bleaching), it bleaches red colour to
Re

Cl 2 + 2I- (aq) ¾® 2Cl - + I2 finally colourless.


Fi

40 °C
40 °C
40. Ca(OH)2 + Cl 2 ¾¾® CaOCl 2 + H2O 53. (i) Ca(OH)2 + Cl 2 ¾® CaOCl 2 + H2 O

41. (i) 2KI + Cl 2 ¾¾® 2KCl + I2 (ii) 3Cu + 8HNO3 (dil) ¾® 3Cu(NO3 )2 + 4H2 O + 2NO
(ii) 2KClO3 + I2 ¾¾® 2KIO3 + Cl 2 (iii) 2NaCl + 2H2 SO4 + MnO2 ¾® Na 2 SO4 + MnSO4
42. (i) Due to small size and high electron density of fluorine + 2H2O + Cl 2
atom, there exist a significant repulsions between D
fluorine atoms in F2 , they have greater tendency to get (iv) Al 2 O3 + 3C + 3Cl 2 ¾® 2AlCl 3 + 3CO
apart. Hence, bond energy of F2 is less than that of Cl 2 .
This is against to bond-length bond-energy
relationship,.

Download Chapter Test


http://tinyurl.com/y2yzc2dp or
17
Transition and
Inner-Transition Elements

w
Flo
Objective Questions I (Only one correct option) KOH, O2
9 A ¾4¾ ¾¾ ¾® 2B + 2H 2O
(Green)
1. Thermal decomposition of a Mn compound (X ) at 513 K

ree
results in compound (Y ), MnO2 and a gaseous product. MnO2 3B ¾ ¾¾® 2C
4 HCl
+ MnO2 + 2H 2O
reacts with NaCl and concentrated H2 SO4 to give a pungent (Purple)

F
gas Z. X , Y and Z, respectively, are (2019 Main, 12 April II) 2C ¾H¾
2O, KI
¾¾® 2 A + 2KOH + D
(a) K 3MnO4 , K 2MnO4 and Cl 2 (b) K 2MnO4 , KMnO4 and SO2
In the above sequence of reactions, A and D, respectively, are

or
(c) KMnO4 , K 2MnO4 and Cl 2 (d) K 2MnO4 , KMnO4 and Cl 2
ur
2. The pair that has similar atomic radii is
(2019 Main, 11 Jan II)

f
(a) KI and KMnO4 (b) MnO2 and KIO3
(a) Mn and Re (b) Ti and Hf
(c) KI and K2MnO4 (d) KIO3 and MnO2
ks
(c) Sc and Ni (d) Mo and W
10. The element that usually does not show variable oxidation
Yo
3. The correct order of the first ionisation enthalpies is
states is (2019 Main, 11 Jan I)
oo
(2019 Main, 10 April II)
(a) Sc (b) Cu (c) Ti (d) V
(a) Mn < Ti < Zn < Ni (b) Ti < Mn < Zn < Ni
eB

st
(c) Zn < Ni < Mn < Ti (d) Ti < Mn < Ni < Zn 11. The 71 electron of an element X with an atomic number of 71
4. The highest possible oxidation states of uranium and enters into the orbital (2019 Main, 10 Jan II)

plutonium, respectively, are (2019 Main, 10 April II)


(a) 4 f (b) 6p (c) 5d (d) 6s
r

(a) 7 and 6 (b) 6 and 7 12. The effect of lanthanoid contraction in the lanthanoid series
ou
ad

(c) 6 and 4 (d) 4 and 6 of elements by and large means (2019 Main, 10 Jan I)
2+ 2+ 3+ 3+ (a) increase in atomic radii and decrease in ionic radii
Y

5. Consider the hydrated ions of Ti , V , Ti and Sc . The


(b) decrease in both atomic and ionic radii
correct order of their spin-only magnetic moment is
(c) increase in both atomic and ionic radii
(2019 Main, 10 April I)
nd

(d) decrease in atomic radii and increase in ionic radii


Re

3+ 3+
(a) Sc < Ti < Ti < V 2+ 2+
(b) Sc < Ti 3+ < V2+ < Ti 2+
3+
13. The transition element having least enthalpy of atomisation
(c) Ti 3+ < Ti 2+ < Sc3+ < V2+ (d) V2+ < Ti 2+ < Ti 3+ < Sc3+
Fi

is (2019 Main, 9 Jan II)


6. The maximum number of possible oxidation states of (a) Zn (b) V (c) Fe (d) Cu
actinoides are shown by (2019 Main, 9 April II)
14. In the following reactions, ZnO is respectively acting as a/an
(a) berkelium, (Bk) and californium (Cf) (2017 Main)
(b) nobelium (No) and lawrencium (Lr) (i) ZnO + Na 2O ¾® Na 2 ZnO2
(c) actinium (Ac) and thorium (Th) (ii) ZnO + CO2 ¾® ZnCO3
(d) neptunium (Np) and plutonium (Pu) (a) base and acid (b) base and base
7. The lanthanide ion that would show colour is (c) acid and acid (d) acid and base
(2019 Main, 8 April I) 15. Sodium salt of an organic acid ‘X ’ produces effervescence
3+ 3+
(a) Gd (b) Sm with conc. H2SO 4. ‘X ’ reacts with the acidified aqueous
(c) La 3+ (d) Lu3+ CaCl2 solution to give a white precipitate which
8. The correct order of atomic radii is (2019 Main, 12 Jan II) decolourises acidic solution of KMnO 4. ‘X ’ is (2017 Main)
(a) Ho > N > Eu > Ce (b) N > Ce > Eu > Ho (a) C6H5COONa (b) HCOONa
(c) Eu > Ce > Ho > N (d) Ce > Eu > Ho > N (c) CH3COONa (d) Na 2C2O4
234 Transition and Inner-Transition Elements

16. Which of the following combination will produce H 2 gas? 25. The colour of light absorbed by an aqueous solution of
(2017 Adv.) CuSO4 is (2012)
(a) Fe metal and conc. HNO3 (a) orange-red (b) blue-green
(b) Cu metal and conc. HNO3 (c) yellow (d) violet
(c) Au metal and NaCN (aq) in the presence of air 26. Which of the following will not be oxidised by O3 ? (2005)
(d) Zn metal and NaOH (aq) (a) KI (b) FeSO4 (c) KMnO4 (d) K 2MnO4
17. Which of the following compounds is metallic and 27. Which of the following pair is expected to exhibit same
ferromagnetic? (2016 Main) colour in solution? (2005, 1M)
(a) CrO2 (b) VO2 (a) VOCl 2; FeCl 2 (b) CuCl 2; VOCl 2
(c) MnO2 (d) TiO2 (c) MnCl 2; FeCl 2 (d) FeCl 2; CuCl 2

w
18. The reaction of zinc with dilute and concentrated nitric acid, 28. When I- is oxidised by MnO-4 in alkaline medium,
respectively, produce (2016 Main) I- converts into (2004)
(a) NO2 and NO (b) NO and N2O (a) IO-3 (b) I2 (c) IO-4 (d) IO-
(c) NO2 and N2O (d) N2O and NO2

Flo
29. The pair of compounds having metals in their highest
19. The geometries of the ammonia complexes of Ni 2 + , Pt 2 + and
oxidation state is (2004, 1M)
Zn 2+ , respectively, are (b) [MnO4 ]- , CrO2Cl 2

ree
(2016 Main) (a) MnO2 , FeCl 3
(a) octahedral, square planar and tetrahedral
(c) [Fe(CN)6 ]3- , [Co(CN)3 ] (d) [NiCl 4 ]2- , [CoCl 4 ]-
(b) square planar, octahedral and tetrahedral

F
(c) tetrahedral, square planar and octahedral 30. (NH4 )2 Cr2 O7 on heating gives a gas which is also given by
(d) octahedral, tetrahedral and square planar (a) heating NH4NO2 (2004, 1M)

or
ur
20. Which of the following compounds is not yellow coloured? (b) heating NH4NO3
(2015 Main) (c) Mg3N2 + H2O

f
(a) Zn 2 [ Fe (CN)6 ] (b) K 3 [ Co (NO2 )6 ] (d) Na(comp.) + H2O2
ks
(c) (NH4 )3 [ As (Mo3O10 )4 ] (d) BaCrO 4
31. When MnO2 is fused with KOH, a coloured compound is
Yo
21. Which series of reactions correctly represents chemical formed, the product and its colour is (2003, 1M)
oo
relations related to iron and its compound? (2014 Main) (a) K 2MnO4, purple green (b) KMnO4, purple
eB

Dil. H SO 2 4 2 H SO ,O Heat (c) Mn 2O3, brown (d) Mn 3O4, black


(a) Fe 2 4
¾¾¾¾¾® FeSO4 ¾¾¾¾¾® Fe2 (SO4 ) 3 ¾¾¾® Fe

O2 , Heat Dil. H 2 SO4


32. Amongst the following, identify the species with an atom in
(b) Fe ¾¾¾¾® FeO ¾¾¾¾¾® FeSO4 Heat
¾¾¾® Fe + 6 oxidation state (2000, 1M)
NiF62 -
r

Cl 2 , Heat Heat, air (a) MnO-4 (b) Cr(CN)63- (c) (d) CrO2Cl 2
(c) Fe ¾¾¾® FeCl 3 ¾¾¾® FeCl 2 Zn
¾¾¾® Fe
ou
ad

O , Heat CO, 600°C CO, 700°C 33. On heating ammonium dichromate, the gas evolved is
2
(d) Fe ¾¾¾¾® Fe3O4 ¾¾¾¾® FeO ¾¾¾¾¾® Fe
Y

(a) oxygen (b) ammonia (1999, 2M)


22. Four successive members of the first row transition elements (c) nitrous oxide (d) nitrogen
listed below with atomic numbers. Which one of them is
nd

34. In the dichromate dianion (1999, 2M)


Re

expected to have the highest E °M 3+ / M 2+ value? (2013 Main) (a) 4 Cr—O bonds are equivalent
(b) 6 Cr—O bonds are equivalent
Fi

(a) Cr (Z = 24) (b) Mn (Z = 25)


(c) all Cr—O bonds are equivalent
(c) Fe (Z = 26) (d) Co (Z = 27)
(d) all Cr—O bonds are non-equivalent
23. Consider the following reaction, (2013 Main)
35. Which of the following compounds is expected to be
z
xMnO-4 + y C2 O24 - + zH+ ¾® xMn 2+ + 2 y CO2 + H2 O coloured? (1997, 1M)
2 (a) Ag2SO4 (b) CuF2 (c) MgF2 (d) CuCl
The values of x, y and z in the reaction are, respectively
(a) 5, 2 and 16 (b) 2, 5 and 8 36. Ammonium dichromate is used in some fireworks. The green
(c) 2, 5 and 16 (d) 5, 2 and 8 coloured powder blown in the air is (1997, 1M)
(a) CrO3 (b) Cr2O3 (c) Cr (d) CO
24. Which of the following arrangements does not represent the
correct order of the property stated against it? (2013 Main) 37. The reaction which proceed in the forward direction is
(a) V2+ < Cr 2+ < Mn 2+ < Fe2+ : paramagnetic behaviour (a) Fe2O3 + 6HCl ¾® 2FeCl 3 + 3H2O (1991, 1M)

(b) Ni 2+
< Co 2+
< Fe
2+
< Mn 2+
: ionic size (b) NH3 + H2O + NaCl ¾® NH4Cl + NaOH
(c) Co < Fe < Cr < Sc : stability in aqueous solution
3+ 3+ 3+ 3+ (c) SnCl 4 + Hg2Cl 2 ¾® SnCl 2 + 2HgCl 2
(d) Sc < Ti < Cr < Mn : number of oxidation states (d) 2CuI + I2 + 4H+ ¾® 2Cu2 + + 4KI
Transition and Inner-Transition Elements 235

38. Zinc-copper couple that can be used as a reducing agent is (b) Vapours when passed into NaOH solution gives a yellow
obtained by (1984, 1M) solution of Na 2CrO4
(a) mixing of zinc dust and copper gauge (c) Chlorine gas is evolved
(b) zinc coated with copper (d) Chromyl chloride is formed
(c) copper coated with zinc
48. Which of the following alloys contains Cu and Zn?
(d) zinc and copper wires welded together
(a) Bronze (b) Brass (1993, 1M)
39. How many unpaired electrons are present in Ni 2 + ? (c) Gun metal (d) Type metal
(a) 0 (b) 2 (1981, 1M)
49. The aqueous solution of the following salts will be coloured
(c) 4 (d) 8
in case of (1990, 1M)
40. One of the constituent of German silver is (1980, 1M) (a) Zn(NO3 )2 (b) LiNO3 (c) Co(NO3 )2 (d) CrCl 3
(a) Ag (b) Cu (c) Mg (d) Al (e) potash alum

w
41. Which of the following dissolves in concentrated NaOH 50. Potassium manganate (K 2 MnO4 ) is formed when (1988, 2M)
solution? (1980, 1M)
(a) chlorine is passed into aqueous KMnO4 solution
(a) Fe (b) Zn (c) Cu (d) Ag

Flo
(b) manganese dioxide is fused with KOH in air
(c) formaldehyde reacts with potassium permanganate in the
Objective Questions II presence of strong alkali

ree
(One or more than one correct option) (d) potassium permanganate reacts with conc. H2SO4
42. The correct statement(s) about Cr 2+ and Mn 3+ is/are [atomic

F
number of Cr = 24 and Mn = 25] (2015 Adv.) Assertion and Reason
2+
(a) Cr is a reducing agent Read the following questions and answer as per the direction

or
given below :
(b) Mn 3+ is an oxidising agent
ur (a) Statement I is true; Statement II is true; Statement II is
(c) both Cr 2+ and Mn 3+ exhibit d 4 electronic configuration

f
the correct explanation of Statement I.
(d) when Cr 2+ is used as a reducing agent, the chromium ion
(b) Statement I is true; Statement II is true; Statement II is
ks
attains d 5 electronic configuration
not the correct explanation of Statement I.
Yo
43. Fe3+ is reduced to Fe2+ by using
oo
(2015 Adv.) (c) Statement I is true; Statement II is false.
(a) H2O2 in presence of NaOH (d) Statement I is false; Statement II is true.
eB

(b) Na 2O2 in water 51. Statement I Zn 2 + is diamagnetic.


(c) H2O2 in presence of H2SO4 Statement II The electrons are lost from 4s orbital to
(d) Na 2O2 in presence of H2SO4 form Zn 2 + . (1998, 2M)
r

44. Which of the following halides react(s) with AgNO3 ( aq ) to


ou

52. Statement I To a solution of potassium chromate if a


ad

give a precipitate that dissolves in Na 2 S2 O3 ( aq ) ? strong acid is added, it changes its colour from yellow to
Y

(a) HCl (b) HF (c) HBr (d) HI orange.


45. Reduction of the metal centre in aqueous permanganate ion Statement II The colour change is due to the change in
oxidation state of potassium chromate. (1988, 2M)
nd

involves (2011)
Re

(a) three electrons in neutral medium


(b) five electrons in neutral medium Fill in the Blanks
Fi

(c) three electrons in alkaline medium 53. The compound Y Ba 2 Cu 3 O7 which show super conductivity
(d) five electrons in acidic medium
has copper in oxidation state …… assuming that the rare earth
46. Which of the following statement (s) is/are correct? (1998) element Yttrium in its usual +3 oxidation state. (1994, 1M)
(a) The electronic configuration of Cr is [Ar] 3d 5 4s1 (Atomic 54. The outermost electronic configuration of Cr is ……
number of Cr = 24) (1994, 1M)
(b) The magnetic quantum number may have a negative value
55. Fehling’s solution A consists of an aqueous solution of
(c) In silver atom, 23 electrons have a spin of one type and 24 of
copper sulphate while Fehling’s solution B consists of an
the opposite type (Atomic number of Ag = 47)
alkaline solution of …… (1990, 1M)
(d) The oxidation state of nitrogen in HN3 is – 3
56. The salts ……… and …… are isostructural. (FeSO4 ×7H2 O,
47. Which of the following statement(s) is/are correct when a
mixture of NaCl and K 2 Cr2 O7 is gently warmed with CuSO4 × 5H2 O,MnSO4 × 4H2 O,ZnSO4 × 7H2 O (1990, 1M)

conc. H2 SO4 ? (1998, 2M) 57. Mn 2 + can be oxidised to MnO-4 by ………


(a) A deep red vapours is formed (SnO2 , PbO2 , BaO2 ) (1981, 1M)
236 Transition and Inner-Transition Elements

True/False 67. Complete and balance the following reactions


(i) [MnO4 ]2- + H+ ¾® ..... + [MnO4 ]- + H2O
58. Dipositive zinc exhibit paramagnetism due to loss of two
electrons from 3d-orbitals of neutral atom. (1987, 1M) (ii) SO2 (aq) + Cr2O27 - + 2H+ ¾® … + … + (1994, 2M)
2+
59. Copper metal reduces Fe in an acidic medium. (1982, 1M) 68. Complete and balance the following reaction.
Integer Answer Type Question (NH4 )2 S2 O8 + H2 O + MnSO4 ¾® …… + …… + ……
(1993, 1M)
60. In neutral or faintly alkaline solution, 8 moles of
permanganate anion quantitative oxidise thiosulphate anions 69. Write the balanced chemical equations for the following
to produce X moles of a sulphur containing product. The reactions.
magnitude of X is ... . (2016 Adv.) (i) A mixture of potassium dichromate and sodium chloride

w
is heated with concentrated H2SO4.
61. In dilute aqueous H2 SO4 the complex diaquadioxalatoferrate
(ii) Potassium permanganate is added to a hot solution of
(II) is oxidised by MnO–4 . For this reaction, the ratio of the rate manganous sulphate. (1990, 2M)
of change of [H+ ] to the rate of change of [MnO-4 ] is

Flo
70. Complete and balance the following reactions.
(2015 Adv.)
(i) Mn 2+ + PbO2 ¾® MnO-4 + H2O
62. Consider the following list of reagents, acidified K 2 Cr2 O7 ,

ree
(ii) Ag+ + AsH3 ¾® H3AsO3 + H+ (1987, 2M
alkaline KMnO4 , CuSO4 , H2 O2 , Cl 2 , O3 , FeCl 3 , HNO3 and
Na 2 S2 O3 . The total number of reagents that can oxidise 71. Give reason in one or two sentences

F
aqueous iodide to iodine is (2014 Adv.) “Most transition metal compounds are coloured.”(1986, 1M)
72. Show with balanced equations for the reactions when

or
Subjective Questions
ur (i) potassium permanganate interacts with manganese
Moist air Zn

f
63. (B ) ¬¾¾ MCl 4 ¾® (A) dioxide in the presence of potassium hydroxide.
W hite fumes ( M = Transition (purple colour) (ii) potassium ferricyanide is heated with concentrated
ks
with pungent smell element colourless)
sulphuric acid. (1985, 2M)
Yo
Identify the metal M and hence MCl 4 . Explain the difference
oo
73. State the conditions under which the following preparations
in colours of MCl 4 and A. (2005)
are carried out. Give necessary equations which need not be
eB

64. Give reasons : CrO3 is an acid anhydride. (1999, 2M ) balanced.


65. A compound of vanadium has a magnetic moment of “Potassium permanganate from manganese dioxide”
(1983, 1M)
1.73 BM. Work out the electronic configuration of the
r

vanadium ion of the compound. (1997) 74. Complete and balance the following reactions (1983, 2M)
ou
ad

66. Write balanced equations for the following (i) Zn + NO3- ¾® Zn 2+


+ NH+4
Y

(i) Oxidation of hydrogen peroxide with potassium (ii) Cr2O72- + C2H4O ¾® C2H4O2 + Cr 3+
permanganate in acidic medium.
(ii) Reaction of zinc with dilute nitric acid.
nd

(1997, 2M)
Re
Fi

Answers
1. (c) 2. (d) 3. (d) 4. (b) 37. (a) 38. (b) 39. (b) 40. (b)
5. (a) 6. (d) 7. (b) 8. (c) 41. (b) 42. (a, b, c) 43. (a, b) 44. (a, c, d)
9. (b) 10. (a) 11. (c) 12. (b) 45. (a, c, d) 46. (a, b, c, d) 47. (a, b, d) 48. (b, c)
13. (a) 14. (d) 15. (d) 16. (d) 49. (c, d) 50. (b, c) 51. (b) 52. (c)
17. (a) 18. (d) 19. (a) 20. (a) 53. x = +7 / 3 5
54. 3d 4s 1
55. Rochelle salt
21. (d) 22. (d) 23. (c) 24. (a) 56. FeSO 4 × 7 H 2O and ZnSO 4 × 7H 2O 57. PbO 2
25. (a) 26. (c) 27. (b) 28. (a) 58. F 59. F 60. (6) 61. (8)
29. (b) 30. (a) 31. (a) 32. (d) 62. (7)
33. (d) 34. (b) 35. (b) 36. (b)
Hints & Solutions
1. Thermal decomposition of Mn compound (X), i.e. KMnO4 at 6. The maximum number of possible oxidation states of actinoids
513 K results in compound Y(i.e. K 2MnO4), MnO2 and a are shown by neptunium (Np) and plutonium (Pu). These
gaseous product. MnO2 reacts with NaCl and concentrated actinoids exhibit oxidation states of +3, +4, +5 and +6.
H2SO4 to give a pungent gas Z(i.e. Cl2). The reactions involved 7. The lanthanide ion that would show colour is Sm3+ . Colour of a
are as follows : compound depends on the number of electrons in 4 f -orbitals.
2KMnO4 ¾513
¾¾ K
® K2MnO4 + MnO2 + O2 (g ) Electronic configuration of given lanthanides are as follows:
D
( X) (Y )
Gd 3+ = 4 f 7
MnO2 + 4NaCl + 4 H2SO4 ¾®
Sm 3+ = 4 f 5
MnCl 2 + 4NaHSO4 + 2H2O + Cl 2 (g )
La 3+ = 4 f 0

w
(Z )
Pungent gas. Lu 3+ = 4 f 14
2. The pair that has similar atomic radii is Mo and W. It is due to Gd 3+
have half-filled 4 f -orbitals.
lanthanoid contraction. The factor responsible for lanthanoid

Flo
3+
La have no electron in 4 f -orbitals.
contraction is the imperfect shielding of one electron by another
3+
in the same set of orbitals. Shielding of 4 f is very less due to its Lu have fully-filled 4 f -orbitals.

ree
diffused shape. As a result, nuclear charge increases. Hence, Mo Only Sm 3+ contain 4 f 5. The electrons can easily undergoes
and W have similar atomic radii.
excitation. That result in a formation of colour.
3. The 3d-transition series is

F
8. The correct order of atomic radii is
Sc Ti V Cr Mn Fe Co Ni Cu Zn
Atomic number 21 22 23 24 25 26 27 28 29 30
Europium (Eu) > Cerium (Ce) > Holmium (Ho) > Nitrogen (N)

or
ur
Þ

199 pm 183 pm 176 pm 65 pm


Outermost 3d24s2 3d54s2 3d84s2 3d104s2

f
Electronic Note
Configuration (i) N being the member of p-block and second period, have the
ks
In 1st ionisation, one electron will be removed from 4 s2 smallest radii.
Yo
subshell/orbital. (ii) Rest of all the 3 members are lanthanides with Eu having
oo
With increase in atomic number (Z ), i.e. with increase in number stable half-filled configuration thus with bigger size than rest
of protons in the nucleus, effective nuclear charge (Z* ) also
eB

two.
increases from Sc to Zn. (iii) Among Ce and Ho, Ce has larger size which can be
IE µ Z* explained on the basis of “Lanthanoid contraction”.
9. When MnO2(A) is fused with alkali in presence of air then
r

So, IE order of the given elements will be,


ou

potassium manganate (B) is formed. Potassium manganate (B)


ad

Ti < Mn < Ni < Zn


is of green colour which disproportionate in a neutral or acidic
4. Actinoids show a variety of oxidation states due to comparable
Y

solution to produce potassium permanganate (C). Potassium


energies of 5 f , 6d and 7s energy levels. permanganate (C) in presence of acidic medium oxidises iodide
In the actinoids family (5 f -block), uranium (U) neptunium to iodate.
nd

(Np), plutonium (Pu) and americium (Am) have highest possible


Re

The reaction can be shown as:


oxidation states of +6, + 7, + 7 and +6 respectively. 0
+ 4 4 KOH, O2 +6 -2 -2
5. The spin only magnetic moment (m ) of each ion can be (i) 2 Mn O2 ¾¾¾¾® + 2H2O
Fi

2K 2 Mn O4
calculated as : (A ) D (B )
Potassium manganate
m = n (n + 2) BM (Green)
[Q n = No. of unpaired electron(s)] Þ m µ n, i.e. higher the +6
4 HCl
+ 7 +7
number of unpaired electron, higher will be the value of m. (ii) 3K2 Mn O4 ¾¾® 2K Mn O4 + Mn O2 + 2H2O + 4KCl
(B ) (C )
Potassium
Metal ion Z n (for metal ion) M (BM) Nature permanganate
2+ 2 (purple)
Ti 22 2 (3d ) 8 Paramagnetic
-1
2+ 3 + 7 K I , H2O + 4 +5
V 23 3 (3d ) 15 Paramagnetic (iii) 2K MnO4 ¾ ¾ ¾ ¾
¾® 2 MnO2 + 2KOH + K IO3
(C ) (A ) (D)
Ti 3+ 22 1 (3d 1 ) 3 Paramagnetic
Thus, A and D are MnO2 and KIO3 respectively.
Sc3+ 21 0 (3d 0 ) 0 Diamagnetic
10. The most stable oxidation states in the compounds of the given
Thus, the correct order of spin only magnetic moments of given transition metals of 3d-series are,
hydrated ions will be Sc : + 3; Ti : + 3, + 4; V : +2, + 3, + 4 , + 5; Cu : + 1, + 2
Sc3+ < Ti 3+ < Ti 2+ < V2+
238 Transition and Inner-Transition Elements

The electronic configuration of Sc (Z = 21) is [Ar] 3d 1 , 4 s2. 19. [Ni(NH3 )6 ] 2+ sp3d 2 octahedral
Due to the presence of only one 3d-electron [Pt(NH3 )4 ] 2+ dsp2 square planar
(no pairing energy) and two 4s-electrons, they easily ionise to
achieve most stable +3 oxidation state. [Zn(NH3 )4 ] 2+ sp3 tetrahedral
20. Zn 2[ Fe(CN)6 ], K3[ Co(NO2 )6 ] and (NH4 )3As [ Mo3O10 ] 4 show
11. In the lanthanoid series, atomic number of fourteen 4 f -block
colour due to d-d transition while BaCrO4 is coloured due to
elements ranges from 58 (Ce) to 71 (Lu). charge transfer phenomenon.
Ytterbium, Yb(Z = 70) has electronic configuration : Further according to spectrochemical series the strong ligand
[ Xe ] 4 f 14 6s2. So, the 71nth electron of lutetium, Lu (Z = 71) possessing complex has higher energy and hence lower
should enter into 5d orbital and its (here, Lu is ‘X ’) electronic wavelength. Therefore, complexes containing NO2 , NH4+ , O2-
configuration will be : [ Xe ]4 f 14 5d 1 6s2. It happens so, because etc., ligands show yellow colour while CN- forces the complex
f -block elements have general electronic configuration, to impart white colour.

w
(n - 2) f 1 - 14 (n - 1)d 1- 10ns2. Therefore, option (c) is correct. Spectrochemical series
12. Lanthanoid contraction in the lanthanoid series takes place due I- < Br - < S2- < SCN- < Cl - < NO3- < N3- < F- < OH-
to the presence of electron(s) in the 4 f -orbitals. f -orbitals have < C2O42- » H2O < NCS- < CH3CN < py < NH3 < en
< bipy < Phen < NO-2 < PPh 3 < CN- » CO

Flo
poor shielding effect. As a result, the effective nucleur charge
will be more experienced by the 5d and 6s- electrons and it will 21. PLAN Analyse each reaction given in the question and choose the
cause contraction or decrease in both atomic and ionic radii.

ree
correct answer on the basis of oxidation state and stability of
iron compounds. Use the concept of Ellingham diagram to
13. For transition metals, solve this problem.
°
DH Atomisation µ Strength of metallic bonding

F
The correct reactions are as follows:
µ Number of unpaired electrons in the metal atom (a) Fe + dil. H2SO4 ¾® FeSO4 + H2
For the given 3d-transition metals, 1

or
V Fe Cu Zn
ur H2SO4 + 2FeSO4 + O2 ¾® Fe2 (SO4 )3 + H2O
2

f
D
3d 34 s2 3d64 s2 3d 104 s1 3d 104 s0 Fe2 (SO4 )3 ¾¾® Fe2O3 (s) + 3SO3 ­
n=3 n=4 n=0 n=0
ks
The given reaction is incorrect in question
[Q n = no. of unpaired electrons]
Yo
22. SRP value normally increases from left to right in the period of
oo
D° H Atomisation (kJ mol -1) = 515 418 339 130 d-block elements. Some SRP value are exceptionally higher due
So, absence of unpaired d-electrons and larger size of Zn atoms, to stability of product ion. e.g.
eB

make the crystal lattice of Zn less closely packed. E° = + 1.57 V; E°


Mn 3+ / Mn 2+
= + 1.97 V
Co3+ / Co2 +
14. Zinc oxide (ZnO) when react with Na 2O it act as acid while with Thus, EM° 3+ / M2+ is highest for Co.
r

CO2 it act as base. Therefore, it is an amphoteric oxide.


ou

ZnO + Na 2 O ¾® Na 2 ZnO2 23. The half equations of the reaction are


ad

Acid Base Salt MnO -4 ¾® Mn 2+


Y

ZnO + CO2 ¾® ZnCO3 C2O42- ¾® CO2


Base Acid Salt
The balanced half equations are
nd

15. The reaction takes place as follows MnO-4 + 8H+ + 5e- ¾® Mn 2+ + 4H2O
Re

Na 2C2O4 + H2SO4 ¾® Na2 SO4 + H2O + CO ­ + CO2 ­


(X) (Conc.) Effervescence C2O24- ¾® 2CO2 + 2e-
Fi

Na 2C2O4 + CaCl 2 ¾® CaC2O4 + 2NaCl On equating number of electrons, we get


(X) White ppt. 2 MnO-4 + 16H+ + 10e- ¾® 2Mn 2+ + 8H2O
5CaC2O4 + 2KMnO4 + 8H2SO4 ¾® K2SO4 + 5CaSO4
Purple 5C2O42- ¾® 10CO2 + 10e-
+ 2MnSO4 + 10CO2 + 8H2O On adding both the equations, we get
Colourless
Hence, X is Na 2C2O4. 16
2MnO-4 + 5C2O-4 + 16H+ ¾® 2Mn 2+ + 2 ´ 5CO2 + H2O
16. Zn + 2NaOH ¾® Na 2ZnO2 + H2 2
Amphoteric Thus x , y and z are 2, 5 and 16 respectively.
17. Only three elements iron (Fe), cobalt (Co) and nickel (Ni) show 24. (a) V2+ = 3 unpaired electrons
ferromagnetism at room temperature. CrO2 is also a metallic and Cr 2+ = 4 unpaired electrons
ferromagnetic compound which is used to make magnetic tapes Mn 2+ = 5 unpaired electrons
for cassette recorders.
Fe2+ = 4 unpaired electrons
18. Zn + 4HNO3 ¾® Zn (NO3 )2 + 2H2O + 2NO2 Hence, the order of paramagnetic behaviour should be
(Conc.)
V2+ < Cr 2 + < Fe2 + < Mn 2 +
4Zn + 10HNO3 ¾® 4Zn (NO3 )2 + N2O + 5H2O
(Dil.) (b) Ionic size decreases from left to right in the same period.
Transition and Inner-Transition Elements 239

(c) (As per data from NCERT) 40. German silver is an alloy of copper (56%), Zn (24%) and
Co3+ / Co2+ = 1.97; Ni(20%).
Fe 3+
/Fe 2+
= 0.77; 41. Zn being amphoteric, dissolves in both acid and base :
Cr 3+ / Cr 2+ = - 0.41 Zn + 2NaOH ¾® Na 2ZnO2 + H2
Sc 3+
is highly stable (It does not show + 2). 42. In aqueous solution Cr 2+ (3d 4 ) acts as a reducing agent,
(d) The oxidation states increases as we go from group 3 to oxidising itself to Cr 3+ (3d 3 ) that gives a completely half-field
group 7 in the same period. t2g level in octahedral ligand field of H2O.
(b) Mn 3+ (3d 4 ) is an oxidising agent as it is reduced to
25. The aqueous solution of CuSO4 consist of the complex
Mn 2+ (3d 5 ) , a completely half-filled stable configuration.
[Cu(H2O)4 ]2+ ion which absorbed in orange-red region and
impart deep blue colouration to solution. (c) Both Cr 2+ and Mn 3+ have d 4 configuration.

w
26. KMnO4 is itself a very strong oxidising agent, O3 cannot oxidise it. (d) 3d 4 Cr 2+ (aq) ¾R.¾
¾A
® Cr 3+ (aq)+ e–
Hence (d) is wrong statement.
27. In CuCl 2, Cu 2 + has d 9 configuration, exhibit d-d transition and
show colour. Similarly in VOCl 2, V 4 + has d 1 configuration, can 43. H2O2 is alkaline medium acts as reducing agent, reduces Fe3+ to

Flo
exhibit d-d transition and show colour. Fe2+. In acidic medium the same H2O2 oxidises Fe2+ to Fe3+.

28. MnO-4 + I- + OH- ¾® MnO24- + IO3- 44. Solubilities of silver halides in water decreases from fluoride

ree
(AgF) to iodide (AgI). Silver fluoride is readialy soluble in
29. In MnO-4 , Mn 7+ is in highest oxidation state possible for Mn. In water, hence when AgNO3 solution is added to HF solution (HF

F
CrO2Cl 2, Cr6+ is in highest oxidation state possible for Cr. being weak acid, its solution maintain very low concentration of
F - ) no precipitate of AgF is formed.
30. Ammonium dichromate on heating produces N2 (g). NH4NO2
HCl, HBr and HI being all strong acid, forms precipitates of

or
also gives N2 on heating :
D
ur AgCl, AgBr and AgI when AgNO3 solution is added to their
(NH4 )2 Cr2O7 ¾® N2 + Cr2O3 + 4H2O aqueous solution.

NH4NO2 ¾® N2 + 2H2O
D

f
HCl(aq) + AgNO3(aq) ¾® AgCl (s) + HNO3(aq)
ks
Curdy white
Yo
31. K2MnO4 (purple green) is formed which is the first step of HBr (aq) + AgNO3(aq) ¾® AgBr (s) + HNO3(aq)
oo
preparation of KMnO4. Pale yellow

Hl (aq) + AgNO 3(aq) ¾® AgI (s) + HNO 3(aq)


eB

2MnO2 + 4KOH + O2 ¾® K2MnO4 + 2H2O


Yellow
Purple green
The solubilities decreases from AgCl to AgI, AgCl dissolves in
32. In CrO2Cl 2 , Cr is in + 6 oxidation state because Cl is in (–1) and, aqueous ammonia, AgBr dissolves only slightly in concentrated
r

oxygen is in (–2) oxidation states. ammonia while AgI does not dissolve in ammonia solution.
ou
ad

Heat
Na 2S2O3 solution dissolve all three, AgCl, AgBr, AgI by
33. (NH4 )2 Cr2O7 ¾¾® N2 + Cr2O3 + 4H2O forming complex [Ag(S2O3 )2 ]3- as S2O32- is a stronger
Y

34. The structure of dichromate ion is : complexing agent than ammonia.


O O 45. In neutral medium
nd

MnO–4 ® MnO2 (Mn 7 + + 3e– ® Mn 4+ )


Re

Cr Cr
O O O In alkaline medium
MnO–4 ® MnO2 (Mn 7 + + 3e– ® Mn 4+ )
Fi

O
– O

Cr2O72– In acidic medium
Exhibit resonance phenomena. Except the bridged Cr—O—Cr, MnO–4 ® Mn2+ (Mn7 + + 5e– ® Mn2+ )
all Cr—O bonds are equivalent.
46. Cr : [Ar]3d 5 4 s1
35. Cu 2+ (3d 9 ) undergo d-d transition, exhibit colour. Magnetic quantum number : – l……0……+ l.
36. Ammonium dichromate [(NH4 )2 Cr2O7 ] on heating decomposes Ag(4 d 10 5s1 ) All paired electrons have opposite spin. The last
producing green powder of Cr2O3 and N2 (g ) is evolved. one has unpaired spin.
37. Fe2O3 is a basic oxide, neutralised by HCl spontaneously 47. 4NaCl + K2Cr2O7 + 6H2SO4 ¾® 2CrO2Cl 2
forming FeCl 3 and water. Chromyl chloride
(red vapour)
38. Zinc coated with copper is used as a reducing agent. + 4NaHSO4 + 2KHSO4 + 3H2O
39. The valence shell electronic configuration of Ni 2+ is : CrO2 Cl 2 + 4NaOH ¾® Na 2 CrO4 + 2NaCl + 2H2 O
[Ar] ; two unpaired yellow solution
electrons 48. Brass = Cu and Zn Gun metal = Cu, Sn, Zn
3d 8 4s0
Bronze = Cu and Sn Type metal = Pb, Sn, Sb
240 Transition and Inner-Transition Elements

49. Co2+ (3d 7 ) and Cr 3+ (3d 3 ) have allowed d-d transition, Na 2S2O3 is a strong reducing agent which on reaction with I 2
therefore produces coloured aqueous solution. produces I - .
Na 2S2O3 + I2 ¾® 2I- + Na 2S4O6
D
50. 2KOH + MnO2 + O2 ¾® K2MnO4 + H2O Therefore, no reaction takes place between Na 2S2O3 and iodide
HCHO + KMnO4 + 2KOH ¾® K2MnO4 + H2O ion.
Hence, correct integer is (7).
+ HCOOH
51. Both Statement I and Statement II are independently true but 63. A = [Ti(H2O)6 ]3+ and M = Ti, B = TiO2, Ti(IV) has no
Statement II is not the correct explanation of Statement I. electron in 3d-orbital, no d-d transition is possible, therefore
Diamagnetism is due to lack of unpaired electron in MCl 4 is colourless. In A, there is one electron in 3d-orbital and
Zn 2+ (3d 10 ). its d-d transition is responsible for colour.
52. Statement I is true but Statement II is false : 64. CrO3 is anhydride of chromic acid :

w
K2CrO4 + H2SO4 ¾® K2Cr2O7 + K2SO4 + H2O CrO3 + H2O ¾® H2CrO4
Yellow Orange Chromic acid

In both K2CrO4 and K2Cr2O7 , chromium is in +6 oxidation state. 65. m = n (n + 2) BM where ‘n’ is number of unpaired electrons.

Flo
53. Y = +3, 2Ba = 2 ´ 2 = 4 1.73 = n (n + 2) Þ n = 1; V4+ = 3d 1
7 ‘O’ = 7 ´ (-2) = - 14

ree
7 66. (i) 2KMnO4 + 5H2O2 + 3H2SO4 ¾® K2SO4
3 + 4 + (-14 ) + 3x = 0 Þ x=+
3 + 2MnSO4 + 5O2 + 8H2O
5 1

F
54. 3d 4 s (ii) 4Zn + 10HNO3 ¾® 4Zn(NO3 )2 + N2O + 5H2O
55. Rochelle salt. 67. (i) 3MnO24- + 4H+ ¾® MnO2 + 2MnO-4 + 2H2O

or
56. FeSO4 × 7H2O and ZnSO4 × 7H2O
ur (ii) 3SO2 (aq) + Cr2O72- + 2H+ ¾® 3SO24- + 2Cr 3+ + H2O
57. PbO2, a strong oxidising agent, oxidises Mn 2+ to MnO-4 .

f
68. (NH4 )2 S2O8 + 2H2O + MnSO4 ¾® MnO2
58. Zn 2+ (3d 10 ) has no unpaired electron–diamagnetic.
ks
+ 2H2SO4 + (NH4 )2 SO4
Yo
59. Cu cannot reduce Fe2+
69. (i) K2Cr2O7 + 4NaCl + 6H2SO4 ¾® 2CrO2Cl 2
oo
60. In neutral or faintly alkaline solution, MnO-4 is reduced to MnO2
+ 4NaHSO4 + 3H2O + 2KHSO4
and S2O32- is oxidised to SO2-
eB

4 .
(ii) 2KMnO4 + 3MnSO4 + 2H2O ¾® 5MnO2
Change in ON = 4 units + K2SO4 + 2H2SO4
– 2– 2– +
MnO4 + 1/2 S2 O3 SO 4 MnO2
70. (i) 2Mn 2+ + 5PbO2 + 4H+ ¾® 2MnO-4 + 2H2O + 5Pb2+
r

+7 +6 +4
+2
ou

(ii) 6Ag+ + AsH3 + 3H2O ¾® 6Ag + H3AsO3 + 6H+


ad

Change in ON = 3 units

71. Most transition metals have partially filled d-orbitals which


Y

3
Thus, 4MnO4- + S2O32- ¾® 3SO24- + 4 MnO2 absorb in visible region and undergo d-d transition, which is
2
responsible for colour.
or 8MnO4- + 3S2O32- ¾® 6SO24- + 8MnO2
nd
Re

-
72. (i) 2KMnO4 + 4KOH + MnO2 ¾® 3K2MnO4 + 2H2O
4 formed by 8 moles of MnO 4 = 6
Thus, moles of SO2-
(ii) K4Fe(CN)6 + 6H2SO4 + 6H2O ¾®
Fi

61. The balanced redox reaction is 2K2SO4 + FeSO4 + 3(NH4 )2 SO4 + 6CO
MnO-4 + [Fe(H2O)2 (C2O4 )2 ]2 - + 8H+ ¾® Mn 2+ + Fe3+ 73. Potassium permanganate can be prepared from MnO2 under the
+ 4CO2 + 6H2O following conditions :
r [ H+ ] 8 Heat
MnO2 + KOH + O2 ¾® K2MnO4 + H2O
Þ –
= =8
r [ MnO 4 ] 1
K2MnO4 + Cl 2 ¾® KMnO4 + KCl
62. Acidified K2Cr2O7 , CuSO4 , H2O2, Cl 2, O3, FeCl 3 and HNO3 74. (i) 4Zn + NO-3 + 10H+ ¾® 4Zn 2+ + NH+4 + 3H2O
oxidise aq. iodide to iodine. Alkaline KMnO4 oxidise aq. iodide (ii) Cr2O27 - + 3C2H4O + 8H+ ¾® 3C2H4O2 + 2Cr 3+ + 4H2O
to IO-3 .

Download Chapter Test


http://tinyurl.com/y4wesqs5 or
18
Coordination Compounds

w
Topic 1 Nomenclature and Isomerism of Coordination Compounds

Flo
Objective Questions I (Only one correct option) 6. The total number of isomers for a square planar complex
1. The coordination numbers of Co and Al in [CoCl(en)2 ]Cl and [M(F)(Cl)(SCN)(NO 2 )] is (2019 Main, 10 Jan I)

ree
(a) 12 (b) 16 (c) 4 (d) 8
K 3 [Al(C2 O4 )3 ], respectively, are (en = ethane-1, 2-diamine)
(2019 Main, 12 April II) 7. The oxidation states of Cr, in [Cr(H2 O)6 ]Cl 3 , [Cr(C6 H6 )2 ],

F
(a) 5 and 3 (b) 3 and 3 (c) 6 and 6 (d) 5 and 6 and K 2 [Cr(CN)2 (O)2 (O2 )(NH3 )] respectively are
2. The species that can have a trans-isomer is (en = ethane -1, (2018 Main)
2-diamine, ox = oxalate) (a) +3, +4 and +6 (b) +3, +2 and +4

or
(2019 Main, 10 April I)
(a) [Pt(en)Cl 2 ]
ur
(b) [Cr(en)2 (ox)]+ (c) +3, 0 and +6 (d) +3, 0 and +4

f
(c) [Pt(en)2 Cl 2 ]2+ (d) [Zn(en)Cl 2 ] 8. Consider the following reaction and statements :
[Co(NH3 ) 4Br2 ]+ + Br - ¾® [Co(NH3 )3 Br3 ] + NH3
ks
3. The maximum possible denticities of a ligand given below
Yo
towards a common transition and inner-transition metal ion, I. Two isomers are produces if the reactant complex ion is
oo
respectively, are (2019 Main, 9 April II) a cis-isomer.
sooc coos II. Two isomers are produced if the reactant complex ion is
eB

a trans-isomer.
N N N
sooc coos III. Only one isomer is produced if the reactant complex ion
coos is a trans-isomer.
r

(a) 8 and 8 (b) 8 and 6 (c) 6 and 6 (d) 6 and 8 IV. Only one isomer is produced if the reactant complex ion
ou
ad

4. The one that will show optical activity is (en = ethane-1, is a cis-isomer.
2-diamine) (2019 Main, 9 April I) The correct statements are (2018 Main)
Y

A A (a) (I) and (II) (b) (I) and (III)


A B B B (c) (III) and (IV) (d) (II) and (IV)
nd

(a) M (b) M
Re

9. Which one of the following complexes shows optical


B A B B
isomerism? (2016 Main )
Fi

B A (a) cis [Co(en)2 Cl 2 ]Cl (b) trans [Co(en)2 Cl 2 ]Cl


A A (c) [Co(NH3 )4 Cl 2 ]Cl (d) [Co(NH3 )3 Cl 3 ]
A
(c) M en (d) en M en
10. The number of geometric isomers that can exist for square
B planar [Pt(Cl)(py)(NH3 )(NH2 OH)]+ is (py = pyridine).
(2015 Main)
B A (a) 2 (b) 3 (c) 4 (d) 6
5. The following ligand is 11. Which of the following complex species is not expected to
NEt2 exhibit optical isomerism? (2013 Main)
N (a) [Co(en)3 ]3+ (b) [Co(en)2 Cl 2 ]+
(c) [Co (NH3 )3Cl 3 ] (d) [Co(en)(NH3 )Cl 2 ]+
O– –
O 12. As per IUPAC nomenclature, the name of the complex
[Co (H2 O)4 (NH3 )2 ]Cl 3 is (2012)
(2019 Main, 8 April I) (a) tetraaquadiaminecobalt (III) chloride
(a) hexadentate (b) tetradentate (b) tetraaquadiamminecobalt (III) chloride
(c) bidentate (d) tridentate
242 Coordination Compounds

(c) diaminetetraaquacobalt (III) chloride (b) Statement I is true; Statement II is true; Statement II is not the
(d) diamminetetraaquacobalt (III) chloride correct explanation of Statement I.
13. Geometrical shapes of the complexes formed by the reaction (c) Statement I is true; Statement II is false.
of Ni 2+ with Cl - , CN- and H2 O, respectively, are (2011)
(d) Statement I is false; Statement II is true.

(a) octahedral, tetrahedral and square planar 20. Statement I The geometrical isomers of the complex
(b) tetrahedral, square planar and octahedral [ M (NH3 )4 Cl 2 ] are optically inactive.
(c) square planar, tetrahedral and octahedral Statement II Both geometrical isomers of the complex
(d) octahedral, square planar and octahedral [ M (NH3 )4 Cl 2 ] possess axis of symmetry. (2008, 3M)

14. The correct structure of ethylenediaminetetraacetic acid Passage Based Question


(EDTA) is (2010)
HOOCCH2 CH2COOH Passage

w
(a)
HOOCCH2
N — CH == CH — N
CH2COOH The coordination number of Ni 2 + is 4.
HOOC COOH NiCl 2 + KCN (excess) ¾® A (cyano complex)
(b) N — CH — CH — N
HOOC COOH NiCl 2 + conc. HCl (excess) ¾® B (chloro complex)

Flo
HOOCCH2 CH2COOH 21. The IUPAC name of A and B are (2006, 3 ´ 4M =12M)
(c) N — CH2 — CH2 — N (a) potassium tetracyanonickelate (II), potassium
HOOCCH2 CH2COOH

ree
COOH
tetrachloronickelate (II)
H 2C (b) tetracyanopotassiumnickelate (II),
HOOC—H2C H tetrachloropotassiumnickelate (II)

F
(d) N—CH—CH—N
H CH2—COOH (c) tetracyanonickel (II), tetrachloronickel (II)
CH2 (d) potassium tetracyanonickel (II), potassium

or
HOOC
ur tetrachloronickel (II)
15. The ionisation isomer of [Cr(H2 O)4 Cl(NO2 )]Cl is

f
(2010)
(a) [Cr(H2O)4 (O2N)]Cl 2 (b) [Cr(H2O)4 Cl 2 ](NO2 ) Fill in the Blank
ks
(c) [Cr(H2O)4 Cl(ONO)]Cl (d) [Cr(H2O)4 Cl 2 (NO2 )] × H2O 22. The type of magnetism exhibited by [Mn(H2 O)6 ]2+ ion is …
Yo
16. The IUPAC name of [Ni(NH3 )4 ][NiCl 4 ] is (2008, 3M) (1994, 1M)
oo
(a) Tetrachloronickel (II)-tetraamminenickel (II) Integer Answer Type Questions
(b) Tetraamminenickel (II)-tetrachloronickel (II)
eB

23. The possible number of geometrical isomers for the complex


(c) Tetraamminenickel (II)-tetrachloronickelate (II) [CoL2 Cl 2 ]- ( L= H2 NCH2 CH2 O- ) is (are) ... (2016 Adv.)
(d) Tetrachloronickel (II)-tetraamminenickelate (0)
24. Among the complex ions,
r

17. Which kind of isomerism is shown by Co(NH3 )4 Br2 Cl? [Co(NH2 CH2 CH2 ¾ NH2 )2 Cl 2 ]+ , [CrCl 2 (C2 O4 )2 ]3- ,
ou
ad

(2005, 1M)
(a) Geometrical and ionisation (b) Optical and ionisation [Fe(H2 O)4 (OH)2 ]+ , [Fe(NH3 )2 (CN)4 ]- ,
Y

(c) Geometrical and optical (d) Geometrical only [Co(NH2 ¾ CH2 ¾ CH2 ¾ NH2 )2 (NH3 ) Cl]2+ and
[Co(NH3 )4 (H2 O)Cl]2+ the number of complex ion(s)
Objective Questions II
nd

that show(s) cis-trans isomerism is


Re

(2015 Adv.)
(One or more than one correct option)
25. The volume (in mL) of 0.1 M AgNO3 required for complete
18. The pair(s) of coordination complexes/ions exhibiting the
Fi

same kind of isomerism is/are (2013 Adv.)


precipitation of chloride ions present in 30 mL of 0.01 M
(a) [Cr(NH3 )5 Cl]Cl 2 and [Cr(NH3 )4 Cl 2 ]Cl solution of [Cr(H2 O)5 Cl]Cl 2 , as silver chloride is close to
(2011)
(b) [Co(NH3 )4 Cl 2 ]+ and [Pt(NH3 )2 (H2O)Cl]+
(c) [CoBr2Cl 2 ]2- and [PtBr2Cl 2 ]2- 26. Total number of geometrical isomers for the complex
[RhCl(CO)(PPh 3 )(NH3 )] is (2010)
(d) [Pt(NH3 )3 (NO3 )]Cl and [Pt(NH3 )3 Cl]Br
19. The compound(s) that exhibit(s) geometrical isomerism Subjective Questions
is/are (2009)
(a) [Pt(en)Cl2] (b) [Pt(en)2]Cl2
27. Write the formulae of the following complexes :
(i) Pentamminechlorocobalt (III) ion
(c) [Pt(en)2Cl2]Cl2 (d) [Pt(NH3)2]Cl2
(ii) Lithium tetrahydridoaluminate (III) (1997, 2M)
Assertion and Reason 28. Write the IUPAC name for [Cr(NH3 )5 CO3 ]Cl. (1996, 1M)
Read the following questions and answer as per the direction 29. Write the IUPAC name of the following compounds :
given below :
(i) [Co(NH3 )5 ONO] Cl 2
(a) Statement I is true; Statement II is true; Statement II is the
(ii) K 3[Cr(CN)6 ] (1995, 2M)
correct explanation of Statement I.
Topic 2 Bonding and Important Property of
Coordination Compounds
Objective Questions I (Only one correct option) absorb light in the visible region. The correct order of the
1. The compound used in the treatment of lead poisoning is wavelength of light absorbed by them is (2019 Main, 10 April I)
(2019 Main, 12 April II) (a) II > I > III (b) I > II > III
(a) D-penicillamine (b) desferrioxime-B (c) III > I > II (d) III > II > I
(c) cis-platin (d) EDTA 7. The degenerate orbitals of [Cr(H2 O)6 ]3+ are
2. Complete removal of both the axial ligands (along the z-axis) (2019 Main, 9 April I)
from an octahedral complex leads to which of the following (a) d 2 and dxz (b) dxz and d yz
z

w
splitting patterns? (relative orbital energies not on scale). (c) d 2 2 and dxy (d) d yz and d 2
x - y z
(2019 Main, 12 April I) 8. The calculated spin only magnetic moments (BM) of the
dx2 – y2 dz2 anionic and cationic species of [Fe(H2 O)6 ]2 and [Fe(CN)6 ],
respectively, are

Flo
(2019 Main, 8 April II)
dxy dx2 – y2
(a) E (b) E (a) 0 and 4.9 (b) 2.84 and 5.92
dz2 dxz, dyz (c) 0 and 5.92 (d) 4.9 and 0

ree
dxz, dyz dxy 9. The compound that inhibits the growth of tumors is
(2019 Main, 8 April II)

F
dx2 – y2 dx2 – y2 (a) trans-[Pt(Cl)2 (NH3 )2 ] (b) cis-[Pd(Cl)2 (NH3 )2 ]
dz2 dz2 (c) cis-[Pt(Cl)2 (NH3 )2 ] (d) trans-[Pd(Cl)2 (NH3 )2 ]
(c) E (d) E

or
10. The correct order of the spin only magnetic moment of metal
dxy
ur dyz, dxz
ions in the following low spin complexes, [V(CN)6 ]4- ,

f
dxz, dyz dxy
[Fe(CN)6 ]4- , [Ru(NH3 )6 ]3+ , and [Cr(NH3 )6 ]2+ , is
ks
(2019 Main, 8 April I)
3. The complex ion that will lose its crystal field stabilisation
Cr 2+ > Ru3+ > Fe2+ > V2+
Yo
(a)
energy upon oxidation of its metal to +3 state is
oo
(b) V2+ > Cr 2+ > Ru3+ > Fe2+
(c) V2+ > Ru3+ > Cr 2+ > Fe2+
eB

(d) Cr 2+ > V2+ > Ru3+ > Fe2+


(Phen =
N N 11. The magnetic moment of an octahedral homoleptic Mn(II)
complex is 5.9 BM. The suitable ligand for this complex is
r

Ignore pairing energy (2019 Main, 12 April I)


(a) CN- (b) ethylenediamine
ou

(a) [Co(phen)3 ]2+ (b) [Ni(phen)3 ]2+


ad

(c) NCS- (d) CO


(c) [Zn(phen)3 ]2+ (d) [Fe(phen)3 ]2+
Y

12. The pair of metal ions that can given a spin-only magnetic
4. The crystal field stabilisation energy (CFSE) of moment of 3.9 BM for the complex [M (H2 O)6 ]Cl 2 , is
[Fe(H2 O)6 ]Cl 2 and K 2 [NiCl 4 ], respectively, are (2019 Main, 12 Jan I)
nd

(a) Co2+ and Fe2+ (b) Cr 2+ and Mn 2+


Re

(2019 Main, 10 April II)


(a) - 0.4 D o and - 12
. Dt (b) - 0.4 D o and - 0.8 D t (c) V2+ and Co2+ (d) V2+ and Fe2+
(c) - 2.4 D o and - 12
. Dt (d) - 0.6 D o and - 0.8 D t
Fi

13. The metal d-orbitals that are directly facing the ligands in
5. The incorrect statement is (2019 Main, 10 April II) K 3 [Co(CN)6 ] are (2019 Main, 12 Jan I)
(a) the gemstone, ruby, has Cr 3 + ions occupying the octahedral (a) dxz , d yz and d 2 (b) d 2 2 and d 2
z x - y z
sites of beryl (c) dxy , dxz and d yz (d) dxz and d 2 2
x - y
(b) the color of [CoCl(NH3 )5 ]2 + is violet as it absorbs the yellow 14. Mn 2 (CO)10 is an organometallic compound due to the
light presence of (2019 Main, 12 Jan I)
(c) the spin only magnetic moments of Fe(H2O)6 ]2 + and (a) Mn ¾ C bond (b) Mn ¾ O bond
[Cr(H2O)6 ]2 + are nearly similar (c) C¾ O bond (d) Mn ¾ Mn

(d) the spin only magnetic moment of [Ni(NH3 )4 (H2O)2 ]2 + is 15 The number of bridging CO ligand(s) and Co ¾ Co bond(s)
in Co2 (CO )8 , respectively are (2019 Main, 11 Jan II)
2.83 BM
(a) 2 and 0 (b) 0 and 2 (c) 4 and 0 (d) 2 and 1
6. Three complexes,
16. The coordination number of Th in K 4 [Th(C2 O4 )4 (OH2 )2 ] is
[CoCl(NH3 )5 ]2+ (I), [Co(NH3 )5 H2 O]3+ (II) and
(C2O24 - = Oxalato) (2019 Main, 11 Jan II)
3+
[Co(NH3 )6 ] (III) (a) 14 (b) 10 (c) 8 (d) 6
244 Coordination Compounds

17. Match the metals (Column I) with the coordination 26. On treatment of 100 mL of 0.1 M solution of
compound(s)/enzyme(s) (Column II). (2019 Main, 11 Jan I) CoCl3 .6H2O with excess of AgNO3 ; 1.2 ´ 1022 ions
Column I Column II are precipitated. The complex is (2017 Main)

(A) Co (i) Wilkinson catalyst (a) [Co(H2O)4 Cl 2 ] Cl × 2H2O (b) [Co(H2O)3 Cl 3 ] × 3H2O
(B) Zn (ii Chlorophyll (c) [Co(H2O)6 ]Cl 3 (d) [Co(H2O)5 Cl] Cl 2 × H2O
(C) Rh (iii) Vitamin B 12 27. The pair having the same magnetic moment is
(D) Mg (iv) Carbonic anhydrase [at. no. Cr = 24, Mn = 25, Fe = 26 and Co = 27]
A B C D (a) [Cr(H2O)6 ]2+ and [Fe(H2O)6 ]2+ (2016 Main)
(a) (i) (ii) (iii) (iv) (b) [Mn(H2O)6 ]2+ and [Cr(H2O)6 ]2+
(b) (iv) (iii) (i) (ii) (c) [CoCl 4 ]2- and [Fe(H2O)6 ]2+

w
(c) (iii) (iv) (i) (ii)
(d) [Cr(H2O)6 ]2+ and [CoCl 4 ]2-
(d) (ii) (i) (iv) (iii)
18. The difference in the number of unpaired electrons of a metal ion 28. Among [Ni(CO)4 ], [ NiCl 4 ]2 - , [ Co(NH3 )4 Cl 2 ] Cl,
Na 3 [ CoF6 ], Na 2 O2 and CsO2 , the total number of

Flo
in its high-spin and low-spin octahedral complexes is two. The
metal ion is (2019 Main, 10 Jan II) paramagnetic compounds is (2016 Adv.)
(a) Mn 2+ (b) Fe2+ (c) Ni 2+ (d) Co2+ (a) 2 (b) 3 (c) 4 (d) 5

ree
19. A reaction of cobalt (III) chloride and ethylene diamine in a 1 : 2 29. The colour of KMnO4 is due to (2015 Main)
mole ratio generates two isomeric products A (violet coloured) (a) M ® L charge transfer transition

F
and B (green coloured). A can show optical activity, but B is (b) d ® d transition
optically inactive. What type of isomers does A and B represent ? (c) L ® M charge transfer transition

or
(d) s ® s* transition
(2019 Main, 10 Jan II)
(a) Ionisation isomers
ur
(b) Coordination isomers
(c) Geometrical isomers (d) Linkage isomers 30. The equation which is balanced and represents the
20. Wilkinson catalyst is (2019 Main, 10 Jan I)
f
correct product(s) is (2014 Main)
ks
(a) [(Et 3P)3 RhCl] (b) [(Et 3P)3 IrCl](Et = C2H5) (a) Li 2O + 2KCl ¾® 2LiCl + K 2O
Yo
(c) [(Ph 3P)3 RhCl] (d) [(Ph 3P)3 IrCl] (b) [CoCl(NH3 )5 ]+ + 5H+ ¾® Co2+ + 5NH+4 + Cl -
oo

21. Homoleptic octahedral complexes of a metal ion ‘M 3 + ’ with three (c) [Mg(H2 O)6 ]2+ + (EDTA)4 - ¾¾¾¾¾®
Excess NaOH
eB

monodentate ligands L1 , L2 and L3 absorb wavelengths in the


region of green, blue and red respectively. The increasing order of [Mg(EDTA)]2+ + 6H2 O
the ligand strength is (2019 Main, 9 Jan II) (d) CuSO4 + 4KCN ¾® K 2 [Cu(CN)4 ] + K 2SO4
(a) L1 < L2 < L3 (b) L2 < L1 < L3
r

31. The octahedral complex of a metal ion M 3+ with four


(c) L3 < L1 < L2 (d) L3 < L2 < L1
ou
ad

monodentate ligands L1 , L2 , L3 and L4 absorb


22. The complex that has highest crystal field splitting energy ( D ), is wavelengths in the region of red, green, yellow and blue,
Y

(2019 Main, 9 Jan II) respectively. The increasing order of ligand strength of
(a) [Co(NH3 )5 Cl] Cl 2 (b) [Co(NH3 )5 (H2O)]Cl 3 the four ligands is (2014 Main)
(c) K 3[Co(CN)6 ] (d) K 2[CoCl 4 ] (a) L4 < L3 , L2 < L1 (b) L1 < L3 < L2 < L4
nd
Re

23. The highest value of the calculated spin only magnetic moment (c) L3 < L2 < L4 < L1 (d) L1 < L2 < L4 < L 3
(in BM) among all the transition metal complexes is
Fi

(2019 Main, 9 Jan I)


32. Consider the following complex ions, P, Q and R.
(a) 5.92 (b) 3.87 (c) 6.93 (d) 4.90 P = [ FeF6 ]3 - , Q = [ V(H 2O )6 ]2 + and R = [ Fe(H 2O )6 ]2 +
24. Two complexes [Cr(H 2O)6 ]Cl3 (A) and [Cr(NH3 )6 ]Cl3 (B) are The correct order of the complex ions, according to their
violet and yellow coloured, respectively. The incorrect statement spin-only magnetic moment values (in BM) is
(2013 Adv.)
regarding them is (2019 Main, 9 Jan I) (a) R < Q < P (b) Q < R < P
(a) D o value for (A) is less than that of (B) (c) R < P < Q (d) Q < P < R
(b) both absorb energies corresponding to their complementary colours
33. NiCl 2 {P(C2 H5 )2 (C6 H5 )}2 exhibits temperature
(c) D o values of (A) and (B) are calculated from the energies of violet
and yellow light, respectively dependent magnetic behaviour
(d) both are paramagnetic with three unpaired electrons
(paramagnetic/diamagnetic) the coordination
geometries of Ni 2+ in the paramagnetic and
25. The recommended concentration of fluoride ion in drinking water diamagnetic states respectively, are (2012)
is up to 1 ppm as fluoride ion is required to make teeth enamel (a) tetrahedral and tetrahedral
harder by converting [3Ca 3 (PO4 )2 × Ca(OH)2 ] to : (2018 Main) (b) square planar and square planar
(a) [CaF2 ] (b) [3(CaF2 ) × Ca(OH)2 ] (c) tetrahedral and square planar
(c) [3Ca 3 (PO4 )2 × CaF2 ] (d) [3{Ca 3 (PO4 )2} × CaF2 ] (d) square planar and tetrahedral
Coordination Compounds 245

34. Among the following complexes (K-P), (2011) 44. The complex ion which has no ‘d’-electrons in the central
K 3 [Fe(CN)6 ] (K), [Co(NH3 )6 ]Cl 3 (L), metal atom is (2001, 1M)

Na 3 [Co (ox)3 ] (M), [Ni(H2 O)6 ] Cl 2 (N), (a) [MnO4 ]– (b) [Co(NH3 )6 ]3+
K 2 [Pt(CN)4 ](O), [Zn(H2 O)6 ](NO3 )2 (P) (c) [Fe(CN)6 ]3– (d) [Cr(H2O)6 ]3+
the diamagnetic complexes are
(a) K, L, M, N (b) K, M, O, P 45. The geometry of Ni(CO)4 and Ni(PPh 3 )2 Cl 2 are (1999, 2M)
(c) L, M, O, P (d) L, M, N, O (a) both square planar
(b) tetrahedral and square planar, respectively
35. The complex showing a spin only magnetic moment of
(c) both tetrahedral
2.82 BM is (2010)
(d) square planar and tetrahedral, respectively
(a) Ni(CO)4 (b) [NiCl 4 ]2-
(d) [Ni(CN)4 ]2- 46. Which of the following is formed when excess of KCN is

w
(c) Ni(PPh 3 )4
added to aqueous solution of copper sulphate? (1996, 1M)
36. The spin only magnetic moment value (in Bohr magneton (a) Cu (CN)2
units) of Cr(CO)6 is (2009) (b) K 2[Cu(CN)4 ]

Flo
(a) 0 (b) 2.84 (c) K[Cu(CN)2 ]
(c) 4.90 (d) 5.92 (d) K 3[Cu(CN)4 ]
47. Among the following ions, which one has the highest

ree
37. Among the following, the coloured compound is (2008, 3M)
(a) CuCl paramagnetism? (1993, 1M)
(b) K 3[Cu(CN)4 ] (a) [Cr(H2O)6 ]3+ (b) [Fe(H2O)6 ]2+

F
(c) CuF2 (c) [Cu(H2O)6 ]2+ (d) [Zn(H2O)6 ]2+
(d) [Cu(CH3CN)4 ]BF4
48. Amongst Ni(CO)4 , [Ni(CN)4 ]2– and NiCl 2–

or
ur
38. Both [Ni(CO)4 ] and [Ni(CN)4 ]2- are diamagnetic. The
4 (1991, 1M)

hybridisations of nickel in these complexes respectively, are (a) Ni(CO)4 and NiCl 2–
4 are diamagnetic and [Ni(CN)4 ]
2–
is
(a) sp3 , sp3 (b) sp3 , dsp2 (2008, 3M)
f
paramagnetic
ks
2
(c) dsp , sp 3 2
(d) dsp , dsp 2 (b) [NiCl 4 ]2- and [Ni(CN)4 ]2– are diamagnetic and Ni(CO)4 is
Yo
paramagnetic
oo
39. Among the following metal carbonyls, the C—O bond order
is lowest in (2007, 3M)
(c) Ni(CO)4 and [Ni(CN)4 ]2– are diamagnetic and [NiCl 4 ]2- is
eB

(a) [Mn(CO)6 ]+ (b) [Fe(CO)5 ] paramagnetic


(c) [Cr(CO)6 ] (d) [V(CO)6 ] - (d) Ni(CO)4 is diamagnetic and [NiCl 4 ]2- and [Ni(CN)4 ]2– are
paramagnetic
40. If the bond length of CO bond in carbon monoxide is
r

49. Amongst the following, the lowest degree of paramagnetism


ou

1.128 Å, then what is the value of CO bond length in


ad

Fe(CO)5? (2006) per mole of the compound at 298 K will be shown by


(1988, 2M)
Y

(a) 1.15 Å (b) 1.128 Å


(a) MnSO4 × 4H2O (b) CuSO4 × 5H2O
(c) 1.72 Å (d) 1.118 Å
(c) FeSO4 × 6H2O (d) NiSO4 × 6H2O
41. Spin only magnetic moment of the compound Hg[Co(SCN)4 ]
nd
Re

is (2004, 1M) Objective Question II


(a) 3 (b) 15
Fi

(One or more than one correct option)


(c) 24 (d) 8
50. The correct statement (s) regarding the binary transition
42. The compound having tetrahedral geometry is (2004, 1M) metal carbonyl compounds is (are) (Atomic numbers :
(a) [Ni(CN)4 ]2– (b) [Pd(CN)4 ]2– Fe = 26, Ni = 28) (2018 Adv.)
(c) [PdCl 4 ]2– (d) [NiCl 4 ]2– (a) Total number of valence shell electrons at metal centre in
Fe(CO)5 or Ni(CO) 4 is 16
43. Mixture X = 0.02 mole of [Co(NH3)5SO4]Br and 0.02 mole of (b) These are predominantly low spin in nature
[Co(NH3)5Br]SO4 was prepared in 2 L of solution. (c) Metal-carbon bond strengthens when the oxidation state of the
1 L of mixture X + excess AgNO3 ® Y metal is lowered
1 L of mixture X + excess BaCl2 ® Z (d) The carbonyl C—O bond weakens when the oxidation state of
Number of moles of Y and Z are (2003) the metal is increased
(a) 0.01, 0.01 (b) 0.02, 0.01
(c) 0.01, 0.02 (d) 0.02, 0.02
246 Coordination Compounds

51. The correct option(s) regarding the complex 55. The hybridisation of A and B are
[Co(en)(NH3 )3 (H2 O)]3+ (en = H2 NCH2 CH2 NH2 ) is (are) (a) dsp2 , sp3 (b) sp3 , sp3
(2018 Adv.) (c) dsp2 , dsp2 (d) sp3d 2 , d 2sp3
(a) It has two geometrical isomers
(b) It will have three geometrical isomers, if bidentate ‘en’ is
replaced by two cyanide ligands
Match the Columns
(c) It is paramagnetic 56. Match each set of hybrid orbitals from List–I with complexes
(d) It absorbs light at longer wavelength as compared to given in List-II.
[Co(en)(NH3 )4 ]3+ List–I List–II
52. Addition of excess aqueous ammonia to a pink coloured P. dsp2 1. [FeF6 ]4-
aqueous solution of MCl 2 × 6H2 O( X ) and NH4 Cl gives an Q. sp3 2. [Ti(H2O)3 Cl 3 ]

w
octahedral complex Y in the presence of air. In aqueous 3 2
R. sp d 3. [Cr(NH3 )6 ]3+
solution, complex Y behaves as 1 : 3 electrolyte. The reaction
of X with excess HCl at room temperature results in the S. d 2sp3 4. [FeCl 4 ]2-

Flo
formation of a blue colured complex Z. The calculated spin 5. [Ni(CO)4 ]
only magnetic moment of X and Z is 3.87 B.M., whersas it is 6. [Ni(CN)4 ]2-
zero for complex Y .

ree
Among the following options, which statement(s) is (are) The correct option is (2018 Adv.)
(a) P ® 5; Q ® 4, 6; R ® 2, 3; S ® 1
correct? (2017 Adv.)
(b) P ® 5,6; Q ® 4; R ® 3; S ® 1,2

F
(a) The hybridisation of the central metal ion in Y is d 2sp3
(c) P ® 6; Q ® 4, 5; R ® 1; S ® 2, 3
(b) Addition of silver nitrate to Y given only two equivalents of (d) P ® 4,6; Q ® 5, 6; R ® 1,2; S ® 3

or
silver chloride
ur 57. Match each coordination compound in Column I with an
(c) When X and Y are in equilibrium at 0°C, the colour of the
appropriate pair of characteristics from Column II and

f
solution is pink
select the correct answer using the codes given below the
ks
(d) Z is a tetrahedral complex
Columns (en = H2 NCH2 CH2 NH2 ; atomic numbers : Ti = 22;
Yo
Cr = 24; Co = 27; Pt = 78) (2014 Adv.)
oo
Assertion and Reason
Column I Column II
Read the following questions and answer as per the direction
eB

given below : (A) [Cr(NH3 )4 Cl 2 ]Cl 1. Paramagnetic and exhibits


(a) Statement I is true; Statement II is true; Statement II is the ionisation isomerism
correct explanation of Statement I. (B) [Ti(H2O)5 Cl](NO3 )2 2. Diamagnetic and exhibits
cis-trans isomerism
r

(b) Statement I is true; Statement II is true; Statement II is not the


ou

(C) [Pt(en)(NH3 )Cl]NO3 3. Paramagnetic and exhibits


ad

correct explanation of Statement I.


cis-trans isomerism
(c) Statement I is true; Statement II is false.
Y

(d) Statement I is false; Statement II is true. (D) [Co(NH3 )4 (NO3 )2 ]NO3 4. Diamagnetic and exhibits
ionisation isomerism
53. Statement I [Fe(H2 O)5 NO]SO4 is paramagnetic.
nd

Codes
Re

Statement II The Fe in [Fe(H2 O)5 NO]SO4 has three A B C D A B C D


unpaired electrons. (2008, 3M) (a) 4 2 3 1 (b) 3 1 4 2
Fi

(c) 2 1 3 4 (d) 1 3 4 2
Passage Based Questions 58. Match the complexes in Column I with their properties listed
The coordination number of Ni 2 + is 4. in Column II. (2007, 6M)
NiCl 2 + KCN (excess) ¾® A (cyano complex)
Column I Column II
NiCl 2 + conc. HCl (excess) ¾® B (chloro complex)
(A) [Co(NH3 )4 (H2O)2 ]Cl 2 p. Geometrical isomers
54. Predict the magnetic nature of A and B.
(B) [Pt(NH3 )2 Cl 2 ] q. Paramagnetic
(a) Both are diamagnetic
(b) A is diamagnetic and B is paramagnetic with one unpaired (C) [Co(H2O)5 Cl]Cl r. Diamagnetic
electron (D) [Ni(H2O)6 ]Cl 2 s. Metal ion with +2
(c) A is diamagnetic and B is paramagnetic with two unpaired oxidation state
electrons
(d) Both are paramagnetic Fill in the Blanks
59. The IUPAC name of [Co(NH3 )6 ] Cl 3 is …… (1994, 1M)
Coordination Compounds 247

True/False 67. Deduce the structures of [NiCl 4 ]2– and [Ni(CN)4 ]2–
60. Both potassium ferrocyanide and potassium ferricyanide are considering the hybridisation of the metal ion. Calculate the
diamagnetic. magnetic moment (spin only) of the species. (2002, 5M)
(1989, 1M)
61. The electron density in the xy plane in 3dx 2 - orbital is 68. A metal complex having composition Cr(NH3 )4 Cl 2 Br has
2
y
zero. (1986, 1M) been isolated in two forms A and B. The form A reacts with
AgNO3 to give a white precipitate readily soluble in dilute
Integer Answer Type Questions aqueous ammonia, whereas B gives a pale yellow precipitate
soluble in concentrated ammonia.
62. For the octahedral complexes of Fe3+ in SCN-
(thiocyanato-S) and in CN- ligand environments, the Write the formula of A and B and state the hybridisation of
difference between the spin only magnetic moments in Bohr chromium in each. Calculate their magnetic moments
(spin-only value).

w
magnetons (when approximated to the nearest integer) is (2001, 5M)
[atomic number of Fe = 26 ] (2015 Adv.) 69. Draw the structures of [Co (NH3 )6 ]3+ , [Ni(CN)4 ]2– and
63. In the complex acetylbromidodicarbonylbis [Ni(CO)4 ] . Write the hybridisation of atomic orbitals of the
(triethylphosphine) iron (II), the number of Fe¾ C bond (s) transition metal in each case.

Flo
(2000, 4M)
is (2015 Adv.) 70. A, B and C are three complexes of chromium (III) with the
64. EDTA 4- is ethylenediaminetetraacetate ion. The total empirical formula H12 O6 Cl 3 Cr. All the three complexes have

ree
- water and chloride ion as ligands.
number of N ¾ Co ¾ O bond angles in [Co(EDTA)]
complex ion is (2013 Adv.) Complex A does not react with concentrated H2 SO4 , whereas

F
complexes B and C lose 6.75% and 13.5% of their original
Subjective Questions mass, respectively, on treatment with concentrated H2 SO4 .

or
Identify A, B and C.
ur
65. NiCl 2 in the presence of dimethyl glyoxime (DMG) gives a
(1999, 2M)

71. Identify the complexes which are expected to be coloured.

f
complex which precipitates in the presence of NH4 OH,
Explain (1994, 2M)
giving a bright red colour.
ks
(i) [Ti(NO3 )4 ] (ii) [Cu(NCCH3 )]+ BF4
(a) Draw its structure and show H-bonding
Yo
(b) Give oxidation state of Ni and its hybridisation (iii) [Cr(NH3 )6 ] Cl3 (iv) K3 [VF6 ]
oo
(c) Predict whether it is paramagnetic or diamagnetic (2004, 4M) 72. Give reasons in two or three sentences only for the
eB

66. Write the IUPAC name of the compound following :


K 2 [Cr(NO)(CN)4 (NH3 )] . Spin magnetic moment of the “The species [CuCl 4 ]2– exists, while [CuI4 ]2– does not
complex m = 1.73 BM. Give the structure of anion.
(2003, 4M)
exist.” (1992, 1M)
r
ou
ad

Answers
Y

Topic 1 17. (c) 18. (d) 19. (c) 20. (c)


1. (d) 2. (c) 3. (d) 4. (c)
nd

21. (c) 22. (c) 23. (a) 24. (c)


Re

5. (b) 6. (a) 7. (c) 8. (b) 25. (c) 26. (d) 27. (a) 28. (b)
Fi

9. (a) 10. (b) 11. (c) 12. (d) 29. (c) 30. (b) 31. (b) 32. (b)
13. (b) 14. (c) 15. (b) 16. (c) 33. (c) 34. (c) 35. (b) 36. (a)
17. (a) 18. (b,d) 19. (c,d) 20. (b) 37. (c) 38. (b) 39. (b) 40. (a)
21. (a) 22. paramagnetism 23. (5) 41. (b) 42. (d) 43. (a) 44. (a)
24. (6) 25. (6) 26. (3) 45. (c) 46. (d) 47. (b) 48. (c)
Topic 2 49. (b) 50. (b, c) 51. (a, b, d) 52. (a, b, d)
1. (d) 2. (a) 3. (d) 4. (b) 53. (a) 54. (c) 55. (a) 56. (c)
5. (a) 6. (b) 7. (b) 8. (a) 57. (b) 58. A ® p, q, s B ® p, r, s C ® q, s D ® q, s
9. (c) 10. (b) 11. (c) 12. (c) 59. hexaammine cobalt (III) chloride 60. F
13. (b) 14. (a) 15. (d) 16. (b) 61. F 62. (4) 63. (3) 64. (8)
Hints & Solutions
Topic 1 Nomenclature and Isomerism of
Coordination Compounds A A
B
1. B
Key Idea The total number of ligands to which the metal is en M en
directly attached is called coordination number. M
B
The coordination numbers of Co and Al in [Co(Cl)(en)2]Cl and B
K 3[ Al(C2O4 )3 ] are 5 and 6 respectively.

w
In first complex, ‘en’ is a didentate ligand and ‘Cl’ is a A A
unidentate ligand. (b) (d)
B
[Co(Cl)(en)2]Cl, coordination number = 1 + 2 ´ 2 = 1 + 4 Þ 5 A B

Flo
So, the coordination number is 5.
M
For K 3 [Al(C2O4 )3 ], ‘C2O24 - ’ is a didentate ligand.

ree
Coordination number = 3 ´ 2 = 6. B A

Hence, coordination number is 6.

F
A
2. Key Idea Square planar complexes of general formulae : (a)
[ M(a - a)b2 ] and [ M(a - a) (b - b) ] do not show geometrical

or
ur
isomerism. Whereas, an octahedral complex of general Only molecule (c) does not possess any plane of symmetry.
formula [ M (a - a)2 b2 ] can show geometrical (cis-trans) Hence, it is a chiral molecule and shows optical activity.

f
isomerism.
Key Idea Denticity of ligand is defined as donor sites or
ks
5.
[ Pt(en)2 (Cl 2 )]2+ with formula [ M (a - a)2 b2 ] will show number of ligating groups.
Yo
geometrical isomerism as follows:
oo
The given ligand is tetradentate. It contains four donor atoms. It
2+
2+ Cl can bind through two nitrogen and two oxygen atom to the
eB

en
central metal ion.
Cl Ligand bound to the central atom or ion through coordinate bond
Pt en Pt en in the coordination entity. It act as a Lewis base. The attacking
r

site of the given ligand is given in bold.


ou

Cl
ad

en NEt2
Cl
Y

cis- trans- N
(optically active) (optically inactive)
(I)
O- -O
nd
Re

3. The maximum possible denticities of given ligand towards a


common transition and inner transition metal ion, are 6 and 8
Fi

respectively.
6. A square planar complex of general formula, M abcd gives three
–OOC COO– geometrical isomers only.
–OOC
N N N
Let, a = F- , b = Cl - , c = S CN- , d = NO-2
COO–
COO–
SCN- and NO-2 are ambidentate ligands and they also show
linkage isomerism (structural). Considering both linkage and
The given ligand act as hexadentate ligand in transition metal geometrical isomerism.
ion because the common oxidation state shown by them is +3. Total number of possible isomers given by the complex,
Whereas in case of inner transition metal ion, its denticity is 8
because their common oxidation state is +4. = 3 ´ (2 + 2) = 12

4. Optical activity is the ability of a chiral molecule to rotate the 7. Let the oxidation state of Cr in all cases is ‘ x’
plane of polarised light, measured by a polarimeter. A chiral (i) Oxidation state of Cr in [Cr(H2O)6 ]Cl 3
molecule does not have any plane of symmetry. If a molecule x + (0 ´ 6) + (-1 ´ 3 ) = 0
possess any plane of symmetry, then it is an achiral molecule. or x + 0 - 3 = 0 or x = + 3
Given options (a), (b) and (d) possess plane of symmetry. (ii) Oxidation state of Cr in [Cr(C6H6 )2 ]
x + (2 ´ 0) = 0 or x = 0
Coordination Compounds 249

(iii) Oxidation state of Cr in 10. [Pt(Cl)(py)(NH3 )(NH2OH)]+ is square planar complex. The
K2[Cr(CN)2 (O)2 (O2 )(NH3 )] structures are formed by fixing a group and then arranging all
1 ´ 2 + x + (-1 ´ 2 ) + (-2 ´ 2 ) + (-2 ) + 0 = 0 the groups.
or 2 + x - 2 - 4 - 2 = 0 or x - 6 = 0 Py Py
NH3 NH3
hence x = + 6
Thus, +3, 0 and +6 is the answer. Pt Pt
8. If the reactant is cis isomer than following reaction takes place.
Cl NH2OH HOH2N Cl
Br Br Br
NH3 Br NH3 Br NH3 Br
Py Cl
Br–
+
Pt
NH3 NH3 NH3 Br NH3

w
NH3 HOH2N NH3
NH3 NH3 Br
Cis-isomer Facial Meridonial Hence, this complex shows three geometrical isomers.
i.e. two isomers are produced. If the reactant is trans isomer than 11. Optical isomerism is exhibited by only those complexes which

Flo
following reaction takes place. lacks elements of symmetry. [Co(NH3 )3 Cl 3] shows facial as
Br Br well as meridional isomerism. But both the forms contain plane

ree
NH3 NH3 NH3 NH3 of symmetry. Thus, this complex does not exhibit optical
Br–
isomerism.
12. First of all, the compound has complex positive part

F
NH3 NH3 NH3
‘‘[Co(H2O)4 (NH3 )2 ]3+ therefore, according to IUPAC
Br
conventions, positive part will be named first. Secondly, in

or
Br writing name of complex, ligands are named first in alphabetical
Br
Trans
ur Meridonial order, irrespective of its charge, hence “ammine” will be written

f
i.e. only 1 isomer is produced. Thus, statement (I) and (III) are prior to “aqua”. Therefore, name of the complex is
correct resulting to option (b) as the correct answer. [Co(H2O)4 (NH3 )2 ]Cl 3. Diamminetetraaqua cobalt (III)
ks
chloride.
Yo
9. Cl Cl
NOTE In alphabetical order, original name of ligands are
oo
Cl
considered not the initials of prefixes. Also, special precaution
en Co en Co en should be taken in spelling name of NH3 ligand as it is ammine.
eB

en 13. Ni 2+ + 4 CN– ® [ Ni(CN)4 ]2–


Cl
cis-[Co(en)2Cl2]Cl trans-[Co(en)2Cl2]Cl Here, Ni 2+ has d 8 -configuration with CN– as strong ligand.
r

(optically active) (optically inactive due 3d 4s 4p


to plane of symmetry)
ou
ad

:
:
:

[Co(NH3 )4 Cl 2 ]Cl can exist in both cis and trans forms that are
given below: dsp2
Y

Cl + NH3 +
d 8 -configuration in strong ligand field gives dsp2-hybridisation,
H3N NH3 H 3N Cl
nd

hence square planar geometry.


Re

Co Co Ni 2+ + 4Cl – ¾¾® [NiCl 4 ]2–


Fi

H3N NH3 H 3N Cl Here, Ni 2+ has d 8 -configuration with Cl – as weak ligand.


Cl NH3 3d
:

:
:
:

trans-[Co(NH3)4Cl2]Cl cis-[Co(NH3)4Cl2]Cl
(optically inactive) (optically inactive) sp2
[Co(NH3 )3 Cl 3 ] exists in fac and mer-isomeric forms and both d -configuration in weak ligand field gives sp 3-hybridisation,
8
are optically inactive. hence tetrahedral geometry. Ni 2+ with H 2O forms
NH3 NH3 [Ni(H 2O)6] 2+ complex and H 2O is a weak ligand.
Cl NH3 Cl NH3 3d
Co Co
:

:
:
:

4s 4p 4d
Cl NH3 Cl Cl
:

:
:
:

:
:

Cl NH3
fac-isomer mer-isomer sp3d2
(optically inactive) (optically inactive)
Therefore, [Ni(H 2O)6] 2+ has octahedral geometry.
250 Coordination Compounds

14. HOOCH2C CH2COOH 19. Both [Pt(en)2 Cl 2 ]Cl 2 and [Pt(NH3 )2 Cl 2 ] are capable of showing
N—CH2CH2—N
HOOCH2C CH2COOH geometrical isomerism.

15. Ionisation isomers are the complexes that produces different Cl + Cl +


ions in solution, i.e. they have ions interchanged inside and Cl
outside the coordination sphere. en Pt en en Pt
[Cr(H2O)4 Cl(NO2 )]Cl and [Cr(H2O)4 Cl 2 ](NO2 ) have different
ions inside and outside the coordinate sphere and they are
Cl en
isomers.
trans cis
Therefore, they are ionisation isomers.
H3N Cl H3N Cl
16. [Ni(NH3 )4 ]2 + = tetraamminenickel (II)
[NiCl 4 ]2- = tetrachloronickelate (II) Pt Pt

w
Cationic part is named first, hence : H3N Cl Cl NH3
cis trans
tetraamminenickel (II)-tetrachloronickelate(II)
17. [Co(NH3 )4 Br2 ]Cl and [Co(NH3 )4 BrCl]Br are ionisation isomers. Square planar complex

Flo
Br Br 20. Both statements are true. However, axis of symmetry is not a
H 3N NH3
+
H3N Br +
criteria of optical isomerism. Optical inactivity of the two
geometrical isomers of [ M (NH3 )4 Cl 2 ] is due to the presence of

ree
Co Co plane of symmetry.
Cl NH3

F
H 3N NH3 H 3N NH3
Br NH3 H3N NH3 H 3N Cl

or
Geometrical isomers M M
18.
ur
PLAN Depending on the structure of the complex,different types of
H3N NH3

f
isomerism are shown. H3N Cl
Cl NH3
ks
Complex Isomerism
Trans Cis
Yo
A. [Cr(NH3 )5 Cl]Cl 2 Neither of structural nor Both have several planes of symmetry
oo
[Cr(NH3 )4 Cl 2 ] Cl stereoisomerism
21. A = K2[Ni(CN)4 ] ; B = K2[NiCl 4 ]
B. [Co (NH3 )4 Cl 2 ]+ Cl
eB

H3N NH3 A : Potassium tetracyanonickelate (II)


Co B : Potassium tetrachloronickelate (II)
H3N NH3
Cl 22. Paramagnetism : In [Mn(H2O)6 ]2+ , Mn(II) has 3d 5 configuration.
r

trans w.r.t. Cl Since, H2O is a weak ligand, all five d-electrons are unpaired :
ou
ad

NH3 3d 4s 4p 4d
H3N Cl
Mn(II) :
Y

Co
H3N Cl (weak ligand field)
NH3
23. Ligand is CH2 — NH2
nd

cis w.r.t. Cl
Re

[Pt(H2O) × (NH3 )2 Cl]+ H2O NH3 ½


Pt CH2 O
Fi

Cl NH3 Geometrical isomers are


cis

H2O NH3
Pt NH2
NH2 NH2
H3N Cl Cl Cl NH2 O
3+
Co3+ O O Co3+
trans
Co
O NH2 O Cl
C. [Co Br2Cl 2 ]2- sp3 tetrahedral
Cl Cl O
NH2
Cl
[PtBr2Cl 2 ]2- dsp2 square planar
Cis Trans Cis
D. [Pt (NH3 )3 (NO3 )]Cl [ Pt (NH3 )3 (NO3 )Cl ] Cl
Cl
[ Pt (NH3 )3 Cl ] NO3 ] NH2
NH2 NH2 O
ionisation Co3+
Co3+
[Pt(NH3 )3 Cl]Br [ Pt (NH3 )3 Cl ] Br O NH2 O
O
[ Pt (NH3 )3 Br ] Cl
ionisation Cl
Cis Trans
Coordination Compounds 251

24. All six complex will show cis-trans isomerism 27. (i) [Co(NH3 )5 Cl]2+ (ii) Li[AlH4 ]
Cl
en + + 28. [Cr(NH3 )5 CO3 ]Cl
Cl : pentaamminecarbonatochromium (III) chloride.
Co 3+ and en Co 3+ en 29. (i) [Co(NH3 )5 ONO]Cl 2
: pentaamminenitritocobalt (III) chloride.
Cl (ii) K3[Cr(CN)6 ] : potassium hexacyanochromate (III)
en
cis Cl
trans
O
Topic 2 Bonding and Important Property
O
O 3–
Cl 3–
of Coordination Compounds
O Cl O O 1. The compound used in the treatment of lead poisoning is EDTA.

w
O O
Medication occurs through chelation therapy. Calcium
Cr3+ and Cr3+ disodium ethylenediamine tetraacetic acid chelates divalent
metal ion such as Pb2 + from plasma and interstitial body fluids.
Cl O O O
O

Flo
O Cl The metal displaces Ca and is chelated, mobilised and usually
O
O
trans excreted. Less then 5% CaNa 2EDTA is absorbed in the
O
gastrointestinal tract and it possibly increases the absorption of

ree
+ + – Pb present in the tract. Therefore, it is not recommended for oral
OH2 OH CN
use. It is usually given intravenously.
H2 O OH H2 O OH2 NC NH3

F
Fe3+ and Fe3+ Fe3+ and 2. Complete removal of both the axial ligands (along the z-axis)
NC NH3 from an octahedral complex leads to the following splitting
H2 O OH H2 O OH2
pattern.

or
OH CN
OH2
cis trans
ur cis
dx2–y 2

f
– 2+ 2+
NH3 Cl
ks
en
dxy
CN NH3 Energy
Yo
NC eg
Co 3+ and en Co 3+ en
Fe3+
oo
NC CN Cl
en dz2
NH3 t2g
eB

NH3
cis trans
trans
2+
2+
NH3 Cl dxzdyz
H3 N
r

NH3
H3N Cl The single electron in the dx2 - y2 orbital is being repelled by four
ou

Co3+ and Co3+


ad

NH3
ligands, while the electron in the dz2 orbital is only being
H3N OH2 H3N
repelled by two ligands. Thus, the energy of the dx2 - y2 increases
Y

OH2
NH3 relative to that of dz2 . A more stable arrangement arises when
cis trans
both the eg electrons pair up and occupy the lower energy dz2
25. mmol of complex = 30 ×0.01 = 0.3 orbital. This leaves the dx2 - y2 orbital empty.
nd
Re

Also, 1 mole of complex [Cr(H2O)5Cl]Cl2 gives only two moles Thus, four ligands can now approach along + x , - x, + y and - y
of chloride ion when dissolved in solution directions without any difficulty as dx2 - y2 orbital is empty.
Fi

However, ligands approaching along + z and -z directions meet


[Cr(H2O)5 Cl]Cl 2 ¾® [Cr(H2O)5 Cl]2+ + 2Cl – very strong repulsive forces from filled dz2 orbitals. Thus, only
Þ mmol of Cl – ion produced from its 0.3 mmol = 0.6 four ligands succeed in bonding to the metal. A square planar
complex is formed, the attempt to form an octahedral complex
Hence, 0.6 mmol of Ag+ would be required for precipitation. being unsuccessful.
Þ 0.60 mmol of Ag+ = 0.1M × V(in mL) Þ V = 6 mL Key Idea Crystal field splitting occurs due to the presence
3.
Cl PPh3 Cl CO of ligands in a definite geometry. In octahedral complexes
26.
the energy of two, eg orbitals will increase by (0.6) D o and
Rh Rh that of three t2g will decrease by (0.4) D o .

H3N CO H3N PPh3 The complex ion that will lose its crystal field stabilisation
energy upon oxidation of its metal to +3 state is [ Fe(phen)3 ]2+ .
Cl PPh3
-e-
Rh [Fe(phen)3 ]2+ ¾¾® [Fe(phen)3 ]3+

OC H3N
252 Coordination Compounds

In [Fe(phen)3 ]2+ , electronic configuration of Fe2+ is 3d6 4 s0. (b) [Co(Cl)(NH3 )5 ]2+ is a low spin octahedral complex of Co3+ .
Phenanthrene is a strong field symmetrical bidentate ligand. The It absorbs low energy yellow light and high energy
splitting of orbital in Fe2+ is as follows: complementary violet light will be shown off. Thus,
statement (b) is correct.
eg
(c) [Fe(H2O)6 ]2+ and [Cr(H2O)6 ]2+ are the high-spin octahedral
+0.6 Do complexes of Fe2+ (3d6 , n = 4 ) and Cr 2+ (3d 5 , n = 5) ions
Fe2+ and weak field ligand, H2O respectively. So, spin-only
–0.4 Do magnetic moment = n(n + 2) of the complexes.
t2g [Fe(H2O)6 ]2+ , m 1 = 4 (4 + 2)
CFSE = 6 ´ -0.4 Do = -2.4 Do . (n = 4 ), = 24 = 4.89 BM
The splitting of orbital and arrangement of electrons in Fe3+ is as [Cr(H2O)6 ]2+ , m 2 = 5(5 + 2)

w
follows :
(n = 5), = 35 = 5.92 BM
eg So, m 1 » m 2.Thus, statement (c) is correct.
(d) [Ni(NH3 )4 (H2O)2 ]2+ is also a high-spin octahedral complex

Flo
+0.6 Do
Fe3+
of Ni2+ (3d 8 , n = 2)
–0.4 Do m = 2(2 + 2) = 8 = 2.83 BM

ree
t2g Thus, statement (d) is correct.

CFSE = 5 ´ -0.4 Do = -2. 0 Do Key Idea The wavelength (l ) of light absorbed by the

F
6.
Fe2+ upon oxidation of its metal to +3 state lose its CFSE from complexes is inversely proportional to its D 0 CFSE
-2.4 D o to -2.0D o . (magnitude). D 0 (CFSE) µ 1 / l

or
ur
4. Key Idea Crystal field stabilisation energy (CFSE) for The complexes can be written as:

f
octahedral complexes = (-0.4 x + 0.6 y)D o I. [CoCl(NH3 )5 ]2+ º [Co(NH3 )5 (Cl)]2+ ]
ks
where, x = number of electrons occupying t2g orbital. II. [Co[NH3 ]5 H2O]3+ º [Co(NH3 )5 (H2O)]3+
Yo
y = number of electrons occupying eg orbital. III. [Co(NH3 )5 ]3+ º [Co(NH3 )5 (NH3 )]3+
oo
CFSE for tetrahedral complexes So, the differentiating ligands in the octahedral complexes of
= (-0.6x + 0.4 y)D t Co (III) in I, II and III are Cl s, H2O and NH3 respectively. In the
eB

where, x = number of electrons occupying e orbital. spectrochemical series, the order of this power for crystal field
y = number of electrons occupying t orbital. splitting is Cl - < H2O < NH3.
r

In [Fe(H2O)6 ]Cl2, H2O is a weak field ligand, so it is a high So, the crystal field splitting energy (magnitude) order will be
ou

spin (outer orbital) octahedral complex of Fe2+ . DCFSE (I) < DCFSE (II) < DCFSE
ad

0 0 0 (III)
eg and the order of wavelength (l ) of light absorbed by the
Y

Fe2+(3d6) = complexes will be


t2g é 1ù
l (I) > l (II) > l (III) êëQ Energy (D 0 ) µ l úû
CFSE
nd
Re

\ CFSE = (-0.4 x + 0.6 y)D o


= [ -0.4 ´ 4 + 0.6 ´ 2 ]D o = - 0.4 Do 7. The degenerate orbitals of [Cr(H2O)6 ]3+ are dxz and d yz.
Fi

In K2[NiCl 4 ], Cl - is a weak field ligand, so it is a high spin Electronic configuration of Cr3 + is 3d 5 4 s1. The five d-orbitals
in an isolated gaseous atom or ion have same energy, i.e. they
tetrahedral complex of Ni2+ .
are degenerate. This degeneracy has been removed due to the
t ligand electron–metal electron repulsions in the octahedral
Ni2+(3d6)= complex to yield three orbitals of lower energy, t2g set and two
e orbitals of higher energy, eg set.
(eg)
\CFSE = (-0.6 ´ 4 + 0.4 ´ 4 )D t = - 0.8D t
dx 2–y2 dz2
5. The explanation of given statements are as follows : D0
(a) Ruby, a pink or blood-red coloured gemstone belongs to Free metal ion (t2g)
corundum (Al 2O3 , alumina) system which has trigonal dxy dxz dyz
crystalline lattice containing the repeating unit of
Al 2O3 – Cr 3+ . So, ruby does not belong to beryl lattice 8. [ Fe(H2O)6 ]2 Þ It will form 2 cationic species. i.e.
II
(Be3Al 2Si6O18 ). I. (i)As [ Fe(H2O)6 ]2+ Þ High spin octahedral complex of Fe2+ .
Thus, statement (a) is incorrect. Fe2+ : 3d6 , x = 4 (unpaired electrons)
m = 4 (4 + 2) BM = 4.9 BM
Coordination Compounds 253
III
or (ii) as [ Fe(H2O)6 ]3+ = High spin octahedral complex of Fe3+ . 11. The magnetic moment of the magnitude 5.9 BM suggest the
presence of 5 unpaired electrons in Mn(II). This can be cross
Fe3 + : 3d5 , x = 5, m = 5 (5 + 2) = 5.92 BM verified by putting the value (5) of unpaired electrons in the
[H2O is a neutral weak field ligand] formula, m = n(n + 2) BM
So, [ Fe(H2O)6 ]2+ will be the cationic specie, m = 4.9 BM. Thus, the valence electronic configuration of Mn(II) in the
[ Fe(CN)6 ] will have two anionic complexes complex is
II
II. (i) [ Fe(CN)6 ]4– Þ Low spin, octahedral complex of Fe2+ . Mn2+=
3d 4s 4p 4d
As CN- is a strong ligand it will pair up the electrons.
d2sp3
The octahedral homoleptic complex suggests
Fe2+(3d6) sp3d 2-hybridisation in the complex, i.e.
sp3d2-hybridisation

w
n=0, m=0 Mn2+=
III
or, (ii) [ Fe(CN)6 ]3– Þ Low spin octahedral complex of Fe3+ . 3d 4s 4p 4d

d2sp3 Thus, 5 unpaired electrons are present in the complex which

Flo
Fe3+ Þ (3d5)
suggest the presence of a weak ligand like NCS - .
12. As H2O is a weak field ligand. It readily forms high spin

ree
n=1, m=Ö1(1+2) complexes. In[M (H2O)6 ]Cl2, M exist in +2 oxidation state. The
=1.73 BM arrangement of electrons in the given metal ions are as follows:
(Not in options)

F
[CN - is an anionic strong field ligand] Metal Configuration Number of Spin only
So,the anionic species is [Fe(CN)6 ]4- , m = 0 ions unpaired Magnetic

or
electrons moment (in
ur
Thus, the calculated spin only magnetic moments (BM) of the
anionic and cationic species of [ Fe(H2O)6 ]2 and [ Fe(CN)6 ] BM) = n (n + 2)

f
respectively are 4.9 and 0. Co2+ (d 7 ) = t25geg2 3 3.9
9. cis-[Pt(Cl)2 (NH3 )2 ] is known as cis-platin. It is a s-bonded
ks
6
Fe 2+
(d ) = t24geg2 4 4.9
organo-metallic compound and is used as an anti-tumor agent in
Yo
Cr2+ (d 4 ) = t23geg1 4 4.9
oo
the treatment of cancer.
5
H3N II
Cl Mn 2+
(d ) = t23geg2 5 5.9
eB

Pt 3
H3N Cl V 2+
(d ) = t23geg0 3 3.9

10. Key Idea In presence of strong field ligands, D 0 > p, for Therefore, Co 2+ and V 2+ contains same value of magnetic
r

moment (3.9 BM).


ou

fourth electron it is more energetically favourable to occupy


ad

t2 g orbital with configuration t24geg0and form low spin complexes. 13. In K3 [Co(CN)6 ], Co have +3 oxidation state and electronic
Y

The correct order of the spin only magnetic moment of metal ions configuration of Co3 + is [Ar ] 18 3d 6.
in the given low-spin complexes is V2+ > Cr 2+ > Ru 3+ > Fe2+ . 3d6 4s0 4p0
All the given complexes possess strong field ligands
nd

Co3+=
Re

(CN, NH3 ). Hence, readily form low spin complexes.


As, CN - is a strong field ligands so it pairs up the de- s
Fi

Complex Oxidation Configuration Orbital No. of


state splitting unpaired 3d6
electrons \ [Co(CN)6]3–= XX XX XX XX XX XX

eg Inner orbital
d2sp3 -hybridised
complex
[V(CN)6]4– V
2+ 3
t2g eg
0
3 (6e- pairs donated
by 6 CN- ligands)
t2g
In an octahedral complex, the metal is at the centre of the
eg octahedron and the ligands are at the six corners. The lobes of the
[Cr(NH3)6]2+ Cr2+ 4
t2g eg0 2 eg orbitals (dx2 - y2 and dz2 ) point along the axes x , y and z under
t2g
the influence of an octahedral field, the d- orbitals split as follow.
eg
5 0
dx2-y2, dz 2
[Ru(NH3)6]3+ Ru3+ t2g eg 1
t2g

eg
Average dxy, dyz, dxz
d-orbitals energy of
[Fe(CN)6]4– Fe2+ 6 0
t2g eg 0 in free ion Splitting of d-orbitals
the d-orbitals
t2g in a spherical in an octahedral
crystal field crystal field
254 Coordination Compounds

As the d-orbitals, i.e. dx2 - y2 and dz2 are vacant. Hence, these Total number of bridging CO ligands = 2
and the Co ¾ Co bond = 1
both orbitals are directly facing the ligands in K3 [Co(CN)6 ].
14. Mn 2 (CO)10 is an organometallic compound due to the 16. Coordination number is defined as the total number of ligands to
presence of Mn¾ C bond. The metal-carbon bond in which the metal is directly attached.
organometallic compounds possess both s and-p character. Here, C2O24 - is a bidentate ligand,
The M¾ C s bond is formed by the donation of lone pair of
¾

electrons from the carbonyl carbon into a vacant orbital of C¾O ¾
the metal. The M¾ C p-bond is formed by the donation of

¾
M (Metal)
C¾O ¾
pair of electrons from a filled d-orbital of metal into vacant O ¾
¾
antibonding p* orbital of CO. The M ¾ L bonding creates a
and H2 O is a monodentate ligand, H2O M
synergic effect which strengthens the bond between CO and
So, total number of sites offered by C2O24 - and H2O ligands

w
the metal.
The structure of Mn 2(CO)10 is shown below : around Th(IV) = Coordination number of Th (IV)
CO
CO
CO
CO = 4 ´ 2(by C2O24 -) + 2 ´ 1(by H2O) = 10

Flo
CO Mn Mn CO 17. (A) Co is present in vitamin B12 (iii) having molecular formula,
CO II
CO CO CO C63H88 CoN14O14P.

ree
15. The structure of Co2 (CO)8 (a polynuclear metal carbonyl) can (B) Zn is present in carbonic anhydrase (iv) in which three
be written as: histidine units and the —OH group coordinate with one Zn

F
(II) ion.
OC C CO (C) Rh is present in Wilkinson catalyst (i) having molecular

or
OC——Co——–Co——CO
ur formula [(Ph 3P)3 RhCl] .
(D) Mg is present in chlorophyll (ii) having molecular formula

f
OC C CO II
C55H70O6N4 Mg (chlorophyll-b).
ks
Yo
oo
18. The difference in the number of unpaired electrons of different metal ions in their high spin and low spin octahedral complexes are given in
the table below :
eB

Metal ion Number of e- in high spin complex ( n1 ) Number of e- in low spin complax ( n2 ) n2 - n1
Mn 2+
3d 4s 4p 3d 4s 4p 5 -1 = 4
r
ou
ad

4d 4d
Y

n1=5 n2=1

Fe 2+
3d 4s 4p 3d 4s 4p 4-0 = 4
nd
Re

4d 4d
Fi

n1=4 n2=0

Ni 2+ 3d 4s 4p

4d

n1=2
2+
Ni does not form low spin octahedral complexes.
Co 2+
3d 4s 4p 3d 4s 4p 3 -1 = 2

4d 4d

\ n1=3 \ n1=1
Coordination Compounds 255

19. According to the situation given in question, reactions are as The highest value of n in transition metal complex is 5 in its
follows: d 5-configuration.
III III \ m = 5(5 + 2) BM = 5.916 BM
CoCl3 + 2en ¾® [Co(en)2Cl2]Cl
·· ù
CH2 — N H2ú 24. ‘A’ absorbs yellow light of less energy and emits violet light of
high energy (complementary colour) because H2O is a weak
en = ethylene-1, 2-diamine, ½ ú
field ligand. But in case of ‘B’, due to presence of strong field
CH2 — NH2 ú
·· úû ligand (NH3 ), it absorbs high energy violet light and emits low
energy complementary yellow colour.
Cl
en D(CFSE) is measured with help of wavelength of the colour
Cl absorbed by the given coordination compound, as
III c
D O = hn = h ´

w
Co Cl en Co en Cl
l
Cl Both the complexes contain three unpaired electrons. Therefore,
en
Cl both are paramagnetic.

Flo
‘A’ Optically active ‘B’ Optically inactive
(cis-form) (trans-form) 25. Fluoride ions help in making teeth enamel harder by converting
(Violet) (Green) [3Ca 3 (PO4 )2 × Ca(OH)2 ] i.e. Hydroxy apatite to

ree
[3Ca 3 (PO4 )2 × CaF2 ] i.e., Fluorapatite (Harder teeth enamel) via
20. Wilkinson’s catalyst is a s-bonded organometallic compound following reaction:
[(Ph 3P)3 RhCl] . It is commercially used for hydrogenation of [3Ca 3 (PO4 ) 2 × Ca(OH) 2 ] + 2F - ¾® [3Ca 3 (PO4 ) 2 × CaF2 ] + 2OH-

F
From
alkenes and vegetable oils (unsaturated). drinking
water
IUPAC name Chloridotris (triphenylphosphene) rhodium (I).

or
ur 26. Molarity (M) =
Number of moles of solute
21. In homoleptic complexes, the metal atom/ion is linked to only Volume of solution (in L)

f
one type of ligand. Assuming, ligands are neutral, the octahedral
complexes of M 3+ can be, \ Number of moles of complex
ks
[ M (L1 )6 ]3+ ,[ M (L2 )6 ]3+ and [ M (L3 )6 ]3+ Molarity ´ volume (in mL)
Yo
=
oo
(I) (II) (III)
1000
l Absorption Green Blue Red (wavelength) . ´ 100
01
eB

= = 0.01 mole
So, 3 > lL1 > lL2
lLIII I II 1000
1 Number of moles of ions precipitate
\ D°absorption : DLII2 > DLI1 > DLIII3 [Q Energy (D, CFSE) µ ]
l
r

. ´ 1022
12
We know, ligand strength µ D°absorption = = 0.02 moles
ou
ad

6.02 ´ 1023
So, the increasing order of the ligand strength will be,
Y

L3 < L1 < L2 \Number of Cl - present in ionisation sphere


Number of moles of ions precipitated 0.02
22. All of the complex given are the octahedral complexes of Co = = =2
nd

Number of moles of complex 0.01


Re

(III) except K 2[CoCl 4], which is a tetrahedral complex of Co


(II) (sp3-hybridised). \2 Cl - are present outside the square brackets, i.e. in ionisation
Fi

é 4 ù sphere. Thus, the formula of complex is


We know, Dt < Do êëQ D t = 9 D o úû
[Co(H2 O) 5 Cl]Cl 2 ×H2 O.
So, the octahedral complexes (a, b, c) have higher D o values 27.
than that of tetrahedral, K2[CoCl 4 ]. Complex ion Electronic Number of unpaired
Now, for the complexes, a, b and c, configuration of electrons (n)
the magnitude of D o µ ligand strength, which is based on their metal ion
positions in the spectrochemical series.
[Cr(H2O)6 ]2+ Cr 2+ ; [Ar] 3 d 4 ;4
Cl - < H2O < NH3 < CN-
Hence, K3[Co(CN)6 ] will have the highest D value. [Fe(H2O)6 ]2+ Fe 2+ ; [Ar] 3 d6 ; 4

23. The spin only magnetic moment (m) (in BM) is given by [Mn(H2O)6 ]2+ Mn2+ ; [Ar] 3 d 5 ;5
m (in BM) = n(n + 2)
where, n = number of unpaired electrons [CoCl 4 ]2- Co 2+ ; [Ar] 3 d 7 ;3
256 Coordination Compounds

28. 1
µ
Compounds Hybridisation Unpaired Magnetic Wavelength of light absorbed
electron(s) character l L1 L2 L3 L4
Ni(CO)4 sp3 No Diamagnetic Absorbed light Red Green Yellow Blue
[NiCl 4 ]2 - sp3 two Paramagnetic Wavelength of absorbed light decreases.
[Co(NH3 )4 Cl 2 ]Cl 3 2
sp d No Diamagnetic \ Increasing order of energy of wavelengths absorbed reflect
3 2 greater extent of crystal field splitting, hence, higher field
Na 3[CoF6 ] sp d three Paramagnetic
strength of the ligand.
Na 2O2 — No Diamagnetic Energy blue (L4 ) > green (L 2 ) > yellow (L 3 ) > red (L 1 )
(O22 - ) \ L4 > L2 > L3 > L1 in field strength of ligands.
CsO2 — One Paramagnetic
PLAN Spin only magnetic moment have the formula n( n + 2) BM,

w
32.
O-2 (superoxide where N is the number of unpaired electrons. In the presence of
ion is weak ligand (as H 2O, Cl - , F - ) there is no pairing of electrons,
paramagnetic) and electrons donated by ligands are filled in outer vacant
orbitals.

Flo
29. KMnO4 ¾® K + + MnO-4 In the presence of strong ligand (as CN- , CO, NH3 , en)
electrons are paired and electrons from ligands are filled in
\ In MnO-4 , Mn has + 7 oxidation state having no electron in

ree
available inner orbitals
d-orbitals.

number of

Unpaired
electrons
Complex
It is considered that higher the oxidation state of metal, greater is

F
Atomic

Magnetic
O.N.

E.C.
the tendency to occur L ®M charge transfer, because ligand is

moment
able to donate the electron into the vacant d-orbital of metal.

or
ur
Since, charge transfer is laporate as well as spin allowed,
therefore, it shows colour. P : [FeF6 ]3 - +3

f
26 [Ar]3d 5 5 35 BM
Time saving Technique There is no need to check all the four weak ligand
ks
options. Just find out the oxidation state of metal ion. If Q : [V(H2O)6 ]2+ 23 +2 [Ar] 3 15 BM
Yo
oxidation state is highest and ligand present there is of electron
weak ligand
oo
donating nature, gives LMCT, which shows more intense
colour. R :[Fe(H2O)6 ]2+ 26 +2 [Ar]3d6 4 24 BM
eB

30. This problem is based on conceptual mixing of properties of Thus, order of spin-only magnetic moment = Q < R < P
lithium oxide and preparation, properties of coordination
33. In the given complex, NiCl 2 {P (C2H5 )2 (C6H5 )}2 nickel is in
compounds. To answer this question, keep in mind that on
+ 2 oxidation state and the ground state electronic configuration
r

adding acid, ammine complexes get destroyed.


ou

of Ni 2+ ions in free gaseous state is


ad

(a) Li 2O + KCl ¾® 2LiCl + K2O


This is wrong equation, since a stronger base K2O cannot be 3d 8 4s0 4p0
Y

generated by a weaker base Li 2O. Ni2+


(b) [CoCl(NH3 )5 ]+ + 5H+ ¾® Co2+ ( aq) + 5 NH+4 + Cl -
nd

For the given four coordinated complex to be paramagnetic, it


Re

This is correct. All ammine complexes can be destroyed by


must possess unpaired electrons in the valence shell. To satisfy
adding H Å . Hence, on adding acid to [CoCl(NH3 )5 ], it gets
this condition, four lone pairs from the four ligands occupies the
Fi

converted to Co2+ ( aq)+ NH+4 and Cl - . four sp3-hybrid orbitals as :


OH-
(c) [Mg (H2O)6 ]2+ + EDTA4- ¾¾® [ Mg( EDTA)]2+ + 6H2O Ni2+
:

:
:
:

Excess
This is wrong, since the formula of complex must be sp3
[Mg(EDTA)]2+ as EDT. Therefore, geometry of paramagnetic complex must be
(d) The 4th reaction is incorrect. It can be correctly tetrahedral. On the otherhand, for complex to be diamagnetic,
represented as there should not be any unpaired electrons in the valence shell.
2CuSO4 + 10KCN ¾® 2K3[Cu(CN)4 ] This condition can be fulfilled by pairing electrons of
+ 2K2SO4 + (CN)2 ­ 3d-orbitals against Hund’s rule as
31. Arrange the complex formed by different ligands L 1 , L 2 , L 3 and Ni2+
:

:
:

L4, according to wavelength of their absorbed light, then use of


the following relation to answer the question. 3d 8 dsp2
Ligand field strength µ Energy of light absorbed The above electronic arrangement gives dsp2-hybridisation and
therefore, square planar geometry to the complex.
Coordination Compounds 257

34. For a diamagnetic complex, there should not be any unpaired 42. [NiCl 4 ]2 - : Ni 2+ (3d 8 )
electron in the valence shell of central metal. Tetrahedral
sp3-hybridisation
In K3[Fe(CN)6], Fe (III) has d 5-configuration (odd electrons),
hence it is paramagnetic.
In [Co(NH3)6]Cl3, Co (III) has d 6-configuration in a strong
ligand field, hence all the electrons are paired and the complex is 3d 8 4s
diamagnetic. Under influence of weak ligand field
In Na3[Co(ox)3], Co (III) has d 6-configuration and oxalate being
In all other complexes, hybridisation at central metal is dsp2 and
a chelating ligand, very strong ligand and all the six electrons
complexes have square planar geometries.
remains paired in lower t2g level, diamagnetic.
43. In 1 L solution, there will be 0.01 mole of each [Co(NH3 )5 SO4 ]
In [Ni(H2O)6]Cl2, Ni (II) has 3d 8 -configuration and H2O is a
Br and [Co(NH3 )5 Br]SO4. Addition of excess of AgNO3 will

w
weak ligand, hence
give 0.01 mole of AgBr. Addition of excess of BaCl 2 will give
3d 0.01 mole of BaSO4.
44. In MnO-4 , Mn + 7 has 3d 0 configuration.

Flo
Paramagnetic
45. In Ni (CO)4, Ni is in 3d 10 state due to strong ligand field produced
:

:
:
:

:
:

3 2
sp d by CO. Hence, Ni is sp3-hybridised and complex is tetrahedral. In

ree
NiCl 2 (PPh 3 )2 , Ni 2+ has 3d 8 -configuration. Due to weak ligand
In K2[Pt(CN)4], Pt(II) has d 8 -configuration and CN– is a strong
ligand, hence all the eight electrons are spin paired . Therefore, field, Ni is sp3-hybridised and complex is tetrahedral.

F
complex is diamagnetic.
46. Cu 2+ + CN- ¾® CuCN ¯
In [Zn(H2O)6](NO3)2, Zn (II) has 3d 10configuration with all the CuCN + 3CN- ¾® [Cu(CN)4 ]3 -

or
ten electrons spin paired, hence diamagnetic.
ur 47. Fe in [Fe(H2O)6 ]2+ has maximum (four) unpaired electrons, has
35. Magnetic moment = 2.83 BM indicates that there is two

f
highest paramagnetism.
unpaired electrons.
48. In Ni (CO)4, Ni has 3d 10-configuration, diamagnetic. In
ks
u = n(n + 2) BM = 8 BM = 2.82 BM Ni (CN)4 ]2- , Ni has 3d 8 -configuration but due to strong ligand
Yo
field, all the d-electrons are spin paired giving
oo
In [NiCl 4 ]2- , Ni has d 8 configuration and Cl - is a weak ligand : dsp2-hybridisation, diamagnetic.
3d In [NiCl 4 ]2- , Ni has 3d 8 -configuration and there is two unpaired
eB

Ni2+
:

:
:
:

electrons (weak chloride ligand do not pair up d - electrons),


4s 4p hence paramagnetic.
3
r

sp -hybridisation 49. Salt with least number of unpaired electrons in d - orbital of


ou
ad

6 central metal will show lowest degree of paramagnetism


36. In Cr(CO)6 : 3d , has no unpaired electrons, zero magnetic
moment. Mn 2+ (3d 5, 5 unpaired electrons)
Y

37. CuF2 : Cu 2+ has 3d 9 -configuration, allowed d-d transition, Cu 2+ (3d 9 , 1 unpaired electrons)
hence, coloured. Fe2+ (3d6, 4 unpaired electrons)
nd
Re

Ni 2+ (3d 8 , 2 unpaired electrons)


38. In Ni(CO)4, Ni is sp3-hybridised while in [Ni(CN)4 ] 2- , Ni 2+ is
Hence, CuSO4 × 5H2O has lowest degree of paramagnetism.
dsp2-hybridised.
Fi

50. Statement wise explanation is


39. Greater the extent of dp - pp back bonding, smaller will be the (i) Statement (a) The total number of valence shell electrons at
bond order of CO bond in metal carbonyls. In Fe(CO)5, there is metal centre in Fe(CO)5 or Ni(CO)4 is 8 instead of 16 as
maximum number of valence shell electrons (d-electrons), shown below
greatest chances of pp - dp back bonding, lowest bond order of Valence shell having 8 electrons
CO bond. in complex formation

40. In CO, bond order = 3. In metal carbonyls like Fe(CO)5, due to Fe(3d6,4s2) Ni(3d8,4s2)
dp - pp back-bonding, bond order of CO decreases slightly 3d 4s 4p 3d 4s 4p
therefore, bond length increases slightly.
41. In Hg [Co(SCN)4 ], Co2+ has 3d 7 configuration. SCN - produces [Fe(CO)5] [Ni(CO)4]
weak ligand field, no pairing of electrons in d-orbitals occurs
against Hund’s rule, hence : Rearrangement Rearrangement
2+ dsp3 sp3
Co : m = 3 (3 + 2) BM = 15 BM
3d7 Hence, this statement is incorrect.
258 Coordination Compounds

(ii) Statement (b) Carbonyl complexes are predominantly low Statement (d) According to CFT, absorption of light by
spin complexes due to strong ligand fields. Hence, this coordination complexes depends upon CFSE i.e., crystal field
statement is correct. splitting energy (D 0)as
(iii) Statement (c) For central metal lowering of oxidation state 1
D0 µ
results to increase in electron density on it. This in turn l
results to increase in extent of synergic bonding. Thus, we
Among the complexes given [Co (en) (NH3)4 ]3+ has more D 0
can say ‘‘metal carbonyl bond strengthens, when oxidation
state of metal is lowered’’. value as compared to complex [Co(en) (NH3)3(H2O) ]3+ . Thus,
Hence, it is a correct statement. [Co (en) (NH3)3(H2O) ]3+ absorbs the light at longer wavelength
(iv) Statement (d) Increase in positive charge on metal (i.e., for d-d transition.
increase in oxidation state) results to decrease in synergic Hence, this statement is also correct.
bonding strength. Note : For any complex, the value of D 0 can be calculated via the

w
This in turn makes C—O bond stronger instead of weaker. difference or gap between eg and t2g values.
Hence this statement is also incorrect.
Excess NH 4 OH / NH 4 Cl
51. Statement wise explanation is 52. [Co(H2O) 6 ] Cl 2 ¾¾¾¾¾¾¾® Co(NH3 ) 6 ]Cl 3
O 2 (Air)
Pink (X) Y
Statement (a) [Co(en)(NH3 )3 H2O ]3+ have following 2

Flo
-
geometrical isomers. [Co(H2 O) 6 ] 2+
+ 4Cl ¾® [CoCl 4 ]2 -
X (Excess) blue Z
NH3 NH3

ree
NH3 H2O
(a) Since NH3 is moderately strong ligand, hybridisation of
cobalt in Y is d 2sp3.
en Co en Co
(b) Cobalt is sp3-hybridised in [CoCl 4 ]2- .

F
NH3 NH3 (c) [Co(NH 3 ) 6 ]Cl 3 + 3AgNO3 (aq) ¾® 3AgCl ¯
H2O NH3 Y

or
Fac
ur
[ M ( AA )b3 c] type complex
Mer (d) [CoCl 4 ]2- + 6H2O q [Co(H2O)6 ] 2+ + 4Cl - ; DH < 0
Blue Pink

f
Hence, this is correct statement. 53. In the complex [Fe(H2O)5 NO]SO4, Fe is in +1 oxidation state
ks
Statement (b) If bidentate ligand ‘en’ is replaced by two because NO is in +1 state. Also NO is a strong ligand, complex
cyanide ligands then [Co(NH3 )3 (H2O)(CN)2 ]+ is formed.
Yo
has 3d 7 -configuration at Fe(I) as :
oo
It is [ Ma3 b2 c ] type complex which has following 3 geometrical
isomers.
eB

NH3 NH3
N C NH3 N C NH3
3d7 Strong ligand field
Co Co
r

N C NH3 NH3
ou

H2O
ad

H2O CN
Y

(Fac) Fac with respect to NH3 and


Mer with respect to —CN Three unpaired electrons
NH3
54. A is diamagnetic, square planar complex because of strong
nd

N C H2O
ligand field of CN- .
Re

Co
N C NH3 Ni(CN)42 –(Ni2+) :
Fi

NH3
(Mer) dsp2
Hence, this statement is also correct. Diamagnetic
7 2 B is paramagnetic, tetrahedral complex because of weak ligand
Statement (c) Co metal has [ Ar ]3d 4 s configuration while in
[Co(en)(NH3 )3 (H2O )]3+ it is in +3 oxidation state. Thus, Co 3+ field of Cl - .
has [Ar]3d6 configuration. NiCl42 –(Ni2+) :
3d 4s
Co3+ = sp3
Paramagnetic
As en is a strong ligand, so pairing will occur 55. Described in 2, A has dsp2 hybridisation while B has
3d 4s
sp3-hybridisation of Ni.
56. For, P i.e. dsp2, It is seen in [Ni(CN)4 ]2 -
Due to the presence of all paired electrons it show diamagnetic
behaviour rather than paramagnetic. Ni ¾[Ar]3d 8 4 s2
Hence, this statement is incorrect. Ni2+ ¾[Ar]3d 8
Coordination Compounds 259

3d 4s 4p As F - is a weak field ligand hence, when it approaches towards


Ni2+ central metal atom, pairing of its electrons does not take place.
Thus, in [FeF6 ]4 -
as CN - is a strong ligand so when it approaches towards central
metal pairing of unpaired electrons takes place. sp3d2 hybridisation

Thus, in [Ni(CN)4 ]2 - 3d 4s 4p 4d
Fe2+
dsp2-hybridisation
F– F– F– F– F– F–
3d 4s 4p
Ni2+ Structure : Octahedral
So, 1 is the correct match for R.
CN– CN– CN– CN–
For S i.e., d 2 sp 3

w
Structure : Square planar
It is seen in [Ti(H2O)3 Cl3 ] and [Cr(NH3 )6 ]3+
So correct match for P is 6.
Ti ¾[Ar]3d 2 4 s2
For Q i.e., sp3
Ti3+ ¾[Ar]3d 1
It is seen in [FeCl4 ]2 - and Ni(CO)4

Flo
3d 4s 4p
Fe - [Ar]3d6 4 s2
Ti3+

ree
6
Fe 2+
- [Ar]3d
3d 4s 4p Here, both H2O and Cl are weak ligands

F
Fe2+ So, in [Ti(H2O)3 Cl3 ]
d2sp3 hybridisation
As Cl - is a weak ligand so when it approaches towards central

or
ur
metal pairing of unpaired electrons does not take place.
Ti3+
3d 4s 4p
Thus, in [FeCl4 ]2 -

f
H2O H2O H2O Cl– Cl– Cl–
sp3 hybridisation
ks
Structure Octahedral
Yo
3d 4s 4p
Fe2+ Likewise in [Cr(NH3)6]3+
oo

Cl– Cl– Cl– Cl– Cr ¾ [Ar]3d 5 4 s1


eB

Structure-Tetrahedral Cr 3+ ¾ [Ar]3d 3 4 s0
Likewise in Ni(CO)4 3d 4s 4p
Ni ¾[Ar]3d 8 4 s2
r

Cr3+
ou
ad

3d 4s 4p
Ni
Here, NH 3 is also a weak field ligand so due to its approach no
pairing takes place in Cr.
Y

As CO is a strong ligand, hence when it approaches towards Thus, In [Cr(NH3 )6 ]3+


central metal atom pairing of unpaired electron of central atom
d2sp3 hybridisation
nd
Re

takes place.
Thus, in Ni(CO)4
3d 4s 4p
Fi

sp3 hybridisation
Cr3+–
3d 4s 4p
NH3 NH3 NH3 NH3 NH3 NH3
Ni
CO CO CO CO So for, S-2 and 3 are the correct match.

Structure Tetrahedral 57. PLAN This problem is based on concept of VBT and magnetic
properties of coordination compound.
So, for Q-4 and 5 are correct match.
Draw VBT for each coordination compound.
For R i.e., sp3d 2
If unpaired electron is present then coordination compound will
It is seen in [FeF6]4- be paramagnetic otherwise diamagnetic.
Fe ¾ [Ar]3d6 4 s2 Coordination compounds of [MA4 B2 ] type show geometrical
Fe 2+ ¾ [Ar]3d6 isomerism. Molecular orbital electronic configuration (MOEC)
for various coordination compound can be drawn using VBT as
3d 4s 4p 4d
Fe2+
260 Coordination Compounds

A. MO EC for [Cr(NH3 )4 Cl 2 ]Cl is 58. (A) [Co(NH3 )4 (H2O)2 ]Cl 2:Co2+ , 3d 7


3d 4s 4p show geometrical isomerism, paramagnetic.
×× ×× ×× ×× ×× ×× (B) Pt(NH3 )2 Cl 2:Pt 2+ has d 8 -configuration with all paired
NH3NH3 NH3 NH3Cl Cl electrons. Show geometrical isomerism, diamagnetic.
(C) [Co(H2O)5 Cl]Cl : Co2+ , 3d 7
Number of unpaired electrons (n) = 3
Cannot show geometrical isomerism, paramagnetic.
Magnetic properties = paramagnetic
(D) [Ni(H2O)6 ]Cl 2: Ni 2+ , 3d 8 , weak ligand, has two unpaired
Geometrical isomers of [Cr(NH3 )4 Cl 2 ]+ are
electrons. Paramagnetic but cannot show geometrical
Cl Cl isomerism.
+ +
H3N Cl H 3N NH3 59. Complex part is cationic, named first : [Co(NH3 )6 ]Cl 3 :
Cr and Cr hexaammine cobalt (III) chloride.

w
H3N NH3 H 3N NH3 60. False : Cyanide (CN- ) is a strong ligand, brings about pairing of
NH3 Cl 3d electrons.
3d 6 4s2

Flo
Cis Trans
K4[Fe(CN)6] Fe
B. n=1 Potassium ferrocyanide
Magnetic properties = paramagnetic

ree
Ionisation isomers of [Ti(H2O)5 Cl](NO3 )2 are Fe(CN)64 –
[Ti(H2O)5 Cl](NO3 )2 and [Ti(H2O)5 (NO3 )]Cl(NO3 )
d 2 sp3

F
C. MOEC of [Pt(en)(NH3 )Cl]NO3 is No unpaired electron, diamagnetic

or
K3[Fe(CN)6] : Fe
××
en
××
en
ur
×× ××
××
Potassium ferricyanide
NH3 Cl 3d 6 4s2

f
n=0
[Fe(CN)6]3 - :
ks
Magnetic property = diamagnetic
Yo
Ionisation isomers are [Pt(en)2 (NH3 )Cl]NO3 d 2 sp3
oo
and [Pt(en)2 NH3 (NO3 )]Cl Has one unpaired electron, paramagnetic

D. MOEC of [Co(NH3 )4 (NO3 )2 ]NO3


eB

61. False : Lobes of 3dx2 - y2 orbitals lies in X Y plane on the X and


×× ×× ×× ×× ×× ×× Y coordinate axes, therefore electron density of dx2 - y2 orbital in
X Y plane is non-zero.
NH3NH3 NH3 NH3NO3NO3
r

n=0 62. When S is donor atom of SCN- , it produces weak ligand field
ou
ad

and forms high spin complex as


Magnetic property = Diamagnetic
[ Fe(SCN)6 ]3- : Fe3+ (3d 5 ) =
Y

Geometrical isomers are


4s 4p 4d
NO3 NH3
nd

3d 5(n-5)
Re

H3N NH3 NO3 NH3 sp3d2


CO and CO Spin only magnetic moment (m s ) = 5 (5 + 2) BM = 35 BM
Fi

H3N NH3 NO3 NH3


In case of CN- ligand, carbon is the donor atom , it produces
NO3 NH3 strong ligand field and forms low spin complex as
Trans Cis [Fe(CN)6 ]3- : Fe3+ (3d 5 )
Thus, magnetic property and isomerism in given coordination 3d 4s 4p
compound can be summarised as
(P) [Cr(NH3 )4 Cl 2 ]Cl ® Paramagnetic and exhibits cis-trans d2sp3
isomerism (3) Spin only magnetic moment (m s ) = 1 (1 + 2) BM = 3 BM
(Q) [Ti(H2O)5 Cl](NO3 )2 ® Paramagnetic and exhibits
ionisation isomerism (1) Hence, difference in spin only magnetic moment
(R) [Pt(en)(NH3 )Cl]NO3 ® Diamagnetic and exhibits = 35 - 3 » 4 BM
ionisation isomerism (4) C O
63.
(S) [Co(NH3 )4 (NO3 )2 ]NO3 ® Diamagnetic and exhibits Br O
==

(II)
cis-trans isomerism (2) Et3P Fe C ¾ CH 3
\ P ® 3, Q ® 1, R ® 4, S ® 2 acetylbromidodicarbonylbis
Et3P
Hence, (b) is the correct choice. C O (triethylphosphine)iron (II)
Coordination Compounds 261

64. PLAN EDTA is a multidentate ligand as it can donate six pairs of m = n (n + 2) BM = 8 BM


electrons – two pair from the two nitrogen atoms and four pair [Ni(CN)4 ]2 - : Ni 2+ (3d 8 ) , strong ligand field.
from the four terminal oxygens of the ¾ COO - groups.

OOCCH2 CH COO – 2 Ni2+(3d 8) :
N—CH2CH2—N

OOCCH2 CH2COO – dsp2
2+ Square planar
The structure of a chelate of a divalent Co with EDTA is
shown as m = 0 (no unpaired electron)
H 2C H 2C CH2 68. In complexes A and B, one halide (Cl - or Br - ) is outside
CH2
coordination sphere, i.e. complexes are :
H 2C N N
CO CO [Cr(NH3 )4 Br2 ]Cl and [Cr(NH3 )4 BrCl]Br

CH2
CO 1 A gives white precipitate AgCl with excess of AgNO3 which
4
O Co O

w
2 dissolve in excess ammonia. Therefore, A must be
3 CO [Cr(NH3 )4 Br2 ]Cl.
O O
Each N has four N ¾ Co ¾ O bonds thus total eight N ¾ Co ¾ O B gives a pale yellow precipitate with excess of AgNO3, which
bonds. dissolve in concentrated ammonia solution. Therefore,

Flo
H precipitate is AgBr and complex B is [Cr(NH3 )4 ClBr]Br.
65. H
O O In both A and B, hybridisation of chromium is d 2sp3 and
magnetic moment : m = n (n + 2) BM = 0

ree
H3C N N CH 3
C C
(3d6, strong ligand, no unpaired electron)
Ni
NH3

F
C C 69. 3+ 2–
H3C N N CH 3 H 3N NH3 NC CN
O O

or
Co Ni
H H
ur
Ni-DMG complex H 3N NH3 NC CN
Oxidation state of Ni is +2 and hybridisation is dsp . 2

f NH3
ks
d 2sp3-octahedral dsp2-square planar
m = 0 (no unpaired electron) hence, diamagnetic. CO
Yo
66. The spin-only magnetic moment (m) of the complex is 1.73 BM.
oo

It indicates that nucleus of complex, chromium ion has one Ni


CO
unpaired electron. So, the ligand NO is unit positively charged.
eB

OC CO
K2[Cr(NO) (CN)4 (NH3 )] sp3-tetrahedral
potassium amminetetracyanonitrosoniumchromate (I)
70. A has no water molecules of crystallisation.
Cr+1 :
r

Hence, A is [Cr(H2O)6 ]Cl 3.


ou
ad

5 0 Both B and C loses weight with concentrated H2SO4, therefore,


3d 4s
both B and C have some water molecules of crystallisation.
Cr(I) “under influence of strong ligand field”.
Y

Moreover, weight loss with C is just double of the same with B


indicates that number of water molecules of crystallisation of C
d 2sp3
is double of the same for B. Therefore, B has one and C has two
nd
Re

Octahedral water molecules of crystallisation.


NO 2– B = [Cr(H2O)5 Cl]Cl 2 × H2O, C = [Cr(H2O)4 Cl 2 ]Cl × 2H2O
Fi

NC CN 71. (i) [Ti(NO3 )4 ]: Ti 4+ (3d 0 ) No d-electron, no d-d transition


Cr possible, colourless.
(ii) [Cu(NCCH3 )]BF4 : Cu + (3d 10 ) All d-orbitals are
NC NH3 completely filled, no d-d transition possible, colourless.
CN (iii) [Cr(NH3 )6 ]Cl 3 : Cr 3+ (3d 3 ) Complex has allowed
Octahedral geometry d-d-transitions from t2g to eg level, hence coloured.
67. [NiCl 4 ] 2 - : Ni 2+ (3d 8 ), weak ligand field. (iv) K3[VF6 ]: V3+ (3d 2 ) Complex has allowed d-d-transitions
from t2g to eg level, hence coloured.
Ni2+(3d 8) :
72. I- is a strong reducing agent, reduces Cu 2+ to Cu + and
sp3 precipitate out as stable CuI.

Download Chapter Test


http://tinyurl.com/y4mcjkm6 or
19
Extraction of Metals

w
Objective Questions I (Only one correct option) (a) Only the reason is correct.

Flo
1. The correct statement is (b) Both the assertion and reason are correct explanation for the
assertion.
(a) leaching of bauxite using concentrated NaOH solution gives
sodium aluminate and sodium silicate. (2019 Main, 12 April II) (c) Both the assertion and reason are correct and the reason is the

ree
correct explanation for the assertion.
(b) the hall-heroult process is used for the production of
aluminium and iron. (d) Only the assertion is correct.

F
(c) pig iron is obtained from cast iron. 7. The ore that contains the metal in the form of fluoride is
(d) the blistered appearance of copper during the metallurgical (2019 Main, 9 April I)
(a) magnetite (b) sphalerite (c) malachite (d) cryolite

or
process is due to the evolution of CO2.
ur 8. The Mond process is used for the (2019 Main, 8 April II)
2. The idea of froth floatation method came from a person X and

f
this method is related to the process Y of ores. X and Y , (a) purification of Ni (b) extraction of Mo
ks
respectively, are (2019 Main, 12 April I) (c) purification of Zr and Ti (d) extraction of Zn
Yo
(a) fisher woman and concentration 9. With respect to an ore, Ellingham diagram helps to predict the
oo
(b) washer woman and concentration feasibility of its (2019 Main, 8 April I)
(c) fisher man and reduction (a) electrolysis (b) zone refining
eB

(d) washer man and reduction (c) vapour phase refining (d) thermal reduction
3. The correct statement is (2019 Main, 10 April II) 10. The pair that does not require calcination is
(a) zone refining process is used for the refining of titanium. (2019 Main, 12 Jan II)
r

(b) zincite is a carbonate ore. (a) ZnO and MgO (b) ZnO and Fe2O3 × xH2O
ou
ad

(c) sodium cyanide cannot be used in the metallurgy of silver. (c) ZnCO3 and CaO (d) Fe2O3 and CaCO3 × MgCO3
(d) aniline is a froth stabiliser. 11. In the Hall-Heroult process, aluminium is formed at the
Y

4. Match the refining methods Column I with metals Column II. cathode. The cathode is made out of (2019 Main, 12 Jan I)
(a) platinum (b) carbon
nd

Column I Column II (c) pure aluminium (d) copper


Re

(Refining Methods) (Metals)


12. The reaction that does not define calcination is
Fi

I. Liquation (A) Zr D
(a) Fe2O3 × XH2O ¾® Fe2O3 + XH2O (2019 Main, 11 Jan II)
II. Zone refining (B) Ni D
(b) ZnCO3 ¾® ZnO + CO2
III. Mond process (C) Sn D
(c) CaCO3 × MgCO3 ¾® CaO + MgO + 2CO2
IV. van Arkel method (D) Ga
D
(d) 2Cu2S + 3O2 ¾® 2Cu2O + 2SO2
(2019 Main, 10 April I)
(a) I- (C) ; II-(D); III-(B) ; IV-(A) 13. Match the ores ( Column A ) with the metals (Column B).
(b) I- (B) ; II-(C); III-(D) ; IV-(A)
(c) I- (C) ; II-(A); III-(B) ; IV-(D) Column A Column B
(d) I- (B) ; II-(D); III-(A) ; IV-(C) Ores Metals
5. The one that is not a carbonate ore is (2019 Main, 9 April II) A. Siderite P. Zinc
(a) siderite (b) calamine (c) malachite (d) bauxite B. Kaolinite Q. Copper
6. Assertion For the extraction of iron, haematite ore is used. C. Malachite R. Iron
Reason Heamatite is a carbonate ore of iron. D. Calamine S. Aluminium
(2019 Main, 9 April II)
(2019 Main, 11 Jan I)
Extraction of Metals 263

(a) A - P; B- Q; C - R; D- S (b) Al 2O3 is mixed with CaF2 which lowers the melting point of
(b) A - R; B- S; C - P; D- Q the mixture and brings conductivity
(c) A - Q; B- R; C - S; D- P (c) Al 3+ is reduced at the cathode to form Al
(d) A - R; B- S; C - Q; D- P (d) Na 3AlF6 serves as the electrolyte
14. The electrolytes usually used in the electroplating of gold 21. The metal that cannot be obtained by electrolysis of an
and silver, respectively, are (2019 Main, 10 Jan II) aqueous solution of its salts is (2014 Main)
(a) [Au(OH)4 ]- and [Ag(OH)2 ]- (a) Ag (b) Ca
(b) [Au(NH3 )2 ]+ and [Ag(CN)2 ]- (c) Cu (d) Cr
(c) [Au(CN)2 ]- and [Ag(CN)2 ]- 22. Sulphide ores are common for the metals (2013 Adv.)
(d) [Au(CN)2 ]- and [AgCl 2 ]- (a) Ag, Cu and Pb (b) Ag, Cu and Sn
(c) Ag, Mg and Pb (d) Al, Cu and Pb

w
15. Hall-Heroult’s process is given by (2019 Main, 10 Jan I)
Coke, 1673 K 23. In the cyanide extraction process of silver from argentite ore,
(a) ZnO + C ¾¾¾¾® Zn + CO
the oxidising and reducing agents used are (2012)
(b) Cr2O3 + 2Al ¾¾® Al 2O3 + 2Cr (a) O2 and CO respectively

Flo
(c) 2Al 2O3 + 3C ¾
¾® 4Al + 3CO2 (b) O2 and Zn dust respectively
(d) Cu2+ (aq) + H2 (g ) ¾
¾® Cu(s) + 2H+ (aq) (c) HNO3 and Zn dust respectively

ree
(d) HNO3 and CO respectively
16. The correct statement regarding the given Ellingham
diagram is (2019 Main, 9 Jan II) 24. Oxidation states of the metal in the minerals haematite and

F
magnetite, respectively, are (2011)
Cu 2O
O2 ®2 (a) II, III in haematite and III in magnetite
4Cu+
(b) II, III in haematite and II in magnetite

or
–300
ur (c) II in haematite and II, III in magnetite

f
(d) III in haematite and II, III in magnetite
DGº (kJ/mol)

2C
+O
2ZnO ®2
ks
–600 O2 ®
2
CO 25. Native silver metal forms a water soluble complex with a
2Zn+ dilute aqueous solution of NaCN in the presence of
Yo
oo
3 Al 2
O3 (a) nitrogen (b) oxygen (2008, 3M)
® 2/
l+O 2 (c) carbon dioxide (d) argon
4/3 A
eB

–1050
500ºC 800ºC 2000ºC 26. Extraction of zinc from zinc blende is achieved by
Temperature (ºC) (a) electrolytic reduction (2007, 3M)
(b) roasting followed by reduction with carbon
r

(a) At 800°C, Cu can be used for the extraction of Zn from ZnO


(c) roasting followed by reduction with another metal
ou
ad

(b) At 1400°C, Al can be used for the extraction of Zn from ZnO


(d) roasting followed by self-reduction
(c) At 500°C, coke can be used for the extraction of Zn from ZnO
Y

(d) Coke cannot be used for the extraction of Cu from Cu 2O 27. Which ore contains both iron and copper? (2005, 1M)
(a) Cuprite (b) Chalcocite
17. The ore that contains both iron and copper is
(c) Chalcopyrite (d) Malachite
nd

(2019 Main, 9 Jan I)


Re

(a) malachite (b) azurite 28. The methods chiefly used for the extraction of lead and tin
(c) dolomite (d) copper pyrites
Fi

from their ores are respectively (2004, 1M)


18. Which one of the following ores is best concentrated by froth (a) self-reduction and carbon reduction
floatation method? (2016 Main) (b) self-reduction and electrolytic reduction
(a) Siderite (b) Galena (c) carbon reduction and self-reduction
(c) Malachite (d) Magnetite (d) cyanide process and carbon reduction
19. From the following statements regarding H2 O2 choose the 29. In the process of extraction of gold,
incorrect statement. (2015 Main) O2
(a) It can act only as an oxidising agent
Roasted gold ore + CN– + H2O ¾® [ X ] + HO
(b) It decomposed on exposure to light [ X ] + Zn ¾® [Y ] + Au
Identify the complexes [X] and [Y]. (2003, 1M)
(c) It has to be stored in plastic or wax lined glass bottles in dark - 2-
(d) It has to be kept away from dust (a) X = [Au(CN)2 ] , Y = [Zn(CN)4 ]
(b) X = [Au(CN)4 ] 3 – ,Y = [Zn (CN)4 ] 2–
20. In the context of the Hall-Heroult process for the extraction
of Al, which of the following statements is false? (c) X = [Au(CN)2 ] - , Y = [Zn(CN)6 ] 4-
(a) CO and CO2 are produced in this process (2015 Main) (d) X = [Au(CN)4 ] - , Y = [ Zn(CN)4 ] 2-
264 Extraction of Metals

30. Anhydrous ferric chloride is prepared by (2002) 39. Type of bonds present in CuSO4 × 5H2O are only (1983)
(a) heating hydrated ferric chloride at a high temperature (a) electrovalent and covalent
in a stream of air (b) electrovalent and coordinate covalent
(b) heating metallic iron in a stream of dry chlorine gas (c) electrovalent, covalent and coordinate covalent
(c) reaction of ferric oxide with hydrochloric acid (d) covalent and coordinate covalent
(d) reaction of metallic iron with hydrochloric acid 40. In metallurgy of iron, when limestone is added to the blast
31. Which of the following process is used in extractive furnace, the calcium ion ends up in (1982)
metallurgy of magnesium? (2002, 3M) (a) slag
(a) Fused salt electrolysis (b) gangue
(b) Self-reduction (c) metallic calcium
(c) Aqueous solution electrolysis (d) calcium carbonate

w
(d) Thermite reduction 41. Iron is rendered passive by treatment with concentrated
32. The chemical composition of ‘slag’ formed during the (a) H2SO4 (b) H3PO4 (1982)
smelting process in the extraction of copper is (2001, 1M)
(c) HCl (d) HNO3

Flo
(a) Cu2O + FeS (b) FeSiO3
(c) CuFeS2 (d) Cu2S + FeO Objective Questions II

ree
(One or more than one correct option)
33. Electrolytic reduction of alumina to aluminium by
Hall-Heroult process is carried out (2000, 1M) 42. Extraction of copper from copper pyrite (CuFeS 2 ) involves

F
(a) in the presence of NaCl (2016 Adv.)
(b) in the presence of fluorite (a) crushing followed by concentration of the ore by
(c) in the presence of cryolite which forms a melt with lower froth-floatation

or
melting temperature
ur (b) removal of iron as slag

f
(d) in the presence of cryolite which forms a melt with higher (c) self reduction step to produce ‘blister copper’ following
melting temperature evolution of SO2
ks
34. The chemical process in the production of steel from (d) refining of ‘blister copper’ by carbon reduction
Yo
haematite ore involve (2000, 1M) 43. Copper is purified by electrolytic refining of blister copper.
oo
(a) reduction The correct statement(s) about this process is/are (2015 Adv.)
(b) oxidation
eB

(a) impure Cu strip is used as cathode


(c) reduction followed by oxidation (b) acidified aqueous CuSO4 is used as electrolyte
(d) oxidation followed by reduction (c) pure Cu deposits at cathode
(d) impurities settle as anode-mud
r

35. In the commercial electrochemical process for aluminium


ou

44. Upon heating with Cu 2 S, the reagent(s) that give copper


ad

extraction, the electrolyte used is (1999, 2M)


(a) Al(OH)3 in NaOH solution metal is/are (2014 Adv.)
Y

(b) an aqueous solution of Al 2 (SO4 )3 (a) CuFeS2 (b) CuO


(c) a molten mixture of Al 2O3 and Na 3AlF6 (c) Cu2O (d) CuSO4
nd

(d) a molten mixture of AlO(OH) and Al(OH)3 45. The carbon-based reduction method is not used for the
Re

36. The major role of fluorspar (CaF2 ) which is added in small extraction of (2013 Adv.)
(a) tin from SnO2
Fi

amount in the electrolytic reduction of alumina dissolved in


(b) iron from Fe2O3
fused cryolite (Na 3 AlF6 ) is (1993, 1M)
(c) aluminium from Al 2O3
(a) as a catalyst
(d) magnesium from MgCO3 , CaCO3
(b) to make the fused mixture very conducting
(c) to increase the temperature of the melt 46. Extraction of metal from the ore cassiterite involves (2011)

(d) to decrease the rate of oxidation of carbon at the anode


(a) carbon reduction of an oxide ore
(b) self-reduction of a sulphide ore
37. Hydrogen gas will not reduce (1985, 1M) (c) removal of copper impurity
(a) heated cupric oxide (d) removal of iron impurity
(b) heated ferric oxide 47. Addition of high proportions of manganese makes steel
(c) heated stannic oxide useful in making rails (1998)
(d) heated aluminium oxide (a) gives hardness to steel
38. In the alumino-thermite process, aluminium acts as (b) helps the formation of oxides of iron
(a) an oxidising agent (b) a flux (1983, 1M)
(c) can remove oxygen and sulphur
(d) can show highest oxidation state of + 7
(c) a reducing agent (d) a solder
Extraction of Metals 265

48. Of the following, the metals that cannot be obtained by 55. Match each of the reactions given in Column I with the
electrolysis of the aqueous solution of their salts are corresponding product(s) given in Column II. (2009)
(a) Ag (b) Mg (c) Cu (1990, 1M)
(d) Al (e) Cr Column I Column II

49. In the electrolysis of alumina, cryolite is added to (1986, 1M) A. Cu + dil. HNO3 p. NO
(a) lower the melting point of alumina B. Cu + conc. q. NO2
(b) increase the electrical conductivity
HNO3
(c) minimise the anode effect
(d) remove impurities from alumina C. Zn + dil. HNO3 r. N2 O
D. Zn + conc. s. Cu(NO3 )2
Assertion and Reason HNO3

w
Read the following questions and answer as per the direction t. Zn(NO3 )2
given below :
(a) Statement I is correct; Statement II is correct; Statement II is 56. Match the conversions in Column I with the type(s) of
the correct explanation of Statement I. reaction(s) given in Column II. (2008, 6M)

Flo
(b) Statement I is correct; Statement II is correct; Statement II is
not the correct explanation of Statement I. Column I Column II
(c) Statement I is correct; Statement II is incorrect.

ree
(d) Statement I is incorrect; Statement II is true.
A. PbS ¾® PbO p. Roasting

50. Statement I Al(OH)3 is amphoteric in nature. B. CaCO3 ¾® CaO q. Calcination

F
Statement II Al—O and O—H bonds can be broken with C. ZnS ¾® Zn r. Carbon reduction
equal ease in Al(OH)3 . (1998)
D. Cu 2 S ¾® Cu s. Self-reduction

or
ur
Passage Based Questions 57. Match the extraction processes listed in Column I with
Passage
f
metals listed in Column II. (2006, 6M)
ks
Copper is the most noble of the first row transition metals and
Yo
Column I Column II
occurs in small deposits in several countries. Ores of copper include
oo
chalcanthite (CuSO4 × 5H2 O), atacamite (Cu 2 Cl(OH)3 ), cuprite A. Self-reduction p. Lead
(Cu 2 O), copper glance (Cu 2 S) and malachite (Cu 2 (OH)2 CO3 ).
eB

B. Carbon reduction q. Silver


However, 80% of the world copper production comes from the ore
chalcopyrite (CuFeS2 ). The extraction of copper from chalcopyrite C. Complex formation and r. Copper
involves partial roasting, removal of iron and self-reduction. displacement by metal
r

(2010) D. Decomposition of iodide s. Boron


ou
ad

51. Partial roasting of chalcopyrite produces


(a) Cu 2S and FeO (b) Cu 2O and FeO 58. Each entry in Column X is in some way related to the entries in
Y

(c) CuS and Fe2O3 (d) Cu 2O and Fe2O3 Columns Y and Z. Match the appropriate entries. (1988, 3M)
52. Iron is removed from chalcopyrite as
nd
Re

(a) FeO (b) FeS (c) Fe2O3 (d) FeSiO3 Column X Column Y Column Z

53. In self-reduction, the reducing species is A. Invar p. Co, Ni m. Cutlery


Fi

2- 2-
(a) S (b) O (c) S (d) SO2 B. Nichrome q. Fe, Ni n. Heating element
C. Stainless r. Fe, Cr, Ni o. Watch spring
Match the Columns steel
54. Match the anionic species given in Column I that are present
59. Match the following choosing one item from Column X and
in the ore (s) given in Column II. (2015 Adv.)
the appropriate item from Column Y. (1983, 2M)
Column I Column II
Column X Column Y
A. Carbonate p. Siderite
A. Al p. Calamine
B. Sulphide q. Malachite
B. Cu q. Cryolite
C. Hydroxide r. Bauxite
C. Mg r. Malachite
D. Oxide s. Calamine
D. Zn s. Carnalite
t. Argentite
266 Extraction of Metals

60. Match the following metals listed in Column I with 76. A1 and A2 are two ores of metal M . A1 on calcination gives
extraction processes listed in Column II. (1979, 2M) black precipitate, CO2 and water.
n Black solid + CO2 + H2O
Column I Column II tio
l c ina
A. Silver p. Fused salt electrolysis Ca
Roasting
Dil. H ; A2 Metal + gas
A1
B. Calcium q. Carbon reduction Cl
KI
C. Zinc r. Carbon monoxide I2 + ppt. K 2Cr2O7
+ H2SO4
reduction
D. Iron s. Amalgamation Green colour

E. Copper t. Self-reduction Identify A1 and A2 . (2004, 4M)

w
77. Which of the two, anhydrous or hydrated AlCl3 is more
Fill in the Blanks soluble in diethyl ether? Justify using the concepts of
bonding in not more than 2 or 3 sentences. (2003)
61. Silver jewellery items tarnish slowly in the air due to their

Flo
reaction with………… (1997)
78. Write the balanced chemical reactions involved in the
extraction of lead from galena. Mention oxidation state of
62. In the extractive metallurgy of zinc, partial fusion of ZnO lead in litharge.

ree
(2003, 2M)
with coke is called …… and reduction of the ore to the
molten metal is called …… (smelting, calcining, roasting, 79. Write the balanced chemical equation for developing
photographic films.

F
(2000)
sintering). (1988, 1M)

63. Silver chloride is sparingly soluble in water because its 80. Write the chemical reactions involved in the extraction of
silver from argentite. (2000, 2M)

or
lattice energy greater than ……….. energy.
ur (1987)
81. Work out the following using chemical equations.
64. Galvanisation of iron denote coating with …… (1983)

f
In moist air, copper corrodes to produce a green layer on the
65. Cassiterite is an ore of …… (1980, 1M) surface.
ks
(1998)
66. In the thermite process …… is used as a reducing agent.
Yo
82. When the ore haematite is burnt in air with coke around
(1980, 1M)
oo
2000°C along with lime, the process not only produces steel
67. In the basic Bessemer process for the manufacture of steel, but also produces a silicate slag, that is useful in making
eB

the lining of the converter is made up of … . The slag formed building materials such as cement. Discuss the same and
consists of …… (1980, 1M) show through balanced chemical equations. (1998, 4M)
68. AgCl dissolve in excess of KCN solution to give ………… 83. Give balance equation for the reaction of aluminium with
r

complex compound. (1980) aqueous sodium hydroxide. (1997)


ou
ad

True/False 84. Write a balanced equation for the reaction of argentite with
KCN and name the products in the solution.
Y

(1996)
69 Cu + disproportionate to Cu 2+ and elemental copper in
85. Give reasons for the following
solution.
nd

(1991) “Although aluminium is above hydrogen in the


Re

70. Silver chloride is more soluble in very concentrated sodium electrochemical series, it is stable in air and water.”
(1994, 1M)
Fi

chloride solution than in pure water. (1984, 1M)


86. Complete the following reaction :
71. Dilute HCl oxidises metallic Fe to Fe2+ . (1983, 1M) Sn + 2KOH + 4H2 O ¾® ...... + ....... (1994)
72. Silver fluoride is fairly soluble in water. (1982)
87. Give briefly the isolation of magnesium from sea water by
the Dow’s process.
Subjective Questions Give equations for the steps involved. (1993, 3M)
73. Give the coordination number of Al in the crystalline state of 88. Complete and balance the following reaction :
AlCl3. (2009, 2M)
Copper reacts with HNO3 to give NO and NO2 in the molar
74. Give the number of water molecule (s) directly bonded to the ratio of 2:1
metal centre in CuSO4 × 5H2O. (2009, 2M) Cu + HNO3 ¾® KK + NO + NO2 + KK (1992)

75. Write balanced chemical equation for developing a black and 89. Write balanced equation for the extraction of “Copper from
white photographic film. Also give reason, why the solution copper pyrites by self reduction.” (1990, 2M)
of sodium thiosulphate on acidification turns milky white 90. Give balanced equations for the extraction of “Silver from
and give balance equation of this reaction. (2005, 2M) silver glance by cyanide process.” (1988, 1M)
Extraction of Metals 267

91. Answer the following questions briefly (i) Metals can be recovered from their ores by chemical methods
(i) What is the actual reducing agent of haematite in blast furnace? (ii) High purity metals can be obtained by zone refining method.
(ii) Give the equation for the recovery of lead from galena by air (1984, 2M)
reduction. 94. Give reason for the following in one or two sentences :
(iii) Why is sodium chloride added during electrolysis of fused “Silver bromide is used in photography.” (1983)
anhydrous magnesium chloride? 95. State the conditions under which the preparation of
(iv) Zinc, not copper is used for the recovery of metallic silver from alumina from aluminium is carried out. Give the necessary
complex [Ag(CN)2 ]– , explain. equations which need not be balanced. (1983, 2M)
(v) Why is chalcocite roasted and not calcinated during recovery of 96. Write the chemical equations involved in the extraction of
copper? (1987, 5M)
lead from galena by self reduction process. (1979, 2M)
92. Write balanced chemical equation for the following
97. Write balanced equation involved in the preparation of tin

w
“Gold is dissolved in aqua regia.” (1987)
metal from cassiterite. (1979)
93. Each of the following statement is true, only under some
specific conditions. Write the condition for each subquestion in
not more than 2 sentences.

Flo
Answers

ree
1. (a) 2. (b) 3. (d) 4. (a) 53. (c) 54. A ® p, q, s; B ® t; C ® q; D ® r
5. (d) 6. (d) 7. (d) 8. (a) 55. A ® p, s; B ® q, s; C ® r, t; D ® q, t

F
9. (d) 10. (a) 11. (b) 12. (d)
56. A ® p; B ® q; C ® r, s; D ® p, s
13. (d) 14. (c) 15. (c) 16. (b)
57. A ® p, r; B ® p; C ® q; D ® s

or
17. (d) 18. (b) 19. (a) 20. (d)
21. (b) 22. (a)
ur
23. (b) 24. (d)
58. A ® q, o; B ® p, n; C ® r, m
59. A ® q; B ® r; C ® s; D ® p

f
25. (b) 26. (b) 27. (c) 28. (a)
60. A ® s; B ® p; C ® q; D ® q, r; E ® t
ks
29. (a) 30. (b) 31. (a) 32. (b)
61. H 2S 62. Sintering, Smelting
Yo
33. (c) 34. (a) 35. (c) 36. (b)
oo
37. (d) 38. (c) 39. (c) 40. (a) 63. Hydration 64. Zn 65. Sn 66. Al
67. Lime, calcium phosphate 68. K [Ag(CN) 2 ]
41. (d) 42. (a,b,c) 43. (b,c,d) 44. (b,c,d)
eB

69. T 70. T 71. T 72. T


45. (c,d) 46. (a,d) 47. (a,c) 48. (b,d)
73. (6) 74. (4) 78. (2)
49. (a,b) 50. (b) 51. (b) 52. (d)
r
ou
ad

Hints & Solutions


Y

1. The correct statement is ‘‘leaching of bauxite using concentrated This method is based upon the preferential wetting properties
NaOH solution gives sodium aluminate and sodium silicate’’. with the frothing agent (collector) and water.
nd
Re

Bauxite usually contains SiO2, iron oxides and titanium oxide 3. The explanation of given statements are as follows:
(TiO2 ) as impurities. Concentration is carried out by digesting the
(a) Zone refining process is used for the refining of B, Ga, In,
powdered ore with a concentrated solution of NaOH at 473-523 K
Fi

and 35-36 bar pressure. Al2O3 is leached out as sodium aluminate Si and Ge.
(and SiO2 too as sodium silicate) leaving the impurities behind. Ti is refined by van Arkel method.
Al 2O3 (s) + 2NaOH(aq) + 3H2O(l ) ¾® 2Na[Al(OH)4 ](aq) Thus, statement (a) is incorrect.
The aluminate in solution is neutralised by passing CO2 gas and (b) Zincite (ZnO) is an oxide ore of Zn.
hydrated Al 2O3 is precipitated. Here, the solution is seeded with Thus, statement (b) is incorrect.
freshly prepared samples of hydrated Al 2O3 which induces (c) NaCN is used in the hydrometallurgy of silver. It is known
precipitation. as Mc. Arthur Forrest process.
2Na[Al(OH)4 ](aq) + CO2 (g ) ¾® The reactions occuring during the process are as follows:
Al 2O3 × xH2O(s) + 2NaHCO3 (aq) Ag2S + 4NaCN ¾® 2Na[Ag(CN)2 ] + Na 2S
The sodium silicate remains in the solution and hydrated alumina 4Na 2S + 2H2O + 5O2 ¾® 2Na 2SO4 + 4NaOH + 2S
is filtered, dried and heated to give back pure Al 2O3.
2Na[Ag(CN)2 ] + Zn ¾® Na 2[Zn(CN)4 ] + 2Ag
Al 2O3 × xH2O(s) ¾1470
¾¾ K
® Al 2O3 (s) + xH2O(g ) Thus, statement (c) is incorrect.
2. The idea of froth floatation method came from a person ‘washer (d) Aniline and cresol help in stabilising the froth in froth
woman’ ( X ) and this method is related to the process concentration floatation process.
(Y ) of ores. Thus, statement (d) is correct.
268 Extraction of Metals

4. Refining of crude metals results pure metals and its impurities 9. With respect to an ore, Ellingham diagram helps to predict the
get separated out. feasibility of its thermal reduction. It is a graph representation of
I. Liquation In this method low melting metals like Gibbs energy change versus absolute temperature.
Sn, Pb, Bi and Hg can be made to flow down through a 0
sloping surface leaving behind the higher melting impurities –100 2Cu 2O
on the hearth. –200 4Cu+O 2
2FeO
e + O 2
II. Zone refining The basic principle of the method is, –300 2F C+O2 CO2
impurities are more soluble in the molten metal than in the –400

DGº/kJ mol–1 of O2
2C+O
2CO 2 2
solid state of the metal. This method is useful to produce –500 +O 2
2Zn
O 2CO
2CO
semiconductors and ultra-pure metals like B, Ga, In, Si and –600
2Zn
+O 2
Ge. –700
III. Mond process –800
l +O 3

w
–900 2/3 A 2
l+O 2
330-350 K 450-470 K
Crude nickel (s) + 4CO(g) Ni(CO)4(g) A O
–1000 4/3 2Mg
(Impure) (Volatile compound) +O 2
–1100 2Mg
Recycled Ni(s)

Flo
4CO(g) + –1200
(Pure)

IV. van Arkel method 0°C 400°C 800°C 1200°C 1600°C 2000°C
273 K 673 K 1073 K 1473 K 1873 K 2273 K

ree
500-600 K 1700-1800 K Temperature
M (s) + 2I2 (g) MI4(g)
(Crude (Volatile compound) Gibbs energy (DGº) versus T plots (schematic)
metal) for the formation of some oxides (Ellingham diagram)

F
Recycled M(s)
2I2(g) +
(Ultra-pure) Generally, the diagram consists of plots of DG ° versus T for the
formation of oxides of elements

or
Here, M = Zr, Hf, Ti
ur 2xM (s) + O2 (g ) ¾¾® 2M x O(s)
Hence, the correct matching is

f
Thermal reduction product
I® (C), II®(D), III ®(B), IV ® (A). In this reaction, amount of gas decreases thus, randomness
ks
5. Bauxite is not a carbonate ore. Its chemical formula is Al 2O3 or decreases. Hence, DS becomes negative. Therefore, the value of
Yo
free energy increases with increase in temperature. There is a
AlOx(OH)3 - 2x , where 0 < x < I. Chemical formula of other ores
oo
point in a curve below which DG is negative. So, M xO is stable.
given in options are as follows:
Above this point, M xO will decompose on its own.
eB

Siderite-FeCO3
Calamine-ZnCO3 10. The hydroxide, hydrated oxides and carbonate ores, after
concentration, are subjected to calcination. In the process, the
Malachite-CuCO3 × Cu(OH)2
ore is heated below its melting point in the limited supply or
r

6. Only assertion is correct and reason is incorrect. Haematite is absence of air. As the result, these are converted into their
ou
ad

not a carbonate ore. It is an oxide ore, i.e. Fe2O3. Cast iron is oxides.
extracted chiefly from its oxide ore (haematite) by heating in the So, among the given options, the options having either
Y

presence of coke and limestone in a blast furnace. carbonates (e.g. ZnCO3 and CaCO3 × MgCO3) or hydrated oxide
7. Cryolite ore (Na 3AlF6, sodium hexafluoroaluminate) contain (e.g. Fe2O3 × xH2O), require calcination while pair of option (a),
i.e. ZnO and MgO does not require calcination.
nd
Re

fluorine while other given options such as malachite


(Cu 2 (CO)3 (OH)2 ), sphalerite ((Zn,Fe)S) and bauxite (Al 2O3 ) 11. In the Hall - Heroult’s process, aluminium in formed at the
Fi

does not contain fluorine. cathode. The cathode is made out of carbon. In this method,
Al 2O3 is melted with cryolite, Na 3[AlF6 ] and electrolysed in a
8. Mond process is used in the purification of Ni. It is a vapour graphite lined steel tank, which serves as the cathode. The anode
phase refining process. is also made of graphite.
It is based on the principle that Ni is heated in the presence of The cell runs continuously and at intervals molten aluminium is
carbon monoxide to form nickel tetracarbonyl, which is a drained from the bottom of the cell and more bauxite is added.
volatile complex. This complex is then decomposed by The electrolytic reactions are as follows:
subjecting it to a higher temperature (450-470 K) to obtain pure
nickel metal. At cathode Al3+ + 3e- ¾® Al

Crude nickel (s) + 4 CO (g) At anode C(s) + O 2- (melt) ¾® CO(g)+ 2e-


(Impure) Recycled C(s) + 2O 2- (melt) ¾® CO2(g)+ 4 e-
330-350 K
12. Calcination is one of the pyrometallurgical process, like
450-470 K Ni (s) roasting by which a concentrated ore gets converted into its
[Ni (CO)4] (g) Pure + 4 CO (g) oxide.
(Volatile compound)
In calcination, a hydrated carbonate or bicarbonate ore or a
hydrated ore is heated at lower temperature (compared to
Extraction of Metals 269

roasting) in absence of air to give its oxide as in options (a), (b) 19. H2O2 acts as an oxidising as well as reducing agent, because
and (c). Here, volatile non-metallic oxides like H2O, CO2, are oxidation number of oxygen in H2O2 is -1. So, it can be oxidised
also produced. to oxidation state 0 or reduced to oxidation state –2.
Roasting is valid mainly for sulphide ores like option (d), where
SO2 gets liberated. In this reaction, calcination cannot be used. H2O2 decomposes on exposure to light. So, it has to be stored in
plastic or wax linked glass bottles in dark for the prevention of
13. The correct match is: A ® R; B ® S; C ® Q; D ® P. exposure. It also has to be kept away from dust.
(A) Siderite is an ore of iron with molecular formula FeCO 3 (R). 20. (a) In Hall-Heroult process for extraction Al, carbon anode is
(B) Kaolinite is an ore of aluminium with molecular formula oxidised to CO and CO2.
Al 2Si 2(OH)2O 5 (S). (b) When Al 2O3 is mixed with CaF2, it lowers the melting point
(C) Malachite is an ore of copper with molecular formula of the mixture and brings conductivity.
CuCO 3 × Cu(OH)2 (Q). (c) Al 3+ is reduced at cathode to form Al.

w
(D) Calamine is an ore of zinc with molecular formula (d) Here, Al 2O3 is an electrolyte, undergoing the redox process.
ZnCO 3 (P). Na 3AlF6 although is an electrolyte but serves as a solvent, not
14. Electroplating is a process of coating one metal or metal object electrolyte.

Flo
with a very thin layer of another metal typically applying a 21. Higher the position of element in the electrochemical series
direct electric current. more difficult is the reduction of its cations.
Electrolytes used in the electroplating of gold and silver are
If Ca 2+ (aq) is electrolysed, water is reduced in preference to it.

ree
given in the table below:
Hence, it cannot be reduced electrolytically from their aqueous
Pure metal solution.

F
Article to block acts an
be plated anode by which
Electrolyte Ca 2+ ( aq) + H2O ¾® Ca 2+ + OH- + H2 ­
Process (aqueous
out acts as electroplating
22.

or
solution) Element Ores Name
cathode
ur
will be done
Ag Ag 2 S Argentite

f
(a) Gold Article Au( s ) I
Na[A u(CN)2 ] Cu CuFeS2 Copper pyrites
plating
ks
(Sodium Pb PbS Galena
Yo
auro-cyanide) Sn SnO2 Cassiterite
oo
(b) Silver Article Ag( s ) I
Na[A g(CN)2 ] Mg MgCO3 × CaCO3 Dolomite
plating
eB

(Sodium argento Al Al 2 O3 × xH2 O Bauxite


cyanide)
23. The reactions involved in extraction of silver by cyanide process
r

15. Hall-Heroult’s process is an electro-reduction process by which are


Ag2S + CN- + O2 ¾® [Ag(CN)2 ]- + SO2
ou

pure alumina (Al 2O3 ) is reduced to crude Al. …(i)


ad

In this process, electrolysis of a fused mixture of Al 2O3, [Ag (CN)2 ]- + Zn ¾® [Zn (CN)4 ]2- + Ag …(ii)
Y

Na 3[AlF6 ] (cryolite) and CaF2 (fluorspar) is carried out at carbon


cathode and graphite anode. In reaction (i), sulphide is oxidised to SO2 by oxygen. In the
reaction (ii), silver ion (Ag+ ) is reduced to Ag by Zn. Therefore,
The overall reaction is represented as:
nd
Re

O2 is oxidising agent and Zn is reducing agent.


2Al 2O3 + 3C ¾® 4Al + 3CO2
24. Haematite is Fe2O3, in which oxidation number of iron is III.
Fi

16. From the Ellingham diagram, we can say that any oxide with Magnetite is Fe3O4 which is infact a mixed oxide (FeO × Fe2O3),
lower value of DG ° is more stable than a oxide with higher value hence iron is present in both II and III oxidation state.
of DG °. We can also predict that the oxide placed higher in the
diagram can be reduced by the element involved in the 25. A water soluble complex with silver and dilute aqueous solution
formation of its oxide placed lower at that temperature in the of NaCN is Na[Ag(CN)2 ]. In the cyanide process, the native
diagram. silver is crushed and treated with aqueous NaCN solution and
aerated.
It is happening in case of ZnO for its reduction by Al at 1400°C.
4Ag + 8NaCN + 2H2O + O2 ¾® 4Na [Ag(CN)2 ]
17. The formulae of the given ores are as follows: + 4NaOH
Malachite : CuCO3 × Cu(OH)2
26. Zinc blende contain ZnS which is first roasted partially and then
Copper pyrites : CuFeS2 subjected to reduction with carbon
Dolomite : CaMg(CO3 )2 ZnS + O2 ¾® ZnO + SO2 Roasting
Azurite : Cu 3 (CO)3 (OH)2
D
ZnO + C ¾® Zn + CO ­ Carbon reduction
18. Sulphide ores are concentrated by froth floatation method e.g.
Galena (PbS) 27. Chalcopyrite contain both iron and copper.
270 Extraction of Metals

28. Lead is mainly extracted by self-reduction process while tin is 43. (a) is wrong statement. Impure copper is set as anode where
extracted by carbon reduction method. copper is oxidised to Cu 2+ and goes into electrolytic
29. Au + 2CN- ¾® [Au(CN)2 ]- solutions.
X
(b) CuSO4 is used as an electrolyte in purification process.
(c) Pure copper is deposited at cathode as:
2Au(CN)2- + Zn ¾® [Zn(CN)4 ] 2-
+ 2Au
Y Cu 2+ + 2e– ¾® Cu : (At cathode)
30. Heating iron in stream of dry chlorine gas gives FeCl 3 in (d) Less active metals like Ag, Au etc settle down as anode mud.
anhydrous form. In all other cases (a and c) hydrated FeCl 3 is 1100 °C
obtained while in (d), FeCl 2 is formed. 44. (b) 4CuO ¾¾¾® 2Cu 2O + O2
31. Mg is extracted by electrolysis of molten MgCl 2. D
2Cu 2O + Cu 2S ¾® 6Cu + SO2

w
32. Iron present in copper pyrite is removed by forming FeSiO3 as D
(c) Cu 2S + 2Cu 2O ¾® 6Cu + SO2
slag.
720 °C 1
33. Cryolite is added to alumina in order to lower the melting point. (d) CuSO4 ¾¾¾® CuO + SO2 + O2
2

Flo
34. Haematite ore contain Fe2O3 which is reduced by CO in the blast 1100 °C
furnace as 4CuO ¾¾¾® 2Cu 2O + O2
Fe2O3 + CO ¾® Fe + CO2 D
2Cu 2O + Cu 2S ¾® 6Cu + SO2

ree
35. Al 2O3 mixed with cryolite Na 3[AlF6 ]is fused and electrolysed in Reaction is believed to proceed as
the extraction of Al. Cu 2S r 2Cu + + S2-

F
36. Fluorspar (CaF2 ) improve the electrical conductivity during 2Cu 2O r 4 Cu + + 2O2-
electrolytic reduction of alumina.
S2- + 2O2- ¾® SO2 + 6e-

or
ur
37. Al itself is a very strong reducing agent.
6Cu + + 6e- ¾® 6Cu ; Ecell o
= 0.52

f
38. In thermite welding, Al acts as a reducing agent Here, copper sulphide is reduced to copper metal. Solidified
2Al + Fe2O3 ¾® Al 2O3 + 2Fe + Heat
ks
copper has blistered appearance due to evolution of SO2 and
Yo
39. The actual representation of CuSO4 × 5H2O (blue vitriol) is thus obtained copper is known as blister copper.
oo
[Cu(H2O)4 ]SO4 × H2O and it has covalent, ionic and coordinate
Other compounds which give Cu are
covalent bonds.
eB

1100 °C
40. Ca 2+ end up in CaSiO3 (slag). (i) CuO as 4CuO ¾¾¾® 2Cu 2O + O2
41. Iron is rendered passive by concentrated HNO3 due to formation D
2Cu 2O + Cu 2S ¾® 6Cu + SO2
of a thick protective layer of Fe3O4.
r

720 °C 1
(ii) CuSO4 as CuSO4 ¾¾® CuO + SO2 +
ou

O2
ad

42. CuFeS2 (copper pyrite) is converted into copper into following 2


steps: D
Y

4CuO ¾® 2Cu 2O + O2
Step I Crushing (grinding ) followed by concentration by
froth-floatation process. D
2Cu 2O + Cu 2S ¾® 6Cu + SO2
nd

Step II Roasting of ore in the presence of SiO2 which removes iron


Re

While CuFeS2 will not give Cu on heating. The heating in the


as slag (FeSiO3). presence of O2 gives Cu 2S and FeS with the evolution of SO 2.
Fi

2CuFeS2 + O2 ¾® Cu 2S + 2FeS + SO2


45. Al has greater affinity for oxygen, hence oxide is not reduced by
2FeS + 3O2 ¾® 2SO2 + 2FeO carbon. MgO and CaO (formed in the calcination from
FeO + SiO2 ¾® FeSiO3 (Slag) carbonates) are stable species and not reduced by carbon.
Step III Self-reduction in Bessemer converter
1300 °C
2Cu 2S + 3O2 ¾® 2Cu 2O + 2SO2 During Smelting SnO2 + C ¾¾¾® Sn + CO
2Cu 2O + Cu 2S ¾® 6Cu + SO2 D 4Fe + 3 CO
2Fe2O3 + 3 C ¾® 2
Copper obtained is blister copper (98% pure).
Step IV Refining of blister copper is done by electrolysis 46. The important ore of tin is cassiterite (SnO2). Tin is extracted
from cassiterite ore by carbon reduction method in a blast furnace.
Impure copper—Anode
SnO2 + 2C ¾ ¾® Sn + 2CO
Pure copper— Cathode
At anode : Cu ¾® Cu 2+ + 2e- The product often contain traces of iron which is removed by
2+ -
blowing air through the melt to oxidise to FeO which then floats
At cathode : Cu + 2e ¾® Cu to the surface.
Carbon-reduction method is not used. Thus, (d) is incorrect. 2Fe + O2 ¾
¾® 2FeO
Extraction of Metals 271

47. Addition of manganese to iron improve hardness of steel as well 57. Extraction Metals
as remove oxygen and sulphur. methods extracted
A. Self reduction r. Copper, (P) Lead
48. Magnesium and aluminium are both highly electropositive,
more electropositive than water cannot be obtained by B. Carbon reduction p. Lead
electrolysis of aqueous solution of their salts. C. Complex formation q. Silver :
49. Alumina (Al 2O3 ) has very high melting point and it is poor and displacement Ag 2 S + NaCN
by metal ¾® Na[Ag(CN)2 ]
conductor of electricity. Both these factors posses difficulty in
electrolysis of molten alumina. Na[Ag(CN)2 ] + Zn
Cryolite, Na 3AlF6, when mixed with alumina, lowers melting ¾® Na 2 [Zn(CN)4 ]
point as well as improve electrical conductivity, hence helps in + Ag
electrolysis of Al 2O3. D. Decomposition of s. Boron :

w
iodide D 3
50. Al(OH)3 is amphoteric BI 3 ¾® B + I2
2
Al(OH)3 + 3HCl ¾® AlCl 3 + 3H2O

Flo
Base
Al(OH)3 + NaOH ¾® Na[Al(OH)4 ] 58. Column X Column Y Column Z
Acid
Invar Fe, Ni Watch spring

ree
High charge and small size of Al 3+ makes Al—O and O—H
bonds equally ionisable. Nichrome Co, Ni Heating element

F
51. 2CuFeS2 + O2 ¾® Cu 2S + 2FeS + SO2 ­ Stainless steel Fe, Cr, Ni Cutlery
2Cu 2S + 3O2 ¾® 2Cu 2O + 2SO2 ­

or
2FeS + 3O2 ¾® 2FeO + 2SO2 ­
ur 59. Column X (Metals) Column Y
(Ores)
52. FeO + SiO2 ¾® FeSiO3 (Slag)
f
A. Al q. Cryolite
ks
53. Cu 2S + 2Cu 2O ¾® 6Cu + SO2 B. Cu r. Malachite
Yo
oo
In Cu 2S, sulphur is S2- and in SO2, sulphur is in +4 state. C. Mg s. Carnalite
2-
Hence, S is acting as reducing agent. D. Zn p. Calamine
eB

54. Siderite =FeCO3, Malachite = CuCO3 × Cu(OH)2 60. A. Silver is extracted by amalgamation process
Bauxite = Al 2O3 × 2H2O2 consisting some Al (OH)3 Distillation
Ag + Hg ¾® Ag(Hg) ¾¾¾® Ag(s) + Hg(v ) ­
r
ou

Calamine = ZnCO3, Argentite = Ag2S Amalgam


ad

55. A. 3Cu + 8HNO3 ¾® 3Cu(NO3 )2 + 2NO + 4H2O B. Calcium is extracted by electrolysis of fused CaCl 2.
Y

Dil. C. Zinc is extracted by carbon reduction method


B. Cu + 4HNO3 ¾® Cu(NO3 )2 + 2NO2 + 2H2O ZnO + C ¾® Zn + CO
nd

Conc. D. Iron is extracted by both carbon reduction method and CO


Re

C. 4Zn + 10HNO3 ¾® 4Zn(NO3 )2 + N2O + 5H2O reduction methods


Fe2O3 + 3C ¾® 2Fe + 3CO
Fi

Dil.
D. Zn + 4HNO3 ¾® Zn(NO3 )2 + 2NO2 + 2H2O Fe2O3 + 3CO ¾® 2Fe + 3CO2
Conc. E. Copper is extracted by self reduction methods
O2
56. A. PbS ¾® PbO + SO2, roasting Cu 2S + O2 ¾® Cu 2O + SO2
B. CaCO3 ¾® CaO + CO2 ­; calcination Cu 2O + Cu 2S ¾® Cu + SO2
C. ZnS ¾® Zn, can be done by carbon reduction 61. H 2S Ag2S (black) is formed on the surface.
or self reduction
D
D. Cu 2S ¾® Cu, roasting followed by self reduction 62. ZnO + C ¾¾® Zn + CO = Smelting
sintering
272 Extraction of Metals

63. Hydration energy Energy required to break the crystal lattice 77. Anhydrous AlCl 3 is more soluble in diethyl ether as the oxygen
during dissolving process comes from hydration. If lattice atom of ether donate its lone-pair of electrons to the vacant
energy is very high and hydration energy is low, salt becomes orbital of Al in electron deficient AlCl 3. In case of hydrated
sparingly soluble.
AlCl 3, Al is not electron deficient as oxygen of water molecule
64. Zn Galvanisation involves coating of iron with zinc metal in has already donated its lone-pair of electrons to compensate
order to prevent if from rusting. electron deficiency of Al.
65. Sn Cassiterite is an ore of tin. Cl Cl
66. Al Aluminium reduces Fe2O3 to Fe. ½ ·· ½ ··
Cl — Al ¬¾ OH2 Cl — Al ¬¾ OEt 2
67. Lime, calcium phosphate In basic Bessemer process, the ·· ··
½ ½
Bessemer converter is lined with lime but in acid Bessemer Cl Cl
process, it is lined with silica. In basic Bessemer process, hydrated anhydrous

w
phosphorus is slagged off as calcium phosphate :
78. The reactions involved in the extraction of lead from galena
P4 + 5O2 ¾® P4O10
(PbS) by self reduction are
6CaO + P4O10 ¾® 2Ca 3 (PO4 )2
2PbS + 3O2 ¾® 2PbO + 2SO2

Flo
Thomas slag
PbS + 2PbO ¾® 3Pb + SO2
68. K [Ag(CN)2 ] : AgCl + 2KCN ¾® K[Ag(CN)2 ] + KCl PbS + 2O2 ¾® PbSO4 (side reaction)

ree
69. True: Cu + is unstable PbSO4 + PbS ¾® 2Pb + 2SO2
H+ In litharge (PbO), the oxidation state of Pb is +2
2Cu + ¾® Cu 2+ + Cu

F
79. The common photographic film is coated with AgBr and during
70. True Complex (Na[AgCl 2 ]) formation increases solubility of developing of photographic film, the unreacted AgBr is
otherwise sparingly soluble AgCl.

or
removed by Na 2S2O3 as
ur AgBr + 2Na 2S2O3 ¾® Na 3[Ag(S2O3 )2 ] + NaBr
71. True Iron is more electropositive than hydrogen
Fe + 2HCl ¾® FeCl 2 + H2 ­ 80.
f 4NaCN + Ag2S ¾® 2Na[Ag(CN)2 ] + Na 2S
ks
72. True : Solubility of silver halides decreases down in the group 2Na[Ag(CN)2 ] + Zn ¾® Na 2[Zn(CN)4 ] + 2Ag
Yo
81. 2Cu + H2O + CO2 + O2 ¾® Cu(OH)2 × CuCO3
oo
Solubility : AgF > AgCl > AgBr > AgI
Green
(basic copper carbonate)
73. In crystalline state, AlCl 3 has rock-salt like structure with
eB

coordination number of Al = 6. 2000 °C


82. C + O2 ¾¾¾® CO
74. Four, the complex has formula [Cu(H2O)4 ] SO4 × H2O 3CO + Fe2O3 ¾® 2Fe + 3CO2
r

CaCO3 ¾® CaO + CO2


ou

75. (a) 2AgBr + C6H4 (OH)2 ¾® 2Ag + 2HBr + C6 H4O2


ad

Hydroquinone Quinone
CaO + SiO2 ¾® CaSiO3
(developer) Slag
Y

H2O
AgBr + 2Na 2S2O3 ¾® Na 3[Ag(S2O3 )2 ] + NaBr 83. Al + NaOH ¾¾® NaAlO2+ 32 H2
(b) Na 2S2O3 + 2H+ ¾® 2Na + + H2SO3 + S ¯
nd
Re

Colloidal 84. 4KCN + Ag2S ¾® 2K[Ag(CN)2 ] + K2S


sulphur
Potassium
dicyanoargentate (I)
Fi

76. A1 is basic copper carbonate (Cu(OH)2 × CuCO3 ) while A2 is


Cu 2S. The confirmatory reactions are : 85. Due to formation of protective, inert layer of Al 2O3 on surface.
D 86. Sn + 2KOH + 4H2O ¾® K2[Sn(OH)6 ] + 2H2
(i) CuCO3 × Cu(OH)2 ¾® CuO + CO2 ­ + H2O
Black
HCl 87. Sea water (contain MgCl 2) + Ca(OH)2
(ii) CuCO3 × Cu(OH)2 ¾® CuCl 2 + CO2 ­ + H2O
KI
¾® Mg(OH)2 ¯ + CaCl 2 ( aq )
CuCl 2 ¾® Cu 2 I2 ¯ + KCl + I2
D (i) Mg(OH)2 + 2HCl ¾® MgCl 2 + 2H2O
Roasting Heat to
A2 ¾¾¾® Cu 2O + SO2 ­ ¾¾¾® MgCl 2 ( s )
Dryness
Cu 2S + Cu 2O ¾® Cu + SO2 ­
Fusion electrolysis
SO2 is a reducing gas that gives green colour with acidified (ii) MgCl 2 (s) ¾¾¾® Mg2+ + 2Cl - ¾¾¾® Mg
NaCl
K2Cr2O7 as (At cathode)
3SO2 + K2Cr2O7 + H2SO4 ¾® K2SO4 + Cr2 (SO4 )3
Green 88. 7Cu + 20HNO3 ¾® 7Cu(NO3 )2 + 4NO + 2NO2 + 10H2O
+ 4H2O
Extraction of Metals 273

89. 2CuFeS2 + O2 ¾® Cu 2S + 2FeS + SO2 However, if the metal is highly electropositive, e.g. Al, Mg
Copper pyrite etc., no reducing agent exist for reduction of their ions
2Cu 2S + 3O2 ¾® 2Cu 2O + 2SO2 (Al 3+ , Mg2+ ) and they are obtained by electrolytic
Roasting
2FeS + 3O2 ¾® 2FeO + 2SO2 reduction of their molten salt.
FeS + SiO2 ¾® FeSiO3 (ii) Metals like Ge is required in high purity, can be readily
Flux Slag melted and can easily crystallise out from the melt form.
2Cu 2O + Cu 2S ¾® 6Cu + SO2 Bessemerisation
94. AgBr is sensitive to visible light.
90. Ag2S + 2NaCN ¾® 2AgCN + Na 2S
hn 1
AgBr ¾¾® Ag + Br2
AgCN + NaCN ¾® Na[Ag(CN)2 ] 2
2Na[Ag(CN)2 ] + Zn ¾® Na 2[Zn(CN)4 ] + 2Ag A photographic plate coated with AgBr, when exposed to
light, gets blackened due to the above reaction.

w
91. (i) Carbon monoxide :
C + O2 ¾® CO 95. Al + NaOH(aq) ¾® NaAlO2 + H2
CO + Fe2O3 ¾® CO2 + Fe NaAlO2 + CO2 (aq) ¾® Na 2CO3 + Al(OH)3 ¯

Flo
(ii) 2PbS + 3O2 ¾® 2PbO + 2SO2 D
Al(OH)3 ¾® Al 2O3 + H2O
2PbO + PbS ¾® 3Pb + SO2
96. In the first step, galena is heated in presence of O2

ree
(iii) To improve electrical conductivity of melt.
(iv) A metal which is much more electropositive than Ag can (limited quantity) in a reverberatory furnace, where PbS is
only replace Ag + completely from [Ag(CN)2 ] - as partially oxidised to PbO :

F
3
Zn + 2[Ag(CN)2 ]- ¾® [Zn(CN)4 ]2- + 2Ag PbS + O ¾® PbO + SO2
2 2

or
(v) Chalcocite is a sulphide ore of copper, during roasting, SO2 In the second step, more PbS is added and heated in absence
ur
is liberated, which is not possible in calcination. of O2 , where the following self reduction takes place

f
92. 2Au + 3HNO3 + 11HCl ¾® 2HAuCl 4 + 6H2O + 3NOCl PbS + 2PbO ¾® 3Pb + SO2
ks
93. (i) If the metal is moderately electropositive, e.g. Fe, Sn, Pb or 97. SnO2 + 2C ¾® Sn + 2CO( g ), Carbon reduction method.
Yo
Cu, they can be obtained from their ore by chemical
oo
reduction methods.
eB

Download Chapter Test


http://tinyurl.com/yyev8wd6 or
r
ou
ad
Y
nd
Re
Fi
20
Qualitative Analysis

w
Objective Questions I (Only one correct option) 7. A solution of a metal ion when treated with KI gives a red
precipitate which dissolves in excess KI to give a colourless
1. An organic compound X showing the following solubility solution. Moreover, the solution of metal ion on treatment
profile is

Flo
(2019 Main, 8 April I)
with a solution of cobalt (II) thiocyanate gives rise to a deep
Water
Insoluble blue crystalline precipitate. The metal ion is (2007, 3M)
(a) Pb2+ (b) Hg2+ (c) Cu2+ (d) Co2+

ree
5% HCl
Insoluble
X 8. MgSO4 on reaction with NH4 OH and Na 2 HPO4 forms a
10% NaOH
Soluble

F
white crystalline precipitate. What is its formula? (2006)
10% NaHCO3
Insoluble (a) Mg(NH4 )PO4 (b) Mg3 (PO4 )2
(c) MgCl 2 × MgSO4 (d) MgSO4

or
(a) o -toluidine
(c) m-cresol
ur
(b) oleic acid
(d) benzamide 9. CuSO4 decolourises on addition of KCN, the product is

f
2. When metal ‘M ’ is treated with NaOH, a white gelatinous (a) [Cu(CN)4 ]2 - (2006, 3M)
ks
precipitate ‘X ’ is obtained, which is soluble in excess of (b) Cu2 + get reduced to form [Cu(CN)4 ]3 -
Yo
NaOH. Compound ‘X ’ when heated strongly gives an oxide
(c) Cu(CN)2
oo
which is used in chromatography as an adsorbent. The metal
(d) CuCN
‘M ’ is (2018 Main)
eB

(a) Zn (b) Ca (c) Al (d) Fe 10. A solution when diluted with H2 O and boiled, it gives a
white precipitate. On addition of excess NH4 Cl / NH4 OH,
3. In the following reaction sequence in aqueous solution, the
the volume of precipitate decreases leaving behind a white
species X, Y and Z, respectively, are (2016 Adv.)
gelatinous precipitate. Identify the precipitate which
r

Ag + Ag +
ou

dissolves in NH4 OH / NH4 Cl. (2006, 3M)


ad

With time
S2 O23 - ¾® X ¾® Y ¾¾® Z (a) Zn(OH)2 (b) Al(OH)3
(Clear solution) (White ppt.) (Black ppt.)
Y

(c) Mg(OH)2 (d) Ca(OH)2


(a) [Ag(S2O3 )2 ]3- , Ag2S2O3 , Ag2S
(b) [Ag(S2O3 )3 ]5- , Ag2SO3 , Ag2S
11. A metal nitrate reacts with KI to give a black precipitate
nd

which on addition of excess of KI convert into orange colour


Re

(c) [Ag(SO3 )2 ]3- , Ag2S2O3 , Ag solution. The cation of metal nitrate is (2005, 1M)
(d) [Ag(SO3 )3 ]3- , Ag2SO4 , Ag (a) Hg2+ (b) Bi 3+ (c) Sn 2+ (d) Pb2+
Fi

4. In Carius method of estimation of halogens 250 mg of an 12. (NH4 )2 Cr2 O7 on heating gives a gas which is also given by
organic compound gave 141 mg of AgBr. The percentage of (2004, 1M)
bromine in the compound is (atomic mass Ag = 108, Br = 80) (a) Heating NH4NO2 (b) Heating NH4NO3
(2015 Main) (c) Mg3N2 + H2O (d) Na(comp. ) + H2O2
(a) 24 (b) 36 (c) 48 (d) 60
13. A sodium salt of an unknown anion when treated with
5. Upon treatment with ammoniacal H 2S , the metal ion that MgCl 2 gives white precipitate only on boiling. The anion is
precipitates as a sulphide is (2013 Adv.) (2004, 1M)
(a) Fe (III) (b) Al (III) (a) SO2–
4 (b) HCO–3 (c) CO2-
3 (d) NO-3
(c) Mg (II) (d) Zn(II)
14. [ X ] + H2 SO4 ® [Y ] a colourless gas with irritating smell
6. Passing H2 S gas into a mixture of Mn 2+ , Ni 2+ , Cu 2+ and
[Y ] + K 2 Cr2 O7 + H2 SO4 ¾® green solution
Hg 2+ ions in an acidified aqueous solution precipitates
[ X ] and [Y ] are (2003, 1M)
(a) CuS and HgS (b) MnS and CuS (2011)
(a) SO23- , SO2 (b) Cl - , HCl
(c) MnS and NiS (d) NiS and HgS
(c) S2- , H2S (d) CO23- , CO2
Qualitative Analysis 275

15. A gas X is passed through water to form a saturated solution. (a) Mn 2+ shows the characteristic green colour in the flame
The aqueous solution on treatment with silver nitrate gives a test
white precipitate. The saturated aqueous solution also (b) Only Cu 2+ shows the formation of precipitate by passing
dissolves magnesium ribbon with evolution of a colourless H2 S in acidic medium
gas Y. Identify X and Y. (2002, 3M) (c) Only Mn 2+ shows the formation of precipitate by passing
(a) X = CO2, Y = Cl 2 (b) X = Cl 2 , Y = CO2 H2 S in faintly basic medium
(c) X = Cl 2 , Y = H2 (d) X = H2 , Y = Cl 2
(d) Cu 2+ / Cu has higher reduction potential than Mn 2+ / Mn
16. An aqueous solution of a substance gives a white precipitate (measured under similar conditions)
on treatment with dilute hydrochloric acid, which dissolves
25. The reagent(s) that can selectively precipitate S2 - from a
on heating. When hydrogen sulphide is passed through the
hot acidic solution, a black precipitate is obtained. The mixture of S2 - and SO24 - in aqueous solution is (are)
(2016 Adv.)

w
substance is a (2000, 1M)
+ (a) CuCl 2 (b) BaCl 2
(a) Hg2+
2 salt (b) Cr 2+
salt (c) Ag salt (d) Pb 2+
salt
(c) Pb(OOCCH3 )2 (d) Na 2[ Fe(CN)5 NO]
17. In nitroprusside ion the iron and NO exist as Fe (II) and NO+ 26. The pair(s) of ions where both the ions are precipitated upon

Flo
rather than Fe (III) and NO. These forms can be
passing H2S gas in presence of dilute HCl, is (are)
differentiated by (1998, 2M)
(2015 Adv.)
(a) estimating the concentration of iron

ree
(a) Ba 2+ , Zn 2+ (b) Bi 3+ , Fe3+
(b) measuring the concentration of CN
(c) measuring the solid state magnetic moment (c) Cu2+ , Pb2+ (d) Hg2+ , Bi 3+

F
(d) thermally decomposing the compound 27. For the given aqueous reaction which of the statement(s) is
18. An aqueous solution FeSO4 × Al 2 (SO4 )3 and chrome alum is Dilute H2 SO4
(are) true? Excess KI + K 3 [Fe(CN)6 ] ¾¾¾¾¾®

or
ur
heated with excess of Na 2 O2 and filtered. The materials
Brownish-yellow solution
obtained are (1996, 1M)

f
(a) a colourless filtrate and a green residue ¯ ZnSO4
ks
(b) a yellow filtrate and a green residue (White precipitate + Brownish-yellow filtrate)
¯ Na 2 S2 O3
Yo
(c) a yellow filtrate and a brown residue
oo
(d) a green filtrate and brown residue Colourless solution (2012)
19. The brown ring complex compound is formulated as (a) The first reaction is a redox reaction
eB

+ (b) White precipitate is Zn 3[ Fe(CN)6 ]2


[Fe(H2 O)5 (NO) ] SO4 . The oxidation state of iron is
(c) Addition of filtrate to starch solution gives blue colour
(1987, 1M)
(a) 1 (b) 2 (c) 3 (d) 0 (d) White precipitate is soluble in NaOH solution
r

28. A solution of colourless salt H on boiling with excess NaOH


ou

20. Which one amongst the following pairs of ions cannot be


ad

separated by H2 S in dilute HCl? (1986, 1M) produces a non-flammable gas. The gas evolution ceases
(a) Bi 3+ , Sn 4+ (b) Al 3+ , Hg2+ after sometime. Upon addition of Zn dust to the same
Y

solution, the gas evolution restarts. The colourless salt(s) H


(c) Zn 2+ , Cu2+ (d) Ni 2+ , Cu2+
is (are) (2008, 4M)
nd
Re

21. The compound insoluble in acetic acid is (1986, 1M) (a) NH4NO3 (b) NH4NO2
(a) calcium oxide (b) calcium carbonate (c) NH4Cl (d) (NH4 )2 SO4
Fi

(c) calcium oxalate (d) calcium hydroxide


29. Which of the following statement(s) is(are) correct when a
22. The ion that cannot be precipitated by both HCl and H2 S is mixture of NaCl and K 2 Cr2 O7 is gently warmed with
(a) Pb2– (b) Cu+ (1982, 1M) conc. H2 SO4 ? (1998, 2M)
(c) Ag+ (d) Sn 2+ (a) A deep red vapour is evolved
(b) The vapour when passed into NaOH solution gives a
23. For the equilibrium, 2H 2O - H3O+ + OH - , the value yellow solution of Na 2CrO4
of DGº at 298 K is approximately (2019 Main 11 Jan II) (c) Chlorine gas is evolved
(a) - 80 kJ mol - 1 (b) 100 kJ mol - 1 (d) Chromyl chloride is formed
(c) 80 kJ mol - 1 (d) - 100 kJ mol - 1 30. Which of the following statement(s) is (are) correct with
reference to the ferrous and ferric ions ? (1998, 2M)
Objective Questions II (a) Fe3+ gives brown colour with potassium ferricyanide
(One or more than one correct option) (b) Fe2+ gives blue precipitate with potassium ferricyanide
(c) Fe3+ gives red colour with potassium thiocyanate
24. The correct option(s) to distinguish nitrate salts to Mn 2+ and
(d) Fe2+ gives brown colour with ammonium thiocyanate
Cu 2+ taken separately is (are) (2018 Adv.)
276 Qualitative Analysis

31. The reagents, NH4 Cl and aqueous NH3 will precipitate medium. However, it gave a precipitate (R) with H2 S in an
(1991, 1M) ammoniacal medium. The precipitate R gave a coloured solution
(a) Ca 2+ (b) Al 3+ (c) Bi 3+ (d) Mg2+ (S ), when treated with H2 O2 in an aqueous NaOH medium.
2+
(e) Zn 36. The precipitate P contains (2013 Adv.)
(a) Pb2+ (b) Hg2+
2 (c) Ag+ (d) Hg2+
Assertion and Reason
37. The coloured solution S contains
Read the following questions and answer as per the direction (a) Fe2 (SO4 )3 (b) CuSO4 (c) ZnSO4 (d) Na 2CrO4
given below :
(a) Statement I is correct Statement II is correct Statement II is Passage 3
a correct; explanation of Statement I When a metal rod M is dipped into an aqueous colourless
(b) Statement I is correct; Statement II is correct Statement II is concentrated solution of compound N , the solution turns light blue.

w
not the correct explanation of Statement I. Addition of aqueous NaCl to the blue solution gives a white
(c) Statement I is correct; Statement II is incorrect. precipitate O. Addition of aqueous NH 3 dissolves O and gives an
(d) Statement I is incorrect; Statement II is correct. intense blue solution. (2011)

Flo
32. Statement I Sulphate is estimated as BaSO4 , not as MgSO4 . 38. The metal rod M is
(a) Fe (b) Cu (c) Ni (d) Co
Statement II Ionic radius of Mg 2+ is smaller than that
39. The compound N is

ree
of Ba 2+ . (1998, 2M) (a) AgNO3 (b) Zn(NO3 )2
33. Statement I A very dilute acidic solution of Cd 2+ and (c) Al(NO3 )3 (d) Pb(NO3 )2

F
Ni 2+ gives yellow precipitate of CdS on passing H2 S. 40. The final solution contains
(a) [Pb(NH3 )4 ]2+ and [CoCl 4 ]2-
Statement II Solubility product of CdS is more than that

or
of NiS.
ur (1989, 2M) (b) [Al(NH3 )4 ]3+ and [Cu(NH3 )4 ]2+

f
(c) [Ag(NH3 )2 ]+ and [Cu(NH3 )4 ]2+
Passage Based Questions (d) [Ag(NH3 )2 ]+ and [Ni(NH3 )6 ]2+
ks
Passage 1
Yo
Passage 4
oo
An aqueous solution of metal ion M 1 reacts separately with
reagents Q and R in excess to give tetrahedral and square planar p-amino-N, N-dimethylaniline is added to a strongly acidic
eB

complexes, respectively. An aqueous solution of another metal ion solution of X. The resulting solution is treated with a few drops of
M 2 always forms tetrahedral complexes with these reagents. aqueous solution of Y to yield blue colouration due to the formation
Aqueous solution of M 2 on reaction with reagent S gives white of methylene blue. Treatment of the aqueous solution of Y with the
precipitate which dissolves in excess of S. The reactions are reagent potassium hexacyanoferrate (II) leads to the formation of
r
ou

summarised in the scheme given below an intense blue precipitate. The precipitate dissolves on excess
ad

Q R addition of the reagent. Similarly, treatment of the solution of Y


Tetrahedral M1 excess Square planar
Y

excess with the solution of potassium hexacyanoferrate (III) leads to a


Q R
brown colouration due to the formation of Z.
Tetrahedral M2 excess Tetrahedral
excess 41. The compound X, is (2009, 1M)
nd
Re

(a) NaNO3 (b) NaCl


S, stoichiometric amount
(c) Na 2SO4 (d) Na 2S
Fi

S
White precipitate excess
Precipitate 42. The compound Y, is (2009, 1M)
dissolves (a) MgCl 2 (b) FeCl 2
34. M1 , Q and R, respectively are (c) FeCl 3 (d) ZnCl 2
(a) Zn 2+ , KCN and HCl (b) Ni 2+ , HCl and KCN 43. The compound Z, is (2009, 1M)
2+ 2+ (a) Mg2[Fe(CN)6 ] (b) Fe[Fe(CN)6 ]
(c) Cd , KCN and HCl (d) Co , HCl and KCN
(c) Fe4[Fe(CN)6 ]3 (d) K 2 Zn 3[Fe(CN)6 ]2
35. Reagent S is
(a) K 4[Fe(CN)6 ] (b) Na 2HPO4
(c) K 2CrO4 (d) KOH
Fill in the Blanks
44. The formula of the deep red liquid formed on warming
Passage 2 dichromate with KCl in concentrated sulphuric acid is...
An aqueous solution of a mixture of two inorganic salts,when (1993, 1M)
treated with dilute HCl, gave a precipitate (P) and filtrate (Q). The 45. If metal ions of group III are precipitated by NH4 Cl and
precipitate (P) was found to dissolve in hot water. The filtrate (Q) NH4 OH without prior oxidation by conc. HNO3 . …… is not
remained unchanged, when treated with H2 S in a dilute mineral acid completely precipitated. (1984, 1M)
Qualitative Analysis 277

True/False Mixture on performing flame test gives lilac colour. Identify


the compounds (A), (B), (C) and (D). (2003)
46. From the solution containing copper (+2) and zinc (+2) ions,
copper can be selectively precipitated using sodium 53. When a white crystalline compound X is heated with
sulphide. (1987, 1M) K 2 Cr2 O7 and concentrated H2 SO4 , a reddish brown gas A is
evolved. On passing A into caustic soda solution, a yellow
47. Addition of ammonium chloride to a solution containing
coloured solution B is obtained. Neutralising the solution of
ferric and magnesium ions is essential for selective
B with acetic acid and on subsequent addition of lead acetate
precipitation of ferric hydroxide by aqueous ammonia.
(1985, 1/2M)
a yellow precipitate C is obtained.
When X is heated with NaOH solution, colourless gas is
Integer Answer Type Question evolved and on passing this gas into K 2 HgI4 solution, a
reddish brown precipitate D is formed. Identify A, B, C, D and

w
48. Among PbS, CuS, HgS, MnS, Ag 2 S, NiS, CoS, Bi 2 S3 and
X. Write the equations of reactions involved. (2002)
SnS2 the total number of black coloured sulphides is
(2014 Adv.) 54. A white substance A reacts with dilute H2 SO4 to produce a
colourless gas B and a colourless solution C. The reaction

Flo
Subjective Questions between B and acidified K 2 Cr2 O7 solution produces a green
solution and a slightly coloured precipitate D. The substance
moist air Zn
¬¾¾¾ ¾¾®

ree
49. B MCl 4 A D burns in air to produce a gas E which reacts with B to yield
(White fumes M = ( Transition (Purple
D and a colourless liquid. Anhydrous copper sulphate is
having smell) element colourless) colour)
turned blue on addition of this colourless liquid. Addition of

F
Identify the metal M and hence MCl 4 . Explain the difference aqueous NH3 or NaOH to C produces first a precipitate,
in colours of MCl 4 and A. (2005, 4M) which dissolves in the excess of the respective reagent to

or
SCN- (excess)
50. Fe3+ ¾¾¾¾¾® Blood red (A)
ur produce a clear solution in each case. Identify A, B, C, D and
E. Write the equations of the reactions involved. (2001, 10M)

f
F - (excess) 55. Write the chemical reactions associated with the ‘borax
¾¾¾¾® Colourless ( B)
ks
bead test’ of cobalt (II) oxide. (2000, 3M)
Yo
Identify A and B.
56. An aqueous blue coloured solution of a transition metal
oo
(i) Write IUPAC name of A and B. sulphate reacts with H2 S in acidic medium to give a black
(ii) Find out spin only magnetic moment of B. (2005) precipitate A, which is insoluble in warm aqueous solution
eB

51. A1 and A2 are two ores of metal M. A1 on calcination gives of KOH. The blue solution on treatment with KI in weakly
black precipitate, CO2 and water. acidic medium, turns yellow and produces a white
precipitate B. Identify the transition metal ion. Write the
r

Calcination
¾¾¾¾® Black solid + CO2 + OH2 chemical reactions involved in the formation of A and B.
ou
ad

A1 ¾¾ (2000, 4M)
dil. HCl 57. Write the chemical reactions associated with the ‘brown
Y

¾¾¾¾® I2 + ppt
KI ring test’. (2000, 1M)
Roasting 58. An aqueous solution containing one mole of HgI2 and two
A2 ¾¾¾® Metal + Gas
nd
Re

moles of NaI is orange in colour. On addition of excess NaI


K 2Cr2O7 + H2SO4 the solution becomes colourless. The orange colour
Fi

reappears on subsequent addition of NaOCl. Explain with


Green colour
equations. (1999, 3M)
Identify A1 and A2 . (2004)
59. During the qualitative analysis of a mixture containing Cu 2+
52. A salt mixture consists of a yellow solid (A) and a colourless and Zn 2+ ions, H2 S gas is passed through an acidified
solid (B). The aqueous solution of the mixture
solution containing these ions in order to test Cu 2+ alone.
(i) On passing H2S, we get a black precipitate of (C), which
dissolves only in aqua-regia. On extraction and reaction Explain briefly. (1998, 2M)
with SnCl2 a greyish white precipitate is obtained. 60. Aluminium sulphide gives a foul odour when it becomes
(ii) On treatment with ammonium hydroxide a reddish brown damp. Write a balanced chemical equation for the reaction.
precipitate (D) is obtained. (1997)
The sodium extract of the solution gives the following tests: 61. A soluble compound of a poisonous element M, when
(i) On reaction with AgNO3 it gives a yellow precipitate which heated with Zn/H2SO4 gives a colourless and extremely
is insoluble in NH3. poisonous gaseous compound N, which on passing through
(ii) On shaking with FeCl3 and CCl4 a violet colouration in CCl4 a heated tube gives a silvery mirror of element M. Identify M
layer is obtained. and N. (1997)
278 Qualitative Analysis

62. A colourless inorganic salt (A) decomposes completely at solution of K 2 HgI4 . The aqueous solution of the mixture on
about 250° C to give only two products (B) and (C), leaving treatment with BaCl 2 gives a white precipitate which is
no residue. The oxide (C) is a liquid at room temperature and sparingly soluble in conc. HCl.
neutral to moist litmus paper, while the gas (B) is a neutral On heating the mixture with K 2 Cr2 O7 and conc. H2 SO4 , red
oxide. vapours A are produced. The aqueous solution of the
White phosphorus burns in excess of (B) to produce a strong mixture gives a deep blue colouration B with potassium
white dehydrating agent. Write balanced equations for the ferricyanide solution. Identify the radicals in the given
reactions involved in the above process. (1996, 3M) mixture and write the balanced equations for the formation
63. Gradual addition of KI solution of Bi(NO3 )3 solution of A and B. (1991, 4M)

initially produces a dark brown precipitate which dissolves 71. Write the balanced chemical equations for the following
in excess of KI to give a clear yellow solution. Write (i) Silver chloride is treated with aqueous sodium cyanide and

w
chemical equations for the above reactions. (1996, 2M) the product thus formed is allowed to react with zinc in
64. A scarlet compound A is treated with conc. HNO3 to give a alkaline medium.
(ii) Cobalt (II) solution reacts with KNO2 in acetic acid
chocolate brown precipitate B. The precipitate is filtered
medium. (1989, 2M)

Flo
and the filtrate is neutralised with NaOH. Addition of KI to
the resulting solution gives a yellow ppt C. The brown ppt B 72. Give reasons for, “The colour of mercurous chloride,
on warming with conc. HNO3 in the presence of Mn(NO3 )2 Hg 2 Cl 2 , changes from white to black when treated with

ree
produces a pink coloured solution due to the formation of D. ammonia.” (1988, 1M)
Identify A, B, C and D. Write the reaction sequence. 73. A mixture of two salts was treated as follows :

F
(1995, 4M)
(i) The mixture was heated with maganese dioxide and
65. An orange solid A on heating gave a green residue B, a concentrated sulphuric acid, when yellowish green gas was

or
liberated.
ur
colourless gas C and water vapour. The dry gas C on passing
over heated Mg gave a white solid D. D on reaction with (ii) The mixture on heating with sodium hydroxide solution

f
water gave a gas E which formed dense white fumes with gave a gas which turned red litmus blue.
HCl. Identify A to E and give the reaction involved. (iii) Its solution in water gave blue precipitate with potassium
ks
(1993, 3M) ferricyanide and red colouration with ammonium
Yo
thiocyanate.
oo
66. The acidic aqueous solution of ferrous ion forms a brown
complex in the presence of NO-3 , by the following two (iv) The mixture was boiled with potassium hydroxide and the
eB

liberated gas was bubbled through an alkaline solution of


steps : K2HgI4 to give brown precipitate.
[Fe(H2 O)6 ]2+ + NO–3 + H+ ¾® K + [Fe(H2 O)6 ]3+ Identify the two salts. Give ionic equations for reactions
involved in the tests (i), (ii) and (iii). (1987, 5M)
+ K ¾® ...... + H2 O
r

2+
[Fe(H2 O)6 ]
ou

74. Write balanced equation for the following “potassium


ad

Complete and balance the equation. (1993, 2M)


permanganate is reacted with warm solution of oxalic acid
Y

67. A light bluish green crystalline compound responds to the in the presence of sulphuric acid.” (1987, 1M)
following tests
75. Mention the products formed in the following:
(i) Its aqueous solution gives a brown precipitate or
nd

(i) Zinc oxide is treated with excess of sodium hydroxide


Re

colouration with K2[HgI4 ].


solution.
(ii) Its aqueous solution gives a blue colour with K3[Fe(CN)6 ].
Fi

(ii) Iodine is added to a solution of stannous chloride.


(iii) Its solution in hydrochloric acid gives a white precipitate
(iii) Sulphur dioxide gas, water vapour and air are passed over
with BaCl 2.
heated sodium chloride. (1986, 3M)
Identify the ions present and suggest the formula of the
compound. (1992, 4M)
76. What happen when
(i) hydrogen sulphide is bubbled through an aqueous solution
68. In the following reaction, identify the compounds/reaction of sulphur dioxide.
conditions represented by the alphabets A and B.
(ii) aqueous ammonia is added dropwise to a solution of copper
Heat in B
PbS ¾¾¾® A + PbS ¾® Pb + SO2 sulphate till it is in excess.
air (1991, 1M) (iii) tin is treated with concentrated nitric acid.
69. Give reason in one or two sentences for the following (iv) CrCl 3 solution is treated with sodium hydroxide and then
“The hydroxides of aluminium and iron are insoluble in with hydrogen peroxide.
water. However, NaOH is used to separate one from other. (v) Pb3O4 is treated with nitric acid. (1985, 5M)
(1991, 2M)
77. Write down the balanced equations for the reactions, when,
70. The gas liberated, on heating a mixture of two salts with ‘a mixture of potassium chlorate, oxalic acid and sulphuric
NaOH, gives a reddish brown precipitate with an alkaline acid is heated. (1985, 1M)
Qualitative Analysis 279

78. When 16.8 g of white solid, X were heated, 4.4 g of acid gas (vi) Residue D from step (iii) is heated on charcoal in a reducing
A, that turned lime water milky was driven off together with flame. It gives a magnetic substance.
1.8 g of a gas B which condensed to a colourless liquid. Name the compound A, B, C, D and E. (1980, 4M)

The solid that remained, Y, dissolved in water to give an 80. Explain the following in not more than two sentences.
alkaline solution, which with excess barium chloride A solution of FeCl 3 in water gives a brown precipitate on
solution gave a white precipitate Z. The precipitate standing. (1980, 1M)
effervesces with acid giving of carbon dioxide. Identify A, B 81. The precipitation of second group sulphides in qualitative
and Y and write down the equation for the thermal analysis is carried out with hydrogen sulphide in the presence
decomposition of X. (1984, 4M) of hydrochloric acid but not in nitric acid. Explain. (1979, 2M)
79. Compound A is a light green crystalline solid. It gives the 82. A white amorphous powder A on heating yields a colourless,
non-combustible gas B and a solid C. The later compound

w
following tests
(i) It dissolves in dilute sulphuric acid. No gas is produced. assumes a yellow colour on heating and changes to white on
(ii) A drop of KMnO4 is added to the above solution. The pink cooling. C dissolves in dilute hydrochloric acid and the
colour disappears. resulting solution gives a white precipitate with K 4 Fe(CN)6

Flo
solution. A dissolves in dil. HCl with the evolution of gas,
(iii) Compound A is heated strongly. Gases B and C, with
which is identical in all respect with B.
pungent smell, come out. A brown residue D is left behind.

ree
(iv) The gas mixture (B and C) is passed into a dichromate The gas B turns lime water milky, but the milkiness
solution. The solution turns green. disappears with the continuous passage of gas. The solution
of A as obtained above, gives a white ppt E on addition of

F
(v) The green solution from step (iv) gives a white precipitate E
with a solution of barium nitrate. NaOH solution, which dissolves on further addition of base.
Identify the compounds A, B, C, D and E. (1979, 4M)

or
ur
Answers
f
ks
1. (c) 2. (c) 3. (a) 4. (a) 29. (a, b, c, d) 30. (b, c) 31. (b, c) 32. (b)
5. (d) 6. (a) 7. (b) 8. (a)
Yo
33. (c) 34. (b) 35. (d) 36. (d)
(b) (a) (b) (a)
oo
9. 10. 11. 12.
37. (d) 38. (b) 39. (a) 40. (c)
13. (b) 14. (a) 15. (c) 16. (d)
(c) (c) (a) (a) 41. (d) 42. (c) 43. (b) 44. CrO2Cl 2
eB

17. 18. 19. 20.


21. (c) 22. (c) 23. (c) 24. (b,d) 45. Fe3+ 46. True 47. True 48. (6 or 7)
25. (a) 26. (c, d) 27. (a, c, d) 28. (a, b) 50. (5.92 BM)
r
ou

Hints & Solutions


ad
Y

1. m-cresol is the organic compound that shows the following O


solubility profile. ½½
Benzamide (C6H5C NH2) is insoluble in 5% HCl, 10% NaOH
nd
Re

Water
Insoluble and 10% NaHCO3 due to the presence ¾CONH2 group.
CH3
o-toluidine is soluble in 5%. HCl due to presence of basic group
Fi

5% HCl
Insoluble ( ¾ NH2 ) attached to ring.
10% NaOH
Soluble 2. Among the given metals Al forms white gelatinous ppt. with
OH
m-cresol NaOH.
10% NaHCO3
Insoluble Hence, the probable metal can be Al. This ppt. is dissolved in
excess of NaOH due to the formation of sodium metal
m-cresol on reaction with 10% NaOH forms 3-methyl
Aluminate. Both the reactions are shown below.
sodiumphenoxide ion.
Al 3+ ¾NaOH
¾¾® Al(OH)3 ¾Excess
¾ ¾of¾¾
NaOH
® NaAlO2
OH O–Na+ White gelatinous Sodium
ppt. (X) of metaaluminate)
aluminium soluble
10% NaOH hydroxide

CH3 CH3 Aluminium hydroxide on strong heating gives alumina (Al 2O3 )
which is used as an adsorbent in chromatography. This reaction
It does not react with H2O, 5% HCl and 10% NaHCO3. can be seen as :
Oleic acid (C18 H34O2 ) is soluble in 10% NaOH and 10% D
2Al(OH)3 ¾® Al 2O3 + 3H2O
NaHCO3 due to the presence of COOH group.
280 Qualitative Analysis

Thus, metal M is Al. I-


11. Bi 3+ + 3I- ¾® BiI3 ¯ ¾¾®
excess
[BiI4 ]-
Ca, being below sodium in electrochemical reactivity series, Black Orange
cannot displaces Na from its aqueous solution. solution
Zn reacts with NaOH to form sodium zincate which is a soluble
compound.
12. Both (NH4 )2 Cr2O7 and NH4NO2 on heating gives nitrogen gas.
Fe reacts with sodium hydroxide to form tetrahydroferrate (II) 13. A sodium salt of an unknown anion when treated with MgCl
sodium which is again a soluble complex. gives white precipitate (MgCO3 ) only on boiling. Hence, the
3. 2S2O32- +
+ Ag ¾® [ Ag(S2O3 )2 ] 3- action must be HCO-3 ion.
(Clear solution) D
MgCl 2 + 2NaHCO3 ¾® MgCO3 + 2NaCl + H2O + CO2
3- +
[ Ag(S2O3 )2 ] + 3Ag ¾® 2Ag2S2O3
(White ppt.) 14. SO23- + H2SO4 ¾® SO2 ­ + H2O + SO42-

w
X Y
Ag2S2O3 + H2O ¾® Ag2S + H2SO4
(Black ppt.) SO2 is a colourless gas with irritating odour.
4. Given, Weight of organic compound = 250 mg 3SO2 + K2Cr2O7 + H2SO4 ¾® K2SO4 + Cr2 (SO4 )3 + H2O
Weight of AgBr = 141mg Y Green

Flo
solution
\ According to formula of % of bromine by Carius method 15. Cl 2 + H2O ¾® HCl + HOCl

ree
Atomic weight of Br X
% of Br =
Molecular weight of AgBr HCl + AgNO3 ¾® AgCl ¯ (white) + HNO3

F
´
Weight of AgBr
´ 100 2HCl + Mg ¾® MgCl 2 + H2 (g ) ­
Weight of organic bromide
16. PbCl 2 is soluble in hot water and PbS (black) is formed on

or
80 141 1128000
\ % of Br = ´
188 250
´ 100 =
47000
= 24%
ur passing H2S(g ) through acidic solution.

f
17. Fe(II) and Fe(III) will have different values of magnetic moment
5. PLAN Ksp (ZnS) is very high and Zn 2+ is precipitated as ZnS by high due to different number of unpaired electrons in their d-orbitals.
ks
concentration of S 2- formed when H 2S is passed in ammoniacal
solution. 18. Yellow filtrate contains CrO5 and brown residue contain Fe2O3.
Yo
H 2S a Zn + + S 2- (I)
oo

H + OH - a H 2O (II)
+ 19. The total positive valency is +2 (because the only anion is
SO2-
4 ) . Therefore, oxidation state of Fe must be +1.
eB

Reaction (I) is favoured in forward side if H+ is removed


immediately by OH- (NH4OH) . 20. Both Bi 3+ and Sn 4+ belongs to same analytical group II.
2+ 2-
Zn +S ¾® ZnS ¯ 21. CaC2O4 is insoluble in acetic acid. This distinguishes Ca 2+ from
r

White ppt
Ba 2+ ion.
ou
ad

Fe3+ and Al 3+ are precipitated as hydroxide.


6. In acidic medium, H2S is very feebly ionised giving very small 22. Ag+ is precipitated by HCl only while all others are precipitated
Y

concentration of sulphide ion for precipitation. Therefore, the by passing H2S in the presence of HCl.
most insoluble salts CuS and HgS are precipitated only. 23. We know that,
nd

DG° = - 2.303 RT log K


Re

7. Hg2+ + 2I- ¾® HgI2 (red)


Also, given equilibrium is
HgI2 + 2KI ¾® K2[HgI4 ]
Fi


Soluble 2H2O - H3O+ + O H
Hg 2+
+ Co(SCN)2 ¾® Co[Hg(SCN)4 ] [H+ ][OH- ] = 10-14 or K = 10- 14
Blue
\ DG ° = - 2.303 ´ 8.314 JK–1mol-1 ´ 298K ´ log10-14
8. MgSO4 + NH4OH + Na 2HPO4 ¾® Mg(NH4 )PO4 ¯
White = 79881.8J mol - 1 = 79.8kJ » 80 kJ mol - 1
+ Na 2 SO4 + H2 O
24. Statement wise explanation is
9. CuSO4+ 2KCN ¾® Cu(CN)2 + K2SO4 Statement (a) Mn 2+ produces yellow-green colour in flame
CN test while Cu 2+ produces bluish-green colour in flame test.
2Cu(CN)2 ¾® 2Cu(CN) + ½ (Cyanogen) Thus, due to the presence of green colour in both the cases,
Unstable CN flame test is not the suitable method to distinguish between
CuCN + 3KCN ¾® 3K+ + [Cu(CN)4 ]3- nitrate salts ofCu 2+ and Mn 2+. Hence this statement is wrong.
Statement (b) Cu 2+ belong to group II of cationic or basic
10. Zn 2+ + 2H2O ¾® Zn(OH)2 ¯ + 2H+ radicals. It gives black ppt. of CuS if H2S is passed through it in
White the presence of acid (e.g HCl). Mn 2+ does not show this
Qualitative Analysis 281

property hence this can be considered as a suitable method to 28. NH4NO3 + NaOH ¾® NaNO3 + NH3 + H2O
distinguish between Mn 2+ and Cu 2+.
NH4NO2 + NaOH ¾® NaNO2 + NH3 + H2O
Hence, this statement is correct.
Statement (c) In faintly basic medium when H2S is passed both 29. When mixture of NaCl is heated with K2Cr2O7 in concentrated
Cu 2+ and Mn 2+ forms precipitates. Thus, it is not suitable H2SO4, red vapour of chromyl chloride (CrO2Cl 2 ) is produced.
method to distinguish between them. Vapours of chromyl chloride when passed through NaOH,
Hence, this statement is incorrect solution turns yellow due to formation of Na 2CrO4. Some
Statement (d) The standard reduction potential of Cu 2+/Cu is chlorine gas is also evolved owing to the following side
+0.34 V while that of Mn 2+/Mn is –1.18V. This can be used to reaction :
distinguish between Cu 2+ and Mn 2+. In general less 6Cl - + Cr2O72- + 14H+ ¾® 3Cl 2 (g ) ­ + 2Cr 3+ + 7H2O
electropositive metals have higher SRP.
30. The blue precipitate of Fe2+ ion with potassium ferricyanide is

w
Hence, this statement is correct.
due to formation of Turnbull’s blue KFe[Fe(CN)6 ].
25. S2– + CuCl 2 ¾® CuS ¯ (black ppt.)
Fe2+ + K 3 [Fe(CN)6 ] ¾® KFe[Fe(CN)6 ] + 2K +
SO24- + CuCl 2 ¾® Soluble, Thus

Flo
(a) CuCl 2 selectively precipitates S2– . The red colour of Fe3+ ion with potassium thiocyanate is due to
formation of [Fe(SCN)3 ].
(b) S2– + BaCl 2 ¾® BaS ¯ (soluble)

ree
Fe3+ + 3KSCN ¾® [Fe(SCN)3 ] + 3K +
SO24- + BaCl 2 ¾® BaSO4 ¯ (white ppt.) Red coloured
(b) precipitates SO2- 2-
4 and not S .

F
3+ 3+
31. Both Al and Bi are precipitated as their hydroxides.
(c) S2– + Pb2+ ¾® PbS ¯ (black ppt.)
32. As MgSO4 is soluble in water, so not used for estimation of SO2-
SO24- + Pb2+ ¾® PbSO4 ¯ (white ppt.)
4

or
S2- and SO2-
ur ion.
4 , both are precipitated. 33. Cation Cd 2+ belongs to group II while Ni 2+ belongs to group

f
2-
(d) S + Na 2 [ Fe (CN)]5 NO ¾® Na 4 [ Fe (CN)]5 NOS] III of analytical group. Group II radicals are precipitated by
ks
Sodium nitroprusside (Purple colour) passing H2S(g ) through acidic solution of salt but radicals of
Yo
But no colour with SO2- group III are precipitated by passing H2S(g ) in NH3 /NH4Cl
4 .
oo
buffer solution of salt due to greater solubility products of later
26. Only radicals of I and II group of qualitative analysis get salts.
eB

precipitated with H2S in the presence of dilute HCl. 34. PLAN This problem can be solved by using concept of chemical
+
(c) Cu 2+ + H2S ¾H¾® CuS ¯ reactions of transition metal ions (,) colour and structure of
Black transition metal compounds.
+
r

Pb2+ + H2S ¾H¾® PbS ¯ Here, among given four option Ni 2+ and Zn 2+ has ability to form
ou
ad

Black tetrahedral as well as square planar complex depending upon


+
(d) Hg2+ + H2S ¾H¾® HgS ¯ types of reagent used.
Y

Black
Ni 2+ on reaction with KCN forms square planar complex
H+
Bi 3+
+ H2S ¾ ¾® Bi 2S3 ¯ [Ni(CN)4 ]2- due to strong field strength of CN.
nd

Brown ppt
Re

Ba , Zn
2+ 2+
and Fe 3+
are not precipitated as sulphide. Ni 2+ + KCN ¾® [Ni(CN)4 ]2-
Square planar
Fi

+3 2+
27. K3 [Fe(CN)6 ] + KI (excess) ¾® While on reaction with HCl, Ni forms stable tetrahedral
+2 complex [Ni(Cl)4 ]2-.
K4[Fe(CN)6 ] + KI3 (redox)
Brownish yellow Zn 2+ , on the other hand, on reaction with KCN as well as HCl
solution produces tetrahedral complex because of its d 10 electronic
K4[Fe(CN)6 ] + ZnSO4 ® K2Zn 3[Fe(CN)6 ]2 configuration.
HCl
or K2 Zn[Fe(CN)6 ] [ZnCl4]2–
White ppt KCN
Zn2+ [Zn(CN)4]2–
I-3 + 2Na 2S2O3 ¾® Na 2 S4O6 + 2NaI + I2 KOH
[Zn(OH)4]2–
Brownish Clear (Turns starch excess
White ppt
yellow solution solution
filtrate blue) Complete reaction sequence can be shown as
K2Zn[Fe(CN)6 ] reacts with NaOH as
K2Zn[Fe(CN)6 ] + NaOH ¾® [Zn(OH)4 ]2 - + [Fe(CN)6 ] 4-
282 Qualitative Analysis

[NiCl4]2–
HCl(Q)
Ni2+
KCN(R)
[Ni(CN)4]2– 42. Y is FeCl 3.
excess excess
Tetrahedral (M1) Square planar 43. Compound Z is Fe[Fe(CN)6 ].
HCl(Q) KCN(R) [Zn(CN)4]2–
[ZnCl4]2– Zn2+
excess (M2)
excess 44. Heating chloride salt with K2Cr2O7 in conc. H2SO4 gives off a
Tetrahedral Tetrahedral
KOH (s)
deep vapour of chromyl chloride (CrO2Cl 2 ) .

Zn(OH)2 KOH [Zn(OH)4]2– 45. Because Fe(OH)2 is soluble.


White ppt excess
Soluble
46. ZnS is soluble in dil. HCl but CuS does not dissolve in dil. HCl.
35. Zn 2+ on treatment with excess of KOH produces [Zn(OH)4 ]2- . 47. In absence of NH4Cl, both Fe(OH)3 and Mg(OH)2 will be
precipitated.
36, 37. PLAN PbCl 2 is soluble in hot water.

w
In ammoniacal medium, cations of group III and IV may be 48. From qualitative analysis of the different metal ions it is found
precipitated as hydroxide or sulphide and dissolved in H2O2 due that PbS, CuS, HgS, Ag2S, NiS, CoS are black coloured.
to oxidation. MnS — dirty pink/buff coloured, SnS2—yellow coloured.
(i) HCl
Mixture of two inorganic salts ¾¾¾® Bi 2S3 — brown/black (brownish black) coloured.

Flo
(ii) Filter
Hence, correct integer is (6 or 7).
¾¾¾¾¾¾¾¾
½

ree
H2 O2 NH4 OH/H2 S ¯ Zn
CrO24 - ¬¾¾ Cr(OH)3 ¬¾¾¾¾¾ Filtrate ppt 49. MCl 4 ¾® Purple coloured compound ( A )
Yellow NaOH ppt Colourless

F
(S ) Moist
M ¾¾® B (white fumes)
2+ Air
Thus, Q. 35. P is Pb . Q. 36. S is Na 2CrO4. Transition metal
Þ M = Ti, 3+
A = [Ti(H2O)6 ] ; B = TiO2

or
ur
38. Appearance of blue colour on addition of the metal rod M to the
solution of N is an indication that metal may be Cu. Later Ti (IV) contains no d-electron, while d-d transition of single

f
formation of white precipitate on addition of NaCl and electron of Ti (III) will cause colour change.
ks
dissolution of this precipitate in aqueous ammonia confirm that
50. (i) Fe3+ + 3SCN- ¾® Fe(SCN)3 (blood red colouration)
while precipitate is of AgCl. This implies that the solution N is
Yo
Iron (III)
oo
of AgNO3. This confirm that the metal M is only Cu. thiocyanate
A
39. The compound N is AgNO3, explained above.
Fe(SCN)3 + F- (excess) ¾® [FeF6 ]3- + 3SCN-
eB

40. Since, in the beginning, concentrated AgNO3 solution was (Hexafluoroferrate)


taken, some AgNO3 remain in solution which gives white B
precipitate of AgCl on addition of NaCl. The precipitate finally (ii) Magnetic moment (m S ) = n (n + 2) BM
r

dissolve in ammonia and Cu2+ present in solution also forms


ou

= 35 BM = 5.92 BM
ad

complex with ammonia as


51. A1 = Cu(OH)2 × CuCO3 and A2 = Cu 2S
Y

Cu (NO3 )2 + 4NH3 (aq) ® [ Cu(NH3 )4 ] 2+ + 2NO-3


Calcination
Blue A1 ¾¾¾¾® 2CuO + CO2 + H2O
AgCl(s) + 2NH3 (aq) ® [Ag(NH3 )2 ] + + Cl –
nd

Black
Re

Dil. HCl
The comprehension describing methylene-blue test. A1 ¾¾¾® CuCl 2 + CO2 + H2O
Fi

NMe2 2CuCl 2 + 4KI ¾® 2CuI + I2 + 4KCl


2Cu 2S ( A2 ) + 3O2 ¾® 2Cu 2O + 2SO2
S2– + 2 D
+ Fe(III) Cu 2S + 2Cu 2O ¾® 6Cu + SO2

52. A + B ¾® lilac colour in flame


123
NH2 Mixture
+
Me2N S NMe2 H2 S
(i) A + B ¾® C ¯
123 Black
Mixture
N Soluble Evaporation
C ¾¾¾¾® Soluble ¾¾¾¾® Residue
Blue solution in aqua-regia
SnCl 2
Therefore, ¾¾® Greyish black ¯ (D )
41. X is Na 2S. Þ C is HgS.
Qualitative Analysis 283

NH OH 56. The transition metal is Cu 2+ . The compound is CuSO4 × 5H2O. It


¾¾¾®
4
(ii) A+ B Brown ppt.
123 dissolves in water to give blue coloured solution due to presence
Solution of
mixture of Cu 2+ . On passing H2S(g ) in acid medium of salt solution
AgNO NH black precipitate of CuS is obtained which is not soluble in
Sodium extract ¾¾®
3
Yellow ppt. ¾¾®
3
Insoluble aqueous KOH solution.
H+
CCl 4 /FeCl 3 CuSO4 + H2S ¾® CuS ¯ + H2SO4
Sodium extract of salt ¾¾¾¾® Violet layer black
Þ A = KI and B = HgI2. On adding KI solution to aqueous solution of CuSO4, yellow
solution of CuI2 is formed in the beginning which decompose
53. X = NH4Cl; A = CrO2Cl 2; B = Na 2CrO4; C = PbCrO4; into white ppt of CuI.
D = H2N(HgO)HgI. CuSO4 + 2KI ¾® CuI2 + K2SO4

w
Reactions involved : 2CuI2 ¾® 2CuI ¯ + I2
White
(i) 4NH4Cl ( X ) + K2Cr2O7 + 6H2SO4 ¾® 2CrO2Cl 2 ( A )
57. NaNO3 + H2SO4 ¾® NaHSO4 + HNO3

Flo
+ 4NH4HSO4 + 2KHSO4 + 3H2O
2HNO3 + 6FeSO4 + 3H2SO4 ¾® 3Fe2 (SO4 )3 + 2NO + 2H2O
(ii) CrO2Cl 2 ( A ) + 2NaOH ¾® Na 2CrO4 ( B ) + 2HCl
FeSO4 + NO ¾® [Fe(NO)SO4 ]

ree
(iii) Na 2CrO4 ( B ) + (CH3COO)2 Pb ¾® PbCrO4 ¯ (C ) Brown ring
(nitrosoferrous sulphate)
+ 2CH3COONa

F
58. NaI on reaction with HgI2 gives a complex salt :
(iv) NH4Cl ( X ) + NaOH ¾® NaCl + H2O + NH3
2NaI + HgI2 r Na 2[HgI4 ]
(v) 2K2HgI4 + NH3 + 3KOH ¾® H2N(HgO)HgI (D )

or
The orange colour is due to residual HgI2. On addition of excess
ur + 7KI + 2H2O NaI, whole HgI2 is converted to complex Na 2[HgI4 ] as colour

f
disappear. The orange colour of HgI2 reappear due to conversion
54. Since, the white substance A gives a colourless gas B with of Na 2[HgI4 ] into HgI2 on treatment with NaOCl.
ks
dil. H2SO4, such gas may be H2S. So, the substance A may be a
3Na 2[HgI4 ] + 2NaOCl + 2H2O ¾® 3HgI2 + 2NaCl
Yo
metal sulphide (Na/K/Zn, etc.)
oo
+ 4 NaOH + 2NaI
When H2S gas reacts with acidified K2Cr2O7 , it gives green
coloured solution of Cr2 (SO4 )3 alongwith slightly yellow ppt of 59. K sp (solubility product) of CuS is less than K sp of ZnS. On
eB

D as sulphur. passing H2S(g ) in acidic solution, dissociation of H2S is


suppressed due to common ion effect and it provide small
K2Cr2O7 + 4H2SO4 + 3H2S ¾® K2SO4 + Cr2 (SO4 )3
concentration of S2- , sufficient for precipitation of CuS but
r

Green
insufficient for precipitation of ZnS.
ou
ad

+ 3S + 7H2
D 60. Al 2S3 + 6H2O ¾® 2Al(OH)3 ¯ + 3H2S ­
Y

S on burning in air gives SO2 (E). Substance E on reaction with Foul odour

B (H2S) produces D (s) : 61. The poisonous element M may be As. On the basis of given
information :
nd

2H2S + SO2 ¾® 2H2O (l ) + 3S ¯


Re

B E D Zn
AsCl 3 + 6H ¾¾® AsH3 + 3HCl
Fi

Anhydrous CuSO4 produces blue colour in water. H2 SO4 N


Solution C produces ppt first with NH3 /NaOH which dissolve in D
2AsH3 ¾¾® 2As + 3H2 ­
excess NH3 /NaOH. Hence, A must be ZnS. M
ZnS + dil. H2SO4 ¾® ZnSO4 (aq) + H2S(g ) 250 °C
A 62. A ¾¾® B + C
C
Inorganic salt Neutral Neutral oxide
oxide (liquid)
ZnSO4 + 2NaOH ¾® Zn(OH)2 ¯ + Na 2SO4
C White Oxide C is liquid and neutral to litmus, so it is H2O. White
2- phosphorus burns in excess of B to give P4O10.
Zn(OH)2 + 2NaOH ¾® [Zn(OH)4 ] + 2Na +
Therefore, B is N2O.
D D
55. Na 2B4O7 ¾® 2NaBO2 + B2O3 NH4NO3 ( A ) ¾® N2O + 2H2O (l )
D B
CoO + B2O3 ¾® Co(BO2 )2
Blue P4 + 10N2O ¾® P4O10 + 10N2
284 Qualitative Analysis

63. On addition of KI solution to a Bi(NO3 )3 solution, firstly a dark 70. Formation of a reddish-brown precipitate with alkaline K2HgI4
brown precipitate of BiI3 is formed that dissolve in excess of KI solution indicates the presence of NH+4 ion and the gas liberated
forming a clear yellow solution of BiI-4 : is ammonia :

Bi 3+ + 3I- ¾® BiI3 ¯ ¾®
I-
BiI-4 NH+4 + OH- ¾® H2O + NH3 (g )
Dark Clear yellow NH2
brown solution Hg
C× HNO3 Filtered 2K2HgI4 + 3NaOH + NH+4 ¾® O ¯
64. A (Pb3O4 ) ¾¾® B (PbO2 ) ¾¾® Pb(NO3 )2 Hg
Scarlet Chocolate Filtrate I
Filtrate is neutralised with NaOH and on reaction with KI, gives + 4KI + 3NaI + 3OH-
yellow ppt of PbI2.
On treatment with BaCl 2, a white precipitate is formed which

w
Pb(NO3 )2 + 2KI ¾® PbI2 ¯ + 2KNO3 indicates the presence of SO2-
4 anion :
Yellow
PbO2 on warming with conc. HNO3 in presence of Mn(NO3 )2 SO24- + BaCl 2 ¾® BaSO4 ¯ + 2Cl -
White
produced pink solution due to formation of Pb(MnO4 )2 (I).

Flo
5PbO2 + 2Mn(NO3 )2 + 4HNO3 ¾® Pb(MnO4 )2 With K2Cr2O7 and conc. H2SO4, red vapour of CrO2Cl 2 is
evolved. This indicates presence of Cl - ion.
+ 4Pb(NO3 )2 + 2H2O

ree
On treatment with potassium ferricyanide, formation of deep
Þ A = Pb3O4, B = PbO2 , C = PbI2 and D = Pb(MnO4 )2. blue solution indicates presence of Fe (II) ion :
D Fe2+ + Fe(CN)63- ¾® Fe4[Fe(CN)6 ]3

F
65. A ¾® B ¯ + C­ + H2O (v ) ­
Orange solid Green Colourless Blue

71. (i) AgCl + 2NaCN ¾® NaCl + Na[Ag(CN)2 ]

or
C + Mg ¾® D (white solid)
ur
HCl 2Na[Ag(CN)2 ] + Zn ¾® Na 2[Zn(CN)4 ] + 2Ag

f
D + H2O ¾® E (g ) ¾® White fumes.
(ii) Co2+ + 3KNO3 + 4NO2- + 2H+ ¾® K3[Co(NO2 )6 ]
ks
Hence, E must be ammonia gas so D must be Mg3N2 and C is
+ H2O + NO
Yo
N2 (g ). This N2 is obtained on strong heating of (NH4 )2 Cr2O7
oo
because (NH4 )2 Cr2O7 is orange solid and produces green Cr2O3 72. Mercurous chloride changes from white to black when treated
residue on heating. with ammonia due to formation of metallic Hg.
eB

D
(NH4 )2 Cr2O7 ¾® Cr2O3 + N2 + 4 H2O 73. (i) Mixture + MnO2 + H2SO4 ¾® Yellowish green gas.
Orange (A) Green (B) C conc.
D H2 O The above reaction suggest that the mixture contain Cl - .
r

N2 + Mg ¾® Mg3N2 ¾¾® NH3 + Mg(OH)2


2Cl - + MnO2 + H2SO4 ¾® MnSO4
ou
ad

D E
+ Cl 2 ­ + H2O
NH3 + HCl ¾® NH4Cl (white fumes)
Y

E
(ii) Mixture + NaOH ¾® Gas turning red litmus blue.
The above reaction indicates that the gas is ammonia.
66. 3[Fe(H2O)6 ]2+ + NO-3 + 4H+ ¾® NO + 3[Fe(H2O)6 ]3+
NH+4 + NaOH ¾® NH3 + H2O + Na +
nd
Re

+ 2H2O
(iii) Solution of mixture + K3[Fe(CN)6 ] ¾® blue ppt.
[Fe(H2O)6 ]2+ + NO ¾® [Fe(H2O)5 NO]2+ + H2O
Fi

The above reaction suggest that mixture contain a Fe(II) salt.


67. (i) Brown precipitate or colour with K2HgI4, indicates the Fe(II) salt react with K3[Fe(CN)6 ] to give blue ppt of
presence of NH+4 ion. Prussian blue complex
(ii) Aqueous Fe(II) solution gives a blue colour with 3Fe2+ + 2Fe(CN)63- ¾® Fe3[Fe(CN)6 ]2 ¯
K3[Fe(CN)6 ] due to formation of Fe[Fe(CN)6 ]- ion : Blue
Red colouration with NH4SCN suggests that some Fe(III) is
Fe2+ + Fe(CN)63- ¾® Fe[Fe(CN)6 ]- also present. It is likely that a part of Fe(II) is oxidised to
Blue colouration
Fe(III) by air :
(iii) Aqueous solution of SO2-
ion in HCl gives white ppt with
Fe3+ + 3NH4SCN ¾®
4
BaCl 2 solution. Fe(SCN)3 + 3NH+4
Red colouration
SO24- + BaCl 2 ¾® BaSO4 ¯ + 2Cl - K 2 HgI 4
(iv) Mixture + KOH ¾® gas ­ ¾¾¾® brown ppt.
Heat in Heat in
68. PbS ¾¾® PbO + PbS ¾¾¾¾® Pb + SO2 The above reaction indicates that the gas is NH3, i.e. mixture
air A absence of air
contain NH+4 ion. Hence, the mixture contains Fe2+ , NH+4
69. In excess of NaOH, Al(OH)3 dissolves forming NaAlO2 while and Cl - ions with some impurity of Fe3+ ion.
Fe(OH)3 remains insoluble. The two salts are FeCl 2 and NH4Cl.
Qualitative Analysis 285

74. 2KMnO4 + 3H2SO4 + 5H2C2O4 ¾® K2SO4 + 2MnSO4 (v) The sulphuric acid (formed in previous step) gives white ppt.
with Ba(NO3 )2 due to formation of BaSO4 (E) :
+ 8H2 O + 10CO2
H2SO4 + Ba(NO3 )2 ¾® BaSO4 ¯ + 2HNO3
75. (i) ZnO + 2NaOH ¾® Na 2ZnO2 + H2O White ( E )
(ii) Sn 2+ + I2 ¾® Sn 4+ + 2I- (vi) The residue D when heated on charcoal in a reducing flame
1 reduces to iron (Fe) which is a magnetic substance.
(iii) SO2 + H2O + O2 + 2NaCl ¾® Na 2SO4 + 2HCl
2 Hence, A = FeSO4, B = SO2, C = SO3, D = Fe2O3 and
76. (i) SO2 + 2H2S ¾® 3S + 2H2O E = BaSO4.
(ii) CuSO4 + 4NH3 ¾® [Cu(NH3 )4 ]SO4 80. FeCl 3 is a salt of strong acid and weak base. In water, it hydrolyses
(iii) Sn + 3HNO3 ¾® H2SnO3 + 4NO2 + H2O slowly producing brown ppt of Fe(OH)3.

w
Conc. Metastannic
acid FeCl 3 + 3H2O ¾® 3HCl + Fe(OH)3 ¯
(iv) H2O2 + CrCl 3 + NaOH ¾® Na 2CrO4 + NaCl + H2O Brown
Yellow

Flo
81. Nitric acid is a strong oxidising agent, oxidises H2S to S.
(v) Pb3O4 ¾® 2PbO + PbO2
2PbO + PbO2 + 4HNO3 ¾® 2Pb(NO3 )2 + 2H2O+ PbO2 ¯ 82. (i) The compound C produced by heating A is white in colour and

ree
changes to yellow on heating, thus compound C may be ZnO.
77. 3KClO3 + 3H2SO4 ® 3KHSO4 + HClO4 + 2ClO2 + H2O C with dil. HCl and K4[Fe(CN)6 ] gives white ppt. This
(COOH)2 + C × H2SO4 ¾® CO + CO2 + H2O confirms that the compound C must be ZnO.

F
D
78. (X) is NaHCO3 (M = 84). The reactions involved are : A ¾® ZnO + B (gas)
‘C ’

or
ur
2NaHCO3 ¾® Na 2CO3 (s) + CO2 (g ) + H2O (g )
ZnO + 2HCl ¾® ZnCl 2 + H2O
X Y A B

f
C
168 g 106 g 44 g 18 g 2ZnCl 2 + K4[Fe(CN)6 ] ¾® 4KCl + Zn 2[Fe(CN)6 ] ¯
ks
Þ 16.8 g 10.6 g 4.4 g 1.8 g
White ppt
Yo
CO2 + Ca(OH)2 ¾® CaCO3 ¯ + H2O
oo
(ii) The gas B turns lime water milky and milkiness disappear with
H2O (g ) is condensed to liquid water. continuous passage of gas. Hence, the gas is CO2 and
eB

Na 2CO3 + BaCl 2 ¾® BaCO3 + 2NaCl compound A in ZnCO3.


Y Z
CO2 + Ca(OH)2 ¾® H2O + CaCO3 ¯
BaCO3 + 2HCl ¾® BaCl 2 + H2O + CO2 B
r

Z
CaCO3 + CO2 + H2O ¾® Ca(HCO3 )2
ou
ad

79. Compound A is a light green crystalline solid, so it may be


FeSO4. D
ZnCO3 ¾® ZnO + CO2
Y

(i) FeSO4 is a salt of strong acid and weak base, so it A C B


hydrolyses in dil. H2SO4 but no gas is evolved. (iii) The solution of A gives white ppt of ZnS D with NH4OH and
(ii) FeSO4 is a strong reducing agent, thus decolourises KMnO4
nd

excess of H2S.
Re

solution :
ZnCO3 + HCl ¾® CO2 ­ + ZnCl 2
5Fe2+ + MnO4- + 8H+ ¾® 5Fe3+ + Mn 2+ + 4H2O
Fi

Colourless
B
Purple

(iii) FeSO4 on strong heating gives both SO2 (B) and SO3 (C ) NH4 OH
ZnCl 2 + H2S ¾¾¾® 2HCl + ZnS¯ (white)
gases alongwith a residue of Fe2O3 (D ). D
D
2FeSO4 ¾® Fe2O3 + SO2 + SO3 (iv) The solution of A also gives initially a white ppt E with NaOH,
D B C which dissolve in excess of reagent.
(iv) The gaseous mixture reduced dichromate solution to green
ZnCl 2 + 2NaOH ¾® Zn(OH)2 ¯ + 2NaCl
solution and also gives H2SO4 in solution : E (white)
Cr2O72- + 3SO2 + 2H+ ¾® 3SO24- + H2O + 2Cr 3+
Green Zn(OH)2 + 2NaOH ¾® Na 2[Zn(OH)4 ]
Soluble
H2O + SO3 ¾® H2SO4

Download Chapter Test


http://tinyurl.com/y626t8yl or
21
Organic Chemistry Basics

w
Topic 1 Nomenclature and Isomerism

Flo
Objective Questions I (Only one correct option) 5. What is the IUPAC name of the following compound?
1. The IUPAC name for the following compound is CH3 CH3

ree
CH3 H
H
H3 C CH Br
CH3

F
(2019 Main, 10 Jan II)
(a) 3-bromo-3-methyl-1,2-dimethylprop-1-ene
CH3
(b) 3-bromo-1,2-dimethylbut-1-ene

or
CH2
ur (2019 Main, 12 April II) (c) 2-bromo-3-methylpent-3-ene
(a) 3-methyl-4-(3-methylprop-1-enyl)-1-heptyne (d) 4-bromo-3-methylpent-2-ene
(b) 3, 5-dimethyl-4-propylhept-6-en-1-yne

f CO2H
ks
(c) 3-methyl-4-(1-methylprop-2-ynyl)-1-heptene
H OH
Yo
(d) 3, 5-dimethyl-4-propylhept-1-en-6-yne
H Cl
oo
2. The correct IUPAC name of the following compound is 6. The absolute configuration of is (2016 Main)
NO2
CH3
eB

(a) (2S, 3R) (b) (2S, 3S)


(c) (2R, 3R) (d) (2R, 3S)
7. The IUPAC name of the following compound is
r

Cl
ou
ad

CH3 (2019 Main, 9 April I) OH


(a) 2-methyl-5-nitro-1-chlorobenzene
Y

(b) 3-chloro-1-methyl-1-nitrobenzene
(c) 2-chloro-1-methyl 1-4-nitrobenzene
nd

(d) 5-chloro-4-methyl 1-1-nitrobenzene CN


Re

3. Which of the following compounds will show the maximum Br


Fi

‘enol’ content? (2019 Main, 8 April II) (2009)


(a) CH3COCH3 (a) 4-bromo-3-cyanophenol
(b) 2-bromo-5-hydroxybenzonitrile
(b) CH3COCH2COCH3
(c) 2-cyano-4-hydroxybromobenzene
(c) CH3COCH2COOC2H5
(d) 6-bromo-3hydroxybenzonitrile
(d) CH3COCH2CONH2
8. The number of stereoisomers obtained by bromination of
4. The IUPAC name of the following compound is trans-2-butene is (2007, 3M)
CH3 OH (a) 1 (b) 2 (c) 3 (d) 4
½ ½
H3C ¾ CH ¾ CH ¾ CH 2 ¾ COOH 9. The IUPAC name of C6 H5 COCl is (2006, 3M)
(2019 Main, 8 April I)
(a) benzoyl chloride
(a) 4,4 - dimethyl -3-hydroxybutanoic acid
(b) 2-methyl-3-hydroxypentan-5-oic acid (b) benzene chloro ketone
(c) 3- hydroxy -4- methylpentanoic acid (c) benzene carbonyl chloride
(d) 4-methyl-3-hydroxypentanoic acid (d) chloro phenyl ketone
Organic Chemistry Basics 287

1
CH3 21. The IUPAC name of the compound
H CH2 == CH ¾ CH(CH3 )2 is (1987, 1M)
10. H
3 (a) 1,1-dimethyl-2-butene (b) 3-methyl-1-butene
2 (c) 2-vinyl propane (d) None of these
H H
22. An isomer of ethanol is (1986, 1M)
CH3 (a) methanol (b) diethyl ether
C 2 is rotated anti-clockwise 120° about C 2 -C 3 bond. The (c) acetone (d) dimethyl ether
resulting conformer is (2004, 1M) 23. The IUPAC name of the compound having the formula is
(a) partially eclipsed (b) eclipsed CH3
(c) gauche (d) staggered ½
11. Which of the following compounds exhibits, H3 C ¾ C ¾ CH==CH2

w
½
stereoisomerism? (2002, 3M) CH3
(a) 2-methylbutene-1 (b) 3-methylbutyne-1 (1984, 1M)
(c) 3-methylbutanoic acid (d) 2-methylbutanoic acid (a) 3, 3, 3-trimethyl-1-propene

Flo
(b) 1, 1, 1-trimethyl-2-propene
12. The number of isomers for the compound with molecular (c) 3, 3-dimethyl-1-butene
formula C2 BrClFI is (2001, 1M) (d) 2, 2-dimethyl-3-butene

ree
(a) 3 (b) 4 (c) 5 (d) 6
24. Which of the following compounds will exhibit cis-trans
13. Which of the following compounds will exhibit geometrical (geometrical) isomerism ? (1983, 1M)

F
isomerism ? (2000, 1M) (a) 2-butene (b) 2-butyne
(a) 1-phenyl-2-butene (b) 3-phenyl-1-butene (c) 2-butanol (d) butanal
(c) 2-phenyl-1-butene (d) 1,1-diphenyl-1-propene

or
ur
14. The optically active tartaric acid is named as D-(+)-tartaric
25. The compound which is not isomeric with diethyl ether is
(a) n-propyl methyl ether (b) butane-1-ol (1981, 1M)

f
acid because it has a positive (1999, 2M) (c) 2-methyl propane-2-ol (d) butanone
(a) optical rotation and is derived from D-glucose
ks
(b) pH in organic solvent Objective Questions II
Yo
oo
(c) optical rotation and is derived from D-(+)-glyceraldehydes (One or more than one correct option)
(d) optical rotation when substituted by deuterium
26. The IUPAC name(s) of the following compound is (are)
eB

15. How many optically active stereoisomers are possible for (2017 Adv.)
butane-2, 3-diol ? (1997, 1M)
(a) 1 (b) 2 (c) 3 (d) 4 H3C Cl
r
ou

16. Isomers which can be interconverted through rotation


ad

(a) 4-methylchlorobenzene
around a single bond are (1992, 1M)
(b) 4-chlorotoluene
Y

(a) conformers (b) diastereomers


(c) 1-chloro-4-methylbenzene
(c) enantiomers (d) positional isomers
(d) 1-methyl-4-chlorobenzene
17. The enolic form of acetone contains
nd

(1990, 1M)
Re

27. The correct combination of names for isomeric alcohols


(a) 9 sigma bonds, 1 pi bond and 2 lone pairs
with molecular formula C4 H10 O is/are (2014 Adv.)
Fi

(b) 8 sigma bonds, 2 pi bonds and 2 lone pairs


(a) tert-butanol and 2-methylpropan-2-ol
(c) 10 sigma bonds, 1 pi bond and 1 lone pair
(b) tert-butanol and 1,1-dimethylethan-1-ol
(d) 9 sigma bonds, 2 pi bonds and 1 lone pair
(c) n-butanol and butan-1-ol
18. The number of isomers of C6 H14 is (1987, 2007, 3M) (d) iso-butyl alcohol and 2-methylpropan-1-ol
(a) 4 (b) 5 (c) 6 (d) 7
28. Which of the given statement(s) about N, O, P and Q with
19. If two compounds have the same empirical formula but respect to M is/are correct? (2012)
different molecular formulae, they must have (1987, 1M)
(a) different percentage composition Cl
HO HO
(b) different molecular weight H H H
(c) same velocity OH
Cl CH3
(d) same vapour density HO H HO
H OH H
20. Which of the following will have least hindered rotation
about carbon-carbon bond ? (1987, 1M) CH3 CH3 CH3
(a) Ethane (b) Ethylene M N O
(c) Acetylene (d) Hexachloroethane
288 Organic Chemistry Basics

CH3 CH3 34. Which of the following have asymmetric carbon atom?
H OH HO H (1989, 1M)
Cl Br H Cl
½ ½ ½ ½
(a) H ¾ C ¾ C ¾ H (b) H ¾ C ¾ C ¾ Cl
HO H HO H ½ ½ ½ ½
Cl Cl H H H H
P Q H Cl H H
½ ½ ½ ½
(a) M and N are non-mirror image stereoisomers (c) H ¾ C ¾ C ¾ D (d) H ¾ C ¾ C ¾ CH3
(b) M and O are identical ½ ½ ½ ½
H H Br OH
(c) M and P are enantiomers
(d) M and Q are identical 35. Keto-enol tautomerism is observed in (1988, 2M)

w
O O
29. The correct statement(s) about the compound (2009) ½½ ½½
H3 C(HO) HC ¾ CH == CH ¾ CH(OH)CH3 ( X ) is/are (a) C6H5 ¾ C ¾ H (b) C6H5 ¾ C ¾ CH3

(a) The total number of stereoisomers possible for X is 6 O O O

Flo
½½ ½½ ½½
(b) The total number of diastereomers possible for X is 3 (c) C6H5 ¾ C ¾ C6H5 (d) C6H5 ¾ C ¾ CH2 ¾ C ¾ CH3
(c) If the stereochemistry about the double bond in X is trans, the
36. Only two isomeric monochloro derivatives are possible for

ree
number of enantiomers possible for X is 4 (1986, 1M)
(d) If the stereochemistry about the double bond in X is cis, the (a) n-butane (b) 2, 4-dimethyl pentane

F
number of enantiomers possible for X is 2 (c) benzene (d) 2-methyl propane
30. The correct statement(s) concerning the structures
Numerical Value Based Question

or
E , F and G is/are
ur 37. For the given compound X, the total number of optically
H 3C O H 3C OH H3C CH3

f
active stereoisomers is ……… .
ks
H 3C CH3 H3C CH3 H 3C OH HO
HO
Yo
(E) (F) (G)
oo
(a) E , F and G are resonance structures (2008, 4M)
(b) E , F and E , G are tautomers
eB

(c) F and G are geometrical isomers


(d) F and G are diastereomers HO
HO
31. The correct statement(s) about the compound given below
r

This type of bond indicates that the configuration at the


is/are
ou

specific carbon and the geometry of the double bond is fixed


ad

Cl H
¾¾¾

This type of bond indicates that the configuration at the specific


¾ ¾¾¾

CH
H3C ¾
carbon and the geometry of the double bond is not fixed
¾ 3
Y

¾
(2018 Adv.)
¾¾

H
¾¾¾

Cl
(2008, 4M) Assertion and Reason
nd
Re

(a) the compound is optically active


Read the following questions and answer as per the direction
(b) the compound possesses centre of symmetry given below :
Fi

(c) the compound possesses plane of symmetry (a) Statement I is correct; Statement II is correct; Statement II is
(d) the compound possesses axis of symmetry the correct explanation of Statement I
32. Tautomerism is exhibited by (1998, 2M) (b) Statement I is correct; Statement II is correct; Statement II is
not the correct explanation of Statement I
¾
(a) ¾
¾ CH== CH ¾ OH (b) O ¾ ¾O
¾ (c) Statement I is correct; Statement II is incorrect
¾ (d) Statement I is incorrect; Statement II is correct
¾ 38. Statement I Molecules that are non-superimposable on
(c) ¾
¾O (d) ¾
¾O their mirror images are chiral.
Statement II All chiral molecules have chiral centres.
¾
¾

¾
¾

O (2007, 3M)
O
33. Which of the following compounds will show geometrical Fill in the Blanks
isomerism ? (1998, 2M)
39. Isomers which are ……… mirror images are known as
(a) 2-butene (b) propene
……… (superimposable, non-superimposable,
(c) 1-phenyl propene (d) 2-methyl-2-butene
enantiomers, diastereomers, epimers) (1988, 1M)
Organic Chemistry Basics 289

True/False 46. (a) Draw Newman’s projection for the less stable staggered
form of butane.
40. 2, 3, 4-trichloropentane has three asymmetric carbon atoms.
(1990, 1M)
(b) Relatively less stability of the staggered form is due to
(i) Torsional strain
41. m-chlorobromobenzene is an isomer of m-bromochloro
benzene. 1 (ii) van der Waals’ strain
(1985, M)
2 (iii) Combination of the above two (2004, 5M)

Integer Answer Type Questions 47. Glycerine contain one ……… hydroxy group. (2004)

42. The total number of stereoisomers that can exist for M is 48. Identify the pairs of enantiomers and diastereomers from the
(2015 Adv.) following (2000, 2M)
H 3C CH3

w
CH3 CH3 CH3
H OH HO H HO H

HO H HO H H OH

Flo
CH3 CH3 CH3
H 3C M I II III
O

ree
43. The total number(s) of stable conformers with non-zero 49. Write tautomeric forms of phenol. (1992, 1M)
dipole moment for the following compound is/are

F
50. Write the IUPAC name of the following compound:
Cl CH3
Br CH3 ½

or
Br Cl
ur H3 C ¾ N ¾– C ¾ C H2 ¾ CH3
CH3 ½ ½

f
(2014 Adv.)
CH3 C2H5 (1991, 1M)
ks
Subjective Questions 51. Give the IUPAC name of the following compound :
Yo
44. Give the total number of cyclic structural as well as
oo
Me
stereoisomers possible for a compound with the molecular
Me Me
eB

formula C5 H10 . (2009)


(Me = methyl)
45. m obs = S m i xi Me Me
where m i is the dipole moment of stable conformer and xi is (1990, 1M)
r

the mole fraction of that conformer. 52. Write the IUPAC name of CH3 CH2 CH==CH ¾ COOH .
ou
ad

(a) Write stable conformer for Z ¾ CH2 ¾ CH2 ¾ Z in Newman’s (1986, 1M)
Y

projection.
53. Write the structure of all the possible isomers of
If m solution = 1.0 D and mole fraction of anti form = 0.82, find dichloroethene. Which of them will have zero dipole
m gauche
nd

moment ? (1985, 2M)


Re

(b) Write most stable meso conformer of (CHDY )2. If 54. Write structural formulae for the isomeric alcohols having
Fi

(i) Y = CH3 about C 2 -C 3 rotation and the molecular formula C4 H10 O. (1984, 2M)

(ii) Y = OH about C1 -C 2 rotation. (2005, 6M)

Topic 2 General Organic Chemistry


Objective Questions I (Only one correct option) 2. An organic compound A is oxidised with Na 2 O2 followed by
1. 25 g of an unknown hydrocarbon upon burning produces boiling with HNO3 . The resultant solution is then treated
88 g of CO2 and 9 g of H2 O. This unknown hydrocarbon with ammonium molybdate to yield a yellow precipitate.
contains (2019 Main, 12 April II) Based on above observation, the element present in the
(a) 20 g of carbon and 5 g of hydrogen given compound is (2019 Main, 12 April I)
(b) 22 g of carbon and 3 g of hydrogen (a) nitrogen
(c) 24 g of carbon and 1 g of hydrogen (b) phosphorus
(d) 18 g of carbon and 7 g of hydrogen (c) fluorine
(d) sulphur
290 Organic Chemistry Basics

3. The increasing order of nucleophilicity of the following 10. Which amongst the following is the strongest acid?
nucleophiles is (2019 Main, 10 April II) (2019 Main, 9 Jan I)
È (a) CHBr3 (b) CHI3
(1) CH3 CO2È (2) H2O (3) CH3 SO3È (4) O H
(c) CHCl 3 (d) CH(CN)3
(a) (1) < (4) < (3) < (2) (b) (2) < (3) < (1) < (4)
(c) (4) < (1) < (3) < (2) (d) (2) < (3) < (4) < (1)
11. Which of the following compounds will be suitable for
Kjeldahl’s method for nitrogen estimation? (2018 Main)
4. In chromatography, which of the following statements is
NH2
incorrect for R f ? (2019 Main, 10 April II)
(a) R f value depends on the type of chromatography (a) (b)
(b) Higher R f value means higher adsorption N
(c) R f value is dependent on the mobile phase
(d) The value of R f can not be more than one NO2 N+2Cl–

w
5. The principle of column chromatography is (c) (d)
(2019 Main, 10 April I)
(a) differential absorption of the substances on the solid phase
12. Which of the following molecules is least resonance

Flo
(b) differential adsorption of the substances on the solid phase
(c) gravitational force stabilised? (2017 Main)
(d) capillary action

ree
6. In the following compound, (a) (b)
O
a

F
NH2
dN
N b (c) (d)
N

or
e N N
ur O
H c

f
13. The distillation technique most suited for separating
the favourable site/s for protonation is/are glycerol from spent lye in the soap industry is (2016 Main)
ks
(2019 Main, 11 Jan II) (a) fractional distillation
Yo
(a) (a) and (e) (b) (b), (c) and (d) (b) steam distillation
oo
(c) (a) and (d) (d) (a) (c) distillation under reduced pressure
7. The correct match between items I and II is (d) simple distillation
eB

Item - I Item II 14. The correct order of acidity for the following compounds is
(Mixture) (Separation method) (2016 Adv.)
A. H2O : Sugar P. Sublimation CO2H
r

CO2H
ou

B. H2O : Aniline Q. Recrystallisation


ad

HO OH
C. H2O : Toluene R. Steam distillation
I II
Y

S. Differential extraction OH
(2019 Main, 11 Jan I)
nd

(a) (A) ® (Q); (B) ® (R); (C) ® (S)


Re

CO2H CO2H
(b) (A) ® (Q); (B) ® (R); (C) ® (P)
(A) ® (S); (B) ® (R); (C) ® (P)
Fi

(c) OH
(d) (A) ® (R); (B) ® (P); (C) ® (S) III IV
8. An organic compound is estimated through Dumas method
and was found to evolved 6 moles of CO2 , 4 moles of H2 O OH
and 1 mole of nitrogen gas. The formula of the compound is
(2019 Main, 11 Jan I) (a) I > II > III > IV
(a) C6H8 N (b) C12H8 N (c) C12H8 N2 (d) C6H8 N2 (b) III > I > II > IV
9. If dichloromethane (DCM) and water (H 2O) are used for (c) III > IV > II > I
differential extraction, which one of the following (d) I > III > IV > II
statements is correct? (2019 Main, 10 Jan I) 15. For the estimation of nitrogen, 1.4 g of an organic compound
(a) DCM and H2O would stay as lower and upper layer was digested by Kjeldahl's method and the evolved ammonia
respectively in the S.F. was absorbed in 60 mL of M /10 sulphuric acid. The unreacted
(b) DCM and H2O would stay as upper and lower layer acid required 20 mL of M /10 sodium hydroxide for complete
respectively in the separating funnel (S.F.) neutralisation. The percentage of nitrogen in the compound is
(c) DCM and H2O will be miscible clearly (a) 6% (b) 10% (2014 Main)

(d) DCM and H2O will make turbid/colloidal mixture (c) 3% (d) 5%
Organic Chemistry Basics 291

16. A gaseous hydrocarbon gives upon combustion 0.72 g of 24. The correct acidity order of the following is (2009)
water and 3.08 g of CO 2. The empirical formula of the OH OH COOH COOH
hydrocarbon is (2013 Main)
(a) C2H4 (b) C3H4 (c) C6H5 (d) C7 H8
17. The order of stability of the following carbocations

CH2 (I) (III)


Cl CH3
CH2 CH CH2 ; CH3 CH2 CH2 ; is
(II) (IV)
(I) (II) (a) (III) > (IV) > (II) > (I) (b) (IV) > (III) > (I) > (II)
(III) (2013 Main)
(c) (III) > (II) > (I) > (IV) (d) (II) > (III) > (IV) > (I)

w
(a) III > II > I (b) II > III > I
(c) I > II > III (d) III > I > II 25. Hyperconjugation involves overlap of the following orbitals
(2008, 3M)
18. Arrange the following compounds in the order of decreasing (a) s - s (b) s - p (c) p-p (d) p - p

Flo
acidity (2013 Main)
OH OH OH OH 26. The correct stability order for the following species is
Ž Å ½

ree
; ; ;
O
(I) (II)
Cl CH3 NO2 OCH3

F
(I) (II) (III) (IV)
Å ½ ½
O Å
(a) II > IV > I > III (b) I > II > III > IV

or
(c) III > I > II > IV
ur
(d) IV > III > I > II
(III) (IV)
(2008, 3M)
(a) (II) > (IV) > (I) > (III) (b) (I) > (II) > (III) > (IV)

f
19. A solution of ( - l ) 1-chloro-1-phenylethane in toluene
racemises slowly in the presence of a small amount of SbCl 5, (c) (II) > (I) > (IV) > (III) (d) (I) > (III) > (II) > (IV)
ks
due to the formation of 27. Among the following, the least stable resonance structure is
Yo
(2013 Main)
(a) carbanion (b) carbene
oo
(2007, 3M)
(c) carbocation (d) free radical Å
Å O Å O
eB

20. In allene (C3 H4 ), the type(s) of hybridisation of the carbon (a) N (b) Å N
½ ½
atoms, is (are) (2012)
O O
3 2
(a) sp and sp (b) sp and sp
r

(c) only sp3 (d) sp2 and sp3


O
ou

Å O Å
ad

21. Among the following compounds, the most acidic is (2011) (c) Å N (d) Å N
½ ½
Y

(a) p-nitrophenol (b) p-hydroxybenzoic acid O


O
(c) o-hydroxybenzoic acid (d) p-toluic acid
22. The correct stability order of the following resonance 28. When benzene sulphonic acid and p-nitrophenol are treated
nd
Re

structure is + – +
with NaHCO3, the gases released respectively are (2006)

(I) H2 C == N == N (II) H2 C ¾ N == N (a) SO2, NO2 (b) SO2, NO
Fi

– + – + (c) SO2, CO2 (d) CO2, CO2


(III) H2 C ¾ N ºº N (IV) H2 C ¾ N == N (2009)
(a) (I) > (II) > (IV) > (III) 29. Which of the following is obtained when 4-methylbenzene
(b) (I) > (III) > (II) > (IV)
sulphonic acid is hydrolysed with excess of sodium acetate?
(2005, 1M)
(c) (II) > (I) > (III) > (IV)
(d) (III) > (I) > (IV) > (II) (a) H3C— —COONa
23. In the following carbocation; H/CH 3 that is most likely to
migrate to the positively charged carbon is (2009) (b) H3C— + SO3
H H
1 ½ + ½ 5
H3 C¾ C2 ¾ C3 ¾ C4 ¾ CH3 (c) H3C— —SO3Na + CH3COOH
½ ½ ½
OH H CH3
(a) CH3 at C-4 (b) H at C-4 (d) H3C— —SO2OCOCH3 + NaOH
(c) CH3 at C-2 (d) H at C-2
292 Organic Chemistry Basics

30. For 1-methoxy-1, 3-butadiene, which of the following HOOC


resonating structure is least stable? (2005, 1M) O–
s Å
(a) CH2 ¾ CH ¾ CH ==CH ¾ O ¾ CH3
s Å
(b) CH2 ¾ CH== CH ¾ CH ==O ¾ CH3 (c)
Å s O 2N
(c) CH2 == CH ¾ CH— CH ¾ O ¾ CH3
s Å CH
(d) CH2 == CH ¾ CH ¾ CH== O ¾ CH3
31. H3N+ +
O–
NH3
Z HOOC
Y
OH

w
COOH
X
Arrange in order of increasing acidic strength (2004, 1M) (d) O2N
(a) X > Z > Y (b) Z < X > Y

Flo
(c) X > Y > Z (d) Z > X > Y C–
32. Among the following, the molecule with the highest dipole

ree
moment is (2003, 1M) O–
(a) CH3Cl (b) CH2Cl 2 (c) CH2Cl 2 (d) CCl 4 35. Which of the following acids has the smallest dissociation

F
constant? (2002)
33. Which of the following represent the given mode of
2 2 (a) CH3CHFCOOH
hybridisation sp - sp - sp - sp from left to right? (b) FCH2CH2COOH

or
ur (2003, 1M) (c) BrCH2CH2COOH
(a) H2C==CH ¾ C ºº N (b) HC ºº C¾ C ºº CH

f
d) CH3CHBrCOOH
(c) H2C==C==C==CH2 (d)
½½

36. Identify the correct order of boiling points of the following


½½
½

ks
compounds:
Yo
34. HOOC
CH3CH2CH2CH2OH (1) CH3CH2CH2CHO (2)
oo
OH CH3CH2CH2COOH (3) (2002)
eB

(a) 1 > 2 > 3 (b) 3 > 1 > 2


(c) 1 > 3 > 2 (d) 3 > 2 > 1
O2N 2 moles of NaNH2
37. Which of the following hydrocarbons has the lowest dipole
r

CH moment? (2002)
ou
ad

(a) cis-2-butene (b) 2-butyne


(c) 1-butyne (d) H2C==CH—CººCH
Y

OH
The product A will be (2003) 38. The correct order of basicities of the following compounds is
– NH
OOC H3C¾ C CH3CH2NH2
nd
Re

NH2 (2 )
OH (1) (2001)
Fi

(CH3 )2 NH CH3CONH2
(3 ) (4 )
(a) O 2N (a) 2 > 1 > 3 > 4 (b) 1 > 3 > 2 > 4
CH (c) 3 > 1 > 2 > 4 (d) 1 > 2 > 3 > 4
39. Among the following, the strongest base is (2000)
– (a) C6H5NH2
O

OOC (b) p-NO2C6H4NH2
OH (c) m-NO2—C6H4NH2
(d) C6H5CH2NH2

(b)
40. Which of the following, has the most acidic hydrogen?
O 2N (2000)
– (a) 3-hexanone
C (b) 2, 4-hexanedione
(c) 2, 5-hexanedione
(d) 2, 3-hexanedione
OH
Organic Chemistry Basics 293

41. The most unlikely representation of resonance structures of 47. The number of sigma and pi-bonds in 1-butene 3-yne are
p-nitrophenoxide ion is (1999) (1989, 1M)
– – (a) 5 sigma and 5 pi (b) 7 sigma and 3 pi
O O O O
+
(c) 8 sigma and 2 pi (d) 6 sigma and 4 pi
N N
48. The compound which gives the most stable carbonium ion
on dehydration is (1989, 1M)
(a) (b) CH3
½
(a) CH3 ¾ CH ¾ CH2OH (b) CH3 ¾ C¾ OH
½ ½
O– O– CH3 CH3
O – O O –
O– OH

w
+ + ½
N N (c) CH3CH2 ¾ CH2 ¾ CH2OH (d) CH3 ¾ CH ¾ CH2CH3
49. Polarisation of electrons in acrolein may be written as
(c) (d) (1988, 1M)

Flo
d+
O O
d- ½½ d- ½½
O– O

ree
(a) H2C== CH ¾ C ¾ H (b) H2C== CH ¾ C ¾ H
d+
42. Among the following compounds, the strongest acid is d-
O O

F
(1998)
(a) HCººCH (b) C6H6 d- d+ ½½ d+ ½½
(c) H2C== CH ¾ C ¾ H (d) H2C== CH ¾ C ¾ H
(c) C2H6 (d) CH3OH

or
43. In the following compounds
ur (1997)
50. The bond between carbon atom (1) and carbon atom (2) in
compound

f
OH OH OH OH Nºº C ¾ CH== CH2
1 2
ks
involves the hybridisation as (1987, 1M)
Yo
(a) sp2 and sp2 (b) sp3 and sp
oo
(c) sp and sp2 (d) sp and sp
NO 2
eB

CH 3 NO 2
Objective Questions II
(I) (II) (III) (IV ) (One or more than one correct option)
r

The order of acidity is 51. Among P, Q, R and S, the aromatic compounds(s) is/are
ou
ad

(a) III > IV > I > II Cl


(b) I > IV > III > II
Y

AlCl3 NaH
(c) II > I > III > IV P Q
(d) IV > III > I > II
nd
Re

O
44. What is the decreasing order of strength of the bases? (NH4)2CO3 HCl
S
OH - , NH2- , H ¾ Cºº C - and CH3 ¾ CH2- R
Fi

(1997) 100-115°C
O O
(a) CH3 ¾ CH2- > NH2- > H ¾ Cºº C > OH - -
(2013 Adv.)
(a) P (b) Q (c) R (d) S
(b) H ¾ Cºº C - > CH3 ¾ CH2- > NH2- > OH -
52. The hyperconjugative stabilities of tert-butyl cation and
(c) OH - > NH2- > H ¾ Cºº C - > CH3 ¾ CH2- 2-butene, respectively, are due to (2013 Adv.)
(d) NH2- > H ¾ Cºº C > OH > - -
CH3 ¾ CH2- (a) s ® p (empty) and s ® p* electron delocalisations
(b) s ® s* and s ® p electron delocalisations
45. The hybridisation of carbon atoms in C ¾ C single bond (c) s ® p (filled) and s ® p electron delocalisations
H ¾ C ºº C ¾ CH==CH2 is (1991, 1M)
(d) p (filled) ® s* and s ® p* electrons delocalisations
(a) sp3 - sp3 (b) sp2 - sp3
53. Amongst the given options, the compound(s) in which all
(c) sp - sp2 (d) sp3 - sp the atoms are in one plane in all the possible conformations
(if any), is/are (2011)
46. Amongst the following, the most basic compound is H H H
(a) benzylamine (b) aniline (1990, 1M) (a) C¾ C (b) H ¾ C ºº C¾ C
(c) acetanilide (d) p-nitroaniline H2C CH2 CH2
(c) H2C == C == O (d) H2C == C == CH2
294 Organic Chemistry Basics

54. In the Newman’s projection for 2,2-dimethylbutane 57. Phenol is less acidic than (1986)
X (a) acetic acid (b) p-methoxy phenol
H3C CH3
(c) p-nitrophenol (d) ethanol

H H Assertion and Reason


Y Read the following questions and answer as per the direction
X and Y can respectively be (2010) given below :
(a) H and H (a) Statement I is correct; Statement II is correct; Statement II is
(b) H and C2H5 the correct explanation of Statement I
(c) C2H5 and H (b) Statement I is correct; Statement II is correct; Statement II is
(d) CH3 and CH3 not the correct explanation of Statement I

w
55. The molecules that will have dipole moment are (c) Statement I is correct; Statement II is incorrect
(1992, 1M)
(a) 2, 2-dimethyl propane
(d) Statement I is incorrect; Statement II is correct
(b) trans-2-pentene 58. Statement I p-hydroxybenzoic acid has a lower boiling
point than o-hydroxybenzoic acid.

Flo
(c) cis-3-hexene
Statement II o-hydroxybenzoic acid has intramolecular
(d) 2,2,3,3-tetramethyl butane
hydrogen bonding.

ree
56. The compound in which C uses its sp 3 -hybrid orbitals for 59. Statement I p-nitrophenol is a stronger acid than
bond formation is (2000, 1M) o-nitrophenol.

F
(a) HCOOH (b) (H2N)2 CO
Statement II Intramolecular hydrogen bonding make the
(c) (CH3 )3 COH (d) CH3CHO o-isomer weaker acid than p-isomer.

or
ur
f
Match the Columns
ks
60. Match the reactions in Column I with appropriate types of steps/reactive intermediate involved in these reactions as given in
Yo
Column II. (2011)
oo

Column I Column II
eB

H3C O O
O
A. p. Nucleophilic substitution
r

aq NaOH
ou
ad
Y

18
O O
B. ½½ 18
CH2CH2CH2OH
H2SO4 q. Electrophilic substitution
nd
Re
Fi

O O
½½
C. CH2CH2CH2Cl CH3MgI r. Dehydration
CH3

CH2CH2CH2C(CH3)2
D. H2SO4 s. Nucleophilic addition
OH

H3C CH3 t. Carbanion


Organic Chemistry Basics 295

61. Match the reaction in Column I with appropriate options in Column II. (2010)

Column I Column II
p. Racemic mixture
—N2Cl + —OH
A.
0°C NaOH/H2O

—N N— —OH

OH OH O q. Addition reaction
½ ½ H 2SO 4 CH3
B. H3C — C — — C — CH3 ¾¾¾®

w
C
½ ½ H3C C—CH3
CH3 CH3 CH3

O r. Substitution reaction

Flo
1. LiAlH4
—–C
2. H3O +
CH3
C.

ree
OH
—–HC
CH3

F
s. Coupling reaction
SH— —Cl Base S
D.

or
ur t. Carbocation intermediate

Fill in the Blanks


f
ks
Integer Answer Type Questions
Yo
62. The kind of delocalisation involving sigma bond orbitals is
oo
called…… (1994, 1M)
73. The total number of contributing structures showing
hyperconjugation (involving C — H bonds) for the
63. The bond dissociation energy needed to form the benzyl
eB

following carbocation is
radical from toluene is ……… than the formation of the (2011)
methyl radical from methane. (1994, 1M) H 3C Å CH2CH3
64. The structure of the enol form of ½
r
ou
ad

CH3 ¾ CO ¾ CH2 ¾ CO ¾ CH3


Y

with intermolecular hydrogen bonding is ………


(1993, 1M) 74. Amongst the following, the total number of compounds
soluble in aqueous NaOH is (2010)
65. The IUPAC name of succinic acid is ………
nd

(1990, 1M)
Re

H3C CH3
66. The shape of (CH3 )+ is ……… (1990, 1M) N COOH NO2
OH
Fi

67. A…… diol has two hydroxyl groups on ……carbon atoms.


(1985, 1M)
68. The terminal carbon atom in butane is …… hybridised.
(1985, 1M)
N
69. ………ring is most strained. (cyclopropane, cyclobutane, H 3C CH3
cyclopentane) (1981, 1M) OCH2CH3 OH CH2CH3
2
70. The compound having both sp and sp -hybridised carbon CH2OH CH2CH3
atoms is ……. (propane, propene, propadiene). (1981, 1M)
COOH
71. In acidic medium, ……….. behaves as the strongest base.
(nitrobenzene, aniline, phenol) (1981, 1M)

72. Among the given cations, ……… is most stable.


(sec-butyl carbonium ion, tert-butyl carbonium ion, n-butyl
carbonium ion) (1981, 1M)
296 Organic Chemistry Basics

Subjective Questions p-methylbenzoic acid 36.2 ´ 10 -5


p-methoxybenzoic acid 10.2 ´ 10 -5 (2003)
75. Which of the following is more acidic and why?
+ +
79. Give reasons for the following:
NH3 NH3 CH2 ==CH - is more basic than HCºº C - . (2000)

80. Explain, why o-hydroxybenzaldehyde is a liquid at room


or temperature while p-hydroxybenzaldehyde is a high melting
solid? (1999)

81. Discuss the hybridisation of carbon atoms in allene (C3 H4 )


F OH (2003)
and show the p-orbital overlaps. (1999, 3M)
76. Draw the resonating structures of . (2003)
82. Give reasons for the following in one or two sentences.

w
77. You have an ether solution containing 4-hydroxybenzoic The central carbon-carbon bond in 1, 3-butadiene is shorter
acid and 4-aminobenzoic acid. Explain, how will you than that of n-butane. (1998)
separate the two in not more than 3 steps? Give
83. Although phenoxide ion has more number of resonating
confirmatory tests with reagents and conditions for

Flo
structures than benzoate ion, benzoic acid is a stronger acid
functional groups of each. (2003)
than phenol. Why? (1997)
78. Match the following with their K a values

ree
84. Arrange the following in the order of their increasing
Benzoic acid 4.2 ´ 10 -5 basicity.
p-nitrobenzoic acid 3.3 ´ 10 -5 p-toluidine, N,N-dimethyl-p-toluidine, p-nitroaniline,

F
p-chlorobenzoic acid 6.4 ´ 10 -5 aniline. (1985, 1M)

or
ur
Answers
Topic 1 17. (d)
f18. (c) 19. (c) 20. (b)
ks
1. (d) 2. (c) 3. (b) 4. (c) 21. (c) 22. (b) 23. (d) 24. (a)
Yo
oo
5. (d) 6. (a) 7. (b) 8. (a) 25. (b) 26. (d) 27. (a) 28. (d)
9. (c) 10. (c) 11. (d) 12. (d) 29. (c) 30. (c) 31. (a) 32. (a)
eB

13. (a) 14. (c) 15. (b) 16. (a) 33. (a) 34. (a) 35. (c) 36. (b)
17. (a) 18. (b) 19. (b) 20. (a) 37. (b) 38. (b) 39. (d) 40. (b)
21. (b) 22. (d) 23. (c) 24. (a) 41. (b) 42. (d) 43. (d) 44. (a)
r
ou

25. (d) 26. (b,c) 27. (a,c,d) 28. (a,b,c) 45. (d) 46. (a) 47. (b) 48. (b)
ad

29. (a,d) 30. (b,c,d) 31. (a,d) 32. (a,c,d) 49. (d) 50. (c) 51. (a,b,c,d) 52. (a)
Y

33. (a,c) 34. (c,d) 35. (b,d) 36. (d) 53. (b,c) 54. (b,d) 55. (b,c) 56. (c,d)
37. (7) 38. (c) 57. (a,c) 58. (d) 59. (a)
nd

60. A ® r, s, t; B ® p, s; C ® r, s; D ® q, r
Re

39. Non-superimposable, Enantiomers 40. False


41. False 42. (2) 43. (3) 44. (7) 61. A ® r, s; B ® t; C ® p, q; D ® r
Fi

62. hyperconjugation
Topic 2
63. less 64. cyclic 65. butanedioic acid
1. (c) 2. (b) 3. (c) 4. (b)
66. triangular planar 67. geminal, same
5. (b) 6. (b) 7. (a) 8. (d)
68. sp3 69. cylopropane 70. propene 71. aniline
9. (a) 10. (d) 11. (b) 12. (d)
13. (c) 14. (a) 15. (b) 16. (d) 72. tert-butyl carbonium ion, 73. (6) 74. (4)
Hints & Solutions
Topic 1 Nomenclature and Isomerism While naming the compound, the longest chain that have
1. The IUPAC name for the given compound is principal functional group ¾ COOH is choosen and numbered
3, 5-dimethyl-4-propylhept-1-en-6-yne. in such a manner that the principal functional group gets the
CH3 lowest possible number. ¾ OH act as substituent and used as
H3 C 7 CH prefix in nomenclature.
5
Longest possible
4 6 chain containing 5. While naming the compound, alkene gets priority over
2 3
double and functional group ( ¾ Br ) and numbering starts from alkene side.
CH3 triple bond
1 Hence, IUPAC name: 4-bromo-3-methyl pent-2-ene
CH2

w
1
CH3 CH3
If both double and triple bonds are present in the compound, the
2 H
endings like-en-yne, a (numeral) dien-(numeral)-yne etc., are 3
4
used. Numbers as low as possible are given to double and triple H

Flo
bonds as a set. CH3 Br
5
2. The IUPAC name of the given compound is
6. 1

ree
NO2
COOH
2
4 H OH
Incorrect

3
Correct

3 H Cl

F
way

way

4
2 CH3

Cl
1 For C-2, order of priority of substituents is

or
CH3
ur OH > CH(Cl) (CH3 ) > COOH

f
2- chloro-1-methy l- 4-nitrobenzene For C-3, order of priority of substituents is
Here, the given compound contains two or more functional Cl > CH(OH)COOH > CH3
ks
groups. So, the numbering is done in such a way that the sum of
Hence, according to CIP rules,
Yo
the locants is the lowest.
oo
O OH 3
3. COOH
1
eB

H 2 1
OH 2S
Acetone No additional stability H Cl 3R
with the ‘enol’ form
3
H CH3
O O
r

O O OH
ou
ad

5
Acetylacetone
This enol-form is highly stable
CN has the highest priority.
Y

because of intramolecular 7. 1
H-bonding (6-membered stability)
and due to extended conjugation. 2 CN
nd

Br
Re

O O (–R) (2-bromo-5-hydroxybenzonitrile)
O O
Fi

O Br
1 2
Ethyl acetoacetate 1 2 NH2 H
(EAA)
CH3 C
H H 3C
In both of the compounds, C -2 of C == O group is a part of the acid Br2
C==C CCl4
C H
O O 8. CH3
½½ ½½
H3C H
Br
derivative (ester, ¾ C ¾ OEt and acid amide, ¾ C ¾ NH2). So,
CH3
C2 O does not take part in enolisation, because it is already in
H Br
resonance (–R) with the derivative group itself.
H Br
4. The IUPAC name of the given compound is CH3
3-hydroxy-4-methylpentanoic acid. (meso)
CH3 OH O
½ 4 ½½
5 ½3 2 1 9. C6H5 ¾ C ¾ Cl : Benzene carbonyl chloride
H3 C ¾ C H ¾ C H ¾ C H2 ¾ C OOH
Principal chain
298 Organic Chemistry Basics
1
CH3 H 19. Compounds with same empirical formula but different
molecular formula have same percentage composition of
H H H H elements but different molecular weight.
3 120° 3
10. 2 2 H H
Anti-clockwise 1
H H H ½ ½
H3C
20. H— C — C — H ; Ethane has the smallest sized group (H)
CH3 CH3 ½ ½
H (gauche) H H
½* bonded to carbons, hence there will be least hindered rotation
11. CH3 — CH2 — C ¾ COOH : Has a chiral carbon, optically active. about C—C bond.
½
CH3 1 2 3 4
(2-methylbutanoic acid) 21. CH2 == CH— CH — CH3 : 3-methyl-1-butene
½

w
F Cl F I CH3
12. C == C + C == C Geometrical isomers
22. Ethers and alcohols (saturated, acyclic) with same number of
Br I Br Cl carbons are always isomeric.

Flo
F Br F I CH3 — O— CH3 and CH3 — CH2 — OH are functional isomers.
C == C + C == C Geometrical isomers

ree
23. Double bond has preference over alkyl group hence :
Cl I Cl Br
CH 3
F Cl F Br
½3 2

F
4 1
C == C + C == C Geometrical isomers H3 C— C ¾ CH == CH2 : 3,3-dimethyl-l-butene
I Br I Cl ½
CH3

or
H3C CH2 — Ph H
ur CH2 — Ph H3 C CH3 H CH3

f
13. C == C C == C
24. C== C C== C
ks
H H H3C H
cis trans H H H3 C H
Yo
cis-2- butene trans-2- butene
oo
1-phenyl-2-butene
14. The ‘D’ term in name is derived from D-glyceraldehyde. 25. Diethyl ether (C2H5OC2H5 ) will be isomeric with all 4-carbon
eB

15. M saturated alcohols. Butanone (CH3CH2COCH3 ) is unsaturated,


CH3 CH3 CH3 has two hydrogen less than the diethylether.
H OH HO H H OH 26. Since, there is no principal functional groups, numbering of
r

HO H H OH H OH disubstituted benzene is done in alphabetical order as


ou
ad

Cl
CH3 CH3 CH3
Y

1
I II III
6 2
I and II are optically active while III is optically inactive
(meso form ).
nd

5 3
Re

16. Conformers can be interconverted through rotation about C—C 4


bond. H CH3
Fi

· ·
·O · Hence IUPAC name of this compound is 1-chloro-4-methyl
½ benzene.
17. H— C == C ¾ CH3 :
½ Also, toluene is an acceptable name in IUPAC, hence this
H compound can also be named as 4-chloro toluene.
Enol of acetone
It has 9 s (6 with H, two C—C and one C ¾ O), one p-bond and 27. PLAN This problem is based on structure and nomenclature of
two lone-pairs. organic compound.
Draw structure of each compound and write IUPAC name of the
given compound.
18.
I II Match the molecular formula of given compound with
III molecular formula of compound given in choices.
The combination of names for possible isomeric alcohols with
molecular formula C4H10O is/are
IV V
Organic Chemistry Basics 299

Formula Names M and P are non-superimposable mirror images, hence,


enantiomers.
CH3CH2CH2CH2OH n-butyl alcohol / n-butanol CH3 HO H
/ butan-1-ol HO H
CH3 ¾ CH ¾ CH2 ¾ OH Iso-butyl alcohol / 2-methyl H3C Cl
½ propan-1-ol
CH3 HO H HO
H
Cl
CH3 ¾ CH2 ¾ CH ¾ OH Secondary butyl alcohol /
½ butan-2-ol Q HO H Cl
CH3
CH3 Tertiary butyl alcohol / HO Cl HO H
H OH
½ tert butanol / 2-methyl H OH
CH3 ¾ C ¾ OH

w
propan-2-ol CH3
½ CH3
CH3 Q
Hence, choices (a), (c) and (d) are correct. M and Q are non-identical they are distereomers.

Flo
28. Converting all of them into Fischer projection. 29. CH3 CH3 CH3 CH3
HO H Cl

ree
HO OH
HO H
C C H C C OH
Cl
HO C==C C==C
HO H H H HO H

F
H H H H H
CH3 CH3 (I) (II + III)
(meso) (enantiomeric)

or
M
ur M
HO H CH3 CH3 CH3
HO H HO
f C H
HO
C H
ks
CH3
H
OH H OH C==C C==C
Yo
H OH H H
oo
Cl Cl H C H C
N N
eB

Since, M and N have ¾ OH on same side and opposite side CH3 H CH3 OH
(IV + V) (VI)
respectively, they cannot be miror image, they are diastereomers. (enantiomeric) (meso)
Cl
HO H Cl
r

H Total six isomers. In both cis and trans forms, there are two
ou
ad

OH enantiomers each.
Cl HO H
HO HO 30. E, F and G are not resonance structures because movement of
HO H
Y

H H
hydrogen between E and F are involved.
CH3 CH3 CH3
E, F and E, G are tautomers in which E is keto form and both F
nd

O O O
Re

and G are enol form of the same E.


M and O are identical.
NOTE Fischer projection represents eclipse form of Sawhorse F and G are geometrical isomers.
Fi

projection. F and G are distereomers as they are stereo isomers but not
For comparision purpose, similar types of eclipse conformers related by mirror image relationship.
must be drawn i.e. both vertically up or both vertically down.
CH3 H OH 31. The compound is optically active as well as it possesses a
H OH two-fold axis of symmetry.
H 3C Cl O
HO H HO 32. (a) —CH==CH—OH r —CH2 ==C—H
H
Cl
P H OH Cl

Cl H OH (c) ==O r —OH


H
OH H OH
CH3 CH3
O HO
P
300 Organic Chemistry Basics

38. Molecules that are non-superimposable on its mirror image are


(d) ==O r —OH
optically active and known as chiral molecule. However, for
chirality of molecule, presence of chiral centre is not essential,
O O e.g.
H3C CH3
H3C CH3 H3C H
33. (a) C == C + C == C C==C==C
H H H CH3
cis-2- butene trans-2- butene
H H
H H Ph Molecule is chiral but does not possesses any chiral carbon.
H
(c) C == C + C == C 39. Non-superimposable, Enantiomers.
Ph CH3 H CH3

w
cis-1-phenylpropene trans-1- phenylpropene
True / False
34. A carbon bonded to four different atoms or groups is called 40. It has only two asymmetric carbon, carbon no.3 is not
asymmetric.
asymmetric carbon :

Flo
Cl H 41. They are identical.
½ ½*
(c) CH3 — C*— D (d) BrCH2 — C— CH3 42. Although the compound has two chiral carbons (indicated by

ree
½ ½ stars), it does not has four optically active isomers as expected.
H OH It is due to its existence in cis-form only.

F
CH3
35. Carbonyl compounds containing a-H show keto-enol CH3 CH3
tautomerism. H3 C
O OH *

or
½½ ½
ur *
(b) C6H5 ¾ C ¾ CH3 r C6H5 — C == CH2

f
H3 C CH3
O O O O
ks
½½ ½½ cis-form trans-form
(d) C6H5 ¾ C ¾ CH2 ¾ C ¾ CH3 r
Yo
(only hypothetical)
O OH
oo
½½ ½ The above shown transformation does not exist due to restricted
C6H5 ¾ C ¾ CH== C— CH3 rotation about the bridge head carbons, hence only cis-form and
eB

CH3 CH3 its mirror image exist.


½ Cl 2 ½ 43. PLAN This problem can be solved by using concept of conformational
36. CH3 — C — CH3 ¾¾® CH3 — C — CH 2Cl analysis of given organic compound. To solve the question
r

½ hn ½ draw the stable conformational structures of organic


ou

H H
ad

CH3 compound and determine the net resultant dipole moment.


½ The conformations of the given compound are as follows
Y

+ CH3 — C — CH3
Cl
½
Cl Br Cl
Cl CH3 Br
nd
Re

37. (7) As given in the question 3 stereocentres are visible, i.e. Br CH3 Cl Br
= =
Br Cl Br Cl
OH OH
Fi

CH3 CH3
H3C CH3
* *
H H H
* 1
½ ½ ½ 5
HO C H3 ¾ C*2 ¾ C*3 ¾ C*4 ¾ CH3
OH
½ ½ ½
Hence, the total number of stereoisomers = 23 = 8 Cl Cl Cl
But out of these the following one is optically inactive due to Cl CH3 Br
symmetry
Br Cl Br Cl Br Cl
OH OH

CH3 Br Br Cl Cl CH3
CH3 CH3 CH3
(Me-Me)gauche (Br-Me)gauche (Cl-Me)gauche
HO
OH
Hence, total number of optically active stereoisomers=7
Organic Chemistry Basics 301

These three have non-zero dipole moment due to 48. I and III are mirror images of one another as well as they are
non-cancellation of all dipole moment created by C—Cl and non-superimposable while II is a meso form.
C—Br bond. Þ (I + III) = Enantiomers
44. CH2CH3 CH3 CH3 I + II and II + III = Diastereomers
CH3 OH
49. O

CH3 CH3 H CH3 Mirror CH3 H


enol keto

CH3 CH3
H H CH3 H H
½ 2 1

w
meso 50. H3C— N — C—
3
CH2 — CH3
pair of enantiomers ½ ½ 5
Þ Total seven isomers CH3 CH2CH3
4

Flo
45. (a) 3-(N, N-dimethyl)-3-methyl pentanamine
Z Z or 3-methyl-3-(N, N-dimethyl) pentanamine
H H H Z 4 Me

ree
2
5 1
Me 3 Me
H H H H 51. 8 10
Me Me

F
6
Z H 7 9
5,6-diethyl-3-methyl decane
anti gauche

or
Mole fraction of anti form = 0.82
ur 5 4 3
52. CH3 CH2 — CH== CH— COOH
2 1

f
Mole fraction of gauche form = 0.18 2-pentenoic acid or pent-2-enoic acid
m obs = 1 D
ks
H Cl Cl Cl Cl H
Þ 1 = m anti ´ 0.82 + m gauche ´ 0.18
Yo
53. C== C C== C C== C
oo
Q m anti = 0 Þ 1 = m gauche ´ 0.18 H Cl H H H Cl
1 I II III
Þ m gauche = = 5.55 D
eB

018
. III is non-polar, has zero dipole moment.
H 54. CH3 — CH2 — CH2 — CH2OH CH3 — CH2 — CH — CH3
CH3 O ½
r

H D O OH
(b) D
ou

H enantiomeric ( ± )
ad

OH
½
D H
Y

H D CH3 — C ¾ CH3 ; CH3 — CH — CH2 OH


H ½ ½
CH3 CH3 CH3
nd

(i) (ii)
Re

Total number of isomers (including stereoisomers) = 5


Structure (ii) is more stable than its anti conformer because of
intramolecular H-bonding.
Fi

Topic 2 General Organic Chemistry


CH3
1. Hydrocarbon containing C and H upon burning produces CO2
H CH3 and water vapour respectively. The equation is represented as
46. (a) CxH y + (x + y / 4 )O2 ¾® xCO2 + ( y / 2)H2O
H H 12
Mass of carbon = ´ mass of CO2
44
H
less stable staggered form of butane 12
= ´ 88 g = 24 g
(b) The less stability of above mentioned conformer is due to 44
2
van der Waals’ repulsion between the adjacent methyl groups. Mass of hydrogen = ´ mass of H2O
18
47. 2
HO OH = ´9 =1g
18
OH
glycerine So, the unknown hydrocarbon contains 24 g of carbon and 1g of
hydrogen.
It contains a secondary (2°) hydroxy group.
302 Organic Chemistry Basics

2. Organic compound ‘A’ contain phosphorus as it gives positive At ‘a’ and ‘e’, N-atoms are sp3 (s% = 25) hybridised, whereas at ‘
test with ammonium molybdate. Phosphorus present in organic b’, ‘c’ and ‘d’, N-atoms are sp2 (s% = 33) hybridised. So, ‘b’, ‘c’
compound ‘A’ get oxidised with Na 2O2 and form Na 3PO4. and ‘d’ are the favourable sites for protonation
2P + 5Na 2O2 ¾® 2Na 3PO4 + 2Na 2O
(H+ is a Lewis acid, i.e. electrons acceptor).
Compound Sodium phosphate
Na 3PO4 in presence of HNO3 form H3PO4 and NaNO3. 7. The correct option is :
Na 3PO4 + 3HNO3 ¾¾® H3PO4 + 3NaNO3 (A) ® (Q); (B) ® (R); (C) ® (S)
Upon cooling, a few drops of ammonium molybdate solution are (A) H2O and sugar mixture They do not react chemically. On
added. A yellow ppt. confirms the presence of phosphorus in the heating, solubility of sugar in H2O increases and on rapid
organic compound. cooling of saturated solution, sugar recrystallises (Q).
H3PO4 +12 (NH4 )2 MoO4 + 21HNO3 ¾® (B) H2O and aniline mixture Aniline is steam volatile but
(NH4 )3 PO4.12MoO3 ¯ +21NH4NO3+12H2O insoluble in H2O. So, steam distillation (R) is employed for

w
Yellow ppt. their separation.
3. Higher the basicity of a base, stronger will be its nucleophilic (C) H2O and toluene mixture Toluene is steam non-volatile
power.
and also insoluble in H2O. So, differential extraction method
Again we know, a weaker acid produces a stronger base

Flo
(S) can be used to separate them.
(conjugate), i.e. a stronger nucleophile.
- HÅ 8. In Dumas method, organic compound is heated with dry cupric
Acid ¾¾® Conjugate base

ree
oxide in a combustion tube in the atmosphere of CO2. Upon
H3O+ H2O heating, C and H present are oxidised to CO2 and water vapours
CH3SO3H CH3SO3È while N 2 is set free. Let, the molecular formula of the organic

F
CH3CO2H CH3COÈ2 compound (1 mol) be C xH yN z. In Dumas method,
Cx H y N z + æç 2x + ö÷ CuO ® xCO2 + H 2 O + N 2 + æç 2x + ö÷ Cu
y y z y
H2O È
OH è ø è ø

or
2 2 2 2
Acid
ur
Base strength
1 mol
y z
strength º Nucleophilicity x mol mol mol

f
2 2
Thus, the correct order of nucleophilicity of given nucleophiles y z
Now, x = 6, = 4 Þ y = 8 and = 1Þ z = 2
ks
are as follows : 2 2
Yo
-
H2O(2) < CH3SO3- (3) < CH3CO-2 (1) < O H(4) \ Molecular formula of the compound is C6H8 N2.
oo

4. In chromatography, the expression of retention factor (R f ) is 9. Dichloromethane, DCM (CH2Cl 2 ) is heavier (density
eB

Distance travelled by the compound from origin = 13266


. g cm-3) than water (density = 1 g cm -3 ). So, DCM and
Rf = <1
Distance travelled by the solvent from origin H2O will stay as lower and upper layer respectively in the
The value of R f signifies the relative ratio of migration of each separating funnel (SF).
r

component of the mixture with respect to the developing solvent 10. We know, a stronger acid produces its stable or weaker conjugate
ou
ad

used. R f value depends on the type of adsorption base. Here, CH(CN)3 produces the most stable conjugate base
chromatography like TLC (Thin-Layer Chromatography), (NC)3 C- . Stronger -R and -I effects of the CN - group, make
Y

paper chromatography etc. The R f value is also the


the carbanion (conjugate base) very stable. The resonance
characteristic of a compound (sample) for a given developing
hybrid structure of [(NC)3 C]- is as follows:
solvent at a given temperature.
nd
Re

When the compound in the sample (usually less polar) is weakly CN


d– d+ –
adsorbed the spot will travel a shorter distance from the origin (NC) C H NC C y +H Å (NC)2C C N
Fi

3
and hence the R f value will be decreased. (–I)
C N (–R)
5. In column chromatography, separation of mixture of
compounds (adsorbate) takes place over a column of solid y
adsorbent (silica gel and Al 2O3 ) packed in a glass tube. N
When an appropriate eluant (liquid) is allowed to flow down the C
column, the compounds present in the mixture get adsorbed to N C C
different extent on the adsorbent column and thus complete C
separation takes place. N
Thus, column chromatography is based on the differential
adsorption of the substance on the solid phase. Resonance hybrid structure of [NC)3C]–
However, halogen (X = Cl, Br, I) show -I effect but + R effect
6. All sites (a, b, c, d, e) of the given molecule have lone pair on
of halogens, destabilises the carbanion, X 3C – (conjugate base of
N-atoms. Higher the ease of donation of lp of electrons of N,
more favourable will be the site for protonation. Ease of the haloform, HCX 3 ).
donation of lp of e s, i.e. Lewis basicity inversely depend on the s
percentage of s-character in hybridisation of ‘N’ which will X2C X (+R,–I )
decide the electronegativity of ‘N’. Thus, CH(CN)3 is the strongest acid among the given options.
Organic Chemistry Basics 303

11. Estimation of nitrogen through Kjeldahl’s method is not suitable ammonium sulphate and ammonium sulphate to sodium
for organic compounds containing nitrogen in ring or nitrogen in sulphate.
nitro or azo groups. It is because of the fact that nitrogen of these (b) Calculate millimoles (m moles) of N present in organic
compounds does not show conversion to Ammonium sulphate compound followed by mass of N present in organic
((NH4 )2 SO4 ) during the process. Hence, among the given compound using the concept of stoichiometry.
compounds only aniline can be used suitably for estimation of (c) At last, calculate % of N present in organic compound using
nitrogen by Kjeldahl’s method. formula
12. Aromatic compounds are stable due to resonance while Mass of N ´ 100
non-aromatics are not. According to Huckel’s rule (or 4 n + 2 % of N =
Mass of organic compound
rule), “For a planar, cyclic compound to be aromatic, its p cloud
must contain (4 n + 2)p electrons, where, n is any whole Mass of organic compound = 1.4 g
number.” Thus, Let it contain x m mole of N atom.

w
Organic compound ¾® NH3
and x m mole
,
O N 2NH3 + H2SO4 ¾® (NH4 )2 SO4 …(i)
(6pe– system) (4pe– + 2e– system) (6pe– system) 6 m mole

Flo
initially taken
are aromatic and stabilised by resonance.
They follow Huckel’s rule. H2SO4 + 2 NaOH ¾® Na 2SO4 + 2 H2O ...(ii)

ree
2 m mole NaOH reacted.
is non-aromatic, hence, Hence, m moles of H2SO4 reacted in Eq. (ii) = 1
least stabilised by resonance.

F
Þ m moles of H2SO4 reacted from Eq. (i) = 6 - 1
O = 5 m moles

or
ur
13. Glycerol with high boiling point (290°C) can be separated from Þ m moles of NH3 in Eq. (i) = 2 ´ 5 = 10 m moles
Þ m moles of N atom in the organic compound

f
spent lye by distillation under reduced pressure. This process is
used to purify liquids having very high boiling points. By this = 10 m moles
ks
process, liquid is made to boil at lower temperature than its Þ Mass of N = 10 ´ 10-3 ´ 14 = 0.14 g
Yo
boiling point by lowering the pressure on its surface.
oo
14. ¾ OH group displays both kinds of effect; an electron Mass of N present in
organic compound
withdrawing acid-strengthening inductive effect from the % of N = ´100
eB

meta-position and an electron-releasing acid weakening Mass of organic compound


resonance effect from the para-position (at this position, 0 .14
resonance effect overweighs the inductive effect). Þ % of N = ´ 100
1.4
r

Thus, III > IV.


ou

= 10%
ad

o-hydroxybenzoic acid (II) is far stronger than the


corresponding meta and para isomers as the carboxylate ion is 16. 18 g H2 O contains 2g H
Y

stabilised by intramolecular H-bonding.


2,6-dihydroxybenzoic acid (I) forms carboxylate ion which is \ 0.72 g H2O contains 0.08 g H
further stabilised by intramolecular H-bonding, Thus, correct 44 g CO2 contains 12 g C
nd
Re

order is \ 3.08 g CO2 contains 0.84 g C


I > II > III > IV 0.84 0.08
Fi

d- \ C:H = :
O O d+ d- d- 12 1
C d+ O O d+
H H C H = 0.07 : 0.08 = 7 : 8
O O O \ Empirical formula = C7 H8
17. The order of stability of carbocation will be
CH2+
II I
(Most stable)

15. THINKING PROCESS This problem is based on the estimation of > CH2 CH CH2 > CH3 CH2 CH2
percentage of N in organic compound using Kjeldahl’s method. Use
the concept of stoichiometry and follow the steps given below to solve Benzyl Allyl Propyl
the problem. ( resonance ( resonance (stabilised by
(a) Write the balanced chemical reaction for the conversion of N stabilised with more stabilised) +I effect)
present in organic compound to ammonia, ammonia to canonical forms)
(III) (I) (II)
304 Organic Chemistry Basics

OH OH OH OH 22. I is most stable because it has more covalent bonds and negative
charge on electronegative nitrogen. III is more stable than II and
IV due to greater number of covalent bonds. Between II and IV,
18. > > >
II is more stable since, it has negative charge on electronegative
atom and positive charge on electropositive atom. Hence,
NO2 Cl CH3 OCH3 overall stability order is
(–M, –I ) ( –I ) (+I, (+M) I > III > II > IV
(hyper conjugation)
(III) (I) (II) (IV) 23. H at C 2 will migrate giving resonance stabilised carbocation.
Electron releasing group decreases while electron withdrawing H H
group increases acidic strength by destabilising and stabilising ½ + ½ H-
+

the phenoxide ion formed respectively. H3C ¾ C ¾ C ¾ C ¾ CH3 ¾® H3C ¾ C ¾ CH2CH(CH3 )2


shift
½ ½ ½ ½

w
SbCl 5 Å OH H CH 3 OH
19. Cl ¾ CH ¾ CH3 ¾¾® Ph ¾ CH ¾ CH3 + SbCl6-
½ Toluene (Carbocation) ¬® H3C ¾ C ¾ CH2 ¾ CH(CH3 )2
Ph (planar) ½½
+

Flo
(- ) OH
¾® Ph ¾ CH ¾ CH3
½ —OH < Cl— —OH <

ree
Cl 24.
( d and l ) mixture
I II
+ SbCl 5

F
20. Allene is the name given to propdiene, H2C == C == CH2 . H3C— —COOH < —COOH
Allene

or
ur
Hybridisation of an atom is determined by determining the
IV III
number of hybrid orbitals at that atom which is equal to the + I effect

f
number of sigma (s) bonds plus number of lone pairs at the decreases acid strength
ks
concerned atom.
Cl has overall electron withdrawing effect, increases acid
Pi(p ) bonds are not formed by hybrid orbitals, therefore, not
Yo
strength.
counted for hybridisation.
oo

H p H
eB

H s H –
s s
C C C s 25. H—C—CH—CH2 H—C==CH—CH2
s
H s H +
H H
r
ou

26. The s-electron of C ¾ H bond is delocalised with p-orbitals of


ad

Here, the terminal carbons have only three sigma bonds p-bond.
Y

associated with them, therefore, hybridisation of terminal carbons


is sp3. The central carbon has only two sigma bonds associated, + 3°, has resonance
O O stability
hence hybridisation at central carbon is sp. +
nd

I
Re

21. A mono-substituted benzoic acid is stronger than a +


mono-substituted phenol as former being a carboxylic acid. has no resonance stability but 2°
Fi

Among the given substituted benzoic acid, ortho-hydroxy acid II


is strongest acid although —OH causes electron donation by
+
resonance effect which tends to decrease acid strength. 2°, has resonnce
O O
It is due to a very high stabilisation of conjugate base by + stability
III
intramolecular H-bond which outweigh the electron donating
resonance effect of —OH.
+ 1°, has no resonance stability
O
IV
COOH –
O Therefore, overall stability order is : I > III > II > IV
H+ +
H 27. The following structure has like charge on adjacent atoms,
OH O therefore, least stable
The overall order of acid-strength of given four acids is + O
+
ortho-hydroxybenzoic acid (pKa = 2.98) > Toluic acid N
-
pKa = 4.37) > p-hydroxybenzoic acid (pKa = 4.58) >
p-nitrophenol (pKa = 7.15) O-
Organic Chemistry Basics 305

28. —SO3Na + H2O


35. Weakest acid BrCH2CH2COOH has smallest dissociation
—SO3H + NaHCO3
constant.
+ CO2
36. Butanoic acid forms more exhaustive H-bonds than butanol and
O2N— —OH + NaHCO butanal do not form intermolecular H-bonds. Hence, boiling
3
point order will be 3 > 1 > 2.

O2N— —ONa + H2O + CO2 37. CH3 ¾ C ºº C ¾ CH3


symmetric

29. A spontaneous neutralisation will occur between strong acid and 38. I is most basic due to formation of resonance stabilised
strong base as conjugate acid .
+
NH NH2

w
+
H3C— H3C ¾ C H
¾® H3C ¾ C
—SO3H + CH3COONa
strong base NH2 N H2
strong acid ··
NH2

Flo
– + ¬® H3C ¾ C
H3C— —SO3Na + CH3COOH
weak acid NH2
+

ree
weak base resonance stabilised
+ -
conjugate acid
30. H2C== CH ¾ CH ¾ CH ¾ O ¾ CH3 IV is amide, least basic.

F
-
Lone pair of oxygen is not the part of this mode of O O
delocalisation. ½½ · · ½ +
CH3 ¾ C ¾ N H2 ¬® CH3 ¾ C == NH2

or
+
ur
31. Carboxylic acid is stronger acid than ammonium ion, hence
+
lone pair is part of delocalisation
—COOH(X) is most acidic. Z (NH3 ) is more acidic than Y (NH3 )

f
Also, among alkyl amines, 2° is more basic than 1° amine.
due to – I effect of —COOH on Z. Hence, overall acid strength Hence, overall order of basic strength is
ks
order is 1>3>2>4
Yo
X > Z > Y
oo
39.
32. CH3Cl has highest dipole moment. —CH2—NH2
eB

33. H2C ==CH ¾ C ºº N


­ ­2 ­ ­ Lone pair is not taking part in resonance, most basic. In other
sp2 sp sp sp cases, lone pair of nitrogen is part of delocalisation which
decreases Lewis base strength.
r

34. In general, the order of acid strength is


ou
ad

¾ C ºº CH < ROH < H2O < PhOH < R ¾ COOH


40. A methylene ( ¾ CH2 ¾ ) with carbonyl on both side is highly
Therefore, during stepwise neutralisation of given acid, acidic.
Y

—COOH will be neutralised first. O O


In the second step, the phenolic —OH, assisted by – I effect of ½½ ½½
nd

CH3 ¾ C ¾ CH2 ¾ C ¾ CH2 ¾ CH3


Re

¾ NO2 at meta position will be neutralised. *


­
HOOC has very acidic H.
Fi

OH O

41. O— ==N
O2N 2 moles of O
CH NaNH2 octet of nitrogen is violated.

– 42. Although alcohols are weaker acid than water, it is stronger than
OOC ammonia and terminal alkynes.
OH OH
43. Nitro group from para position exert electron withdrawing
+ 2NH3 resonance effect, increases acidity of phenol the most. This is
followed by meta nitrophenol in which nitro group exert
O 2N
electron withdrawing effect on acidity. CH3— is an electron
CH donating group, decreases acid strength. Hence, the overall
order is

O– IV > III > I > II


306 Organic Chemistry Basics

44. CH3CH-2 > NH-2 > H ¾ C ºº C - > HO- n (4 n + 2) p electrons


It is because the order of acid-strength of their conjugate acid is P – 0 2 2
CH3CH3 < NH3 < H ¾ C ºº C ¾ H < H2O : Acid strength. Q – 1 6 6 (including lone pair)
R – 1 6 6 (including lone pair on N)
45. H ¾ Cºº C ¾ CH == CH2
S – 1 6 6
­ ­
sp sp3 In all cases there is complete delocalisation of p-electrons.
52. PLAN Spreading out charge by the overlap of an empty p-orbital with
46. —CH2—NH2 an adjacent s-bond is called hyperconjugation. This overlap
(the hyperconjugation) delocalises the positive charge on the
carbocation, spreading it over a larger volume, and this
benzylamine
stabilises the carbocation.
Lone pair is not involved in resonance, most basic. In all other Vacant

w
cases, lone pair of nitrogen is involved in resonance, less basic. p-orbital
H r
CH3—C—CH2—H CH3—C
r
CH2 H
H ½
47. C == C ¾ C ºº C ¾ H : It has 7 sigma and 3 pi bonds. CH3 CH3

Flo
H tertiary butyl carbocation has one vacant p-orbital, hence, it is
1- butene-3-yne stabilised by s-p (empty) hyperconjugation.

ree
CH3 CH3 Å –
½ +
½ H—CH
s 2—CH==CH—CH
p 3 H CH2 ==CH—CH—CH3
H
48. CH3 ¾ C — OH ¾¾® CH3 ¾ C+ In 2-butene, stabilisation is due to hyperconjugation between

F
- H2 O ½
½ s - p* electron delocalisation.
CH3 CH3
53. In both (b) and (c), all the atoms are present in one single plane

or
(3°, most stable alkyl carbocation)
ur
- sp2
O O Od - H sp2 H

f
½½ + ½ d+ ½½ H—C C—C C C O
49. CH 2 == CH ¾ CH ¬® CH 2CH == CH ¬® CH 2 == CH ¾ C ¾ H H straight
straight
ks
C H
50. N ºº C ¾ CH == CH2
Yo
1 2 H
oo
In (a) 1, 3-butadiene, conformational change is possible
sp sp2 between C2 — C3 bond in which atoms will be present in more
eB

than one single plane.


51. PLAN A species is said to have aromatic character if In (d) allene, the terminals H—C—H planes are perpendicular
(a) ring is planar to one another.
(b) their is complete delocalisation of p-electrons
r

CH3
(c) Huckel rule i.e. ( 4 n + 2) rule is followed.
ou


ad

where, n is the number of rings 54. H3C—CH2—C—CH3


4 3 2 1

( 4 n + 2) = p electron delocalised.
Y

Cl CH3
+ On C2 — C3 axis, X = CH3 , Y = CH3
On C1 — C2 axis, X = H, Y = C2H5
nd
Re


+ AlCl3 + AlCl4
CH3
½
Fi

Aromatic
P 55. H3C ¾ C — CH3 m =0
– ½
CH3
+ NaH + H2 H3C H
C == C m ¹0
Aromatic
Q H C2H5
unsymmetrical
O OH
H5C2 C2H5
r C == C m ¹0
O (NH4)2CO3 HCl
O D, 100–115°C ; H H
N
Aromatic CH3 CH3
H S ½ ½
Aromatic CH3 ¾ C — C — CH3 m =0
R ½ ½
CH3 CH3
symmetrical
Organic Chemistry Basics 307

O O +
O 18 +
OH
½½ ½½ OH H
56. H ¾ C ¾ OH H 2N ¾ C ¾ NH2 C. Ph Ph
­ ­ :OH
Nucleophilic addition
sp2 sp2
CH3 –H+ Ph –H2 O+
O Ph
½ ½½ HO O O
CH3 ¾ C — O ¾ H CH3 ¾ C ¾ H 18
Dehydration
½
sp 3 CH3 sp 3 H
+
D. Ph –H2O
57. Phenol is less acidic than a carboxylic acid (acetic acid). Nitro OH +
group from para position exert electron withdrawing resonance

w
Friedel-
effect, increases acid strength. Therefore, phenol is less acidic Craft alkylation
than p-nitro phenol.
Electrophilic
On the other hand, methoxy group from para position, donate

Flo
substitution
electrons by resonance effect, decreases acid strength of phenol.
Also ethanol is weaker acid than phenol due to resonance
stabilisation in phenoxide ion. 61. +

ree
—N2 + —OH
Hence,
ethanol < p-methoxyphenol < phenol < p-nitrophenol < acetic

F
acid —N==N— —OH
¾¾increasing acid strength ¾¾®

or
ur
58. Statement I is incorrect; Statement II is correct. Intramolecular this is an example of electrophilic substitution at para position
H-bonding in ortho-hydroxy benzoic acid lowers the boiling of phenol, giving a coupling product.

f
point. O H O–
ks
59. Statement I is correct; Statement II is correct; Statement II is the
Ph—C—CH3 + H—Al–—H
Yo
correct explanation of Statement I. Ph—C—CH3
oo

Intramolecular H-bonding discourage release of H+ to some H H


Nucleophilic addition
eB

extent, hence weaker acid than its para isomer.


H OH
O O–
Ph—C—CH3
r

+
N
ou
ad

H
O Pianacol-pinacolone rearrangement, occur through carbocation
Y

intermediate.
60.
O O Nucleophilic addition occur at sp2 (planar) carbon, generating a
nd
Re

A. Ph—C—CH2—CH2—C—CH3 + OH – chiral centre, hence product will be a racemic mixture.


– S
:

Fi

O CH HS— —Cl Base


—S– —Cl
2
O H2O OH + Cl
O
Ph Ph
Nucleophilic
carbanion substitution
–H2O O
Ph 62. Hyperconjugation.
Dehydration 63. Less, stable free radical is formed.
O– 64. Cyclic
O H
CH3MgBr
Cl Ph O O
B. Ph CH
Cl C
Ph O H3C CH3
SN 2
H3C
308 Organic Chemistry Basics

O O 77. OH OH
½½ ½½ OH NH2
65. HO ¾ C ¾ CH2 ¾ CH2 ¾ C ¾ OH
butanedioic acid Distillation

66. Triangular planar; carbon is sp2-hybridised and

67. geminal, same COOH COOH


3 COOH COOH (ether layer)
68. sp -hybridised +
ether
NH3Cl– NH2
69. Cyclopropane: here the C—C—C bond angle is 60° while the solution
requirement is 109°. HCl NaOH(aq)
70. Propene : CH2 == CH ¾ CH3

w
­ ­
sp2 sp3
COOH COOH
(aqueous layer)
71. Aniline

Flo
72. Tert-butyl carbonium ion because the three methyl group
stabilises carbocation by + I effect. Tests of functional groups HOOC— —OH

ree
2
73. These are total 6a-H to sp carbon and they all can participate in FeCl 3
¾¾® violet colouration confirm phenolic group
hyperconjugation.

F
NaHCO3 lime
+
+ ¾¾¾® CO2 ­ ¾¾® milky ¾® — COOH
H water
+ H
NH2 NC
H+

or
ur
+ +

f
+ CHCl3 + KOH – NH2 group confirmed.
Three structures Two structures
ks
74. Aromatic alcohols and carboxylic acids forms salt with NaOH,
Yo
COOH COOH
oo
will dissolve in aqueous NaOH :
characteristic
foul smell of isocyanide
eB

COOH OH OH COOH
78. p-methoxy benzoic acid is the weakest and p-nitrobenzoic acid
is the strongest acid among these acids. Chloro group has overall
electron withdrawing effect on ring, therefore, increases acid
r

strength of benzoic acid. Methyl group decreases acid strength


ou
ad

N of benzoic acid by + I effect. Therefore,


Ka
Y

H3C CH3
p-methoxy benzoic acid 3.3 ´ 10- 5
+ p-methyl benzoic acid 4.2 ´ 10- 5
NH3 6.4 ´ 10- 5
nd

benzoic acid
Re

p-chlorobenzoic acid 10. 2 ´ 10- 5


p-nitrobenzoic acid 36.2 ´ 10- 5
Fi

75. is more acidic due to – I effect of F.


79. H ¾ C ºº C ¾ H is more acidic than CH2 == CH2.
F 80. Intramolecular H-bonding in ortho-hydroxybenzaldehyde
decreases its melting point as well as boiling point.
+ Molecules of p-hydroxybenzaldehyde is symmetrical,
OH O¾H
associated together by intermolecular H-bonds, has higher
+ boiling point and melting point.
- O¾H
76. O
H
- O

H
Intramolecular H-bonding
Organic Chemistry Basics 309

H H H H stable than phenoxide ion in which negative charge also moves


81. C—C—C ºº C==C==C on carbon atoms.
-
H H H H O O
½½ ½
Allene sp sp2 CH3 ¾ C ¾ O- ¬® CH3 ¾ C == O

82. -

+ —O ==O – ==O
-
+

The above shown resonance introduces some double bond ==O


character to central C—C bond. Therefore, the central C—C

bond in 1, 3-butadiene is shorter (stronger) than C—C bond in

w
butane. 84. p-nitroaniline < aniline < p-toluidine < N,N-dimethyl-p-toluidine.
83. In case of acetate ion, both the resonance structures are Nitro group, by electron withdrawing resonance effect decreases
equivalent and negative charge always remains on the basic strength. Methyl group by electron donating inductive
effect, increases basic strength.

Flo
electronegative oxygen. These factors makes acetate ion more

ree
Download Chapter Test
http://tinyurl.com/y6tyd62g or

F
or
ur
f
ks
Yo
oo
r eB
ou
ad
Y
nd
Re
Fi
22
Hydrocarbons

w
5. 1-bromo-3-chlorocyclobutane when treated with two
Topic 1 Saturated Hydrocarbons equivalents of Na, in the presence of ether which of the
Objective Questions I (Only one correct option) following will be formed? (2005, 1M)

Flo
Br Cl
1. In the following skew conformation of ethane,
H¢ ¾ C ¾ C ¾ H¢¢ dihedral angle is (2019 Main, 12 April II)

ree
(a) (b)
H
H H¢ 29°

F
H
(c) (d)
H¢¢ H

or
6. How many chiral compounds are possible on mono
(a) 58° (b) 149°
ur
(c) 151° (d) 120° chlorination of 2-methyl butane ? (2004, 1M)

f
(a) 2 (b) 4
2. Which of these factors does not govern the stability of a
(c) 6 (d) 8
ks
conformation in acyclic compounds? (2019 Main, 10 April II)
7. Consider the following reaction
Yo
(a) Electrostatic forces of interaction
oo
(b) Torsional strain ·
(c) Angle strain H3 C ¾ CH ¾ CH ¾ CH3 + Br ¾® X + HBr
½ ½
eB

(d) Steric interactions


D CH3
3. Isomers of hexane, based on their branching, can be divided
into three distinct classes as shown in the figure. (2014 Adv.) Identify the structure of the major product X (2002, 3M)
r

· ·
(a) H3C¾ CH¾ CH ¾ CH2 (b) H3C¾ CH¾ C ¾ CH3
ou
ad

and ½ ½ ½ ½
I. and II. D CH3 D CH3
Y

· ·
(c) H3C¾ C ¾ CH ¾ CH3 (d) H3C¾ CH ¾ CH ¾ CH3
III. ½ ½ ½
D CH3 CH3
nd
Re

The correct order of their boiling point is 8. Benzyl chloride (C6H5CH2Cl) can be prepared from toluene
Fi

(a) I > II > III by chlorination with (1998)


(b) III > II > I (a) SO2Cl2 (b) SOCl2
(c) II > III > I (c) Cl2 (d) NaOCl
(d) III > I > II
9. (CH3)3CMgCl on reaction with D2O produces (1997)
CH3 Cl 2 , hv (a) (CH3)3CD (b) (CH3)3OD
4. ¾¾¾® N (isomeric products) C5 H11 Cl
(c) (CD3)3CD (d) (CD3)3OD
H3C 10. When cyclohexane is poured on water, it floats because
(a) cyclohexane is in ‘boat’ form (1997, 1M)
CH3
(b) cyclohexane is in ‘chair’ form
Fractional distillation (c) cyclohexane is in ‘crown’ form
¾¾¾¾¾¾¾® M (isomeric products) (d) cyclohexane is less dense than water
What are N and M ? (2006, 5M)
11. The C ¾ H bond distance is the longest in (1989, 1M)
(a) 6, 6 (b) 6, 4
(c) 4, 4 (d) 3, 3 (a) C2H2 (b) C2H4
(c) C2H6 (d) C2H2Br2
Hydrocarbons 311

12. The compound which has one isopropyl group, is (1989, 1M) 15. The compound with highest boiling point is (1982, 1M)
(a) 2,2,3,3-tetramethyl pentane (b) 2,2-dimethyl pentane (a) 2-methyl butane (b) n -pentane
(c) 2,2,3-trimethyl pentane (d) 2-methyl pentane (c) 2, 2-dimethyl propane (d) n-hexane
13. The highest boiling point is expected for (1986, 1M) 16. Marsh gas mainly contains (1980, 1M)
(a) iso-butane (b) n-octane (a) C2H2 (b) CH4
(c) 2, 2, 3, 3-tetramethyl butane (d) n-butane (c) H2S (d) CO
14. Which of the following compounds does not dissolve in
conc. H2 SO4 even on warming ? (1983, 1M)
Integer Answer Type Question
(a) Ethylene (b) Benzene 17. The maximum number of isomers (including stereoisomers)
(c) Hexane (d) Aniline that are possible on mono-chlorination of the following
compound, is CH3 CH2 CH( CH3 ) CH2 CH2 (2011)

w
Topic 2 Unsaturated Hydrocarbons

Flo
Objective Questions I (Only one correct option) 5. Which one of the following alkenes when treated with HCl
1. Consider the following reactions, yields majorly an anti Markownikov product?

ree
Ag2O
(2019 Main, 8 April II)
ppt
D (a) Cl ¾ CH == CH2 (b) H2N ¾ CH == CH2
A (c) CH3O ¾ CH == CH2 (d) F3C¾ CH == CH2

F
Hg2+/H+ NaBH4 ZnCl2
B C Turbidity within
Conc. HCl 5 minutes 6. The correct order for acid strength of compounds
A is CH ºº CH, CH3 ¾ C ºº CH and CH 2 == CH 2 is as follows :

or
(2019 Main, 12 April II)
(a) CH ºº CH
ur
(b) CH3 ¾ C ºº C ¾ CH3 (2019 Main, 12 Jan I)

f
(c) CH3 ¾ C ºº CH (d) CH2 == CH2 (a) CH3 ¾ Cºº CH > CH 2 == CH 2 > HCºº CH
2. But-2-ene on reaction with alkaline KMnO4 at elevated (b) CH3 -- C ºº CH > CH ºº CH > CH 2 == CH 2
ks
temperature followed by acidification will give (c) HC ºº CH > CH3 -- C ºº CH > CH 2 == CH 2
Yo
(d) CHºº C H > CH2 == CH2 > CH3 -- Cºº CH
oo
(2019 Main, 12 April I)
7. The trans-alkenes are formed by the reduction of alkynes
(a) CH3 ¾ CH¾ CH ¾ CH3
eB

½ ½ with (2018 Main)


OH OH (a) H2-Pd/C, BaSO4 (b) NaBH4
(b) one molecule of CH3CHO and one molecule of CH3COOH (c) Na/liq. NH3 (d) Sn-HCl
(c) 2 molecules of CH3COOH
r

8. The reaction of propene with HOCl (Cl2 + H2O) proceeds


ou

(d) 2 molecules of CH3CHO


ad

through the intermediate (2016 Main)


3. The major product of the following addition reaction is + +
Y

Cl 2 / H 2O (a) CH3 ¾ CH ¾ CH2 ¾ Cl (b) CH3 ¾ CH(OH) ¾ CH2


H3C ¾ CH == CH 2 ¾¾® + +
(c) CH3 ¾ CHCl ¾ CH2 (d) CH3 ¾ CH ¾ CH2 ¾ OH
(2019 Main, 12 April I)
nd
Re

(a) CH3 ¾ CH ¾ CH2 9. Which of the following compounds will exhibit geometrical
½ ½ isomerism? (2015 Main)
Fi

Cl OH
(a) 1-phenyl-2-butene (b) 3-phenyl-1-butene
(b) CH3 ¾ CH ¾ CH2 (c) 2-phenyl-1-butene (d) 1, 1-diphenyl-1-propane
½ ½
OH Cl 10. Which compound would give 5-keto-2-methyl hexanal upon
O
ozonolysis? (2015 Main)

(c) H3C CH3 CH3


CH3
O (a) (b)
(d)

H3C CH3 CH3


CH3 CH3
4. The major product of the following reaction is
¾ (¾
CH 3C ºº CH ¾(i)¾DCl l equiv.)
¾¾®
(ii) DI (2019 Main, 9 April I) (c) (d)
(a) CH3CD(Cl)CHD(I) (b) CH3CD2CH(Cl)(I)
(c) CH3CD(I)CHD(Cl) (d) CH3C(I)(Cl)CHD2 CH3
312 Hydrocarbons

11. The major organic compound formed by the reaction of Hg 2+


1,1,1-trichloroethane with silver powder is (2014 Main) 20. Ph ¾ C ºº C ¾ CH3 ¾¾® A; A is
H+ (2003, 3M)
(a) acetylene (b) ethene O
(c) 2-butyne (d) 2-butene Ph Ph
(a) (b) ==O
12. The number of optically active products obtained from the
complete ozonolysis of the given compound, is (2012) H 3C H 3C
OH
CH3 H Ph Ph
(c) (d) —OH
CH3 CH CH C CH CH C CH CH CH2 H3C H 3C
21. Identify a reagent from the following list which can easily
H CH3 distinguish between 1-butyne and 2-butyne.

w
(2002, 3M)

(a) 0 (b) 1 (c) 2 (d) 4 (a) bromine, CCl 4 (b) H2 , Lindlar catalyst
(c) dilute H2SO4 , HgSO4 (d) ammoniacal CuCl 2 solution
13. The synthesis of 3-octyne is achieved by adding a
22. In the presence of peroxide, hydrogen chloride and hydrogen

Flo
bromoalkane into a mixture of sodium amide and an alkyne.
iodide do not give anti-Markownikoff’s addition to alkenes
The bromoalkane and alkyne respectively are (2010)
because (2001, 1M)
(a) BrCH2CH2CH2CH2CH3 and CH3CH2C ºº CH

ree
(a) both are highly ionic
(b) BrCH2CH2CH3 and CH3CH2CH2C ºº CH
(b) one is oxidising and the other is reducing
(c) BrCH2CH2CH2CH2CH3 and CH3C ºº CH (c) one of the steps is endothermic in both the cases

F
(d) BrCH2CH2CH2CH3 and CH3CH2C ºº CH (d) all the steps are exothermic in both the cases
14. The number of stereoisomers obtained by bromination of 23. The reaction of propene with HOCl proceeds via the addition

or
trans-2-butene is
ur (2007) of (2001)
(a) H+ in the first step (b) Cl+ in the first step

f
(a) 1 (b) 2
(c) 3 (d) 4 (c) OH– in the first step (d) Cl+ and OH– single step
ks
15. The reagent(s) for the following conversion, 24. Hydrogenation of the adjoining compound in the presence of
Yo
poisoned palladium catalyst gives
oo
Br ?
H H H3 C H
Br CH3
eB

is/are (2007, 3M)


(a) alcoholic KOH
H 3C H
H (2001, 1M)
(b) alcoholic KOH followed by NaNH2 (a) an optically active compound
r

(c) aqueous KOH followed by NaNH2 (b) an optically inactive compound


ou
ad

(d) Zn/CH3OH (c) a racemic mixture


16. CH3 — CH == CH2 + NOCl ¾® P; Identify the adduct. (d) a diastereomeric mixture
Y

(2006, 3M) 25. Propyne and propene can be distinguished by (2000)


(a) CH3 —CH—CH2 (b) CH3— CH—CH2 (a) conc. H2SO4 (b) Br2 in CCl4
nd

½ ½ ½ ½
Re

Cl NO (c) dil. KMnO4 (d) AgNO 3 in ammonia


NO Cl
Cl 26. Which one of the following alkenes will react fastest with H2
Fi

½
(c) CH3—CH2 ¾ CH (d) CH2 —CH2—CH2 under catalytic hydrogenation condition ? (2000, 1M)
½ ½ ½ R R R H
NO NO Cl
(a) (b)
17. Cyclohexene is best prepared from cyclohexanol by which of H H R H
the following? (2005)
R R R R
(a) conc. H3PO4 (b) conc. HCl / ZnCl2 (c) (d)
(c) conc. HCl (d) conc. HBr
R H R R
18. 2-hexyne gives trans-2-hexene on treatment with (2004, 1M)
(a) Li/NH3 (b) Pd/BaSO4
27. The product(s) obtained via oxymercuration
(c) LiAlH4 (d) Pt/H2
(HgSO4 + H2 SO4 ) of 1-butyne would be (1999, 2M)
O
19. 2-phenyl propene on acidic hydration, gives (2004, 1M) ½½
(a) 2-phenyl-2-propanol (a) CH3 ¾ CH2 ¾ C ¾ CH3
(b) 2-phenyl-1-propanol (b) CH3 ¾ CH2 ¾ CH2 ¾ CHO
(c) 3-phenyl-1-propanol (c) CH3 ¾ CH2 ¾ CHO + HCHO
(d) 1-phenyl-2-propanol (d) CH3 ¾ CH2 ¾ COOH + HCOOH
Hydrocarbons 313

28. In the compound, H , the C2-C3 bond is of the (a) (M and O) and (N and P) are two pairs of enantiomers
type (1999, 2M) (b) Bromination proceeds through trans-addition in both the
(a) sp - sp2 (b) sp3 - sp3 (c) sp - sp3 (d) sp2 - sp3 reactions
(c) O and P are identical molecules
29. The reaction of CH3 CH==CH OH with HBr (d) (M and O) and (N and P) two pairs of diastereomers
gives (1998, 2M)
Assertion and Reason
(a) CH3CHBrCH2 OH
Read the following questions and answer as per the direction
given below :
(b) CH3CHBrCH2 Br (a) Statement I is correct Statement II is correct Statement II is
a correct explanation of Statement I.

w
(b) Statement I is correct Statement II is correct Statement II is
(c) CH3CH2CHBr OH
not the correct explanation of Statement I.
(c) Statement I is correct Statement II is incorrect.
(d) CH3CH2CHBr Br (d) Statement I is incorrect Statement II is correct.

Flo
30. Which one of the following has the smallest heat of 38. Statement I Addition of bromine to trans-2-butene yields
hydrogenation per mole ? meso-2, 3-dibromo butane.

ree
(1993, 1M)
(a) 1-butene (b) trans-2-butene Statement II Bromine addition to an alkene is an
(c) cis-2-butene (d) 1, 3-butadiene electrophilic addition. (2001, 1M)

F
31. The number of structural and configurational isomers of a 39. Statement I Dimethyl sulphide is commonly used for the
bromo compound, C5 H9 Br, formed by the addition of HBr to reduction of an ozonide of an alkene to get the carbonyl

or
2-pentyne respectively, are
ur (1988, 1M) compound.
(a) 1 and 2 (b) 2 and 4 (c) 4 and 2 (d) 2 and 1 Statement II It reduces the ozonide giving water soluble
32. Acidic hydrogen is present in (1985, 1M)

f
dimethyl sulphoxide and excess of it evaporates. (2001, 1M)
ks
(a) ethyne (b) ethene (c) benzene (d) ethane
40. Statement I 1-butene on reaction with HBr in the presence
Yo
33. Baeyer’s reagent is (1984, 1M) of a peroxide produces 1-bromobutane.
oo
(a) alkaline permanganate solution
Statement II It involves the formation of a primary radical.
(b) acidified permanganate solution
eB

(2000, 1M)
(c) neutral permanganate solution
(d) aqueous bromine solution 41. Statement I Addition of Br2 to 1-butene gives two optical
34. When propyne is treated with aqueous H2 SO4 in the presence isomers.
r

of HgSO4 , the major product is Statement II The product contains one asymmetric carbon.
ou

(1983, 1M)
ad

(a) propanal (b) propyl hydrogen sulphate


(1998 , 1M)
(c) acetone (d) propanol
Y

35. The compound 1, 2-butadiene has (1983, 1M) Passage Based Questions
(a) only sp-hybridised carbon atoms
nd
Re

(b) only sp2-hybridised carbon atoms Passage 1


(c) both sp and sp2-hybridised carbon atoms Pd-BaSO4 (i) B2H6
Fi

(d) sp, sp2 and sp3-hybridised carbon atoms C8H6


H2
C8H8
(ii) H2O2,NaOH, H2O
X

H2O
36. Which of the following will decolourise alkaline KMnO4 HgSO4,H2SO4
solution? (1980, 1M) C8H8
(i) EtMgBr, H2O
Y
(ii) H+, Heat (2015 Adv.)
(a) C3H8 (b) CH4 (c) CCl 4 (d) C2H4
42. Compound X is
Objective Questions II OH
(One or more than one correct option) (a) CH3 (b) CH3
37. The correct statement(s) for the following addition reactions
is (are) (2017 Adv.)
OH
H3C H (c) (d) CHO
H3C H
Br2/CHCl3 Br2/CHCl3
(i) M and N (i) O and P
H CH3 H CH3
314 Hydrocarbons

43. The major compound Y is (i) NaNH2 (excess)


(ii) CH3CH2I (1 equivalent)
(a) CH3
(b) CH3 H X (Scheme 1)
(iii) CH3I (1 equivalent)
HO M (iv) H2, Lindlar's catalyst
CH3
CH3
(i) NaNH2 (2 equivalent)
CH3
(c) (d) OH
CH3 (ii)
Br
H Y (Scheme 2)
(iii) H3O+, (mild)
N (iv) H2, Pd/C
Passage 2 (v) CrO3
An acyclic hydrocarbon P, having molecular formula C 6H10, 46. The correct statement with respect to product Y is

w
gave acetone as the only organic product through the (a) it gives a positive Tollen’s test and is a functional isomer of X
following sequence of reactions, in which Q is an (b) it gives a positive Tollen’s test and is a geometrical isomer of X
intermediate organic compound. (c) it gives a positive iodoform test and is a functional isomer of X

Flo
(i) dil. H2SO4/HgSO4 (d) it gives a positive iodoform test and is a geometrical isomer of X
P
(ii) NaBH4/ethanol 47. The product X is
(C6H10) (iii) dil. acid

ree
H3CO H
(i) dil. H2SO4(catalytic
O
amount) (–H2O) (a) (b)
Q 2 C

F
(ii) O3 /ethanol H
(iii) Zn/H2O H3C CH3 (2011)
H H H3CO

or
H
ur CH3CH2O
44. The structure of the compound Q is

f
(c) (d)
H3C OH H3C OH H
ks
½ ½ CH3CH2O
(a) H ¾ C¾ C ¾ CH2CH3 (b) H3C¾ C¾ C ¾ CH3 H H
Yo
½ ½
oo
H3C
H H3C H Fill in the Blanks
H3C OH
eB

½ 48. 1,3-butadiene with bromine in molar ratio of 1 : 1 generate


(c) H ¾ C — CH2 CH CH3 (d) predominantly …… . (1997, 1M)
H3C
49. Addition of water to acetylene compounds is catalysed by ……
r

OH
and …… (1993, 1M)
ou

½
ad

CH3CH2CH2 CH CH2CH3
50. Kolbe’s electrolysis of potassium succinate gives CO2 and …… .
Y

45. The structure of compound P is (1993, 1M)


(a) CH3CH2CH2CH2 ¾ C ºº C¾ H 51. The terminal carbon atom in 2-butene is …… hybridised.
nd
Re

(b) H3CH2C¾ C ºº C¾ CH2CH3 (1985, 1M)

H3C 52. Acetylene is treated with excess sodium in liquid ammonia. The
Fi

(c) H ¾ C¾ C ºº C¾ CH3 product is reacted with excess methyl iodide. The final product is
…… . (1983, 1M)
H3C
53. …… is most acidic. (Ethane, Ethene, Ethyne) (1981, 1M)
H3C
(d) H3C¾ C¾ C ºº C¾ H True/False
H3C 54. Moist ethylene can be dried by passing it through concentrated
Passage 3 sulphuric acid. (1981)
Schemes 1 and 2 describe sequential transformation of
alkynes M and N. Consider only the major products formed in Integer Answer Type Question
each step for both schemes. 55. The total number of cyclic isomers possible for a hydrocarbon with
the molecular formula C4 H6 is (2010)
Hydrocarbons 315

Subjective Questions this information two isomeric structures can be proposed for
alkene (A). Write their structures and identify the isomer which
56. Monomer A of a polymer on ozonolysis yields two moles on catalytic hydrogenation (H2/Pd - C) gives a racemic mixture.
of HCHO and one mole of CH3 COCHO. (2001)
(a) Deduce the structure of A.
(b) Write the structure of all cis form of polymer of compound 61. What would be the major product in the following reaction?
A. (2005, 2M)
H2
57. A biologically active compound, Bombykol (C16H30O) is Lindlar’s catalyst
obtained from a natural source. The structure of the
CH3
(2000, 1M)
compound is determine by the following reactions.
62. Complete the following reactions with appropriate reagents :
(a) On hydrogenation, Bombykol gives a
compound A, C16H34O, which reacts with

w
acetic anhydride to give an ester. (a) ==
(b) Bombykol also reacts with acetic anhydride to
give another ester, which on oxidative
D D OH CH3

Flo
ozonolysis (O3/H2O2) gives a mixture of
(b) CH3 C==C
butanoic acid, oxalic acid and 10-acetoxy H3C D—C—C—H
decanoic acid. H 3C CH3
H 3C D

ree
Determine the number of double bonds in CH3 CH3 (1999)
Bombykol. Write the structures of compound

F
A and Bombykol. How many geometrical 63. Complete the following reactions with appropriate structures of
isomers are possible for Bombykol? products/reagents :
Br

or
(i) NaNH (3 equivalent)
write their structures.
ur
58. Identify X, Y and Z in the following synthetic scheme and 2
C6H5CH == CH2 ¾® [ A ] ¾¾¾¾¾¾¾¾¾®
2
[B]
(ii) CH3I (1998, 2M)

CH3 CH2 ¾ C ºº C ¾ H
(i) NaNH
¾¾¾¾®
2

f
64. Write the intermediate steps for each of the following reactions :
ks
(ii) CH3CH2Br H 3O +
(i) C6 H5 CH(OH)C ºº CH ¾® C6 H5 ¾ CH == CH ¾ CHO
Yo
H2 /Pd BaSO4 Alkaline KMnO4
oo
X ¾¾¾¾¾® Y¾¾¾¾¾¾® Z
H+
Is the compound Z optically active? Justify your answer.
eB

(ii)
(2002)
OH O CH3

65. The hydrocarbon A, adds one mole of hydrogen in the presence


r

59. (a) Identify (A), (B), (C), (D) and (E) in the following schemes
ou

and write their structures : of a platinum catalyst to form n-hexane. When A is oxidised
ad

vigorously with KMnO4 , a single carboxylic acid, containing


Br2/Cl4
Y

three carbon atoms, is isolated. Give the structure of A and


explain. (1997, 2M)
nd

NaNH2 HgSO4/H2SO4 66. Complete the following, giving the structures of the principal
Re

A B C organic products : (1997, 1M)


H2 NHNCONH2 NaOD/D2 O (excess)
Fi

C ¾¾¾¾¾® D ¾¾¾¾¾¾¾® E Ph H
(b) Identify (X), (Y) and (Z) in the following synthetic scheme A
(i) + KNH2
and write their structures. Explain the formation of labelled
formaldehyde (H2C*O) as one of the products when Ph Br
compound (Z) is treated with HBr and subsequently R R
ozonolysed. Mark the C* carbon in the entire scheme.
BaC*O3 + H2SO4 ® (X) gas
B
(ii) + HClO4
[C* denotes C14] [R = nPr]
(i) Mg/ether LiAlH4
H2C == CH ¾ Br ¾¾¾¾® (Y ) ¾¾¾® (Z ) O
(ii) X
(2001)
(iii) H3 O+ C
(iii) + CHBr3 + t-BuOK
60. An alkene (A) C16H16 on ozonolysis gives only one
product (B) C8H8O. Compound (B) on reaction with
NaOH/I2 yields sodium benzoate. Compound (B) reacts 67. An alkyl halide, X, of formula C6 H13 Cl on treatment with
with KOH/NH2NH2 yielding a hydrocarbon (C) C8H10. potassium tertiary butoxide gives two isomeric alkenes Y and Z
Write the structures of compounds (B) and (C). Based on (C6 H12 ) . Both alkenes on hydrogenation gives 2, 3-dimethyl
butane. Predict the structures of X, Y and Z. (1996, 3M)
316 Hydrocarbons

68. Give the structure of the major organic products obtained (iii) CH3 COCH3 (iv) CH3 ¾ CH ¾ CHO
from 3-ethyl-2-pentene under each of the following reaction |
conditions : (1996) CH3
(a) HBr in the presence of peroxide What are the structures of A, B and C ? (1986, 4M)
(b) Br2 / H2O 76. How would you convert acetylene to acetone? (1985, 1M)
(c) Hg(OAc)2 / H2O, NaBH4 77. Give the chemical test to distinguish between 2-butyne and
69. Write down the structure of the stereoisomers formed when 1-butyne. (1985, 1M)
cis-2-butene is reacted with bromine. (1995) 78. Following statements are true, only under some specific
70. An organic compound E (C5 H8 ) on hydrogenation gives conditions. Write the condition for each subquestion in not
compound F (C5 H12 ). Compound E on ozonolysis gives more than two sentences :

w
formaldehyde and 2-keto propanal. Deduce the structure of (i) 2-methyl propene can be converted into isobutyl bromide
compound E. (1995) by hydrogen bromide.
71. When gas A is passed through dry KOH at low temperature, a (ii) Ethyne and its derivatives will give white precipitate with
deep red coloured compound B and a gas C are obtained. The ammoniacal silver nitrate solution. (1984,1M ´ 2 = 2M)

Flo
gas A, on reaction with but-2-ene, followed by treatment with 79. Give reasons for the following in one or two sentences :
Zn / H2 O yields acetaldehyde. Identify A, B and C. (i) Methane does not react with chlorine in the dark.

ree
(1994, 3M) (ii) Propene reacts with HBr to give isopropyl bromide but
72. Give the structures of A, B and C (explanation are not required) does not give n-propyl bromide. (1983,1M ´ 2 = 2M)

F
(i) A (C4H8 ) which adds on HBr in the presence and in the absence 80. State with balanced equation, what happens when “propene
of peroxide to give same product.
is bubbled through a hot aqueous solution of potassium

or
permanganate.”?
ur
(ii) B (C4H8 ) which when treated with H2O / H2SO4 gives C4H10O
(1982, 1M)
which cannot be resolved into optical isomers. 81. One mole of a hydrocarbon A reacts with one mole of

f
(iii) C (C6H12 ), an optically active hydrocarbon which on catalytic bromine giving a dibromo compound, C5 H10 Br2 . Compound
ks
hydrogenation gives an optically inactive compound C6H14. A on treatment with cold dilute alkaline potassium
Yo
(1993,1M ´ 3 = 3M) permanganate solution forms a compound, C5 H12 O2 . On
oo
ozonolysis A gives equimolar quantities of propanone and
73. Write the balanced chemical equation for the following
ethanal. Deduce the structural formula of A. (1981, 1M)
“Ethylene glycol is obtained by the reaction of ethylene with
eB

potassium permanganate.” (1991, 1M) 82. Write the structural formula of the major product in each of
the following cases
74. Give a chemical test and the reagents used to distinguish (i) Ethene mixed with air is passed under pressure over a
r

between cyclohexane and cyclohexene. (1991, 1M)


silver catalyst.
ou
ad

75. A white precipitate was formed slowly when silver nitrate (ii) The compound obtained by hydration of ethyne is treated
was added to compound A with molecular formula C6 H13 Cl. with dilute alkali. (1981 , 2 ´ 1 / 2 M = 1M)
Y

Compound A on treatment with hot alcoholic potassium 83. Outline the reaction sequence for the conversion of ethene to
hydroxide gave a mixture of two isomeric alkenes B and C, ethyne (the number of steps should not be more than two).
nd

having formula C6 H12 . The mixture of B and C, on


Re

(1981, 1M)
ozonolysis, furnished four compounds
84. Give one characteristic test which would distinguish CH4
Fi

(i) CH3 CHO (ii) C2 H5 CHO


from C2 H2 . (1979, 1M)

Answers
Topic 1 17. (a) 18. (a) 19. (a) 20. (a)
1. (b) 2. (c) 3. (b) 4. (b) 21. (d) 22. (c) 23. (b) 24. (b)
5. (d) 6. (c) 7. (b) 8. (c) 25. (d) 26. (a) 27. (a) 28. (d)
9. (a) 10. (d) 11. (c) 12. (d) 29. (c) 30. (b) 31. (b) 32. (a)
13. (b) 14. (c) 15. (d) 16. (b) 33. (a) 34. (c) 35. (d) 36. (d)
17. (8) 37. (a,b,d) 38. (b) 39. (a) 40. (c)
41. (a) 42. (c) 43. (d) 44. (b)
Topic 2 45. (d) 46. (c) 47. (a)
1. (c) 2. (c) 3. (b) 4. (d)
48. 3,4-dibromo-1-butene 49. H 2SO 4 , HgSO 4
5. (d) 6. (c) 7. (c) 8. (a)
50. ethene 51. sp 3 52. 2-butyne
9. (a) 10. (b) 11. (c) 12. (a)
13. (d) 14. (a) 15. (b) 16. (a) 53. Terminal alkyne (ethyne) 54. False 55. (5)
Hints & Solutions
Topic 1 Saturated Hydrocarbons
1. A dihedral angle is the angle between two C ¾ H bonds
projected on a plane orthogonal to the C ¾ C bond. In the given
skew conformation, having Newman’s projection the dihedral and
angle is
H¢¾ C ¾ C ¾ H¢¢ III II
= (H¢¾ C ¾ C ¾ Ha ) + (Ha ¾ C ¾ C ¾ H¢¢)

w
= 29°+120° = 149°
and
H
H H¢ I

Flo
29º (ethane, C2H6)
Ha On moving left to right (III to I)
120º · branching increases

ree
H¢¢ H · surface area decreases
b

· boiling point decreases

F
2. The four types of strains viz (a) electrostatic force of attraction,
Hence, correct choice is (b).
(b) torsional strain, (c) angle strain, (d) steric stain, are
responsible for the stability or energy barriers of conformers. In 4. CH3 CH3

or
ur
cyclic compounds, all types of strains may be present. CH3 Cl2 CH2Cl
+
hn

f
Compound Type of strains/forces H3C CH3
I
a+ c
ks
CH3 CH3 CH2Cl
Yo
CH3 + CH3 + CH3
oo
a+ b+ c
H3C H3C Cl CH3
eB

OH F a+ b+ c+ d Cl
(II + III) IV (V + VI)
chiral chiral
Since, fractional distillation cannot separate enantiomers
r

In a cyclic or open-chain compounds, angle strain (c) is absent. e.g.


(II + III and V + VI), M = 4 and N = 6.
ou
ad

Compound Types of strains/forces Cl


Y

H a Na / D
H 5. Wurtz’s reaction
Br
nd

H
Re

1-bromo-3-chlorocyclobutane
H H
H
CH3
Fi

CH3 a+ b CH3 hn
6. + Cl2
H CH3 H 3C I Cl
2-methyl butane
H H Cl
H

OH a+ b+ d + + +
H OH Cl
Cl
II III IV
H H
H
Out of the four products formed above, II and IV are chiral,
produced in pairs, giving total of six mono-chlorination
3. PLAN This problem is based on boiling point of isomeric alkanes.
As we know more the branching in an alkane, lesser will be its
products.
surface area and lesser will be the boiling point
318 Hydrocarbons

7. Bromination is highly selective, occur at the carbon, where the Due to this method of formation, methane is also known as marsh
most stable free radical is formed : gas.
CH3 ¾ CHD ¾ CH — CH3 + Br · ¾® 17. *
Cl +
½
CH3 Cl I
· Cl
CH3 ¾ CHD ¾ C— CH3 + HBr * * + +
½
CH3 II III IV Cl
(a tertiary free radical) I has one chiral carbon = two isomers
8. Toluene on treatment with Cl 2 in the presence of heat or light II has two chiral carbons and no symmetry = four isomers.
III and IV have no chiral carbon, no stereoisomers.
undergo free-radical chlorination at benzylic position, giving

w
benzyl chloride
CH3 CH2Cl Topic 2 Unsaturated Hydrocarbons
1. According to the given conditions, the compound should be
hv alkyne with triple bond present at the terminal. The chemical

Flo
+ Cl2 + HCl
or heat reactions involved are as follows:
Toluene Benzyl chloride Step 1

ree
CH3 CH3 C CH
Ag2O
CH3 C C Ag
½ Prop-1-yne (Precipitates)
9. (CH3)3 CMgCl + D2O ¾® CH3 ¾ C — D + Mg(OD)Cl

F
(A)
½
CH3 Step 2

or
ur
10. Alkanes are all less dense than water, floats over water. CH3 C
Hg2+
CH dil. H SO

f
2 4
11. C—H bond with sp3-C will be longest in C2H6. OH O
Tauto-
ks
CH3 CH3 CH3 merisation
CH3 C CH3
Yo
CH3 C CH2
½ ½ ½
oo
(Propan-2-one)
12. CH3 ¾ C — C — CH2 ¾ CH3 CH3 ¾ C — CH2CH2CH3 (B)
½ ½ ½
eB

CH3 CH3 CH3 NaBH4


2, 2, 3, 3-tetramethyl pentane 2, 2-dimethyl pentane
(no isopropyl group) (no isopropyl group) CH3 CH CH3
r

CH3 OH
CH3 CH3
ou

(C) (2° alcohol)


½ ½ ½
ad

CH3 ¾ C — CH— CH2CH3 CH3 ¾ CH — CH2CH2CH3 Conc. HCl


+
½
Y

ZnCl2
CH3 Isopropyl group
2, 2, 3-trimethyl pentane 2-methyl pentane CH3 CH CH3
(no isopropyl group)
nd
Re

Cl
13. Boiling point of alkane increases with molar mass. Among Turbidity within 5 minutes
(Insoluble in Lucas reagent)
Fi

isomeric alkanes, branching decreases boiling point. Therefore,


n-octane has highest boiling point, higher than In step-1, prop-1-yne reacts with Ag2O to form
2, 2, 3, 3-tetramethyl-butane (an isomer of n-octane). CH3 ¾ C ºº C ¾ Ag, that forms white precipitates.
14. Ethylene absorb H2SO4 forming CH3 ¾ CH2OSO3H and In step 2, prop-1-yne in presence of mercuric sulphate and
dil × H2SO4 produces carbonyl compound (CH3 )2 C == O which
dissolve. Benzene, with warm H2SO4, undergo sulphonation and
produces (CH3 )2CH ¾ OH in presence of NaBH4. 2ºalcohol
dissolve.
on reaction with Lucas reagent produces turbidity in about
Aniline, with H2SO4, forms anilinium sulphate salt and dissolve. 5 min.
Hexane, a hydrophobic molecule, does not react with H2SO4,
remains insoluble. 2. But-2-ene on reaction with alkaline KMnO4 at elevated
temperature followed by acidification will give acetic acid
15. Among alkanes, boiling point increases with molar mass. (CH3COOH). Hot alkaline solution of potassium
Among isomeric alkanes, branching decreases boiling point. permanganate followed by acidification oxidatively cleaved
Therefore, n-hexane has highest boiling point among these. alkenes. The reaction proceed as follows :
16. Methane is produced due to the decay of vegetables or animal Alk. KMnO4 , heat
organisms present in swamps and marsh, by the action of bacteria. CH3 ¾ CH == CH ¾ CH3 ¾¾¾¾¾¾® 2CH3COOH
But-2 -ene H3O+ Acetic acid
Hydrocarbons 319

3. The major product of the given addition reaction is +R d+ d–


H3C ¾ CH ¾ CH2.
r
Cl –
— CH2 H—Cl Cl —
Cl — CH — — CH — CH3
Slow
½ ½
fast
OH Cl
Cl
In this reaction, H2O is used as a solvent and the major product Cl — CH — CH3
of the reaction will be a vicinal halohydrin. A halohydrin is an
organic molecule that contains both OH group and a halogen. In Similarly,
a vicinal halohydrin, the OH and halogen are bonded to adjacent d+ d–
+
·· H—Cl
carbons. H2N — CH —
— CH2 NH2 — CH — CH3
Cl /H O (+R) Cl
H3 C ¾ CH == CH2 ¾¾¾® H3C ¾ CH ¾ CH2 +
2 2

½ ½
Cl
NH2 — CH — CH3
OH d+ d– Fast

w
Cl H—Cl +
A chlorohydrin (major product) CH3O — CH —
— CH2 CH3O — CH — CH3
(+R)
CH3 ¾ CH ¾ CH2 Cl

CH3O — CH — CH3
½ ½ Fast

Flo
Cl Cl Cl
(Minor product) d+
d–

ree
The reaction proceeds through following mechanism : Cl — H

+ HCl r
Cl < CH —
F3C — — CH2 Slow F3C — CH2 — CH2

F
Slow H2 O (– I) – Cl
s
CH3—CH=
=CH2 + Cl—Cl CH3—CH—CH2 Fast s
Cl
Chloronium – F3C — CH2CH2Cl
+ Cl Slow

or
ur ion

6. Ethene (H2C == CH2 ) is sp 2-hybridised and ethyne

f
H2 O +
CH3—CHCH2—Cl Fast
CH3CH—CH2—Cl + H3O (HC ºº CH) is sp-hybridised. In ethyne, the sp-hybridised
| |
ks
+ carbon atom possesses maximum s-character and hence,
OH OH
maximum electronegativity. Due to which, it attracts the shared
Yo
| Chlorohydrin
electron pair of C¾H bond to a greater extent and makes the
oo
H
removal of proton easier. Hence, alkyne is much more acidic
4. The major product obtained in the given reaction is than alkene.
eB

CH3C (I) (Cl) CHD2. Presence of electron donating group in alkyne (H3C ¾ C ººCH)
1. DCl (1 equiv.)
decreases the acidic strength of compound. Hence, the correct
CH3C ºº CH ¾¾¾¾® CH3C (I)(Cl)CHD2
2. DI order of acidic strength is:
r

HC ºº CH > H3C ¾ C º CH > CH2 == CH2


ou

Addition in unsymmetrical alkynes takes place according to


ad

Markovnikov’s rule. 7. Sodium metal in liquid ammonia reduces alkynes with anti
Y

Reaction proceed as follows : stereochemistry to give trans alkenes. The reduction is


Cl selectively anti since the vinyl radical formed during reduction
DCl (1 equiv.) ½ DI is more stable in trans configuration.
CH3 C ºº CH ¾¾¾¾¾® CH3 C == CHD ¾¾
nd
Re

Prop-1-yne Mechanism
Cl
Fi

R R H
½ H–NH2
R C C R¢ C C C C
CH3 ¾ C ¾ CHD2 –NH2
Na+ R¢ R¢
½ Na Vinylic
I radical
Product Sodium atom donates an electron to
alkyne which after H-abstraction from
5. Attachment of electron donating group (+ R or + I) with NH3 forms vinylic radical. Transfer of
sp2-carbon of an unsymmetrical alkene supports Markownikov’s another electron gives a vinylic anion,
which is more stable in trans form. This Na Na+
addition rule through electrophilic-addition-pathway. in turn gives trans-alkene after
But, attachment of electron-withdrawing group (- R or - I) for H-abstraction from NH3.
the same will follow anti-Markownikov’s pathway (even in
absence of organic peroxide which favours free radical addition) R H R H
H–NH2
through electrophilic addition pathway. C C –NH2 – C C
The product formed by given alkenes when treated with HCl. H R¢ R¢
Vinylic
anion
320 Hydrocarbons

8. d– d+ 16. NOCl undergo electrophilic addition on alkene as:


HO—Cl +
— 2
CH3—CH—CH CH3—CH—CH2—Cl
(Electrophilic NOCl ¾® +
N==O + Cl -
(Intermediate)
addition) +
Cl -

OH CH3 ¾ CH==CH2 + + NO ¾® CH3 ¾ CH ¾ CH2 ¾®
½
CH3—CH—CH2—Cl NO

OH CH3 ¾ CH ¾ CH2
½ ½
9. PhCH2 CH3 PhCH2 H Cl NO
C C C C
H H H CH3 17. Cyclohexanol on treatment with concentrated H3PO4 undergo
cis form trans form acid catalysed dehydration giving cyclohexene.

w
10. CH3 CH3 OH
O3 O conc.H3PO4

Flo
Zn/H2O CHO

CH3 CH3 cyclohexanol cyclohexene

ree
5-keto-2-methyl hexanal
18. Alkynes on treatment with alkali metals in liquid ammonia gives
11. The reaction is

F
trans hydrogenation product:
6Ag
2CH3 ¾ CCl 3 ¾ ¾
¾® CH3 ¾ C ºº C ¾ CH3 + 6 AgCl CH3 ¾ C ºº C ¾ CH2 ¾ CH2 ¾ CH3 ¾¾¾®
3 Li/NH
D But-2-yne

or
ur 2-hexyne
12. Ozonolysis of the given triene occur as follows : H3 C H

f
CH3 CH3 C==C
ks
H CH2 CH2 CH3
Yo
H3 C CH CH C CH CH C CH CH CH3 trans-2-hexene
oo
19. Reaction proceeds through carbocation intermediate:
H H +
H+
eB

CH3 H O
CH3 ¾ C== CH2 ¾® CH3 ¾ C ¾ CH3 ¾®
2

O3 ½ ½
2 CH3 CHO + 2 OHC C CHO Ph Ph
Zn/H2O 2-phenyl propene 3 °, resonance stabilised
r
ou

H OH
ad

Since, none of the above dial is chiral, no optically active ½


CH3 ¾ C — CH3
Y

product is obtained.
½
13. CH3CH2C ºº CH + Br — CH2CH2CH2CH3 Ph
nd

2-phenyl-2-propanol
Re

¾¾® CH3CH2 — C ºº C— CH2CH2CH2CH3


— HBr 3-octyne 20. Reaction proceeds through carbocation intermediate :
Fi

+ H2 O
14. Br2 undergo anti-addition on C == C bonds as: Ph ¾ C ºº C ¾ CH3 + H+ ¾® Ph ¾ C == CH ¾ CH3 ¾®
Br Resonance stabilised
CH3 OH O
H3C H C ½ ½½
CH3
H
C==C
CH3
+ Br H C H
Ph ¾ C == CH ¾ CH3
Unstable enol
e
(Tautomerisation)
Ph ¾ C ¾ CH2 ¾ CH3

Br 21. Ammoniacal CuCl 2 forms red precipitate with terminal alkynes,


(meso isomer) can be used to distinguish terminal alkynes from internal alkynes:
Br Alc. KOH NH ( aq )
NaNH2 CH3 ¾ CH2 ¾ C ºº C ¾ H + CuCl 2 ¾¾®
3
15. CH2 ==CHBr
Br
H—C ººC—H CH3 ¾ CH2 ¾ C ºº C - Cu + ¯
red ppt.
Hydrocarbons 321

22. In addition of HBr to an alkene, in the presence of peroxide, both 28. According to the IUPAC conventions, compound can be
the propagation steps are exothermic : numbered as:
1 2 3 4 5 6
HBr + HO· ¾® H2O + Br · H2 C== CH ¾ CH2 ¾ CH2 ¾ C ºº C ¾ H
Propagation Here, C-2 is sp2 and C-3 is sp3-hybridised.
·
ì ·
ï CH3 ·¾ CH == CH2 + Br ¾® CH3 ¾ CH ¾ CH2Br; DH < 0 29. Electrophilic addition on C == C is governed by stability of
ï
íCH3 ¾ CH ¾ CH2Br + HBr ¾® CH3 ¾ CH2 ¾ CH2Br carbocation:
ï + Br ·; DH < 0 H+
ïî CH3—CH==CH— —OH
In case of addition of HCl and HI, one of the propagation step is +
endothermic, reaction fail to occur. CH3—CH2—CH— —OH

w
- +
23. HOCl ¾® HO + Cl
Cl +

CH3 ¾ CH==CH2 + Cl + ¾¾® CH3 ¾ CH ¾¾¾ CH2 Br– +


CH3—CH2—CH== ==OH

Flo
Cl
½ Br (resonance stabilised carbocation)
HO-
¾¾® CH3 ¾ CH — CH2

ree
½ CH3CH2—CH— — OH
OH
i.e. reaction is initiated by Cl + (chloronium ion electrophile)

F
30. Among alkenes-heat of hydrogenation depends on :
24. Hydrogenation with poisoned palladium brings about cis (a) The number of double bonds-greater, greater the amount of
hydrogenation of alkyne and does not affect double bonds : heat evolved in hydrogenation.

or
CH3 H
ur Hence, 1, 3-butadiene has highest heat of hydrogenation

f
CH3 among these.
H2/Pd (b) Relative stability of alkenes-greater the stability, smaller the
ks
heat evolved in hydrogenation. trans-2-butene is most stable
Yo
H3C H H CH3 H among three given butenes, has least heat of hydrogenation.
oo
H3C CH3 31. CH3 ¾ C ºº C ¾ CH2CH3 + HBr ¾®
eB

H3C C2H5 H3C Br


H HH C == C + C == C
H
H Br H CH
meso-compound 1 44444444244444444235
r

(optically inactive) geometrical isomers


ou
ad

25. Terminal alkynes forms silver salt with Tollen’s reagent while H3C C2H5 H3C H
alkene does not react with Tollen’s reagent. C == C + C == C
Y

NH3 ( aq ) Br H Br C2H5
CH3 ¾ C ºº C ¾ H + AgNO3 ¾¾® CH3 ¾ C ºº CAg ¯ 1 444444442444444443
white ppt. geometrical isomers
nd
Re

Therefore, Tollen’s reagent can be used to distinguish a terminal Therefore, two structural and four configurational isomers.
alkyne like propyne from alkene as well as from internal alkynes. 32. Terminal alkynes are slightly acidic, forms salt with very strong
Fi

26. Ease of catalytic hydrogenation depends upon the size of groups base like Na, NaNH2 etc.
present at the doubly bonded carbon. Larger the size of groups, D 1
H ¾ C ºº C ¾ H + Na ¾® H ¾ C ºº C - Na + + H2 ­
difficult the hydrogenation. Therefore, in the given situation, ethyne 2
disubstituted reacts at faster rate than tri and tetra substituted
alkenes. Among disubstituted, the stability order is : 33. Baeyer’s reagent is cold, dilute, alkaline permanganate solution,
used to detect presence of olefinic bonds.
cis < geminal < trans.
34. Alkynes undergo Markownikoff’s addition of water in the
27. Oxymercuration-demercuration brings about Markownikoff’s presence of H2SO4 / HgSO4 :
addition of water as :
HgSO4 ù
CH3 ¾ CH2 ¾ C ºº C ¾ H + H2 SO4 ¾¾® é
ê
OH ú
ê ½ ú
é OH ù O CH3 ¾ C ºº C ¾ H + H2SO4 ¾¾®
HgSO4 ê CH3 ¾ C == CH2 úúû
ê ½ ú ½½ êë
Unstable enol
êCH3 ¾ CH2 ¾ C== CH2 ú ¬® CH3 ¾ CH2 ¾ C ¾ CH3
ê ú O
butanone ½½
ê ú
ë
unstable enol
û e
(Tautomerisation)
CH3 ¾ C ¾ CH3
Acetone
322 Hydrocarbons

35. Structural formula of 1, 2-butadiene is : Both Statement I and Statement II are correct and Statement II is
correct explanation of Statement I.
H2C == C == CH¾ CH3
­ ­ ­ ­ 40. CH3 ¾ CH2 ¾ CH== CH2 + Br · ¾®
sp2 sp sp2 sp3 ·
CH3 ¾ CH2 ¾ CH ¾ CH2 Br
36. Unsaturated compounds which contain C == C or C ºº C, a secondary radical
decolourises the purple colour of alkaline KMnO4 solution. Therefore, Statement I is correct but Statement II is incorrect.
CH2 ¾ OH 41. CH3 ¾ CH2 ¾ CH == CH2 + Br2 ¾®
HO-
CH2 == CH2 + KMnO4 ¾¾® ½ + MnO2 ¯
purple CH2 ¾ OH CH3 ¾ CH2 ¾ CH ¾ CH2 Br
coloured ½
Br
37. Addition of halogen at double bond occur in antiorientation via has one chiral carbon

w
cyclic halonium ion intermediate.
Both Statement I and Statement II are correct and Statement II is
(i) correct explanation of Statement I.
r Br
CH3 H Br CH3 H

Flo
Br2 CH3 Br –
Passage 1
C C CH3
H H The reaction condition indicates that starting compound is phenyl
H CH3 H3C H Br acetylene.

ree
trans Meso (M and N)
42.
(ii) r ¾ CH2
CH ¾
CººC—H OH

F
H H Br
Br2 CH3 Pd/BaSO4 (i) B2H6
C C H2 (ii) H2O2, NaOH, H2O
H
CH3 CH3 H

or
cis
ur
CH3 (2-phenyl ethanol)
(X)
H CH3

f
Br Hydroboration oxidation brings about anti-Markonikoff’s
Br
Br –
H CH3 hydration of alkene.
ks
+
CH3 Br 43.
Yo
H H
Br CH3
O
oo
O P
C ºº CH
ó
ô

ô ó
í

Enantiomers CH3
eB

H2O, HgSO4
Here, (M + O) and (N + P) are pair of diastereomers. H2SO4

+ CH3
Br CH3
H 3C
r

H C
ou

C==C + Br2 CH3 CH3


ad

38. CH3
H+/D
CH3 C C2H5 MgBr, H2O
H H OH
Y

trans H
CH3
Statement I is correct. Statement II is also correct. Meso form of
nd
Re

the product is due to anti addition of Br - on cyclic bromonium


Passage 2
ion intermediate, hence Statement II is not correct explanation The final ozonolysis product indicates that the alkene before
Fi

of Statement I. ozonolysis is
O
Br
H3C CH3
CH3 O3
2 C
C C C Zn–H2O
Br– CH3 H3C CH 3 H 3 C CH3
H C P
H
Also P(C6H10) has two degree of unsaturation and
Br oxymercuration demercuration hydration indicates that it is an
(meso) alkyne. As alkyne, on hydration, gives a carbonyl compound
which on reduction with NaBH4 gives a 2º alcohol.
39. O
O OH
C==C + O3 C C CH3—S—CH3
(i) NaBH4
—C C— + H2O —C—CH2— + — C—CH2—
O O (ii) H
ozonide H
2° alcohol
C ==O + O == C + (CH3 )2 SO
Hydrocarbons 323

The secondary alcohol that can give above shown alkene on acid Hence, using the concept of regioselectivity we come on the
catalysed dehydration is conclusion that final product is correctly represented by
structure (a).
CH3 OH
+ + 48. 3, 4-dibromo-1-butene :
H Me-shift
CH3 — C — CH — CH3 CH3 — C — CH— CH3
–H2O + Br2
Br
CH3
Q 2° carbocation Br
CH3 CH3
+ HgSO
+ –H 49. CH ºº CH + H2SO4 ¾¾®
4
CH3CHO
CH3 —C—CH—CH 3 CH3 — C C — CH3
CH2 ¾ COOK
CH3 Electrolysis
CH3 50. ½ ¾¾¾® CH2 == CH2 + 2CO2

w
CH2 ¾ COOK ethene
44. Explained in the beginning.
CH3 CH3 O sp 3

HgSO4 (i)NaBH4
51. CH3 ¾ CH== CH ¾ CH3
CH3—C— C—CH3 Q
45. CH3—C—C CH 2- butene

Flo
H2SO4 (ii) H +

CH3 CH3 52. 2-butyne :


(P) + - CH I

ree
H ¾ C ºº C ¾ H + Na ¾® Na C ºº C - Na + ¾¾®
3
excess excess
Passage 3
CH3 ¾ C ºº C ¾ CH3

F
46. PLAN This problem can be solved by using the concept of iodoform
53. Terminal alkyne (ethyne) is most acidic among these.
test and functional isomerism.
Iodoform test The compound containing — COCH3 or 54. Sulphuric acid undergo addition to alkene.

or
¾CH(OH) group will undergo iodoform test
ur
f
Br OH
NaNH2 55.
H C s
ks
Iequivalent NaNH2(1eq) , , , ,
N
Yo
oo
CH3
½
s
H2Pd/C O
OH O3
56. (a) CH2 == C — CH == CH2 ¾¾® 2HCHO
eB

H3Os
mild (A ) Zn -H2 O

OH Me O
CrO3 O
½½
r

Y + CH3 ¾ C ¾ CHO
ou
ad

• Thus, X and Y are functional isomers of each other and Y gives


iodoform test due to the presence of CH 3CO group as Indicated. H3C H H3C H
Y

Hence, correct choice is (c). (b) C == C C == C


H2C CH2 ¾¾ H2C CH2
47. PLAN This problem can be solved by using the concept of nucleophilic
nd

substitution reaction, oxidation reaction and reduction reaction natural rubber


Re

including strength of nucleophile and regioselectivity.


Reaction of Scheme 1 can be completed as 57. From oxidation products, structure of starting compound can be
Fi

1. NaNH2 deduced as :
H C3 H8 ¾ COOH + HOOC ¾ COOH
II
O butanoic acid oxalic acid O
HO M I
½½
Among two nacked nucleophilic group I and II, II is more + HOOC ¾ (CH2 )8 ¾ CH2 O ¾ C ¾ CH3
nucleophilica and then will react selectively as follows 10-acetoxy decanoic acid
1. CH3CH2—I O 3 / H 2O 2
CH3 CH2 CH2 CH==CH ¾ CH==CH ¾ (CH2 )8
O O
CH3I ¾ CH2 OCOCH3
H2 Lindlar’s Therefore, Bombykol is :
catalyst (produces
cis alkene)
CH3O CH3CH2CH2 ¾ CH== CH ¾ CH== CH ¾ (CH2 )8 ¾ CH2OH
H H Bombykol
H2
CH3 ¾-
( CH2 )14 ¾ CH2OH
CH3O
(A )
324 Hydrocarbons

C2H5
H H alkaline KMnO4 H OH
(i) NaNH Pd/BaSO
58. CH3CH2 ¾ C ºº C ¾ H ¾¾¾¾®
2
CH3CH2 ¾ C ºº C ¾ CH2CH3 ¾¾¾¾®
4
C== C H OH
(ii) CH3 CH2 Br X H2 C2H5 C2H5 C2H5
Y (meso diol)
‘ Z’
O
Br
Br2 NaNH2 HgSO4
59. (a) —C ººCH B —C—CH3
CCl4 H2SO4
A Br B C
O

w
CH3 O C
H2NHNCONH2 NaOD
—C==N—NH—C—NH2 D CD3
D2O

Flo
E D
D
* *

ree
(b) Ba CO3 + H2 SO4 ¾® CO2 + BaSO4
X
O

F
(i) Mg / ether ½
½ LiAlH4 *
CH2 == CH ¾ Br ¾¾¾® CH2 ==CH ¾ C ¾ OH ¾¾® CH2 == CH ¾ CH2 ¾ OH
(ii) X * Z

or
ur
(iii) H3O+ Y

60. B + NaOH + I2 ¾® C6H5 ¾ COONa (Iodoform reaction)


O
f
ks
½½
\ B is C6H5 ¾ C ¾ CH3
Yo
oo
KOH Hydrocarbon (C ) “Wolff-Kishner Reduction”
B + N2H4 ¾¾®
C8 H10
eB

\C is C6H5 ¾ CH2 ¾ CH3


Hence, A can be one of the following :
H5 C6 C6 H5 H5 C6 CH3
r
ou

C==C or C==C
ad

H3 C CH3 H3 C C6 H5
I II
Y

I on catalytic hydrogenation, would give meso compound while II on catalytic hydrogenation, would produce racemic mixtures.
nd
Re

CH3
61. Lindlar’s
Fi

CH3 catalyst H
H
cis hydrogenation

O OH

Zn-H2O LiAlH4 conc. H2SO4


62. (a) + O3 D

D OH
D D CH3
H 2O 2
(b) CH3 C==C + BHR2 – CH3 C—C
HO
CH3 H
H3C H3C
CH3 CH3 D

R = (CHl 3 )2 CHCH2
Hydrocarbons 325

Br CH2CH3 C2H5
½ ½ ½
HBr
63. C6H5 ¾ CH == CH2 + Br2 ¾® C6H5 ¾ CH ¾ CH2 68. CH3CH == C ¾ CH2 CH3 ¾¾® CH3 ¾ CH ¾ CH ¾ C2H5
½ 3-ethyl-2-pentene peroxide ½
Br Br
A [A]
NaNH2 CH2 CH3
¾¾¾® C6H5 ¾ C ºº C ¾ CH3
CH3 I B Br 2 /H2O ½
+ ¾¾¾® CH3 ¾ CH ¾ C — CH2 ¾ CH3
OH OH2 ½
½ ½ ½
H+ Br OH
64. (i) C6H5 ¾ CH ¾ C ºº CH ¾® C6H5 ¾ CH ¾ C ºº CH
[B]
– H2 O +
H+ C2H5
¾¾® C6H5 ¾ CH ¾ C ºº CH ¾®

w
H2 O Hg(OAc) 2 /H2 O ½
¾¾¾¾¾® CH3 ¾ CH2 ¾ C — CH2 CH3
C6H5 ¾ CH == CH ¾ CHO e
(Tautomerisation)
C6H5 ¾ CH == C == CH
½
OH
NaBH4
½
OH

Flo
unstable enol [C ]
69. H3C CH3
H+ C==C + Br2

ree
(ii) + H cis H
OH O CH3 Br H CH3

F
H Br
H C C
CH3

or
C C
H
ur + H3C
H
+
Br

f
Br CH3
O CH3 H
Pair of enantiomers
ks
65. Oxidation product indicates that alkene is symmetrical : O CH2
Yo
½½ ½½
oo
H+ 70. CH3 ¾ C ¾ CHO + O== C ¾ H ¬¾ CH3 C ¾ CH == CH2
+ KMnO4 2CH3CH2COOH
3-hexene 2-ketopropanal ½ E
eB

‘A’ H
Formaldehyde
Ph H Gas A
71. CH3 ¾ CH == CH ¾ CH3 ¾¾¾® 2CH3 ¾ CHO
Zn - H2 O
r

66. (i) C==C + KNH2 Ph—C C—Ph


\ Gas A is ozone (O3 )
ou
ad

A
Ph Br Also; Gas A + KOH (dry) ¾® B + Gas C
Y

R R deep red coloured


\ 4O3 + 4KOH ¾® 4KO3 + 2H2O + O2
(ii) + HClO4 2R—COOH B C
nd
Re

B 72. (i) A must be a symmetrical alkene :


O Br
Fi

Br ½
CH3 CH==CHCH3 + HBr ¾® CH3 CH2 ¾ CH ¾ CH3
CH3 CH3
(iii) + CHBr3 + t-BuOK ½ ½
(ii) CH3 ¾ C == CH2 + H2SO4 + H2O ¾® CH3 ¾ C — CH3
C ½
OH
CH3 CH3 Achiral
Cl (CH3)3COK
CH3 CH2 CH3
67.
H3C H3C ½ Pt
(iii) CH3 ¾ CH2 ¾ C — CH == CH 2 + H2 ¾®
CH3 CH3 D
‘X’ ‘Y’ ½
H
CH3 CH3 has one chiral carbon CH3
CH3 CH3 ½
H2/Pt CH3CH2 ¾ C — CH 2CH 3
+ H3C H3C ½
CH3 CH3 H
‘Z’
Achiral
326 Hydrocarbons

73. CH2 == CH2 + KMnO4 + H2O ¾® CH2 ¾ CH2 + MnO2 79. (i) Free radical chlorination of alkane require energy which is
½ ½ supplied either in the form of heat or radiation.
OH OH (ii) Addition of HBr proceeds through carbocation intermediates.
Ethylene glycol + -
CH3 ¾ CH == CH2 + H+ ¾® CH3 ¾ CH ¾ CH3 ¾®
Br
74. Baeyer’s reagent (cold, dilute, alkaline permanganate) can be used 2 ° carbocation
to distinguish between alkanes and alkenes. Alkenes decolourises
purple colour of Baeyer’s reagent while alkanes do not. CH3 ¾ CH ¾ CH3
½
75. The alkenes are : Br
Isopropyl bromide
CH3 ¾ CH == O + O == CH ¾ CH(CH3 )2
CH3 CH3 D CH ¾ CH ¾ CH + MnO
80. CH3CH == CH2 + KMnO4 (aq) ¾® 3 2 2
½ ½ ½ ½

w
CH3 ¾ CH == CH ¾ CH — CH3 ; CH3CH2CH ==O + O == C ¾ CH3 OH OH
I (B )
CH3 81. Ozonolysis products are the key of identification :
½ CH3 CH3

Flo
CH3 ¾ CH2 ¾ CH== C — CH3 ½ ½
O3
II (C ) CH3 ¾ C== O + O == CH ¾ CH3 ¬¾¾ CH3 ¾ C== CH ¾ CH3
Since, both alkenes I and II are obtained by b-elimination of Propanone Ethanal Zn -H2 O A

ree
same halides, the halides must be :
CH3 Other products are:
½ CH3

F
CH3 ¾ CH2 ¾ CH ¾ CH — CH3 ½ dil. KMnO4 Br 2
½ CH3 ¾ C — CH ¾ CH3 ¬¾¾¾¾ A ¾®
Cl -
½ ½ Cold, OH

or
ur
(A ) OH OH CH3 Br
Heat ( D )

f
3 CH I ½ ½
76. H ¾ C ºº C ¾ H + Na ¾¾¾® ¾¾® CH3 ¾ C ºº CH CH3 ¾ C — CH — CH3
ks
O ½
½½ Br
Yo
H2 SO4
¾¾¾® CH3 ¾ C ¾ CH3 O
oo
HgSO4 Acetone 1 Ag
77. 1-butyne (terminal) can be distinguished from 2-butyne 82. (i) CH2 == CH2 + O2 ¾¾® H2C ¾¾ CH2
eB

2 heat Ethylene oxide


(internal) by either Tollen’s test or through Fehling’s test. (oxirane)
AgNO3 +
NH3(aq)
CH3—CH2—C ºº CAg¯ (ii) CH ºº CH + H2SO4 ¾¾®
4 HgSO
CH3 ¾ CHO ¾¾®
dil. OH
r

White ppt.
CH3—CH2—C ººC—H
ou

OH
ad

CuCl2
CH3—CH2—C ºº CCu¯ ½
NH3(aq) CH3 ¾ CH ¾ CH2 ¾ CHO
Y

Red ppt.
(Aldol)
Peroxide
78. (i) CH3 ¾ C == CH2 + HBr ¾¾¾® CH3 ¾ CH ¾ CH2Br NaNH
nd

2
83. CH2 == CH2 + Br2 ¾® CH2 ¾ CH2 ¾¾¾®
Re

½ ½
CH3 CH3 ½ ½ H ¾ C ºº C ¾ H
2-methyl propene Isobutyl bromide Br Br
Fi

In the absence of peroxide, HBr would be added giving 84. Acetylene can be distinguished from methane using Tollen’s reagent :
tertiary butyl bromide. NH (aq)
(ii) Tertiary alkynes are slightly acidic, forms silver salt with C2H2 + AgNO3 ¾¾®
3
H ¾ C ºº CAg ¯
White ppt.
ammoniacal solution of silver nitrate :
No such reaction occur with methane.
NH ( aq )
R ¾ C ºº C ¾ H + AgNO3 ¾¾®
3
R ¾ C ºº CAg ¯
White ppt.

Download Chapter Test


http://tinyurl.com/yyjzocvw or
23
Alkyl Halides

w
1. Heating of 2-chloro-1-phenyl butane with EtOK/EtOH gives 5. Which of the following potential energy (PE) diagrams
X as the major product. Reaction of X with represents the SN 1reaction? (2019 Main, 9 April II)
Hg(OAc)2 / H 2O followed by NaBH 4 gives Y as the major

Flo
product. Y is (2019 Main, 12 April II)
OH (a) PE (b) PE

ree
(a) Ph (b) Ph
Progress of reaction Progress of reaction

F
OH
OH

or
(c) (d)
(c) Ph (d) Ph
ur PE PE

f
2. Which one of the following is likely to give a precipitate with Progress of reaction Progress of reaction
AgNO3 solution?
ks
(2019 Main, 12 April II)
(a) CH2 == CH ¾ Cl (b) CCl 4 6. The major product of the following reaction is
Yo
(c) CHCl 3 (d) (CH3 )3 CCl
oo
CH2CH3
3. The major product Y in the following reaction is
eB

(2019 Main, 10 April II) NaOEt


H3C C Cl D
Cl EtONa HBr
X Y
Heat COOCH2CH3
r
ou
ad

(2019 Main, 12 Jan II)


Br
(a) Br (b) (a) CH3 CH2C == CH2
½
Y

CO2CH2CH3
(b) CO2CH2CH3
nd

HO
½
Re

(c) (d)
CH3 C == CHCH3
Fi

Br CH2CH3

4. The major product of the following reaction is (c) H3C C OCH2CH3


CH3
½ CH OH COOCH2CH3
CH3 ¾ C ¾ CHCH3 ¾ ¾3 ¾
¾®
½ ½ OCH2CH3
H Br (2019 Main, 10 April I)
CH3 CH3 (d) H3C H2C C CO2CH2CH3
½ ½
(a) CH3 ¾ C == CHCH3 (b) CH3 ¾ C ¾ CH == CH 2 CH3
½
H
CH3
7. The major product in the following conversion is
CH3
½ ½ HBr (excess)
(c) CH3 ¾ C ¾ CH 2 CH3 (d) CH3 ¾ C ¾ CHCH3 CH3O CH CH CH3 Heat ?
½ ½ ½
OCH3 H OCH3 (2019 Main, 12 Jan II)
328 Alkyl Halides

12. The major product of the following reaction is


(a) CH3O CH CH2 CH3

Br CH3O
CH2Cl
(i) AlCl3 (anhyd.)
(ii) H2O
(b) HO CH2 CH CH3
(2019 Main, 10 Jan I)
Br

(c) CH3O CH2 CH CH3 CH3O CH3O


(a) (b)
Br CH3
(d) HO CH CH2 CH3 OCH3

w
CH3 CH3O
Br (c) (d)
8. The major product of the following reaction is

Flo
13. The major product of the following reaction is
(i) KOH alc.
CH3 CH2 CH ¾ CH2 ¾ ¾ ¾ ¾ ¾ ¾®
½ ½ (ii) NaNH2 Br

ree
Br Br in liq. NH3
(2019 Main, 12 Jan II) NaOMe
(a) CH3CH2 C H ¾ CH2 (b) CH3CH == CHCH2NH2 MeOH

F
½ ½ (2018 Main)
NH2 NH2
OMe
(c) CH3CH == C == CH2 (d) CH3CH2C ºº CH

or
9. The major product of the following reaction is
ur (a) (b)

f
(2019 Main, 12 Jan I)
CH3O
ks
OMe
(i) Cl2/CCl4
Yo
(c) (d)
(ii) AlCl3 (anhyd.)
oo
Cl
14. The increasing order of reactivity of the following halides for
eB

the SN 1 reaction is (2017 Main)


CH3O CH3O
(a) (b) I. CH3CH(Cl)CH2CH3
Cl II. CH3CH2CH2Cl
r

Cl III. p-H3CO ¾ C6H4 ¾ CH2Cl


ou
ad

CH3O (a) (III) < (II) < (I)


(c) (d) CH3O
Y

(b) (II) < (I) < (III)


Cl (c) (I) < (III) < (II)
nd

(d) (II) < (III) < (I)


Re

10. Which hydrogen in compound (E) is easily replaceable


during bromination reaction in presence of light? 15. Which of the following, upon treatment with tert-BuONa
Fi

CH3 ¾ CH 2 ¾ CH ==CH 2 (2019 Main, 10 Jan I) followed by addition of bromine water, fails to decolourise
d g b a the colour of bromine ? (2017 Main)
(E) C6H5
O
(a) b-hydrogen (b) d-hydrogen
(a) (b)
(c) g-hydrogen (d) a-hydrogen
Br Br
11. The major product of the following reaction is
Br O
KOH alc. (excess)
D
(c) (d)
Ph Br Br
Br 16. 3-methyl-pent-2-ene on reaction with HBr in presence of
(2019 Main, 10 Jan I)
peroxide forms an addition product. The number of possible
stereoisomers for the product is (2017 Main)
(a) Ph (b) Ph (a) six (b) zero
(c) two (d) four

(c) Ph (d) Ph
Alkyl Halides 329

17. The major product obtained in the following reaction is 23. The major product of the following reaction is (2008, 3M)
Br H3C Br
H t
BuOK
C6H5 D
C6H5
(2017 Main)
F
PhSNa
t
(a) (± ) C6 H5CH(O Bu)CH2C6 H5 dimethyl formamide
(b) C6 H5CH == CHC6 H5
(c) (+ ) C6H5CH(Ot Bu)CH2C6 H5 NO2
(d) (- )C6 H5CH(Ot Bu)CH2C6 H5 H 3C SPh H 3C SPh
18. 2-chloro-2-methylpentane on reaction with sodium
F F
methoxide in methanol yields (2016 Main)

w
(a) (b)
CH3
½
I. C2 H5 CH2 C ¾ OCH3 II. C2 H5 CH2 C == CH2 NO2 NO2
½

Flo
½
CH3 CH3 H 3C Br H 3C SPh

III. C2 H5 CH == C ¾ CH3

ree
SPh SPh
½ (c) (d)
CH3

F
(a) Both I and III (b) Only III
(c) Both I and II (d) All of these NO2 NO2

or
19. The product of the reaction given below is
ur (2015 Adv.)
24. The following compound on hydrolysis in aqueous acetone
will give

f
(2005, 1M)
(i) NBS/hn
ks
X CH3 H CH3
(ii) H2O/K2CO3
Yo
MeO— —NO2
oo

OH O CO2H
eB

H Cl CH3
(a) (b) (c) (d) CH3 H CH3
(K) MeO— —NO2
r
ou
ad

20. The synthesis of alkyl fluorides is best accomplished by H OH CH3


(2015 Main)
CH3 H CH3
Y

(a) free radical fluorination (b) Sandmeyer’s reaction


(c) Finkelstein reaction (d) Swarts reaction (L) MeO— —NO2
nd

21. In SN 2 reactions, the correct order of reactivity for the


Re

following compounds CH3 Cl,CH3 CH2 Cl, (CH3 )2 CHCl and OH H CH3
(CH3 )3 CCl is
Fi

(2014 Main)
CH3 H CH3
(a) CH3Cl > (CH3 )2 CHCl > CH3CH2Cl > (CH3 )3 CCl
(b) CH3Cl > CH3CH2 Cl > (CH3 )2 CHCl > (CH3 )3 CCl (M) MeO— —NO2
(c) CH3CH2Cl > CH3Cl > (CH3 )2 CHCl > (CH3 )3 CCl
H CH3 OH
(d) (CH3 )2 CHCl > CH3CH2Cl > CH3Cl > (CH3 )3CCl
It mainly gives
22. KI in acetone, undergoes S N 2 reaction with each P, Q, R and (a) K and L (b) only K
S. The rates of the reaction vary as (2013 Adv.) (c) L and M (d) only M

O 25. The product of following reaction is


Cl OH
H3C—Cl —Cl
Cl C2H5O–
+ C2H5I
P Q R S anhy. C2H5OH
(2003, 1M)
(a) P > Q > R > S (b) S > P > R > Q (a) C6H5OC2H5 (b) C2H5OC2H5
(c) P > R > Q > S (d) R > P > S > Q (c) C6H5OC6H5 (d) C6H5I
330 Alkyl Halides

26. Identify the set of reagents/reaction conditions X and Y in the Objective Questions II
following set of transformations– (2002, 3M)
(One or more than one correct option)
X
CH3 ¾ CH2 ¾ CH2 Br ¾® Product ¾®
Y CH ¾ CH ¾ CH
3 3 37. For the following compounds, the correct statement(s) with
½ respect to nucleophilic substitution reaction is(are)
Br
(2017 Adv.)
(a) X = dilute aqueous NaOH, 20°C,Y = HBr/acetic acid, 20°C CH3 CH3
(b) X = concentrated alcoholic NaOH, 80°C,
Y = HBr/acetic acid, 20°C Br Br
H 3C C Br
Br
(c) X = dilute aqueous NaOH, 20°C, Y = Br2 / CHCl 3 , 0°C CH3
(d) X = concentrated aqueous NaOH, 80°C, I II III IV
Y = Br2/CHCl 3 , 0°C
(a) Compound IV undergoes inversion of configuration

w
27. The compound that will react most readily with NaOH to (b) The order of reactivity for I, III and IV is : IV > I > III
form methanol is (2001, 1M) (c) I and III follow S N 1 mechanism
(a) (CH3 )4 N + I – (b) CH3OCH3 (d) I and II follow S N 1 mechanism

Flo
(c) (CH3 )3 S+ I – (d) (CH3 )3 Cl 38. Compound(s) that on hydrogenation produce(s) optically
28. An SN 2 reaction at an asymmetric carbon of a compound inactive compound (s) is/are (2015 Adv.)

ree
always gives (2001) H Br H Br
(a) an enantiomer of the substrate (a) (b) H2C CH3
(b) a product with opposite optical rotation H3C CH3

F
(c) a mixture of diastereomers
H Br Br H
(d) a single stereoisomer
(c) H2C (d) H2C CH3
29. The order of reactivities of the following alkyl halides for a

or
SN 2 reaction is–
ur (2000, 1M) CH3
CH3

f
(a) RF > RCl > RBr > RI (b) RF > RBr > RCl > RI
39. In the following reaction, the major product is (2015 Adv.)
(c) RCl > RBr > RF > RI (d) RI > RBr > RCl > RF
ks
CH3
30. Which of the following has the highest nucleophilicity? CH2 1 equivalent HBr
Yo
oo
(2000) H2C
(a) F - (b) OH - (c) CH3- (d) NH2- CH3 CH3
eB

CH3
31. A solution of (+)-2-chloro-2-phenylethane in toluene (a)
H2C
(b)
H3C
racemises slowly in the presence of small amount of SbCl5, Br
Br
due to the formation of (1999)
CH3 CH3
r

(a) carbanion (b) carbene


ou

(c)
ad

(c) free-radical (d) carbocation Br (d)


H2C H3C Br
32. Which of the following is an organometallic compound?
Y

(a) Lithium methoxide (b) Lithium acetate (1997)


40. The compounds used as refrigerant are (1990, 1M)
(c) Lithium dimethylamide (d) Methyl lithium (a) NH3 (b) CCl 4 (c) CF4 (d) CF2Cl 2
(e) CH2F2
nd

33. (CH3 )3 CMgCl on treatment with D2 O produces


Re

(1997, 1M)
(a) (CH3 )3 CD (b) (CH3 )3 COD (c) (CD)3 CD (d) (CD)3 COD Assertion and Reason
Fi

34. 1-chlorobutane on reaction with alcoholic potash gives 41. An Assertion and a Reason are given below. Choose the
(a) 1-butene (b) 1-butanol (1991, 1M)
correct answer from the following options.
(c) 2-butene (d) 2-butanol
Assertion (A) Vinyl halides do not undergo nucleophilic
35. n-propyl bromide on treatment with ethanolic potassium
substitution easily.
hydroxide produces (1987, 1M)
(a) propane (b) propene (c) propyne (d) propanol Reason (R) Even though the intermediate carbocation is
stabilised by loosely held p-electrons, the cleavage is
36. The reaction condition leading to the best yield of C2 H5 Cl
difficult because of strong bonding. (2019 Main, 12 April II)
are– (1986, 1M) (a) Both (A) and (R) are wrong statements.
UV light (b) Both (A) and (R) are correct statements and (R) is correct
(a) C2H6 (excess) + Cl 2 ¾¾¾®
explanation of (A).
dark
(b) C2H6 + Cl 2 (excess) ¾¾¾® (c) Both (A) and (R) are correct statements but (R) is not the
room temp.
correct explanation of (A).
UV light
(c) C2H6 + Cl 2 (excess) ¾¾¾® (d) (A) is a correct statement but (R) is a wrong statement.
UV light
(d) C2H6 + Cl 2 ¾¾¾®
Alkyl Halides 331

Match the Columns 44. Match the following : (2006, 3M)

42. List-I contains reactions and List-II contains major products. Column I Column II
A. CH3 ¾ CHBr ¾ CD3 on p. E1 reaction
List-I List-II treatment with alc. KOH gives
P. 1. CH2 == CH ¾ CD3 as a major
+
ONa Br OH product.
B. Ph—CHBr—CH3 reacts faster q. E2 reaction
Q. + HBr
2. than Ph—CHBr—CD3
Br
OMe C. Ph—CH2 ¾ CH2Br on treatment r. E1CB
with C2H5 OD /C2H5 O- gives reaction
R. + NaOMe
3.
Ph ¾ CD ==CH2 as the major

w
Br OMe
product.
S. + MeBr 4. D. PhCH2CH2Br and PhCD2CH2Br s. First order
ONa react with same rate. reaction

Flo
5. O
Fill in the Blanks

ree
45. Vinyl chloride on reaction with the dimethyl copper
Match each reaction in List-I with one or more products in gives ………. (1997)

F
List-II and choose the correct option. (2018 Adv.) 46. The starting material for the manufacture of polyvinyl
(a) P ® 1, 5; Q ® 2; R ® 3; S ® 4 chloride is obtained by reacting HCl with …… (1983, 1M)
(b) P ® 1, 4; Q ® 2; R ® 4; S ® 3

or
(c) P ® 1, 4; Q ® 1,2; R ® 3,4; S ® 4
ur 47. The halogen which is most reactive in the halogenation of
alkanes under sunlight is … (1981 , 1M)

f
(d) P ® 4, 5; Q ® 4; R ® 4; S ® 3,4
True/False
ks
43. Match the chemical conversion in Column I with the
appropriate reagents in Column II and select the correct
Yo
48. Photobromination of 2-methylpropane gives a mixture of
oo
answer using the code given below the lists. (2013 Adv.) 1-bromo-2-methyl propane and 2-bromo-2-methyl propane
in the ratio 9 : 1. (1993, 1M)
eB

Column I Column II
49. During S N 1 reactions, the leaving group leaves the molecule
1. (i) Hg(OAc)2 ; before the incoming group is attached to the molecule.
Cl
P. (ii) NaBH4 (1990, 2M)
r

50. The reaction of vinyl chloride with hydrogen iodide to give


ou
ad

ONa OEt 1-chloro-1-iodoethane is an example of anti-Markownikoff’s


Q. 2. NaOEt rule. (1989, 2M)
Y

51. Iodide is better nucleophile than bromide. (1985)


OH
52. Carbon tetrachloride is inflammable. (1985, 1/ 2M)
nd

R. 3. Et-Br
Re

53. Carbon tetrachloride burns in air when lighted to give


phosgene. (1983, 1M)
Fi

4. (i) BH3 ; Integer Answer Type Question


S. (ii)H2 O2 / NaOH
54. In the following monobromination reaction, the number of
OH
possible chiral product(s) is (are)...
Codes CH 2 CH 2 CH 3
P Q R S Br 2 (1.0 mole)
H Br
(a) 2 3 1 4 300°C
(b) 3 2 1 4 CH3
(c) 2 3 4 1 (1.0 mole) (Enantiomerically pure)
(d) 3 2 4 1
55. The total number of alkenes possible by dehydrobromination
of 3-bromo-3-cyclopentylhexane using alcoholic KOH is
(2011)
332 Alkyl Halides

Subjective Questions 62. Draw the stereochemical structure of product in the


following reaction
56. Identify X, Y and Z in the following synthetic scheme and
write their structures CH3
NaOH
2 (i) NaNH Br ¾¾¾ H ¾¾¾® (1994)
CH3 CH2 C ºº CH ¾¾¾¾¾® X SN 2
(ii) CH3 CH2 Br
C2 H5
H2 / Pd - BaSO4 alkaline
X ¾¾¾¾¾® Y ¾¾¾¾® Z 63. Aryl halides are less reactive than alkyl halides towards
KMnO4 (2002, 5M) nucleophilic reagents. Give reason. (1994, 2M)
57. What would be the major product in each of the following 64. Identify the major product in the following reaction.
reactions? H

w
CH3 Alc. KOH HBr
C6 H5 ¾ CH2 ¾¾¾ CH3 ¾¾¾® ? ¾¾® ? (1993, 1M)
½ C2 H5 OH D
(i) CH3 ¾ C ¾ CH2 Br ¾¾¾® (2000, 2M) Br
½ D

Flo
CH3 65. Write the structures of the major organic product expected
from each of the following reactions :
58. Write the structures of the products :

ree
CH3
Alc. KOH Alc. KOH
C6 H5 CH2 CHClC6 H5 ¾¾¾¾® (1998, 2M) (i) H3 C ¾¾¾ CH2 CH3 ¾¾¾®

F
59. Which of the following is the correct method for synthesising Cl
aq. alkali
methyl-t-butyl ether and why?

or
(ii) CH3 CH2 CHCl 2 ¾¾¾®
ur boil
(1992, 2M)
(CH3 )3 CBr + NaOMe ¾®

f
66. Arrange the following in order of their
or CH3 Br + NaO -t -Bu ¾® (1997, 2M)
(i) Increasing basicity
ks
60. Predict the structure of the product in the following reaction H2O, OH- , CH3OH, CH3O–
Yo
oo
Br (ii) Increasing reactivity in nucleophilic substitution reactions
H Ph CH3 F, CH3 I, CH3 Br, CH3 Cl (1992, 2M)
eB

NaI
(1996, 1M)
MeO H acetone 67. Chloroform is stored in dark coloured bottles. Explain in not
more than two sentences. (1980 , 1M)
Ph
68. Show by chemical equations only, how would you prepare
r
ou

61. Optically active 2-iodobutane on treatment with NaI in the following from the indicated starting materials? Specify
ad

acetone gives a product which does not show optical activity. the reagents in each step of the synthesis. (1979, 2M)
Y

Explain briefly. (1995, 2M) (i) Chloroform from carbon disulphide.


(ii) Hexachloroethane (C2Cl6) from calcium carbide.
nd
Re
Fi

Answers
1. (c) 2. (d) 3. (b) 4. (c) 33. (a) 34. (a) 35. (b) 36. (a)
5. (b) 6. (b) 7. (d) 8. (d) 37. (a, d) 38. (b,d) 39. (d) 40. (a,d)
9. (c) 10. (c) 11. (d) 12. (d) 41. (c) 42. (b) 43. (a)
13. (b) 14. (b) 15. (a) 16. (d) 44. A ® q, B ® q, C ® r, D ® p, s
17. (b) 18. (d) 19. (a) 20. (d) 45. propene 46. ethyne 47. chlorine 48. False
21. (b) 22. (b) 23. (a) 24. (a) 49. True 50. True 51. True 52. False
25. (a) 26. (b) 27. (a) 28. (d) 53. False 54. (5) 55. (5)
29. (d) 30. (c) 31. (d) 32. (d)
Hints & Solutions
1. Heating of 2-chloro-1-phenylbutane with EtOK/EtOH gives 3. The given reaction takes place as follows :
1-phenyl but-1-ene(X).
Cl
Reaction of X with Hg(OAc)2 / H2O followed by NaBH4 gives H
EtONa/D
1-phenyl butan-1-ol (Y). b-H for b-elimination
Saytzeff elimination –HCl
Reaction involved is as follows: CH2
H b-H for
Cl –+ Hoffmann elimination
EtOK
EtOH Markownikov’s
Ph Ph centre for Brs

w
+
OH
1. Hg (OAc)2,H2O 3-methyl but-2-ene 3-methyl-but-1-ene
major (X) minor
2. NaBH4
Ph (Saytzeff product) (Hoffmann product)
(Y)

Flo
Electrophilic addition
HBr
Mechanism (Markownikov’s rule)
Br

ree
Cl OAc
s
Br +
OEt
+ Hg OAc
Ph dehydro- 2-bromo-3-methyl 2-bromo-2-methyl

F
Ph
H halogenation butane (minor) butane (major) (Y)

Oxymercuration 4. In the given question, the substrate is a 2º-halide (bromide) and the

or
ur OAc
medium, CH3OH (as well as a poor nucleophile) is protic in nature,
So, the reaction will follow mainly S N1 pathways via the formation

f
of a carbocation intermediate (I).
Hg+
ks
Ph
H Me Me Me Me
Yo
r r CH3OH
–Br
C H C CH
oo
rds r
Me –H
H Me (2°)
Fast
H H C2H5 H Br H (I)
eB

H2O
Me
CH—CH—Me
Ph CH CH Hg OAc Me
OCH3
r

(Minor)
ou

OH C2 H5
ad

The intermediate, I can be rearranged into the more stable form I¢


(3°) by a-hydride shift. I ¢ will give the major product.
Y

H
s s Me
Me r a-H shift r
OAc C CH—Me C—CH2—Me
nd

Me Me
Re

Ph CH CH HgOAc
I¢ CH3OH –Hr
H
(3° carbocation)
Fi

OH C2H5 I Me
(2° carbocation)
C—CH2—Me
Demercuration NaBH4 Me
s OCH3
Ph CH CH2 + AcO
(Major)

OH C2H5 (Y)
5. The potential energy (PE) diagram for S N1 reaction is
2. (CH3 )3CCl gives a precipitate with AgNO3 solution because Carbocation
it forms stable carbocation. (i.e. tertiary) that readily combines intermediate
with AgNO3 to give precipitates of AgCl.
CH3
½ + PE R++X–
CH3 ¾ C ¾ Cl + AgNO3 ¾® (CH3 )3 C + AgCl –
+Nu
½ (White ppts.)
CH3 R¾ X
R ¾ Nu
CH2 == CH ¾ Cl forms unstable carbocation. Progress of reaction
Hence, it does not readily react with AgNO3.
334 Alkyl Halides

S N1 reaction has two steps. In the first step, the carbon halogen Key Idea Both alc. KOH and NaNH2 in liquid NH3 are
bond breaks heterolytically, with the halogen retaining the 8.
dehydrohalogenating reagents. On comparative terms
previously shared pair of electron. In the second step, the NaNH2/liquid NH3 is stronger in action.
nucleophile reacts rapidly with the carbocation formed in the
first step. The reaction proceeds as :
In the above graph, the alkyl halide is the only species that Alc. KOH
CH3 CH2 CH CH2
CH3 CH2 CH CH2
participates in the rate limiting step. Here, the rate of reaction
depends on the concentration of the alkyl halide and does not Br Br Br
depend on the concentration of nucleophile. NaNH2/liquid NH3

6. Presence of strong base (EtO- ) and heat indicates elimination. CH3 CH2 C CH
H+
CH3CH2C CNa+
Thus, the compound undergo dehydrohalogenation and alkene Thus, option (d) is the correct answer.
is produced. As per the position of Cl in substrate, following 2 9. The given reactant in presence of Cl2/CCl4, given vicinal

w
alkenes are formed as product: dihalide. Chlorine adds up to alkene via electrophilic addition
(i) CH3CH2 C == CH2 reaction involving cyclic chlorinium ion formation.
½ Cl

Flo
COOCH2CH3
H3CO H3CO
Cl2/CCl4
(ii) CH3 CH == C ¾ CH3
Cl
½

ree
COOCH2CH3 The vicinal dihalide in presence of anhyd. AlCl3 results in the
In accordance with Saytzeff rule formation of carbocation that rearranges itself to form a cyclic

F
compound.
CH3 CH == C ¾ CH3 will be the major product
Cl
½

or
H3CO
COOCH2CH3
ur + AlCl3
(Anhyd.)
Cl

f
Cl
7. Key Idea The excess of HBr and high temperature in given H3CO
reaction serves for dual purpose:
ks
+ –
r + AlCl
(i) Hydrolysis of ether via SN 2 mechanism, i.e. Zeisel’s 4
Yo
method.
oo

(ii) Markownikoff addition at double bond of the branch.


CH3O CH3O
eB

The road map of complete reaction is as follows:


–H+
Cl Cl
H
CH3 O CH CH CH3
r

10. The compound (E ) has two allyl-hydrogen atoms (g ). When E


ou

H+ HBr excess/Heat
ad

reacts with Br2 / hn, it readily undergoes allylic free radical


substitution and forms 3, 3-dibromobut-1-ene
Y

+
CH3 O CH CH2 CH3 d g b a Br / hn
C H3 ¾ C H2 ¾ C H == C H2 ¾ ¾2 ¾®
–HBr
Br– But-1-ene
nd

Br
Re

| Br / hn
CH3 O CH CH2 CH3 C H3 ¾CH ¾ C H ==C H2 ¾ ¾ ¾®
2

d g b a
Fi

–HBr
Br 3-bromo-but-1-ene
Br
H+ |
Br–
C H3 ¾ Cg ¾ C H == C H2
d b a
H |
d– +
Br
BF d BB d–
Br C O CH CH2 CH3 3,3- dibromobut-1-ene
BF
H H H d+ 11. The reaction follows a, b-elimination mechanism to give a more
Br
Transition state substituted stable alkene as a major product. As the substrate is a
a, g -dibromo (1, 3-) compound it gives a conjugated diene.
SN2 Reaction
(Zeisel’s Method)
Br
CH3Br + HO CH CH2 CH3 HOs H Alc. KOH
D
Br Ph H
–2HBr
(It is very stable because
Product
Br sOH the conjugated diene system
Here, BB = Bond breakage present. It is further conjugated
with the benzene ring.)
BF = Bonf formation
Alkyl Halides 335

12. In the given reaction, AlCl3 act as Lewis acid and helps in 15. To show decolourisation, compound must be unsaturated.
generation of carbocation. The resulting carbocation (1°) O – + O
tert-BuONa
rearranges itself to stable carbocation (3°).
MeO Br O—tBu
Cl (Saturated)
AlCl3(anhy.) (cannot decolourise Br2 water)
C6H5 – + C6H5
r s
MeO Cl AlCl3 tert–BuONa
MeO s
–AlCl4
H r Br
(Unsaturated)
(1°) (decolourise Br2 water)
MeO O – + O
a-Hs shift r
tert–BuONa

w
(H2O)
Br
(Unsaturated)
3° carbocation, more stable (decolourise Br2 water)
Protic medium, H2O supports the O O
rearrangement of carbcation

Flo
– +
Intramolecular ArSE2 pathway tert–BuONa
r
Me O MeO
Br

ree
–Hr
(Unsaturated)
(decolourise Br2 water)
H

F
16. The number of stereoisomers in molecules which are not
13. Complete reaction can be represented as divisible into two equal halves and have n number of
asymmetric C-atoms = 2n.

or
Br
Na+OMe

ur + NaBr + MeOH
3-methyl-pent-2-ene on reaction with HBr in presence of peroxide
forms an addition product i.e. 2-bromo-3-methyl pentane. It has

f
MeOH
H two chiral centres. Therefore, 4 stereoisomers are possible
ks
Thus, the given reaction is dehydrohalogenation which is a Br
Yo
2 4 2 4
b-elimination proceeding through E2 mechanism.
oo
1 3 HBr 1 3
5 5
Mechanism The reaction proceeds through the formation of Peroxide
following transition state with simultaneous removal of Br and
eB

3-methylpent-2-ene Anti-Markownikoff's addition


H atoms. Four stereoisomers are possible
Na+ Na+Br– (As molecule has two chiral
centres and asymmetric).
BF (Product)
r

BB Br 17. An alkyl halide in presence of a bulkier base removes a proton


ou
ad

Here, BB = Bond breakage from a carbon adjacent to the carbon bonded to the halogen. This
BF
BF = Bond formation reaction is called E2 ( b-elimination reaction).
Y

– Br
(Product) BB H
BF
OMe H C6H5
(Transition t-BuO–K+ H
C==C
nd

State) C6H5
Re

C6H5 H C6H5
MeOH (Product)
H + tert-BuOH + Br–
14. (i) The rate of S N 1 reaction depends only upon the concentration H
Fi

of the alkyl halide.


-
(ii) S N 1reaction proceeds through the formation of carbocation. 18. Key Idea Strong nucleophile ( OMe) in polar solvent (MeOH)
The reactivity is decided by ease of dissociation of alkyl halide. gives elimination products over substitution products but all
R ¾ X q RÅ + X È products are possible in different yields.
Higher the stability of R + (carbocation), higher would be the
Cl OCH3
reactivity towards S N1 reaction. -+
p - H3CO ¾ C6H4 ¾ CH2Å is the most stable carbocation due CH3—C—CH2CH2CH3
MeONa
CH3—C—CH2—CH2—CH3 +
to resonance and then CH3 CHCH2CH3 (2° carbocation) while MeOH
Å Å
CH3 CH3
CH3CH2 CH2(1°) is least stable.
(Less yield)
Thus, the correct increasing order of the reactivity of the given
halides towards the S N 1reaction is CH3 —C== CH—CH2—CH3+ CH2 ==C—CH2—CH2—CH3
CH3CH2CH2Cl < CH3 CHCH2CH3 < p-H3COC6H4CH2Cl
| CH3 CH3
Cl (More yield)
(II) (I) (III)
336 Alkyl Halides

19. NBS 23. Nucleophile PhS- substitute the Br - through SN 2 mechanism


hv wih inversion of configuration at a-C.
H3C Br
H3C SPh
Br Br
HO OH H O/K CO F
2 2 3 + PhS
– F
+
SN2

20. Alkyl fluorides can be prepared by action of mercurous fluoride NO2


NO2
or antimony trifluorides (inorganic fluorides) on corresponding
alkyl halide. This reaction is known as Swarts reaction. 24. Reaction proceed through carbocation intermediate

w
CH3 H CH3
CH3Br + AgF ¾® CH3F + AgBr
Methyl fluoride
CH3O— —NO2 – Cl+
But, when aciton of NaI/acetone takes place on alkyl chloride of
bromide, alkyl iodide forms. This reaction is called ‘Finkelstein

Flo
reaction’ H Cl CH3

C2H5Cl ¾¾®
NaI C2H5I + NaCl CH3 H CH3

ree
acetone
+
Free redical fluorination is highly explosive reaction, so not CH3O— —NO2 H shift
preferred for the preparation of fluoride. +

F
21. Steric hindrance (crowding) is the basis of SN 2 reaction, by H CH3
2°(I)
using which we can arrange the reactant in correct order of their CH3 H CH3

or
reactivity towards SN 2 reaction.
ur I
H2O
K
1 CH3O— —NO2

f
H2O
Rate of S N 2 µ II L +
Steric crowding of 'C'
ks
CH3 H CH3
Yo
| 3°(II)
CH3Cl > CH3CH2Cl > CH3 ¾ CH ¾ Cl > CH3 ¾ C ¾ Cl
oo
stabilised by + R effect of – OCH3
(Less 1° | |
crowded) CH3 CH3 OH OC2H5
eB

2° (More crowded)
As steric hinderance (crowding) increases, rate of S N 2 reaction C2H5O–
decreases. 25. + C2H5I
anhy. C2H5OH
r

Note The order of reactivity towards SN 2 reaction for alkyl


ou

SN2 reaction
ad

halides is
Primary > Secondary > Tertiary Alcoholic NaOH
26. CH3 ¾ CH2 ¾ CH2Br ¾¾¾¾¾® CH3 ¾ CH == CH2
Y

halides halides halides


(1° ) (2 ° ) (3 ° ) 80 °C
HBr
¾¾¾® CH3 ¾ CH ¾ CH3
22. PLAN Acetone is an aprotic solvent and can dissolve both the
nd

½
Re

nucleophile and the substrate and thus SN2 reaction is favoured.


Also Br
CH3
Fi

1° 2° 3°
SN 2 Alkyl halides ½
SN1 27. H3C ¾ N + ¾ CH3I- ¾® CH3OH + (CH3 )3 N+ I-
½
S. O 1° alkyl halide but (C ¾ Cl). CH3
Cl B.E. is decreased by electron HO-
withdrawing [C 6H5CO ] group,
Lack of b-H on quaternary ammonium iodide leads to SN 2
(a case of I-effect). Thus,
reaction otherwise E2 elimination usually takes place.
maximum rate in SN 2 reaction
Q. 2° alkyl halide, rate is minimum 28. SN 2 reaction at asymmetric carbon occur with inversion of
Cl configuration and a single steroisomer is formed because the
reactant and product are not enthtiomer. Therefore the sign of
P. CH3 — Cl 1° alkyl halide optical rotation may or may not change.
R. 1° allylic halide but allylic 1° 29. If alkyl groups are same, the order of leaving ability of halides in
Cl
carbocation is resonance SN 2 reaction is
stabilised in SN 1reaction RI > RBr > RCl > RF
Thus, reactivity order is S > P > R > Q
Alkyl Halides 337

30. CH-3 , being the strongest base, has highest nucleophilicity. 40. Both NH3 and CF2Cl 2 are used as refrigerant.
Cl
½ + 41. Vinyl halide (CH2 ==CH ¾ Cl) do not undergo nucleophilic
31. C6H5 ¾ C ¾ CH3 + SbCl 5 l C6H5 ¾ C ¾ CH3 + SbCl6- T substitution reactions. This is because it forms highly unstable
½ ½ Å
H H carbocation (CH2 == CH). It cannot delocalise its p-electron. In
(I) vinyl halide C ¾ Cl bond possess double bond character also.
he planar carbocation (I), when return back, forms racemic s r
mixture of the starting compounds. CH2 CH Cl CH2 CH Cl

32. Compound in which metal is directly bonded to carbon, is


42.
known are organometallic compound, e.g. CH3Li. For P, i.e. –+
ONa + Br
CH3 CH3

w
½ D O ½ For this reaction 1 and 4 are probable products.
33. CH3 ¾ C - MgCl + ¾¾¾®2
CH3 ¾ C ¾ D
½ ½ Product 1 i.e., is formed due to substitution while
CH3 CH3 OH

Flo
product 4 i.e., is formed due to elimination. A tertiary
C2 H5 OH
34. CH3 ¾ CH2 ¾ CH2 ¾ CH2 ¾ Cl + KOH ¾¾¾®
carbocation i.e, + formed during the reaction. Remember for
CH3CH2 ¾ CH == CH2

ree
3° carbocation ions elimination product predominates.
1- butene
For Q, i.e. + HBr
35. An alkyl halide containing at least one b-H, on treatment with

F
OMe
ethanolic KOH, undergoes dehydrohalogenation, giving
alkene.
Correctly matched product for this reaction is 2 i.e., .

or
Ethanol
CH3 ¾ CH2 ¾ CH2 ¾ Br + KOH ¾¾¾®
ur The reaction proceeds as
Br

CH3 ¾ CH == CH2

f OMe
+ HBr MeOH +
ks
36. During chlorination of alkane, if excess of alkane is treated Br
with Cl 2 (g ) in presence of light or heat, chance of
Yo
mono-chlorination predominate. For R i.e., + NaOMe
oo
Br
UV-light
C2H6 (excess) + Cl 2 ¾¾¾® CH3CH2Cl + HCl
eB

Correctly matched product is 4 i.e., . It is a normal


37. (a) Both I and II are 1° halide, undergos SN 2 reaction. elimination reaction and proceeds as
(d) III is a tertiary halide, undergoes SN 2 reaction. I is benzylic
r

bromide, it is very reactive in SN 1also as if produces stable +–


+ NaOMe
ou

benzylic carbonation.
ad

Br (Alkali in
alcohol)
38. In both cases, hydrogenation of olefinic bond will render
Y

compound achiral as two identical ethyl group will come at the 3° alkyl halide preferes elimination.
a-carbon which was earlier chiral carbon. However, in (a) and
For S i.e., + MeBr
(c), chirality will be retained even after hydrogenation.
nd
Re

H
ONa
H Br Br
Fi

H2
(a) H3C H2C H3C CH3
Chiral The correct match is 3 i.e., OMe . The reaction proceeds as
H Br
H2
H Br + MeBr + NaBr
(b) H2C H3C – +
CH3 ONa OMe
CH3
CH3
CH3 43.
Chiral
3. Column I Column II Explanation
39. Since, there is no mention of temperature, room temperature
will be considered and thermodynamically controlled product NaOEt(2)
P. O Et (strong
would be the major product as: Cl
nucleophile) causes
CH3 CH3 dehydrohalogenation
+
CH2 H CH2 of 3° alkyl halide
CH2 Br– CH3
3º-carbocation Q. EtBr (3) 3° butoxide undergoes
ONa OEt
CH3 SN reaction with 1°
CH3 alkyl halide
CH2
CH3 CH3 Br
338 Alkyl Halides

R. OH (i) Hg(OAC) 2 Mercuration- 54. Given compound undergoes free-radical bromination under
(ii) NaBH4 demercuration adds given conditions, replacing H by Br.
(1) H2O by
Markownikoff’s rule
C* is chiral carbon.
without rearrangement CH2CH2CH3 CH2CH2CH3
S. (i) BH3 Hydroboro-oxidation Br2
H Br H Br
(ii) H2O 2 /OH- adds H2O by
anti-Markownikoff’s
OH (4) rule
CH3 CH2Br
I. Chiral
Thus, P — (2), Q — (3), R — (1), S — (4) Br
Alc. KOH CH2CH2CH3
44. A. CH3 ¾ CHBr ¾ CD3 ¾¾¾® CH2 == CH ¾ CD3

w
E2 CH—CH2CH3
Br Br
E2 reaction is a single-step reaction in which both H Br
deprotonation from b-C and loss of leaving group from a-C CH3
occur simultaneously in the rate-determining step. CH3

Flo
C-D bond is stronger than C—H bond, C—H is preferably II. Achiral III. Chiral
broken in elimination. Br

ree
B. Ph ¾ CHBr ¾ CH3 reacts faster than Ph ¾ CHBr ¾ CD3 in CH2—CH2CH2CH3
E2 reaction because in latter case, stronger C—D bond is to CH2— CH—CH3

F
H Br
be broken in the rate determining step. H Br
C2 H5 OD CH3

or
C. Ph ¾ CH2 ¾ CH2Br ¾¾¾¾®
-
ur
Ph ¾ CD == CH2
C2 H5 O
CH3
IV. Chiral V. Chiral

f
Deuterium incorporation in the product indicates E1CB
mechanism (III) has two chiral centres and can have two structures.
ks
C2 H5 O- -
CH2CH3 CH2CH3
Yo
Ph ¾ CH2 ¾ CH2Br l Ph ¾ CH ¾ CH2Br
oo
carbanion H Br H Br
C2 H5 OD
eB

H Br Br H
l Ph ¾ CHD ¾ CH2Br
I
D CH3 CH3
C2 H5 O- ½ (III) A (III) B
r

I ¾¾¾® Ph ¾ C ¾ CH2 ¾ Br ¾® Ph ¾ C == CH2 (IV) has also two chiral centres and can have two structures.
ou

-
½
ad

D CH3 CH3
Y

D. Both PhCH2CH2Br and PhCD2CH2Br will react at same


H Br H Br
rate in E1 reaction because C—H bond is broken in fast non
rate determining step. Also E1 reaction follow first order H Br Br H
nd
Re

kinetics.
CH3 CH3
45. Propene is produced (IV)A (IV)B
Fi

(CH3 )2 Cu + CH2 == CHCl ¾® CH3 ¾ CH== CH2 It has plane of symmetry thus, achiral.
46. Vinyl chloride is obtained by the reaction of HCl with ethyne. Thus, chiral compounds are five. I, III A, III B, IV B and V.

47. Chlorine is most reactive. 55. The substrate has three different types of B—H, therefore, first
three structural isomers of alkenes are expected as :
48. 2-bromo-2-methylpropane is formed as major product.
H H
49. In SN 1 reaction, leaving group is detached in the first step Br
forming carbocation intermediate. H
KOH
50. True + +
51. Larger the size of donor atom, greater is its polarisability, 3 types of B-H
I II III
stronger is the nucleophile.
52. False The last two alkenes II and III are also capable of showing
geometrical isomerism, hence two geometrical isomers for each
53. CCl 4 is fire retardent, used as fire-extinguisher. of them will be counted giving a total of five isomers.
Alkyl Halides 339

(i) NaNH2 Above equilibrium is established which has equilibrium constant


56. CH3CH2C ºº CH ¾¾¾¾¾® CH3CH2 ¾ C ºº C ¾ CH2CH3 equal to 1. Therefore, equilibrium mixture will have both the
(ii) CH3 CH2 Br X
enantiomers in equal amount giving racemic mixture.
C2H5
62. SN 2 reactions leads to inversion of configuration at a-C
H2/Pd–BaSO4 H H Alk. KMnO H OH
4
X C ==C CH3 CH3
C2H5 H OH NaOH
H5C2 Br H H OH
SN2
Y C2H5
meso-diol C2H5 C2H5
product with inverted
57. Unimolecular reaction occur configuration
CH3 CH3 63. X X+
½ D ½ +

w
CH3 ¾ C ¾ CH2 ¾ Br l CH3 ¾ C ¾ CH2

½ ½
CH3 CH3
primary carbocation

Flo
CH3 Aryl halide

methyl shift ½ Due to the above resonance phenomena, C—X bond acquire
¾¾¾¾® CH3 ¾ C ¾ CH2 CH3 partial double bond character and becomes difficult to break in

ree
+ the rate determining step of SN 2 reaction.
I
CH3 Alc. KOH
64. C6H5CH2 ¾ CH ¾ CH3 ¾¾¾® C6H5 ¾ CH == CH ¾ CH3

F
D
E1 ½
CH3—C==CH—CH3 Br
¯ HBr

or
I
ur CH3 C6H5 ¾ CH ¾ CH2 ¾ CH3
½
SN 1

f
CH3—C—CH2CH3 Br
CH3 CH3
ks
OC2H5 Alc. KOH ½
65. (i) H3C ¾¾¾ CH2CH3 ¾¾¾® CH3 ¾ C == CHCH3
Yo
Alc. KOH E2
oo
58. C6H5CH2 ¾ CH ¾ C6H5 ¾¾¾® C6H5 ¾ CH == CH ¾ C6H5 Cl
E2
½ OH
eB

Cl ½
Aq. NaOH
(ii) CH3CH2CHCl 2 ¾¾¾® [CH3CH2 ¾ CH ¾ OH]
CH3 boil unstable
½ E2 CH3 -H O
59. CH3 ¾ C ¾ Br + CH3 O- ¾¾® CH2 == C ¾¾¾®
2
CH3CH2CHO
r

½ CH3
ou

66. (i) H2O < CH3OH < HO- < CH3O-


ad

CH3
CH3 (ii) CH3F < CH3Cl < CH3Br < CH3I
Y

CH3
½ SN 2 ½ 67. Chloroform in presence of air and sunlight, oxidises slowly to
CH3 ¾ C ¾ O- + CH3 Br ¾¾® CH3 ¾ C ¾ OCH3 form a highly poisonous compound called phosgene
nd

½ ½
Re

hn
CH3 CH3 2CHCl 3 + O2 ¾¾® 2COCl 2 + 2HCl
(Phosgene)
Fi

60. Br I To prevent the above oxidation reaction, chloroform is kept in


H Ph Ph H dark bottles.
NaI
68. (i) CS2 + Cl 2 ¾® CCl 4 + S2Cl 2
MeO H acetone MeO H
SN2 CS2 + 2S2Cl 2 ¾® CCl 4 + 6S
Ph Ph
inversion of configuration Fe/H2 O
occur only at a-carbon
CCl 4 + 2 [H] ¾¾¾® CHCl 3 + HCl
(Chloroform)

61. H 3C CH3 (ii) CaC2 + H2O ¾® C2H2 + Ca(OH)2


– K=1
C—I + I I—C H /Ni Cl 2 (excess)
H H C2H2 ¾¾¾® C2H6 ¾¾¾¾® Cl 3C ¾ CCl 3
2
hn (Hexachloroethane)
H5C2 C2H5

Download Chapter Test


http://tinyurl.com/y54uy6l6 or
24
Alcohols and Ethers

w
Objective Questions I (Only one correct option) 5. The major product of the following reaction is
(2019 Main, 8 April I)
1. The major product of the following reaction is

Flo
(2019 Main, 10 April I)
O
O Br
HI (excess) NaBH4

ree
D MeOH, 25°C

F
NC O OH OH
Br OMe
OH I

or
(a) (b)
ur (a) (b)

NC
f OMe O
ks
NC OH OH
Yo
(c) (d)
I OH
oo

(c) (d)
eB

6. The major product of the following reaction is


NC I NC I (2019 Main, 8 April I)
OCH3
r

Conc. HBr (excess)


2. The synonym for water gas when used in the production of
ou
ad

Heat
methanol is (2019 Main, 10 April I)
(a) natural gas (b) laughing gas
Y

CH==CH2
(c) syn gas (d) fuel gas
Br OH
3. The major product of the following reaction is
nd

(a) (b)
Re

CH3 CH == CHCO2 CH3 ¾LiAlH


¾ ¾4 ® (2019 Main, 9 April I)
Fi

(a) CH3 CH == CHCH2 OH (b) CH3 CH2 CH2 CH2 OH Br—CHCH3 Br—CHCH3
(c) CH3 CH2 CH2 CO2 CH3 (d) CH3 CH2 CH2 CHO Br OH
(d)
4. The major product of the following reaction is (c)
(2019 Main, 9 April I)
1. PBr3 CH2CH2Br CH2CH2Br
OH 2. KOH (alc.)

OH
O
½
7. CH3CH 2 ¾ C ¾ CH3 cannot be prepared by
(a) (b) OH ½
Ph (2019 Main, 12 Jan I)
HO O
(a) CH3 CH2 COCH3 + PhMgX
(c) (d) (b) PhCOCH3 + CH3 CH2 MgX
(c) PhCOCH2 CH3 + CH3 MgX
O O (d) HCHO + PhCH(CH3 ) CH2 MgX
Alcohols and Ethers 341

8. The major product of the following reaction is


(2019 Main, 11 Jan II)
OH I
HO (a) (b)
(i) HCI
(ii) AlCl3 (Anhyd.) OH I

HO Cl I
(c) OH (d)
(a) (b)
I OH

Cl
12. The acidic hydrolysis of ether (X) shown below is fastest
HO
(d) when (2014 Adv.)
(c)

w
9. Which is the most suitable reagent for the following
transformation? (2019 Main, 10 Jan II)

Flo
Acid
OH OR OH + ROH
½
CH3 ¾ CH ==CH ¾ CH2 ¾ C H ¾ CH3 ¾®

ree
CH3 ¾ CH ==CH ¾ CH2CO2H
(a) Tollen’s reagent (b) I2 / NaOH

F
[X ]
(c) Alkaline KMnO4 (d) CrO2 Cl 2 / CS2
(a) one phenyl group is replaced by a methyl group
10. The major product of the following reaction is

or
ur (2019 Main, 10 Jan II) (b) one phenyl group is replaced by a para-methoxyphenyl
group

f
O CH3
(i) Dil. HCl/D
(c) two phenyl groups are replaced by two
ks
(ii) (COOH)2/
CH3O OH Polymerisation para-methoxyphenyl groups
Yo
(d) no structural change is made to X
oo
13. An unknown alcohol is treated with the “Lucas reagent” to
(a) —O
determine whether the alcohol is primary, secondary or
eB

n
tertiary. Which alcohol reacts fastest and by what mechanism?
(a) Secondary alcohol by SN 1 (2013 Main)
OH
(b) Tertiary alcohol by SN 1
r
ou

O— (c) Secondary alcohol by SN 2


ad

(b) —O
n (d) Tertiary alcohol by SN 2
Y

14. The major product of the following reaction is


OH
RCH2OH
nd
Re

Å
H (anhydrous)
(c) O
O
—O n
Fi

(a) a hemiacetal (b) an acetal


(c) an ether (d) an ester
OCH3 15. (I) 1, 2-dihydroxy benzene
O— (II) 1, 3-dihydroxy benzene
(d) —O
n (III) 1, 4-dihydroxy benzene
(IV) Hydroxy benzene
OCOCH3
The increasing order of boiling points of above mentioned
11. The major product formed in the following reaction is alcohols is (2006, 3M)
(a) I < II < III < IV (b) I < II < IV < III
O
(c) IV < I < II < III (d) IV < II < I < III
HI
Heat 16. The best method to prepare cyclohexene from cyclohexanol
O
(2018 Main) is by using (2005, 1M)
(a) conc. HCl + ZnCl 2 (b) conc. H3 PO4
(c) HBr (d) conc. HCl
342 Alcohols and Ethers

17. When phenyl magnesium bromide reacts with tert butanol, 23. In CH3 CH2 OH, the bond that undergoes heterolytic cleavage
which of the following is formed? (2005, 1M) most readily is (1988, 1M)
(a) Tert butyl methyl ether (a) C ¾ C (b) C ¾ O
(b) Benzene (c) C ¾ H (d) O ¾ H
(c) Tert butyl benzene
24. Hydrogen bonding is maximum in (1987, 1M)
(d) Phenol
CH3 (a) ethanol (b) diethyl ether
H+ Br2/CCl4 (c) ethyl chloride (d) triethyl amine
18. [F ] C4H8Br2
/ H2O 25. HBr reacts fastest with (1986, 1M)
H 3C OH 5 such products
are possible (a) 2-methyl propan-2-ol (b) propan-1-ol
(c) propan-2-ol (d) 2-methyl propan-1-ol

w
How many structures of F is possible? (2003, 1M)
26. An industrial method of preparation of methanol is
(a) 2 (b) 5 (c) 6 (d) 3
(1984, 1M)
19. Compound ‘A’ (molecular formula C3H8O) is treated with (a) catalytic reduction of carbon monoxide in presence of

Flo
acidified potassium dichromate to form a product ‘B’ ZnO-Cr2 O3
(molecular formula C3H6O) ‘B’ forms a shining silver mirror (b) by reacting methane with steam at 900°C with nickel
on warming with ammoniacal silver nitrate. ‘B’ when treated
catalyst

ree
with an aqueous solution of H2NCONHNH2 and sodium
(c) by reducing formaldehyde with LiAlH4
acetate gives a product ‘C’. Identify the structure of ‘C’.
(d) by reacting formaldehyde with aqueous sodium
(a) CH3 CH2 CH == NNHCONH2

F
(2002, 3M)
hydroxide solution
(b) H3 C ¾ C == NNHCONH2
½ 27. Diethyl ether on heating with conc. HI gives two moles of

or
CH3
ur (a) ethanol (b) iodoform (1983, 1M)
(c) ethyl iodide (d) methyl iodide

f
(c) H3 C ¾ C == NCONHNH2
½ 28. The compound which reacts fastest with Lucas reagent at
ks
CH3 room temperature is (1981 , 1M)
Yo
(a) butan-2-ol (b) butan-1-ol
oo
(d) CH3 CH2 OH + NCONHNH2
(c) 2-methyl propan-1-ol (d) 2-methyl propan-2-ol
20. 1-propanol and 2-propanol can be best distinguished by
eB

(a) oxidation with alkaline KMnO4 followed by reaction 29. Ethyl alcohol is heated with conc. H2 SO4 . The product
with Fehling solution (2001, 1M) formed is
(b) oxidation with acidic dichromate followed by reaction (a) CH3 COOC2 H5 (b) C2 H2 (1980 , 1M)
r

with Fehling solution (c) C2 H4 (d) C2 H6


ou
ad

(c) oxidation by heating with copper followed by reaction 30. Which of the following is soluble in water? (1980, 1M)
with Fehling solution
Y

(a) CS2 (b) C2 H5 OH


(d) oxidation with concentrated H2 SO4 followed by (c) CCl 4 (d) CHCl 3
reaction with Fehling solution
nd
Re

21. Which one of the following will most readily be dehydrated Objective Questions II
in acidic condition? (2000, 1M) (One or more than one correct option)
Fi

O OH OH 31. In the following reaction sequence, the correct structure (s)


of X is (are) (2018 Adv.)
(a) (b)
Me N3
O O (1) PBr3, Et2O
X
(2) NaI, Me2CO
(3) NaN3, HCONMe2
(c) (d) Enantiomerically
pure

OH OH Me OH Me OH
(a) (b)
22. The products of combustion of an aliphatic thiol (RSH) at
298 K are (1992)
(a) CO2 ( g ), H2 O( g ) and SO2 ( g )
(b) CO2 ( g ), H2 O( l ), and SO2 ( g )
(c) CO2 ( l ), H2 O( l ) and SO2 ( g ) (c) Me OH (d) Me OH
(d) CO2 ( g ), H2 O( l ) and SO2 ( l )
Alcohols and Ethers 343

32. The correct statement(s) about the following reaction Compound J upon reaction with KOH gives benzyl alcohol
sequence is (are) and a compound L, whereas K on reaction with KOH gives
(i) O2 CHCl 3 / NaOH only M.
Cumene (C9 H12 ) ¾¾¾®
+
P ¾¾¾¾® O
(ii) H 3O
H3C Ph
NaOH M=
Q (major) + R (minor), Q ¾¾¾® S
PhCH 2 Br (2016 Adv.) Ph H
(2008, 3 ´4M = 12M)
(a) R is steam volatile
(b) Q gives dark violet colouration with 1% aqueous 36. The structures of compounds J , K and L respectively, are
FeCl 3 solution (a) PhCOCH3 , PhCH2COCH3 and PhCH2COO- K+
(c) S gives yellow precipitate with (b) PhCHO, PhCH2CHO and PhCOO- K+
2, 4-dinitrophenylhydrazine (c) PhCOCH3 , PhCH2CHO and CH3COO- K+

w
(d) S gives dark violet colouration with 1% aqueous (d) PhCHO, PhCOCH3 and PhCOO- K+
FeCl 3 solution
37. The structure of compound I is
33. The following ether, when treated with HI produces

Flo
(1999, 3M) Ph CH3 H3C Ph
(a) (b)

ree
—O—CH2— + HI H Ph H Ph

Ph CH3 H 3C CH3

F
(c) (d)
(a) CH2I (b) CH2OH
H CH2Ph Ph H

or
ur 38. Compound H is formed by the reaction of

f
(c) I (d) OH
O
ks
(a) ; PhMgBr
Yo
34. The products of reaction of alcoholic silver nitrate with Ph CH3
oo
ethyl bromide are (1991, 1M)
(a) ethane (b) ethene O
eB

(c) nitroethane (d) ethyl alcohol (b) ; PhCH2MgBr


(e) ethyl nitrite Ph CH3

O
r

Assertion and Reason


ou

(c) ; PhCH2MgBr
ad

Read the following question and answer as per the direction Ph H


given below :
Y

O CH2
(a) Statement I is correct; Statement II is correct; Statement II
(d) ;
is a correct explanation of Statement I.
nd

Ph H Ph MgBr
Re

(b) Statement I is correct; Statement II is correct; Statement II


is not the correct explanation of Statement I.
Fill in the Blanks
Fi

(c) Statement I is correct; Statement II is incorrect.


39. Glycerine contains one ……… hydroxyl group. (1997, 1M)
(d) Statement I is incorrect; Statement II is correct.
35. Statement I Solubility of n-alcohol in water decreases 40. Aliphatic ethers are purified by shaking with a solution of
with increase in molecular weight. ferrous salt to remove ……… which are formed on prolonged
standing in contact with water. (1992, 1M)
Statement II The relative proportion of the hydrocarbon
part in alcohols increases with increasing molecular weight 41. A …… diol has two hydroxyl groups on …… carbon atoms.
(1986, 1M)
which permit enhanced hydrogen bonding with water.
(1988 , 2M) 42. Ethanol vapour is passed over heated copper and the product is
treated with aqueous NaOH. The final product is ………
Passage Based Questions (1983, 1M)

Passage 1 True or False


A tertiary alcohol H upon acid catalysed dehydration gives a 43. Sodium ethoxide is prepared by reacting ethanol with aqueous
product I. Ozonolysis of I leads to compounds J and K. sodium hydroxide. (1985, 1M)
344 Alcohols and Ethers

44. The yield of a ketone when a secondary alcohol is oxidised is 54. When t-butanol and n-butanol are separately treated with a
more than the yield of aldehyde when a primary alcohol is few drops of dilute KMnO4 in one case only, the purple
oxidised. (1983, 1M) colour disappears and a brown precipitate is formed. Which
of the two alcohols gives the above reaction and what is the
Subjective Questions brown precipitate? (1994, 2M)
45. O 55. Compound X (molecular formula, C5 H8 O) does not react
—OH
appreciably with Lucas reagent at room temperature but
+
gives a precipitate with ammoniacal silver nitrate with excess
H /D (i) O3 NaOH
X Y of MeMgBr, 0.42 g of X gives 224 mL of CH4 at STP.
(ii) Zn/CH3COOH Treatment of X with H2 in presence of Pt catalyst followed by
boiling with excess HI, gives n-pentane. Suggest structure
Identify X and Y. (2005, 2M) for X and write the equation involved. (1992, 5M)

w
46. An organic compound P having the molecular formula 56. Arrange the following in increasing order of boiling point :
C5H10O when treated with dil H2SO4 gives two compounds, n-butane, n-butanol, n-butylchloride, iso-butane. (1988, 1M)
Q and R both gives positive iodoform test. The reaction of

Flo
C5H10O with dil, H2SO4 gives reaction 1015 times faster than 57. How may be the following transformation be carried out (in
ethylene. Identify organic compound of Q and R. Give the not more than six steps)?
reason for the extra stability of P. “Ethyl alcohol to vinyl acetate.”

ree
(2004) (1986, 3M)

47. Cyclobutylbromide on treatment with magnesium in dry 58. Write down the main product of the following reaction :
ether forms an organometallic compound (A). The

F
2I /NaOH
organometallic reacts with ethanal to give an alcohol (B) Ethanol ¾¾¾¾® (1985, 1M)
after mild acidification. Prolonged treatment of alcohol (B)
59. Give a chemical test to distinguish between methanol and

or
ur
with an equivalent amount of HBr gives 1-bromo-1-
methylcyclopentane (C). Write the structures of (A), (B) and ethanol. (1985, 1M)

f
explain how (C) is obtained from (B). (2001, 5M) 60. Suggest a reason for the large difference between the boiling
points of butanol and butanal, although they have almost the
ks
48. Explain briefly the formation of products giving the
same solubility in water.
Yo
(1985, 2M)
structures of the intermediates.
oo
HCl 61. An alcohol A, when heated with conc. H2 SO4 gives an alkene
(i)
OH Cl B. When B is bubbled through bromine water and the product
eB

obtained is dehydrohalogenated with excess of sodamide, a


+ CH2Cl + etc. new compound C is obtained. The compound C gives D when
CH3 CH3 treated with warm dilute H2 SO4 in presence of HgSO4 . D can
r

also be obtained either by oxidising A with KMnO4 or from


ou

HCl
ad

(ii) acetic acid through its calcium salt. Identify A, B, C and D.


OH Cl (1983, 4M)
Y

only (1999, 3M)


62. State the conditions under which the following preparations
49. Write the structures of the products : are carried out. Give necessary equations which need not be
nd

HI (excess)
Re

balanced.
(CH3 )2 CH ¾ OCH3 ¾¾¾®
heat (1998, 2M) (i) Ethanol from acetylene
Fi

50. Give reasons for the following in one or two sentences. (ii) Lead tetraethyl from sodium-lead alloy
“Acid catalysed dehydration of t-butanol is faster than that of (iii) Methyl chloride from aluminium carbide (1983, 3M)
n-butanol. (1998, 2M) 63. A compound ‘X’ containing C, H and O is unreactive
51. 2, 2-dimethyloxirane can be cleaved by acid (H+ ). Write towards sodium. It does not add with bromine. It also does
mechanism. (1997, 2M) not react with Schiff’s reagent. On refluxing with an excess
of HI, ‘X’ yields only one organic product ‘Y’. ‘Y’ on
52. A compound D(C8 H10 O) upon treatment with alkaline
solution of iodine gives a yellow precipitate. The filtrate on hydrolysis yields a new compound ‘Z’ which can be
acidification gives a white solid E (C7 H6 O2 ). Write the converted into ‘Y’ by reaction with red phosphorus and
structures of D, E and explain the formation of E. (1996, 2M) iodine. The compound ‘Z’ on oxidation with potassium
permanganate gives a carboxylic acid. The equivalent weight
53. 3, 3-dimethylbutan-2-ol losses a molecule of water in the of acid is 60. What are the compounds ‘X’, ‘Y’ and ‘Z’ ?
presence of concentrated sulphuric acid to give Write chemical equations leading to the conversion of ‘X’ to
tetramethylethylene as a major product. Suggest a suitable ‘Y ’. (1981, 3M)
mechanism. (1996, 2M)
Alcohols and Ethers 345

64. An organic liquid ‘A’ containing C, H and O with boiling Integer Type Question
point 78°C, possessing a rather pleasant odour, on heating
with concentrated sulphuric acid gives a gaseous product ‘B’ 65. The number of hydroxyl group(s) in Q is (2015 Adv.)
with the empirical formula, CH2 . ‘B’ decolourises bromine Aqueous dilute
H+ KMnO4 (excess)
water as well as alkaline permanganate solution and takes up H P Q
Heat 0°C
one mole of H2 (per mole of B) in presence of finely divided HO
nickel at high temperature. Identify the substances A and B. H3C CH3
(1979 , 2M)

Answers
1. (d) 2. (c) 3. (a) 4. (c) 25. (a) 26. (a) 27. (c) 28. (d)

w
5. (d) 6. (b) 7. (d) 8. (a) 29. (c) 30. (b) 31. (b) 32. (b,c)
9. (b) 10. (c) 11. (d) 12. (c) 33. (a,d) 34. (c,e) 35. (c) 36. (d)
13. (b) 14. (b) 15. (c) 16. (b) 37. (a) 38. (b) 39. secondary 40. Peroxides

Flo
17. (b) 18. (d) 19. (a) 20. (c) 41. vicinal; same 42. aldol 43. False 44. False
21. (a) 22. (b) 23. (d) 24. (a) 65. (4)

ree
Hints & Solutions

F
1. The given reaction takes place as follows: 3. Key Idea LiAlH4 reagent is used for the reduction of ¾ CHO,

or
ur Cleavage-I O
| H+ | ½½

f
+
O O — C OCH3. It does not reduce double bonds.
HI (excess) H
ks
Cleavage-II
The reaction of an ester with LiAlH4 produces two alcohols, one
Cleavage-I
Yo
corresponding to the acyl portion of the ester and one
oo
+ H corresponding to the alkyl portion.
NC O H+ NC O
Cleavage-II O
eB

½½ LiAlH 4
ava
ge-
II
CH3CH == CH C — OCH3 ¾¾¾®
cle
via SN1 pathway
[via cleavage I]
CH3 CH == CHCH2OH + CH3OH
r

But -2-en -1-ol Methanol


ou

Thus, the major product of the given reactant


ad

I OH
+ CH3OH + s
I O
+ CH3 CH3I
½½
Y

+
+ CH (CH3)2CHI
+ (CH3)2CHOH CH3CH == CHC OCH3 in presence of LiAlH4 is
NC OH CH3 CH3 NC I CH3CH ==CH CH2OH and CH3OH. The reaction proceeds
nd

through following mechanism.


Re

Product-1 Product-2
Mechanism
Fi

Product-2 is formed because O Nucleophilic acyl


– substitution reaction
(i) Cleavage-I will give more stable aryl carbocation. CH3CH CH C OCH3 + H AlH3
(ii) Cleavage- I will give intermediate which is in conjugation s
with ring. O

2. The production of methanol from water gas is as follows: CH3CH CH C OCH3


ZnO + Cr2 O 3
[CO + H2 ] + H2 ¾¾¾¾¾® CH3OH O H
300 atm, 300º C
Water gas Methanol –
or CH3CH CH C + CH3O
syn gas
An aldehyde
It is an industrial process used for the production of methanol H Nu
cle H– –
op AlH
where volume ratio of the reactant gases is maintained as: h
rea ilic a 3 s
cti dd O
syn gas : H2 = 2 : 1 and ZnO-Cr2O3 act as catalysts on itio
n
Thus, water gas is also called syn gas because it is used for CH3 CH CH CH2OH
H+
CH3CH CH C H
H2 O
synthesis of methanol. Primary alcohol
H
346 Alcohols and Ethers

Key Idea PBr3 reagent is used for the substitution of —Br As conc. HBr is in excess. So, reaction will take place at both the
4. substituents.
group while alc. KOH reagent is used to carry out
elimination reaction. Mechanism
Step 1 Protonation of ether to form oxonium ion.
The given reaction proceed in following manner:
H
Step I In presence of PBr3, alcohols undergo substitution |r
s
reactions to give halides. Reagent PBr3 is usually generated O—CH3 O—CH3 + Br
H—Br
insitu by the reaction of red phosphorus with bromine.
PBr3
OH Br CH=
=CH2 CH=
=CH2
Oxonium ion

O O

w
Step 2 Attack of nucleophile at the protonated ether.
3-bromohexanone

Step II 3-bromohexanone in presence of alc. KOH undergoes H H


s
|r |+
elimination reaction and gives cyclohex-2-en-1-one. O—CH3 O CH3

|
Flo
SN2
Alc. KOH Br-+ Slow
Br Br

ree
CH=
=CH2 CH=
=CH2
O O
Cyclohex-2-en-1-one

F
OH
5. Major product obtained in the given reaction is
CH3—Br +

or
ur O CH=
=CH2

f
Step 3 As HBr is in excess, so, reaction will also take place at
ks
NaBH4 in the reaction is used for the reduction by addition of a alkene.
hydride ion and a proton. Carbon-oxygen double bonds are
Yo
easily reduced by sodium borohydride. The actual reducing OH OH
oo
Markownikov’s rule, HBr
agent in these reductions is hydride ion (H- ). Hydride ion adds Addition reaction
eB

to the carbonyl carbon and the alkoxide ion that is formed is


subsequently protonated by water. In other words, the carbonyl CH=
=CH2 Br CH3
group is reduced by adding an H- followed by an H+ . The
mechanism of the given reaction is as follows : OH
r

O– ½
ou

O
ad

7. CH 3CH2 ¾ C ¾ CH 3 cannot be prepared by HCHO and


C – C—CH2Br ½
Y

CH2Br H from NaBH4 H2 O


Ph
H PhCH(CH3 )CH2MgX . This can be easily illustrated by
following reaction.
nd

OH
Re

C—CH2 — Br O
Fi

EtOH, 25°C
d– d+ d – d+
O Intramolecular H H C H + PhCH CH2 MgX Ph CHCH2 CH2 O MgX
cyclisation
CH3 CH3

H3O+
6. Key Idea Ethers are least reactive functional groups. The
cleavage of C ¾ O bond in ethers take place under drastic Ph CH (CH2)2 OH
conditions with excess of HX .
CH3
The major product obtained in the reaction is as follows :
The obtained product is not the required substance. While option
OCH3 OH (a), (b) and (c) can readily prepare the required substance.
Conc. HBr (excess)
Heat The reactions are as follows :

CH=
=CH2 Br—CH— CH3
Alcohols and Ethers 347

d – d+ Here, the haloform reaction will give following reaction:


O O MgX
2 4 6 s s
d– d+ 1 5 I2/OH I2/OH
Ph C CH2 CH3 + CH3MgX Ph C CH2CH3 3 [O]
OH O
CH3
CI3 s 2 4 OH + CHI3
OH 5
OH 1 3 (G)
H3O+ O O (Yellow ppt.)
Ph C CH2CH3
(i) Tollen’s reagent (AgNO3 + NH4OH) is a mild oxidising
CH3 agent. It does not react with ¾ C H ¾ CH3 group
d – d+ ½
O O MgX
OH
d– d+

w
Ph C CH3 + CH3CH2MgX Ph C CH3 (2°-alcohol).
(ii) Alkaline KMnO4 cannot perform the oxidative cleavage,
CH2CH3 rather it will hydroxylate the C == C.
OH

Flo
OH
½
H3O+
Ph C CH3 C H3 ¾ C H== C H ¾ C H2 ¾ C H ¾ C H3
6 5 4 3 2 1

ree
OH OH OH
CH2CH3
½ ½ ½
KMnO4
d – d+ ¾¾¾ ¾® C H3 ¾ C H ¾ C H ¾ C H2 ¾ C H ¾ C H3

F
O O MgX OH- 6 5 4 3 2 1

d – d+ (iv) CrO2Cl2 / CS 2 will not react here.


CH3CH2 C CH3 + PhMgX CH3CH2 C CH3

or
ur 10. In the given reaction, ester get cleaved in presence of dil. HCl
Ph and readily forms alcohol. This alcohol on reaction with oxalic
OH
f
acid undergoes polymerisation reaction.
ks
Hydrolysis of
H3O+ the ester group
Yo
CH3CH2 C CH3 H CH3
oo
O OH
Dil. HCl/D
Ph
Me – CH3COOH
eB

O
O OH
8. In the given reaction, AlCl 3 helps in the generation of
{

Ether group O O
electrophile that further undergoes ArSE 2 reaction to give the remains unaffected
by dil. HCl (hot)
required product. OH OH C C OH
r
ou

AlCl3
ad

HO HO Me
H—Cl O
–AlCl s O H OH C C OH
–H2O (COOH)2/Polymerisation
Y

O O
Intramolecular
ArSE2 reaction O
nd

H O
Re

O O
HO –HCl
–AlCl3
Fi

O n
AlCls4

H OCH3
It does not show SN reaction
with HCl because of double bond 11. Key idea The reaction given is a nucleophilic substitution
nature of the O—C bond, (due reaction in which cleavage at C¾O bond is visible. The
to +R-effect of the —OH group.) product formation can be visualised with the help of
following analysis.
9. The most suitable reagent to carry out given transformation is
I2/NaOH These two atoms are directly
attached to benzene ring.
OH Hence, development of
a O
½ any charge on these atoms is
stabilised by the ring itself due
C H3 C H== C H ¾ C H2 ¾ C H ¾ C H3 ¾Strong
¾ ¾ oxidation, [O]
¾¾¾¾ ¾® to conjugation (Resonance)
6 5 4 3 2 1 (Oxidative cleavage) O b
O If any one properly visualise the fact written with figure
½½ above, than a conclusion can be made that C¾O bonds
C H3 ¾ C H== C H ¾ C H2 ¾ C ¾ OH marked (a) and (b) in the figure will undergo heterolysis
6 5 4 3 2
during the reaction.
+CHI3 ¯ + CO2 ­
348 Alcohols and Ethers

The reaction can be represented as RCH2OH


14. H+
+

: :
O O + –H O OCH2R
O HI I Acetal
Resonance
Heat + I + CH3OH stabilisation
O
OH
Mechanism O
+
Step I The reaction begins with the attack of H + of HI on
oxygen to form oxonium ion as 15. All dihydroxy benzene will have higher boiling points than
+ monohydroxy benzene. Also, among dihydroxy benzenes, 1,
O O 2,-di-hydroxy benzene has lowest boiling point due to
HI
D H intramolecular H-bonding.
O O

w
O
Oxonium ion
Step II This oxonium ion undergoes lysis and addition of I- to H
form two products as O

Flo
+
O
I– I + CH OH H
H 3 (intramolecular H-bonding in

ree
O O 1,2-dihydroxy benzene)
.

Step III Similar pathway is followed at the other oxygen atom, OH

F
which can be visualised as conc. H3PO4
16.
I HI I I– I

or
+ CH3CH2I
O
D +
O
ur Concentrated H3PO4 solution does not involve any substitution
O product while with others, substitution products are also formed.
H H
f
17. C6 H5 MgBr + (CH3 )3 COH ¾® C6 H6 + Mg[(CH3 )3 CO]Br
ks
Note Mechanism of a reaction is always a logical sequencing of events
Yo
which may occur simultaneously as well. 18. CH3
oo
H+
12. PLAN This problem can be solved by using the concept of stability of +
H2O
eB

carbocation and SN 1 reaction. OH trans


H 3C
When two phenyl groups are replaced by two para methoxy H H
group, carbocation formed will be more stable. As the + C==C
stability of carbocation formed increases, rate of acidic
r

H3 C CH3
hydrolysis increases.
ou

cis
ad

OCH3 19. A is an alcohol and its oxidation product gives Tollen’s test, i.e.
B must be an aldehyde (CH3CH2CHO).
Y

CH3CH2CHO + H2NHN ¾ CO ¾ NH2 ¾®


OCH3 group indicated semicarbazide
nd

CH3CH2 ¾ CH== N ¾ NH ¾ CONH2


Re

by arrow increases
C C the stability of
carbocation Cu/ D Fehling
20. 1-propanol ¾¾® CH3 ¾ CH2 ¾ CHO ¾¾¾® Cu 2O ¯
Fi

electronically (by + M effect) solution red


O
Cu/ D ½½ Fehling
OCH3 2-propanol ¾¾® CH3 ¾ C ¾ CH3 ¾¾¾® No reaction
More stable carbocation solution
O OH O O
Hence, (c) is the correct choice. 21. H+ + – H+

13. The reaction of alcohol with Lucas reagent is mostly an SN 1 – H 2O


I conjugated
reaction and the rate of reaction is directly proportional to the
O O O
stability of carbocation formed in the reaction. Since 3° H+ – H+
R ¾ OH forms 3° carbocation (most stable), hence it will
– H 2O
react fastest. +
II
OH
Alcohols and Ethers 349

Although both reactions are giving the same product, Now if the given product is
carbocation I is more stable than II. Me N3
*
22. Thiol, (RSH), on combustion produces CO2 (g ), SO2 (g ) and
H2O(l ). At 298 K, H2O will be in liquid phase.
23. OH ¾® O- + H+ (has maximum electronegativity and which is too enantiomerically pure i.e. 100% either
difference) dextrorotatory or leavorotatory form, then the ‘X’ must be
Me OH
24. Ethanol is capable in forming intermolecular H-bonds : *
H O—H O—H
C2H5—O C2H5 C2H5 Note The configuration at carbon * atom in ‘X’ becomes inverted

w
due to SN 2 mechanism which is visible in the product as well.
25. Reaction proceeds via carbocation intermediates :
CH3 CH3 Thus, the probable reactions will be
½ - H2 O ½ Me OH Me Br
CH3 ¾ C — OH + H+ ¾¾® CH3 ¾ C + PBr3

Flo
½ ½ Et2O
CH3 CH3
(2-methylpropan -2-o1) (3 ° carbocation)

ree
Me Br Me I
CH3 NaI
- ½ Me2CO
+ NaBr
Br

F
¾¾® CH3 ¾ C — Br
½ Me I Me N3
CH3

or
ZnO-Cr 2 O3
ur NaN3
HCONMe2
26. CO + H2 ¾¾¾® CH3OH

f
heat
32. CH3
ks
27. CH3 ¾ CH2 ¾ O ¾ CH2 ¾ CH3 + HI ¾® 2C2H5I
CH3
Yo
CH CH3 O—O—CH
oo
28. 2-methyl propan-2-ol is a tertiary alcohol, will react fastest with CH3
Lucas reagent :
O2 H3 O +
eB

CH3 CH3
½ ½
HCl
CH3 ¾ C — OH ¾¾® CH3 ¾ C — Cl
½ ZnCl 2 ½ OH
r

CH3 CH3 O
ou
ad

SN1 reaction
+ CH3—C—CH3
29. On heating with concentrated H2SO4, ethanol would undergo CHCl3/NaOH
Y

Acetone
Phenol D , Reimer-Tiemann
dehydration to produce ethene. reaction
(P)
30. Ethanol is soluble in water due to its ability to form
nd
Re

intermolecular H-bonds with water : OH O O H O H


Fi

31. Key idea All the reactions involved in the problem are H CHO
Nucleophilic substitution of second order i.e., SN 2 which have , NaOH
the speciality of inversion of configuration at the carbon atom
involved. (Q) Major
(Q) Major
CHO
Of the reactions given (R) Minor
Reaction 1 in its generalised format is seen as
PBr
ROH ¾¾¾¾¾¾¾¾®
3
RBr O—Na Br CH2—Ph O—CH2Ph
In diethyl ether (Et 2O)
CHO
Reaction 2 is simple halogen exchange reaction called Williamson's CHO
Finkelstein reaction. Its generalised format is synthesis
In acetone( Me2 CO )
RX + NaI ¾¾¾¾¾¾® RI + NaX (S)

where X =Cl or Br (a) R is not steam volatile, but Q is steam volatile thus, incorrect.
Reaction 3 in its generalised format seen as (b) Q has enolic group thus, gives violet colour with 1% aqueous
RI + NaN3 ¾¾¾¾®
2 HCONMe
RN3 + NaI FeCl 3 solution thus, correct.
350 Alcohols and Ethers

(c) S has carbonyl group hence, gives yellow precipitate with 2, O


4-DNP thus, correct. ½½ H 2O
(d) S does not give colour with FeCl 3 thus, incorrect. 38. Ph ¾ C ¾ CH3 + Ph ¾ CH2 MgBr ¾®
33. OH
½
—O—CH2— + HI Ph ¾ C — CH2 ¾ Ph
½
CH3
—OH + —CH2I H

Phenol does not react further with HI. 39. Secondary : HO OH C–2(OH) is secondary
··
OH
34. CH3CH2Br + O == N ¾ O- ¾® CH3CH2NO2 + CH3CH2ONO

w
40. Peroxides : H2O2 + Fe2+ ¾® H2O + Fe3+
ambident nitroethane ethyl nitrite
nucleophile
41. Vicinal, same
OH
35. R—OH ¬ Hydrophilic

Flo
Cu ½ NaOH
Hydrophobic 42. C2H5OH ¾® CH3CHO ¾¾® CH3 ¾ CH ¾ CH2 ¾ CHO
D H 2O
aldol
Increasing molecular weight increases hydrocarbon (R)

ree
proportion that lowers the solubility in water. 43. Ethanol is weaker acid than water, not neutralised with NaOH.
Passage 1 44. 2°-alcohol on oxidation yields ketone while 1°alcohol on

F
oxidation produces aldehyde which can further be oxidised to
Compound J must be benzaldehyde because it on treatment acid.
with KOH undergoing Cannizzaro’s reaction producing

or
benzyl alcohol and pot-benzoate (L).
ur —OH
+
CH2 +

f
H+ Ring – H+
KOH 45.
ks
C6 H5 ¾ CHO ¾¾® C6 H5 ¾ CH2 OH + C6 H5 COOK( L) expansion
Yo
J benzyl alcohol X
O
oo
Also M is aldol condensation product formed from H
acetophenone O3
eB

[X ]
O Zn/CH3COOH
O
½½ KOH
Y
Ph ¾ C ¾ CH3 ¾¾® O
r

acetophenone ( K )
O NaOH
ou
ad

CH3 O CH3
½
—Ph aldol
½½ - H2 O
Y

Ph ¾ C — CH2 ¾ C ¾ Ph ¾¾®
½ O
OH Ph M +
dil. H2SO4/H
nd
Re

Ph 46. CH3CH2OH + CH3—C—CH3


ÞI = C== CH ¾ Ph O
H3C P CH3 Q R
Fi

I2/NaOH
OH Both gives positive iodoform test.
½
and H = Ph ¾ C — CH2 ¾ Ph Br OH
½ Mg (i) CH3CHO
CH3 47. —CH—CH3
ether (ii) H+/H2O
3 °-alcohol ‘B’
O
½½ - H+ + Ring expansion
+ H–shift
36. J = C6 H5 CHO, K = Ph ¾ C ¾ CH3 , L = PhCOOK + —CH—CH3
2° Br
Ph
37. I = C== CH ¾ Ph + Br–
H3 C

3° ‘C’
Alcohols and Ethers 351

+
H+ Lucas reagent
48. CH2 55. Compound ‘X’ ¾¾¾¾¾® No reaction at room
(i) OH temperature.
Cl–
Ammoniacal Excess of
C5H8 O ¾¾¾¾¾® ppt, X ¾¾¾¾® CH4;
AgNO3 CH3 MgBr
Cl
H2 /Pt
+ Cl– X ¾¾¾® n-pentane
CH2 CH2Cl HI excess

H+
Above information suggest that X has a terminal triple bond and
+ Cl–
it contain primary —OH group.
(ii)
OH 2° Cl Ag(NH3 )+2
Þ H ¾ C ºº C ¾ CH2 ¾ CH2 ¾ CH2OH ¾¾¾®

w
HI X
49. (CH3 )2 CH ¾ O ¾ CH3 ¾® CH3 ¾ CH ¾ CH3 + CH3I
½ Ag ¾ C ºº C ¾ CH2CH2CH2OH
I
56. Isobutane < n-butane < n-butylchloride < n-butanol

Flo
50. Acid catalysed dehydration proceeds via carbocation Conc.H SO
intermediate. Also, greater the stability of reactive intermediate, 57. CH3CH2OH ¾¾¾¾¾®
2 4
CH2 == CH2
D
faster the reaction :

ree
Br 2 NaNH2
CH3 CH3 ¾® CH2 ¾ CH2 ¾¾¾® H ¾ C ºº C ¾ H
½ H+
½ ½ ½
CH3 ¾ C — OH ¾¾® CH3 ¾ C — CH3

F
Br Br
½ - H2 O
+
CH3 (3 ° highly stable)
Hg(CH3COO) 2
¾¾¾¾¾® CH2 ==CH ¾ OCOCH3

or
(t - butanol)
ur CH3COOH vinyl acetate
n-butanol forms less stable (1°) carbocation.

f
2 I /NaOH
H 58. C2H5OH ¾¾¾® CHI3 + HCOONa
ks
iodoform
Yo
O O+ 59. CH3 ¾ CH2 OH + I2 + NaOH ¾® CHI3 ¯ (Iodoform test)
H+ – H+
oo
51. H3C
H3C yellow
eB

H3C H 3C OH2
Iodoform test is not given by methanol.
2,2-dimethyloxirane
CH3 OH 60. Butanol forms intermolecular H-bonds, has higher boiling point
than butanal.
r

H3C—C——CH2 O
ou
ad

½½ D
OH 61. (CH3COO)2 Ca ¾® CH3 ¾ C ¾ CH3 + CaCO3
Y

52. E = C6H5COOH (benzoic acid) D


O
O ½½
nd

½½ H2 SO4
Re

I2 CH3 ¾ C ºº CH ¾¾® CH3 ¾ C ¾ CH3


Þ D = C6H5 ¾ C ¾ CH3 ¾¾® C6H5 ¾ COONa + CHI3 C HgSO4 D
NaOH
Fi

CH3 OH CH3
½ ½ H+
½ + OH
53. CH3 ¾ C — CH ¾ CH3 ¾¾® CH3 ¾ C — CH ¾ CH3 ½ Br 2
Conc.
½ ½ CH3 ¾ CH ¾ CH3 ¾¾® CH3 ¾ CH== CH2 ¾®
CH3 CH3 H2 SO4 B H2 O

Br
CH3 ½
Methyl ½ – H+ HC CH3 CH3 ¾ CH ¾ CH2
¾¾® CH3 ¾ C — CH — CH3 ¾® 3 C == C ½
shift ½ H3C CH3 Br
+
CH3 X
NaNH2
KMnO4 CHO + MnO2¯ X ¾¾® CH3 ¾ C ºº CH
H 2O C
54. OH brown
n-butanol
352 Alcohols and Ethers

62. (i) H ¾ C ºº C ¾ H ¾¾¾®


HgSO Catalyst 64. A is ethanol because B is an alkene (ethene).
4
CH3 ¾ CHO ¾¾¾®
H2 SO4 H2 H /Ni
C2H5OH + Conc.H2SO4 ¾® CH2 == CH2 ¾¾¾®
2

CH3CH2OH
CH3 ¾ CH3
Heat
(ii) 4 C2H5Br + 4 (Na / Pb) ¾® (C2H5 )4 Pb CH2 == CH2 + Br2 ¾ H2O ¾® CH2 ¾ CH2
sodium -lead tetraethyl lead (brown) ½ ½
alloy Br Br
colourless
H O
(iii) Al 4C3 ¾¾®
2
Al(OH)3 + CH4 CH2 == CH2 + alk. KMnO4 ¾® CH2 ¾ CH2
hn purple ½ ½
CH4 + Cl 2 ¾® CH3Cl + HCl OH OH
colourless
63. Compound X must be a symmetrical ether :

w
65.
HI Na H
+ Methyl shift
R ¾ O¾ R ¾® 2R ¾ I ¾® No reaction, not an alcohol H
X Y Heat
HO

Flo
Schiffs’ H3 C CH3 H 3C CH3
¾¾¾® No reaction, not an aldehyde.
reagent

ree
H2 O [O] –H
R ¾ I ¾® R ¾ OH (Z ) ¾® Acid Heat
EW = 60 CH3
½ I2 / P H 3C

F
¯ CH3 CH3
R—I ‘P’
OH

or
Þ Acid is CH3COOH and Z = CH3CH2OH
ur KMnO4(dil)
HO
Þ X = C2H5 ¾ O ¾ C2H5, Y = CH3 ¾ CH2 ¾ I 0°C

f
OH
HI
H3C OH CH3
CH3CH2 ¾ O ¾ CH2CH3 ¾® 2CH3CH2I
ks
‘Q’
Yo
X Y
oo
eB

Download Chapter Test


http://tinyurl.com/yy5jdmfu or
r
ou
ad
Y
nd
Re
Fi
25
Aldehydes and Ketones

w
4. The major product of the following reaction is
Objective Questions I (Only one correct option) (2019 Main, 8 April II)

Flo
1. The major product(s) obtained in the following reaction is/are O (1) t-BuOK
(2019 Main, 12 April I) (2) Conc. H2 SO4 /D
Cl

ree
(i) KOtBu
(ii) O3/Me2S
O O
(b)

F
Br (a)
CHO and OHC ¾ CHO
(a) OHC

or
ur
(b) OHC O O

f
CHO (d)
(c)
ks
(c) OHC CHO
Yo
OtBu
5. An organic compound neither reacts with neutral ferric
oo
(d) OHC chloride solution nor with Fehling solution. It however,
eB

CHO reacts with Grignard reagent and gives positive iodoform


2. In the following reaction, test. The compound is (2019 Main, 8 April I)
HCl
Carbonyl compound + MeOH - acetal O
r

Rate of the reaction is the highest for: (2019 Main, 9 April II) CH3 O
ou

CH3
ad

(a) Acetone as substrate and methanol in excess (a) (b)


H
(b) Propanal as substrate and methanol in stoichiometric amount OH
Y

(c) Acetone as substrate and methanol in stoichiometric amount O


(d) Propanal as substrate and methanol in excess O OH
nd
Re

3. The major product obtained in the following reaction is C2H5 CH3


(2019 Main, 8 April II) (c) (d)
Fi

CH3 O CH3 C2H5


O
NaOH O
OHC D

CH3
6. In the following reactions, products A and B are
H3 C (2019 Main, 12 Jan I)
(a) H (b) H O O
Dil. NaOH
O CH3 O CH2 H 3C H [A]
H3 C CH3
CH3 CH3 H3O+
[A] D [B]

O O
(c) (d) CH3 CH3
(a) A= CH3 ; B= CH3
O O HO
354 Aldehydes and Ketones

O O O
CH3 CH3 OH OH
CH2 C OCH3
(b) A= CH3 ; B= CH3 CH2 CH2 OH
HO (a) (b)
O
O
OH CH3
O OH
C O
H H CH2 C H CH2 CH2 OH
HC
(c) A= 3 ; B= (c) (d)
H 3C H3C
CH3 CH3
O 10. The correct sequence of reagents for the following
O
OH
C H2 C conversion will be (2017 Main)
H H O HO CH3
A= H3C ; B=

w
(d)
H 3C H3 C
CH3 CH3

7. The major product obtained in the following reaction is H3C OH

Flo
CHO
(2019 Main, 11 Jan II) CH3
OH
(a) [Ag(NH3 )2 ]+ OH- , H+ / CH3 OH, CH3 MgBr

ree
LiAlH4
(excess)
(b) CH3 MgBr, H+ / CH3 OH, [Ag(NH3 )2 ]+ OH-
CH3
(c) CH3 MgBr, [Ag(NH3 )2 ]+ OH- , H+ / CH3 OH

F
NO2
(d) [Ag(NH3 )2 ]+ OH- , CH3 MgBr, H+ / CH3 OH
OH OH

or
ur 11. The major product of the following reaction sequence is
(2016 Adv.)

f
(a) (b) O
CH3 CH3
ks
(i) HCHO (Excess)/NaOH, Heat
NO2 OH
Yo
NH2 OH
(ii) HCHO/H+ (Catalytic amount)
oo
OH OH
eB

O O O O OH
(c) (d)
CH3 CH3
(a) (b)
r

NO2 OH NH2 OH
ou
ad

8. The correct match between item ‘I’ and item ‘II’ is


O OH
O O
Y

Item ‘I’ Item ‘II’ HO


(Compound) (Reagent)
(c) (d)
nd
Re

(A) Lysine (P) 1-naphthol


(B) Furfural (Q) Ninhydrin
Fi

(C) Benzyl alcohol (R) KMnO4 OH


(D) Styrene (S) Ceric ammonium nitrate 12. Which compound would give 5-keto-2-methyl hexanal upon
(2019 Main, 10 Jan II) ozonolysis? (2015 Main)

Codes CH3 CH3


A B C D ½ ½
CH3
(a) Q R S P
½

(b) R P Q S (a) (b)


(c) Q P S R
½

(d) Q P R S CH3
CH3 CH3
9. The major product ‘X’ formed in the following reaction is ½
(2019 Main, 10 Jan I)
½
CH3
½

O (c) (d)
O
CH2 C OCH3 NaBH4
X
½

MeOH CH3
Aldehydes and Ketones 355

13. The major product in the following reaction is (2014 Adv.) 17. Cyclohexene on ozonolysis followed by reaction with zinc
O dust and water gives compound E. Compound E on further
(i) CH3MgBr, dry ether, 0°C treatment with aqueous KOH yields compound F.
Cl
(ii) Aqueous acid Compound F is (2007, 3M)
CH3
O —CHO —CHO
(a) (b)
(a) H3C
CH3
OH —COOH
(c) (d)
COOH
(b)
H2C CH3
CH3
18. The smallest ketone and its next homologue are reacted with

w
NH2OH to form oxime (2006)
(c) (a) two different oximes are formed
CH2 (b) three different oximes are formed
O
(c) two oximes are optically active

Flo
CH3
(d) all oximes are optically active
(d)
CH3 19. Butan-2-one can be converted to propanoic acid by which of

ree
O
the following ? (2006)
14. The most suitable reagent for the conversion of (a) NaOH, NaI/H+
R ¾ CH2 ¾ OH ® R ¾ CHO is

F
(2014 Main)
(b) Fehling’s solution
(a) KMnO4 (c) NaOH, I2/H+
(b) K 2 Cr2 O7

or
(c) CrO3
ur (d) Tollen’s reagent

f
(d) PCC (pyridinium chlorochromate) CHO
15. The major product H in the given reaction sequence is CH3COONa
ks
20. + (X )
95% H2 SO4
Yo
CN s
CH3 ¾ CH2 ¾ CO ¾ CH3 ¾¾® G ¾¾¾¾® H MeO
oo
COOH
(a) CH3 ¾ CH == C ¾ COOH
eB

½
CH3 (2012)
MeO
(b) CH3 ¾ CH == C ¾ CN
r

What is X ?
ou

½
ad

CH3 (a) CH3 COOH (b) BrCH2 COOH


(c) (CH3 CO)2 O (d) HOC ¾ COOH
Y

OH
21. The order of reactivity of phenyl magnesium bromide with
½
(c) CH3 ¾ CH2 ¾ C ¾ COOH the following compounds is (2004, 1M)
nd
Re

½ O O O
CH3 ½½ ½½ ½½
Fi

(d) CH3 ¾ CH == C ¾ CO ¾ NH2 H3 C CH3 H3 C H Ph Ph


½ (I) (II) (III)
CH3
(a) (II) > (III) > (I)
16. The number of aldol reaction(s) that occurs in the given (b) (I) > (III) > (II)
transformation is (2012) (c) (II) > (I) > (III)
(d) All of the above
OH
OH 22. A mixture of benzaldehyde and formaldehyde on heating
Conc. aq. NaOH
CH3CHO + 4HCHO with aqueous NaOH solution gives (2001, 1M)
HO (a) benzyl alcohol and sodium formate
OH
(b) sodium benzoate and methyl alcohol
(a) 1 (b) 2 (c) sodium benzoate and sodium formate
(c) 3 (d) 4 (d) benzyl alcohol and methyl alcohol
356 Aldehydes and Ketones

23. The appropriate reagent for the following transformation: Objective Questions II
O (One or more than one correct option)
CH2CH3 33. Reagent(s) which can be used to bring about the following
CH3
transformation is (are) (2016 Adv.)

HO HO
O O O O
(a) Zn (Hg), HCl (b) NH2 NH2 , OH– C O C
(c) H2 /Ni (d) NaBH4
O H O OH
24. Which of the following has the most acidic hydrogen?
(2000, 1M) COOH COOH

w
(a) 3-hexanone (b) 2, 4-hexanedione (a) LiAlH 4 in ( C2 H5 )2 O (b) BH 3 in THF
(c) 2, 5-hexanedione (d) 2, 3-hexanedione (c) NaBH 4 in C2 H5 OH (d) Raney Ni/H 2 in THF
25. The enol form of acetone, after treatment with D2 O, gives 34. The major product of the following reaction is (2015 Adv.)

Flo
(1999, 2M)
O
OD O (i) KOH, H2O
½ ½½

ree
(ii) H+, Heat
(a) H3 C ¾ C ==CH2 (b) D3 C ¾ C ¾ CD3 CH3
O
OH OD

F
½ CH3
½ CH3
O O
(c) H2 C==C ¾ CH2 D (d) D2 C==C ¾ CD3
(a) (b)

or
ur
26. Which of the following will react with water? (1998, 2M)
(a) CHCl 3 (b) Cl 3 CCHO O

f
(c) CCl 4 (d) ClCH2 CH2 Cl O CH3
ks
27. Which of the following compounds is oxidised to prepare (c) CH3 (d)
Yo
methyl ethyl ketone? (1987, 1M)
oo
(a) 2-propanol (b) 1-butanol
(c) 2-butanol (d) t-butyl alcohol 35. After completion of the reactions (I and II), the organic
eB

compound (s) in the reaction mixtures is/are (2013)


28. The compound that will not give iodoform on treatment with
alkali and iodine is (1985, 1M) O
Br2 (1.0 mol)
(a) acetone (b) ethanol
r

(c) diethyl ketone (d) isopropyl alcohol Reaction I H3C CH3 Aqueous/NaOH
ou
ad

29. The Cannizzaro’s reaction is not given by (1983, 1M) O


Br2 (1.0 mol)
Y

(a) trimethyl acetaldehyde H3C CH3 CH3COOH


Reaction II
(b) acetaldehyde (1.0 mol)
(c) benzaldehyde
nd
Re

(d) formaldehyde O O O
Fi

30. When acetaldehyde is treated with Fehling’s solution, it


H3C CH2Br H3C CBr2 Br2C CBr2
gives a precipitate of (1983, 1M)
(a) Cu (b) CuO P Q R
(c) Cu 2 O (d) Cu + Cu 2 O + CuO O O
31. A compound that gives a positive iodoform test is (1982, 1M) CHBr3
BrH2C CH2Br H3 C ONa
(a) 1-pentanol (b) 3-pentanone
S T U
(c) 2- pentanone (d) pentanal
(a) reaction I : P and reaction II : P
32. The reagent with which both acetaldehyde and acetone react
(b) reaction I : U, acetone and reaction II : Q, acetone
easily is (1981, 1M)
(c) reaction I : T , U , acetone and reaction II : P
(a) Tollen’s reagent (b) Schiff ’s reagent (d) reaction I : R, acetone and reaction II : S , acetone
(c) Grignard’s reagent (d) Fehling’s reagent
Aldehydes and Ketones 357

36. Tautomerism is exhibited by (1998) Passage Based Questions


Passage 1
(a) —CH==CH—OH (b) O== ==O
Two aliphatic aldehydes P and Q react in the presence of aqueous
K 2 CO3 to give compound R, which upon treatment with HCN
provides compound S . On acidification and heating, S gives the
(c) ==O (d) ==O product shown below : (2010)
==

==
O O H 3C OH
H 3C
37. A new carbon–carbon bond formation is possible in
(1998, 2M) O
(a) Cannizzaro’s reaction O

w
(b) Friedel-Crafts’ reaction
(c) Clemmensen’s reduction
44. The compounds P and Q respectively are
(d) Reimer-Tiemann reaction CH3
H3C H

Flo
38. Which of the following will undergo aldol condensation? CH H C
(a) Acetaldehyde (1998, 2M)
(a) H3C C and
(b) Propanaldehyde O

ree
O
(c) Benzaldehyde CH3
(d) Trideutero acetaldehyde H
H

F
39. Among the following compounds, which will react with CH H C
(b) H3C C and
acetone to give a product containing C==N¾ ? O

or
(a) C6 H5 NH2
ur
(b) (CH3 )3 N (1998, 2M)
O
H
H3C CH2 H H3C

f
(c) C6 H5 NHC6 H5 (d) C6 H5 NHNH2 CH C C
(c) and
ks
40. Which of the following is an example of aldol condensation? CH3 O O
Yo
Dil.NaOH
(a) 2CH3 CHO ¾¾¾® CH3 CH(OH)CH2 CHO (1989, 1M)
oo
H3C CH2 H H H
OH CH C C
eB

½ (d) and
Dil.NaOH CH3 O
(b) 2CH3 COCH3 ¾¾¾® H3 C ¾ C ¾ CH2 COCH3 O
½ 45. The compound R is
CH3
r

O O
ou

Dil .NaOH
ad

(c) 2HCHO ¾¾¾® CH3 OH + HCOONa


H 3C C H 3C C
(a) C (b) C
Y

Dil .NaOH H H
(d) C6 H5 CHO + HCHO ¾¾¾® C6 H5 CH2 OH +HCOONa H3C H 3C
H 2C CH
41. Which of the following compounds will react with ethanolic OH H3C OH
nd
Re

KCN? (1984, 1M)


(a) Ethyl chloride (b) Acetyl chloride CH3 O CH3 O
Fi

(c) Chlorobenzene (d) Benzaldehyde CH CH


C C
42. Which of the following compounds will give a yellow (c) H3C CH H (d) H3C CH H
precipitate with iodine and alkali ? (1984, 1M)
H2C CH
(a) 2-hydroxy propane OH H3C OH
(b) Acetophenone
(c) Methyl acetate 46. The compound S is
(d) Acetamide CH3 O
O
43. Base catalysed aldol condensation occurs with (1984, 1M)
CH C
(a) propionaldehyde H3C H H3C C
(a) CH (b) C H
(b) benzaldehyde H3C
(c) 2-methyl propionaldehyde H 2C H2C
CN CN
(d) 2, 2-dimethyl propionaldehyde
358 Aldehydes and Ketones

CH3 CN CN 49. The structure of the product S, is


CH CH H 3C CH O
(c) H3C CH OH (d) C OH
H 3C
H 2C H 2C (a) (b)
OH OH
O
Me Me
Passage 2 Me
O O
A carbonyl compound P, which gives positive iodoform test,
undergoes reaction with MeMgBr followed by dehydration to give Me
an olefin Q. Ozonolysis of Q leads to a dicarbonyl compound R, (c) (d)
which undergoes intramolecular aldol reaction to give

w
predominantly S. (2009) Me Me

1. MeMgBr O3 /Zn -H2O OH– Passage 3


P ¾¾¾¾¾® Q ¾¾¾¾¾® R ¾¾¾® S
In the following sequence, product I , J and L are formed. K

Flo
2. H+ , H 2O heat
3. H2SO4 / heat
represents a reagent.
(i) Mg/ether

ree
(i) NaBH4 (ii) CO2 K
47. The structure of the carbonyl compound P, is Hex-3-ynal I J
(ii) PBr3 (iii) H3O+
CH3 H2

F
(a) (b) H3C Cl L
Pd/BaSO4 quinoline
O CH3 O CH3
O

or
(2008, 3 ´ 4M = 12M)
ur 50. The structure of the product I is
O

f
(a) H3C Br
ks
(c) (d)
Yo
(b) H3C
oo
O C2H5 CH3
Br
48. The structures of the products Q and R, respectively, are
eB

(c) H3C Br
O
(d) H3C Br
H
r

(a) CH3
COCH3
ou

51. The structures of compounds J and K , respectively, are


ad

CH3
H3C H3C CH3 (a) H3C COOH and SOCl2
Y

O
(b) H3C COOH and SO2Cl2
nd
Re

H
(b) (c) H3C and SOCl2
CHO
Fi

CH3 COOH
H 3C H3C CH3
(d) H3C COOH
and CH3SO2Cl
O

H
52. The structure of product L is
(c) (a) H3C CHO
CHO
Me Et Et
Me
O (b) H3C CHO
Me
CH3 (c) H3C
(d)
CHO CHO
Me Me Et
(d) H3C CHO
Aldehydes and Ketones 359

Match the Column 59. In the scheme given below, the total number of
intramolecular aldol condensation products formed fromY is
53. Match each of the compounds given in Column I with the
reaction(s) that they can undergo, given in Column II. 1. O3 1. NaOH(aq)
Y
2. Zn, H2O 2. heat
Column I Column II (2010)
A. Br p. Nucleophilic
substitution Subjective Questions
HCl
O 60. (A), C6H12 ¾¾® (B), C6H13Cl + (C ), C6H13Cl
Alcoholic KOH
B. q. Elimination (B) ¾¾¾¾¾® (D), (an isomer of (A))
OH

w
O zonolysis
(D) ¾¾¾¾® (E), (positive iodoform and negative
Fehling’s solution test)
C. CHO r. Nucleophilic addition
Ozonolysis
(A) ¾¾¾¾® (F) + (G), (positive Tollen's test for both)

Flo
Conc. NaOH
OH (F) + (G) ¾¾¾¾® HCOONa + A primary alcohol

ree
D. s. Esterification with Identify the compounds (A) to (D). (2003)
Br
acetic anhydride
61. A compound C9H7O2Cl exists predominantly in enol form

F
NO2 (A) and also in keto form (B). On oxidation with KMnO4 it
gives m-chlorobenzoic acid as one of the products. Identify
t. Dehydrogenation the compounds (A) and (B).

or
(2003)
ur
54. Match the compounds/ions in Column I with their 62. An alkene (A) C16 H16 on ozonolysis gives only one product

f
properties/reactions in Column II. (2007, 6M) (B) C8 H8 O. Compound (B) on reaction with NaOH / I2 yields
ks
sodium benzoate. Compound (B) reacts with KOH/NH2 NH2
Yo
Column I Column II yielding a hydrocarbon (C ) C8 H10 . Write the structures of
oo
A. C6 H5 CHO p. gives precipitate with compounds (B) and (C ). Based on this information two
2, 4-dinitrophenylhydrazine isomeric structures can be proposed for alkene (A). Write
eB

their structures and identify the isomer which on catalytic


B. CH3 C ºº CH q. gives precipitate with AgNO3 hydrogenation (H2 / Pd – C) gives a racemic mixture.

(2001, 5M)
C. CN r. is a nucleophile
r

63. Identify A, B and C, and give their structures.


ou
ad

D. I- s. is involved in cyanohydrin
O
formation CH3
Y

CH3 Br2
Fill in the Blanks A +B
nd

NaOH
Re

55. Fehling’s solution A consists of an aqueous solution of O H+


C(C7H12O)
copper sulphate, while Fehling’s solution B consists of an heat (2000)
Fi

alkaline solution …… (1990, 1M) 64. An organic compound A, C6 H10 O, on reaction with CH3 MgBr
followed by acid treatment gives compound B. The compound
True or False B on ozonolysis gives compound C, which in presence of a
56. The reaction of methyl magnesium iodide with acetone base gives 1-acetyl cyclopentane D. The compound B on
followed by hydrolysis gives secondary butanol. (1987, 1M) reaction with HBr gives compound E. Write the structures of
A, B, C and E. Show, how D is formed from C. (2000, 5M)
57. Benzaldehyde undergoes aldol condensation in an alkaline
medium. (1982 , 1M) 65. What would be the major product in the following reaction ?
Ph
Integer Answer Type Questions base

58. Consider all possible isomeric ketones including O


Br
stereoisomers of MW = 100. All these isomers are (2000, 1M)
independently reacted with NaBH4 . The total number of
66. (a) Compound A (C8 H8 O) on treatment with NH2OH. HCl
ketones that gives a racemic product(s) is/are
given B and C. B and C rearrange to give D and E,
NOTE (Stereoisomers are also reacted separately). (2014 Adv.)
respectively, on treatment with acid. B, C, D and E are all
360 Aldehydes and Ketones

isomers of molecular formula (C8 H9 NO). When D is 73 Complete the following reaction with appropriate structure :
boiled with alcoholic KOH, an oil F (C6H7 N) separates CH3 CH2
out. F reacts rapidly with CH3COCl to give back D. On the (i) KCN/H2SO4
C==O ¾¾¾¾¾®
other hand, E on boiling with alkali followed by (ii) LiAlH4
acidification gives a white solid G (C7 H6O2). Identify H (1996, 1M)
A-G.
(b) Carry out the following transformation in not more than 74. Complete the following reaction with appropriate structure.
three steps. NaOC 2 H5 in absolute
1-butyne ¾® 2-pentanone (1999, 3M) C6H5CHO + CH3COOC2H5 ¾¾¾¾¾¾¾¾® A
C2 H5 OH heat (1995, 1M)
67. Write the intermediate steps for each of the following
reactions 75. Write the structure of the major organic product expected
from the following reaction. (1992, 1M)
H3O+

w
(i) C6H5CH(OH)C ºº CH ¾¾® C6H5CH == CHCHO
KOH
MeO CHO + HCHO

H
(ii)

Flo
76. Arrange the following in the increasing order of expected
OH O CH3 enol content.
68. Complete the following reactions with appropriate structures CH3 COCH2 CHO, CH3 COCH3 , CH3 CHO,

ree
of products/reagents. CH3 COCH2 COCH3 (1992, 1M)

O 77. Give reason in one or two sentences :

F
½½ “Iodoform is obtained by the reaction of acetone with
O CHC6H6
[C] (i) LiAIH4 hypoiodite but not with iodide”. (1991)

or
ur (ii) H +, heat
(1998)
78. A ketone A, which undergoes haloform reaction, gives
compound B on reduction. B on heating with sulphuric acid

f
69. An aldehyde A (C 11 H8 O), which does not undergo self aldol gives compound C, which forms mono-ozonide D. D on
ks
condensation, gives benzaldehyde and two moles of B on hydrolysis in the presence of zinc dust gives only
Yo
ozonolysis. Compound B, on oxidation with silver ion gives acetaldehyde. Identify A, B and C. Write down the reactions
oo
oxalic acid. Identify the compounds A and B. (1998, 2M) involved. (1989, 4M)

70. Acetophenone on reaction with hydroxylamine hydrochloride 79. Answer the followings with suitable equations wherever
eB

can produce two isomeric oximes. Write structures of the necessary


oxime. (1997, 2M) (i) suggest a reagent to distinguish acetaldehyde from acetone.
(ii) what happens when excess chlorine is passed through
r

71. Complete the following, giving the structures of the principal


boiling toluene in the presence of sunlight?
ou

organic products, (1987, 2M)


ad

O
80. Complete the following with appropriate structures
Y

NaOH
? CH CH—CHO
(i) + Ph3P CH2 A
nd
Re

(1986, 1M)

81. How may the following transformation be carried out (in not
(ii) ClCH2CH2CH2COPh + KOH + MeOH ¾® B
Fi

more than six steps) “benzaldehyde to cyanobenzene”?


O (1986, 2M)
C6H5 82. Give reason in one or two sentences for the following:
H3O+
(iii) H3C + NaOH C “Hydrazones of aldehydes and ketones are not prepared in
O (1997, 2M) highly acidic medium”. (1986, 1M)
72. Suggest appropriate structures for the missing compounds.
83. Write down product of the following reaction
(the number of carbon atoms remains the same throughout
the reaction) NaOH
Propanal ¾¾¾®
heat (1985, 1M)
CH3
84. Arrange the following in order of their increasing reactivity
Dil. KMnO4 HIO4 HO– towards HCN :
A B C
CH3CHO, CH3COCH3 , HCHO, C2H5COCH3 (1985, 1M)

CH3 (1996, 3M)


Aldehydes and Ketones 361

85. Write down the reactions involved in the preparation of the treated with bromine in presence of phosphorus yields a
following using the reagents indicated against in parenthesis: compound C which on hydrolysis gives a hydroxyl acid D.
“Acetoxime from acetaldehyde.” This acid can also be obtained from acetone by the reaction
with hydrogen cyanide followed by hydrolysis. Identify the
[K2Cr2O7 / H+ , Ca(OH)2 and NH2OH, HCl] (1984, 2M) compounds A, B, C and D. (1982 , 2M)

86. Show with balanced equation, what happens, when the 88. Outline the reaction sequence for the conversion of methanal
following are mixed : to ethanol (the number of steps should not be more than
“Chloral is heated with aqueous hydroxide” (1984, 2M)
three). (1981, 2M)

87. An alkene A on ozonolysis yields acetone and an aldehyde. 89. Write the structural formula of the main organic product
The aldehyde is easily oxidised to an acid B. When B is formed when methanal reacts with ammonia. (1981, 1/2M)

w
Answers
1. (a) 2. (d) 3. (c) 4. (d) 37. (b,d) 38. (a,b,d) 39. (a,d) 40. (a,b)

Flo
5. (d) 6. (b) 7. (b) 8. (c) 41. (a,b,d) 42. (a,b) 43. (a,c) 44. (b)
9. (b) 10. (a) 11. (a) 12. (b) 45. (a) 46. (d) 47. (b) 48. (a)

ree
13. (d) 14. (d) 15. (a) 16. (c) 49. (b) 50. (d) 51. (a) 52. (c)
17. (a) 18. (b) 19. (c) 20. (c) 53. A ® p, q, t B ® p, s, t C ® r, s D ® p.
A ® p, q, s B ® q, r C ® q, r, s D ® q, r.

F
21. (c) 22. (a) 23. (b) 24. (b) 54.
25. (b) 26. (b) 27. (c) 28. (c) 55. Sodium potassium tartarate
(b) (c) (c) (c) False 57. False

or
29. 30. 31. 32. 56.
33. (c) 34. (a) 35.
ur (c) 36. (a,c,d) 58. (5) 59. (1)

f
ks
Hints & Solutions
Yo
oo

3 Step II moves in the forward direction that results in the formation of


eB

1. CHO
Step I O3/Me2S
2 OHC and acetal. Reaction involved is as follows :
KOtBu Ozonolysis
OHC ¾CHO
1 Et Et OH Et OMe
H+/MeOH MeOH
Br Cyclohex-1, C O C C
r

3-diene Excess
H H OMe H OMe
ou
ad

In step-1 dehydrohalogenation reaction takes place. Here, Acetal


hydrogen is eliminated from b-carbon and the halogen is lost
3. It is an intramolecular aldol condensation reaction.
Y

from a-carbon atom. As a result diene is formed.


This attack will result
H 7-membering. So, it will not
nd


Re

OtBu proceed.
Br Cyclohex-1,3-diene O O
s O
Fi

6 4
OH 1
Cyclohex -1, 3-diene on ozonolysis gives butane-1, 4- dial and OHC CH2 –H2O HC 5 3
2 CH2
s
ethane- 1, 2- dial. H H 7
H
CHO Acidic H-atom
O3 CHO | O
| + (CH2)2 OH– 4 2
Me2S 5 1
CHO | – H 2O O=
=CH 3 s
CHO

Less acidic
2. Key Idea Aldehydes are more reactive than ketones in H s
nucleophilic addition reactions. O
D HO H 2O 4
5 3
For the reaction, –H2O – OH
s 1 2
HCl (option-c)
Carbonyl compound + MeOH Acetal - O Major
O H More acidic O
Rate of reaction is the highest for propanal as substrate and b-H for b-elimination
methanol in excess. Propanal is an aldehyde and more reactive
than ketones. When MeOH is taken in excess then reaction
362 Aldehydes and Ketones

4. –I
O
6. The reactant in presence of dil × NaOH undergoes intramolecular
O
H b-elimination aldol condensation reaction.
Cl s r /D
tBuOK As a result of this, b-hydroxyketone (A) is obtained which on
Acidic hydrogen H
–HCl
hydrolysis followed by heating produces a , b-unsaturated
This benzylic –H activates ketone (B)
r
p-position by +ve H (Conc. H2SO4) O O O O
for ArSO2 Electrophilic addition
dil.
H H H3C H NaOH s H
O O H3 C CH3


r
r 1° O O O
2°, but less stable carbocation
1°, but more stable
because of –I effect of O

w
carbocation D H 2O
s
ArSE 2 ArSE 2 –H2O HO O
H H (B) H (A) H
r O r O

Flo
7. LiAlH4 acts as a nucleophilic reducing agent that can reduce
H H ¾ COOH to ¾ CH2OH, ¾ C == O into ¾ CH ¾ OH and
½ ½

ree
6- membered ring ¾ NO2 into ¾ NH2, but it cannot reduce isolated C==C
D –Hr (more stable) D –Hr
H

F
O O O OH isolated C==C OH

LiAlH4 (excess)

or
Major product
(option-d)
5- membered ring
ur Minor product CH3

f
(less stable) (option-c)
NO2 NH2 OH
ks
5. According to the given conditions, compound (d) neither reacts Compound
(b)
Yo
with neutral ferric chloride solution nor with Fehling solution. It
oo
however reacts with Grignard reagent and gives positive 8. (A) ® Q; B ® (P) ; C ® (S), D ® (R)
iodoform test.
(A) Lysine (R=–(CH2)4–NH2)
eB

● As the compound does not contain any phenolic ¾ OH

group. Hence, it gives negative neutral FeCl 3 test. NH2


Ninhydrin test (B)
R CH Violet
OH COOH O
| colouration
OH
r

CH3 Neutral (Ninhydrin),


ou

no reaction
ad

FeCl3 OH boil
(FeCl3 test for phenolic
C—Et O
group)
Y

O OH
● Compound gives reaction with RMgX as it contains
(B)
¾ C ¾ Et. CHO +
nd
Re

½½ O
(Furfural)
O CH2SO4
OH OH Violet colouration
Fi

D
| |
Molisch Test (P)
CH3 RMgX CH3 (Molisch reagent)
Grignard R
reagent C2H5
C—Et CH2OH Ceric ammonium CHO
OH nitrate (s) [CAN]
O (C)
50% CH3COOH
Compound with CH3CH ¾ group undergoes iodoform test in (Benzyl alcohol) (Benzaldehydel)
½
OH OH
OH
presence of NaOH and I2.

dil. KMnO4/OH /Cold (R)
OH O (D) H
| (Pink) H
H
CH3 NaOH+I2
O–Na+ (Styrene)
CH3I ¯ + +Mn2+(Colourless)
Iodoform
test Yellow Et
C—Et ppts.
O
O
Aldehydes and Ketones 363

9. NaBH4 is a selective reducing agent. It reduces carbonyl CH3


12. (a) ½
C O CH3
group into an alcohol but cannot reduce an isolated

½
O3

½
C == C and an ester group too. D
Zn.H2O2
Carbonyl
group O O O O
O OH
½½ ½½
CH2 C OCH3 CH2 C OCH3 CH3 ¾ C ¾ CH2 ¾ CH2 ¾ CH2 ¾ C ¾ CH3
NaBH4
Heptan -2, 6 dione
Ester MeOH
group
CH3
Isolated ½
C C

½
(b) O3

w
10. D
Zn.H2O2

½
O O O HO CH3 CH3
O

Flo
[Ag (NH3)2]OH Esterification CH3MgBr
CH3OH, H+ 6 ½½5 4 3 2 1
Tollen’s reagent (Excess)
CH3 ¾ C ¾ CH2 ¾ CH2 ¾ CH ¾ CHO
½

ree
CHO COOH COOCH3 OH
CH3
5-keto-2-methyl hexanal
Before final product is formed, intermediate is
CH3

F
HO CH3
½

½
O3

or
ur (c) D
Zn.H2O2

f
½
O CH3 CH3
ks
O CH3
11. ½½ ½
Yo
O O CH3 ¾ C ¾ CH2 ¾ CH ¾ CH2 ¾ CHO
oo
5-keto- 3-methyl hexanal

eB

OH CH3
H
½
(d) CH3
½

H-atom at a carbon Carbanion O3


D
r

O Zn.H2O2
ou
ad

O O
O CH3
Y

H—C—H ½½ ½
H—C—H
CH3 ¾ C ¾ CH ¾ CH2 ¾ CH2 ¾ CHO
5-keto-4- methyl hexanal
nd
Re

13. PLAN This problem includes concept of nucleophilic addition reaction


O to carbonyl compound (ketone here) and intramolecular
O
Fi

nucleophilic substitution reaction.


Complete reaction sequence is as shown below:
Crossed – H2 O
aldol CH2O CH2OH O
Cl CH3MgBr, dry ether, 0°C
a-carbon has no H-atom
hence, next reaction with HCHO is CH3 Nucleophilic
crossed Cannizzaro reaction addition reaction
HCHO, OH-

OH OMgX CH3
O O aq. acid
Cl intramolecular
CH3 nucleophilic
O CH3
HCHO CH3 substitution reaction
Acetal CH2OH + HCOO– + Mg X Cl
formation (By oxidation)
(By reduction)
364 Aldehydes and Ketones

PCC CH2OH
14. R — CH2OH ¾ ¾¾® R — CH == O O
½½ ½
Pyridinium chlorochromate is the mild oxidising agent which H ¾ C ¾ H + – CH ¾ CHO r
causes conversion of alcohol to aldehyde stage. While others ½
causes conversion of alcohol to acid. CH2OH
15. The first step is cyanohydrin reaction O- CH2OH CH2OH
O ½ ½ H 2O
½
½½ H ¾ C ¾ C ¾ CHO ¾® HOCH2 ¾ C ¾ CHO
-
CH3 ¾ CH2 ¾ C ¾ CH3 + CN ¾® ½ ½ ½
H CH2OH CH2OH
O- OH
½ H2O ½ CH2OH O O-
CH3 ¾ CH2 ¾ C ¾ CN ¾¾® CH3 ¾ CH2 ¾ C ¾ CN ½ ½½ ½

w
½ ½ Step IV HOCH2 ¾ C ¾¾¾ C ¾ H + H ¾ C ¾ OH
CH3 CH3 ½ ½
(I) CH2OH H
In the second step the ¾ CN of intermediate (I) is first

Flo
OH
hydrolysed and then dehydrated on heating in the presence of OH
Cannizzaro reaction
conc. H2SO4. + HCOO–

ree
OH OH HO
½ H2SO4 ½ OH
CH3 ¾ CH2 ¾ C ¾ CN ¾¾¾® CH3CH2 ¾ C ¾ COOH

F
½ ½ In the last step, formaldehyde is oxidised and the other aldehyde
CH3 CH3 is reduced giving the desired products.
(I) O

or
17.
D
ur H
¾® CH3 ¾ CH == C ¾ COOH O3

f
½ H
CH3 Zn-H2O
ks
O
E
Yo
16. The given reaction is an example of repeated aldol condensation
oo
followed by Cannizzaro reaction.
KOH CHO
-
Step I CH3CHO + OH- ¾® CH2 ¾ CHO + H2O
eB

H 2O
O
Intramolecular aldol
½½ - condensation reaction
H ¾ C ¾ H + CH2 ¾ CHO r
r

18. Three different oximes are formed out of which two are optically
O-
ou

OH
ad

active i.e.exists as a pair of enantiomers while other is optically


½ ½
H 2O
H ¾ C ¾ CH2 ¾ CHO ¾® CH2 ¾ CH2 ¾ CHO inactive.
Y

½ 19. CH3
NaOH
H H 3C CHI3 + CH3CH2COONa
I2/H2O
- O
nd

Step II HOCH2 ¾ CH2 ¾ CHO + HO r


Re

H+
- CH3CH2COOH
HO ¾ CH2 ¾ CH ¾ CHO + H2O
Fi

O
½½ - 20. X is (CH3CO)2 O and it is an example of Perkin’s reaction.
H ¾ C ¾ H + CH ¾ CHO r
½ 21. The reactivity of carbonyl compound towards nucleophilic
CH2OH
addition of Grignard’s reagent depends on extent of steric
O- OH hindrance at a-carbon. Greater the steric hindrance smaller the
½ H 2O
½ reactivity. Hence, reactivity order is
H ¾ C ¾ CH ¾ CHO ¾¾® CH2 ¾ CH ¾ CHO
½ ½ ½ CH3CHO > CH3 ¾ CO ¾ CH3 > Ph ¾ CO ¾ Ph
H CH2OH CH2OH II I III
O
Step III HOCH2 ¾ CH ¾ CHO + HO- r
½ 22. —CHO + H—C—H
NaOH
—CH2OH
CH2OH
H2O
-
HOCH2 ¾ C ¾ CHO + H2O + H— COONa
½
CH2OH This is an example of cross Cannizzaro reaction in which
formaldehyde is always oxidised.
Aldehydes and Ketones 365

23. O 32. Grignard’s reagent reacts with both aldehydes and ketones
while other three reagents reacts only with aldehydes, not with
NH2–NH2 CH2CH3
CH3 ketones.
HO–
33. Only ¾ CHO group is to be reduced to ¾ CH2OH.
HO HO
It can be done using NaBH4 in C2H5OH.
(Wolff-Kishner reduction)
24. O IV O
O O O O C O
+ H+ H
– O III
II
Highly resonance
H

w
stabilised COOH
I
OH O
½ ½½ (a) LiAlH4 / (C2H5 )2 O reduces I, II and III into ¾ CH2OH, and
25. CH2 == C ¾ CH3 + D2O r D3C ¾ C ¾ CD3
IV into diol.

Flo
All a-H will be replaced by deuterium.
(b) BH3 / THF show same properties as (a).
26. H O (c) NaBH4 / C2H5OH reduces III into ¾ CH2OH.

ree
Cl (d) Raney nickel, same as (a) and (b), thus (c) is correct
Cl3C—CHO + H2O Cl—C—–CH reagent.

F
Cl 34. CH3
H O
O O O
Highly stable hydrate

or
ur HO–

OH O

f
CH3
Aldol type
½ [O] ½½ O
27. CH3 ¾ CH ¾ CH2 ¾ CH3 ¾¾® CH3 ¾ C ¾ CH2 ¾ CH3 Condensation
ks
2- butanol catalyst Ethyl methyl ketone CH3 CH3
Yo
O O O
oo
H+
28. Compounds that contain either CH3 ¾ CO or CH3 ¾ CH ¾ D
½
eB

group gives iodoform test : –H2O


OH
O a, b-unsaturated ketone
½½
CH3 ¾ C ¾ CH3 CH3 ¾ CH ¾ H CH3 ¾ CH ¾ CH3 35. Plan When acetone reacts with Br2 in basic medium, bromoform is
r

formed.
½ ½
ou

Acetone
ad

OH OH Reaction I CH3COCH3 + 3Br2 + 4NaOH


Ethanol Isopropyl alcohol 1 mol 3 mol
Y

1 1 mol
Above three compounds has the desired group for iodoform test. mol
3
Diethyl ketone does not has the required group for iodoform
¾® CH3COONa + CHBr3 + 3NaBr + 3H2O
nd

test.
Re

(T ) (U )
29. Aldehydes lacking presence of a-H undergo Cannizzaro When CH3COCH3 and Br2 are in equimolar quantity, all the Br2
Fi

reaction. When treated with aqueous base CH3CHO does not (limiting reactant) is converted into desired products and
undergo Cannizzaro reaction because it has a-H and in the 2/3 mole of CH3COCH3 remains unreacted, being in excess.
presence of aqueous base it undergoes aldol condensation. When acetone reacts with Br2 in acidic medium, there is
30. CH3 ¾ CHO + Fehling’s solution ¾¾® Cu 2O ¯ monobromination of acetone.
Red Reactions II
O CH COOH
CH3COCH3 + Br2 ¾¾¾¾®
3
CH3COCH2Br + HBr
½½
31. For iodoform test, CH3 ¾ C ¾ group is required 1 mol 1 mol (P )
CH3COCH3 and Br2 react in 1 : 1 mole ratio and (P) is formed.
O
In reaction I, (U ) and (T ) are formed and acetone (reactant)
+ NaOH + I2 remains unreacted. In reaction II, (P) is formed.
H3C CH3
2-pentanone
36. All those carbonyl compounds containing a-H to sp2 carbon
COONa + CHI3¯ show keto-enol tautomerism.
Yellow
366 Aldehydes and Ketones

37. In both Friedel-Craft’s reaction and Reimer-Tiemann reaction 43. For base catalysed aldol condensation, there must be at least one
new carbon-carbon bond is formed. a-H to carbonyl group.
CH3 CH3 ¾ CH2 ¾ CHO,
AlCl3 Propionaldehyde
+ CH3Cl (2 a -H)
CH3 CH3
Friedel-Crafts' alkylation ½ ½
OH OH CH3 ¾ CH ¾ CHO , CH3 ¾ C — CHO
CHO 2-methyl propionaldehyde ½
+
(1 a -H) CH3
H
+ CHCl3 + NaOH 2, 2-dimethyl propionaldehyde
(no a -H)
Passage 1 (For Q. Nos. 44-46)

w
Reimer-Tiemann’ reaction
The given product is an ester, obtained by condensation of a
38. All carbonyl compounds containing a-H or a-D undergo aldol
hydroxy acid obtained through hydrolysis of a cyanohydrin :
condensation. In given example, benzaldehyde does not contain
a-H to —CHO, hence does not undergo aldol condensation.

Flo
CH3
OH
H3C H3C
39. C == O + C6H5NH2 ¾® C == N ¾ C6H5 CH3—C——CH H3C OH
H3C H3C H+

ree
CH2 C==O D
H3C H 3C
+ C6H5NHNH2 ¾® C == N ¾ NHC6H5 HO
O O
OH

F
H3C
OH Acid above is obtained by acid hydrolysis of cyanohydrin S as
½

or
Dil. NaOH
40. 2CH3CHO ¾¾¾® CH3 ¾ CH ¾ CH2CHO
ur CH3 OH CH3 OH
Aldol H+

f
H3C—C——CH—CN H2O
H3C—C——CH—COOH
O CH3 O
½
ks
½½ Dil. NaOH ½½ CH2OH CH2OH
2CH3 ¾ C ¾ CH3 ¾¾¾® CH3 ¾ C — CH2 ¾ C ¾ CH3
Yo
S
½
oo
OH S is obtained by nucleophile addition of HCN on R, hence R is
Aldol
eB

Dil. NaOH CH3 O CH3 OH


2HCHO ¾¾¾® CH3OH + HCOONa
Cannizzaro reaction H3C—C——C—H + HCN H3C—C——CH—CN
Dil. NaOH
r

C6H5CHO + HCHO ¾¾¾® C6H5CH2OH + HCOONa CH2OH CH2OH


ou
ad

Cannizzaro reaction R S
N S 2
41. CH3CH2Cl + KCN ¾® CH3CH2CN + KCl
Y

R is obtained by treatment of P and Q with aqueous K2CO3


O O through aldol condensation reaction as
½½ ½½ CH3 O CH3
nd

CH3 ¾ C ¾ Cl + KCN ¾® CH3 ¾ C ¾ CN + KCl


Re

OH–
O O CH3—CH—CHO + H—C—H OHC—C—CH2OH
½½ ½½
Fi

P+Q CH3
C6H5 ¾ C ¾ H + KCN ¾® C6H5 ¾ C ¾ CH ¾ C6H5
R
½
OH Passage 2 (For Q. Nos. 47-49)
Benzoin
O CH3 CH3
CH3MgBr H+/H2O
½½
42. Aldehyde, ketones containing CH3 ¾ C ¾ group gives iodoform CH3
O CH3
test. Also alcohol containing CH3 ¾ CH ¾ group gives CH3
½ P Q
OH (gives positive iodoform test)
OH O
½ ½½ CHO
iodoform test. CH3 ¾ C — CH3 C6H5 ¾ C ¾ CH3 O3 CH3 OH–
Q
½ Zn-H2O aldol
H O O
Both gives positive iodoform test H 3C CH3
Esters and amides do not give iodoform test. R S
Aldehydes and Ketones 367

Passage 3 (For Q. Nos. 50-52) 54.


Column I Column II
O
1. NaBH4 A. C6H5CHO Gives phenyl hydrazone with 2,
2. PBr3 4-dinitrophenyl hydrazine.
H Br
I Gives precipitate with AgNO3, Tollen’s
(i) Mg/ether test forms cyanohydrin.
(ii) CO2 B. CH3 ¾ C ºº CH Gives precipitate (CH3 ¾ C ºº CAg)
COOH
(iii) H3O+ J with AgNO3 .
A nucleophile, undergo electrophilic attack.
SOCl2
J C. CN- Forms AgCN with AgNO3.
K C—Cl A nucleophile is involved in cyanohydrin

w
formation.
O
H H D. I- Gives AgI precipitate with AgNO3 and it is
H2
C==C a nucleophile.

Flo
Pd/BaSO4
H5C2 CH2CH2CHO 55. Sodium potassium tartarate
L

ree
O CH3
Column I Column II ½½ H2 O ½
53. 56. CH3MgI + CH3 ¾ C ¾ CH3 ¾® CH3 ¾ C — OH

F
Undergo nucleophilic substitution of ½
Br CH3
Br - . Undergo elimination of HBr. Tertiary alcohol
A.

or
O
ur
Does not undergo nucleophilic
57. Benzaldehyde, lacking a-H does not undergo aldol
addition. Does not esterify with

f
condensation, rather it undergoes Cannizzaro reaction.
acetic anhydride, can be
ks
dehydrogenated. 58. Molecular weight of the ketone is 100
So, molecular formula = C6 H12O
Yo
Undergo nucleophilic substitution 12
oo
OH Degree of unsaturation = (6 + 1) - =1
B. with SOCl 2 , PCl 5 etc. 2
Does not undergo elimination.
eB

According to question, compound contains ketone group. Since,


Does not undergo nucleophilic the compound which contain chiral centre lead to the formation
addition. of diastereomer while other produces enantiomers. Various
Undergo esterification with acetic isomers and their possible reduced product are as shown below.
r
ou

anhydride.
ad

(1) n-butyl–C—CH3 n-butyl–CH—CH3


Undergo dehydrogenation to give
OH
Y

C6 H5 CHO. O
All are
(2) Iso-butyl–C—CH3 Iso-butyl–CH—CH3 + racemic
CHO Does not undergo nucleophilic mixture
nd
Re

C. substitution, there is no leaving O NaBH4 OH


group. (3) 3° butyl–C—CH3 3° butyl–CH—CH3
OH
Fi

Does not undergo elimination.


O OH
Undergo nucleophilic addition at
carbonyl carbon of —CHO. CH3 CH3
Undergo esterification with acetic *
(4) CH3CH2—CH—C—CH3 CH3—CH2—CH—CH—CH3
anhydride. * *
Does not undergo dehydrogenation. O OH
R (2°butyl) 2-alcohols (R,R) and (R,S)
Undergo aromatic nucleophilic diastereomeric pair
Br CH3 NaBH4
D. substitution (SN Ar) CH3
Does not undergo elimination, (5) CH3CH2—CH—C—CH3
NO2 * *
nucleophilic addition, esterification CH3—CH2—CH—CH—CH 3
O *
or dehydrogenation. S (2°butyl)
OH
2-alcohols (S,S) and (S,R)
(Here, (*) represents chiral centre) diastereomeric pair
368 Aldehydes and Ketones

Ph Ph Ph Ph
n-butyl–CH—Et A= C == C + C == C
(6) n-propyl–C—Et H3C CH3 H3C CH3
(±) racemic
OH mixture I II
O NaBH4 H
(7) Iso-propyl–C—Et Iso-propyl–CH—Et H
O (±)racemic Ph + Enantiomer
OH II + H2/Pd
mixture CH3
= Racemic mixture
Ph CH3
While in case of (4) and (5) they do not produce enantiomer
due to the presence of stereogenic centre on ketone.
63.
59. O O O O
CH3
CH3

w
O3 OH– CH3 Br2
Zn-H2O
NaOH
COONa + CHBr3
O O ‘B’
Only product

Flo
Y ‘A’ O
CH3 CH3 Cl CH3
½ ½ ½ H+

ree
60. CH2 == CH ¾ C — CH3 CH3 ¾ CH — C — CH3 Heat
+ CO2
½ ½
CH3 CH3 C

F
A B
64. O
Cl CH3 CH3

or
½ ½
ur ½ OH– C
CH3 ¾ CH — C — CH3 CH3 ¾ C == C — CH3 C CH3

f
Aldol
½ ½
CH3 CH3
ks
D
C D CHO O
Yo
CH3 CH3
oo
O CH3
½½ ½ Þ C= B=
CH3 ¾ C ¾ CH3 CH3 ¾ C — CHO HCHO
eB

E ½ G
CH3 MeMgBr H+
A (C6H10O) ¾¾¾® ¾¾® B
F
O
r

61.
ou
ad

H
O O O Þ + CH3MgBr
Y

CHO
H A HO CH3 CH3
nd
Re

H+
Fi

Cl Cl
A B B
COOH
KMnO4
65. Ph Ph
Base
_
O O
Cl Br Br
Enolisation
62. B + NaOH / I2 ¾® C6H5COONa + CHI3
Ph
O
CH2CH3 O
C
N2H4/OH–
Þ B= CH3
Aldehydes and Ketones 369

66. (a) G is benzoic acid C6H5 ¾ COOH , B and C are two 68. O
stereomeric oximes which undergo Beckmann’s rearrangement O CH—Ph
on treatment with acid to give amides D and E. (i) NaOH
O (ii) C6H5CHO
½½ OH-
C6H5 ¾ C ¾ NHCH3 ¾¾® C6H5COOH + CH3NH2 CH—Ph
E H2 O G (i) LiAlH4
O
H5C6 OH (ii) H+, heat
+
H
C==N C6H5—C—NHCH3
H3 C E 69. Aldehyde A does not has any a-H but undergo ozonolysis to
give two moles of compound B and benzaldehyde. Compound B
C on oxidation gives oxalic acid, so A is

w
O
H5C6 O3
H+ C6H5 ¾ CH == CH¾ C ºº C ¾ CHO ¾®
C==N CH3—C—NHC6H5 A
H 3C OH
D C6H5 ¾ CHO + 2COOH

Flo
½
B KOH CHO
CH3COOH + C6H5NH2
H2O B
F

ree
F + CH3COCl ¾® C6H5NHCOCH3 Ag +

D B ¾¾® COOH
O ½

F
½½ COOH
Þ A = C6H5 ¾ C ¾ CH3
70.
(b) CH3 ¾ CH2 ¾ C ºº CH

or
O
O
ur H5C6 OH

f
B2 H6 ½½ (i) CH3 MgBr
C6H5—C—CH3 + H2NOH C==N
¾¾¾® CH3CH2CH2 ¾ C ¾ H ¾¾¾¾¾® H 3C
H2 O2 /OH-
ks
+
(ii) H3 O
Yo
OH H5C6
oo
½
CH3CH2CH2 ¾ CH ¾ CH3 + C==N
eB

O H 3C OH
[O] ½½ O CH2
¾® CH3CH2CH2 ¾ C ¾ CH3
2-pentanone.
r
ou

71. (i) (Wittig reaction)


ad

67. (i) C6H5 ¾ CH ¾ C ºº C ¾ H + Ph3P==CH2


½
Y

OH A
+ + O
H
¾¾® C6H5 ¾ CH ¾ C ºº C ¾ H ½½ CH3O-
nd

- H2 O
Re

(ii) Cl ¾ CH2 ¾ CH2 ¾ CH2 ¾ C ¾ Ph ¾¾®


··
OH2 Cl Ph
Fi

O O
CH2
-H+ ½½ Ph—C—
¾® C6H5 ¾ CH== CH ¾ C ¾ H CH2
HC B
C6 H5 ¾ CH == C == C ¾ OH
e
tautomerisation ½
H O
COPh
O
(iii)
(ii) H+ C6H5
OH– –
+ H 3C H2C C==O
OH O O
Ph
H O O
H+/D
– H+ OH
—Ph
Ph C
O
370 Aldehydes and Ketones

72. 76. CH3 ¾ CO ¾ CH2 ¾ COCH3 has highest enol content due to
CH3 CH3 CH3 resonance and formation of six membered ring through
intramolecular H-bonding
Dil. KMnO4 OH HIO4 O O O H
OH O O O
CH3 CH3 CH3 H3C CH3
A B
O (A) H3C C H3
(I)
HO–
B Also, enol content depends upon the number of substituents on
aldol double bond, greater the number of substituents, greater the
stability, higher the enol content.
C

w
73. KCN + H2SO4 ¾® KHSO4 + HCN Therefore, CH3COCH2CHO forms next most stable enol

O OH 77. Iodoform reaction is an oxidation reaction in which hypoiodite


½½ ½ OI- acts as oxidising agent :

Flo
CH3CH2 ¾ C ¾ H + HCN ¾® CH3CH2 ¾ C — CN O
OH ½ ½½
H R ¾ C ¾ CH3 + OI- ¾® R ¾ COO- + CHI3
½

ree
LiAlH4 I
¾¾® CH CH ¾ C — CH NH
3 2 2 2 Iodide (I- ) is a reducing agent, does not give iodoform reaction.
½
H O

F
78. D ¾®
3 Zn CH CHO (only)
¾®
II 3
H2 O
I is formed as racemic mixture.
C = CH3 ¾ CH == CH ¾ CH3,

or
O
ur
½½ O

f
C2 H5 ONa
74. C6H5CHO + CH3 ¾ C ¾ OC2H5 ¾¾¾® D = CH3 ¾ CH CH ¾ CH3
C2 H5 OH
ks
½ ½
O
Yo
O ¾¾ O
½½
oo
C6H5 ¾ CH == CH ¾ C ¾ OC2H5 OH O
½ ½½
eB

75. B = CH3 ¾ CH ¾ CH2CH3 and A = CH3 ¾ C ¾ CH2CH3


KOH 79. (i) Tollen’s reagent gives grey precipitate of Ag, acetone
MeO— —CHO + HCHO does not.
r

CH3 CCl3
ou
ad

hn
(ii) + Cl2 + HCl
MeO— —CH2OH + HCOOK (excess)
Y

(Cannizzaro reaction) 80. NaOH


C6H5CHO + CH3CHO —CH==CH—CHO
nd

In cross-Cannizzaro reaction, methanal is always oxidised.


Re

O O Cross-aldol condensation
H
Fi

O O 81.
H CHO COOH
[O] NaOH
B H3C H
II K2Cr2O7/H+ CaO/D

II is less stable than I because II is less substituted enol. NO2


HNO3
Acetone has greater enol content than ethanal
O OH H2SO4
½½ ½
CH3 ¾ C ¾ CH3 s CH2 == C ¾ CH3 (more substituted) NO2 NH2
C Fe/HCl NaNO2
O OH HCl/0°C
½½ ½ +
CH3 ¾ C ¾ H s CH2 == C — H (less substituted) N2Cl– CN
D KCN
Therefore, overall enol-content order is D < C < B < A CuCN
Aldehydes and Ketones 371

O O OH
½½ R ½½ HCN
½ H+
82. R ¾ C ¾ R + PhNHNH2 ¾® C ==N ¾ NHPh 87. CH3 ¾ C ¾ CH3 ¾¾® CH3 ¾ C — CN ¾¾®
R Hydrazone ½ H2 O
CH3 OH
In acidic medium, hydrazine reacts to form salt and hydrazone is
½
hydrolysed back to aldehyde/ketone. CH3 ¾ C — COOH
OH- ½
83. CH3CH2CHO ¾¾® CH3 ¾ CH2 ¾ CH == C ¾ CHO CH3
heat ½
Br D
CH3 CH3
Aldol followed by ½ ½
dehydration Þ C = CH3 ¾ C — COOH , B = CH3 ¾ CH — COOH
½

w
84. Steric hindrance at carbonyl carbon determine the reactivity CH3
towards nucleophilic addition reaction. Greater the steric
hindrance, smaller the reactivity. CH3 CH3
½ ½
C2H5COCH3 < CH3COCH3 < CH3CHO < HCHO Þ A = CH3 ¾ C== CH ¾ CH ¾ CH3

Flo
H+ Ca(OH) O
85. CH3CHO ¾¾® CH3COOH ¾¾®2 (CH3COO)2 Ca
K 2 Cr 2 O7 ½½ E ther

ree
O 88. H ¾ C ¾ H + CH3MgBr ¾¾® CH3CH2OMgBr
heat ½
½ H2 NOH
(CH3COO)2 Ca ¾® CH3 ¾ C ¾ CH3 ¾¾® H+
¾¾® CH3CH2OH

F
H2 O
H3C
C == N ¾ OH 89. N

or
ur
H3C acetoxime
‘‘(CH2)6N4’’
6CH2O + 4NH3 N
86. Cl 3C ¾ CHO + NaOH(aq) ¾® Cl 3C ¾ CH2OH

f
N N
Cannizzaro reaction
ks
Hexamine
+ Cl 3C ¾ COONa
Yo
oo
eB

Download Chapter Test


http://tinyurl.com/yx9d32yd or
r
ou
ad
Y
nd
Re
Fi
26
Carboxylic Acids and
Their Derivatives

w
Flo
Topic 1 Carboxylic Acids

ree
Objective Questions I (Only one correct option) 5. The compound that undergoes decarboxylation most
1 Which dicarboxylic acid in presence of a dehydrating agent readily under mild condition is (2012)

F
is least reactive to give an anhydride? (2019 Main, 10 Jan I) COOH COOH
CH2 CH2COOH O
CO2H (a) (b)

or
CH2
(a)
ur
(b)
COOH

COOH

f
CO2H CH2 COOH CH2COOH
CH2 COOH O
ks
O (c) (d)
Yo
COOH C
oo
CH2 OH
(c) (d) 6. The carboxyl functional group ( ¾ COOH) is present in
CH2 OH
eB

COOH C (a) picric acid (2012)


(b) barbituric acid
O
(c) ascorbic acid
2. In the reaction,
r

(d) aspirin
ou
ad

LiAlH 4 PCl 5 Alc. KOH


CH3 COOH ¾¾® A ¾¾® B ¾¾¾® C 7. In the following reaction sequence, the correct structures
The product C is of E , F and G are
Y

(2014 Main)
(a) acetaldehyde (b) acetylene O O
(c) ethylene (d) acetyl chloride Heat I2
nd

Ph OH [E] [F] + [G]


Re

3. The compound that does not liberate CO2 , on treatment with * NaOH
aqueous sodium bicarbonate solution, is (2013 Adv.) (* implies 13 C labelled carbon)
Fi

(2008, 3M)
(a) benzoic acid O O
(b) benzenesulphonic acid
(a) E = F= Å G = CHI3
(c) salicylic acid Ph * CH3 Ph * ONa
(d) carbolic acid (Phenol)
O O
4. An organic compound A upon reacting with NH3 gives B. On
(b) E = * F= Å G = CHI3
heating, B gives C. C in the presence of KOH reacts with Br2 Ph CH3 Ph ONa
to give CH3 CH2 NH2 . A is (2013 Main)
(a) CH3 COOH O O
(b) CH3 CH2 CH2 COOH *
(c) E = * F= Å G = CHI3
(c) CH3 ¾ CH ¾ COOH Ph CH3 Ph ONa
½
CH3 O O
*
(d) CH3 CH2 COOH (d) E = * F= Å G = CH3I
Ph CH3 Ph ONa
Carboxylic Acids and Their Derivatives 373

8. When benzene sulphonic acid and p-nitrophenol are treated List–I List–II
with NaHCO3 , the gases released respectively, are (2006, 3M)
P. Ph 1. I2 , NaOH
(a) SO2 , NO2 (b) SO2 , NO (c) SO2 , CO2 (d) CO2 , CO2
HO
9. An enantiomerically pure acid is treated with racemic Me
Ph + H2SO4
mixture of an alcohol having one chiral carbon.The ester OH
formed will be (2003, S, 1M) Me
(a) optically active mixture (b) pure enantiomer
Q. Ph 2. [Ag(NH 3 )2 ] OH
(c) meso compound (d) racemic mixture
H2 N
10. Benzoyl chloride is prepared from benzoic acid by H
Ph + HNO2
(a) Cl 2 , hn (b) SO2 Cl 2 (2000, S, 1M) OH
(c) SOCl 2 (d) Cl 2 , H2 O Me

w
11. When propionic acid is treated with aqueous sodium R. Ph 3. Fehling
bicarbonate, CO2 is liberated. The C of CO2 comes from HO solution
Ph
(a) methyl group (1999, 2M) Me + H2SO4

Flo
(b) carboxylic acid group OH
Me
(c) methylene group
(d) bicarbonate group

ree
S. Ph 4. HCHO, NaOH
12. Which of the following is basic ? (1980, 1M) Br
H
(a) CH3 CH2 OH (b) H2 O2 Ph + AgNO3

F
OH
(c) HOCH2 CH2 OH (d) CH3 COOH
Me

or
Matching Type Questions
ur 5. NaOBr

f
13. The desired product X can be prepared by reacting the major
The correct option is
product of the reactions in List-I with one or more
(a) P ® 1; Q ® 2, 3; R ® 1, 4; S ® 2,4
ks
appropriate reagents in List-II.
(b) P ® 1, 5; Q ® 3,4; R ® 4, 5; S ® 3
Yo
(given, order of migratory aptitude : aryl > alkyl > hydrogen)
oo
(c) P ® 1, 5; Q ® 3,4; R ® 5; S ® 2,4
O (d) P ® 1, 5; Q ® 2, 3; R ® 1,5; S ® 2,3
eB

Ph
OH
Me Ph Subjective Questions
X 14. How will you bring about the following conversion?
r

(2018 Adv.)
“Ethanal to 2-hydroxy-3-butenoic acid.’’
ou

(1990, 2M)
ad
Y

Topic 2 Acid Derivatives


Objective Questions I 2. The major product of the following reaction is
nd
Re

(2019 Main, 8 April I)


(Only one correct option) O Cl
Fi

1. The major product of the following reaction is


(2019 Main, 9 April II) (i) AlCl3, heat
O+
OH (ii) H2O

CH2OH H2SO4(cat.) O
CO2Et CHCl3
O
O O CO2H Cl
(a) (b)
(a) (b)
Cl
CO2Et COOH
O O
OH OH
O Cl O
(c) (d)
OEt O (c) (d)
O O COOH Cl
O
374 Carboxylic Acids and Their Derivatives

3. The increasing order of the reactivity of the following with 8. Different possible thermal decomposition pathways for
LiAlH 4 is (2019 Main, 12 Jan II) peroxyesters are shown below. Match each pathway from
O O Column t I with an appropriate structure from Column II and
(A) (B) select the correct answer using the code given below the lists.
(2014 Adv.)
C2H5 NH2 C2H5 OCH3 P
R· + R¢O·
O O O –CO2

(C) (D) O Q
C2H5 Cl C2H5 O C2H5 O R· + R¢O· R· + X·
–CO2
R O R¢ + Carbonyl compound
(a) ( A ) < ( B ) < ( D ) < (C )
(b) ( A ) < ( B ) < (C ) < ( D ) (Peroxyester) R
RCO· 2+ R¢O· R· + X¢·
–CO2

w
(c) ( B ) < ( A ) < ( D ) < (C ) + Carbonyl
compound
(d) ( B ) < ( A ) < (C ) < ( D ) S
RCO2· + R¢O· R· + R¢O·
4. The major product obtained in the following reaction is –CO2

Flo
(2019 Main, 10 Jan II)
CO2Et
NaOEt/D Column I Column II

ree
O
O O
O O P. Pathway P 1. O

F
C6H5CH2 O CH3
(a) (b)
CO2Et
O

or
CO2Et
ur Q. Pathway Q 2. O
C6H5 O CH3

f
O O

O
ks
(c) (d)
CO2Et
O
Yo
CH3
R. Pathway R 3. C6H5CH2 O
oo
CO2Et CH3
CH2C6H5
5. The decreasing order of ease of alkaline hydrolysis for the
eB

following esters is (2019 Main, 10 Jan I) O


O CH3
COOC2H5 Cl COOC2H5 S. Pathway S 4. C6H5 O
r

CH3
C6H5
ou
ad

I II

Codes
Y

O2N COOC2H5 CH3O COOC2H5


P Q R S
III IV (a) 1 3 4 2
nd

(b) 2 4 3 1
Re

(a) III > II > IV > I (b) III > II > I > IV
(c) 4 1 2 3
(c) II > III > I > IV (d) IV > II > III > I (d) 3 2 1 4
Fi

6. The compounds A and B in the following reaction are, 9. A compound with molecular mass 180 is acylated with
respectively CH 3 COCl to get a compound with molecular mass 390. The
HCHO+HCl AgCN number of amino groups present per molecule of the former
A B
compound is (2013 Main)
(2019 Main, 9 Jan I)
(a) 2 (b) 5
(a) A = Benzyl alcohol, B = Benzyl isocyanide
(c) 4 (d) 6
(b) A = Benzyl alcohol, B = Benzyl cyanide
(c) A = Benzyl chloride, B = Benzyl isocyanide 10. Which of the following reactants on reaction with
(d) A = Benzyl chloride, B = Benzyl cyanide conc. NaOH followed by acidification gives the following
7. The major product of following reaction is lactone as the only product? (2006, 5M)
O
(i) AIH( i - Bu) 2
R ¾ C ºº N ¾¾¾¾¾¾¾®?
(ii) H 2O (2019 Main, 9 Jan I)
O
(a) RCHO (b) RCONH2
(c) RCOOH (d) RCH2 NH2
Carboxylic Acids and Their Derivatives 375

COOCH3 COOH 17. Identify the binary mixture(s) that can be separated into
(a) (b) individual compounds, by differential extraction, as shown
in the given scheme. (2012)
COOH CHO
NaOH (aq)
CHO COOH
Binary mixture containing
(c) (d) Compound 1 and Compound 2 NaHCO3 (aq)
CHO COOH
Compound 1 + Compound 2
11. Benzamide on treatment with POCl 3 gives (2004, S, 1M) Compound 1 + Compound 2
(a) aniline (b) benzonitrile
(c) chlorobenzene (d) benzyl amine (a) C6 H5 OH and C6 H5 COOH

w
CH3MgBr (b) C6 H5 COOH and C6 H5 CH2 OH
12. Ethyl ester ¾¾¾® P, the product ‘P’ will be
(excess) (2003, S, 1M) (c) C6 H5 CH2 OH and C6 H5 OH
H 3C CH3 H3C C2H5 (d) C6 H5 CH2 OH and C6 H5 CH2 COOH

Flo
(a) (b)
H 3C OH H5C2 OH 18. Reaction of RCONH2 with a mixture of Br2 and KOH gives
R ¾ NH2 as the main product. The intermediates involved in

ree
H5C2 C2H5 H5C2 C2H5 this reaction are (1992, 1M)
(c) (d)
(a) RCONHBr
H5C2 OH H7C3 OH

F
(b) RNHBr
13. The product of acid hydrolysis of P and Q can be (c) R ¾ N == C == O
distinguished by (2003, S, 1M)

or
(d) RCONBr2
OCOCH
ur
H3C
3

f
P : H2C== ; Q: Assertion and Reason
CH3
ks
OCOCH3 Read the following questions and answer as per the direction
Yo
(a) Lucas reagent (b) 2, 4-DNP given below :
oo
(c) Fehling’s solution (d) NaHSO 3 (a) Statement I is true; Statement II is true; Statement II is a
correct explanation of Statement I.
eB

14. Hydrogenation of benzoyl chloride in the presence of Pd on (b) Statement I is true; Statement II is true; Statement II is not
BaSO4 gives (1992, 1M) the correct explanation of Statement I.
(a) benzyl alcohol (b) benzaldehyde (c) Statement I is true; Statement II is false.
r

(c) benzoic acid (d) phenol (d) Statement I is false; Statement II is true.
ou
ad

15. Acetamide is treated separately with the following reagents. 19. Statement I p-hydroxybenzoic acid has a lower boiling
Y

Which one of these would give methyl amine ? (1983, 1M) point than o-hydroxybenzoic acid.
(a) PCl 5 (b) NaOH + Br2 Statement II o-hydroxybenzoic acid has intramolecular
(c) Sodalime (d) Hot conc. H2 SO4 hydrogen bonding. (2007, 3M)
nd
Re

Objective Questions I 20. Statement I Acetic acid does not undergo haloform reaction.
Statement II Acetic acid has no alpha hydrogen. (1998, 2M)
Fi

(One or more than one correct options)


16. With reference to the scheme given, which of the given Passage Based Questions
statement(s) about T, U, V and W is/are correct? (2012)
Passage 1
O
The reaction of compound P with CH3 MgBr (excess) in (C2 H5 )2 O
O followed by addition of H2 O givesQ. The compoundQ on treatment
with H2 SO4 at 0° C gives R. The reaction of R with CH3 COCl in the
H3C
T presence of anhydrous AlCl 3 in CH2 Cl 2 followed by treatment with
LiAlH4 (excess) H2O produces compound S . [Et in compound P is ethyl group]
CrO3/H+ (CH3CO)2O
V U W (H3C)3C
(a) T is soluble in hot aqueous NaOH CO2Et Q R S
(b) U is optically active
P
(c) Molecular formula of W is C10H18 O4
(d) V gives effervescence on treatment with aqueous NaHCO3
376 Carboxylic Acids and Their Derivatives

O
21. The product S is (2017 Adv.)

COCH3 H 3C (c) O and


CH3
(a) (H3C)3C CH3 (b) (H3C)3C
O (W)
(V)
COCH3
HO3S H3COC HOH2C
(c) (H3C)3C O CH3 (d) (H3C)3C H 3C CH3 (d) and
CH2OH CH2OH
(V) (W)

w
COCH3
Passage 3
22. The reactions, Q to R and R to S , are RCONH2 is converted into RNH 2 by means of Hofmann’s

Flo
(a) Aromatic sulfonation and Friedel-Crafts acylation bromamide degradation.
(b) Friedel-Crafts alkylation and Friedel-Crafts acylation O O
(c) Friedel-Crafts alkylation, dehydration and Friedel-Crafts Cl— Cl

ree
acylation NH2 NHBr
(i) (ii)
(d) Dehydration and Friedel-Crafts acylation

F
O
Passage 2
C O
P and Q are isomers of dicarboxylic acid C4 H4 O4 . Both decolourize

or
ur
Br2 / H2 O. On heating, P forms the cyclic anhydride. N Cl Cl

f
Upon treatment with dilute alkaline KMnO4 . P as well as Q could (iii) N—Br
(iv)
produce one or more than one form S, T and U. (2013 Adv.)
ks
Yo
COOH COOH COOH – +
oo
OM
H OH H OH HO H O
eB

H OH HO H H OH N Cl NH2 Cl
H
COOH COOH COOH (v) (vi)
S T U
r

In this reaction, RCONHBr is formed from which this


ou

23. Compounds formed from P and Q are, respectively


ad

reaction has derived its name. Electron donating group at


(a) Optically active S and optically active pair (T, U)
phenyl activates the reaction. Hofmann’s degradation
Y

(b) Optically inactive S and optically inactive pair (T, U)


reaction is an intramolecular reaction. (2006, 3 ´ 4M = 12M)
(c) Optically active pair (T, U) and optically active S
(d) Optically inactive pair (T, U) and optically inactive S 25. How can the conversion of (i) to (ii) be brought about?
nd
Re

(a) KBr
24. In the following reaction sequences V and W are respectively (b) KBr + CH 3 ONa
Fi

H 2 / Ni (c) KBr + KOH


Q ¾¾¾® V
D (d) Br 2 + KOH
AlCl3 (anhydrous) (i) Zn—Hg/HCl 26. Which is the rate determining step in Hofmann’s
+V W bromamide degradation?
(ii) H3PO4
(a) Formation of (i)
O
(b) Formation of (ii)
(c) Formation of (iii)
(a) O and (d) Formation of (iv)
27. What are the constituent amines formed when the mixture of
O O
(1) and (2) undergoes Hofmann’s bromamide degradation?
(V) (W)
CH2OH 15
CONH2 CONH2
(b) and
CH2OH D
(1) (2)
(V) (W)
Carboxylic Acids and Their Derivatives 377

15 *
(a) NH2, NH2, NH2, formaldehyde (H2 CO) as one of the products when
compound (Z) is treated with HBr and subsequently
D D ozonolysed. Mark the C* carbon in the entire scheme.
15
NH2 * *
Ba CO3 + H2 SO4 ¾® X ( C = C14 gas )
15
(b) NH2, NH2 (i) Mg/ether
4 LiAlH
CH2 == CH ¾ Br ¾¾¾¾® Y ¾¾¾® Z
(ii) X
D (ii) H3 O+ (2001 Main, 5M)
15
(c) NH2, NH2 36. Write the structures of the products A and B.
O
½½ 18 H3O+

w
15
(d) NHD, NH2 CH3 ¾ C ¾ O C2H5 ¾¾® A+ B (2000 Main, 3M)
37. Explain briefly the formation of the products giving the
structures of the intermediates
Fill in the Blank

Flo
O
28. Formic acid when heated with conc. H2 SO4 produces O
OC2H5
…………………

ree
(1983, 1M) OC2H5
C (i) OH–
NaOEt Br
H 2C
True/False C OC2H5
(ii) H+

F
COOH
29. The boiling point of propanoic acid is less than that of n-butyl
alcohol, an alcohol of comparable molecular weight. O

or
ur (1991, 1M) COOH
(1999, 5M)
30. Hydrolysis of an ester in the presence of a dilute acid is

f
38. Write the structures of the products :
known as saponification. (1983, 1M)
ks
(CH3CO) 2 O, heat
CH3 CH2 NH2 ¾¾¾¾¾¾®
Yo
(1998)
Integer Type Question
oo
39. An ester A (C4H8O2), on treatment with excess methyl
31. The total number of carboxylic acid groups in the product P
magnesium chloride followed by acidification, gives an
eB

is (2013 Adv.)
alcohol B as the sole organic product. Alcohol B, on
O O oxidation with NaOCl followed by acidification, gives acetic
(i) H3O+ , D
acid. Deduce the structures of A and B. Show the reactions
r

O (ii) O3
P involved. (1998)
ou
ad

(iii) H2O2 40. Complete the following, giving the structures of the principal
O O organic products
Y

Me
Subjective Questions (i) + (COOEt)2 + EtONa A
nd
Re

CH2Cl NO2
32.
Fi

conc. H2SO 4
+ (ii) (COOH)2 + (CH2OH)2 ¾¾¾¾® ¾® B
KCN C2H5ONa/EtOH H3O /D
(A) (B) ( C) H O+
Heat
DMF C6H5CHO (iii) H3CCOCOC6H5 + NaOH ¾¾®
3
C (1997, 2M)
SOCl2
(D) 41. A hydrocarbon A of the formula C8 H10 , on ozonolysis gives
CH3NH2
Identify A to D. (2004, M)
compound B (C4 H6 O2 ) only. The compound B can also be
obtained from the alkyl bromide C (C3 H5 Br) upon treatment
33. Compound A of molecular formula C9 H7 O2 Cl exists in keto with magnesium in dry ether, followed by carbon dioxide
form and predominantly in enolic form B. On oxidation with and acidification. Identify A, B and C and also give equations
KMnO4 , A gives m-chlorobenzoic acid. Identify A and B. for the reactions. (1996, 3M)
(2003 Main, 2M)
42. Complete the following sequence of reactions with
34. (±) 2-phenylpropanoic acid on treatment with (+) 2-butanol appropriate structures
gives (A) and (B). Deduce their structures and also establish
Red-P A
CH3 ¾ CH2 ¾ COOH ¾¾¾®
stereochemical relation between them. (2003)
Br 2
35. Identify X and Y in the following synthetic scheme and
(i) Alc. KOH (excess)
write their structures. Explain the formation of labelled A ¾¾¾¾¾¾¾® B
(ii) H+ (1995, 2M)
378 Carboxylic Acids and Their Derivatives

43. Which of the following carboxylic acids undergoes and 1.36 Å and both the carbon-oxygen bonds in sodium
decarboxylation easily ? Explain briefly. formate have the same value, i.e. 1.27Å.” (1988, 2M)
(i) C6H5COCH2COOH 51. Write balanced equations for the following reaction :
(ii) C6H5COCOOH “Acetamide is reacted with bromine in the presence of
(iii) C6H5CH(OH)COOH potassium hydroxide.” (1987, 1M)
(iv) C6H5CH(NH2) COOH (1995, 2M) 52. A liquid X, having a molecular formula C6 H12 O2 is hydrolysed
44. Predict the major product in the following reaction : with water in the presence of an acid to give a carboxylic acid
(i) CH MgBr (excess) Y and an alcohol Z. Oxidation of Z with chromic acid gives Y.
3
C6H5 ¾ CH2COCH3 ¾¾¾¾¾¾¾® (1994, 1M) What are the structures of X, Y and Z ? (1986, 3M)
(ii) H+
53. An ester A (C4H8O2) on treatment with excess of methyl
45. In the following reactions, identify the compounds A, B, C magnesium chloride followed by acidification, gives an alcohol

w
and D. B as the sole organic product. Alcohol B, on oxidation with
(i) PCl5 + SO2 ¾® A + B NaOCl followed by acidification, gives acetic acid. Deduce
(ii) A + CH3COOH ¾® C + SO2 + HCl structures of A and B. Show the reactions involved. (1998)

Flo
(iii) 2C + (CH3 )2 Cd ¾® 2D + CdCl2 (1994, 1M ´ 4 = 4M) 54. Complete the following with appropriate structures :
46. Complete the following sequence of the reactions with C H OH
(CH3CO)2 O ¾¾¾®
2 5
CH3COOH + ? (1986, 1M)
appropriate structures

ree
55. Arrange the following in order of their increasing ease of
Fuming 1. NaOH (Fuse) hydrolysis : (1986, 1M)
(i) —SO3H

F
+
H2SO4 2. H CH3COOC2H5 , CH3COCl, (CH3CO)2 O, CH3CONH2

P2O5
+
H , H2O 56. Give reasons in one or two sentences for the following :

or
(ii) —CONH2
ur D “Formic acid is a stronger acid than acetic acid.” (1985, 1M)

f
(1992, 1M) 57. Write down the reactions involved in the preparation of the
47. In the following identify the compounds/reaction following using the reagents indicated against in parenthesis.
ks
conditions represented by the alphabets A, B, and C : “Propionic anhydride from propionaldehyde”
Yo
[AgNO3 , NH4 OH, P2 O5 ]
oo
(1984, 2M)
5 PCl 3 NH
C6 H5 COOH ¾¾® A ¾¾® 58. Give reasons for the following in one or two sentences.
eB

P2 O5 H2 /Ni ‘‘Acetic acid can be halogenated in the presence of P and Cl 2 ,


B ¾¾® C6 H5 CN ¾¾® C (1991, 2M)
but formic acid cannot be halogenated in the same way.’’
48. Arrange the following as stated: Why? (1983, 1M)
r

“Increasing order of acidic strength.”


59. State with balanced equation, what happens when, “Acetic
ou
ad

ClCH2COOH, CH3CH2COOH, ClCH2CH2COOH, anhydride reacts with phenol in presence of a base.” (1982, 1M)
(CH3 )2 CHCOOH, CH3COOH
Y

(1991, 1M)
60. Write the structural formula of main organic product formed
49. How will you bring about the following conversion? when ethyl acetate is treated with double the molar quantity of
methyl magnesium bromide and the reaction mixture is poured
nd

“Ethanoic acid to a mixture of methanoic acid and diphenyl


Re

ketone.” (1990, 2M)


into water. (1981, 1/2 M)

61. Write the chemical equation to show what happens when,


Fi

50. Give reasons for :


“Carbon-oxygen bond lengths in formic acid are 1.23 Å “Ethyl acetate is treated with sodium ethoxide in ethanol and
the reaction mixture is acidified”. (1981, 2 M)

Answers
Topic 1 5. (b) 6. (c) 7. (a) 8. (a)
1. (b) 2. (c) 3. (d) 4. (d) 9. (b) 10. (c) 11. (b) 12. (a)
5. (b) 6. (d) 7. (c) 8. (d) 13. (c) 14. (b) 15. (b) 16. (a,c,d)
9. (d) 10. (c) 11. (d) 12. (a) 17. (b,d) 18. (a,c) 19. (d) 20. (c)
13. (d) 21. (a) 22. (c) 23. (b) 24. (a)
Topic 2 25. (d) 26. (d) 27. (b)
1. (d) 2. (d) 3. (a) 4. (b) 28. H2O and CO gas 29. False 30. False
31. (2)
Hints & Solutions
Topic 1 Carboxylic Acids NOTE

Ordinary carboxylic acid require soda-lime catalyst for
COOH
1. Heating of (CH2)4 (adipic acid) with a dehydrating agent, decarboxylation.
COOH ●
Final step of decarboxylation in the above shown mechanism
decarboxylates ( ¾CO2 ) to give a ketone (cyclopentanone), not involve tautomerism, therefore, for decarboxylation of b-keto
an anhydride. acid by above mechanism, the acid must contain an a-H].

COOH D 6. Structures of the various compounds are


O
COOH –CO2
–H2O OH O

w
Codes O 2N NO2
HN NH
P ® 1, Q ® 3, R ® 4, S ® 2
Thus, (a) is the correct choice.
O O

Flo
2. This problem is based on successive reduction, chlorination and NO2 barbituric acid
elimination reaction. To solve such problem, use the function of (B)
picric acid
the given reagents.

ree
(A)
(i) LiAlH 4 causes reduction
(ii) PCl 5 causes chlorination OH COOH

F
(iii) Alc. KOH causes elimination reaction HO
LiAlH
NO2 O CH3
CH3COOH ¾ ¾ ¾
¾4® CH3CH2OH

or
ur
(A )
HO OH
O

f
PCl 5 Alc.KOH
¾¾® CH3CH2Cl ¾¾¾® CH2 == CH2 ascorbic acid aspirin
(B ) - HCl (C )
ks
(C) (D)
Ethylene
Yo
PLAN NaHCO 3 s Na + + HCO-3 O
oo
3. O O
Heat * I2
HCO-3 is decomposed by acid releasing CO2 Ph—C—CH3
eB

7. NaOH
HCO-3 + H+ ¾® H2O + CO2 Ph * OH E
If acid is stronger than HCO-3 then CO2 is released. Phenol is less *
Ph—COONa + CHI 3
acidic and thus does not liberate CO2 with NaHCO3. F G
r
ou
ad

O
½½ NH3 SO3H SO3Na
4. CH3CH2 ¾ C ¾ OH ¾®
Y

(A ) 8. + NaHCO3
O
½½
nd

D + H2O + CO2­
Re

CH3CH2COONH4 ¾® CH3 ¾ CH2 ¾ C ¾ NH2


(B ) (C ) OH ONa
Fi

Br 2 , KOH + NaHCO3
¾¾¾¾¾¾¾® CH3 ¾ CH2 ¾ NH2
Hoffmann’ s bromamide
reaction O 2N O 2N
+ H2O + CO2­
5. It is a b-keto acid which undergo decarboxylation in very mild
condtion, i.e. on simple heating. This occur through a six O O-
member cyclic transition state as ½½ H ½
- H+
9. R ¾ C ¾ OH + O R ¢ ¾¾® R ¾ C — OH
OH ··
H ½
O O D Tautomerism OR ¢
–CO2
R C CH2
C C Reaction occur at planar sp2 carbon giving racemic mixture of
R CH2 O O
product.
Six membered cyclic R C CH3
transitionstate of a 10. C6H5COOH + SOCl 2 ¾® C6H5 ¾ COCl
b-keto acid
*
11. CH3 ¾ CH2 ¾ COOH + NaHCO3 ¾® CH3CH2COONa
*
+ H2 O + CO2
380 Carboxylic Acids and Their Derivatives

12. Ethanol is the weakest acid among these, hence it is most basic. The reaction proceed as

13. (d) Ph
Ph Ph Ph O
HO Me Pinacol
For P, i.e. + H2SO4 Pinacolone
HO OH Me
Ph OH + H2SO4 Me Me rearrangement Ph
Me
Me (Phenyl shift)
The correct match is 1 i.e., I2 , NaOH and 5 i.e., NaOBr
H Both
The reactions proceed as aO haloform
/N
I2 reactions
Ph Me Ph O
Pinacol
+ H2SO4
HO OH Pinacolone Me Me Ph O NaOBr
Ph Me rearrangement Ph

w
(Methyl shift) Me
OH
Ph
Both
X
H
aO

haloform
Ph
/N

Flo
reactions
I2

For S, i.e. Br H
Ph + AgNO3

ree
OH
NaOBr
Me
Ph O The correct match is 2, 3.

F
Me OH The reaction proceed as
Ph Ph

or
ur‘ X’ Br H

f
Ph H
Ph OH + AgNO3
+ HNO2
ks
For Q, i.e. Me
NH2 OH dibromination
Ph Me
Yo
rearrangement
oo
The correct match is 2 i.e. [Ag(NH3)2]OH and 3 i.e., Fehling’s Ph
solution.
eB

O
The reactions proceed as Me
Ph H Ph O Ph H
Deamination
+ HNO2
r

rearrangement Me Rest procedure is same as seen for Q above i.e., via oxidation.
H2N H
Ph Me OH
ou

Ph
ad

(Methyl shift) OH
tio
n ½ D
NaOH
Y

o lu 14. CH3CHO + HCHO ¾¾® H ¾ C — CH2 ¾ CHO ¾®


g'ss
lin ½ Aldol
h Both H
Fe oxidation
nd
Re

Reactions OH
[Ag(NH3)2]OH
(Tollens's HCN ½
CH2 == CH ¾ CHO ¾¾® CH2 == CH ¾ CH ¾ CN
Fi

O reagent)
Ph Cyanohydrin
Me
OH OH OH
Ph
‘X’ ½ H+ ½
CH2 == CH ¾ CH ¾ CN ¾® CH2 == CH ¾ CH ¾ COOH
Ph Ph H2 O

For R, i.e. HO + H2SO4


OH
Me Me
The correct match is 1, 5 again.
Carboxylic Acids and Their Derivatives 381

This leaving group ability (weak conjugate


Topic 2 Acid Derivatives base) corresponds directly to the reactivity order.
1. Given reaction involves acidic hydrolysis of esters followed by Hence, the correct reactivity order is:
the intramolecular cyclisation. The chemical equation is as follows:
O O
O
OH OH
½½
C2H5 ¾ C ¾ Cl > C2H5 O C2H5
CH2OH H2SO4(cat.) CH2OH Most reactive
CHCl3 +C2H5OH (D)
(C)
COEt C OH
O O
O O ½½ ½½
OH OH > C2H5 ¾ C ¾ OCH3 > CH3 C NH2
Least reactive
(B)

w
CH2 OH –H2O (A)
O Note The -I effect of ¾ Cl and + m effects of
C OH
O
O
O · · ½½
¾ N H2¾ O ¾ C ¾ C2H5 and ¾ OCH3 groups while

Flo
2. The major product of the given reaction is (d).
This reaction proceed via Friedel-Craft acylation. Here, ¾ Cl attached to ¾ C ¾ group to be reduced. Also add to the
group present on chlorobenzene is ortho and para-directing. It can ½½

ree
be easily understood by resonating structures of chlorobenzene. O
r r r group leaving ability.

F
Cl Cl Cl Cl Cl
4. In presence of strong base, acidic H can easily be removed that
s s
result in formation of anion. The resulting anion undergoes

or
intramolecular nucleophilic addition which on hydrolysis
s
ur followed by heating gives the required product.

f
The given reaction proceed as follows : More acidic
ks
O H H
O + CO2Et O
Yo
s r
C EtONa
oo
–EtOH CO2Et
O + AlCl3 – O s
O----Al + Cl3 O O
eB

O (Nucleophilic
O s addition)
Cl EtO
–H
O cidic b O
H
more a H 2O
CO2Et
r

s CO2Et
–OH
ou
ad

OH s
O
O
O
Y

O
C a, b-eliminatioin
H2O
–EtOH CO2Et
Cl C—O – + Cl –OHs
nd

COOH
Re

—AlC
l3 5. Alkaline hydrolysis of an ester (carboxylic acid derivative)
O
follows acyl S N2 mechanism.
Fi

3. All the given compounds are acid derivatives, thus contain


O s O O
carbonyl group in them. LiAlH4 reduces these compounds OH
C C + ROs C + ROH
through nucleophilic substitution via addition elimination Os
OR OH
(S NA E) reaction. The rate of reaction depends upon the following
factors : Rate of S N 2 mechanism depends on the polarity of C O
(i) Size of alkyl group. group of ¾COOR group. Electron withdrawing group
(ii) Steric hinderance around the > C == O group. (- R > - I ) increases the rate of S N2 reaction whereas electron
(iii) (+ ) ve charge on the C-atom of > C == O group. donating group (+ R > + I ) decreases the rate of S N 2 reaction.
The alkyl groups are more or less same in the given compounds. Here, the nature of functional groups attached para to the
Thus, the reactivity order of given compounds depends upon benzene ring are:
2nd and 3rd factor written above. The cumulative effect of
these two factors results to leaving group ability (LGA ) of the — NO2 > —Cl > — OCH3
(- R ) (- I ) (+ R )
substituents in the following order :
O So, the order of hydrolysis will be,
½½ - III > II > I > IV
Cl- > O- ¾ C ¾ CH3 >O ¾ CH3 > NH2 (- R ) (- I ) (+ R )
Good leaving Poor leaving
group group
382 Carboxylic Acids and Their Derivatives

6. The mechanism of the given reaction is as follows:


SN2
Å y
it is electrophilic substitution CH2 OH CH2Cl CH2CN CH2N—C
[CH2==OH]Cl , (Ar SE2): Haloformylation –H2O Ag CN
+
–HCl HCl (SN1) More covalent
[CH2==O + H Cl] (Ag—CººN )
(A) –AgCl (Minor) B (Major)
(Benzyl chloride) (Benzyl isocyanide)

Thus, both benzyl cyanide and benzyl isocyanide are the products of reaction but benzyl isocyanide being the major product gives the correct
option as (c).

7. Key Idea DIBAL-H is diisobutyl aluminium hydride, [(CH3 )2 CHCH2 ]2 AlH. It is a selective reducing agent. It reduces carboxylic acids,

w
carboxylic acid derivatives and nitriles into aldehydes. It is an electrophilic reducing agent.

The mechanism of the reaction is as follows:


H

Flo
(sp2) s
H 2O
Al (DIBAL-H) Al Al
(An electrophilie) R CH O
–NH3

ree
[Aldehyde]
r –DIBAL-H
H N
R C N
R C N Hydride donor
(Reducing agent) CH (An imine)

F
(A nucleophile)
[Nitrile]
R

or
So, R ¾ C H is the correct answer.
½½
ur
f
O
ks
8. PLAN T his problem can be solved by using the stability of radical obtained after frag mentation of peroxyester.
Yo
Allylic radical are more stable than alkyl radical, so when there is a possibility of formation of allyl radical, it will undergo fragmentation
oo
through formation of allyl radical. i.e. fragmentation produces stable radical.
On the basis of stability of radical, fragmentation can be done as
eB

Column I Column II Explanation

O
r
ou

O
ad

P. 1. C6H5H2C CH3
O C6H5 — CH2+ CO2+ CH3O
Y

O
O CH3
C6H5 — CH2+ CO2+ Ph—CH2 — C — CH3 Ph —CH2+ CH3—CO—CH3
nd

O CH3
Re

Q. 3. C6H5H2C O
CH2C6H5 CH3
Fi

O O
CH3
O CH3 C6H5 — CO2+ CH3—C—CH3 Ph + CH3—CO—Ph + CH3 + CO2
R. 4. C6H5 O
–CO2

C6H5 C6H5

O
C6H5 —CO2+ CH3O
O
S. 2. C6H5 O CH3 C6H5+ CO2
Carboxylic Acids and Their Derivatives 383

O O 16. (a) T undergoes an ester hydrolysis in hot aqueous alkali as


½½ ½½
9. R ¾ NH2 + CH3 ¾ C ¾ Cl ¾¾® R ¾ NH ¾ C ¾ CH3 O
(- HCl)
CH3O
O NaOH(aq) C ONa
Since, each ¾ COCH3 group displace one H atom in the hot HO
O H 3C Soluble in aqueous NaOH
T
½½
reaction of one mole of CH3 ¾ C ¾ Cl with one ¾ NH2 group, (b) LiAlH4 reduces ester to alcohol as
the molecular mass increases with 42 unit. “U ” No chiral carbon optically inactive.
Since the mass increases by (390 - 180) = 210, hence the (c) U on treatment with excess of acetic anhydride forms a
210 diester as
number of ¾ NH2 group is = 5. (d) U on treatment with CrO3 |H+ undergo oxidation to diacid
42
which gives effervescence with NaHCO3.

w
CHO COO – O
OH –
10. H+ OH NaHCO3
U + CrO3 CO2
CHO CH2OH COOH

Flo
H 3C
O ‘V’
17. For separation by differential extraction one of the component

ree
H+
O must form salt with the given base so that the salt will be
extracted in aqueous layer leaving other component in organic

F
layer.
O (a) Both phenol and benzoic acid forms salt with NaOH, hence
½½ this mixture can’t be separated.

or
POCl 3
11. C6H5 ¾ C ¾ NH2 ¾¾® C6H5 ¾ CN
ur (b) Benzoic acid forms salt with NaOH while benzyl alcohol
does not, hence the mixture can be separated using NaOH.

f
O OMgBr Also benzoic acid forms salt with NaHCO3 but benzyl
½½ ½
ks
CH3 MgBr alcohol does not, hence NaHCO3 can be used for
12. CH3 ¾ C ¾ OC2H5 ¾¾¾® CH3 ¾ C — CH3 separation.
excess
Yo
½ (c) Neither benzyl alcohol nor phenol forms salt with NaHCO3,
oo
CH3 mixture cannot be separated using NaHCO3.
(d) C6H5CH2COOH forms salt with NaOH, C6H5CH2OH does
eB

CH3
½ not, hence mixture can be separated using NaOH.
H 2O C6H5CH2COOH forms salt with NaHCO3. C6H5CH2OH
¾® CH3 ¾ C — OH
does not, hence mixture can be separated using NaHCO3.
½
r

CH3 O O
ou
ad

½½ -
½½
OH O 18. R ¾ C ¾ NH2 + OH + Br2 ¾® R ¾ C ¾ NHBr
Y

H+ ½ ½½ (A )
13. P ¾¾® CH3 ¾ C == CH2 r CH3 ¾ C ¾ CH3 O O
H2 O
NaOH ½½ ½
½ -
nd

¾¾® CHI3 ¯ R ¾ C ¾ NHBr + OH- ¾® R ¾ C ¾ N ¾ Br


Re

I2 Yellow (B )
H+ ¾® R ¾ N == C == O + Br -
Fi

Q ¾¾® CH3 ¾ CH== CH ¾ OH r CH3 CH2 OH (C )


H 2O
Fehling solution 19. p-hydroxy benzoic acid has higher boiling point than o-hydroxy
¾¾¾¾¾® Cu 2 O ¯ benzoic acid because former prefers intermolecular H-bonding
Red while the latter prefer intramolecular H-bonding.
H O
COCl CHO
H2 HOOC— —O C— —OH
14. Pd/BaSO4 ‘‘Rosenmund H—O
reduction’’ intermolecular H-bonding
OH
O
½½ C
15. CH3 ¾ C ¾ NH2 + Br2 + NaOH ¾® CH3NH2 “Hofmann’s O
bromamide reaction”.
H
O
intramolecular H-bonding
384 Carboxylic Acids and Their Derivatives

O
½½ COOH COOH COOH
20. Compounds with CH3 ¾ C ¾ or CH3 ¾ CH(OH)–group gives
haloform reaction but this reaction is given only by aldehydes, H—C H2 O + O H—C—OH OH—C—H
ketones and alcohols, so acetic acid does not give haloform +
reaction. However acetic acid has three a-H, therefore, C—H dilute alkaline HO—C—H H—C—OH
KMnO4
statement I is true but statement II is false.
COOH COOH COOH
Passage-1
(fumaric acid) (T) d (+) (U) l (–)
OH
(I) CH3MgBr(ex) T and U (in 1 : 1 molar ratio) form optically inactive (racemic
OEt+ (ii) H2O mixture) due to external compensation.

w
Q
24. PLAN Ni / H2 reduces (C == C) bond.
O H+/0°C
Benzene undergoes Friedel-Crafts reaction Zn-Hg/HCl reduces
carbonyl group (Clemmensen reduction)

Flo
CH3COCl
AlCl3 O
R(F.C. Alkylation) CHCOOH CH2COOH CH2C
D

ree
Ni/H2 O
O
S(F.C. Acylation) HOOCHC CH2COOH CH2C
O

F
22. (a) succinic anhydride (V)

22. (c) O

or
23. PLAN A lkenes decolourise Br2 water
ur O
anhydrous C

f
dil× KMnO4 CH2—C AlCl3 CH2
c is -isomer ¾¾¾¾® M e s o isomers by s y n addition + O
ks
dil× KMnO4 CH2—C CH2COOH
t r a n s -isomer ¾¾¾¾® d (+ ) and l(- ) isomers by s y n
Yo
oo
a d d it io n t h u s , r a c e m ic m ix t u r e . O
Formation of anhydride from dicarboxylic acid indicates Å
eB

c is -isomer. Zn–Hg/HCl H3PO4


P and Q are isomers of dicarboxylic acids. Clemmensen
2Br water HO
P , Q ¾¾¾¾® decolourised O
r

O
P and Q have ( C ==C ) bond (W)
ou
ad

D
P ¾® anhydride O
Y

Thus, P is cis-isomer.
25. Cl— —C—NH2 + KOH + Br2
CHCOOH Br2 Water Br—CHCOOH CHCO
D
nd

O
Re

O
(i)
CHCOOH Br—CHCOOH CHCO
Cl— —C—NHBr
Fi

COOH COOH
an intermediate (ii)
H—C H 2O + O H—C—OH 26. Rearrangement of (iii) to (iv) is the rate determining step :
O
H—C dilute alkaline H—C—OH
KMnO4 Slow Cl—
Cl— —C – —N==C==O + Br–
COOH COOH
N—Br
(P) (S)
(iii) (iv)
(maleic acid) Optically inactive due to
internal compensation of
rotation (meso-isomer) 27. The rate determining step of Hofmann’s bromamide reaction is
unimolecular rearrangement of bromamide anion (iii) and no
cross-products are formed when mixture of amides are taken.
Carboxylic Acids and Their Derivatives 385

15
KOH
33. Compound A of molecular formula C9 H7 O2Cl exist in keto and
—CONH2 + —CONH2 predominantly in enolic form B. Hence, A must be a carbonyl
Br2
compound which contain a-H. Enolic forms of B predominates
D (i) 15 because of presence of intramolecular H-bonding.
—NH2 + —NH2 Cl Cl Cl
[O] tautomerism
D
D
28. HCOOH + H2SO4 ¾® H2O + CO ­ COOH
conc. O O
29. Propanoic acid has higher boiling point than n-butanol because H H
of more exhaustive H-bonding in former case. CHO O

w
30. Saponification is hydrolysis of ester in presence of dilute base 34. The two stereoisomers of 2-phenyl propanoic acid in the
rather in presence of dilute acid. racemic mixture are :
31. PLAN R eactant is cyclic anhydride and chang es to dicarboxylic acid *

Flo
COOH COO CH(CH3 )Et
on hydrolysis. ½ * ½
CH3 C H(OH)Et
Also there is decarboxylation on heating if there is keto group H ¾ C — Ph ¾¾¾¾¾¾® H ¾ C — Ph

ree
w.r.t ¾ COOH group. Ozonolysis cleaves (C== C) bond and ½ ½
H2O2 oxidises ¾ CHO to ¾ COOH group. CH3 CH3
O O O O (A )

F
*
(i) H3O+ OH COOH COOCH(CH3 )Et
O ½ * ½

or
CH3C H(OH)Et
ur OH
and Ph ¾ C — H ¾¾¾¾¾® Ph ¾ C — H
O ½ ½

f
O O O
CH3 CH3
Anhydride
ks
O O (B )
Yo
D decrbo- Here A and B are diastereomers.
oo
xylation O3 OHC O
OHC (i) Mg/ether ½½
eB

35. H2C == CH ¾ Br ¾¾¾¾® H2C == CH ¾ C ¾ OH


(ii) X *
O O (iii) H3O+ Y
O
LiAlH4 *
¾¾® H2 C==CH ¾ CH2 OH
r

H2O2 HOOC
ou
ad

Z
HOOC O O
½½ 18 ½½
Y

H3O+ 18
O 36. CH3 ¾ C ¾ OC2H5 ¾¾® CH3 ¾ C ¾ OH + C2H5 ¾ OH
(P)
(A ) (B )
Thus, number of ¾COOH groups in P = 2.
nd
Re

CH2Cl CH2CN O
O
Fi

KCN C2H5ONa/EtOH C OC2H5 COOC2H5 OC2H5


32. NaOEt –
DMF C6H5CHO 37. CH2 CH
Br
C OC2H5 COOC2H5
(A)

OH O
C6H5 C6 H 5
H3O+/D COOH
C6H5—CH—CH—C6H5 C==C (i) OH–
O CH3 COOC2H5 (ii) H+
HOOC H
CN (C) H5C2O—C—CH—CH—COOC2H5
(B) COOH
C6H5 C6 H 5 O
SOCl2 ½½
C==C (CH3 CO) 2 O
CH3NH2 38. CH3CH2NH2 ¾¾¾¾® CH3CH2NH ¾ C ¾ CH3
CH3HN—C H Heat

O (D )
386 Carboxylic Acids and Their Derivatives

O CH3 OH
½ O
½½ MeMgBr /H2 O ½½ ½
39. H ¾ C ¾ O ¾ CH — CH3 ¾¾¾¾¾® CH3 ¾ CH ¾ OH (i) CH3 MgBr
excess 44. C6H5CH2 ¾ C ¾ CH3 ¾¾¾¾® C H ¾ CH ¾ C — CH3
A ½ (ii) H+
6 5 2
CH3 ½
B
CH3

NaOCl
45. (i) PCl 5 + SO2 ¾® POCl 3 + SOCl 2
¾¾¾® CH3COOH + CHCl 3 B A
– (ii) SOCl 2 + CH3COOH ¾® CH3COCl + SO2 + HCl
CH3 CH2
EtO – (COOEt)2 (A )
40. (i) O
½½
NO2 NO2 (iii) 2CH3COCl + Cd(CH3 )2 ¾® 2CH3 ¾ C ¾ CH3 + CdCl 2

w
O SO3H SO3H OH
CH2—C—COOEt
fuming NaOH
46. (i)

Flo
H2SO4 fusion
NO2 SO3H OH
A

ree
CONH2 CN COOH
HO
O O
COOH CH2 H+
(ii) + P 2O 5 H+

F
(ii)
COOH CH2 H2O
HO O O

or
B
O O
ur
CH3 O
½ PCl 5 ½½

f
½½ ½½ NH3
NaOH 47. C6H5COOH ¾¾® C6H5COCl ¾¾® C6H5 ¾ C ¾ NH2
(iii) CH3 ¾ C ¾ C ¾ C6H5 ¾¾® C6H5 ¾ C ¾ COOH
ks
H2 O A B
½
OH P2O5
Yo
H2 /Ni
¾¾® C6 H5 ¾ CN ¾¾® C6 H5 CH2 NH2
oo
C
C
41. B is C3H5COOH and A is C3H5 ¾ C ºº C ¾ C3H5
eB

48. Electron withdrawing inductive effect increases acid strength


Also A on ozonolysis gives B indicates that there is no olefinic while electron donating inductive effect decreases acid strength.
bond in C3H5-unit of A and it is cyclopropyl group.
+ I-effect
r

Þ A= —C==C—
ou

H3C ¾ CH ¾ COOH < CH3CH2COOH < CH3COOH


ad

½
—COOH CH3
Y

B=
+ I-effect + I-effect
and C= —Br + I-effect
nd
Re

< C H2 ¾ CH2COOH < C H2 ¾ COOH


red-P ½ ½
CH3CH2COOH ¾¾¾® CH3 ¾ CH ¾ COOH
Fi

42. Cl Cl
Br 2 ½ - I-effect - I-effect
A Br
(HVZ reaction) (greater distance)
O
(i) alc.KOH
¾¾¾¾® CH2 ==CH ¾ COOH PCl 5 ½½ C6 H6
(ii) H+ B 49. CH3 ¾ COOH ¾® CH3COCl ¾¾® C6H5 ¾ C ¾ CH3
AlCl 3
43. A b-keto acid undergo very fast decarboxylation OH C6H5
OH C6 H5 MgBr
½ ½
H+
H ¾¾¾¾® C6H5 ¾ C — CH3 ¾® C6H5 ¾ C==CH2
O O H2 O D
– CO2 ½
C6H5—C==CH2 C6H 5
C enol
O O O
C6H5
O3 /H2 O2 ½½
C6H5—C==CH3 ¾¾¾® C6H5 ¾ C ¾ C6H5 + H ¾ COOH
keto
Carboxylic Acids and Their Derivatives 387

50. Both formic acid and sodium formate exhibit the phenomenon Hydrolysis is an example of nucleophilic acyl substitution,
of resonance as : hence the reactivity towards hydrolysis is :
O O- CH3CONH2 < CH3COOC2H5 < (CH3CO)2 O < CH3COCl
½½ · · ½ +
H ¾ C ¾ O ¾ H ¬® H ¾ C == O ¾ H O O
·· ·· ½½ ½½
O O- 56. CH3 ¾ C ¾ OH < H ¾ C ¾ OH
does not affect acidity.
½½ ½ +I -effect
H ¾ C ¾ O- ¬® H ¾ C == O decreases acid-strength

In formic acid, the extent of delocalisation is less compared to NH4 OH


57. CH3CH2CHO + AgNO3 ¾¾® CH3CH2COOH
sodium formate because of charge separation in the former case.
P2O5
Due to this reason, the bond length between carbon and sp3 ¾¾® (CH3 CH2 CO)2 O
D

w
oxygen in formic acid is slightly greater than the same between
carbon and sp2 oxygen. In formate ion, there is no separation of Red-P
58. CH3 ¾ COOH + Cl 2 ¾¾® CH 2 ¾ COOH (HVZ reaction)
charge and both the resonance structures are equivalent giving ½
equal bond length of both carbon oxygen bonds. Cl

Flo
NaOH
51. CH3CONH2 + Br2 ¾¾® CH3NH2 + Na 2CO3 For this reaction to occur, presence of a a-H is essential
Hoffman bromamide requirement. Formic acid does not has any a-H, fails in HVZ

ree
reaction reaction.

52. X is an ester and both its acid and alcohol fragments have same OH O CH3

F
OH–
number of carbons. Hence, X is : 59. + (CH3CO)2O
+ O
H
CH3CH2COOCH2CH2CH3 ¾¾®

or
CH3CH2COOH
X
ur
H2 O Y O OMgBr
½½ ½

f
+ CH3CH2CH2OH 60. CH3 ¾ C ¾ OC2H5 + 2CH3MgBr ¾® CH3 ¾ C ¾ CH3
ks
[O]
Z ½
Z ¾¾® CH3CH2COOH CH3
Yo
oo
Y OH
O CH3 ½
H 2O
½½ ½ ¾¾® CH3 ¾ C — CH3
eB

MeMgCl H+ /H2 O
53. H ¾ C ¾ O ¾ CH — CH3 ¾¾¾® ¾¾¾®
A
excess ½
CH3
OH
r

½ C H O- -
61. CH3COOC2H5 ¾¾®
2 5
ou

CH3 ¾ C — CH3 CH2COOC2H5


ad

½
O
Y

H O
½½
B ½½
CH3 ¾ C ¾ OC2 H2
B + NaOCl ¾® CHCl 3 + CH3COOH ¾¾¾¾¾¾® CH3 ¾ C ¾ CH2 ¾ COO2H5
nd

Claisen condensation
Re

O
C2H5 OH ½½ O
54. (CH3CO)2 O ¾¾¾® CH3COOH + CH3 ¾ C ¾ OC2H5
Fi

H+ ½½
¾¾¾® C2H5OH + CH3 ¾ C ¾ CH2COOH
55. Among acid derivatives, the reactivity towards nucleophilic H2 O
acyl substitution is in the order of :
Amide < Ester < Anhydride < Acid chloride

Download Chapter Test


http://tinyurl.com/y2y69czp or
27
Aliphatic Compounds
Containing Nitrogen

w
Flo
Objective Questions I (Only one correct option) 5. Which of the following amines can be prepared by Gabriel

ree
1. The major product of the following reaction is phthalimide reaction? (2019 Main, 8 April I)
(a) n - butylamine
OH
(b) triethylamine
½

F
Ethyl formate (1 equiv.) (c)
CH3 CH CH2CH2NH2 ¾¾¾¾¾¾¾¾® t-butylamine
Triethylamine
(2019 Main, 10 April I) (d) neo -pentylamine
OH

or
½
ur 6. In the following compounds, the decreasing order of basic
(a) CH3 CHCH2CH2NHCHO (b) CH3CH== CH ¾ CH2NH2 strength will be (2019 Main, 8 April I)

f
O (a) C2H5NH2 > NH3 > (C2H5 )2 NH
OH
ks
(b) (C2H5 )2 NH > NH3 > C2H5NH2
½
Yo
(c) CH3 ¾ CH ¾ CH == CH2 (d) O H (c) (C2H5 )2 NH > C2H5NH2 > NH3
oo
(d) NH3 > C2H5NH2 > (C2H5 )2 NH
CH3CHCH2CH2NH2
7. The major product of the following reaction is
eB

2. Ethylamine (C2H5 NH2) can be obtained from (2019 Main, 12 Jan II)
N-ethylphthalimide on treatment with (2019 Main, 10 April I) H3 C O (i) NaNO2/H+
NH2
(a) NaBH4 (b) NH2NH2 (c) H2O (d) CaH2 (ii) CrO3/H+
r

O
(iii) H2SO4 (conc.), D
ou
ad

3. Hinsberg’s reagent is (2019 Main, 9 April II)


(a) SOCl 2 (b) C6H5COCl (c) C6H5SO2Cl (d) (COCl)2 O O
Y

4. The major products A and B for the following reactions are, (a) CH3 O (b) HO
respectively (2019 Main, 9 April II)
nd

O O
Re

I H2/Pd
HO CH3 O
KCN
[A] [B] (c) (d)
Fi

DMSO
O O O
O OH
CN CH2NH2 8. A compound ‘X’ on treatment with Br2 / NaOH, provided
(a) ; C3 H9N, which gives positive carbylamine test. Compound ‘X’
is (2019 Main, 11 Jan II)
HO CN HO CH2 NH2
I H
(a) CH3COCH2NHCH3
(b) ; (b) CH3CH2CH2CONH2
(c) CH3CON(CH3 )2
O O (d) CH3CH2COCH2NH2
CN CH2NH2
(c) ; 9. The major product of the following reaction is
(2019 Main, 10 Jan II)
HO CN HO CH2 NH2 O
I I CH3N NaBH4
(d) ;
Aliphatic Compounds Containing Nitrogen 389

OH COCH3
(a) CH3N OH OH

(b) CH3NH (a) (b)


NH2 NHCOCH3
OH
OCOCH3
(c) CH3NH
OH
(c) (d)
(d) CH3N NHCOCH3 NH2

10. The correct structure of product ‘P’ in the following reaction is 13. Major product of the following reaction is
NEt 3

w
Cl
Asn -Ser + (CH3CO)2 O ¾® P
(Excess) Cl NH2
(2019 Main, 10 Jan I) (i) Et3N
+ H 2N
O O O (ii) Free radical
polymerisation

Flo
O NH2 O
H (2019 Main, 9 Jan I)
N Cl O O
(a) H3C N OH Cl

ree
H (a) n (b)
O NH2
OCOCH3 N
O n
O
H

F
OCOCH3
O O HN NH2
N Ol
(b) H3C N H OH Cl Cl

or
H
O
ur NHCOCH3 (c) (d)
n n

f
O
OCOCH3 HN O
ks
O O NH2 NH2
H N
Yo
N O H
O
oo
(c) H3C N OH
H NH2
O 14. The increasing order of basicity of the following compounds is
eB

O NH2
I. NH
O II.
O NHCOCH3 NH2
r

O IV.
III.
ou

NH NHCH3
ad

(d) H3C N NH
H OH (2018 Main)
O OCOCH3 (a) (I) < (II) < (III) < (IV)
Y

(b) (II) < (I) < (III) < (IV)


11. The major product formed in the reaction given below will be (c) (II) < (I) < (IV) < (III)
nd
Re

NH2 (d) (IV) < (II) < (I) < (III)


NaNO2
15. The order of basicity among the following compounds is
Fi

Aq. HCI, 0-5°C


(2017 Adv.)
(2019 Main, 10 Jan I) O NH2

(a) NO2 (b) OH H3C NH2 N NH HN N H 2N NH

(I) (II) (III) (IV)


NO2 OH
(c) (d) (a) II > I > IV > III (b) I > IV > III > I
(c) IV > I > II > I (d) IV > I > II > III
16. In the Hofmann-bromamide degradation reaction, the number of
12. The major product obtained in the following reaction is moles of NaOH and Br2 used per mole of amine produced are
(2016 Main)
OH
(a) four moles of NaOH and two moles of Br2
(CH3CO)2O/pyridine (1 eqv.) (b) two moles of NaOH and two moles of Br2
NH2 room temperature (c) four moles of NaOH and one mole of Br2
(2019 Main, 9 Jan II) (d) one mole of NaOH and one mole of Br2
390 Aliphatic Compounds Containing Nitrogen

17. Considering the basic strength of amines in aqueous solution, 23. The correct order of basicities of the following compounds is
which one has the smallest pK b value? (2014 Main) NH
(a) (CH3 )2 NH (b) CH3NH2 H3C ¾ C , CH3 ¾ CH2 ¾ NH2 ,
(c) (CH3 )3 N (d) C6H5NH2 NH2 2
1
18. On heating an aliphatic primary amine with chloroform and
ethanolic potassium hydroxide, the organic compound formed is O
½½
(a) an alkanol (b) an alkanediol (2014 Main) (CH3 )2 NH, H3C ¾ C ¾ NH2
(c) an alkyl cyanide (d) an alkyl isocyanide 4 (2001, 1M)
3
19. The gas leaked from a storage tank of the Union Carbide plant in (a) 2 > 1 > 3 > 4 (b) 1 > 3 > 2 > 4 (c) 3 > 1 > 2 > 4 (d) 1 > 2 > 3 > 4
Bhopal gas tragedy was (2013 Main) 24. A positive carbylamine test is given by (1999, 2M)
(a) methyl isocyanate (a) N, N-dimethylaniline (b) 2, 4-dimethylaniline

w
(b) methylamine
(c) N-methyl-o-methylaniline (d) p-methylbenzylamine
(c) ammonia
(d) phosgene 25. p-chloroaniline and anilinium hydrochloride can be
distinguished by (1998, 2M)
20. The major product of the following reaction is (2011)

Flo
(a) Sandmeyer reaction (b) NaHCO3
O
(c) AgNO3 (d) Carbylamine test
C

ree
(i) KOH 26. Carbylamine test is performed in alc. KOH by heating a
NH mixture of (1984, 1M)
C (ii) Br— —CH2Cl (a) chloroform and silver powder

F
(b) trihalogenated methane and a primary amine
O (c) an alkyl halide and a primary amine

or
O (d) an alkyl cyanide and a primary amine
C
ur
27. Acetamide is treated separately with the following reagents.

f
(a) N—CH2— —Br Which of these would give methylamine? (1983, 1M)
ks
C (a) PCl 5 (b) Sodalime
(c) NaOH + Br2 (d) Hot, conc. H2SO4
Yo
O
oo
O 28. The compound which on reaction with aqueous nitrous acid at
C
low temperature produces an oily nitrosamine, is
eB

(b) (a) methylamine (b) ethylamine (1981, 1M)


N— —CH2Cl
(c) diethylamine (d) triethylamine
C
r

O Objective Question II
ou
ad

O (Only one more than one correct option)


Y

C 29. The major product of the reaction is (2015, Adv.)


(c) N H 3C CO2H
NaNO2, aq. HCl
C
nd

0°C
Re

CH3 NH2
O —CH2— —Br
Fi

O H3C NH2 H3 C CO2H


(a) (b)
C
(d) N CH3 OH CH3 OH

C
H3 C CO2H H3C NH2
O— —CH2Cl (c) (d)
CH3 OH CH3 OH

21. CH3NH2 + CHCl3 + KOH ® Nitrogen containing compound +


30. A positive carbylamine test is given by (1999, 2M)
KCl + H2O. Nitrogen containing compound is (2006)
(a) N, N-dimethyl aniline (b) 2, 4-dimethyl aniline
(a) CH3CN (b) CH3NHCH3
- + + - (c) N-methyl-o-methyl aniline (d) p-methyl benzyl amine
(c) CH3 ¾ N ºº C (d) CH3 ¾ N ºº C
22. Benzamide on treatment with POCl3 gives (2004) Fill in the Blanks
(a) aniline (b) benzonitrile 31. (CH3OH+2 ) is …… acidic than (CH3NH3+ ). (1997 C, 1M)
(c) chlorobenzene (d) benzyl amine
Aliphatic Compounds Containing Nitrogen 391

Match the Columns 38. Arrange the following in increasing order of basic strength:
methylamine, dimethylamine, aniline, N-methylaniline.
32. Match each of the compounds in Column I with its characteristic (1988, 1M)
reaction(s) in Column II. (2016, Adv.)
39. Give a chemical test and the reagent used to distinguish between
Column I Column II the following : “Ethylamine and diethylamine”. (1988, 1M)
(A) CH3 CH2 CH2 CN (p) Reduction with Pd - C/ H2 40. For nitromethane molecule, write structures
(B) CH3 CH2 OCOCH3 (q) Reduction with SnCl 2 / HCl (i) showing significant resonance stabilisation
(ii) indicating tautomerism (1986, 1M + 1M = 2M)
(C) CH3 CH == CHCH2 OH (r) Development of foul smell
on treatment with chloroform 41. State the equation for the preparation of following compounds :
and alcoholic KOH. n-propyl amine from ethyl chloride. (1982, 2 ´ 1M = 2M)

(D) CH3 CH 2 CH2 CH2 NH2 (s) Reduction with

w
diisobutylaluminium Passage Based Questions
hydride (DIBAL-H) Treatment of compound O with KMnO 4 / H+ gave P, which on
(t) Alkaline hydrolysis heating with ammonia gave Q. The compound Q on treatment with

Flo
Br2 / NaOH produced R. On strong heating, Q gave S, which on
Subjective Questions further treatment with ethyl 2-bromopropanoate in the presence of

ree
KOH followed by acidification, gave a compound T.4 (2016 Adv.)
NaNO2 , HCl
33. C5H13N ¾¾¾¾® Y(tertiary alcohol + other products)
– N2
optically active

F
X
Find X and Y. Is Y optically active ? Write the intermediate steps

or
(2005, 4M)
ur
34. Give reasons for the following in one or two sentences.

f
Dimethylamine is a stronger base than trimethylamine. (O)
(1998, 2M)
ks
42. The compound R is
35. Following reaction gives two products. Write the structures of O
Yo
oo
the products. NH2
Br
(CH CO) O, heat (a) (b)
CH3CH2NH2 ¾¾¾¾¾¾®
3 2
eB

(1998, 2M) Br
NH2
36. Give the structure of A. ‘A (C3H9 N) reacts with benzenesulphonyl
O
chloride to give a solid, insoluble in alkali’. (1993, 1M)
O O
r

37. A basic volatile nitrogen compound gave a foul smelling gas


ou
ad

when treated with chloroform and alcoholic KOH. A 0.295 g NHBr


(c) (d) NBr
sample of the substance dissolved in aq. HCl and treated with
Y

NHBr
NaNO2 solution at 0°C, liberated a colourless, odourless gas
whose volume corresponds to 112 mL at STP. O O
nd

After evolution of the gas was complete, the aqueous solution


Re

was distilled to give an organic liquid which did not contain 43. The compound T is
nitrogen and which on warming with alkali and iodine gave a (a) glycine (b) alanine
Fi

yellow precipitate. Identify the original substance assuming that (c) valine (d) serine
it contains one N-atom per molecule. (1993, 4M)

Answers
1. (a) 2. (b) 3. (c) 4. (c) 5. (a) 6. (c) 7. (c) 8. (b)
9. (c) 10. (a) 11. (*) 12. (b) 13. (d) 14. (c) 15. (d) 16. (c)
17. (a) 18. (d) 19. (a) 20. (a) 21. (d) 22. (b) 23. (b) 24. (d)
25. (c) 26. (b) 27. (c) 28. (c) 29. (c) 30. (b,d) 31. (more)
32. A ® p, q, s, t; B ® p, s, t; C ® p; D ® r 42. (a) 43. (b)
Hints & Solutions
1. The mechanism of the given reaction is as follows: 5. n-butylamine (CH3CH2CH2CH2NH2 ) can be prepared by
O Gabriel phthalimide reaction. This method produces only
OH
H—C—OEt
primary amines without the traces of secondary or tertiary
CH3—CH—CH2CH2—NH2 amines. In this method, phthalimide is treated with ethanolic
Et3N
[–NH2 is stronger —EtOs
KOH, it forms potassium salt of phthalimide which on heating
nucleophile than
OH with alkyl halide followed by alkaline hydrolysis forms
—OH]
r corresponding primary amines.
CH3—CH—CH2CH2—NH—CH=
=O CO CO
NH+ Alc. KOH –H O N –K +
H 2

w
CO CO
OH EtOs
nBu—X

CH3—CH—CH2CH2—NH—CHO
COO–Na+ CO

Flo
NaOH(aq.)
The basic mechanism of the reaction is acyl S N 2 because the + n Bu—NH2 N—R
nucleophile, CH3 (OH)CH2CH2 NH2 attacks the sp2 carbon of –
COO Na+ CO
1° amine
the ester (H ¾ CO2Et) and gets substituted. N-alkyl

ree
phthalimide
2. The reaction for the production of ethylamine from
N -ethylphthalimide can be takes place as follows: Triethylamine, t-butylamine and neo-pentylamine cannot be

F
prepared by Gabriel phthalimide reaction.
O H HN
6. Basic strength can be compared by the reaction of an

or
N—C2H5
ur
Hydrazinolysis
alkanamine and ammonia with proton.
H H

f
NH2—NH2
(Hydrazine) | |
H HN R—N + H+ R—N+—H
O
ks
| |
(N-ethylphthalimide) H H
Yo
O
oo
H H
NH | |
| + C2H5—NH2 H—N + H+ H—N+—H
eB

NH | |
Ethylamine
(An aliphatic
H H
O 1°-amine)
Basicity of an amine in aqueous solution depends upon the
solubility of ammonium cation formed by accepting proton
r

This reaction is the second step of Gabriel phthalimide synthesis


from water. The stability of ammonium cation depends upon the
ou

for the preparation of aliphatic 1°-amines and amino acid. In this


ad

step concentrated alkali can also be used in place of hydrazine. following factors.
(a) + I effect (b) Steric effect
Y

3. Hinsberg’s reagent is C6H5SO2Cl (benzene sulphonyl chloride). (c) Solvation effect


This reagent is used to distinguish between primary, secondary In first case, on increasing the size of alkyl group + I effect
nd

and tertiary amines.


Re

increases and the positive charge of ammonium cation gets


4. O O dispersed more easily, Therefore, order of basicity is 2º amine >
1º amine > ammonia. In second case, substituted ammonium
Fi

I CN
Step I
KCN cation is also stabilised by solvation with water molecules.
DMSO Greater the size of ion, lesser will be the solvation and less
stabilised is the ion.
[A ]

Step II H2/Pd
OH2

O H
| C2H5 H----OH2
CH2NH2 +
C2H2—N+—H----OH2 > N
| C2H5 H----OH2
H
2° amine
[B] OH2
1° amine
Step I involves the nucleophilic substitution reaction in which I
(Iodine) is substituted by
Considering both steric and solvation effect, it can be concluded
¾ CN group. In step II, H2/Pd reagent is used for reduction
that greater the number of H-atoms on the N-atom, greater will
process. Here, ¾ CN group reduces itself to ¾CH2NH2. be the H-bonding and hence more stable is the ammonium
Aliphatic Compounds Containing Nitrogen 393

cation. But in case of ¾C2H5 group, + I effect predominates or CH3 CH(CONH2 )CH3 (b).
over H-bonding. Therefore, order is 2º amine > 1° amine > Carbylamine test given by (a) and (b)
ammonia i.e. (C2H5 )2 NH > C2H5NH2 > NH3. D
CH3 CH2 CH2 NH2 + CHCl 3 + 3KOH ¾¾ ®
7. Key Idea The reaction involves: CH3 (CH2 )2 NC + 3KCl + 3H2
A. Deamination in step (i) NH2
B. Oxidation in step (ii) ½
D
C. Hydroysis in step (iii) CH3 ¾ CH ¾ CH3 + CHCl 3 + 3KOH ¾¾ ®
The complete pathway of reactions is as follows: NC
H3C O ½
NH2
CH3 ¾ CH ¾ CH3 + 3KCl + 3H2
(i)
O

w
As (b) is not among the given options So, it is ruled out and
H3C O compound (X) is CH3CH2CH2CONH2.
NaNO2/H+
OH 9. NaBH4 is a selective reducing agent. It can reduce C==O
Deamination O
group into alcohol, N-methyl imine group, Me ¾ N == CH ¾

Flo
H 3C O into 2°-amine but cannot reduce an isolated C == C.
(ii) OH O OH

ree
O N
NaBH4
O NH
H3C H3C H3C
CrO3/H+ O

F
OH N-methyl isolated Keto
Oxidation C C group
O imine

or
10 Acetylation by Ac2O/Et3 N is possible with ¾ NH2 (1°-amine)
HO
H O
ur
H 3C and ¾ OH (alcohol) groups only, but not with ‘¾ NH2’ part of

f
O Conc. H2SO4/D
(iii) Cyclisation O
½½
ks
O
H OH HO
¾ C ¾ NH2 (amide) groups, because the lp of nitrogen in
Yo
Hydrolysis
oo
Intramolecular amide group is involved in resonance with C O part.
dehydration O Os
eB

O r
Final product C NH2 C NH2
Thus, option (c) is the correct answer. O
r

8. The molecular formula, C3H9 N refers to four structural isomers H2 N CH C NH CH COOH


ou
ad

of amines
CH2 C NH2 CH2 OH
(a) CH3CH2CH2NH2 (b) CH3–CH–CH3
Y

NH2 O
Asn Ser
nd

Primary amine
Re

(Asparagine) (Serine)
CH3 Ac2O or (CH3CO)2O (2 moles)
Fi

(c) CH3–NH–CH2CH3 (d) CH3–N–CH3 Et3N


–2 AcOH
Secondary amine Tertiary amine O O
Here, only a and b (primary amines) can be prepared from CH3 C NH CH C NH CH COOH
their respective amides by Hoffmann bromamide (Br2/NaOH)
method and can give positive carbylamine test. CH2 CONH2 CH2O C CH3

O
Br2
Þ CH3CH2CH2—C—NH2 CH3CH2CH2—NH2
NaOH
–Na2CO3 11. No option is the correct answer.
Amines in presence of NaNO2 + dil. HCl undergoes
H2N—C NH2
diazotisation. The diazotised product readily loses nitrogen gas
Br2/NaOH
Þ CH3—CH—CH3 CH3—CH—CH3 with the formation of carbocation. The resulting carbocation
–Na2CO3
rearranges itself to give the expanded ring.
So, ‘X ’ can be CH3 CH2 CH2 ¾ CONH2 (a)
394 Aliphatic Compounds Containing Nitrogen

NH2 NaNO2+dil. HCl Decrease in the delocalisation of p-bonds


0-5°C because of which it undergoes free radical
(Diazotisation) addition (chain growth) polymerisation
(1°-amine)
r s
N2Cl Cl Cl Free Cl
radical
–Cl – polymerisation
Aliphatic diazonium n
–N2 absorbed –H O
salt (unstable) by Et3N O C
4 NH
4 r
3 5 5
3 6 O Cl NH2
6 Ring expansion r
2 1 2
leads to give 1 NH2 O
–Hr H2O thermodynamically NH2
more stable O

w
carbocation O
OH NH2
NH2

14. Key Idea Among the given compounds the basic nature

Flo
(Major)
depends upon their tendency to donate electron pair.
NH
12. Rate of acetylation : ¾ NH2 > ¾ OH because N-bases are Among the given compounds in , Nitrogen is

ree
stronger than O-bases. Size of N-atom is larger than O-atom and sp2-hybridised. This marginally increases the electronegativity
at the same time, N atom is less electronegative than O-atom. of nitrogen which in turn decreases the electron donation
··

F
tendency of nitrogen. Thus making compound least basic.
So, N-atom of the ¾ N H2 group can donate its lone pair of
NH2
electrons (Lewis basicity) more easily than that of O atom of the Among the rest NH is totally different from others as in

or
··
¾ O H group.
ur this compound lone pair of one nitrogen are in conjugation with
p bond i.e. As a result of this conjugation the cation formed after

f
OH OH protonation becomes resonance stabilised
s
ks
CH3 CH3COO
Yo
Protonation
H 2N HN—COCH3 +
oo
C +
H2 N NH2 H2 N NH2
HN NH2
O—COCH3 H Equivalent resonance
eB

s
O—C—CH3
OH
This equivalent resonance in cation makes HN NH2 most
r

basic among all.


ou

CH3COOH +
ad

NHCOCH3 NH2
Absorbed Categorisation is very simple between rest two as
Y

by pyridine
N N CH3
(primary amine) is less basic than (secondary
H amine)
nd
Re

13. The analysis of both the substrates : Hence, the correct order is
Vinylic centre (II) < (I) < (IV) < (III) i.e. option (c) is correct.
Fi

Cl 15. IV is most basic as conjugate acid is stabilised by resonance of


(+R) two ¾ NH2.
r
NH NH2 NH2
O Cl H+
Acyl SN2 centre r
H2N NH2 H2 N NH2 H2N NH2
Acts as NH2
H2N nucleophile
(1°-amine) r
O
This N-atom can H2N NH2
not act as nucleophile
NH2 + R III is least basic as
Amide group H+ r
H N N H N N H
So, the reaction can take place as follows:
Destablised by -I-effect of sp2-carbons.
Aliphatic Compounds Containing Nitrogen 395

16. Hofmann-bromamide degradation reaction is given as: Bromine is not substituted in the above reaction as it is in
RCONH2 + 4NaOH + Br2 ¾® RNH2 resonance with benzene ring giving partial double bond
(1° amine) character to C—Br bond, hence difficult to break.

+

:
+ Na2 CO3 + 2NaBr + 2H2O

: :
ClCH2 Br: ClCH2 Br
Hence, four moles of NaOH and one mole of Br2 are used.
+ -
17. This problem can be solved by using the concept of effect of 21. CH3NH2 + CHCl 3 + KOH ¾® CH3 ¾ N ºº C + KCl + H2O
steric hindrance, hydration and H-bonding in basic strength of isocyanide
amines. Order of basic strength of aliphatic amine in aqueous O
solution is as follows (order of K b ) ½½
·· ·· ·· ·· POCl 3
(CH3 )2 NH > CH3 NH2 > (CH3 )3 N > C6H5 NH2 22. C6H5 ¾ C ¾ NH2 ¾¾® C6H5 ¾ C ºº N + H2O
POCl 3 brings about dehydration of primary amide.

w
As we know, pK b = - log K b
··
So, (CH3 )2 NH will have smallest pK b value. In case of phenyl 23. 4, (acetamide) is least basic because lone pair of nitrogen is
involved in delocalisation.
amine, N is attached to sp2-hybridised carbon, hence it has

Flo
-
highest pK b and least basic strength. O O
½½ · · ½ +
18. This reaction is an example of carbylamine test which includes CH3 ¾ C ¾ NH2 ¬® CH3 ¾ C == NH2

ree
conversion of amine to isocyanide.
·· C2 H 5OH + – ‘1’ is most basic due to formation of resonance stabilised
R ¾ NH2 + CHCl 3 ¾¾¾® R— N ºº C: conjugate acid.

F
1° amine KOH +
Alkyl isocyanide
NH NH2
NOTE The mechanism of this reaction included rearrangement of
H3C—C + H+ H3C—C

or
takes place.
ur
nitrene in which migration of alkyl group from carbon to nitrogen
NH2 NH2
(1)

f
19. Methyl isocyanate CH3 ¾ N == C == O (MIC) gas was leaked NH2
ks
from the storage tank of the union carbide plant in Bhopal gas H3C—C +
Yo
tragedy. NH2
oo
20. It is the first step of Gabriel's phthalimide synthesis. The
hydrogen bonded to nitrogen is sufficiently acidic due to two 3 (secondary amine) is stronger base than 2 (primary amine).
eB

a-carbonyls. Hence, overall order of basic strength is :


O O 1>3>2>4

r

N¾H + OH H 2O + N– 24. H3C— —CH2NH2 + CHCl3 KOH


ou
ad

O O p-methyl benzylamine
Y

(a primary amine) H3C— —CH2—NC


O– O
nd

(isocyanide)
Re

N N
Carbylamine test is not given by secondary or tertiary amine.
Fi

O O– + +
25. C6H5 NH3Cl - + AgNO3 ¾® C6H5 N H3NO3- + AgCl (s) ¯
The conjugate base formed above act as nucleophile in the anilinium
subsequent step of reaction. As shown above, the nucleophile hydrochloride
exist in three resonating form, one may think of oxygen being No such precipitate of AgCl(s) would be formed with
the donor atom in the nucleophilic attack. However, nitrogen act p-chloroaniline.
as donor as it is better donor than oxygen. KOH
26. R ¾ NH2 + CHCl 3 ¾¾® R ¾ NC ‘Carbylamine test’
O isocyanide
SN 2 (foul smell)
N–+ CH2 Br The above test is characteristic of primary amine and used to
distinguish primary amine from other amines.
Cl
O O O
½½ NaOH
N CH2 Br 27. CH3 ¾ C ¾ NH2 + Br2 ¾¾® CH3NH2
acetamide methylamine
O (Hofmann’s bromamide reaction)
396 Aliphatic Compounds Containing Nitrogen

28. A secondary amine, on treatment with aqueous nitrous acid at H C2H5


low temperature produces oily nitrosamine. ½ ½
33. CH3 ¾*C ¾ NH2 or CH3 ¾ CH ¾ CH2NH2
(C2H5 )2 NH + HNO2 ¾® (C2H5 )2 N ¾ NO + H2O ½ *
nitrosamine CH ¾ CH3
29. Reaction proceeds via diazonium salt with neighbouring group ½
participation. CH3
(X )
HO
¾R OH H H CH3
CH3 COOH R C O ½ NaNO2 ½
O ¾
HNO2 *
N¾¾N CH3 ¾ C ¾ NH2 ¾¾¾® CH3 ¾ CH2 ¾ C ¾ CH3
H ½ HCl ½
CH3 H NH2 N¾ N H
r
r
CH ¾ CH3 OH

w
Or ½ Y -optically inactive
R
R COOH COOH CH3
H –H
r ¾O
H
H2O Above reaction proceeds via carbocation intermediate, hence
H OH R
OH (I) rearrangement takes place.

Flo
Conformer of (I)
34. Conjugate acid of dimethylamine is more stable than conjugate
30. Carbylamine test is given by primary amines only. base of triethyl amine due to exhaustive H-bonding with water.

ree
CH3 CH3 NH2
O
N CH3 (CH3 CO) 2 O ½½
35. CH3CH2NH2 ¾¾¾¾® CH3CH2 ¾ NH ¾ C ¾ CH3

F
Heat
+ CH3COOH

or
ur CH3 36. A must be a secondary amine :
N,N-dimethylaniline 2,4-dimethylaniline CH3CH2NHCH3 + benzene sulphonylchloride ¾®

f
(3°-amine) (1°-amine)
CH3
ks
NHCH3 ½ NaOH
Yo
CH3 CH3CH2 ¾ N ¾ SO2 ¾ C6H5 ¾¾® Insoluble
oo
H 3C CH2NH2 37. Starting compound is a primary amine.
eB

R ¾ NH2 + CHCl 3 + KCl ¾® R ¾ NC


N-methyl-o-methylaniline p-methyl benzylamine foul smell
(2°-amine) (1°-amine)
Also, R ¾ NH2 + HNO2 ¾® R ¾ OH + N2
r

31. More : CH3NH2 is stronger base than CH3OH. 112


Moles of N2 = = 5 ´ 10- 3
ou
ad

22500
32. Column I Column II
Q One mole of N2 is obtained from 1.0 mole of R ¾ NH2, mole
Y

(A) CH3 CH2 CH2 CN : Gives amine with Pd-C/ H2 of R ¾ NH2 = 5 ´ 10- 3
Gives aldehyde with Þ 5 ´ 10- 3 mol R ¾ NH2 weigh = 0.295 g
nd
Re

SnCl 2 / HCl Gives amide with


0.295
diisobutyl- aluminium 1 mole of R ¾ NH2 will weigh = = 59 g
5 ´ 10- 3
Fi

hydride. Gives carboxylic


acid on alkaline hydrolysis. In R ¾ NH2, ¾ NH2 has molar mass = 16
(B) CH3 CH2 OCOCH3 : Reduced to alcohol with Þ R— has molar mass = 43
ester Pd - C/ H2 Reduced with Also, the alcohol R—OH gives iodoform test; it must has the
diisobutylaluminium hydride following skeleton :
into aldehyde. Undergo
alkaline hydrolysis. R ¢¾ CH ¾ CH3
½
(C) CH3 CH == CHCH2 OH Reduced to butanol when OH
: treated with Pd-C/ H2 . Þ R ¢— has molar mass = 15 (CH3 )

(D) CH3 CH 2 CH2 CH2 NH2 A primary amine, gives CH3


½
carbylamine test. Þ Original compound is CH3 ¾ CH ¾ NH2
Aliphatic Compounds Containing Nitrogen 397

38. Aniline > N-methylaniline < methylamine < dimethylamine. O


39. Carbylamine test given only by primary amine. NH2
NH2 Br2/NaOH
KOH
CH3CH2 ¾ NH2 + CHCl 3 ¾¾® CH3 ¾ CH2 ¾ NC NH2 D
primary amine foul smell NH2
Nitrosamine test is given only by secondary amine. O R
Q O
(CH3CH2 )2 NH + HNO2 ¾® (CH3CH2 )2 N ¾ NO
oily nitrosamine D
KOH
NH

O O O
+ + +
40. (i) CH3—N CH3—N CH3—N –
O

w
O– O O O S
O OH
+ + Br COOC2H5
(ii) CH3—N CH2==N : Tautomerism N—K
– –KBr
O–

Flo
O O
NaCN 4 LiAlH O
41. CH3CH2Cl ¾¾® CH3CH2CN ¾¾¾® CH3CH2CH2NH2

ree
SN 2 S1 N
42. (a) COOC2H5

F
43. Explanation O
S2

or
O
ur O
NH2

f
– OH NH3 H 3O + OH
MnO4 /H+
D +
D OH
ks
OH COOH
Yo
OP
oo
O (T)
(P)
eB

Download Chapter Test


http://tinyurl.com/y6nlww9e
r

or
ou
ad
Y
nd
Re
Fi
28
Benzene and Alkyl Benzene
Objective Questions I (Only one correct option)

w
1. The major product obtained in the given reaction is 4. The major product of the following reaction is
(2019 Main, 10 April II) (2019 Main, 9 April I)
CH3 AlCl3 CH2CH3
CH3 O CH2 Product
CH2 CH

Flo
(i) Alkaline KMnO4
Cl (ii) H3 O+
CH3

ree
COOH CH2CHO
(a) CH3 O

F
(a) (b)

(b) H3C O CH CH3


COCH3 CH2COOH

or
CH2 CH
ur
f
(c) H3C O (c) (d)
ks
Yo
5. The major product of the following reaction is
oo
CH3 (2019 Main, 9 April I)

(d) H3C O CH2


eB

1. KOH (alc.)
CH2 CH CH2
Cl 2. Free radical
polymerisation
Cl
2. The increasing order of the reactivity of the following CH3
r

compounds towards electrophilic aromatic substitution


ou

(a) (b)
ad

reaction is (2019 Main, 10 April I)


n
n
Y

Cl CH3 COCH3

Cl
nd
Re

CH3 Cl
(I) (II) (III)
Fi

(a) III < I < II (b) II < I < III (c) (d)
(c) III < II < I (d) I < III < II n
n
3. Increasing order of reactivity of the following compounds for
SN 1substitution is (2019 Main, 9 April II)
CH3
CH2 Cl OH
H 3C Cl OH
CH3
(A ) (B) 6. The increasing order of reactivity of the following
compounds towards aromatic electrophilic substitution
Cl Cl reaction is (2019 Main, 9 April I)
H3CO Cl OMe Me CN
(C) (D)
(a) (A) < (B) < (D) < (C)
(b) (B) < (C) < (D) < (A)
(c) (B) < (A) < (D) < (C)
(d) (B) < (C) < (A) < (D) A B C D
Benzene and Alkyl Benzene 399

(a) A < B < C < D (b) B < C < A < D 9. Among the following four aromatic compounds, which one
(c) D < A < C < B (d) D < B < A < C will have the lowest melting point? (2019 Main, 12 Jan I)

7. Polysubstitutiion is a major drawback in (2019 Main, 8 April II) O


(a) Friedel-Craft’s alkylation
OH
(b) Reimer-Tiemann reaction (a) (b)
OH
(c) Friedel-Craft’s acylation
(d) Acetylation of aniline O
CH3
8. The major product of the following reaction is OH
(2019 Main, 8 April II) O
CH3 (c) O (d)

w
(1) Cl2/hn CH3
(2) H2O,D

10. Which of the following compounds is not aromatic?


Cl (2019 Main, 9 Jan II)
CHCl2

Flo
CO2H (a) (b)
N
(a) (b) H

ree
(c) (d)
Cl Cl N

F
CH2 OH CHO 11. In the following sequence of reactions (2015 Main)
KMnO 4 SOCl 2 H 2 / Pd
Toluene ¾¾® A ¾¾® B ¾¾® C, the product C is

or
(c) (d)
ur BaSO 4

(a) C6 H5 COOH (b) C6 H5 CH3


Cl Cl
f
(c) C6 H5 CH3 OH (d) C6 H5 CHO
ks
Yo
12. Match the four starting materials given in Column I with the corresponding reaction schemes provided in Column II and select the
oo
correct answer using the code given below the lists.
eB

Column I Column II
A. H H p. Scheme I
(i) KMnO4 , HO- , heat (ii) H + , H 2O
r

(iii) SOCl 2 (iv) NH 3


? ¾¾¾¾¾¾¾¾¾¾¾¾
¾® C7 H6N2O3
ou
ad

B. OH q. Scheme II
Y

(i) Sn / HCl (ii) CH 3COCl (iii) Conc. H 2SO4


(iv) HNO3 (v) Dil. H 2 SO4 , heat (vi) HO-
? ¾ ¾ ¾ ¾ ¾ ¾ ¾ ¾ ¾ ¾ ¾ ¾ ¾ ¾¾® C6H6N2O2
nd
Re

OH
C. r. Scheme III
Fi

NO2
(i) Red hot iron, 873 K (ii) fuming HNO3 , H 2SO4 , heat
(iii) H 2S × NH 3 (iv) NaNO2 , H 2SO4 (v) hydrolysis
? ¾¾¾¾¾¾¾¾¾¾¾¾¾¾¾¾¾
¾® C6H5NO3

D. NO2 s. Scheme IV
(i) Conc. H 2SO4 ,60°C
(ii) Conc. HNO3 , conc. H 2SO4 (iii) Dil × H 2SO4 , heat
? ¾ ¾ ¾ ¾ ¾ ¾ ¾ ¾ ¾ ¾ ¾ ¾ ¾ ¾ ¾ ¾ ¾® C6H5NO4

CH3

Codes
A B C D A B C D
(a) p s q r (b) r p s q
(c) r s q p (d) s p r q (2014 Adv.)
400 Benzene and Alkyl Benzene

13. For which of the following molecule significant m ¹ 0? COOH OCH3


Cl CN OH SH (d)
HO H3C NO2

NO2 O

C
O NO2
Cl CN OH SH
I II III IV

(a) Only I (b) I and II (2014 Main)


HBr
(c) Only III (d) III and IV 15. In the reaction, —OCH3 the products are,

w
(2010)
14. The compounds P, Q and S
COOH OCH3 (a) Br— —OCH3 and H2 (b) —Br and CH3Br

Flo
H 3C H 3C (c) —Br and CH3OH (d) —OH and CH3Br
P Q O

ree
C 16. In the following reaction,
O

F
O
H3C
S N Conc. HNO3

or
ur H
Conc. H2SO4
X
were separately subjected to nitration using HNO3 / H2 SO4

f
mixture. The major product formed in each case The product X is (2007, 3M)
respectively, is
ks
(2010)
O
(a)
Yo
COOH OCH3 NO2
oo
N
(a) H
eB

HO H3C
O O 2N
NO2 NO2
O
(b)
C
r

O N
ou
ad

H
NO2
Y

O
(c)
COOH OCH3
N
nd

(b)
Re

H
HO NO2 H 3C NO2
Fi

NO2 O
O (d)
O 2N N
C
O H

NO2 AlCl
17. + Cl — CH2 CH2 — CH3 ¾¾¾®
3

COOH OCH3 (i) O / D


P ¾¾¾¾®
2
Q + Phenol
(c) (ii) H 3 O +
HO H3C NO2 P and Q are respectively (2006, 5M)

NO2 O NO2
(a) + CH3CH2CHO
C
O
(b) + CH3COCH3
Benzene and Alkyl Benzene 401

24. The compound that is most reactive towards electrophilic


substitution is (1985, 1M)
(c) + CH3COCH3
(a) toluene (b) benzene
(c) benzoic acid (d) nitrobenzene
25. Among the following, the compound that can be most readily
(d) + CH3CH2CHO sulphonated is (1982)
(a) benzene (b) nitrobenzene
18. O (c) toluene (d) chlorobenzene
HN
H3C CH3 Objective Questions II
Fe/Br2
(One or more than one correct option)

w
26. The reaction(s) leading to the formation of
product on monobromination of this compound is
(2004, 1M) 1,3,5-trimethylbenzene is (are) (2018 Adv.)

O O O

Flo
Conc. H2SO4
HN HN (a)
D
H 3C CH3 H3C CH3
(a) Heated iron tube
(b)

ree
(b) Me —— H
873 K

F
Br Br (1) Br2, NaOH
(2) H3O+
O (c)
HN (3) Sodalime, D

or
H3C
ur
CH3
O O

f
(c)
CHO
ks
Br
Yo
O (d) Zn/Hg, HCl
HN
oo

H 3C CH3 OHC CHO


eB

(d)
27. Among the following reactions(s), which gives(give)
Br tert-butyl benzene as the major product? (2016 Adv.)

19. Identify the correct order of reactivity in electrophilic


r

Br Cl
ou

substitution reactions of the following compounds (a) (b)


ad

Benzene (1), Toluene (2), Chlorobenzene (3) and NaOC2H5 AlCl3


Y

Nitrobenzene (4) (2002)


(a) 1 > 2 > 3 > 4 (b) 4 > 3 > 2 > 1 OH
(c) 2 > 1 > 3 > 4 (d) 2 > 3 > 1 > 4 (c) (d)
nd
Re

H2SO4 BF3,OEt2
20. A solution of (+) -2-chloro-2-phenylethane in toluene
racemises slowly in the presence of small amount of SbCl 5 , 28. The major product U in the following reaction is
Fi

(2015 Adv.)
due to the formation of (1999, 2M)
Radical
(a) carbanion (b) carbene CH2==CH¾ CH 3,H +
T
initiator, O2
U
High pressure, Heat
(c) free-radical (d) carbocation
21. Benzyl chloride (C6 H5 CH2 Cl) can be prepared from toluene H

by chlorination with (1998, 2M) O


O H3 C CH3
(a) SO2Cl 2 (b) SOCl 2 (c) Cl 2 (d) NaOCl CH3 O
22. Chlorination of toluene in the presence of light and heat O H
(a) (b)
followed by treatment with aqueous NaOH gives (1990, 1M)
(a) o-cresol (b) p-cresol H
(c) 2, 4-dihydroxy toluene (d) benzoic acid O
O CH2
23. The reaction of toluene with chlorine in the presence of ferric CH2 O H
chloride (FeCl 3 ) gives predominantly (1986, 1M) O
(c) (d)
(a) benzoyl chloride (b) m-chlorotoluene
(c) benzyl chloride (d) o- and p-chlorotoluene
402 Benzene and Alkyl Benzene

29. Which of the following molecules, in pure form, is/are F


unstable at room temperature? (2012)
(b) (i) NaOH (aq)
F– (liberated)
O2N
(a) (b)
O CH3
O
F
NaOH (aq)
(c) (d) (ii) F – (not liberated)
H3C
30. An aromatic molecule will (1999, 3M)
CH2NO2
(a) have 4n p-electrons

w
(b) have ( 4 n + 2 )p-electrons NO NO NO
(c) be planar NO2
(d) be cyclic Conc. HNO3
(c) (i) +

Flo
Conc. H2SO4
31. Which compound(s) out of the following is/are not aromatic?
(2019 Main, 11 Jan I)
NO2

ree
(a) (b)

Å NO2 NO2

F
Å

Conc. HNO3
(c) (d) (ii)

or
ur Conc. H2SO4
s NO2
32. Toluene, when treated with Br2 /Fe, gives p-bromotoluene as
f
ks
the major product because the CH3 group (1999, 3M)
Yo
oo
(a) is para-directing
Pd/C
(b) is meta-directing (d)
eB

(c) activates the ring by hyperconjugation 3.0 mol


H2
(d) deactivates the ring
33. A new carbon–carbon bond formation is possible in (1998)
r
ou

(a) Cannizzaro’s reaction (b) Friedel-Craft’s alkylation


ad

(c) Clemmensen reduction (d) Reimer-Tiemann reaction


Y

True or False
nd

34. An electron donating substituent in benzene orients the


Re

incoming electrophilic group to the meta-position. not


(1987, 1M)
Fi

35. In benzene, carbon uses all the three p-orbitals for


hybridisation. (1987, 1M)

Subjective Questions (2005)


36. Explain the followings : 37. 7-bromo-1, 3, 5-cycloheptatriene exists as ionic species in
CH3 aqueous solution while 5-bromo-1,3-cyclopentadiene
doesn't ionise even in presence of Ag + ( aq ). Explain. (2004)
Br C2H5OH (aq) acidic solution
(a) (i) CH3 38. What would be the major product in the following reaction?
(2000, Main, 1M)
CH3 O
C2H5OH (aq)
(ii) Br neutral
CH3 solution Br2/Fe(1eq)
N—
Benzene and Alkyl Benzene 403

39. Give reasons for the following : 48. Identify the major product in the following reactions :
(i) tert-butylbenzene does not give benzoic acid on treatment HNO3/H2SO4
with acidic KMnO4. (i) —COO—
mononitration
(ii) Normally, benzene gives electrophilic substitution
reaction rather than electrophilic addition reaction (ii) C6H5COOH + CH3MgI ? + ? (1993, 2M)
although it has double bond. (2000)
49. Write the structure of the major organic product expected
40. Show the steps to carry out the following transformations. from the following reaction
(i) Ethylbenzene ® benzene
AlCl3
(ii) Ethylbenzene ® 2-phenylpropionic acid + (CH3)2CHCH2Cl
(1992, 1M)
41. Write the structures of the products.
50. Give reasons in two or three sentences only for the following:

w
Br 2 /Fe
CH3 CONHC6 H5 ¾¾¾® (1998, 2M) Phenyl group is known to exert negative inductive effect, but
each phenyl ring in biphenyl (C6 H5 ¾ C6 H5 ) is more
42. Give reasons for the following in one or two sentences reactive than benzene towards electrophilic substitution.

Flo
“Nitrobenzene does not undergo Friedel-Craft’s alkylation.” (1992, 1M)
(1998, 2M)
51. Arrange the following in increasing order of reactivity

ree
43. Complete the following, giving the structures of the principal towards sulphonation with fuming sulphuric acid.
organic products. (1997)
Benzene, toluene, methoxy benzene, chlorobenzene.

F
Me (1988, 1M)

½ CH3 52. Answer the following with suitable equation wherever


½ Anhy. AlCl3 necessary

or
+ H3C—C—CH2Br
ur (i) How can you prepare benzene from lime ?
½

f
H (ii) How will you convert toluene to m-nitrobenzoic acid ?
(1987, 2M)
ks
44. Toluene reacts with bromine in the presence of the light to 53. Write down the main product of the following reaction
Yo
give benzyl bromide, while in presence of FeBr3 it gives
oo
CH CH COCl/AlCl
3 2 3
p-bromotoluene. Give explanation for the above Benzene ¾¾¾¾¾¾¾® (1985, 1M)
observation.
eB

(1996)
54. How many sigma and pi-bonds are present in a benzene
45. Predict the structures of the intermediates/products in the molecule ? (1985, 1M)
following reaction sequences (1996)
55. Write down the reaction involved in the preparation of
r

OMe O following using the reagents indicated against in parenthesis.


ou
ad

“Ethyl benzene from benzene.”


(i) AlCl3 Zn(Hg)/HCl
Y

+ O A B [C2 H5 OH, PCl 5 , anhyd. AlCl 3 ] (1984, 2M)


(ii) H3O+
56. Show with balanced equation what happens when the
nd

O ‘p-xylene is reacted with concentrated sulphuric acid and the


Re

O MeO resultant product is fused with KOH’. (1984, 1M)


Fi

H3PO4
B 57. Give reasons for the following in one or two sentences :
‘Although benzene is highly unsaturated, normally it does
not undergo addition reaction.’ (1983, 1M)
46. Predict the major product in the following reactions (1994)

Integer Type Questions


H SO
+ (CH3 )2 CHCH2 OH ¾¾®
2 4
(i) 58. Among the following, the number of aromatic compound(s)
is
C2H5 r
(i) Br2 , heat, light s r
(ii) ¾¾¾¾¾¾®
(ii) NaCN

47. Give reason for the following ‘In acylium ion the structure
R ¾ C ºº O+ is more stable than R ¾ C+ == O’ (1994, 1M) r r
404 Benzene and Alkyl Benzene

Answers
1. (c) 2. (a) 3. (c) 4. (a) 21. (a) 22. (d) 23. (d) 24. (a)
5. (a) 6. (c) 7. (a) 8. (d) 25. (c) 26. (a,b,d) 27. (b,c,d) 28. (b)
9. (b) 10. (b) 11. (d) 12. (c) 29. (b,c) 30. (b,c,d) 31. (b,c,d) 32. (a,c)
13. (d) 14. (c) 15. (d) 16. (b) 33. (b,d) 34. False 35. False 54. (12s, 3p)
17. (c) 18. (b) 19. (c) 20. (d) 58. (5)

Hints & Solutions

w
1. The given reaction takes place as follows :
It will stabilise the carbocation by
(+ve) hyperconjugation

Flo
CH3 O CH3 O
AlCl3 CH3
CH3 O O
Cl – AlCl s

ree
4
r –HCl r
–AlCl3
Intramolecular
ArSE 2-reaction H

F
CH3 –
AlCl4

2. Key Idea In electrophilic aromatic substitution (Ar SE 2) reaction, the aromatic (benzene nucleus here) compound (substrate) acts as

or
ur
a nucleophile which gets attacked by an electrophile in the rate determining step (rds).

f
Higher electron density on the nucleophile, i.e. benzene nucleus will fasten the rate of the reaction. Electron-donating groups (EDG) will
ks
increase electron density in benzene nucleus by any or both of their +R/+M and hyperconjugative (HPC) effects. Now, let us consider the
given substrate.
Yo
oo
O
eB

Cl –I effect dominates the +R effect CH3 + HPC C—CH3 –R effect

(I) (II) (III)


r

As –R effect is more powerful than –I effect, the order of their reactivity, towards an electrophile will be
ou
ad

III I II
(–R) < (–I>+R) < (+ HPC)
Y

3. Reactivity of substitution nucleophilic unimolecular (S N1) reaction depends on the formation of carbocation. Greater the stability of
carbocation, greater will be its ease of formation of alkyl halide and faster will be the rate of reaction. So, the correct order of (S N1) reactivity is
nd
Re

CH2Cl CH2—Cl
Fi

> > > CH3 CH2Cl

Cl
OCH3
(+R)
(C) (D) (A) (B)
In compound C, the carbocation formed is stabilised by activating group ( ¾ OCH3 ). Compound D forms benzyl carbocation (C6H5 ¾ CH2+ )
that is stabilised by resonance. Compound A produces a primary carbocation that further rearranges itself to secondary carbocation.
Nus Rearrangement
r r
Cl 1° carbocation 2° carbocation

Compound B produces primary carbocation which is least stable among all the given options.
Benzene and Alkyl Benzene 405

4. The major product of the given reaction is benzoic acid CH2CH3


(C6H5COOH). On vigorous oxidation of alkyl benzene with C2H5
C2H5Cl (1 eqv.) C2H5
acidic or alkaline KMnO4, aromatic acids are obtained. During
oxidation of alkyl benzene, the aromatic nucleus remains intact AlCl3 (anhyd.)
C2H5Cl
C2H5
and the entire chain is oxidised to ¾COOH group irrespective –HCl
C2H5 AlCl3
+
of the length of carbon chain.
–+
CH2CH3 COOK COOH C2H5Cl (excess) C2H5
AlCl3 (anhyd.)
(i) Alk. KMnO4 (ii) H3O+
C2H5 C2H5
H5
H5C2 C2H5 C2 C
Ethyl Potassium Benzoic acid 2H5Cl
benzoate + AlCl3
benzene

w
5. In presence of alc. KOH, the given halide undergoes (Minor)
C2H5
elimination reaction (Major)
8.

Flo
Alc. KOH CH3
Cl
Cl Cl

ree
The alkene produced undergoes polymerisation via free radical
mechanism. This process involve three steps, i.e. initiation,

F
Cl
propagation and termination.
hn/benzylic free radical
substitution with Cl2

or
n
ur
Polymerisation Cl2(1 eqv.) Cl2(2 eqv.) Cl2(excess)

f
–HCl –2HCl –3HCl
Cl Cl
Polymer CCl3
ks
CH2Cl CHCl2
Yo
6. Aromatic electrophilic substitution reactions are fast in those
oo
compounds in which the attacking site possess high electron Cl Cl Cl
density. Electron withdrawing groups (EWGs ) reduces the s s
s s
–3H –3OH(aq)
–Cl OH(Cl) –2HCl 2H2O/D
eB

electron density in the benzene ring due to its strong –I effect OH


and makes further substitution difficult. Hence, called as OH
deactivating groups. While electron releasing groups (ERGs) CH2OH CH C OH
O—H
increases the electron density in the benzene due to resonance.
r

OH
Therefore, activates the benzene ring for the attack by an
ou

Cl Cl Cl
ad

electrophile. Hence, called as activating groups. (Option-c) Gem-diol –H2O


·· (unstable) OH
Y

In given options, Activating groups are — CH3 < — OCH3


··
(+ I, + R) Strong + R effect –H2O C O
nd

Deactivating groups are


Re

— CN > — Cl CH O
Strong – I effect (- I > - R) Cl
So, the correct order is –
COO
Fi

CN Cl Me OCH3 as
Cl
(Option-d)
Cl
< < <
in basic medium
H3Or
D A C B COOH
7. Through aromatic electrophilic substitution mechanism
(ArS E2) when mono-alkylation (Friedel-Craft’s reaction) is Cl
performed, we get mono-alkylated benzene. As, the alkyl group (Option-b)
is ring activating (towards ArS E 2) in nature, again o- and So, considering the second reaction condition, Cl- , H2O, D in
p-substituted alkyl benzene will be obtained and so on. e.g.
the statement of the question, the correct answer is option (d).
406 Benzene and Alkyl Benzene

9. Melting point of a compound depends on size and force of attraction between the molecules. Compound (c) has largest size and also possess
weak intermolecular association due to dipole - dipole interactions.
Compound (a) is a dicarboxylic acid and possess high melting point due to intermolecular hydrogen bonding.
As a result, it exist as associated molecule. Compound (d) is an alcohol and possess intermolecular H-bonding. No interactions are present in
hydrocarbon (naphthalene) compound (b).
Hence, melting point is lowest for naphthalene (- ~ 80°C), i.e. compound (b).
10. Aromaticity of a compound can be decided by Huckel’s rule. In cyclopentadienyl cation (b), resonance takes place as follows:

H Resonance

w
hybrid

Hence, is anti-aromatic does not follow

Flo
Huckel’s rule as it has conjugated 4p-electron (4 np, n = 1) system. Rest of the species are aromatic as each of them belongs to 6p-electron
[(4 n + 2) p, n = 1] system.

ree
11. CH3 COOH COCl CHO

F
KMnO4 SOCl2 H2/Pd
BaSO4

or
Toluene Benzoic acid
ur Benzoyl
chloride
Benzaldehyde

f
12. PLAN This problem can be solved by using the various concepts synthesis of benzene, electrophilic substitution reaction and directive influence of various
substituents, including oxidation and reduction.
ks
Þ —OH and —NH2 are o/p-directing groups.
Yo
Þ N-acetylation is more favourable than C-acylation.
oo

Þ N-sulphonation is more favourable than C-sulphonation.


eB

Þ NO2 is a meta-directing group.


Þ H2S× NH3 reduces only one NO2 group selectively in the presence of two NO2 groups.
Using above concepts the correct sequence of reaction can be written as
r
ou
ad

NO2 NH2
Y

NaNO2
Red hot Fuming HNO3 / H2SO4 H2S .NH3
A. 3CH CH H2SO4
iron 873K
D selective reduction
nd

NO2 NO2 Here, N-sulphonation is


Re

N+2HSO4– OH more favourable in


comparisons to
C-sulphonation.
Fi

Hydrolysis

NO2 NO2
OH OH OH OH

+
O2N O2N
Conc. H2SO4 NO2 Dil.H2SO4
B.
60°C Here, nitration takes D
OH OH place at o-position of OH OH
less hindered side.
SO3H SO3H
Benzene and Alkyl Benzene 407

NO2 NH2 NHCOCH3 NHCOCH3 NHCOCH3


NO2
Sn/HCl CH3COCl Conc.H2SO4 HNO3
C.
Here, N-acetylation is
more favourable than
C-acetylation
SO3H SO3H
Dil.H2SO4 D

NH2 NHCOCH3

NO2 NO2

w
OH–

NO2 NO2 NO2 NO2 NO2

Flo
KMnO4 H3O+ SOCl2 NH3

ree
D. – OH/D

F
CH3 COO– COOH COCl CONH2
Above reaction proceeds through oxidation, chlorination and amide formation sequentially.

or
ur
\ A ® r, B ® s, C ® q, D ® p Hence, the correct choice is (c).

f
13. Draw the structure of organic compounds indicating net dipole moment which includes lone pair and bond angle also.
OH SH
ks
H H
O O
Yo
oo
In the quinol and thioquinol
same as in thioquinol.
eB

OH SH
— OH groups and —SH groups do not cancel their O O
dipole moment as they exist in different confirmation. H H
r
ou

14. COOH COOH


ad

HNO3
H2SO4
Y

HO HO
P
NO2
nd
Re

— OH is activating while — COOH is deactivating group in SE Ar reaction. Therefore, electrophile attack to ortho of the activating
— OH group.
Fi

OCH3 OCH3
HNO3
H2SO4
H3C H3C NO2
Q

Both — OCH3 and — CH3 are activating ortho/para directing groups but — OCH3 is stronger activator, electrophile attack to ortho of
— OCH3.
O O NO2

C C
O HNO3 O
II H2SO4

I S
Ring II is activated while ring I is deactivated in SE Ar reaction. Therefore, electrophile attack at para to ring-II, the less hindered position.
408 Benzene and Alkyl Benzene

21. SO2Cl 2 brings about free-radical chlorination at —CH3 group.


15. ¾ OCH3 + HBr ¾® CH3Br + ¾ OH
22. Free radical chlorination occur at —CH3.
—OH of phenol is not further substituted by bromide due to
resonance with the ring. hv
—CH3 + Cl2 —CCl3
Heat
16. Ring attached to nitrogen is activated while ring attached to
C==O is deactivated. Also, electrophilic substitution occur
predominantly at para position of the activated ring due to NaOH
—COOH
immense steric hindrance at ortho position.
benzoic acid
O
Conc. HNO3 23. The methyl group in toluene is ortho/para directing activating

w
N group :
Conc. H2SO4
H CH3 CH3 CH3
O 2N
O
Cl

Flo
FeCl3
N + Cl2 +

ree
H
Cl
17.
AlCl3

F
+ ClCH2CH2CH3 P 24. Toluene is most reactive among these. Nitro and carboxylic groups
Friedel-Craft’s
reaction
are deactivating in aromatic electrophilic substitution reaction.

or
O2/heat
ur Q + Phenol, 25. Toluene is most readily sulphonated among these because
H3O+ methyl group is electron donating (+ I effect), activate benzene
O
f
ring for electrophilic aromatic substitution.
ks
P= , Q = CH3—C—CH3 26. Reaction shown in option (a) is aldol condensation in the
Yo
Acetone presence of conc. H2SO 4 at high temperature.
oo

Cumene In summerised way the formation of mesitylene through this can


be visualised as
eB

18. Ring attached to nitrogen is activated by electron donating


resonance effect while ring attached to carbonyl group is CH3
deactivated by electron withdrawing resonance effect : C H CH3
r

O O H2O O C Conc. H2SO4


HN HN
ou

H H D
ad

HH H2O
H3C CH3 H3C CH3 O –3H2O
H—C
CH3 CH3
Y

Fe
Br2 C C
H C Mesitylene
CH3 O CH3
H
nd
Re

H
Br H2O
19. Both chloro and nitro groups are deactivating in electrophilic
Fi

aromatic substitution reaction. Also nitro group is stronger Reaction given in option (b) is simple polymerisation
deactivating group. Methyl group is activator in electrophilic (trimerisation) reaction of alkyne i.e.,
aromatic substitution. Me H or CH3 C CH when passed through heated
Hence, overall order of reactivity is : iron tube at 873K then mesitylene is formed as
nitrobenzene < chlorobenzene < benzene < toluene CH3
H CH3
½ H C
+ Cl - CH
20. C6H5 ¾ C ¾ Cl + SbCl 5 ¾® SbCl6- + C6H5 ¾ C ¾® Heated iron tube
½ HC 873 K
CH3
CH3 C CH3 CH3
planar C
(+)-2-chloro- carbocation HC CH3
2-phenylethane CH3 Mesitylene
(±) -2-chloro-2-phenylethane (racemic mixture)
This reaction is also called aromatisation.
Benzene and Alkyl Benzene 409

(1) and (2) reactions of option (c) combined to give haloform 28. CH3 CH3
reaction while (3) reaction given in this option is O
OH
decarboxylation reaction i.e., + O2 CH3
+ CH2 — CHCH3 H CH3
COCH3
Cumene hydroperoxide
(1) Br2/NaOH Cumene
(2) H3O+ Cumene hydroperoxide formed above is an intermediate in the
H3COC COCH3 synthesis of phenol.

COONa+
29. According to Huckel rule, the compounds which have
4 n (n = 0, 1, 2, 3¼) delocalised p-electrons in a close-loop are
+ 3CHBr3¯ anti-aromatic and characteristically unstable. Compound B
+– – satisfy the criteria of anti-aromaticity as :
COONa+ Bromoform

w
NaOOC
The above product of haloform reaction on decarboxylation gives
4p
benzene as

COONa+

Flo
Compound C is anti-aromatic in its resonance form :
(Sodalime) –
NaOH/CaO O O

ree
+ – – –3Na2CO3
NaOOC COONa+
Benzene

F
+
The reaction given in option (d) is Clemmensen reduction i.e.,
anti-aromatic

¾- Hg
C == O ¾Zn ¾® CH2 + H2O

or
Compound A has 4p-electrons which are also delocalised but do
HCl
ur not constitute close loop, hence non-aromatic.

f
Hence, the final product of this reaction is also mesitylene which Compound D is aromatic, characteristically stable.
can be seen as
ks
CHO CH3 –
Yo
O O
oo
Zn-Hg/HCl
+
eB

OHC CHO H3C CH3 Tropyllium ion, aromatic


Mesitylene
Order of stability Aromatic > Non-aromatic > Anti-aromatic
27. (a) 30. Aromatic molecule must.
r

NaOC2H5
Br (b) have (4 n + 2)p electrons.
ou

(c) be planar.
ad

(d) be cyclic.
C2H5O- (a strong nucleophile) causes E1 reaction to form
Y

isobutene as the major product. 31. A compound is considered to be aromatic, if it follows three
rules:
nd

(a) Must be cyclic and planar.


Re

(b) Cl AlCl3 1, 2-hydride


(b) Must have conjugated system in it.
shift 3° carbocation
(c) It must follow Huckel rule which states that number of
Fi

1° carbocation (Major)
p-electrons = (4 n + 2)

(A) Þ 2p-e- system [(4 n + 2)p , n = 0 ]Þ Aromatic

SE Tertiary butyl benzene(X)


(B) Þ 4p-e- system [ 4 np , n = 1]Þ Anti-aromatic

H+
(c) (X)
(C) Þ 8p-e- system [ 4 np , n = 2 ]Þ Anti-aromatic

OH +
BF3,Et2O (D) Þ The sp3-carbon does not support conjugation and
(d)
sp3 also devoids planarity.
Þ Non-aromatic
410 Benzene and Alkyl Benzene

32. CH3 CH3 + +


N==O N==O N==O
Fe
+ Br2 –
(c) (i)
Toluene
Br –
— CH3 in toluene is para directing group.It activates the ring by Hence, —NO is ortho/para directing although deactivating
both inductive and hyperconjugation effect. group. Therefore, ortho/para nitro derivatives are formed on
treatment with mixed acid.
33. In both Friedel-Craft’s reaction and Reimer-Tiemann reaction,
new carbon-carbon bond is formed : NO2 NO2
R

w
AlCl3 Friedel-Craft’s
+ R—Cl + HCl Conc. H2SO4
reaction (ii) + Conc. HNO3

OH OH NO2

Flo
CHO
– H+ Nitro group is meta directing as well as strongly deactivating.
+ CHCl3 + OH

ree
H2O

Reimer-Tiemann

F
reaction

34. An electron donating substituent in benzene orient ortho/para Pd/C


(d)

or
substitution.
2
ur H2
35. In benzene, carbons are sp -hybridised only, hence only two
p-orbitals are involved in hybridisation.
f
ks
CH3 CH3
Yo
oo
In the above hydrogenation reaction, the anti-aromatic character
C—Br C—OC2H5
of three cyclobutadiene rings are relieved that provide stability
36. (a) (i) + C2H5OH
eB

CH3 CH3 to the hydrogenation product.

+ HBr —Br + Ag+(aq) AgBr¯ + +


Acid 37.
r
ou

7-bromo-1,3,5-cycloheptatriene Tropylium ion


ad

CH3
(ii) Br + C2H5OH No reaction (aromatic, stable)
CH3
Y

F OH
—Br + Ag+ + + AgBr
+ F–
nd

(b) (i) + NaOH


Re

O2N O 2N anti-aromatic
5-bromo-1,3-cyclopentadiene
Fi

(unstable)
CH3 CH3
Nitro group from para position activate aromatic nucleophilic O
substitution.
Br2
F N—
38. Fe
(ii) + NaOH No reaction
deactivated activated
H3C O
CH2NO2
—NO2 is not in resonance with benzene ring, does not activate N— —Br
the aromatic nucleophilic substitution reaction, hence fluoride is
not formed.
Benzene and Alkyl Benzene 411

39. (i) Oxidation of side-chain alkyl group of benzene occur 42. Nitro group is a very strong deactivating group, the very slow
through free radical mechanism and it initiates at a -C as. Friedel-Craft’s reaction does not takes place in presence of nitro
group.
½ ½
Ph — C— H ¾® Ph — C · + H · 43. CH3 CH3 CH3
½ ½ CH3 C—CH3
In tertiary butyl benzene, there is no a - H, required to AlCl3
+ CH3—C—CH2Br CH3
initiate free radical oxidation reaction, hence not oxidised
with KMnO4. H CH3
CH3

C—CH3 +

w
+ KMnO4 No oxidation
CH3
takes place
CH3 C CH3
tert butyl benzene

Flo
(no a-H) CH3
(ii) Pi-electrons of double bonds are involved in aromatic
44. In presence of light, free radical reaction takes place at — CH3

ree
delocalisation (aromaticity), hence electrophilic addition do
not occur as it would destroy aromatic stability. However, group while in presence of FeBr3, electrophilic substitution
electrophilic substitution do not destroy aromaticity. occur in the ring.

F
CH3 CH2Br
E H Br2/heat
E+ Addition
H+ + + HBr or light

or
ur Br
Br2/FeBr3

f
aromaticity retained electrophilic loss of aromaticity CH3
(occur preferably) substitution (not occur)
CH3
ks
Yo
+
CH2CH3 COOH
oo
K2Cr2O7 Br
40. (i)
eB

Br
OCH3 O OCH3
CaO/NaOH
(i) AlCl3 Zn(Hg)
r

Heat 45. + O
ou

(ii) H3O+
ad

HCl
CH2CH3 CH—CH3
Y

Br2 NaCN O
(ii) O COOH
heat Br
Friedel-Craft’s
nd

CH3
Re

reaction
OCH3
CH—CH3 C—COOH
Fi

+
H /H2O
H3PO4
CN H Final product

NHCOCH3 NHCOCH3
Br2/Fe COOH
41. CH3
Br
C—CH3
H2SO4
NHCOCH3 46. (i) + (CH3)2CHCH2OH
CH3
+
Br H
NHCOCH3 is ortho/para directing although deactivating group. CH2CH3 C—CH3
Therefore, Ortho/pare bromoderivatives are fomed or (i) Br2/heat
treatment.
(ii)
(ii) NaCN CN
412 Benzene and Alkyl Benzene
+ +
47. R — C ºº O ¬® R — C==O CaC2 + 2H2O ¾® Ca(OH)2 + C2H2
I II Acetylene
I is more stable resonance structure of acylium ion than Cu
3C2H2 ¾¾® C6H6
II on the following grounds. D Benzene
It has more covalent bonds. (ii) —CH3 in toluene is ortho/para directing group, first
It has complete octet of both carbon and oxygen. oxidised to —COOH in order to make it meta directing and
O then nitrated.
HNO3/H2SO4
—C—O— CH3 COOH COOH
48. (i) mononitration

K2Cr2O7 Conc. HNO3


deactivated activated

w
Conc. H2SO4
ring ring
O NO2

—C—O— —NO2 O

Flo
C—CH2CH3
(ii) C6H5COOH + CH3MgI ¾® CH4 + Mg(OOCC6H5 )I AlCl3
53. + CH3CH2COCl

ree
CH3 CH3
49.
AlCl3 + Hydride Friedel-Craft’s acylation
CH3—C—CH2Cl CH3—C—CH2 H

F
shift
H H H H
It has 12 sigma and 3 pi-bonds.

or
1° carbocation 54.
ur CH3 H H

f
CH3
C—CH3 H
C6H6
ks
CH3—C—CH3
+ CH3 55. CH3CH2OH + PCl 5 ¾® POCl 3 + HCl + CH3CH2Cl
Yo
3° carbocation
oo
(more stable) AlCl 3
C6H6 + CH3CH2Cl ¾¾® C6H5— CH2CH3
+ –
eB

50. — —
— Friedel-Craft’s reaction

+ 56. H3C— —CH3 + Conc. H2SO4


r
ou


ad

— – p-xylene
SO3H OH
Y

The above resonance activate an aromatic ring for electrophilic


KOH
substitution reaction. H3C— —CH3 H3C— —CH3
51. Both — CH3 and — OCH3 are activating groups but chloro is fusion
nd
Re

deactivating group in electrophilic aromatic substitution


reaction. 57. Pi-bonds of benzene are involved in aromaticity, not open for
Fi

Also — OCH3 activate more than — CH3 due to resonance electrophilic addition reaction. Rather, it undergo electrophilic
effect by the former group. Therefore, the overall order of substitution reaction.
reactivity is:
Chlorobenzene < benzene < toluene < methoxy benzene 58. The aromatic systems are

52. (i) Lime is heated with coke at high temperature and then, r s
hydrolysed to form acetylene. This acetylene on passing red r
hot tube polymerises to form benzene :
, , , ,
2000 °C
CaO + 3C ¾¾¾® CaC2 + CO ­
Lime Coke

Download Chapter Test


http://tinyurl.com/y23lzk9x or
29
Aromatic Compounds
Containing Nitrogen

w
Objective Questions I (Only one correct option) 3. Which of the following is not a correct method of the

Flo
preparation of benzylamine from cyanobenzene?
1. Benzene diazonium chloride on reaction with aniline in the (a) H 2 / Ni (2019 Main, 10 April II)
presence of dilute hydrochloric acid gives (2019 Main, 12 April II)
(b) (i) HCl/ H 2O (ii) NaBH 4

ree
(c) (i) LiAlH 4 (ii) H3O+
(a) —NH2
(d) (i) SnCl2 + HCl(gas) (ii) NaBH 4

F
4. Aniline dissolved in dil. HCl is reacted with sodium nitrite
(b) —N==N — at 0°C. This solution was added dropwise to a solution

or
H2N
ur containing equimolar mixture of aniline and phenol in
dil.HCl. The structure of the major product is

f
(2019 Main, 9 April I)
(c) —N==N— —NH2
ks
(a) N N NH2
Yo
oo
(d) —N==N—NH—
(b) N N O
eB

2. The increasing order of the pK b of the following compound is N N


(c) OH
F
r

S
ou
ad

(A) (d) N N NH
N N
Y

CH3O H H
S
5. The major product obtained in the following reaction is
(B)
nd

NH2
Re

N N (i) CHCl3/KOH
(ii) Pd/C/H2
Fi

O2N H H
S
(C) CN O (2019 Main, 8 April II)
N N
H H
NCH3 NCH3
H H
H 3C (a) (b)
S
(D)
N N CN O CN OH
H H (2019 Main, 12 April I) H H
NCHCl2 NCH3
(a) (A) < (C) < (D) < (B)
(c) (d)
(b) (C) < (A) < (D) < (B)
(c) (B) < (D) < (A) < (C)
(d) (B) < (D) < (C) < (A) CN OH H 2N OH
414 Aromatic Compounds Containing Nitrogen

6. The major product in the following reaction is 9. The major product of the following reaction is
(2019 Main, 8 April II) (2019 Main, 11 Jan I)
NH2 O

N OEt (i) Ni/H2


N Base
+CH3I CN (ii) DIBAL- H
N
H N
NH N
(a) (b)
NH2 NH2

N + N H
NCH3 N
(a) (b)
N N O OH
N N (c) CHO (d)
H NH2

w
CH3
NHCH3 NH2 10. What will be the major product in the following mononitration
reaction ? (2019 Main, 10 Jan II)
N N N N
(c) (d)

Flo
N N +
H N N N HNO3
H H

ree
CH3 Conc.H2SO4

7. Coupling of benzene diazonium chloride with 1-naphthol in O2N


NO2

F
alkaline medium will give (2019 Main, 8 April I) (a) (b)
OH N N
H H

or
(a) (b)
ur
(c)
f (d)
ks
N N O2N N N
Yo
H H
OH
oo
N N
NO2
eB

11. An aromatic compound ‘A’ having molecular formula C7H6O2


on treating with aqueous ammonia and heating forms compound
‘B’. The compound ‘B’ on reaction with molecular bromine and
potassium hydroxide provides compound ‘C’ having molecular
r

formula C6H7N. The structure of ‘A’ is (2019 Main, 10 Jan II)


ou
ad

OH
OHC CHO
Y

N (a) (b)
(c) (d) OH
N N OH
nd

N
Re

COOH
(c) (d)
Fi

CH==CHCHO

OH
12. The increasing basicity order of the following compounds is
8. The increasing order of reactivity of the following compounds CH 2CH3
towards reaction with alkyl halides directly is
(2019 Main, 12 Jan I)
½
(A) CH3CH 2NH 2 (B) CH3CH 2NH
O O CN

NH2
NH2 NH2 CH3 CH3
NH ½ ½
(C) H3C ¾ N ¾ CH3 (D) Ph ¾ N ¾ H
O
(A) (B) (C) (D) (a) (D) < (C) < (B) < (A)
(a) (A) < (C) < (D) < (B) (b) (A) < (B) < (C) < (D)
(b) (B) < (A) < (C) < (D) (c) (A) < (B) < (D) < (C)
(c) (B) < (A) < (D) < (C) (d) (D) < (C) < (A) < (B) (2019 Main, 9 Jan II)
(d) (A) < (B) < (C) < (D)
Aromatic Compounds Containing Nitrogen 415

13. The tests performed on compound X and their inferences are : 18. The major product of the following reaction is (2017 Adv.)
OH
Test Inference
(a) 2, 4- DNP test Coloured precipitate (i) NaNO2/HCl/0°C
(ii) (aq. NaOH)
(b) Iodoform test Yellow precipitate
(c) Azo-dye test No dye formation NH2
– +
Compound ‘X ’ is OH ONa
H 3C CH3 H 3C CH3
N N (a) (b)
COCH3
(a) CHO
(b) N N NH2

w
N N OH OH
NH2 OH NH2

CH3 CHO (c) (d)


(c) (d)

Flo
(2019 Main, 9 Jan II)
Cl
14. The major product of the following reaction is

ree
19. In the reaction,
NH2
C

F
NH2 (i) Br2/hn
NaNO2/HCl CuCN/KCN
(ii) KOH (dil.) D E + N2
CH2CH3 (2019 Main, 9 Jan II) 0.5°C D

or
ur
CH3

f
NH
(a) (b) NH The product E is (2015 Main)
ks
CH3 COOH
Yo
CH3
oo
(a) (b) H3C CH3
eB

NH NH
(c) (d)
CH3
15. Arrange the following amines in the decreasing order of CN
r

CH3
basicity:
ou

(2019 Main, 9 Jan I)


ad

(c)
(d)
Y

N N N
(I)
H H CH3
nd

(II) (III)
Re

20. Amongst the compounds given, the one that would form a
(a) I > II > III (b) III > II > I (c) I > III > II (d) III > I > II brilliant coloured dye on treatment with NaNO 2 in dil. HCl
Fi

16. The correct decreasing order for acid strength is followed by addition to an alkaline solution of b-naphthol is
(2019 Main, 9 Jan I) NHCH3
N(CH3)2
(a) FCH2COOH > NCCH2COOH (a) (b)
> NO2CH2COOH > ClCH2COOH
(b) CNCH2COOH > O2NCH2COOH NH2 CH2NH2
> FCH2COOH > ClCH2COOH (c) (d)
(c) NO2CH2COOH > NCCH2COOH H 3C (2011)
> FCH2COOH > ClCH2COOH
(d) NO2CH2COOH > FCH2COOH 21. The species having pyramidal shape is (2010)
> CNCH2COOH > ClCH2COOH (a) SO3 (b) BrF3 (c) SiO2-
3 (d) OSF2
17. Which of the following compounds will give significant amount 22. In the following reaction,
of meta-product during mononitration reaction? (2017 Main)
OH OCOCH3 NH2 NHCOCH3 O

N Conc. HNO3
(a) (b) (c) (d) H X
Conc. H2SO4
416 Aromatic Compounds Containing Nitrogen

The structure of the major product X is (2007, 3M) 25. Benzene diazonium chloride on reaction with phenol in
weakly basic medium gives (1998, 2M)
O
(a) (a) diphenyl ether (b) p-hydroxy azobenzene
NO2 (c) chlorobenzene (d) benzene
N
H 26. Examine the following two structures of anilinium ion and
choose the correct statement from the ones given below :
O 2N (1993, 1M)
O +
(b) NH3 NH3
N +
H

I II

w
O
(c) (a) II is not an acceptable canonical structure because
N carbonium ions are less stable than ammonium ions
H (b) II is not an acceptable canonical structure because it is

Flo
NO2 non-aromatic
(c) II is not an acceptable canonical structure because the
O nitrogen has 10 valence electrons
(d)

ree
(d) II is an acceptable canonical structure
O2N N
H 27. Chlorobenzene can be prepared by reacting aniline with

F
(a) hydrochloric acid (1984, 1M)
(CH3)2NH (b) cuprous chloride
23. F NO2 (A) (c) chlorine in the presence of anhyd AlCl 3

or
DMF,D
ur (d) nitrous acid followed by heating with cuprous chloride
(i) Fe/HCl

f
(ii) NaNO2/HCl/0°C Objective Questions II
(B), B is
ks
(iii) H2/Ni (One or more than one correct option)
Yo
(2003, 1 M)
28. Aniline reacts with mixed acid (conc. HNO 3 and
oo
CH3
(a) H2N N conc. H 2SO 4 ) at 288 K to give P (51%),Q (47%) and R
eB

CH3 (2%). The major product(s) of the following sequence is (are)


(1) Sn /HCl
(1) Ac2 O, pyridine (2) Br2 /H 2 O (excess)
(2) Br2 , CH 3 CO 2 H
(b) H2N NH2 R ¾ ¾ ¾ ¾ ¾ ¾ ¾ ¾® S ¾¾¾¾¾¾¾¾¾®
(3) H 3 O+ (3) NaNO 2 , HCl /273-278 K
r

(4) NaNO 2 , HCl /273-278 K (4) H 3 PO 2


ou
ad

CH3 (5) EtOH, D


(c) O2N N Major product(s) (2018 Adv.)
Y

CH3
Br Br
(d) O2N NH2
nd

Br Br
Re

(a) (b)
24. The most unlikely representation of resonance structures of Br Br
Fi

p-nitrophenoxide ion is (1999, 2M)


_ _ _
Br Br
O + O O + O (d) Br
N N (c) Br

(a) (b) Br Br Br Br
Br
O– O 29. The product(s) of the following reaction sequence is (are)
O O O– + O (2017, Adv.)
N N NH2
(i) Acetic anhydride/Pyridine
(ii) KBrO3/Br
(c) (d)
– (iii) H3O+, Heat
(iv) NaNO2/HCl, 273-278 K
(v) Cu/HBr
O– O
Aromatic Compounds Containing Nitrogen 417

Br Br Br Br 32. p-chloroaniline and anilinium hydrochloride can be


Br Br Br Br distinguished by (1998, 2M)
(a) Sandmeyer reaction (b) NaHCO3
Br (c) AgNO3 (d) Carbylamine test

Br Br
33. When nitrobenzene is treated with Br2 in the presence of FeBr3 ,
(a) (b) (c) (d) the major product formed is m-bromonitrobenzene. Statements
which are related to obtain the m-isomer, are (1992, 1M)
30. In the following reactions, the major product W is (2015 Adv.)
(a) the electron density on meta carbon is more than that on ortho
OH
NH2 and para positions
NaNO2,HCl , NaOH (b) the intermediate carbonium ion formed after initial attack of
0°C
V W
Br + at the meta position is less destabilised

w
(c) loss of aromaticity when Br + attacks at the ortho and
para positions and not at meta position
OH
(d) easier loss of H+ to regain aromaticity from meta position
than from ortho and para positions

Flo
(a) (b)
N=
=N OH N=
=N
Assertion and Reason

ree
Read the following questions and answer as per the direction given
HO below :

F
OH
(a) Statement I is correct; Statement II is correct; Statement
II is the correct explanation of Statement I.
(c) N=
=N (d)
(b) Statement I is correct; Statement II is correct; Statement

or
ur N=
=N
II is not the correct explanation of Statement I.
(c) Statement I is correct; Statement II is incorrect.

f
(d) Statement I is incorrect; Statement II is correct.
ks
31. In the reaction shown below, the major product(s) formed is/are
34. Statement I Aniline on reaction with NaNO2/HCl at 0°C
Yo
(2014 Adv.)
NH2 followed by coupling with b-naphthol gives a dark blue coloured
oo
precipitate.
eB

Acetic Statement II The colour of the compound formed in the


anhydride
NH2 Product(s) reaction of aniline with NaNO2/HCl at 0°C followed by
CH2Cl2 coupling with b-naphthol is due to the extended conjugation
O (2008, 3M)
r

N
ou
ad

H CH3 35. Statement I In strongly acidic solution, aniline becomes more


reactive towards electrophilic reagents.
O
Y

Statement II The amino group being completely protonated in


(a)
NH2 + CH3COOH strongly acidic solution, the lone pair of electrons on nitrogen is
no longer available for resonance.
nd

(2001, 1M)
Re

O
NH2
Match the Column
Fi

(b) H 36. Match the compounds in Column I with their characteristic


N CH3 + CH3COOH test(s)/reaction(s) given in Column II. (2008, 6M)

O O Column I Column II
H
N CH3 Sodium fusion
Å s extract of the
O (A) H2N ¾ NH3 Cl (p) compound gives
(c) H Prussian blue
N CH3 + H2O
colour with FeSO4
O O Å
r s O2N NHNH3Br
NH3CH2COO (B) Gives positive
(q) FeCl3 test
NO2
(d) H
N CH3

O O
418 Aromatic Compounds Containing Nitrogen

Column I Column II 44. What would be the major product in the following reaction?
O
Å Gives white
(C) HO NH3Cl precipitate with
(r) Br2/Fe
AgNO3
N—
Reacts with (2000, Main, 1M)
Å
O2N NHNH3Br aldehydes to form
(D) the corresponding 45. How would you bring about the following conversion (in 3 steps)?
(s)
hydrazone
NO2 derivative Aniline ¾® Benzylamine (2000, 3M)

46. Complete the following reactions with appropriate reagents


Fill in the Blanks O I

w
37. The high melting point and insolubility in organic solvents of
sulphanilic acid are due to its…… structure. (1994, 1M) —NH—C—CH3 —NO2

38. In an acidic medium, ……… behaves as the strongest base. I (1994, 4M)
(nitrobenzene, aniline, phenol) (1981, 1M)

Flo
47. Following reaction gives two products. Write the structures of
the products.
Subjective Questions
Br 2 /Fe

ree
39. Convert CH3 CONHC6 H5 ¾¾¾® (1998, 2M)
NO2 NO2
48. Write the structure of foul smelling compound obtained when

F
aniline is treated with chloroform in the presence of KOH.
(1996, 1M)
to

or
49. Complete the following with appropriate structure:
ur OH
NH2

f
in not more than four steps. Also mention the reaction conditions
and temperature. NO2 (i ) NaNO2 and HCl at 5°C
ks
(2004, 4M)
¾¾¾¾¾¾¾¾¾® A
40. Which of the following is more acidic and why? (ii ) Anisole
Yo
(1995, 1M)
NO2
oo
+ +
NH3 NH3
eB

50. How will you bring about the following conversions?


“Benzamide from nitrobenzene” (1994, 2M)

51. How will you bring about the following conversions?


r

I “4-nitrobenzaldehyde from benzene” (1994, 2M)


ou
ad

F
II (2004, 2M) 52. Outline a synthesis of p-bromonitrobenzene from benzene in
Y

two steps. (1993, 2M)


41. Convert (in not more than 3 steps) : (2003, 2M)
COOH COOH 53. Write the structure of the major organic product expected from
the following reaction.
nd
Re

CH3
+ HNO2
Fi

—N
F
CH3
42. There is a solution of p-hydroxybenzoic acid and p-amino (1992, 1M)
benzoic acid. Discuss one method by which we can separate 54. How will you bring about the following conversion?
them and also write down the confirmatory test of the functional
“4-nitro aniline to 1, 2, 3-tribromobenzene.” (1990, 2M)
group present. (2003, 4M)
43. Write structures of the products A, B, C, D and E in the following 55. Complete the following with appropriate structures: (1986, 1M)
scheme
O Base ?
—NH2 + —COCl
CH2CH2CH3 Cl2/FeCl3 Na-Hg/HCl
A B 56. How would you convert aniline into chlorobenzene? (1985, 1M)
Cl CH2

CHCH2 ONa +
HNO3/H2SO4 57. State the conditions under which the following preparation is
D C
H2O carried out. “Aniline from benzene” (1983, 1M)
H2 /Pd/C
E (2002, 5M)
58. State the equation for the preparation of following compound.
Chlorobenzene from aniline (in 2 steps). (1982, 1M)
Answers
1. (c) 2. (c) 3. (b) 4. (a) 25. (b) 26. (c) 27. (d) 28. (d)
5. (d) 6. (*) 7. (c) 8. (b) 29. (b) 30. (a) 31. (a) 32. (c,d)
9. (b) 10. (b) 11. (c) 12. (d) 33. (a,b) 34. (d) 35. (d)
13. (b) 14. (a) 15. (d) 16. (c) 36. A ® r, s B ® p, q C ® p, q, r D® p
17. (c) 18. (a) 19. (c) 20. (c)
37. zwitter ionic 38. Aniline
21. (d) 22. (b) 23. (a) 24. (c)

Hints & Solutions

w
1. Benzene diazonium chloride on reaction with aniline in the 3. The preparation of benzylamine from cyanobenzene using
presence of dilute hydrochloric acid undergoes coupling given reagents are as follows :
reaction and produces p-amino azobenzene. In this reaction, H2/Ni
Ph ¾ C= N Ph ¾ CH2 ¾ NH2

Flo
benzene diazonium chloride reacts with aniline in which aniline D
(Cyanobenzene) Benzylamine
molecule at its para-position is coupled with diazonium salt to
give p-amino azobenzene (yellow dye).

ree
+ – (i) LiAlH4
N NCl + H NH2 (ii) NaBH4
(ii) H Or
3

F
Benzene Aniline (i) SnCl2 r
[Ph ¾ CH==NH2[2 SnCl 62-
diazonium
(ii) HCl (g)
salt H+ bis (benzaldiminium )
N N NH2+Cl–+H2O

or
hexachlorostannate (IV)
ur
p-amino azobenzene HCl , H O NaBH
PhCN ¾¾¾® PhCOOH ¾¾®
2 4
PhCH2OH

f
(yellow dye)
–NH4Cl Benzoic acid Benzyl alcohol
ks
2. Key Idea pK b value is defined as the minus lagarithm of K b Thus, option (b) is incorrect.
Yo
smallar the value of pK b stronger is the base and vice-versa.
4. Major product of the reaction is
oo
In the given options, +R effect is shown by —CH3 and —OCH3
group ( ¾ OCH3 > ¾ CH3 ). These group increases the electron
eB

N N NH2
density at o and p-positions. Groups such as ¾ F and—NO2
shows –R-effect ( ¾ NO2 > ¾ F). These group decreases the It is obtained by coupling reaction.
electron density at o and p- positions. Reaction of aniline with dil. HCl and sodium nitrite at 0°C is
r

Increase in electron density at p-position makes the unshared shown below :


ou
ad

electron pair of ‘N ’ more available and decrease in electron +–


NH2 NººNCl
density at p-position makes the unshared electron pair of ‘N ’
Y

less available. Compound containing —OCH3 group act as


strongest base and hence possess lowest value of pK b . So, the + NaNO2 + Dil. HCl 0°C
correct increasing order of pK b in the given compound is
nd
Re

Diazonium salt
O2N S Diazonium salt formed when added to equimolar mixture of
>
Fi

aniline and phenol in dil. HCl then aniline couples with


N
H N
H diazonium salt. Reaction is as follows :
(C)
+ +
F S NººNCl + H2N
>
N
H N
H In acidic
H+ medium

(A )
N=
=N —NH2 + HCl
H3C S
>
N
H N
H For an effective coupling, the solution must be so alkaline
that the concentration of diazonium ion is too low. It must not
(D)
be so acidic that the concentration of free amine or phenoxide
H3CO ion is too low. That is why amines couple fastest in mild
S
acidic solution and phenols couple fastest in mild alkaline
N
H N
H solution.
(B)
420 Aromatic Compounds Containing Nitrogen

s NaOH
5. [ Ar ¾ N ºº N ]+ OH - Ar ¾ N == N ¾ OH
1° NH2 responds
H+ Does not couple
NH2 NººC NaOH - +
CHCl3/KOH
(Carbylamine reaction)
- Ar ¾ N == N ¾ O Na
H+ Does not couple

CN O NººC O Following conditions are very useful for coupling reaction to


take place.
Pd/C/H2
(a) The solution must be so alkaline that the concentration of
diazonium ion is too low.
NH—CH3
(b) It must not be so acidic that the concentration of free
phenoxide ion is too low.

8. The reaction of alkyl halide with amine or amides is a

w
H2 N OH
nucleophilic substitution reaction. More nucleophilic nitrogen is
In step (i), 1° amine is converted into isocyanide, when reacts more reactive with alkyl halide. Compound (A) is benzamide
with CHCl3 / KOH. This reaction is known as carbylamine and lone pair of ‘N’ of it, is not available in this compound.
reaction. This reaction is used to detect primary amine in a

Flo
s
compound. O O
In step (ii), isocyanide is reduced to secondary amine and C NH2 C
+
NH2

ree
cyanide is reduced to primary amine, and carbonyl group to
alcoholic group when treated with Pd/C/H2.
d- d+
(A)
6. N is more electronegative than ‘C’. So, H of N ¾ H bond is

F
In compound (D), lone pair of ‘N’ are available but in compound
more acidic than that of C ¾ H bond in the ring.
(C), the lone pair of ‘N’ are not readily available due to the
Again, sp2-N is more electronegative (s% = 33.3) than sp3-N presence of electron withdrawing group ( ¾CN) attach to

or
(s% = 25) of the ¾ NH2 group.
ur benzene ring.

f
sp3 In compound (B), i.e. pthalimide, ‘N’ is not nucleophilic due to
NH2 NH2 the presence of two polar groups attached to it that pulls lone
ks
sp2
N sp2 N
pair towards them. Hence, the correct order is:
Yo
N N
Base ( B ) < ( A ) < (C ) < (D ).
oo
N N sp2 sp2
N N 9. Ni/H 2 can reduce —C ºº N into ¾ CH2 ¾ NH2 (1°-amine) but
eB

Non-aromatic s
ring H Aromatic cannot reduce an ester group ( ¾ CO2Et) whereas DIBAL-H,
ring Both aromatic di-isobutylaluminium hydride, [(CH3 )2CH]2AlH reduces the
rings
s
B Apparently ester group ( ¾ CO2Et) into —CHO (an aldehyde) and
sp3 C2H5 OH.
r

(Base)
d+ d– – BH
ou
ad

CH3–I –I
s

(SN2 attack)
OEt Ni/H2/D OEt
Y

NH2 CºN DIBAL–H


NH2 –EtOH
Note This C C¢ N
nd

N
Re

1 CH
are missing in all 2
the option given. N –H2O NH2
N N
Fi

3 (Nucleophilic addition
elimination reaction)
4
CH3
(Option-b) 10. Here, in mononitration the electrophile produced from mixed
+
7. Coupling of benzene diazonium chloride with 1-naphthol in acid (HNO3 + conc.H2SO4 ) is N O2.
alkaline medium will give the following coupling reaction.
OH OH O –R
A +R

PhN2+ N
B
OH –
H
N==N Ring-(A) is activated, i.e. becomes more nucleophilic by the + R
· ·
- + effect of the ¾ N H ¾ group and it becomes o / p-directing
In the presence of OH ion, Ar N 2 exists in equilibrium with an
towards the electrophile, NO+2 in the ArS E 2 reaction. For
unionised compound (Ar ¾ N == N ¾ OH) and salts Å
+ –
s mononitration, N O2 will preferably come at p-position,
[ ArN == N ] ¾OH derived from it, which do not couple.
Aromatic Compounds Containing Nitrogen 421

whereas the ring-(B) gets deactivated by -R- effect of the Aromatic amines (D) are always weaker bases than aliphatic
O amines, because of the conjugation of lp of electrons of N
½½ (+ R-effect) with the benzene ring.
¾ C ¾ group, So, the correct order is (D) < (C) < (A) < (B).
r
O2N O r 13. It is a 3°-amino group (—NMe2) though
NO2
aromatic which will not respond to
C rds of ArSE2 CH3
N diazotisation (NaNO2 + dil. HCl/5°C).
NH CH3 So, formation of diazonium ion and its
further reaction with alkaline phenol or
H b-napthol at 5-10°C to produce a
O precipitate of azo-dye will not take place.
O2N
r C (b) It is a keto group C==O and so it

w
NH Fast,–Hr
responds to 2, 4-DNP test with Brady’s
CH3 —— C==O reagent (2, 4-dinitrophenyl hydrazine) to
O2N O give a reddish orange precipitate.
O

Flo
C
NH It is a keto-methyl group —C—CH3 and
hence, it responds to iodoform test to

ree
give a yellow precipitate of iodoform
11. Using retro-synthesis, to get the required compounds, A , B and (CHI3).

C. Compound (c) is an alcohol and does not give positive 2, 4-DNP

F
Br
C6H7N ¬¾
¾2 ¾ B test. Hence, eliminated (a) and (d) does not have ¾ CH3CO
(C ) KOH [An amide] group and does not give idoform test. Hence, eliminated.
[1°-amine] Hoffmann (No.of C =7) NH3/D

or
(No. of C = 6 bromamide
1 carbon less) reaction
ur 14. In the given reaction,
(step down)

f
Responds to Hoffmann's
A bromamide degradation and
ks
[Carboxylic acid gets converted into
So, the reactions can be shown as: C 7 H 6 O2 ] [¾NH2 +K2CO3] by Br2/KOH
Yo
C
oo
COOH CONH2 NH2
NH3/D NH2
Br2/KOH CH2 Undergo benzylic free
–H2O
eB

–K2CO3 radical substitution with


(A) (B) –KBr
(i) Br2/hn CH3 Br2/hn to give [¾CH¾CH3]
[C7H6O2] Benzenamide (C) (ii) KOH (dil.)
Benzoic [C6H7N] Br
H
r

acid Aniline
N + H
ou

N
ad

H –Br
12. If we consider Lewis basicity (basicity in aprotic solvents or in H
vapour phase), the order of basicity will be. Br
Y

CH3
Formation of –H+
D A B C
2° carbocation
2° 1° 2° 3° s
nd

by removing Br NH
Re

But, this order does not match with the options given. So, it has
CH3
been asked on basicity of the amines in aqueous solution. When
Fi

no phase is given, then basicity of amine is considered in 15. Key Idea


aqueous solution as they are liquids. In aqueous solution, Basicity µ Ease of donation of lone pair of electrons
basicity of 2°-amines (aliphatic) is maximum because, of the 1 1
thermodynamic stability of its conjugate acid. µ µ
Å È
% of s - character of N Electronegativity of N
Et 2 NH + H 2O c Et 2 N H 2 + OH
(B ) Conjugate acid The % of s-character in the given amines are as follows:
Å
Et 2 N H2 is a sterically symmetric tetrahedral ion as it contains
equal number (two) of bulkier Et-group and small size H-atoms. N N N
sp2-hybridised sp3-hybridised
Here, two H-atoms give additional stability through hydrogen H
[s-character=33.3%]
H [s-character=25%]
bonding with H2O (solvent) molecules. sp2-hybridised
[s-character=33.3%]
Et
d+ d+ Pyridine Pyrrole Piperidine
H d

N d– (I) (II) (III)
O H
H
Et Therefore, piperidine (III) having minimum
H-bonding
422 Aromatic Compounds Containing Nitrogen

% s-character is most basic. Among the rest, pyridine (I) and aromatic amine, on treatment with NaNO2 in dil. HCl forms the
pyrrole (II) the lone pair of electrons of N in pyrrole (II) is corresponding diazonium chloride salt.
involved in delocalisation and follows (4 n + 2) p aromatic + –
NH2 N2 Cl
(n = 1) system. So, the N-atom of pyrrole (II) will show least
NaNO2
basicity. Thus, the order of basicity is as follows: HCl
H3C H 3C
> >
N N N
OH

H H
(III) (I) (II) b-naphthol
N N CH3
16. All the given compounds are a-monosubstituted acetic acid
derivatives and the a-substitutions have been made by strong–I OH

w
groups/atoms. More powerful the - I group, stronger will be the
acid.
-I power of different groups is as follows : 21. F—S==O S


¾ NO2 > ¾ CN > ¾ F > ¾ Cl. F O

Flo
F F
Thus, the correct decreasing order for acid strength is: S is sp3-hybridised Pyramidal
NO2CH2COOH > NCCH2COOH > FCH2COOH SO3 is planar (S is sp2-hybridised), BrF3 is T-shaped and SiO2-
3 is

ree
> ClCH2COOH. planar (Si is sp2-hybridised).
17. Aniline in presence of nitrating mixture

F
O
(conc. HNO3 + conc. H2 SO4) gives significance amount 22. ½½
(» 47%) of meta-product because in presence of H2SO4 its N Conc. HNO3

or
protonation takes place and anilinium ion is formed.
ur I ½ Conc. H2SO4
H

f
NH2 NH3HSO4
II O 2N
ks
H2SO4 O
½½
Yo
N
oo
Aniline Anilinium ion
½
Here, anilinium ion is strongly deactivating group and H
eB

meta-directing in nature. So, it gives meta-nitration product.


Ring-I is activated while ring-II is deactivated towards
NH2 NH3HSO4 electrophilic aromatic substitution reaction.
H3C
r

Conc.H2SO4 (CH3)2NH
ou

23. F NO2
ad

+Conc.HNO3 N— NO2
NO2 DMF
H3C
Y

18. Diazo coupling occur at para-position of phenol.


H3C
OH OH OH (i) Fe/HCl
N— NH2
nd
Re

(ii) NaNO2
(i) NaNO2/HCl/0°C (iii) H2/Ni H3C
(ii) NaOH
Fi

+ O O
NH2 N2Cl– N N N
19. NH2
+
N2 Cl
-
CN Nitrogen has five bonds and 10 valence
24. electrons, not an acceptable resonance structure.
NaNO2/HCl D
0-5°C CuCN/KCN –
diazotisation O
+
CH3 CH3 25. C6H5N2Cl– + C6H5OH
CH3
4-methyl aniline D E
4-methyl diazonium chloride 4-methyl benzocyanide
—N==N— —OH
20. As we know, benzenediazonium salt forms brilliant coloured
dye with b-naphthol, the compound under consideration must be
p-hydroxy azobenzene
p-toluidine (c) as it is a primary aromatic amine. Primary
Aromatic Compounds Containing Nitrogen 423

So, Br
26. In structure II, nitrogen is associated with five bonds and 10
valence electrons, hence not acceptable. NO2 NH2 NH2
Sn/HCl Br2
H H2O (Excess)
Br Br
H—N—H Br Br Br
(S) NaNO2+HCl
+ 273-278K
Br Br

N2+Cl
N-has five bonds H3PO2
and 10 valence electrons

w
Br Br Br Br
CuCl
27. C6H5NH2 + HNO2 ¾® C6H5N2OH ¾® C6H5 ¾ Cl Br Br
Major product
Conc. HNO3 +
28. Given, Aniline ¾ ¾ ¾ ¾ ¾¾ ® P + Q + R 1, 3, 4, 5-tetrabromobenzene
Conc. H 2SO4

Flo
(51%)
(47%) (2%) Hence, only (d) is the correct answer.
Then P, Q and R will be 29. NH 2 NHAc

ree
NH2 NH2 NH2 NH2
Ac2O/Pyridine
NO2

F
Conc. HNO3+
+ +
Conc. H2SO4
KBrO3 + HBr ¾® Br2
NO2 Ac is CH3CO (acetyl), it protects ¾ NH2 group from being

or
NO2
ur oxidised.

f
P (51%) Q (47%) R (2%)
NHAc NHAc NH2
ks
(i) Ac 2O, pyridine
(ii) Br2 , CH 3COOH Br2 water H 3O +
Yo
Its given R ¾ ¾ ¾ ¾ ¾ ¾ ¾ ¾ ¾¾® S + CH3COOH
oo
(iii) H 3O+
(iv) NaNO2 , HCl / 273-278 K
(v) EtOH, D
eB

Br Br
O O NH2 N2Cl Br
NH2 HN C CH3 HN C CH3
r

NaNO2/HCl CuBr/HBr
NO2 NO2 NO2
ou

273-278 K (Sandmeyer
ad

Ac2O Br2 reaction)


(Diazotisation)
Pyridine CH3COOH
Y

Br Br Br
(R )
Br + –
30. NH2 N2Cl
nd
Re

NaNO2 –HCl b-naphthol


H3 O+
–CH3COOH 0°C NaOH
(Hydrolysis)
Fi

V N==N––Ph
+ –
N2Cl NH2 OH
NO2 NO2 NO2
EtOH NaNO2/HCl
D 273-278K W
(Diazotisation)
31. PLAN This problem includes concept of acetylation reaction and
Br Br Br regioselectivity of chemical reaction.
(S)
Regioselectivity means which group will react selectivity in the
Now from S to major products its given. presence of two or more than two functional groups. Here,
(i) Sn / HCl among two functional group ¾ NH2 and ¾ CONH2,
(ii) Br 2 / H2O (Excess)
S ¾ ¾ ¾ ¾ ¾ ¾ ¾ ¾¾® Major product NH2 is more nucleophilic, hence NH 2 group will undergo
(iii) NaNO 2 , HCl / 273-278K reaction faster than CONH2.
(iv) H3PO 2
424 Aromatic Compounds Containing Nitrogen

CH2—NH2 O O Chloride salt gives white precipitate of AgCl on treatment with


CH2Cl2
AgNO3. Hydrazone formation occur effectively at pH = 4.5. The
+ CH3—C—O—C—CH3
O
reaction proceeds in that condition only when H+ concentration
is just sufficient to activate the following enolisation.
C—NH2
CH2—NH—C—CH3 O OH
O ½½ H+ ½
¾ C— l ¾ C+
½
C—NH2 As H+ concentration rises sufficiently, a large number of
molecules of hydrazine gets converted into hydrazinium ion
O
which is not nucleophilic and reaction becomes impossible.
¾ CH2 ¾ NH2 is more nucleophilic than ¾ C ¾ NH2. Further low concentration of H+ (in the case of
||

w
2,4-dinitrophenyl hydrazinium bromide) is not effective to
O
proceed elimination.
Hence, correct choice is (a).
+ 37. Zwitter ionic
32. (C6H5 NH3Cl – ) + AgNO3 ¾® AgCl ¯ H3N—
+
—SO3

Flo
anilinium hydrochloride precipitate
No such precipitate is formed with p-chloroaniline. sulphanilic acid
Also, carbylamine test will not be given by anilinium

ree
hydrochloride but p-chloroaniline give this test. 38. Aniline It is a stronger base than either phenol or nitrobenzene.
33. Nitro group withdraws electrons more from ortho/para position NO2 NO2 NO2

F
than from meta position. Also the s-complex formed from meta
attack is less destabilised than from ortho/para positions. HNO3 NH4HS
39.

or
NO2 NO2
ur NO2 H2SO4
NO2 NH2
heat

f
+ +
+ Br+ H H NO2 NO2
ks
Br Br
Yo
NaNO2 NaOH
oo
Nitro group not destabilising the positive charge by resonance.
HCl/0°C + H2O
NO2 NO2 NO2 N2Cl– OH
eB

40. II is more acidic due to - I effect of F.


+
+
+ Br COOH COOH
+
r

HNO3 Sn-HCl
ou
ad

Br H Br H 41.
H2SO4
Nitro group destabilises the positive charge directly by NO2
Y

resonance, less stable s-complex.


Similar phenomenon is observed with ortho attack. COOH COOH
nd
Re

0 °C
34. C6H5NH2 + NaNO2 / HCl ¾® C6H5N2+ Cl – NaNO2
Fi

OH HBF4, heat
b-naphthol F NH2

N==N—C6H5 COOH COOH


coloured dye

35. In strongly acidic medium, aniline is fully protonated, becomes 42. +


deactivated for SE Ar reaction.
+
NH2 NH3 OH NH2
+
+ H
dissolve in diethyl ether + –
HOOC— —NH3Cl(aq)
Lone pair on nitrogen is not available for resonance. Positive aq HCl
charge makes the group strongly electron withdrawing.
36. Sodium fusion extract gives Prussian blue colouration, nitrogen HO— —COOH
and carbon both present in the compound. Phenolic group and (ether layer)
salt of carboxylic acid gives FeCl 3 test.
Aromatic Compounds Containing Nitrogen 425

COOH

FeCl3(aq)
violet colouration

OH
NH2 NC

+ CHCl3 + KOH

w
COOH COOK
foul smell
O O

Flo
C C CH2CH2CH2CH3
CH2CH2CH3 Cl2 CH2CH2CH3A Na-Hg
43. FeCl3 HCl

ree
Cl Cl Cl
Cl Cl

F
NO2 (A) (B)

CH2CH2CH2CH3 NO2 NO2


HNO3 H2C==CH—CH2ONa

or
H2SO4
ur H2/Pd/C

f
Cl
Cl Cl Cl
ks
(C) O—CH2CH2CH3
O—CH2—CH==CH2
Yo
(D) (E)
oo

O
eB

Br2
44. N__ Fe N__ __
Br
r
ou
ad

2 NaNO CuCN
4 LiAlH
45. C6H5NH2 ¾¾¾® C6H5N2+ Cl – ¾¾® C6H5 — CN ¾¾¾® C6H5 — CH2NH2
Y

HCl/0 °C
NO2
O
nd

NO2
Re

H+ NaNO2 + NaNO2(aq) I2
46. —NH—C—CH3 —NH2 —N2Cl–
H2O HCl/0°C Heat Fe
Fi

I I
O O O

NH—C—CH3 NH—C—CH3 NH—C—CH3


Br2
47. +
Fe
Br Br
48. C6H5NH2 + CHCl 3 + KOH ¾® C6H5NC
Isocyanide
(foul smell)
+
NH2 N2Cl–
NO2 NO2
(i) NaNO2/HCl Anisole
49. N N OCH3
0°C
NO2 NO2
O2N NO2
426 Aromatic Compounds Containing Nitrogen

NO2 Zn-HCl NH2 NaNO CuCN


CN H+/H O COOH NH CONH2
2 2 3
50.
HCl/0°C D

Benzamide

CH3 CH3 CHO

AlCl3 Conc. HNO3 CrO3 H+


51. + CH3Cl
Conc. H2SO4 (CH3CO)2O

NO2 NO2

w
Br Br

Fe HNO3
52. + Br2

Flo
H2SO4

ree
NO2
53. No reaction. Tertiary amine does not react with nitrous acid.

F
NH2 NH2 N2Cl– Br
Br Br NaNO Br Br CuBr Br Br Zn—HCl

or
54. + Br2 (excess)
Fe
ur 2

HCl/0°C

f
ks
NO2 NO2 NO2 NO2
Yo
Br Br
oo

Br Br NaNO2 H3PO2
Br Br
eB

HCl/0°C
r

NH2
ou

O
ad

Base
55. NH2 + COCl NH—C—
Y
nd

NaNO2 /HCl CuCl


Re

56. C6H5 — NH2 ¾¾¾¾® C6H5N2+ Cl – ¾® C6H5 — Cl


0 °C Heat
Fi

Zn – HCl
57. C6H6 + Conc. HNO3 / Conc.H2SO4 ¾® C6H5 — NO2 ¾¾¾® C6H5 — NH2
Aniline
HCl CuCl
58. C6H5NH2 + NaNO2 ¾® C6H5N2+ Cl – ¾® C6H5 — Cl
0 °C Heat

Download Chapter Test


http://tinyurl.com/y437rg2e or
30
Aryl Halides and Phenols

w
Objective Questions I (Only one correct option) OH
NH2
1 What will be the major product when m-cresol is reacted with
(a) (b)
propargyl bromide (HC ººC ¾ CH 2Br ) in presence of

Flo
K 2CO3 in acetone? (2019 Main, 12 April II)
OH
O OH
NH2

ree
(a) (b) (c) (d)

F
CH3 CH3
5. Which of the following compounds reacts with ethyl
OH OH magnesium bromide and also decolourises bromine water
solution

or
(2019 Main, 11 Jan II)
(c) (d)
ur CN OCH3

f
H3C H3C CH2—CO2CH3 CH
ks
(a) (b) CH2
Yo
2. Increasing rate of S N1reaction in the following compounds is
OH
oo
I CN O
I
MeO
eB

(c) (d)
(A) (B)

I
6. Which of the following compounds will produce a
r

precipitate with AgNO3 ? (2019 Main, 11 Jan I)


ou

I
ad

(C) (D)
Br Br Br
H3 C H3CO Br
Y

(2019 Main, 10 April I)


(a) (b) (c) (d)
(a) (A) < (B) < (C) < (D) (b) (B) < (A) < (C) < (D)
(c) (A) < (B) < (D) < (C) (d) (B) < (A) < (D) < (C) N
nd
Re

3. p-hydroxybenzophenone upon reaction with bromine in carbon 7. The major product of the following reaction is
tetrachloride gives (2019 Main, 9 April II) Cl
Fi

O (i) HBr
O
Br (ii) alc.KOH
Br (b) O (2019 Main, 11 Jan I)
(a)
HO Cl Cl
HO OH
O Br O
Br (a) (b) (c) (d)
(c) (d)
O O O
HO HO OH

4. The organic compound that gives following qualitative analysis 8. The major product of the following reaction is
is (2019 Main, 9 April I) OH
Test Inference Br2 (excess)
(i) Dil. HCl Insoluble
(ii) NaOH solution Soluble
SO3H (2019 Main, 11 Jan I)
(iii) Br2 /water Decolourisation
428 Aryl Halides and Phenols

OH OH O CH3
OH O
Br
(a) (b) (a) and CH3—OH (b) H3 C CH3
and
Br Br
Br SO3H
OH OH
OH OH O
Br Br Br Br H3 C CH3
(c) (d) (c) and H3C CH3 (d) and

Br SO3H 13. The major product of the following reaction is

w
9. The major product of the following reaction is (i) KOH (aq.)
CH3 Br
(ii) CrO3/H+
OH (iii) H2SO4/D
(i) aq. NaOH Br (2019 Main, 9 Jan I)

Flo
(ii) CH3I O O
(2019 Main, 10 Jan II)
CH3 CH3 (a) (b)

ree
OH OH
(a) Br
(b) HO
O
CH3 O

F
CH3
CH3 CH3 (c) (d)

OCH3 OH

or
HO
(c) (d)
ur Br

f
14 The major product of the following reaction is
CH3
ks
(i) Br2
Yo
10. The increasing order of the pK a values of the following (ii) EtOH
(2019 Main, 9 Jan I)
oo
compounds is (2019 Main, 10 Jan I) Br OEt
OH OH OH OH OEt
eB

(a) (b)

NO2 OEt OEt


r

NO2 OMe OEt Br


ou
ad

A B C D (c) (d)
(a) D < A < C < B (b) B < C < A < D
Y

(c) C < B < A < D (d) B < C < D < A 15. Phenol on treatment with CO2 in the presence of NaOH
11. The major product of the following reaction is followed by acidification produces compound X as the major
nd

product. X on treatment with (CH3CO)2 O in the presence of


Re

HO
CH3 catalytic amount of H2SO4 produces: (2018 Main)
AlCl3, D
Fi

O +
O
(2019 Main, 9 Jan II) O
O CH3
OH OH
O CH3
H3 C CH3
(a) (b)
(a) (b)
CO2H
CO2H
OH
OH O CO2H
CH3 O CH3
CH3 C
(c) (d) O
(c) OH (d)

CO2H
O CH3
12. The products formed in the reaction of cumene with O2
followed by treatment with dil. HCl are (2019 Main, 9 Jan II) O
Aryl Halides and Phenols 429

16. Phenol reacts with methyl chloroformate in the presence of 19. OH


NaOH to form product A. A reacts with Br 2 to form product

B. A and B are respectively (2018 Main) OC2H5
+ C2H5I
anhy. C2H5OH
OH Br OH (2003, 1M)
(a) and (a) C6H5OC2H5 (b) C2H5OC2H5
OCH3 OCH3 (c) C6H5OC6H5 (d) C6H5I

O O 20. In the reaction of p-chlorotoluene with KNH2 in liq. NH3 , the


major product is (1997, 1M)
O O O O (a) o-toluidine (b) m-toluidine
(b) and (c) p-toluidine (d) p-chloroaniline
O O

w
21. Phenol reacts with bromine in carbon disulphide at low
Br
temperature to give (1988, 1M)
(a) m-bromophenol (b) o- and p-bromophenol
O O O O
(c) p- bromophenol (d) 2, 4, 6-tribromophenol

Flo
(c) and
O O 22. When phenol is treated with excess of bromine water, it gives
Br (1984, 1M)
(a) m-bromophenol (b) o-and p-bromophenol

ree
OH OH (c) 2, 4-dibromophenol (d) 2, 4, 6-tribromophenol
(d) and

F
OCH3 OCH3 Objective Questions II
O Br O (One or more than one correct option)

or
ur 23. The reactivity of compound Z with different halogens under
17. For the identification of b-naphthol using dye test, it is necessary appropriate conditions is given below

f
to use (2014 Adv.) monohalo substituted derivative
ks
(a) dichloromethane solution of b-naphthol OH when X2 = I2
acidic solution of b-naphthol X2
Yo
(b) dihalo substituted derivative
oo
(c) neutral solution of b -naphthol when X2 = Br2
(d) alkaline solution of b-naphthol trihalo substituted derivative
C(CH3)3
eB

18. The major product of the following reaction is Z when X2 = Cl 2


H3C Br The observed pattern of electrophilic substitution can be explain by
r

(a) the steric effect on the halogen


F Å
ou

(b) the steric effect of the tert-butyl group


ad

PhS Na
dimethyl formamide (c) the electronic effect of the phenolic group
Y

(d) the electronic effect of the tert-butyl group

NO2 24. The major product(s) of the following reaction is/are


(2008, 3M)
(2013 Adv.)
nd
Re

H3C SPh H3C SPh OH


Fi

F F Aqueous Br2 (3.0 equivalents)


(a) (b) OH OH

SO3H Br Br Br Br

NO2 NO2 Br
SO3H Br
(P) (Q )
H3C Br H3C SPh
OH OH
SPh SPh Br
(c) (d)
Br Br Br Br
Br SO3H
NO2 NO2 (R) (S)

(a) P (b) Q (c) R (d) S


430 Aryl Halides and Phenols

25. In the following reaction, the product (s) formed is/are 28. When phenol is reacted with CHCl 3 and NaOH followed by
OH acidification, salicylaldehyde is formed. Which of the following
species are involved in the above mentioned reaction as
CHCl3 intermediates ? (1995, 2M)
?
OH– O OH
(2013 Adv.)
H CHCl2
CH3 –
(a) CCl2 (b)
OH O
OHC CHO
O O–
H CHCl2
H3C CHCl2

w
CH3 (Q ) (c) CHCl (d)
(P)
OH
OH OH

Flo
CHO 29. Aryl halides are less reactive towards nucleophilic substitution
reaction as compared to alkyl halide due to (1990, 1M)
(a) the formation of less stable carbonium ion

ree
H3C CHCl2 (b) resonance stabilisation
(R) CH3 (c) longer carbon halogen bond
( S) (d) sp2-hybridised carbon bonded to halogen

F
(a) P (major) (b) Q (minor) (c) R (minor) (d) S (major)
OH Assertion and Reason

or
ur Read the following questions and answer as per the direction
NaOH(aq)/Br2

f
26. In the reaction, given below :
(a) Statement I is correct; Statement II is correct Statement II is
ks
the correct explanation of Statement I
Yo
the intermediate(s) is/are (2010)
(b) Statement I is correct; Statement II is correct Statement II is
oo
s O
O
not the correct explanation of Statement I
eB

Br (c) Statement I is correct; Statement II is incorrect


(a) (b) (d) Statement I is incorrect; Statement II is correct

Br Br 30. Statement I Bromobenzene, upon reaction with Br2/Fe gives


r

Br 1,4-dibromobenzene as the major product.


ou
ad

O
s
O
s Statement II In bromobenzene, the inductive effect of the
bromo group is more dominant than the mesomeric effect in
Y

directing the incoming electrophile. (2008, 3M)


(c) (d)
31. Statement I Phenol is more reactive than benzene towards
nd

Br
Re

electrophilic substitution reaction.


Br Statement II In the case of phenol, the intermediate
Fi

carbocation is more resonance stabilised. (2000, M)


27. The ether O—CH2 ,when treated 32. Statement I Benzonitrile is prepared by the reaction of
chlorobenzene with potassium cyanide.
with HI produces (1999, 3M)
Statement II Cyanide (CN– ) is a strong nucleophile.
(1998, 2M)
(a) CH2I (b) CH2OH
33. Statement I Aryl halides undergo nucleophilic substitution
with ease.
Statement II The carbon halogen bond in aryl halides has
(c) I (d) OH partial double bond character. (1991, 2M)
Aryl Halides and Phenols 431

Passage Based Problems Subjective Questions


Passage 1 42. Carry out the following conversions.
Reimer-Tiemann reaction introduces an aldehyde group, on to the (i) Phenol to aspirin
aromatic ring of phenol, ortho to the hydroxyl group. This reaction (ii) Benzoic acid to meta-fluorobenzoic acid in not more than
involves electrophilic aromatic substitution. This is a general method three steps. (2003)
for the synthesis of substituted salicylaldehydes as depicted below.
43. How would you synthesise 4-methoxyphenol from
OH ONa OH bromobenzene in not more than five steps? State clearly the
½ ½ CHO ½ CHO reagents used in each step and show the structures of the
[I ] aq. HCl intermediate compounds in your synthetic scheme. (2001, 5M)
44. What would be the major product in the following reaction?

w
½ ½ ½ F
CH3 CH3 CH3
(I) (II) (III)
NaOCH3

Flo
34. The structure of the intermediate I is D

ONa ONa

ree
NO2
CH2Cl CHCl2
(a) (b) 45. Explain briefly the formation of the products giving the

F
structures of the intermediates.

or
CH3 CH3
ur NaNH2
ONa ONa —OMe —OMe

f
NH3
CCl3 CH2OH

¾
ks ¾
(c) (d) Br
H 2N
Yo
oo
46. Complete the following, giving the structures of the principal
organic products
CH3 CH3
eB

35. The electrophile in this reaction is heat


Me I + Cu ----
· + · ·
(a) · CHCl (b) CHCl 2 (c) · CCl 2 (d) CCl 3
r
ou
ad

36. Which one of the following reagents is used in the above 47. How will you prepare m-bromoiodobenzene from benzene
reaction ? (in not more than 5-7 steps) ? (1996, 2M)
Y

(a) aq NaOH + CH3Cl (b) aq NaOH + CH2Cl 2


(c) aq NaOH + CHCl 3 (d) aq NaOH + CCl 4 48. Explain the following in one or two sentences only :
“Phenol is an acid, but it does not react with sodium
nd
Re

Integer Type Questions bicarbonate.” (1987, 1M)

37. The number of resonance structures for N is 49. Complete the following with appropriate structures :
Fi

(2015, Adv.)
OH OH OH
NaOH
N CHO
?

Fill in the Blanks (1986, 1M)


38. Amongst the three isomers of nitrophenol, the one that is least 50. A compound of molecular formula C7 H8 O is insoluble in water
soluble in water is ……… (1992, 1M)
and dilute sodium bicarbonate but dissolve in dilute NaOH
39. Phenol is acidic due to resonance stabilisation of its conjugate solution and gives a characteristic colour with FeCl 3. On
base, namely …… (1990, 1M) treatment with bromine water, it readily gives a precipitate of
C7 H5OBr3. Write down the structure of the compound.
40. Formation of phenol from chlorobenzene is an example of (1985, 2M)
……… aromatic substitution. (1989, 1M)
51. Give reason in one or two sentences for the following :
41. The acidity of phenol is due to the ……… of its anion. “o-nitrophenol is steam volatile whereas p-nitrophenol is not.”
(1984, 1M) (1985, 1M)
Answers
1. (a) 2. (b) 3. (c) 4. (b) 25. (b,d) 26. (b,c) 27. (a, d) 28. (a,d)
5. (c) 6. (a) 7. (c) 8. (c) 29. (b,d) 30. (c) 31. (a) 32. (d)
9. (c) 10. (b) 11. (c) 12. (b) 33. (d) 34. (b) 35. (c) 36. (c)
13. (a) 14. (d) 15. (a) 16. (c) 37. (9) 38. ortho-nitrophenol
17. (d) 18. (a) 19. (a) 20. (b) 39. phenoxide ion 40. nucleophilic 41. phenoxide ion
21. (c) 22. (d) 23. (a,b,c) 24. (b)

Hints & Solutions

w
1 The major product when m-cresol reacts with propargyl bromide I CH3
(HC ºº C ¾ CH2Br) in presence of K 2CO3 in acetone is given r Þ Highly

Flo
in the following reaction: –I
s
stable
rds
OH O– CH3O CH3¾O
(D)

ree
Step 2
Step 1 Additional
1. K2CO3 Br CH2 C CH +R effect from
–Brs p-position

F
(DN)
O CH2 C CH
3. p-hydroxy benzophenone upon reaction with bromine in carbon

or
ur tetrachloride gives 3-bromo-4-hydroxy benzophenone.
O O

Major product

f Br
ks
Br2

In step 1 K 2CO3 act as a base and abstract H-atom from ¾ OH CCl4


Yo
HO
HO
oo
group. This leads to the formation of substituted phenoxide ion
(highly stable). ¾ OH group attached on the benzene ring direct the incoming
eB

In step 2 substituted phenoxide ion on reaction with group at ortho and para-positions due to increase in electron
Br ¾ CH2 ¾ C ºº CH gives the required product. density at o and p-positions. ¾ OH group also exhibit –I group
2. More stable the carbocation intermediate, higher will be the rate that reduces the electron density to some extent at o and
p-positions. But overall electron density increases at these
r

of S N1 reaction.
positions of the ring due to resonance. Hence, attack of —Br
ou

The reaction involving carbocation intermediate formation for


ad

occur at ortho position. Resonating structures are as follows:


the given compounds are as follows:
Y

OH +OH +OH
I CH3 Three positive
hyperconjugation
r
2°-benzyl carbocation s
nd

–I s
Re

Þ Stable
rds
s
Fi

(A)
+R
+OH OH
(AN)
I CH3
s
MeO
s
MeO r Þ Less
–I stable
rds

(B) –I effect
from m-position
4. Phenol (ArOH) is insoluble in dil. HCl and readily soluble in
(BN) NaOH solution. It reacts with Br2/water to give 2, 4,
6-tribromophenol. It readily decolourises the yellow colour of
I CH3
Br2 water. Reactions involved are as follows :
r Þ More stable OH
s
–I
rds
H 3C H3 C Dil. HCl
(C ) · no reaction
Additional
three hyperconjugation
Phenol
( C ¢)
(Acidic in nature)
Aryl Halides and Phenols 433

– +
OH ONa
(b) —Br + Br

NaOH Stable carbocation


· (Aromatic, 6p system)

Phenol Sodium (c) —Br + Br


(Acidic in nature) phenoxide
(Unstable carbocation)
OH OH
(+R-effect)
Br Br (d) —Br + Br
Br2 Water N N
· Unstable
(Aryl carbocation)
Phenol Br
Br

w
White ppts.
(2, 4, 6-tribromophenol) So, only produces a precipitate of AgBr with AgNO3 solution.

5. Ethyl magnesium bromide is a Grignard reagent (GR), it


Å 7. In presence of HBr, reactant containing C==C undergoes

Flo
constitutes C2H5– [ C2H5– M gBr in ether/aprotic medium] which electrophilic addition reaction and give substituted alkyl halide.
can act as nucleophile as well as strong base. Bromine water On further reaction with alc. KOH, a ,b-elimination takes place
that give corresponding diene. The diene undergoes enolisation

ree
(Br2/H2O, red) gets decolourised with phenol derivatives
(option, c), anisole derivatives (option, b) etc., as C==C is to give stable product (phenol).
present outside the ring (aliphatic, not aromatic).

F
Cl –I effect at a-position to C==C Cl H
d+ d–
CºN Et group can react HBr
—Br Acidic
b–H

or
(a) CH2—C—OMe
d+ d–
ur O Carbocation formation in the O H
electrophilic addition of HBr

f
Od– –HCl Alc. KOH
–I-effect at b-position to C==C
–HBr (a, b–elimination)
ks
(Br2/H2O does not react)
Yo
OCH3 Br Enolisation to H Conjugated
oo
H attain aromaticity H diene, stable
OCH3 Br2/H2O
H OH O
eB

(excess) Br [Very stable, phenol]


(b)
EtMgBr OH
No reaction
r

OH Br 8. In ipso-substitution takes place with the carbon bearing


ou
ad

Acidic 'H' H
Br2/H2O
H
Y

(excess) SO3H
Br
OH ¾SO 3H group. After the attack of the electrophilic Br + in the
rate determining step (rds) of the ArS E2 pathway
nd
Re

(c) O Mg Br desulphonation ( ¾SO 3 ) takes place with a faster rate.


Et MgBr (+R) O—H OH
O—H
Fi

+ C2H6
[Et acts as (rds) (fast)
a base] Br2/H2O –SO3
d+ d– d– (Br ) –H
NºC O
Br—Br O S Br
SO3H Br
d+ —Br
(d) O O
H
OH
Et group can react but (Br2/H2O) does not react (i) 2 moles of Br2
Br Br
-
6. Only ionic halides (X ) give precipitate of AgX with AgNO3 (ii) Attack at ortho- 1
and ortho-2 by Br+
solution. So, an organic bromide able to produce R Å (stable (iii) –2HBr
carbocation) and Br - in aqueous solution will give precipitate of Br
AgBr with AgNO3. A white ppt. of
2,4,6-tribromophenol
(+ R-effect)
(a) —Br + Br 9. Substituted phenols react with aq.NaOH to form sodium
phenoxides which on reaction with CH3 I undergoes S N 2
(Bond energy Unstable
also increases) (Aryl carbocation) reaction to give 2-methoxy-1-methyl benzene.
434 Aryl Halides and Phenols

CH3 CH3 CH3 12 The given process is cumene process (Hock process) to
s r
OH O Na d+ d– OCH3 synthesise phenol and acetone industrially.
NaOH (aq) CH3 I In Hock p rocess, Ph — group migrate and release H2O.
Acid-base reaction SN2 reaction
–H2O Å
–NaI 2-methoxy-1- + H
—O—O—H —O—O O
methyl benzene
H
10. Acidic strength is inversely proportional to pK a value. The O2 hn H+ Ph-group
5 atm migration
acidity of phenols is due to greater resonance stabilisation of
phenoxide ion relative to phenol. Therefore, any substituent Cumene H 2O
which stabilises the phenoxide ion more by dispersal of negative OH O
charge will tend to increase the acidity of phenol. Electron
OH
withdrawing groups ( ¾ NO2 ) increases the acidic strength of Hydrolysis

w
+
phenol whereas electron donating group ( ¾ OCH3 ) decreases
the acidic strength of phenol. In case of ¾ NO2 group attached
to phenol, the dispersal of negative charge is more pronounced 13. Key Idea The reaction involves hydrolysis or nucleophilic
at o- and p-position than at m-position. substitution in first step followed by oxidation and dehydration

Flo
Thus, order of acidic strength of nitrophenol is: in last step. The most important fact is that, the Br group
p-nitrophenol > o-nitrophenol and the correct order of the pK a attached directly to aromatic ring will not undergo substitution

ree
values of give option is in step 1.
OH OH OH OH The road map of the given reaction is as follows:

F
< < < aq. KOH CrO3/H+
Br OH
SN2
NO2

or
NO2
ur OMe
Br Br
It is aryl halide, remains unaffected

f
(B) (C) (A ) (D) under ordinary SN2 reaction conditions
ks
O
11. It is an aromatic electrophilic substitution reaction (ArS E 2).
O
Yo
The reaction follows Ar S E2 (Aromatic electrophlic H OH H2SO4/D
oo
–H2O
substitution pathway) as shown below :4
(Intramolecular
Br
eB

dehydration) Br
it makes ring activating and p-directing
It becomes ring activating 14 The road map of the given reaction is :
and m-directing
Br
Br
r

Isolated C=C
H—O HO—AlCl3 HO—AlCl3 *
ou
ad

Br2 /CCl4
*
H3C Anti-addition (electrophilic)
AlCl3 –H /+H of Br2 with the isolated
Y

Friedel-Craft's O (non-aromatic) C==C


EtOH (Polar protic
acetylation – medium as well
SN1,–Br as nucleophile)
O reaction
nd

2°-benzyl
Re

O OEt carbocation (Stable)


O Stabilises the complex by Å
* Br Br
hyperconjugation
Fi

EtOH
* Å
*
–H
Racemised ± as well as
O O O—AlCl3
inverted (+ and –)

-AlCl3 OH
O 15. Followed by
O –H2O O + CO2 + NaOH acidification X
H (CH3CO)2O
conc. H2SO4
HO (Catalytic amount)
CH3 H2O
?
OH O
The very first reaction in the above road map looks like Kolbe’s
reaction which results to salicylic acid as
O O OH OH
(i) CO2, NaOH
H (ii) Acidification
COOH
Salicylic acid
(X)
Aryl Halides and Phenols 435

The salicylic acid with acetic anhydride [(CH3CO)2 O] in the 17. PLAN This problem can be solved by using the concept of synthesis of
presence of catalytic amount of conc. H2SO4 undergoes dye using electrophilic aromatic substitution reaction.
acylation to produce aspirin as In basic (alkaline) solution naphthol exists as naphthoxide ion
O which is a strong o, p-directing group.

CH3 C OH O
O O KOH
OH + CH3 C
OCCH3
Conc.
O H2SO4
+ CH3COOH III
COOH -d -d
COOH O
-d
Acetyl salicylic
acid (Aspirin) -d

w
Aspirin is a non-narcotic analgesic (Pain killer). Thus, formation of dye can be shown as
16. Given, N N¾Ph
OH OH
OH [Ph—N N]Cl–

Flo
NaOH Alkaline solution
+ Methyl A
chloroformate Br2
Thus, (d) is the correct choice.

ree
B
H 3C Br H 3C SPh
In the above road map, first reaction appears as acid base

F
reaction followed by SN AE (Nucleophilic substitution through
Addition and Elimination). Both the steps are shown below 18. F F
PhS–
(i) Acid base reaction

or
DMF
OH
ur O–

f
OH
+ H 2O NO2 NO2
ks
S N 2 reaction bring about inversion of configuration.
(ii) S N AE
Yo
19. C6H5OH + C2H5O– ¾® C6H5O–
oo
O
O C2H5I
O– ¾¾® C6H5— OC2H5
eB

O C O CH3
Cl C OCH3

CH3 CH3
r

In the product of SN AE the attached group is ortho and liq. NH3


ou

para-directing due to following cross conjugation


ad

20. + KNH2 (Benzyne mechanism)


O NH2
Y

d–
6 O1 C O 2 CH3 Cl
5 1
OH
nd

4 2 OH
Re

d– 3 d– CS2
21. + Br2
Cross conjugation due to which lone pair of oxygen 1 will be 1000 temp.
Fi

Br
easily available to ring resulting to higher electron density at 2,
4, 6 position with respect to group. However from the stability In carbon disulphide, no phenoxide ion exist, therefore only
point of view ortho positions are not preferred by substituents as monobromination takes place.
group ¾ O ¾ C ¾ O ¾ CH3 is bulky. OH OH
||
O Br Br
Hence, on further bromination of SN AE product para bromo 22. + Br2–H2O
derivative will be the preferred product i.e.
O C O CH3 O C O CH3 Br
precipitate
O O
+ Br2 + HBr Br2 + H2O ¾® HBrO + HBr
It is a reversible reaction, but equilibrium is significantly shifted
Br to left, also indicated as Br2 (aq).
436 Aryl Halides and Phenols

23. PLAN This problem includes concept of effect of steric and electronic
Os OH
effect on reactivity of organic compounds. O
Steric effect of halogens are as follows Cl 2 < Br2 < I2 H CHCl2
s +
Electronic effect of phenolic group directs the approaching + : CCl2 CCl2 +H +
electrophile towards ortho and para positions. Tertiary butyl –H
group has large size so it causes steric effect around aromatic
nucleus. On the basis of above factors the products of the given CH3 CH3 CH3
reactions are as follows: H
OH OH O O
I
X2 = I2 CHO C
OH– H
C(CH3)3

w
or
OH OH
X2 = Br2
Br CH3 CH3
( S)

Flo
C(CH3)3 C(CH3)3 Major as stable due to intramolecular H-bonding.
Br

ree
Os O O O
OH
X2 = Cl2 Cl Cl

F
:CCl2 H2 O

C(CH3)3
s

or
ur Cl CH3 CH3 CH3 CCl2
s
CH3 CHCl2

f
Hence, orientation in electrophilic substitution reaction is Q (minor)
decided by
ks
(a) The steric effect of the halogen Thus, (b) and (d) are correct.
Yo
(b) The steric effect of the tert-butyl group
26. — OH in phenol is ortho/para directing group.
oo
(c) The electronic effect of the phenolic group
So, (a), (b) and (c) are correct choices. O– O– O
eB

24. PLAN ¾ OH group is activating group and is o- and p-directing. Br2 s


Also, ¾ SO3H is a better leaving group and is knocked out by
Br - .
r
ou

OH OH
ad

Br Br
Br2 water Br Br
Y

(3 equivalents) Br2 water O O

Br2
nd
Re

SO3H SO3H –
ortho-attack Br Br
Fi

Br
OH OH
O O– O– O–
Br Br Br Br Br
+ SO3
r O + +
Br S —O—H
Br Br Br Br
O (major)
para-attack
27.
25. PLAN Phenolic compounds in alkaline solution react with chloroform
—O—CH2— —OH +
(CHCl 3) at a temperature lower than that of CHCl 3 to form + HI
ortho-isomer as the major product (due to greater stability
resulting from intramolecular hydrogen bonding).
– s –Cl – —CH2I
HO + H—CCl3 a H2O ¾ CCl3 CCl2
dichloro
carbene
Aryl Halides and Phenols 437

28. O– O O O O O

H
+ CCl2 CCl
– 2 (VI) (V) (IV)
dichlorocarbene (III)

O– O O O

CHCl2
VII IX (VIII)
All the above shown nine resonance structures are different.
29. + 38. Ortho-nitrophenol : Due to intramolecular H-bonding.
X X

w
O
– H Intramolecular H-bonding in
ortho-nitrophenol
+ O–
N

Flo
Above resonance makes X- a poor leaving group. Also, the O
carbon bearing X is sp2- hybridised.
39. Phenoxide ion 40. Nucleophilic

ree
30. Bromo group is deactivating due to dominance of inductive 41. Phenoxide ion :
effect over resonance effect. However, orientation is determined
OH O–

F
by mesomeric effet of —Br.
31. + r H+ +
OH OH O—H

or
+
urH H
Resonance stabilised
conjugate base

f
+ E+ E E
OH O– OH
ks
resonance stabilisation COO– COOH
Yo
NaOH H+
of intermediate carbocation 42. (i)
oo
CO2
32. Statement I is incorrect, aryl halides do not undergo
eB

Kolbe’s reaction
nucleophilic substitution reaction with ease. Cyanide ion (CN - ) OCOCH3
is a strong nucleophile. COOH
(CH3CO)2O
33. Statement I is incorrect, aryl halides do not usually undergo
r

nucleophilic substitution with ease. Statement II is correct, Heat


ou
ad

resonance introduces partial double bond character to C—X Aspirin


bond.
Y

O– O ONa COOH COOH COOH


H CHCl2
nd


Re

34. HNO3 Sn/HCl


+ CCl2 CCl2 (ii)
H2SO4
NO2 NH2
Fi

CH3 CH3 CH3 COOH


Intermediate
NaNO2/HBF4
35. Dichlorocarbene is the electrophile as shown above.
· D
36. CHCl 3 + NaOH ¾® · CCl 2 (Dichlorocarbene) F
electrophile meta-fluorobenzoic acid

37. O O Br Br OCH3
OH
NaOH
HNO3 CH3ONa Zn/HCl
43.
N (I) (II) H2SO4 Heat

NO2 NO2
438 Aryl Halides and Phenols

OCH3 OCH3 OCH3 NO2 NH2

NaNO2/HCl H2O Br2 SnCl2 NaNO2


0°C NaOH Fe HCl HCl/0°C
Br Br
NH2
+
N2Cl– OH N2+Cl– I
4-methoxyphenol
KI(aq)
44. F OCH3
CuI
CH3ONa
Br Br
m-bromoiodo-benzene
+ NaF
heat 48. Phenol is weaker acid than carbonic acid.

w
OH OH
NO2 NO2
CHO
H+

Flo
Nucleophilic aromatic substitution occur which is assisted by 49. + CHCl3 + NaOH
electron withdrawing —NO2 group from para position. H2O

Reimer-Tiemann reaction

ree
45. OCH3 OCH3 OCH3
Br 50. The compound must contain a hydroxy group on the ring with
– NH3 NH–2 – all three ortho/para positions vacant :

F
+ NH2
OH OH
NH2
Benzyne more stable Br Br

or
ur carbanion is formed
+ Br2—H2O

f
2Cu CH3 CH3
46. 2 Me— —I
ks
Heat
Br
Yo
oo
51. Intramolecular H-bonding in ortho-nitrophenol lowers its
CH3— — —CH3 +2CuI
boiling point. No such intramolecular H-bonding is possible
eB

with p-nitrophenol and rather it is associated together by


Ullmann’s reaction intermolecular H-bonding which increases the boiling point.
NO2 O
H
r

+ O–
ou
ad

47. + Conc. HNO3/conc.H2SO4 N


Y

O
ortho-nitrophenol
nd
Re

Download Chapter Test


Fi

http://tinyurl.com/y2xlovfc or
31
Aromatic Aldehydes,
Ketones and Acids

w
Flo
Objective Questions I (Only one correct option) 3. Compound A(C9 H10 O) shows positive iodoform test.
Oxidation of A with KMnO4 / KOH gives acid B (C8 H6 O4 ).
1. The major products of the following reaction are
Anhydride of B is used for the preparation of

ree
(2019 Main, 12 April I)
OH phenolphthalein. Compound A is (2019 Main, 10 April II)
O

F
(1) CHCl3/aq. NaOH
CH2 C H CH3
(2) HCHO, NaOH (conc.)

or
(a) (b)
(3) H3 O+
ur CH3
Cl

f
OH OH
ks
COOH COOH O CH3
Yo
(a) and methanol (b) and methanol CH3 CH3
oo
(c) (d)
CH3
O
eB

Cl OH
OH OH O

4. The major product Y in the following reaction is


r

OH OH
(c) (d) (2019 Main, 10 April II)
ou

and formic acid and formic acid


ad

Ph CH3
NaOCl (i) SOCl2
Y
Y

OH Cl X (ii) Aniline
O
2. The major product of the following reaction is O
nd

(2019 Main, 12 April I)


Re

Ph
N HN Ph
Fi

(i) CrO3
HO (ii) SOCl2/D O
(a) (b)
(iii) D
HO
O
NH2 NH2
O
O
(c) (d)
(a) (b) Ph
HO
HO O Ph
O
5. Major products of the following reaction are
O (2019 Main, 10 April I)
(d) CHO
(c) (i) 50% NaOH
+ HCHO
(ii) H3O+
Cl
Cl
440 Aromatic Aldehydes, Ketones and Acids

(a) CH3OH and HCO2H 9. The major product obtained in the following conversion is
COOH (2019 Main, 11 Jan II)
CH3 O
(b) CH3OH and
O
Br2 (1 eqv.)
CH2OH MeOH
(c) HCOOH and

O
CH2OH COOH O CH3 O
CH3
(d) and O
Br O OMe
(a) (b)
6. The major product of the following reaction is

w
(2019 Main, 12 Jan II) OMe Br
O O O
NaBH4 CH3 O CH3 O

Flo
EtOH Br
O O
OH O
(c) (d)

ree
(a) (b) Br
O O

F
OH OH 10. The major product of the following reaction is
(2019 Main, 11 Jan I)

or
(c) (d)
ur COCH3
(i) KMnO4/KOH,D

f
(ii) H2SO4 (dil.)
OEt CH3
ks
7. In the following reactions, the product S is COOH COCH3
Yo
H3C (a) (b)
oo
(i) O3 (i) NH3 HOOC HOOC
R S
(ii) Zn,H2O COOH COCOOH
eB

(2019 Main, 12 Jan I) (c) (d)


H3C H3C N OHC HOOC
(a) N (b)
r

11. The major product formed in the following reaction is


ou
ad

N O CH3
N
Y

(c) (d)
H3C H3C H3C H Dil. NaOH
+
nd
Re

8. The major product of the following reaction is (2019 Main, 9 Jan II)
O OH
Fi

CN
(i) DIBAL-H
O (ii) H3O+ (a) H 3C

O OH O
(2019 Main, 12 Jan I)
CHO CHO (b) H3C
(a) (b)
O O
OH O
O
CHO (c) H3C H
CH NH
(c) (d) O OH
OH OH
(d) H
OH H3C
CHO
Aromatic Aldehydes, Ketones and Acids 441

12. The major product obtained in the following reaction is 16. Which of the following reactants on reaction with
O conc. NaOH followed by acidification gives following
O lactone as the main product ? (2006, 5M)
DIBAL-H
O
COOH (2017 Main)
OH OH O

(a) CHO (b) CHO


COOCH3 COOH
COOH CHO (a) (b)
(c) (d) COOH CHO

w
CHO CHO

COOH CHO CHO COOH


13. In the following sequence of reaction, (c) (d)

Flo
KMnO SOCl H / Pd CHO COOH
Toluene ¾¾®
4
A ¾¾®
2
B ¾¾®
2
C
BaSO 4 (2015 Main) 17. 4-methyl benzene sulphonic acid reacts with sodium acetate

ree
The product C is to give (2005, 1M)
(a) C6 H5 COOH (b) C6 H5 CH3
(c) C6 H5 CH2 OH (d) C6 H5 CHO CH3 COONa

F
14. Sodium phenoxide when heated with CO2 under pressure at
125°C yields a product which on acetylation produces C. (a) ; CH3CO2H (b) ; SO3

or
ur
125 °
¾® B ¾H¾® C
ONa + CO2 ¾ ¾
+

SO3Na CH3

f
5 atm Ac2O
ks
OCOCH3 SO3H
The major product C would be (2014 Main)
Yo
oo
OCOCH3 OH (c) ; SO3 (d) ; NaOH
eB

COOH COCH3
(a) (b) CH3 CH3
Hg 2+ / H+
r

18. Ph ¾ C ºº C ¾ CH3 ¾¾¾® A , A is (2003, 1M)


ou
ad

COCH3
O
OH OCOCH3
Y

Ph Ph
(a) (b) O
COOCH3
nd

(c) (d) H3C H3C


Re

COOH
O–
Fi

15. Compound ( A ) , C8 H9 Br gives a white precipitate when Ph Ph


warmed with alcoholic AgNO 3 . Oxidation of (A) gives an
acid (B), C8 H6 O4 . (B) easily forms anhydride on heating. (c) (d) OH
Identify the compound (A). (2013 Main) H 3C H3C
CH2Br C2H5
CHO OHC
(a) (b)
Br (i) NaOH/100°C
19.
CH3 (ii) H+/H2O
CH2Br
CHO OHC
CH2Br
(c) (d) Major product (2003, 1M)

CH3
CH3
442 Aromatic Aldehydes, Ketones and Acids

COOH HOOC 22. m-chlorobenzaldehyde on reaction with conc. KOH at room


temperature gives (1991, 1M)
(a) potassium m-chlorobenzoate and m-chlorobenzyl alcohol
(a) (b) m-hydroxy benzaldehyde and m-chlorobenzyl alcohol
(c) m-chlorobenzyl alcohol and m-hydroxy benzyl alcohol
COOH HOOC (d) potassium m-chlorobenzoate and m-hydroxy benzaldehyde

CH2OH HOOC Objective Questions II


(One or more than one correct option)
(b)
23. Compound P and R upon ozonolysis produce Q and S ,
respectively. The molecular formula of Q and S is C8H 8O. Q

w
COOH HOH2C
undergoes Cannizzaro reaction but not haloform reaction,
O whereas S undergoes haloform reaction but not Cannizzaro
O reaction. (2017 Adv.)

Flo
(i) O3/CH2Cl 2 (i) O3/CH2Cl 2
(i) P ¾¾¾¾® Q (ii) R ¾¾¾¾® S
(ii) Zn/H2O (C 8H 8O) (ii) Zn/H2O (C 8H8O)
(c)

ree
The option(s) with suitable combination of P and R,
respectively, is(are)
O

F
H3C CH3
O
CH3 CH3
CH2OH HOH2C (a) and

or
ur CH3

f
CH3
(d)
ks
(b) H3C and H3C
Yo
CH2OH HOH2C
oo
CH3

20. In Cannizzaro’s reaction, the intermediate which is the best


eB

hydride donor is (1997) (c) H3C and


H H CH3
½ ½
r

H3C
(a) C6 H5 ¾ C ¾ O- (b) C6 H5 ¾ C ¾ O-
ou
ad

CH3 CH3
½ ½-
OH O (d) and
Y

H H3C CH3
½
nd

C ¾ O–
Re

24. Positive Tollen’s test is observed for


½ OH
(c) O–
Fi

O 2N H O CHO Ph
Ph O
H H
½ H O
C ¾ O– Ph Ph
½ Br
(d)
O– (a) (b) (c) (d)
MeO
25. The aldehydes which will not form Grignard product with
21. In the Cannizzaro’s reaction given below : one equivalent Grignard reagents are (2019 Main 12 Jan II)

KOH CHO CHO


2Ph ¾ CHO ¾® Ph ¾ CH2OH + PhCO–2 (b)
(a)
The slowest step is (1996, 1M) HO2C
(a) the attack of —OH at the carbonyl group CHO CHO
(b) the transfer of hydride to the carbonyl group (c) (d)
(c) the abstraction of proton from the carboxylic acid HO3CO HOH2C
(d) the deprotonation of Ph ¾ CH2 OH
Aromatic Aldehydes, Ketones and Acids 443

Passage Based Questions 29. The compound S is


Paragraph X
Treatment of benzene with CO / HCl in the presence of anhydrous
AlCl 3 / CuCl followed by reaction with Ac2 O / NaOAc gives (a) (b)
compound X as the major product. Compound X upon reaction
with Br2 / Na 2 CO3 followed by heating at 473 K with moist KOH
furnishes Y as the major product. Reaction of X with H2 / Pd - C, NH2
HN
followed by H3 PO4 treatment gives Z as the major product.
26. The compound Y is (2018 Adv.)
OH NH2 H
(c) (d)

w
N
COBr Br
(a) (b)

HO O

Flo
Passage
Br In the following reactions sequence, the compound J is an
intermediate.

ree
COBr
(c) (d) (CH3 CO) 2 O (i) H2 , Pd/C
I ¾¾¾¾¾® J ¾¾¾¾¾¾® K
Br CH3 COONa (ii) SOCl 2

F
(iii) anhyd. AlCl 3
27. The compound Z is
J (C9 H8 O2 ) gives effervescence on treatment with NaHCO3

or
(a) (b)
ur and positive Baeyer’s test. (2012)
30. The compound K , is
O O
f
ks
OH
(a) (b)
Yo
O O
oo

(c) (d)
(c) (d)
eB

O
O
O
Paragraph A
r

31. The compound I, is


An organic acid P ( C11 H12 O2 ) can easily be oxidised to a dibasic
ou
ad

acid which reacts with ethylene glycol to produce a polymer O H OH


dacron. Upon ozonolysis, P gives an aliphatic ketone as one of the
Y

products. P undergoes the following reaction sequences to furnish (a) (b)


R via Q. The compound P also undergoes another set of reactions to
nd

produce S . H
Re

(2018 Adv.)
O CH3 H
(1) H2/Pd-C H
(2) NH3/D
Fi

(1) H2/Pd-C (1) HCl


(3) Br2/NaOH (2) SOCl2 (2) Mg/Et2O (c) (d)
S P Q R
(4) CHCl3/KOH, D (3) MeMgBr,CdCl2 (3) CO2(dry ice)
(5) H2/Pd-C (4) NaBH4 (4) H3O+

28. The Compound R is Fill in the Blanks


HO2C 32. The structure of the intermediate product formed by the
oxidation of toluene with CrO3 and acetic anhydride, whose
(a) CO2H (b) hydrolysis gives benzaldehyde is …………… (1992, 2M)

True/False
33. Benzaldehyde undergoes aldol condensation in an alkaline
medium. (1982 , 1M)
CO2H CO2H
(c) (d)
444 Aromatic Aldehydes, Ketones and Acids

Subjective Questions Column 1 Column 2 Column 3


34. Five isomeric para-disubstituted aromatic compounds (I) Toluene (i) NaOH/ Br2 (P) Condensation
A to E with molecular formula C8 H8 O2 were given for (II) Acetophenone (ii) Br2/hn (Q) Carboxylation
identification.
(III) Benzaldehyde (iii) (CH3CO)2 O/ (R) Substitution
Based on the following observations, give structures of the
CH3COOK
compounds : (2002, Main, 5M)
(i) Both A and B form a silver mirror with Tollen’s reagent; also, (IV) Phenol (iv) NaOH/CO 2 (S) Haloform
B gives a positive test with FeCl 3 solution.
37. The only CORRECT combination in which the reaction
(ii) C gives positive iodoform test .
proceeds through radical mechanism is
(iii) D is readily extracted in aqueous NaHCO3 solution.
(a) (IV) (i) (Q) (b) (III) (ii) (P)
(iv) E on acid hydrolysis gives 1, 4-dihydroxy benzene.

w
(c) (II) (iii) (R) (d) (I) (ii) (R)
35. An organic compound A, C8H4O3, in dry benzene in the
38. For the synthesis of benzoic acid, the only CORRECT
presence of anhydrous AlCl3 gives compound B. The
combination is
compound B on treatment with PCl5, followed by reaction

Flo
(a) (II) (i) (S) (b) (I) (iv) (Q)
with H2/Pd/(BaSO4) gives compound C, which on reaction
(c) (IV) (ii) (P) (d) (III) (iv) (R)
with hydrazine gives a cyclised compound D (C14H10N2).
30. The only CORRECT combination that gives two different

ree
Identify A, B, C and D. Explain the formation of D from C.
(2000, 5M) carboxylic acids is
36. Explain, why o-hydroxy benzaldehyde is a liquid at room (a) (IV) (iii) (Q) (b) (II) (iv) (R)

F
temperature while p-hydroxy benzaldehyde is a high melting (c) (I) (i) (S) (d) (III) (iii) (P)
solid? (1999, 2M)

or
ur Integer Type Questions
Matching Type Questions

f
40. Among the following the number of reaction(s) that
Answer Q. 22, Q. 23 and Q. 24 by appropriately matching the produce(s) benzaldehyde is (2015 Adv.)
ks
information given in the three columns of the following table. CHCl2
Yo
Column 1, 2 and 3 contain starting materials, reaction conditions, CO, HCl H2 O
oo
I. II.
and type of reactions, respectively. (2017 Adv.) Anhydrous AlCl3 / CuCl 100°C
eB

COCl CO2Me
H2 DIBAL-H
III. IV.
Pd-BaSO4 Toluene, -78°C
H2O
r
ou
ad
Y

Answers
nd
Re

1. (d) 2. (b) 3. (c) 4. (b) 21. (b) 22. (a) 23. (b,c) 24. (a,b,c)
5. (c) 6. (a) 7. (a) 8. (d) 25. (b,d) 26. (c) 27. (a) 28. (a)
Fi

9. (b) 10. (a) 11. (c) 12. (a) 29. (b) 30. (c) 31. (a)
13. (d) 14. (a) 15. (d) 16. (c) 32. C6H5 ¾ CH(OAc)2 33. False 37. (a)
17. (a) 18. (a) 19. (b) 20. (d)
38. (d) 39. (b) 40. (4)
Hints & Solutions
1. The major products of the given reaction are as follows: Further, heating of product leads to intramolecular cyclisation.
OH OH
COCl
(1) CHCl3/aq. NaOH OH C==O
D
(2) HCHO, conc. NaOH and HCOOH
(3) H3O+
HO HO
Cl Cl
In step-I, substituted phenol undergoes Reimer-Tiemann 3. (i) C9H10O shows positive iodoform test thus, ¾ C ¾ CH3
reaction in presence of CHCl3 / aq. NaOH group is present. ||

w
OH OH
O
CHO (ii) C9H10O on strong oxidation (KMnO4 / KOH), gives acid
(1) CHCl3/aq. NaOH
(C8H6O4 ), indicating it can be a dicarboxylic acid. So, ‘A’
contains ¾COCH3 and one ¾CH3 group which get

Flo
oxidised into ¾COOH and ¾COOH respectively.
Cl Cl
(iii) In the preparation of phenolphthalein from phenol, phthalic

ree
The aldehyde obtained in above equation does not possess anhydride is used. So, ‘B’ can be phthalic acid (benzene-1,2-
a-hydrogen. In presence of formaldehyde and conc. NaOH it dicarboxylic acid) which readily forms anhydride.
undergoes Cannizaro reaction. In this reaction, one molecule of

F
Thus, the reaction sequence is as follows :
aldehyde is reduced to alcohol while another molecule is
oxidised to salt of carboxylic acid. CH3
I2/NaOH

or
OH
ur
O–Na+ CH3 (Iodoform test) – +
+ CHI3
COONa Yellow ppts.
CHO

f
Conc.
OH CH3
+ HCHO NaOH + HCOO–Na+ C O
ks
C
Yo
O
Cl Cl A(C9H10O) (i) [O] OH
oo
+ CO2
KMnO4/KOH OH
Upon hydrolysis, following reaction takes place (ii) H3O
r C
eB

O–Na+ OH O
Phthalic acid
H3O+ B(C8H6O4)
OH OH
+ HCOO–Na+
r

+ HCOOH H
D –H2O
ou
ad

(2 mol)/
Cl Cl Conc. H2SO4
O
Y

2. The major product formed in the reaction is as follows : C


OH
O
–H2O C
nd
Re

HO ==O
(1) CrO3
(2) SOCl2/D O
Fi

(3)D Phthalic anhydride


HO HO O
Primary alcohol readily oxidised to corresponding carboxylic
acid with oxidising agent, chromium trioxide (CrO3) in acidic O
medium.
COOH
HO CrO3

HO HO
OH OH
¾ OH group of carboxylic acid get substituted by ¾Cl in (Phenolphthalein
presence of SOCl2 (Thionyl chloride). an indicator)

COOH 4. NaOCl (sodium hypochlorite) is the reagent of haloform


COCl
(chloroform formation) reaction.
SOCl2/D
+ SO2­ + HCl­ 2NaOH + Cl2 ¾® NaOCl + NaCl + H 2O
HO HO
446 Aromatic Aldehydes, Ketones and Acids

The given reaction takes place as follows : 7. Key Idea Grignard reagent usually attacks on > C == O group
O O as :
( C == O + RMgX ¾®
NaOCl s r
Ph—C—CH3 Ph—C—ONa + CHCl3 '
r
H3O (It is missing in the ( C ' OMgX ¾¾¾®
Hydration ( C ' OH
Keto-methyl statement of the question)
group ' (R - Mg (OH) X ' ( R
(Ionic salt)
O
The question is related to above reaction only with the condition
Ph—C—OH that the consumption of RMgX will be more than 1 equivalent in
Benzoic acid
(X)
some of the given cases.
O O Among the given compounds B, i.e.

w
SOCl CHO CHO
2
Ph—C—OH Ph—C—Cl
– SO2 [Acyl SN2
(X) Benzoyl
– HCl pathway]
chloride H2N— HO2C and D, i.e. HOH2C
–HCl

Flo
Aniline
O contain additional groups which can give active hydrogens.
Grignard reagents produce alkanes whenever come in contact
—NH—C—Ph with any group or compound which can give active hydrogen as:

ree
Benzanilide
OR
(Y) ROH + R ¢ MgX ¾Dry
¾¾® R¢ H + Mg '
(
ether X

F
5. The given reaction is a crossed Cannizzaro reaction which is a These reactions are equivalent to acid-base reactions. So, in both
redox reaction too. Oxidation number of carbon atom of the of these compounds more than one equivalent will be required to
¾CHO groups of Ph¾CHO and H¾CHO are +1 and zero

or
form Grignard products. Remember these compounds will give
ur
respectively. So, HCHO is the stronger reducing agent than 2 type of products as:
PhCHO. As a result, HCHO is oxidised to HCOONa (by

f
(i) from the >C == O group
donation of hydride, H- ) and PhCHO (H- acceptor) is reduced
ks
to PhCH 2OH. (ii) from the group which release active hydrogen
The additional reactions involved are:
Yo
+1 0 50% NaOH
Ph ¾ C HO + HC HO ¾¾¾¾®
oo
CHO CHO
-1 +2 - + Dry
Ph ¾ C H2OH + HC OONa (i) + RMg X RH +
eB

ether
½ + HO2C C
¯ H3 O d+ d–
PhCH2OH + HCOOH X MgO
(Major products) O
r

The reaction proceed via following mechanism. CHO CHO


ou

Dry
ad

OH (ii) + RMg X + RH
ether
– | s HOH2C XMgOH2C
Y

OH Slowest
H2C=
=O Slower H—C—O
d+ d– | Ph—CH=
=O
H 8. DIBAL-H (Diisobutylaluminium hydride) is a reducing agent
nd

More polar and less with formula [l-Bu 2AlH ] . At ordinary temperatures, nitriles
Re


crowding for H shift
nucleophilic addition give imines which are readily converted in aldehydes by
OH
hydrolysis whereas lactones are reduced directly to aldehydes.
Fi

|
H—C==O + Ph—CH2—Os H
Fast
Os Al Al
Ö

(DIBAL–H) (DIBAL–H)
| Al
+ + H
H—C==O + PhCH2OH N
CN N
C
6. Reducing agents like LiAlH4, NaBH4, i.e. complex hydrides C H
O
usually does not affect olifenic or p-bonds. Thus, if NaBH4 is O
O O
O
O
O
applied to a compound like then its ( C == O bond will be
' H2O
OH
CHO CHO CHO
+H+ DIBAL–H
(Hs)
reduced only and we get as the OH O O
final product. H
Thus, option (a) is correct answer. O –O
H O
Aromatic Aldehydes, Ketones and Acids 447

OH (Cross Aldol condensation of


9. In presence of Br2/EtOH, the reactant containing double bond CH3CHO and CH3COC6H5
undergoes electrophilic addition reaction via the formation of (iii) CH3—C—C6H5 in which >C=O group of
·· C6H5COCH3 is carbanion
bromonium ion. On further attack of ¾ O Me on bromonium CH2 acceptor).
··
H—C
ion gives the addition product.
H3 C COMe O
O
OH O
Br2(1 eqiv) O MeOH
O in MeOH (Self Aldol condensation
Br (iv) C6H5—C—CH2—C—C6H5 product of C H COCH )
(Protic solvent) 6 5 3
CH3
O –
O O H of base will prefer to attack on ¾CH3 group of CH3CHO
O for the formation of carbanion and as among the > C == O groups

w
available, the > C == O group of CH3CHO is the best carbanion
O acceptor. Hence, self condensation product of CH3CHO will be
OMe the major product.
Br

Flo
O
12. DIBAL-H (Di-isobutyl aluminium hydride) is a reducing agent
(Major)
with formula. This is generally used for the preparation of
aldehydes. Using DIBAL ¾ H, Lactones are reduced directly to

ree
10. In presence of alkaline KMnO4, vigorous oxidation of alkyl or aldehydes.
O
acyl benzene takes place. During oxidation, aromatic nucleus OH

F
O
remains intact but the entire chain is oxidised to —COOH group
DIBAL–H
irrespective of the length of carbon chain. CHO

or
O==C—CH3
Oxidative cleavage
in strong
ur sÅ
O == C—O K COOH
COOH
(MnO4– /OH–/D–)

f
KMnO4/KOH/D 13. Toluene undergoes oxidation with KMnO4, forms benzoic acid.
ks
Strong oxidation, [O] In this conversion, alkyl part of toluene converts into carboxylic
Yo
sÅ group. Further, benzoic acid reacts with thionyl chloride
oo
CH3 COOK (SOCl 2) to give benzoyl chloride which upon reduction with
(also + H2/Pd or BaSO4 forms benzaldehyde (Rosenmund reduction)
oxidisable) H3O (dil. H2SO4)
eB

The conversion look like,


COOH
O Cl
CH3 COOH C
r
ou

KMnO4 SOCl2
ad

+ SO2 + HCl
COOH
Y

(Toluene) (Benzoic Acid) (Benzoyl chloride)


11. In aldol condensation, generally aldehydes react at a faster rate ‘A’ ‘B’
than ketones towards base. In the given case CH3CHO will lose H2/Pd BaSO4
nd
Re

O
O H
C C
Fi

a-hydrogen faster than CH3 due to one more

reason, i.e. conjugation between benzene ring and ‚ + HCl


ƒC == O
group. Along with sterically less hindered nucleophile of (Benzaldehyde)
CH3CHO will also add to the major product formation. ‘C ’

Following four products are possible in the reaction: 14. It is a Kolbe Schmidt reaction.
+ +
H ONa COONa COOH O
½ ½ ½
(i) CH3—C—CH2—CHO (Self Aldol condensation OH OC ¾ CH3
of CH3CHO) H+/Ac2O
+ CO2 125°C
OH 5 atm
H (Cross Aldol condensation of ‘B’ Aspirin
CH3CHO and CH3COC6H5 (pain killer)
(ii) CH3—C—CH2—C—C6H5 in which >C=O group of ‘C’
CH3CHO is carbanion
OH acceptor). The second step of the reaction is an example of acetylation
reaction.
448 Aromatic Aldehydes, Ketones and Acids

15. Compound A gives a precipitate with alcoholic AgNO3, so it 20. Dioxoanion is better hydride donor. Electron donating group at
must contains Br in side chain. On oxidation, it gives C8 H6O4, ortho/para position further promote H– transfer.
which shows the presence of two alkyl chains attached directly –
O
with the benzene nucleus. Since, compound B gives anhydride
on heating, the two alkyl substituent must occupy adjacent (1, 2) O
C—H
position. Thus, A must be
+ C—R
CH2 Br O–
H3CO H O

CH3
C
and the reactions are as follows :
CH2Br CH2 OR + O–

w
Alcoholic H3CO –
+ AgBr O
AgNO3
CH3 CH3
‘A’
+ H— C —R
Oxidation

Flo
O
COOH H
D
O -
O

ree
COOH
O
‘B’ ½½ ½
O
Phthalic anhydride
21. Ph ¾ C ¾ H + HO- ¾® Ph ¾ C ¾ H
½

F
16. CHO COOH OH
OH– I
O- O

or
CHO
ur
Intramolecular
Cannizzaro reaction CH2OH ½ ½½ Slow
Ph ¾ C ¾ H + C ¾ Ph ¾¾¾¾¾®

f
O ½ ½ hydride transfer
OH H
ks
I
H+ O-
Yo
O
O
oo
½½ ½
Ph ¾ C ¾ OH + Ph ¾ C ¾ H
Ester formation
eB

½
17. H
H3C— —SO3H + CH3COONa
Base 22. Cl Cl Cl
r

4-methyl benzene sulphonic


ou

acid
ad

H3C— —SO3Na + CH3COOH + KOH +


Y

CHO COOK CH2OH


H+ + Cannizzaro’s
18. Ph — C ºº C— CH3 ¾® Ph — C==CH— CH3 reaction
nd
Re

Stable carbocation 23.


O
Fi

O3
(b) CH3 CH CH2 CH3 CHO+H2CO
H2O ½½ Zn–H2O
(R)
¾¾® Ph — C — CH2CH3 (P) Gives Cannizzaro
but not haloform reaction
CHO OHC
O
NaOH O3
19. Zn–H2O
C CH3+H2CO
(S)
(Q ) Gives haloform reaction
CHO OHC but not Cannizzaro reaction
COOH HOH2C
O3
+ (c) CH CH CH3 CHO+CH3CHO
H Zn–H2O
(R)
H2 O P
CH3 CH3
CH2OH Gives Cannizzaro
HOOC but not haloform reaction

The above reaction is an example of intramolecular Cannizzaro


reaction.
Aromatic Aldehydes, Ketones and Acids 449

- For this question we require only reaction 1 to 4 written above.


OH
24. RCHO + Ag2O ¾ ¾¾® RCOOH + 2Ag Let us explore them one by one.
(Tollen' s reagent) Silver mirror
Reaction 1 It is called formylation or Gatterman Koch reaction.
Tollen’s test is given by all aldehydes and all reducing sugars as A ¾ CHO group is introduced to benzene ring through this
glucose, fructose and a-hydroxy ketones. reaction as
CHO
OH O
CO+HCl
— CH— C —
a In anhyd.
AlCl3/CuCl
H O Benzene Benzaldehyde
having an aldehyde group gives Tollen's test. +
H The attacking electrophile isH ¾ C == O which is generated as
H

w
(i) CO + HCl q H — C — Cl
Br ½½
(c) O O
CHO +
(ii) H — C — Cl + AlCl 3 q H — C + AlCl -4

Flo
CH C
½½ ½½
OH O O
Aldehyde Reaction 2 It is Perkin condensation which results in a, b

ree
a-hydroxy ketone
give positive Tollen’s test. unsaturated acid as
CHO CH CHCOOH

F
25. O

H3C H3C CH3COO–Na+


C—H + CH3COOH
(i) O3 + (CH3CO)2O

or
180°C
(ii) Zn– H2O
ur CHO

R Cinnamic acid

f
Note Besides CH3COO- Na+, quinoline, pyridine, Na 2CO3,
H3C CHO
ks
triethylamine can also be used as bases in this reaction.
NH3 CH NH
Reaction 3 It is simple addition of bromine to unsaturated acid
Yo
formed through reaction 2.
oo
(X)

In the above reaction, NH3 prefer to attack at aliphatic aldehyde CH CHCOOH CHBr CHBr
eB

group than an less reactive aromatic aldehyde group.


Br2 COO–Na+
O OH
Na2CO3
H 3C
H 3C N—H
r

C Cinnamic acid
H
ou

Tautomerism
ad

Xs NH2 Na2CO 3 works as a base in the reaction to trap H+ to be released


– H 2O OH
Y

O CH—CH—Br
H 3C
N
nd

COO–Na+
Re

by — COOH of the reactant. is also formed


‘S’
Fi

26. Given, in the reaction as the minor product.


1 2 Reaction 4 It is decarboxylation and dehydrohalogenation of
CO+HCl Ac2O
Benzene X product produced by reaction 3 as
anhy. AlCl3/CuCl NaOAc (Major) CHBr CHBr C CH

3
Br2 + COO–Na+
C

5 Na2CO3 Moist KOH


+ Na2CO3 + KBr
d
2 /P

437 K
H

6 Moist KOH C CH
4
O

4
3P

473 K
H

Z Y Hence, Y is i.e., (c) is the correct answer.


(Major product)
450 Aromatic Aldehydes, Ketones and Acids

27. Reaction 5 The Perkin condensation product X is Thus P can be


COOH
CH CHCOOH

i.e., cinnamic acid (Ref. Q. 15)

This compound on hydrogenation with H2 in the presence of Pd CH


activated with charcoal (Pd - C ) gives
C
CH CHCOOH CH2CH2COOH
CH3 CH3

Pd-C IUPAC name : 4-(2-methyl) prop-l-enyl benzoic acid


+ H2
Now look for the reactions

w
3-Phenyl propanoic acid COOH COOH

Reaction 6 The product of reaction 5 on heating with H3PO4 Oxidation

Flo
dehydrates to give
CH2CH2COOH CH2 CH2
COOH

ree
H3PO4 C Terephthalic
O (P)
D acid

F
COOH
Z CHO
COOH
28. (a) Given, O CHO
Ozonolysis

or
C O
(i) C11H12O2 ¾Oxidation
¾ ¾¾® Dibasic acid
ur +
CHO
+ C O +
Aliphatic Glyoxal

f
(An organic acid ‘P’) CHO CHO
ketone
This indicates the presence of alkyl or alkenyl branch in P (Acetone) other oxidised
ks
along with -COOH group. (P) products
Yo
(ii) Dibassic acid produced by oxidation of P
oo
1. H 2 / Pd– C
CH2 CH2 2. SOCl 2
Given, P ¾¾¾¾¾¾¾® Q
eB

OH OH O 3. MeMgBr, CdCl 2
(Ethylene glycol) 4. NaBH 4
O C —C—O—(CH2)2—O— So,
O Cl
r

Dacron
ou
ad

COOH COOH CO
This indicates presence of benzene ring in P; as concluded from
the structure of dacron given below.
Y

H2/Pd––C SOCl2
O
nd

O C C O CH2 CH2 O MeMgBr,


Re

CdCl2
O
Fi

(P)
Dicarboxylic Ethylene Me O
acid component glycol component
Dacron H C OH C Me

Attachment of —COO group in dacron also confirm the para NaBH4


position of branch with respect to —COOH group in P. [Here, Me = CH3]
(iii) P ¾ ¾ ¾ ¾® Aliphatic ketone + other oxidised products.
Ozonolysis

( Q)
This reaction confirms the presence of multiple bonded
branch i.e., alkenyl group in P.
1. HCl
2. Mg/ Et 2O
Further, Q ¾¾¾¾¾® R
3. CO2 (Dry ice)
4. H 3O+
Aromatic Aldehydes, Ketones and Acids 451

Me Me Me In the second step, J on treatment with H2 / Pd / C undergo



H C OH H C Cl H CMg
2+
Cl
– hydrogenation at olefinic bond only as :
J + H2 / Pd ¾® C6 H5 ¾ CH2 ¾ CH2 ¾ COOH
HCl Mg The hydrogenated acid, on treatment with SOCl 2 gives acid
Et2O chloride.
(Dry ether)
C6 H5 — CH2 — CH2 — COOH + SOCl 2 ¾®
(Q) C6 H5 — CH2 — CH2 — COCl + HCl + SO2
Grignard's reagent In the final step, acid chloride formed above undergo
CO2
(Carboxylation) intramolecular Friedel-Craft acylation as:
AlCl3
Me Me
C

w
Cl O O
H C COOH H C COOMgCl ‘K’
32. CH3 CH(OAc)2
H3O+ CrO3
Mg(OH)Cl +

Flo
(Hydrolysis)
(CH3CO)2O
Intermediate

ree
CHO
H 2O
(R )
H+

F
29. Given (In connection with Q. 17)
1. H 2 / Pd– C
33. For aldol condensation, presence of at least one a-H is essential,
2. NH 3 / D

or
which is not available to benzaldehyde.
3. Br2 / NaOH
P ¾¾¾¾¾¾¾® S
ur
4. CHCl 3 / KOH, D 34. CHO

f
5. H 2 / Pd– C
ks
So,
Gives positive Tollen’s test but does not
Yo
COOH COOH CONH2
give FeCl3 test.
oo

H2/Pd–C NH3/D
CH2OH
eB

OHC— —OH Gives positive Tollen’s test


r

(P) and FeCl3 test.


ou
ad

CH3 B
O
Y

NH N C NH2 Br2/NaOH
(Hofmann
bromamide) CH3—C— —OH Gives positive iodoform
H2/Pd–C CHCl3/KOH test.
nd
Re

(Carbylamine) C
Fi

HOOC— —CH3 Can be extracted with


(S) NaHCO3.

Passage HO— —OCH==CH2 On hydrolysis give


1,4-dihydroxy benzene.
Sol for (Q. Nos. 30 to 31) The first step of reaction is
Perkin’s condensation. 35. O
CHO
CH3COONa AlCl3
+ (CH3CO)2O O +
C6H5 CH CH COOH
I
J
J being a carboxylic acid gives effervescence with NaHCO3 . O
A
Also, J has olefinic bond, it will decolourise Baeyer’s
reagent.
452 Aromatic Aldehydes, Ketones and Acids

O O Br
37. C6H5 ¾ CH3 ¾®
2
C6H5 ¾ CH2Br + HBr
hv (Free radical bromination)
PCl5

O
HOOC Cl—C || NaOH / Br2
38. C6H5 ¾ C ¾ CH3 ¾¾¾¾® C6H 5COONa + CHBr3 (P )
(i )
B O (II) Bromoform

O O
O || ||
39. C6H 5CHO+ CH3 ¾ C ¾ O ¾ C ¾ CH3
H2 N 2 H4
Pd/BaSO4 N CH COOK
¾¾¾¾¾¾® C6H5 ¾ CH == CH ¾ COOH
3

w
Perkin's condensation (Cinnamic acid)
H—C N
D
Cinnamic acid shows cis-trans isomerism.
O
C
40. I. Gattermann-Koch reaction.

Flo
36. Intramolecular H-bonding in ortho hydroxy benzaldehyde is II.
responsible for decrease in melting and boiling points. CHCl2 CH(OH)2

ree
H2O
O H 100°C (SN1)
CHO
O gem-diol

F
–H2O
H

or
ur
Intramolecular
H-bonding
III. Rosenmund’s reduction.
IV. Acid chloride, anhydride and ester undergo controlled
p-hydroxy benzaldehyde molecules are associated by
f
reduction with di-iso-butylaluminium hydride (DIBAL-H) at
ks
- 78°C to give aldehydes.
intermolecular H-bonding, has higher melting and boiling
Yo
points.
oo
eB

Download Chapter Test


http://tinyurl.com/y4tw8a24 or
r
ou
ad
Y
nd
Re
Fi
32
Biomolecules and Chemistry
in Everyday Life

w
Topic 1 Biomolecules

Flo
Objective Questions I (Only one correct option) 8. Maltose on treatment with dilute HCl gives
(2019 Main, 08 April I)

ree
1. Which of the given statements is incorrect about glycogen?
(2019 Main, 12 April II)
(a) D-glucose and D-fructose (b) D-fructose
(a) It is straight chain polymer similar to amylose (c) D-galactose (d) D-glucose

F
(b) Only a-linkages are present in the molecule 9. The correct structure of histidine in a strongly acidic
(c) It is present in animal cells solution ( pH = 2 ) is (2019 Main, 12 Jan II)

or
(d) It is present in some yeast and fungi
ur r
(a) H3N CH
s
COO
r
(b) H3N CH COOH
2. Which of the following statement is not true about RNA?

f
r r
(2019 Main, 12 April I) NH2 NH2
ks
(a) It controls the synthesis of protein
N N
(b) It has always double stranded a-helix structure
Yo
r
oo
r s
(c) It usually does not replicate (c) H3N CH COO (d) H3N CH COOH
(d) It is present in the nucleus of the cell +
eB

NH NH
3. Number of stereo-centers present in linear and cyclic
NH
structures of glucose are respectively (2019 Main, 10 April II) Nr
(a) 4 and 5 (b) 4 and 4 H
r

(c) 5 and 4 (d) 5 and 5


10. The correct match between Item I and Item II is
ou
ad

4. Amylopectin is composed of (2019 Main, 10 April I)


Item I Item II
(a) b-D-glucose, C1-C4 and C2-C6 linkages
Y

A. Ester test P. Tyr


(b) a-D-glucose, C1-C4 and C2-C6 linkages
B. Carbylamine test Q. Asp
(c) b-D-glucose, C1-C4 and C1-C6 linkages
nd

C. Phthalein dye test R. Ser


Re

(d) a-D-glucose, C1-C4 and C1 -C6linkages


S. Lys
5. The peptide that gives positive ceric ammonium nitrate and
Fi

(2019 Main, 11 Jan II)


carbylamine tests is (2019 Main, 09 April II)
(a) A ® Q; B ® S; C ® R (b) A ® R, B ® Q; C ® P
(a) Lys-Asp (b) Ser-Lys (c) A ® R; B ® S; C ® Q (d) A ® Q; B ® S; C ® P
(c) Gln-Asp (d) Asp-Gln
11 Among the following compounds, which one is found
6. Which of the following statement is not true about sucrose? in RNA? (2019 Main, 11 Jan I)
(2019 Main, 09 April I)
O O
(a) It is also named as invert sugar.
CH3
(b) The glycosidic linkage is present between C1 of NH NH
(a) (b)
a-glucose and C1 of b-fructose
N O N
(c) It is a non-reducing sugar O
H
(d) On hydrolysis, it produces glucose and fructose H
NH2 O
7. Fructose and glucose can be distinguished by
(2019 Main, 08 April II) N N Me
(c) (d)
(a) Fehling’s test (b) Barfoed’s test N N
O O
(c) Benedict’s test (d) Seliwanoff’s test
H Me
454 Biomolecules and Chemistry in Everyday Life

12. Which of the following tests cannot be used for identifying 20. Which one of the following bases is not present in DNA?
amino acids ? (2019 Main, 10 Jan II) (a) Quinoline (b) Adenine (2014 Main)
(a) Barfoed test (b) Ninhydrin test (c) Cytosine (d) Thymine
(c) Xanthoproteic test (d) Biuret test 21. Synthesis of each molecule of glucose in photosynthesis
13. The correct sequence of amino acids present in the involves (2013 Main)
tripeptide given below is (2019 Main, 9 Jan II) (a) 18 molecules of ATP (b) 10 molecules of ATP
(c) 8 molecules of ATP (d) 6 molecules of ATP
Me Me Me OH
H O 22. The following carbohydrate is
N OH
H2 N N C H OH
O H
HO H O
OH O OH
HO H HO

w
(a) Thr - Ser - Leu (b) Leu - Ser - Thr H H (2011)
(c) Val - Ser - Thr (d) Thr - Ser - Val (a) a ketohexose (b) an aldohexose
14 The increasing order of pK a of the following amino acids (c) an a-furanose (d) an a-pyranose

Flo
in] aqueous solution is Gly, Asp, Lys, Arg 23. The correct statement about the following disaccharide is
(2019 Main, 9 Jan I) CH2OH
(a) Asp < Gly < Arg < Lys (b) Arg < Lys < Gly < Asp

ree
H O H2COH O H
(c) Gly < Asp < Arg < Lys (d) Asp < Gly < Lys < Arg H
H
OH H H HO
15. Glucose on prolonged heating with HI gives (2018 Main) OCH2CH2O

F
HO CH2OH
(a) n-hexane (b) 1-hexene
H OH OH H (2010)
(c) Hexanoic acid (d) 6-iodohexanal (a) (b)

or
ur
16. The predominant form of histamine present in human blood (a) Ring (a) is pyranose with a-glycosidic link
is (pK a , Histidine = 6.0 ) Ring (a) is furanose with a-glycosidic link

f
(2018 Main) (b)
H H (c) Ring (b) is furanose with a-glycosidic link
ks
N N r (d) Ring (b) is pyranose with b-glycosidic link
NH2 NH3
Yo
(a) (b) 24. Two forms of D-glucopyranose, are called (2005, 1M)
oo
r
N N (a) enantiomers (b) anomers
H
(c) epimers (d) diastereomers
eB

H H
N NH2 N r
NH3 25. Which of the following pairs give positive Tollen’s test ?
(c) (d) (2004, 1M)
r
N N (a) Glucose, sucrose (b) Glucose, fructose
r

H
(c) Hexanal, acetophenone (d) Fructose, sucrose
ou
ad

17. Which of the following compounds will behave as a


Objective Question II
Y

reducing sugar in an aqueous KOH solution? (2017 Main)


(One or more than one correct option)
HOH2C O CH2OH HOH2C O CH2OH
26. The Fischer presentation of D-glucose is given below.
nd
Re

(a) HO (b) HO CHO


OCOCH3
Fi

H OH
OH OH HO H
HOH2C H OH
O O CH OCH H OH
HOH2C CH2OH 2 3

(c) (d) CH2OH


HO OH D-glucose
OCH3 OH
The correct structure(s) of b-L-glucopyranose is (are)
OH OH (2018 Adv.)
H H
18. Thiol group is present in (2016 Main)
(a) cystine (b) cysteine O O
HO OH HO OH
(c) methionine (d) cytosine (a) CH2OH (b) CH2OH
19. Which of the vitamins given below is water soluble? H HO H H
(a) Vitamin C (b) Vitamin D (2015 Main) H H H H
(c) Vitamin E (d) Vitamin K
OH H OH OH
Biomolecules and Chemistry in Everyday Life 455

CH2OH H
Integer Answer Type Questions
O O
H OH HO H 30. The total number of distinct naturally occurring amino acids
H (d) CH2OH
(c) obtained by complete acidic hydrolysis of the peptide
OH HO H HO shown below is (2014 Adv.)
HO H H OH
O
H H OH H H
O O H O H O N
27. For ‘invert sugar’, the correct statement(s) is (are) N
N N N
N N
(Given: specific rotations of ( + ) - sucrose, (+) - maltose,
L-( - ) -glucose and L-( + ) -fructose in aqueous solution are N CH2 O O H CH2 O
H
+ 66 °, +140 °, - 52 °and 92° , respectively) (2016 Adv.) H

w
(a) Invert sugar is prepared by acid catalysed hydrolysis O
of maltose
(b) Invert sugar is an equimolar mixture of D-(+) -glucose
31. A tetrapeptide has ¾ COOH group on alanine. This produces
glycine (Gly), valine (Val), phenyl alanine (Phe) and alanine

Flo
and D-(-)- fructose
(c) Specific rotation of invert sugar is - 20 ° (Ala), on complete hydrolysis. For this tetrapeptide, the
(d) On reaction with Br2 water, invert sugar forms number of possible sequences (primary structures) with

ree
saccharic acid as one of the products ¾ NH2 group attached to a chiral centre is (2013 Adv.)

28. The correct statement(s) about the following sugars X and Y 32. The substituents R1 and R2 for nine peptides are listed in the

F
is/are: (2009) table given below. How many of these peptides are
CH2OH
H positively charged at pH = 7.0 ? (2012)
H O O

or
H H3N ¾ CH ¾ CO ¾ NH ¾ CH ¾ CO ¾ NH ¾ CH
H
HOH2C
ur
f
OH H H HO H
CH2OH R1 R2
OH O
¾ CO ¾ NH ¾ CH ¾ COO
ks
H OH OH H
Yo
X H
oo
CH2OH
O Peptide R1 R2
eB

H H
I H H
H
CH2OH II H CH 3
OH H
H O O III CH 2COOH H
OH
r

H IV CH 2CONH 2 (CH 2 ) 4 NH 2
OH
ou

H
ad

OH H V CH 2CONH 2 CH 2CONH 2
OH H VI (CH 2 ) 4 NH 2 (CH 2 ) 4 NH 2
Y

H OH VII CH 2COOH CH 2CONH 2


Y
VIII CH 2OH (CH 2 ) 4 NH 2
(a) X is a reducing sugar and Y is a non-reducing sugar (CH 2 ) 4 NH 2 CH 3
nd

IX
Re

(b) X is a non-reducing sugar and Y is a reducing sugar


(c) The glucosidic linkages in X and Y are a and b, respectively 33. When the following aldohexose exists in its
Fi

(d) The glucosidic linkages in X and Y are b and a, respectively D-configuration, the total number of stereoisomers in its
pyranose form, is (2012)
Assertion and Reason CHO

Read the following questions and answer as per the direction CH2
given below : CHOH
(a) Statement I is correct; Statement II is correct;
Statement II is a correct explanation of Statement I. CHOH
(b) Statement I is correct; Statement II is correct; Statement CHOH
II is not the correct explanation of Statement I.
(c) Statement I is correct; Statement II is incorrect. CH2OH
(d) Statement I is incorrect; Statement II is correct. 34. A decapeptide (Molecular weight 796) on complete
29. Statement I Glucose gives a reddish-brown precipitate hydrolysis gives glycine (Molecular weight 75), alanine and
with Fehling’s solution. phenylalanine. Glycine contributes 47.0% to the total
Statement II Reaction of glucose with Fehling’s solution weight of the hydrolysed products. The number of glycine
gives CuO and gluconic acid. (2007, 3M) units present in the decapeptide is (2011)
456 Biomolecules and Chemistry in Everyday Life

35. The total number of basic groups in the following form of


(i) Draw the structure of L-glucose.
lysine is (2010)
(ii) Give the reaction of L-glucose with Tollen’s reagent.
Å
H3N—CH2—CH2—CH2—H2C O (2004, 2M)

CH—C 38. Following two amino acids lysine and glutamine form
s dipeptide linkage. What are two possible dipeptides?
H2N O (2003, 2M)
Subjective Questions H2 N ¾ CH ¾ COOH
½
36. Which of the following disaccharide will not reduce CH2 CH2 CH2 CH2 NH2
Tollen’s reagent? (2005, 2M)
CH2OH CH2OH
H2 N ¾ CH ¾ COOH
½

w
HO O O OH
H H
CH2 CH2 COOH
(a)
H HO H HO
O 39. Aspartame, an artificial sweetener, is a peptide and has the
H H
following structure

Flo
OH H OH H
CH2OH CH2OH CH2 C6H5
½

ree
OH O O OH
H H H2 N ¾ CH ¾ CONH ¾ CH ¾ COOCH3
(b) ½
H HO OH H
O CH2 ¾ COOH

F
H H
OH H H OH (i) Identify the four functional groups.
(ii) Write the Zwitter ionic structure.

or
37. The structure of D-glucose is as follows :
ur (iii) Write the structures of the amino acids obtained from

f
CHO the hydrolysis of aspartame.
H OH (iv) Which of the two amino acids is more hydrophobic?
ks
(2001, 5M)
Yo
HO H
oo
40. Give the structures of the products in the following reaction
H OH
H+
eB

Sucrose ¾® A + B (2000, 2M)


H OH
OH
41. Write the structure of alanine at pH = 2 and pH = 10.
(2000, 2M)
r
ou
ad

Topic 2 Chemistry in Everyday Life


Y

Objective Questions I (Only one correct option) 3. Which of the following is a condensation polymer?
(2019 Main, 10 April I)
1. Which of the following is a thermosetting polymer?
nd

(a) Nylon-6, 6 (b) Neoprene


Re

(2019 Main, 12 April I)


(a) Bakelite (b) Buna-N (c) Teflon (d) Buna - S
Fi

(c) Nylon-6 (d) PVC 4. Noradrenaline is a/an


2. The correct match between Item-I and Item-II is (a) antidepressant (b) antihistamine
(c) neurotransmitter (d) antacid
Item-I Item-II (2019 Main, 9 April II)
A. High density polythene I. Peroxide catalyst
5. Which of the following compounds is a constituent of the
B. Polyacrylonitrile II. Condensation at high O
temperature and pressure
½½
C. Novolac III. Ziegler-Natta catalyst
[ HN — C — NH — CH 2—
polymer — ]n ?
D. Nylon-6 IV. Acid or base catalyst
(2019 Main, 9 April II)
(2019 Main, 10 April II) (a) N-methyl urea
Codes (b) Methylamine
A B C D A B C D (c) Ammonia
(a) III I IV II (b) IV II I III (d) Formaldehyde
(c) II IV I III (d) III I II IV
Biomolecules and Chemistry in Everyday Life 457

6. The structure of nylon-6 is 12. The correct match between item (I) and item (II) is
O H O H Item - I Item - II
|| | || |
(A) Norethindrone (P) Antibiotic
(a) ¾ ]n (b) ¾
[ (CH 2 )6 ¾ C ¾ N ¾ [ C ¾ (CH 2 )5 ¾ N ¾
]n
(B) Ofloxacin (Q) Antifertility
O H O H (C) Equanil (R) Hypertension
|| | || | (S) Analgesics
(c) ¾ ]n (d) ¾
[ (CH 2 )4 ¾ C ¾ N ¾ [ C(CH 2 )6 ¾ N ¾
]n
(2019 Main, 11 Jan I)
(2019 Main, 8 April II) (a) (A) ® (Q); (B) ® (R); (C) ® (S)
7. The two monomers for the synthesis of (b) (A) ® (Q); (B) ® (P); (C) ® (R)
nylon 6, 6 are (2019 Main, 12 Jan II) (c) (A) ® (R); (B) ® (P); (C) ® (S)
(a) HOOC(CH 2 )4 COOH, H 2N(CH 2 )4 NH 2 (d) (A) ® (R); (B) ® (P); (C) ® (R)

w
(b) HOOC(CH 2 )6 COOH, H 2N(CH 2 )4 NH 2 13. The correct match between Item - I and Item - II is
(c) HOOC(CH 2 )4 COOH, H 2N(CH 2 )6 NH 2
Item I (Drug) Item II (Test)
(d) HOOC(CH 2 )6 COOH, H 2N(CH 2 )6 NH 2
A. Chloroxylenol P. Carbylamine test

Flo
8. Poly-b-hydroxybutyrate-Co-b-hydroxyvalerate (PHBV) is a B. Norethindrone Q. Sodium hydrogen carbonate test
copolymer of …… (2019 Main, 12 Jan I) C. Sulphapyridine R. Ferric chloride test

ree
(a) 3-hydroxybutanoic acid and 2-hydroxypentanoic acid D. Penicillin S. Bayer’s test
(b) 2-hydroxybutanoic acid and 3-hydroxypentanoic acid (2019 Main, 9 Jan I)
(c) 3-hydroxybutanoic acid and 4-hydroxypentanoic acid (a) A®R;B®P;C®S;D®Q

F
(d) 3-hydroxybutanoic acid and 3-hydroxypentanoic acid (b) A®R;B®S;C®P;D®Q
9. The homopolymer formed from 4-hydroxybutanoic acid is (c) A®Q;B®P;C®S;D®R

or
O O
ur O
(d) A®Q;B®S;C®P;D®R

f
14. The formation of which of the following polymers involves
(a) —OC(CH2)3—O— (b) —C(CH2)2C—
n n hydrolysis reaction? (2017 Main)
ks
O O O (a) Nylon-6
Yo
oo
(c) —C(CH2)3—O— (d) —C(CH2)2C—O—n (b) Bakelite
n
(c) Nylon-6, 6
eB

(2019 Main, 11 Jan II) (d) Terylene


10. The correct match between Item I and Item II is 15. Which of the following statements about low density
Item I Item II polythene is false? (2016 Main)
r

A. Allosteric effect P. Molecule binding to the active site (a) It is a poor conductor of electricity
ou
ad

of enzyme. (b) Its synthesis required dioxygen or a peroxide initiator


B. Competitive Q. Molecule crucial for as a catalyst
Y

inhibitor communication in the body. (c) It is used in the manufacture of buckets, dustbins etc.
C. Receptor R. Molecule binding to a site other (d) Its synthesis requires high pressure
than the active site of enzyme.
nd

16. Which of the following is an anionic detergent?(2016 Main)


Re

D. Poison S. Molecule binding to the enzyme


covalently.
(a) Sodium lauryl sulphate
(b) Cetyltrimethyl ammonium bromide
Fi

(a) A ® P; B ® R; C ® S; D ® Q (c) Glyceryl oleate


(b) A ® P, B ® R; C ® Q; D ® S (d) Sodium stearate
(c) A ® R; B ® P; C ® S; D ® Q 17. On complete hydrogenation, natural rubber produces
(d) A ® R; B ® P; C ® Q; D ® S (2019 Main, 11 Jan II) (2016 Adv.)
11. The polymer obtained from the following reaction is: (a) ethylene-propylene copolymer
(2019 Main, 11 Jan I) (b) vulcanised rubber
(i) NaNO2 /H3O
+ (c) polypropylene
NH2 (d) polybutylene
HOOC (ii) Polymerisation
18. Which polymer is used in the manufacture of paints and
O O lacquers? (2015 Main)
H
(a) —C—(CH2)4—N—n (b) —O—(CH2)4—C—n (a) Bakelite
(b) Glyptal
O O O (c) Polypropene
H (d) Polyvinyl chloride
(c) —OC(CH2)4O— (d) —HNC(CH2)4—C—N—
n n
458 Biomolecules and Chemistry in Everyday Life

19. Match the polymers in Column I with their main uses in AcO AcO AcO
Column II and choose the correct answer: HH OH H OH H OO
(c) H H
Column I Column II OAc H O OAc H O OAc H
O H
(A) Polystyrene 1. Paints and lacquers
H OAc H OAc H OAc
(B) Glyptal 2. Raincoats
(C) Polyvinyl chloride 3. Manufacture of toys AcO AcO AcO
(D) Bakelite 4. Computer discs OH H OH H OO
HH H H
Codes (d)
H H O H H O H H
A B C D A B C D O H
(a) 2 1 3 4 (b) 3 1 2 4 OAc OAc OAc OAc OAc OAc
(c) 2 4 3 1 (d) 3 4 2 1

w
20. OCOCH3
Objective Question II
(One or more than one correct option)
COOH is used as 25. Under hydrolysis conditions, the compounds used for
preparation of linear polymer and for chain termination,

Flo
respectively are (2012)
(a) Insecticide (b) Antihistamine (a) CH3 SiCl3 and Si(CH3 )4 (b) (CH3 )2 SiCl2 and (CH3 )3 SiCl

ree
(c) Analgesic (d) Antacid
(c) (CH3 )SiCl2 and CH3 SiCl3 (d) SiCl4 and (CH3 )3 SiCl
21. Which of the following is not an antacid? (2015 Main)
26. The correct functional group X and the reagent/reaction

F
(a) Aluminium hydroxide (b) Cimetidine
(c) Phenelzine (d) Ranitidine conditions Y in the following schemes are (2011)
(i) Y
22. Which one is classified as a condensation polymer?

or
Condensation polymer
ur (2014 Main)
X—(CH2)4—X
O
O

f
(a) Dacron (b) Neoprene (ii) C—(CH2)4—C
(c) Teflon (d) Acrylonitrile HO heat OH
ks
23. Among cellulose, poly (vinyl chloride), nylon and natural (a) X = COOCH3, Y = H2/Ni/heat
Yo
rubber, the polymer in which the intermolecular force of X = CONH2, Y = H2/Ni/heat
oo
(b)
attraction is weakest is (2009) (c) X = CONH2, Y = Br2/NaOH
(a) nylon
eB

(d) X = CN, Y = H2/Ni/heat


(b) poly (vinyl chloride)
(c) cellulose Match the Columns
(d) natural rubber 27. Match the chemical substances in Column I with type of
r
ou

24. Cellulose upon acetylation with excess acetic anhydride/ polymers/type of bond in Column II.
ad

(2007, 6M)
H2 SO4 (catalytic) gives cellulose triacetate whose structure Column I Column II
Y

is (2008, 3M) A. Cellulose p. Natural polymer


AcO B. Nylon-66 q. Synthetic polymer
OO
nd

C. Protein r. Amide linkage


Re

AcO H H D. Sucrose s. Glycoside linkage


(a) O OAcH
O H
Fi

AcO H H Subjective Questions


O OAcH H H OAc
HH O 28. Monomer A of a polymer on ozonolysis yields two moles of
OAc HCHO and one mole of CH3COCHO.
OAcH H H
(2005)
O
(a) Deduce the structure of A.
H OAc (b) Write the structure of ‘all cis’ – form of polymer of
AcO
OO compound A.
AcO H H 29. Name the heterogeneous catalyst used in the polymerisation
(b) O OH H
O H of ethylene. (2003)
AcO H H
OH H H H OH 30. Give the structures of the products in the following reaction.
HH O O (2000, 2M)
OH NOH
O
OH H H H
H OH H+ Polymerisation
C *—D—n*
Answers
Topic 1 Topic 2
1. (a) 2. (b) 3. (a) 4. (d) 1. (a) 2. (a) 3. (a) 4. (c)
5. (b) 6. (b) 7. (d) 8. (d) 5. (d) 6. (b) 7. (c) 8. (d)
9. (a) 10. (d) 11. (a) 12. (a) 9. (c) 10. (d) 11. (b) 12. (b)
13. (c) 14. (d) 15. (a) 16. (d) 13. (b) 14. (a) 15. (c) 16. (a)
17. (a) 18. (b) 19. (a) 20. (a) 17. (a) 18. (b) 19. (b) 20. (c)
21. (a) 22. (b) 23. (a) 24. (b) 21. (c) 22. (a) 23. (d) 24. (a)
25. (b) 26. (d) 27. (b,c) 28. (b,c) 25. (b) 26. (a, b, c, d)

w
29. (c) 30. (1) 31. (4) 32. (4) 27. A ® p, s; B ® q, r; C ® p, r; D ® s
33. (8) 34. (6) 35. (2)

Flo
Hints & Solutions

ree
Topic 1 Biomolecules

F
1. Statement (a) is incorrect. Glycogen is not a straight chain a - D- glucose units and the branches are connected to the main
polymer similar to amylose. It is highly branched structure chain by a - 1,6¢- glycosidic linkages. Its structure can be
similar to amylopectin. It is known to be the storage material of represented as:

or
ur
animals. It is found in liver, muscles and brain. It breaks down to 6
4 CH2OH O
glucose by the action of enzymes when body needs a glucose. It

f
O 2
5
is also found in yeast and fungi. HO
H
1
ks 3 OH a-1, 6¢-glycosidic linkage (Branch)
2. RNA does not have double stranded a-helix structure. Helixes 6
O
6
Yo
present in RNA are single-stranded but sometimes they fold 4 CH2 O 4 CH2OH
O
oo
back on themselves to form a double helix structure. RNA 5 2
H 5 2 H
usually does not replicate. HO
3
HO 1
OH 3 OH O
eB

It is present in the nucleus of the cell. It controls the synthesis of O


protein. RNA molecules are of three types, i.e. messenger’s a-1, 4¢-glycosidic linkage (Chain)
RNA (m-RNA), ribosomal RNA (rRNA), transfer RNA (t-RNA). Structure of Amylopectin
r

3. Key Idea Chiral centre is also called stereo-centre or 5. The peptide that gives positive cerric ammonium nitrate and
ou
ad

stereogenic center. carbylamine tests is ser - lys. The structures of serine and lysine
are,
Linear structure of glucose is as follows :
Y

CHO
HO ¾ CH2 ¾ C H ¾ COOH H2N ¾ (CH2)4 ¾ C H ¾ COOH
| |
H OH NH2 NH2
nd
Re

HO H Serine Lysine
H OH 6. Statement-(b) is not true for sucrose. It is linked through a
glycosidic linkage between C-1 of a-glucose and C-2 of
Fi

H OH
CH2OH
b-fructose. Since, the reducing groups of glucose and fructose are
involved in glycosidic bond formation, sucrose is a non-reducing
Fischer formula Number of stereo-centre (C* ) = 4. sugar.
Cyclic structure of glucose are as follows : CH2OH
CH2OH CH2OH O
H H HOH2C O
H H
O O a b
H H H H H OH OH H
HO Glycosidic
linkage H HO CH2OH
HO OH H OH HO OH H H
H OH

H OH a-D-glucose OH H
H OH
a-D-glucose b-D-glucose b-D-fructose

Haworth formula Number of stereo-centre (C* ) in each anomer = 5. On hydrolysis with acids or enzyme, sucrose gives equimolar
mixture of D-(+)-glucose and D-(–)-fructose.
4. Amylopectin is the water-soluble component of starch. It is a HCl
branched-chain polymer of a-D-glucose. The main chain C12H22O11 + H2O ¾¾® C6H12O 6 + C6H12O 6
consists of an a - 1, 4¢- glycosidic linkages between a - D- D-(+) -glucose D-(–)-fructose
460 Biomolecules and Chemistry in Everyday Life

7. Both fructose and glucose give following test positive.


R C COOH
(i) Fehling’s test (red ppt. of Cu 2O is obtained).
(ii) Barfoed’s test (red ppt. of Cu 2O is obtained) NH2
(iii) Benedict’s test (red ppt. of Cu 2O is obtained)
They have the tendency to loose H+ of their ¾COOH group at
Fehling’s solution :CuSO4 + Na, K-tartrate (Rochelle salt)
alkaline (higher) pH while the ¾ NH2 group present in them
Barfoed’s reagent (CH3COO )2 Cu + CH3COOH + H 2O
(7%) (1%) (92%)
have the tendency to gain H+ at acidic (lower) pH.

Benedict’s solution : CuSO4 + Na-citrate + Na 2CO3 10. The correct match is :


Seliwanoff’s test is used to differentiate between ketose and A ® (Q); (B) ® (S) (C) ® (P)
aldose. The reagent is a solution of resorcinol in concentrated R Þ CH2—COOH C2H5OH/H2SO4/D
HCl. The reagent when heated along with a sugar will produce Asp [Aspartic acid]
[Ester test] (A)
furfural or hydroxy-methylfurfural, which further reacts to give (Q)

w
Sweet smell
red color. Ketose (fructose) reacts more quickly than aldose of ester
(glucose). R Þ (CH2)4—NH2 CHCl /alc.KOH
3
Lys [Lysine]
[Carbylamine test] (B)
8. Maltose on treatment with dil. HCl gives D-glucose. Hydrolysis OOC (S)

Flo
of maltose yields two moles of a- D-glucose. Thus, it is CH— R Foul smell
of isocyanide
composed of two a-D-glucose units in which C-1 of one glucose H3 N O
unit (I) is linked to C-4 of another glucose unit (II). The free General

ree
aldehyde group can be produced at C-1 of second glucose in formula of O, Conc. H2SO4/D
amino acid
solution and it shows reducing properties. So, it is a reducing
R Þ ¾CH2 OH O
sugar.

F
Tyr [Tyrosine] [Phthalein dye test] (C)
CH2OH CH2OH (P) Resultant colourless
O O solution turns pink

or
H H H H H
H
ur (A) Ester test confirms the presence of
OH H O OH H

f
OH OH ¾ COOH group.
(B) Carbylamine test confirms the presence of
ks
H OH H OH
¾ NH2 group (1°).
Yo
CH2OH
(C) Phthalein dye test confirms the presence of phenolic
oo
O
H OH ¾ OH group.
Dil. HCl H
2
eB

OH H H
OH 11. RNA contains, adenine (A), guanine (G), cytosine (C) and uracil
(U). In the given options,
H OH O
a-D-glucose
r

9. Histidine has following structure in


ou

NH
ad

is uracil (present in RNA only)


H2 N CH C OH N O
Y

H
CH2 O
Lone pairs of O
These lone pairs are
nd

both of these N H3 C
Re

nitrogen are not free for donation as NH


freely available N H these are delocalised is thymine (present in DNA)
with-p bonds.
Fi

for donation N O
H
At highly acidic pH, i.e. 2 both the nitrogens with lone pairs will
accept one H+ each and ¾ C ¾ OH will not loose its H+ . Thus, NH2
½½
N
O is cytosine (present in both DNA and RNA)
the final structure of histidine at pH = 2 will be N O
+ H
H3N CH C OH
O
CH2 O
N¾Me
+ is not a pyrimidine base but a derivative of uracil.
H N
N O
NH Me
Thus, option (d) is the correct answer. 12. (i) Barfoed test is used for detecting the presence of
Note Amino acids have following generalise structure: monosaccharides like glucose, fructose etc. Barfoed reagents is
Cu (II) acetate solution.
Biomolecules and Chemistry in Everyday Life 461

CHO COOH 15. HI is a strong reducing agent. It reduces both primary and
secondary alcoholic groups of glucose along with the carbonyl
H OH H OH group to produce n-hexane as
HO H HO H
+ Cu2++ 2H2O + Cu2O +4H+ CHO Carbonyl group CH3
H OH (Cu (II)-acetale) H OH
(Red ppt.) Secondary
H OH H OH HI
alcoholic (CH2)4
(CHOH)4 group D
CH2OH CH2OH (Prolonged)
(D-glucose) (D-gluconic acid) CH2OH Primary CH3
alcoholic
group
(ii) Glucose n-hexane
(Ninhydrin Test)
Violet 16. Our blood is slightly basic in nature with pH range from
Ninhydrin/ D
colouration 7.35-7.4. The structure of histamine is given below :
Amino acid (Xanthoprotic Test) Aliphatic

w
Yellow
(Protein) Conc. HNO3 H amino group
colouration
(Biuret Test) Imidazole N NH2
Violet ring
CuSO4 (aq)+NaOH

Flo
colouration N
13. Formation of the tripeptide (Val-Ser-Thr) can be shown as: Basic nitrogen of imidazole ring

ree
Me Me Me OH It is produced by decarboxylation of histidine having following
CH O CH
structure. It is clearly visible from the above structure that

F
CH OH H HN C CH OH histamine has two basic centres namely aliphatic amino group
H 2N C CH OH H HN C and basic nitrogen of imidazole ring. The aliphatic amino group
O CH2 O has pK a around 9.4. In blood with pH around 7.4 the aliphatic

or
OH
ur amino group of histamine become protonated to give a single
charged cation as shown below

f
Valine (Val) Serine (Ser) Threonine (Thr)
–2H2O H
Me Me Me OH
ks
CH O CH N +
NH3
Yo
CH NH C CH OH
oo
H 2N C C N
CH NH
17. Sugars that have an aldehyde, a ketone, a hemiacetal or a
eB

O CH2 O
OH hemiketal group is able to reduce an oxidising agent. These
Val Ser Thr sugars are classified as reducing sugars.
HOH2C O HOH2C O
r

CH2OH CH2OH
ou

14 Amino acid molecules can be represented as, aq·KOH


ad

COOH
HO –+ OH
–CH3COOK
O–
Y

R CH OCOCH3
NH2
OH OH
Nature of the ‘R’ group will determine the basicity (hence, pK a ) Hemiacetal
nd
Re

of an amino acid.
Fi

Nature of O
Nature HOH2C CH2OH
‘R’ in the amino group of R the amino
acids HO O
1. + Basic More basic (due
NH2 to the presence
¾ (CH2)3¾ NH ¾ C OH
of acetamidine
NH2 (a-hydroxyketone)
group)
(Arginine : Arg)
Hemiacetal can be easily reduced by oxidising agent such as
2. O Acidic Acidic Tollen’s reagent.
CH2 C
O– O
HOH2C CH2OH
(Aspartic acid: Asp) Tollen’s
O positive silver
3. ¾H (Glycine) : Gly Neutral Neutral OH reagent
mirror test.
+
4. ¾ ( CH2 ) 4 ¾ NH3 Basic Basic
OH
(Lysine : Lys) (Reducing sugar)
462 Biomolecules and Chemistry in Everyday Life

18. NH2 H O H O
Cystine HO S COOH C1 C1
C S
H C2 OH HO C2 H
O NH2
COOH HO C3 H H C3 OH
Cysteine
HS
NH2 H C4 OH HO C4 H
NH2 Cytosine
O
N H C5 OH HO C5 H
Methionine S C
H3C OH , N 6CH2OH 6CH2OH
O
NH2 H D-glucose L-glucose

w
Thiol group (SH) is present in cysteine. So, b-L glucopyranose is formed as
H H
19. Vitamin B and C are water soluble while vitamin A,D,E and 5 5
O H O
K are fat soluble or water insoluble. HO 6 CH OH HO H
CH2OH

Flo
4 2 1C O 4 6 1
20. Quinoline is an alkaloid, it is not present in DNA. DNA has H OH H OH OH
H H H
four nitrogen bases in adenine, guanine, cytosine and
2 2

ree
3 3
thymine. OH H OH H
21. 18 ATPs are involved in the formation of 1 glucose molecule b-L-glucopyranose
The a-L-glucopyranose has configurational change at C1 only and

F
as shown in the reaction below :
6CO2 + 12NADPH + 18 ATP ¾® looks like
C6 H12 O6 + 12NADP + 18 ADP H

or
ur O
HO HO Configuration at

f
22. Here, the — OH of hemiacetal group is equatorial therefore, it CH2OH this carbon atom
1
is a b-pyranose of an aldohexose. H OH H is opposite in a
ks
H
and b-forms
23. The six-membered cyclic ether is known as pyranose while
Yo
OH H
the five membered cyclic ether is known as furanose. Hence,
oo
a-L-glucopyranose
ring (a) is a pyranose and it has ether linkage at a-position
that is known as a-glycosidic linkage in carbohydrate
eB

27. If there is inversion of specific rotation from (+ ) to (-), then invert


chemistry.
sugar is formed.
24. “a” and “b” cyclic hemiacetals of D-glucose having (a ) C12H22O11 + H2O ¾® Glucose
(+) Maltose D(+)
r

difference in configuration at C-1 only are called anomers. 52 °


140°
ou
ad

25. Both glucose and fructose are reducing sugars, reduces (b) C12H22O11 + H2O ¾® Glucose + Fructose
(+) Sucrose D(+) L(-)
Tollen’s reagent to metallic silver. + 66 ° 52 ° -92 °
Y

-40 ° for 2 moles mixture


26. (d) A pyranose ring is a 6 membered ring having 5 carbon -20 ° for 1 mole mixture
atoms and one oxygen atom. In glucose, it is formed by the There is formation of invert sugar. Thus, correct.
nd

(c) Specific rotation of invert sugar is -20 ° per mole. Thus,


Re

reaction between >C=O group at position 1 and —OH group


at 5th carbon atom. In general reaction between >C=O group correct.
Fi

and —C—OH looks like (d) Br2 water is a weak oxidising agent. It oxidises ¾ CHO to
¾ COOH. ¾ CH2OH group is not affected.
OH COOH CHO COOH
C O+ C OH C
O C ½ ½ Br water
½
HNO 2
(CHOH)4 ¬¾¾3¾ (CHOH)4 ¾¾¾® (CHOH)4
Carbonyl Alcoholic Hemiacetal or
group group Hemiketal ½ ½ ½
COOH CH2OH CH2OH
The product formed in called hemiacetal Saccharic acid Gluconic acid
(one of the products)
(if>C=O group belongs to an aldehyde) or hemiketal (if
>C=O group belongs to a ketone). L- glucose has the mirror HNO3 (a strong oxidising agent) oxidises invert sugar to
image configuration of D-glucose i.e., saccharic acid. Thus, incorrect.
Biomolecules and Chemistry in Everyday Life 463

28. X is acetal, has no free hemiacetal, hence a non-reducing sugar 32. The amino acid remain completely in Zwitter ionic form at its
while Y has a free hemiacetal group, it is reducing sugar. Also, isoelectric point. Amino acids with additional acidic group have
glucosidic linkage of X is ‘a’ while that of Y is b-linkage. their isoelectric pH less than 7.0 and increasing pH above
isoelectric point makes them anionic.
29. Statement I is correct Presence of — CHO group in glucose is
tested by Fehling’s solution test where a reddish-brown On the other hand, amino acids with additional basic group have
precipitate of Cu 2O is formed. their isoelectric pH greater than 7.0 and decreasing pH below
isoelectric point (by adding acid solution) makes them cationic.
Hence, Statement II is incorrect. The given peptide with followings R1 and R2 are basic, will
30. PLAN This problem can be solved by performing hydrolysis of peptide remain protonated (cationic) at pH = 7.0.
and deciding the nature of product.
Chemical reaction and product formed after hydrolysis of given Peptide R1 R2
peptide can be represented as IV CH 2CONH 2 (CH 2 ) 4 NH 4
O

w
VI (CH 2 ) 4 NH 2 (CH 2 ) 4 NH 4
H
H O H O N VIII CH 2OH (CH 2 ) 4 NH 4
O O
N N IX (CH 2 ) 4 NH 2 CH 3
NH—C NH N

Flo
N
CH2 O O H CH2 O Thus, 4 is the correct integer.
N
H
33. The D-form of given sugar is

ree
O
CHO
H3O+
CMe3 CH2OH

F
+ CH2
+ –
H3N—CH2—CO2+ + O2C—CH—NH3 H O OH
(A) (B) CHOH

or
+
H 3N CO2
ur Cyclisation
*
H
H
*
HO—C—CHNH—CH2—C—OH

f
CHOH
OH H
O CH2 O *
ks
CH2 + H OH
H
Yo
CH2OH D-Pyranose
oo
D- Sugar
(C ) (D )
eB

Configurations at the three chiral carbons (starred) can be changed


(A) is glycine which is only naturally occurring amino acid. While
maintaining D-configuration. Hence, the total number of
(B), (C) and (D) are not the naturally occurring amino acids.
steroisomers of D-pyranose = 23 = 8
Hence, correct integer is (1).
r

Thus, the correct integer is 8.


ou

31. PLAN A peptide linkage is hydrolysed to two free amino acids.


ad

34. A decapeptide has nine peptide (amide) linkage as


O O O O O
Y

O
|| O O O O O
H2N ¾ CH ¾ C ¾ NH ¾ CH ¾ COOH
| |
nd

R¢ —CONH—
Re

R
­ bond
Peptide
Fi

Therefore, on hydrolysis, it will absorb nine water molecules.


O Hence, total mass of hydrolysis product = 796 +18 × 9 = 958
* || *
H2N ¾ CH ¾ C ¾ OH + H2N ¾ CH ¾ COOH
| | Þ mass of glycine in hydrolysis product = 958 ´ 47 = 450
R R¢ 100
Þ number of glycine molecule in one
C* is chiral carbon tetrapeptide has four amino acids joined by 450
three peptide linkage. molecule of decapeptide = =6
75
¾ COOH group is on alanine part, thus it is at fixed C-terminal
position in each combination. 35. — OO- and —NH2 are two basic groups in lysine.
Glycine is optically inactive thus it cannot be on the N¾ 36. In structure (a), one ring has a free hemiacetal group, will
terminal side. Thus, possible combinations are hydrolyse into open chain in aqueous solution and therefore will
reduce Tollen’s reagent. Structure (b) has only acetal groups,
Phe-Gly-Val-Ala, Phe-Val-Gly-Ala,
will not hydrolyse in aqueous solution into open chain, will not
Val-Gly-Phe-Ala, Val-Phe-Gly-Ala
reduce Tollen’s reagent
Thus, in all four combinations are possible.
464 Biomolecules and Chemistry in Everyday Life

37. (i) D-glucose and L-glucose are enantiomers, hence CHO


CO2OH
Mirror H OH
CHO CHO
C O
HO H
H OH HO H HO H
H OH
HO H H OH H OH
H OH
H OH HO H H OH
CH2OH
CH2OH
H OH HO H
pH = 10
CH2OH CH2OH 41. H2N ¾ CH ¾ COO- ¾¾® H2N ¾ CH ¾ COOH
½ ½

w
D-glucose L-glucose
CH3 CH3
CHO COOH Alanine
pH = 2 +
H OH H OH ¾¾® H3N ¾ CH ¾ COOH

Flo
½
HO H + CH3
Ag(NH3)2 HO H
(ii)

ree
H OH H OH
Topic 2 Chemistry in Everyday Life

F
H OH H OH 1. Bakelite is a thermosetting polymer. These polymers are
cross-linked or heavily branched molecules which on heating
CH2OH CH2OH undergo extensive cross linking in moulds and become

or
ur gluconic acid infusible. Once they get set, they cannot be reshaped and reused.
38. The dipeptides are

f
[TiCl + (C H ) Al]
2. (A) n CH2 == CH2 ¾¾¾¾¾¾®
4 2 5 3
¾
[ CH2 ¾] n
HOOC ¾ (CH2 )2 ¾ CH ¾ CO ¾ NH ¾ CH ¾ (CH2 )4 ¾ NH2 Ethylene
Ziegler-Natta catalyst High density
ks
(III) polythene (HDPE)
½ ½
-
Yo
NH3 COO CN CN
+
oo
and | O22- /O2 ½
HOOC ¾ (CH2 )2 ¾ CH ¾ NH ¾ CO ¾ CH ¾ (CH2 )4 ¾ NH2 (B) nCH2 == CH ¾¾¾¾® ¾ [ CH2 ¾ C H ]¾
n
(Catalyst)
eB

Polyacrylonitrile
½ ½ Acrylonitrile (I)
COO- NH3 (PAN)
+ (C) OH OH
O CH2C6H5 CH2OH
½
r

½½ sOH
39. H2N ¾ CH ¾ C ¾ NH ¾ CH ¾ COOCH3 + HCHO Polymerisation/
ou
ad

Acid or H Or
Formaldehyde 3
½ base catalyst
CH2 ¾ COOH Phenol (IV)
Y

Aspartame
OH OH
(i) Aspartame has amine, acid, amide and ester groups. CH2
nd
Re

O CH2C6H5
+ ½½ ½
(ii) H3 N ¾ CH ¾ C ¾ NH ¾ CH ¾ COOCH3 n
Fi

Novolac
½
CH2 ¾ COO -
H+
(D) O
(iii) Aspartame ¾¾® H2N ¾ CH ¾ COOH + NH
H2 O
½
550 K
O 20 atm —C—NH—(CH2)5—
CH2 COOH –H2O Nylon-6 n
I Caprolactum (II)
CH2C6H5
½
Thus, the correct match is as follows :
H2N ¾ CH ¾ COOH + CH3OH (A) ® (III), (B) ® (I), (C) ® (IV), (D) ® (II)
II
3. Nylon-6, 6 (an amide) is a condensation copolymer because it is
(iv) II is more hydrophobic due to the presence of phenyl group.
obtained by condensation between adipic acid and
H+ hexamethylenediamine.
40. Sucrose ¾¾® D-glucose + D-fructose
H2 O
Biomolecules and Chemistry in Everyday Life 465

O O of adipic acid and hexamethylene diamine is the correct answer.


H
Both of these units react as follows to form nylon-6, 6.
nHO—C—(CH2)4—C—OH + nN—(CH2)6—NHH O
H Hexamethylene –nH2O
Adipic acid
diamine nH N (CH2)6 N H + OH C (CH2)4 C OH

O O H H –nH2O H Adipic acid


Hexamethylene O
——C—(CH2)4—C—NH—(CH2)6—NH—— diamine
1 mol n
N (CH2)6 N C (CH2)4 C
nylon-6,6

Neoprene, teflon and buna-S are addition polymers. H H O


n
Nylon-6, 6
4. Noradrenaline is one of the example of neurotransmitters. It
8. Poly-b-hydroxy butyrate Co-b-hydroxyvalerate (PHBV) is a

w
plays a major role in mood changes. If the level of noradrenaline
is low for some reason, then signal-sending activity becomes copolymer of 3-hydroxybutanoic acid and 3-hydroxypentanoic
low and the person suffers from depression. acid. It is used in speciality packaging, orthopaedic devices and in
O controlled release of drugs. PHBV undergoes bacterial degradation

Flo
|| in the environment. The reaction involved is as follows :
5. Monomer of ¾[NH ¾ C ¾ NH ¾ CH2¾ ]n is formaldehyde. OH OH
The polymer is also known as urea-formaldehyde resin. It is ½ ½

ree
made from urea (NH2CONH2) and formaldehyde (HCHO). CH3 ¾ CH ¾ CH2COOH + CH3CH2 CHCH2COOH ¾®
Polymerisation
NH 2CONH 2 + HCHO ¾¾¾¾¾® 3-hydroxybutanoic acid 3-hydroxypentanoic acid

F
Urea Formaldehyde æ O CH2CH 3 ö
O ç ÷
ç ½½ ½ ÷
|| ¾ç O ¾ CH ¾ C H2 C ¾ O ¾ CH ¾ CH2 ¾ C ÷¾

or
ur
¾(NH ¾ C ¾ NH ¾ CH 2 )¾
n ç ½ ½½÷
Urea-formaldehyde resin ç CH3 O÷ n

f
è ø
It is used for making unbreakable cups and laminated sheets.
ks
(PHBV)
6. O
Yo
9. On polymerisation, 4-hydroxy butanoic acid will produce a
oo
condensation homopolymer by loss of H2O molecules.
Polymerisation
NH O O
eB

HO—C—CH2CH2CH2—O H + HO —C—(CH2)3—O H
Caprolactum 4-hydroxy butanoic acid
–H2O
O
r

O
ou

O O
ad

—C—NH—(CH2)5 —C—NH—(CH3)5—
-----—C—(CH2)2CH2—O—C—(CH2)2CH2—O----
Y

or
Ester linkage
O
The homopolymer obtained can also be represented as
nd

—NH—(CH2)5—C—
Re

n O
Nylon-6 ½½

[ C — (CH2)3 O —]n
Fi

Nylon-6 is prepared by ring opening polymerisation of


caprolactum. It is heated about 533 K in an inert atmospheric 10. (A) Molecule binding to a site other than the active site of
nitrogen about 4-5 hrs. Nylon-6 fibres are tough, possessing enzyme is called allosteric effect.
high tensile strength, as well as elasticity and lustre. They are (B) Molecule binding to the active site of enzyme is called
wrinkle proof and highly resistant to abrasion and chemicals competitive inhibitor.
such as acids and alkalis.
(C) Molecule crucial for communication in the body is called
7. Nylon-6,6 has following structure: receptor.
é O O ù (D) Molecule binding to the enzyme covalently is called poison.
ê ½½ ½½ ú
ê N ¾ (CH2)6 ¾ N ¾ C ¾ (CH2)4 ¾ C ¾
ú Thus, the correct match is : A ® R, B ® P, C ® Q, D ® S
¾
ê ½ ½ ú 11. Given amino acid on reaction with NaNO 2 / H3O+ gives
ê H H ú
ë ûn diazotisation reaction which further evolves ¾ N 2 gas along
6 C-atoms 6 C-atoms with formation of carbocation. On further reaction with water, it
form HOOC ¾ (CH2 ) ¾ OH that undergoes polymerisation to
As it is a condensation polymer hence, each of its monomeric
give polymer.
unit must contain 6 carbon atoms in them. Hence, a combination
466 Biomolecules and Chemistry in Everyday Life

NaNO2/H3O+
O
H H CH3
HOOC—(CH2)4—NH2 HOOC—(CH2)4—N2 S
(Diazotisation) (D) R C NH
Aliphatic diazonium ion CH3
(Unstable)
–N2 N COOH
+ O
HOOC—(CH2)3—CH2 H
Penicillin
O –H –H2O NaHCO3(aq)
(Sodium
Becomes soluble
Polymerisation with effervescence
C—(CH2)4—O H HOOC—(CH2)4—OH hydrogen
carbonate) of CO2 gas
OH OH (Q)
–H2O
H O—(CH2)4—C == O Thus, the correct match is: A ® R; B ® S; C ® P; D ® Q
O O O
14. Nylon-6 or perlon is prepared by polymerisation of amino

w
-----O—(CH2)4—C—O—(CH2)4—C---- or —O—(CH2)4—C—
n caproic acid at high temperature. Caprolactam is first
(Polymer)
(An ester group) hydrolysed with water to form amino acid which on heating
12. The correct match is: undergoes polymerisation to give nylon-6.
A ® (Q) B ® (P) C ® (R) O

Flo
O
(A) Norethindrone It is an antifertility drug(Q ) containing NH Hydrolysis + –
synthetic progesterone derivative. [Other similar drug, is H3N—(CH2)5—C—O

ree
ethinylestradiol (novestrol)].
Caprolactam D Polymerisation
(B) Ofloxacin It is an antibiotic (P ), O

F
i.e produced wholly or partly by chemical synthesis with low
concentration of microorganism.[Some other similar drugs : [ HN— (CH2)5—C —
]n
Penicillin, chloramphenicol, salvarsan etc.] Nylon-6

or
ur
(C) Equanil (meprobamate) It is a mild tranquilizer for
15. High density polythene is used in the manufacture of buckets,

f
relieving hypertension. It relieve anxiety, stress, excitement by
inducing a sense of well being. dustbins etc.
ks
(Other similar drug is chlordiazepoxide.) 16. Sodium lauryl sulphate [(CH3 (CH2 )10 CH2OSO3- Na + )] =
Yo
Anionic detergent
oo
13 Cetyltrimethyl + ammonium
Phenolic é CH3 ù
OH –OH group ê ½ ú
eB

FeCl3 ê CH (CH ) ¾ N ¾ CH ú Br - = Cationic detergent


3 2 15 3
(A)
(Ferric chloride)
Violet colouration ê ½ ú
(R) ê CH ú
H 3C CH3 ë 3 û
r

Cl
Glyceryl oleate [(C17 H32COO)3 C3H5 ] = Non-ionic detergent
ou
ad

Chloroxylenol (Dettol) Sodium stearate [C17 H35COO- Na + ] = Anionic soap


17. Natural rubber is formed by polymerisation of isoprene.
Y

OH
H3C CººCH
H Dil. KMnO4/5°C/OH – CH2 C—CH CH2 CH2—C CH—CH2
nd

H
Re

Pink colour of

(Baeyer’s test)
(B) (S)
KMnO4 gets CH3 CH3
H H discharged
Fi

O Polymerise
Norethindrone Ethylene

C C
—CH2—CH— CH2—CH2— —CH2—C CH—CH2—
Propylene

(Double bond)
H2/Pt

CH3 CH3
O
n n
Natural rubber
(C) H 2N S N Ethylene-propylene copolymer
H N
O
Primary Sulphapyridine This co-polymer is formed from propylene and ethylene.
amine (–NH2) CHCl3 + KOH (alc.)
Foul smell
(Carbylamine
of isocyanide n CH2 == CH + nCH2 == CH2 ¾® —CH2—CH CH2—CH2—
test) (P)
½
CH3 CH3
n
Biomolecules and Chemistry in Everyday Life 467

18. (a) Bakelite is used for making gears, protective coating and electrical fittings.
(b) Glyptal is used in the manufacture of paints and lacquers.
(c) PP is used in the manufacture of textile, packaging materials etc.
(d) Polyvinyl chloride (PVC) is used in the manufacture of rain coats, hand bags, leather clothes etc.
19. (a) Polystyreme- manufacturing toys (b) Glyptal- Paints and lacquers
(c) Polyvinyl chloride (PVC)- Raincoats (d) Bakelite- computer discs
Thus, the correct match is A ® (1), B ® (1), C ® (2), D ® (4)
20. The given structure is of aspirin which is used as analgesic.
21. Aluminium hydroxide Al(OH)3, cimetidine and ranitidine are antacids while phenelzine is not.
H CH(NO2)
N

w
N
Me2N S C
S C
N NHMe
N N NHCN O
H
H

Flo
Ranitidine
Cimetidine

ree
Phenelzine is a tranquilizer, not an antacid.
H
N ¾ NH2

F
or
Phenelzine is used as antidepressant drug.
ur
22. Dacron is a condensation polymer of ethylene glycol and methyl terepthalate. Formation of dacron can be shown as

f
ks
Polymerisation
MeO — C C—OMe + nHO — CH2—CH2—OH —O—C C—OCH2—CH2—
Yo
(n-1) MeOH
oo
O O O O n
Dacron
eB

Here, elimination of MeOH occurs as a by product. So, this reaction is known as condensation polymerisation.
23. Cellulose and nylons have H-bonding type of intermolecular attraction while poly (vinyl chloride) is polar. Natural rubber is hydrocarbon
and has the weakest intermolecular force of attraction, i.e. van der Waals’ force of attraction.
r
ou

H OH CH2OH
ad

CH2OAc CH2OAc CH2OAc


C C H H C O O O O
Y

24. H O H O H O
OH H (CH3CO)2O H H H
C C C C
O OH H H
OAc H H OAc H OAc H
nd

O H
Re

H C C C H
O
H OH H OAc H OAc H OAc
Fi

CHOH
Tri-acetylated cellulose Tri-acetylated cellulose

25. CH3 CH3 CH3 CH3 CH3 CH3 CH3 CH3 CH3
Polymerisations (CH3)3 XSiCl
(CH3)2SiCl2 + H2O HO—Si—OH HO—Si—O—Si—O —Si—OH CH3 —Si—O —Si— O—Si—O —Si—O —Si— CH3

CH3 CH3 CH3 CH3 CH3 CH3 CH3 n CH3 CH3


n
Linear chain polymer Chain terminated linear chain silicone

26. (a) When X = COOCH3


H 2 / Ni HOOC—(CH 2 ) 4 —COOH
CH3OOC — (CH2 )4 — COOCH3 ¾ ¾¾® HOCH2 — (CH2 )4 — CH2OH + 2CH3OH ¾¾¾¾¾¾¾¾®
Heat Heat
O O

—O—(CH2)6—O—C—(CH2)4—C—
n
Ester, condensation polymer
468 Biomolecules and Chemistry in Everyday Life

(b) When X = CONH2


O O

H / Ni HOOC—(CH 2 ) 4 —COOH —HN—(CH2)6—NH—C—(CH2)4—C—


H2NOC — (CH2 )4 — CONH2 ¾ ¾¾
2
® H2N ¾ (CH2 )6 — NH2 ¾¾¾¾¾¾¾¾® n
Heat Heat Nylon, condensation polymer
O O
2 Br 2 4 HOOC—(CH ) —COOH
(c) H2NOC — (CH2 )4 — CONH2 ¾¾®
NaOH
H2N— (CH2 )4 — NH2 ¾¾¾¾¾¾¾¾® —HN—(CH2)4—NH—C—(CH2)4—C—
Heat n
Hofmann's bromamide
reaction Nylon, condensation polymer

O O
H 2 / Ni HOOC—(CH 2 ) 4 —COOH
(d) When X = CN NC — (CH2 )4 — CN ¾ ¾¾ ® H2N(CH2 )6 NH2 ¾¾¾¾¾¾¾® —HN—(CH2)6—NH—C—(CH2)4—C—
Heat Heat n

w
In author’s opinion (a) and (b) should also be the answer. Nylon, condensation polymer

O O O O
Heat
(a) HOCH2—(CH2)4—CH2OH + HO—C— (CH2)4—C—OH —O—(CH2)6 —O—C—(CH2)4—C—

Flo
n
Polyester, a condensation polymer

ree
O O
H2/Ni
(b) H2N—C—(CH2)4—C—NH2 H2NCH2—(CH2)4—CH2NH2
Heat

F
O O O O
Heat

or
ur
H2NCH2—(CH2)4—CH2NH2 + HO—C—(CH2)4—C—OH —NH—(CH2)6 —NH—C—(CH2)4—C—
n

f
Nylon, a condensation polymer
ks
27. (A) Cellulose—a natural polymer of a-D-glucose, linked by glycoside linkage.
Yo
(B) Nylon-6, 6—a synthetic polymer of adipic acid and 1,6-diaminohexane. The diacid is linked with diamine through amide linkage.
oo
(C) Protein—a natural polymer of a-amino acids where individual amino acid units are linked by amide linkage.
(D) Sucrose—has glycoside linkage, a disaccharide.
eB

CH3 O
½ O3 ½½
28. (a) H2C == C ¾ CH == CH2 ¾¾¾® 2HCHO + CH3 ¾ C ¾ CHO
Zn - H2 O
r

‘ A ’ Isoprene
ou

CH2 ¾ CH2
ad

H2C CH2
(b) Isoprene ¾® C == C C == C
Y

H3C H H3C H
natural rubber
nd

29. Zeigler-Natta catalyst, which is a mixture of triethylaluminium ‘(C2H5 )3 Al’ and TiCl 4, is used as heterogeneous catalyst in polymerisation of
Re

ethylene.
Fi

30. N—OH
O O
H+ Polymerisation
[–C—(CH2)5—NH–]
NH n
Nylon-6 (D)
Caprolactum
(C)

Download Chapter Test


http://tinyurl.com/y45zhmup or
33
Environmental Chemistry

w
Objective Questions I (Only one correct option) 9. Which is wrong with respect to our responsibility as a
human being to protect our environment?
1. The primary pollutant that leads to photochemical smog is (2019 Main, 8 April I)

Flo
(2019 Main, 12 April II)
(a) Restricting the use of vehicles
(a) acrolein (b) nitrogen oxides
(b) Avoiding the use of floodlighted facilities
(c) ozone (d) sulphur dioxide
(c) Setting up compost tin in gardens

ree
2. The correct set of species responsible for the photochemical (d) Using plastic bags
smog is (2019 Main, 12 April I)
10. The upper stratosphere consisting of the ozone layer

F
(a) N2 , NO2 and hydrocarbons
protects us from the sun’s radiation that falls in the
(b) CO2 , NO2 , SO2 and hydrocarbons
wavelength region of (2019 Main, 12 Jan II)
(c) NO, NO2 , O3 and hydrocarbons

or
(d) N2 , O2 , O3 and hydrocarbons
ur (a) 600 - 750 nm
(c) 0.8 - 15
. nm
(b) 400 - 550 nm
(d) 200 - 315 nm

f
3. Air pollution that occurs in sunlight is (2019 Main, 10 April II) 11. The compound that is not a common component of
(a) acid rain (b) oxidising smog
ks
photochemical smog is (2019 Main, 12 Jan II)
(c) fog (d) reducing smog
(b) H3 C ¾ C ¾ OONO2
Yo
(a) CF2Cl 2
½½
oo
4. The regions of the atmosphere, where clouds form and
O
where we live, respectively, are (2019 Main, 10 April I)
(c) CH2 == CHCHO (d) O3
eB

(a) stratosphere and stratosphere


(b) troposphere and troposphere 12. Water samples with BOD values of 4 ppm and 18 ppm,
(c) troposphere and stratosphere respectively, are (2019 Main, 12 Jan I)
(d) stratosphere and troposphere
r

(a) clean and clean


ou

5. The layer of atmosphere between 10 km to 50 km above the (b) highly polluted and clean
ad

sea level is called as (2019 Main, 9 April II) (c) highly polluted and highly polluted
Y

(a) stratosphere (b) mesosphere (d) clean and highly polluted


(c) thermosphere (d) troposphere
13. The molecule that has minimum/no role in the formation of
6. Excessive release of CO2 into the atmosphere results in photochemical smog, is (2019 Main, 12 Jan I)
nd
Re

(2019 Main, 9 April I) (a) N2 (b) CH2 == O


(a) formation of smog (b) depletion of ozone (c) NO (d) O3
Fi

(c) polar vortex (d) global warming


14. Taj Mahal is being slowly disfigured and discoloured. This
7. The maximum prescribed concentration of copper in is primarily due to (2019 Main, 11 Jan II)
drinking water is (2019 Main, 8 April II) (a) water pollution (b) soil pollution
(a) 5 ppm (b) 0.5 ppm (c) global warming (d) acid rain
(c) 0.05 ppm (d) 3 ppm
15. The higher concentration of which gas in air can cause
8. Assertion (A) Ozone is destroyed by CFCs in the upper stiffness of flower buds? (2019 Main, 11 Jan II)
stratosphere. (a) SO2 (b) CO
Reason (R) Ozone holes increase the amount of UV (c) CO2 (d) NO2
radiation reaching the earth. (2019 Main, 8 April I) 16. Peroxyacetyl nitrate (PAN), an eye irritant is produced by
(a) Assertion and Reason are incorrect.
(2019 Main, 11 Jan I)
(b) Assertion and Reason are both correct and the Reason is the (a) organic waste
correct explanation for the Assertion. (b) acid rain
(c) Assertion and Reason are correct, but the Reason is not the (c) classical smog
explanation for the Assertion. (d) photochemical smog
(d) Assertion is false, but the Reason is correct.
470 Environmental Chemistry
17. The concentration of dissolved oxygen (DO) in cold water (b) A is suitable for drinking, wherease B is not
can go upto (2019 Main, 11 Jan I) (c) Both A and B are suitable for drinking
(a) 14 ppm (b) 10 ppm (d) B is more polluted than A
(c) 8 ppm (d) 16 ppm 20. The pH of rain water, is approximately (2019 Main, 9 Jan II)
18. The reaction that is not involved in the ozone layer depletion (a) 7.5 (b) 6.5
mechanism in the stratosphere is (2019 Main, 10 Jan II) (c) 5.6 (d) 7.0
(a) CH4 + 2O3 ¾® 3CH2 == O + 3H2O 21. The condition for methemoglobinemia by drinking water is
· ·
(b) Cl O(g ) + O(g ) ¾® C l(g ) + O2 (g ) (2019 Main, 9 Jan II)
· ·
(a) > 50 ppm nitrate (b) > 50 ppm chloride
hn
(c) HOCl(g ) ¾® OH(g ) + Cl(g ) (c) > 50 ppm lead (d) > 100 ppm sulphate
hn · ·
(d) CF2Cl 2 (g ) ¾® Cl(g ) + CF2Cl(g ) 22. A water sample has ppm level concentration of the
following metals: Fe = 0.2; Mn = 5.0 ; Cu = 3.0; Zn = 5.0.

w
19. Water filled in two glasses A and B have BOD values of 10 The metal that makes the water sample unsuitable for
and 20, respectively. The correct statement regarding them, drinking is (2019 Main, 9 Jan I)
is (2019 Main, 10 Jan I)
(a) Cu (b) Fe
(a) A is more polluted than B (c) Mn (d) Zn

Flo
Answers

ree
1. (b) 2. (c) 3. (b) 4. (b) 13. (a) 14. (d) 15. (a) 16. (d)

F
5. (a) 6. (d) 7. (d) 8. (c) 17. (b) 18. (a) 19. (d) 20. (c)
9. (d) 10. (d) 11. (a) 12. (d) 21. (a) 22. (c)

or
ur
Hints & Solutions
f
ks
1. The primary pollutant that leads to photochemical smog is O3 (g ) + NO(g ) ¾® NO2(g ) + O2 (g )
Yo
Brown gas (in high
oo
nitrogen oxides. Burning of fossil fuels such as petrol and diesel concentration form haze)
in automobiles, reaction between nitrogen and oxygen and other
3CH4 (g ) + 2O3 (g ) ¾® 3CH2 == O(g )
eB

such reactions result in a variety of pollutants, two main of


which are hydrocarbons (unburnt fuel) and nitric oxide (NO). (Unburnt hydrocarbon) Formaldehyde

N2(g ) + O2 (g ) In petrol and


¾diesel
¾¾ ¾¾® 2NO(g ) + CH2 == CHCH == O+ H2O
engines
Acrolein
r

When the concentration of these pollutants is sufficiently high, a


ou

3. In sunlight oxidising smog or photochemical smog or


ad

chain reaction initiate because of the interaction of sunlight with Los-Angeles smog is formed. This smog is brown in colour. It
oxides of nitrogen.
Y

occurs in warm, dry and sunny climate. In presence of sunlight,


2NO(g ) + O2 (g ) ¾Sunlight
¾ ¾® 2NO2 (g ) NOx (N-oxides), O2 and unburnt hydrocarbons of air combine to
produce photochemical smog which mainly contains peroxyacetyl
NO2(g ) ¾¾
hn
® NO(g ) + [ O]
nd

O
Re

Nascent
oxygen ||
nitrate (PAN). CH3 ¾ C ¾ O ¾ O ¾ NO2 (PAN)
Fi

O3 (g ) + NO(g ) ¾® NO2 (g ) + O2 (g )
Brown gas 4. The lowest region of atmosphere is troposphere which extends
2. The correct set of species responsible for the photochemical upto the height of 10 km (approx) from sea level. We live in the
smog is NO, NO2, O3 and hydrocarbons. Photochemical smog tropospheric region. It contains air, water vapour and dust which
appears in warm, dry and sunny climate which are obtained by can form clouds with the help of strong air movement.
the action of sunlight on unsaturated hydrocarbons and nitrogen Above the troposphere, stratospheric region extends upto 50 km
oxides. Following reactions are involved during the formation from sea level. It contains mainly N2 , O2 , O3 and little water
of photochemical smog. vapour. O3 in the stratosphere absorbs 99.5% of the sun’s harmful
(i) N2 (g ) + O2 (g ) ¾¾¾® 2NO(g ) UV raditions and thus protects the lives on the earth.
(Originates from
burning of fossil fuels) 5. The atmosphere between the heights 10 to 50 km above the sea
Sunlight level is stratosphere. Atmosphere is not of the same thickness at
(ii) 2NO(g ) + O2 (g ) ¾¾¾® 2NO2 (g ) heights.
hn
NO2 (g ) ¾¾¾® NO(g ) + [ O ] 6. The effect of release of CO2 gas into atmosphere is global
Nascent oxygen warming.
(iii) O (g ) + O2 (g ) - O (g ) 3
Reacts rapidly with NO
Environmental Chemistry 471

7. According to W.H.O. and US environmental protection agency 12. The amount of oxygen required by bacteria to break down the
guidelines, maximum allowable concentration of metals in organic matter present in a certain value of a sample of water is
drinking water are as follows : called biochemical oxygen demand (BOD). The amount of
BOD in the water is a measure of the amount of organic material
Metal Maximum concentration in the water, in terms of how much oxygen will be required to
(ppm or mg dm-3) break it down biologically. Clean water would have BOD value
Cd 0.005 of less than 5ppm whereas highly polluted water would have
Mn 0.05 (option-c) BOD value of 17 ppm or more.
Al 0.2
BOD value of clean water = 4 ppm
BOD value of highly polluted water = 18 ppm
Fe 0.2
Cu 3.0 (option-d) 13. N2 molecule has minimum role in the formation of photochemical
smog. While CH2 == O, O3 and NO has major role. When

w
Zn 5.0 (option-a)
fossil fuels are burnt, a variety of pollutants are emitted. Two
8. Ozone is destroyed by CFCs in the upper stratosphere. of them are hydrocarbons (unburnt) and NO. When these
These compounds ultimately reach the stratosphere where they pollutants build upto high levels, a chain reaction occurs from
their interaction with sunlight. The reactions involved in the

Flo
get broken down by powerful UV radiations and release
chlorine free radical. The chlorine free radicals react with ozone formation of photochemical smog are as follows:
and cause its depletion by converting it into chlorine monoxide NO2( g ) ¾¾ hn
® NO( g )+ O( g )

ree
radical and molecular oxygen. O( g )+ O2( g ) dO3( g )
hn ·
CF2Cl2 (g ) ¾¾® Cl (g ) + C F2Cl(g ) NO( g ) + O3 ( g ) ¾® NO2( g ) + O2( g )

F
hn · O 3 reats with unburnt hydrocarbons to produce chemicals such
CFCl3 (g ) ¾¾® CFCl2 (g ) + C l(g ) as formaldehyde, acrolein and PAN.
· · 3CH4 + 2O3 ¾® 3CH2 == O + 3H2O

or
ur
C l(g ) + O3 (g ) ¾¾® Cl O(g ) + O2 (g ) + CH2 ==CCH == O + CH3 C OONO2
½½

f
Ozone holes increase the amount of UV radiation reaching the
earth. These radiations can cause skin cancer, sunburns, ageing of O
ks
(PAN)
skin.
Yo
9. Using plastic bags is wrong with respect to responsibility as a 14. Acid rain (pH = 3.5 - 5.6) constitutes strong acids like HNO3,
oo
human being to protect our environment. Plastic bags are H2SO4 and H2SO3 which slowly react with marble (CaCO3 ) of
non-biodegradable in nature. It remains in the environment as Taj Mahal and make it disfigured and discoloured. Here, CaCO3
eB

such and does not degraded by bacteria. If it is not disposed (marble) gets dissolved in acids.
properly then it may lead serious threat to the environment. The 2 H Å (aq)
activities that can be used to protect our environment are as CaCO3 (s) ¾¾¾® Ca 2 + (aq) + H2O(l ) + CO2 (g ) ­
(Acid rain)
r

follows:
ou
ad


Restricting the use of vehicles. 15. Organic pigments (colourents) present in flower buds retain

Avoiding the use of flood lighted facilities. their colour in the oxidised form of the pigment as their nature is
Y

itself oxidising in nature. When they comes in contact with



Setting up compost tin in gardens. moist SO2 (acid rain) of higher concentration, they get
10. Sun emits UV-radiations, which according to following EM decoloured and stiff.
nd
Re

categorisation have the wavelength range from 1 nm to 400 nm. SO2 + H2O ¾® H2SO4
H2SO4 ¾® H+ + HSO-4
Fi

Type Wavelength range


HSO-4 ¾® H+ + SO24 -
Radio wave > 0.1 m
Microwave 0.1 m to 1 mm Due to the release of H+ ion (acid), the flower get decoloured
and stiff.
Infrared wave 1 mm to 700 nm
As a result, flower eventually falls off from plants.
Visible rays 700 nm to 400 nm
Ultraviolet rays 400 nm to 1 nm 16. Molecular formula of peroxyacetyl nitrate (PAN) is
O
-3
X-rays 1 nm to 10 nm ½½
Gamma rays < 10- 3 nm CH3 ¾ C ¾ O ¾ O ¾ NO2. It is a secondary pollutant. It is
present in photochemical smog (oxidising or Los Angeles
Thus, option (d) with 200-315 nm range is the correct option.
smog). PAN is a powerful lachrymator or tear producer and it
11. Freons or CFCs or chlorofluoro carbons, i.e. CF2Cl2 is not the also causes breathing troubles.
common component of photochemical smog. This smog is
produced as the result of tropospheric pollution while freons are 17. Dissolved oxygen (DO) is the oxygen dissolved in water either
the components of stratospheric pollution. These are infact from atmosphere or by photosynthesis. The lower the
considered as the major cause of ozone layer depletion. concentration of DO in a water sample, the more polluted is the
water sample.
472 Environmental Chemistry
The concentration range of dissolved oxygen (DO) in cold water sample of water. Clean water or drinking water has a BOD value
reaches upto 10 ppm, but that in normal water (at room < 5 ppm.
temperature) is within 5 ppm. So, water filled with A, BOD = 10 ppm is polluted and water
18. CH4 is not present in the stratosphere and also it cannot diffuse filled with B, BOD = 20 ppm, is also polluted. But, B is more
polluted than A.
or escape into the stratosphere like freon-12 (CF2Cl2) from the
atmosphere. 20. In clean air, rain water picks up some acidic oxides like CO 2 and
In the stratosphere, ozone layer depletion take place mainly by SO 2 (obtained from volcanic eruptions). These substance make
chlorofluorocarbons (CFCs) like CF2Cl2 and the mechanism of the rain slightly acidic (pH = 5.6 - 6).
ozone layer depletion can be shown as:
· · 21. According to EEC (European Environment Commission),
hn
(i) CF2Cl2(g ) ¾¾® C l(g ) + C F2Cl(g ) [Option, (d)] excess of NO -3 (> 50 ppm) in drinking water may lead to
· methemoglobinemia (‘Blue baby syndrome’). It also may cause
(ii) C l(g) + O3 (g ) ¾® ClO· (g ) + O2 stomach-cancer.

w
(iii) ClO· (g ) + O(g ) ¾® Cl· (g ) + O2 (g ) [Option (b)] 22. For drinking water, the maximum recommended levels of some
·
metals, set by European Environment Commission (EEC) is
(iv) C l(g ) + H2O(g ) ¾® HOCl(g ) + H· (g )
[Present in the

Flo
stratosphere] Metal Max. concentration in ppm
· Zn 5
hn
(v) HOCl(g ) ¾¾® OH(g ) + Cl· (g ) [Option (c)]

ree
Mn 0.05
Þ One Cl· can destroy or deplete 105 O3 molecules.
Fe 0.2
As (i) reaction is involved in the formation of photochemical

F
smog, not in ozone layer depletion. So option (a) is correct. Cu 3

19. BOD is defined as the amount of oxygen required by bacteria to As the concentration of Mn in the given water sample is more

or
ur
break down the organic matter present in a certain volume of a than the recommended concentration. Thus, it makes water
unsuitable for drinking.

f
ks
Yo
oo
r eB
ou
ad
Y
nd
Re
Fi
JEE ADVANCED
Solved Paper 2019
Paper 1

w
Section 1 (Maximum Marks : 12)

Flo
● This section contains FOUR (04) questions.
Each question has FOUR options. ONLY ONE of these four options is the correct answer.

ree

● For each question, choose the option corresponding to the correct answer.
● Answer to each question will be evaluated according to the following marking scheme:

F
Full Marks : + 3 If ONLY the correct options is chosen.
Zero Marks : 0 If none of the options is chosen. (i.e. the question is unanswered)

or
ur
Negative Marks : - 1 In all other cases.

1. The correct order of acid strength of the following


f
4. Molar conductivity (L m) of aqueous solution of sodium
ks
carboxylic acids is stearate, which behaves as a strong electrolyte, is
Yo
recorded at varying concentrations (C) of sodium
oo
O
O OH stearate. Which one of the following plots provides the
I H II H correct representation of micelle formation in the
eB

OH H solution?
O
III MeO IV H3C OH (critical micelle concentration (CMC) is marked with
r

OH an arrow in the figures)


O
ou
ad

(a) III > II > I > IV


(b) I > II > III > IV
Y

Lm
(c) II > I > IV > III Lm CMC

(d) I > III > II > IV (a) (b)


nd
Re

CMC
2. The green colour produced in the borax bead test of a
chromium (III) salt is due to
Fi

(a) Cr 2 O 3 (b) CrB ÖC ÖC


(c) Cr(BO 2 )3 (d) Cr2 (B 4O 7 )3
3. Calamine, malachite, magnetite and cryolite,
respectively, are CMC
Lm Lm CMC
(a) ZnCO 3 , CuCO 3 , Fe 2O 3 , Na 3 AlF6 (c) (d)
(b) ZnSO 4 , CuCO 3 , Fe 2O 3 , AlF3
(c) ZnSO 4 , Cu(OH)2 , Fe 3O 4 , Na 3 AlF6
(d) ZnCO 3 , CuCO 3 × Cu(OH)2 , Fe 3O 4 , Na 3 AlF6
ÖC ÖC
2 JEE Advanced Solved Paper 2019

Section 2 (Maximum Marks : 32)


● This section contains EIGHT (08) questions.
● Each question has FOUR options. ONE OR MORE THAN ONE of these four option(s) is (are) correct options(s).
● For each question, choose the options(s) corresponding to (all) the correct answer(s).
● Answer to each question will be evaluated according to the following marking scheme:
Full Marks : + 4 If only (all) the correct option(s) is (are) chosen.
Partial Marks : + 3 If all the four options are correct but ONLY three options are chosen.
Partial Marks : + 2 If three or more options are correct but ONLY two options are chosen, both of which are correct and
both of which are correct.
Partial Marks : + 1 If two or more options are correct but ONLY one option is chosen and it is a correct option;

w
Zero Marks : 0 If none of the option is chosen (i.e., the question is unanswered);
Negative Marks : - 1 In all other cases.
● For example, in a question, if (A), (B) and (D) are the ONLY three options corresponding to correct answers, then

Flo
choosing ONLY (A), (B) and (D) will get + 4 marks choosing ONLY (A) and (B) will get + 2 marks
choosing ONLY (A) and (D) will get + 2 marks choosing ONLY (B) and (D) will get + 2 marks
choosing ONLY (A) will get + 1marks choosing ONLY (B) will get + 1marks

ree
choosing ONLY (D) will get + 1marks
choosing no option (i.e., the question is unanswered) will get 0 marks; and

F
choosing any other combination of options will get - 1mark.

or
5. Choose the reaction(s) from the following options, for (c) x1 will deflect towards negatively charged plate
ur
which the standard enthalpy of reaction is equal to the (d) x3 is g-ray

f
standard enthalpy of formation.
8. Which of the following statement(s) is(are) correct
ks
(a) 2C(g ) + 3 H2 (g ) ¾® C 2H6 (g ) regarding the root mean square speed (U rms) and
Yo
(b) 2H2 (g ) + O 2 (g )¾® 2H2O(l ) average translational kinetic energy (E av) of a molecule
oo
3 in a gas at equilibrium?
(c) O 2 (g ) ¾® O 3 (g )
2 (a) Urms is inversely proportional to the square root of its
eB

1 molecular mass
(d) S8 (s ) + O 2 (g ) ¾® SO 2 (g )
8
(b) Urms is doubled when its temperature is increased four times
6. A tin chloride Q undergoes the following reactions (not (c) Eav is doubled when its temperature is increased four times
r

balanced) (d) Eav at a given temperature does not depend on its molecular
ou
ad

mass
Q + Cl - ¾® X
Y

Q + Me3N ¾® Y 9. Choose the correct option(s) for the following set of


Q + CuCl 2 ¾® Z + CuCl reactions.
(i) MeMgBr Conc. HCl
nd

X is a monoanion having pyramidal geometry. Both Y C6H10O Q S


Re

(ii) H2O
(major)
and Z are neutral compounds.
20 % H3PO4, 360 K
Fi

Choose the correct option(s).


(i) H2, Ni HBr, benzoyl peroxide
(a) There is a coordinate bond in Y T R U
(major) (ii) Br2, hn (major) D
(b) The central atom in Z has one lone pair of electrons (major)

(c) The oxidation state of the central atom in Z is + 2 CH3 H C Br CH3


3 H 3C Br
(d) The central atom in X is sp3 hybridised Cl Cl
(a) (b)
7. In the decay sequence.
238 - x1 234 - x2 234 - x3
92 U ¾¾
¾® 90 Th ¾¾
¾® 91 Pa ¾¾
¾® S T U S
234 - x4 230 CH3
Z ¾ ¾¾® 90 Th CH3 H 3C Br
H 3C Cl
Br
x1 , x 2 , x 3 and x 4 are particles/radiation emitted by the Br
(c) (d)
respective isotopes. The correct option(s) is(are)
(a) Z is an isotope of uranium U T
S U
(b) x2 is b -
JEE Advanced Solved Paper 2019 3

10. Which of the following statement(s) is(are) true? (a) Both Y and Z are coloured and have tetrahedral shape
(a) The two six-membered cyclic hemiacetal forms of (b) Y is diamagnetic in nature while Z is paramagnetic
D-(+ ) -glucose are called anomers (c) In both Y and Z, p-bonding occurs between p-orbitals of
(b) Oxidation of glucose with bromine water gives glutamic acid oxygen and d-orbitals of manganese
(c) Monosaccharides cannot be hydrolysed to given (d) In aqueous acidic solution, Y undergoes disproportionation
polyhydroxy aldehydes and ketones reaction to give Z and MnO 2
(d) Hydrolysis of sucrose gives dextrorotatory glucose and
laevorotatory fructose 12. Each of the following options contains a set of four
molecules. Identify the option(s) where all four
11. Fusion of MnO 2 with KOH in presence of O 2 produces molecules posses permanent dipole moment at room
a salt W. Alkaline solution of W upon electrolytic temperature.
oxidation yields another salt X . The manganese (a) SO 2 , C 6H5Cl, H2Se, BrF5 (b) BeCl 2 , CO 2 , BCl 3 , CHCl 3
containing ions present in W and X , respectively, are Y
(c) NO 2 , NH3 , POCl 3 , CH3Cl (d) BF3 , O 3 , SF6 , XeF6

w
and Z. Correct statement(s) is (are)

Flo
Section 3 (Maximum Marks : 18)
● This section contains SIX (06) questions. The answer to each question is a NUMERICAL VALUE.

ree
● Four each question, enter the correct numerical value of the answer using the mouse and the on-screen virtual numeric keypad
in the place designated to enter the answer. If the numerical value has more than two decimal places, truncate/round-off the
value to TWO decimal places.

F
● Answer to each question will be evaluated according to the following marking scheme:
Full Marks : + 3 If ONLY the correct numerical value is entered.

or
Zero Marks
ur
: 0 In all other cases.

f
ks
13. Among B 2H 6, B 3N 3H 6, N 2O, N 2O 4, H 2S2O 3 and H 2S2O 8, 16. For the following reaction, the equilibrium constant K c
Yo
the total number of molecules containing covalent . ´ 10 17 .
at 298 K is 16
oo
bond between two atoms of the same kind is .............
Fe2 + (aq) + S2 - (aq) - FeS (s)
eB

14. On dissolving 0.5 g of a non-volatile non-ionic solute to 39 g When equal volumes of 0 .06 M Fe 2+
(aq) and
of benzene, its vapour pressure decreases from 650
0 .2 M S2 - (aq) solutions are mixed, the equilibrium
mmHg to 640 mmHg. The depression of freezing point
of benzene (in K) upon addition of the solute is ............ concentration of Fe2 + (aq) is found by Y ´ 10 - 17 M. The
r

value of Y is ...........
ou

(Given data : Molar mass and the molal freezing point


ad

depression constant of benzene are 78 g mol - 1 and 17. At 143 K, the reaction of XeF4 with O 2F2 produces a
Y

. K kg mol - 1 , respectively).
512 xenon compound Y . The total number of lone pair(s) of
electrons present on the whole molecule of Y is ............
nd

15. Consider the kinetic data given in the following table


Re

for the reaction A + B + C ¾® Product 18. Schemes 1 and 2 describe the conversion of P to Q and
R to S, respectively. Scheme 3 describes the synthesis of
Fi

Rate of T from Q and S. The total number of Br atoms in a


Experiment [A] [B] [C]
reaction
No. (mol dm- 3 ) (mol dm- 3 ) (mol dm- 3 ) molecule of T is ..............
(mol dm- 3s - 1 )
Scheme 1 NH2
1 0.2 0.1 0.1 6.0 ´ 10 - 5 (i) Br2(excess), H2O
(ii) NaNO2, HCl, 273 K
(iii) CuCN/KCN
2 0.2 0.2 0.1 6.0 ´ 10 - 5 (iv) H3O+, D
Q
(major)
(v) SOCl2, pyridine
3 0.2 0.1 0.2 . ´ 10 - 4
12 P

Scheme 2 (i) Oleum


4 0.3 0.1 0.1 9.0 ´ 10 - 5 (ii) NaOH, D
S
(iii) H+
(iv) Br2, CS2, 273 K (major)
-3
The rate of the reaction for [ A] = 0 .15 mol dm , R
[ B] = 0 .25 mol dm - 3 and [C ] = 0 .15 mol dm - 3 is found Scheme 3
to be Y ´ 10 - 5 mol dm - 3s - 1 . The value of Y is ............ (i) NaOH, D
S T
(ii) Q
(major)
Paper 2
Section 1 (Maximum Marks : 32)
● This section contains EIGHT (08) questions.
● Each question has FOUR options for correct answer(s). ONE OR MORE THAN ONE of these four option(s) is (are) correct
options(s).
● For each question, choose the correct options(s) to answer the question.
● Answer to each question will be evaluated according to the following marking scheme:
Full Marks : + 4 If only (all) the correct option(s) is (are) chosen.
Partial Marks : + 3 If all the four options are correct but ONLY three options are chosen.

w
Partial Marks : + 2 If three or more options are correct but ONLY two options are chosen, both of which are correct
options.
Partial Marks : + 1 If two or more options are correct but ONLY one option is chosen and it is a correct option.

Flo
Zero Marks : 0 If none of the options is chosen (i.e. the question is unanswered).
Negative Marks : - 2 In all other cases.

ree
● For example: in a question, if (A), (B) and (D) are the ONLY three options corresponding to correct answer, then
choosing ONLY (A), (B) and (D) will get +4 marks; choosing ONLY (A) and (B) will get +2 marks;
choosing ONLY (A) and (D) will get +2 marks; choosing ONLY (B) and (D) will get +2 marks;

F
choosing ONLY (A) will get +1mark; choosing ONLY (B) will get +1mark;
choosing ONLY (D) will get +1mark;

or
ur
choosing no option (i.e. the question is unanswered) will get 0 marks; and

f
choosing any other combination of options will -1mark. ks
Yo
1. Choose the correct option(s) for the following reaction 2. Choose the correct option(s) that give(s) an aromatic
oo
sequence compound as the major product.
CHO Hg2+, Br
eB

(i) dil. H2SO4 (i) SOCl2


(ii) AgNO3, NH4OH pyridine NaOEt
Q (a)
(iii) Zn-Hg, conc. HCl (ii) AlCl 3
MeO
Zn-Hg
r

conc. HCl UV, 500 K


R S (b) + Cl2 (excess)
ou
ad

Consider Q, R and S as major products.


Y

(i) Alc. KOH


(c) H 3C Br (ii) NaNH2
(iii) Red hot iron tube, 873 K
Br
nd
Re

(a) MeO MeO


NaOMe
R O S (d)
Fi

CO2H
(b) 3. The ground state energy of hydrogen atom is
MeO MeO
-13.6 eV. Consider an electronic state Y of He+ whose
O
Q R energy, azimuthal quantum number and magnetic
quantum number are -3.4 eV, 2 and 0, respectively.
Which of the following statement(s) is(are) true for the
(c) MeO MeO state Y?
O (a) It is a 4d state
R S
OH (b) The nuclear charge experienced by the electron in this state
is less than 2e, where e is the magnitude of the electronic
CO2H charge
(d) (c) It has 2 angular nodes
MeO
MeO (d) It has 3 radial nodes
Q S
JEE Advanced Solved Paper 2019 5

4. Consider the following reactions (unbalanced). (b) Natural rubber is polyisoprene containing trans alkene units
(c) Cellulose has only a-D-glucose units that are joined by
Zn + Hot conc. H 2SO 4 ¾® G + R + X glycosidic linkages
Zn + conc. NaOH ¾® T + Q (d) Nylon-6 has amide linkages
G + H 2S + NH 4OH ¾® Z (a precipitate) + X + Y
7. The cyanide process of gold extraction involves
Choose the correct option(s).
(a) The oxidation state of Zn in T is +1 leaching out gold from its ore with CN - in the presence
(b) R is a V-shaped molecule of Q in water to form R. Subsequently, R is treated with
(c) Bond order of Q is 1 in its ground state T to obtain Au and Z. Choose the correct option(s).
(d) Z is dirty white in colour (a) Q is O 2 (b) Z is [Zn(CN)4 ]2-
(c) T is Zn (d) R is [Au(CN)4 ]-
5. With reference to aqua-regia, choose the correct option(s).
(a) Aqua-regia is prepared by mixing conc. HCl and conc. HNO 3 8. Which of the following reactions produce(s) propane

w
in 3 : 1 (v / v ) ratio as a major product?
(b) The yellow colour of aqua-regia is due to the presence of (a) H3C Cl Zn, dil. HCl
NOCl and Cl 2
(c) Reaction of gold with aqua-regia produces an anion having Br

Flo
Au in +3 oxidation state (b) H3C Br Zn
(d) Reaction of gold with aqua regia produces NO 2 in the
absence of air (c) H3C

ree
COONa NaOH, CaO, D

6. Choose the correct option(s) from the following.


(a) Teflon is prepared by heating tetrafluoroethene in presence (d) H3C COONa + H2O Electrolysis

F
of a persulphate catalyst at high pressure

or
Section 2 (Maximum Marks : 18
ur
f
● This section contains SIX (06) questions. The answer to each question is a NUMERICAL VALUE.
For each question, enter the correct numerical value of the answer using the mouse and the on-screen virtual numeric keypad

ks
in the place designated to enter the answer. If the numerical value has more than two decimal places, truncate/round-off the
Yo
value to TWO decimal places.
oo
● Answer to each question will be evaluated according to the following marking scheme:
Full Marks : + 3 If ONLY the correct numerical value is entered as answer.
eB

Zero Marks : 0 In all other cases.

9. The decomposition reaction 11. Total number of hydroxyl groups present in a molecule
r
ou

of the major product P is ………


ad

D
2N 2O 5(g) ¾® 2N 2O 4 (g) + O 2 (g) is started in a closed
Y

cylinder under isothermal isochoric condition at an


(i) H2, Pd-BaSO4, quinoline
initial pressure of 1 atm. After Y ´ 10 3 s, the pressure P
(ii) Dil. KMnO4, (excess), 273 K
inside the cylinder is found to be 1.45 atm. If the rate
nd
Re

constant of the reaction is 5 ´ 10 -4 s -1 , assuming ideal


Fi

gas behaviour, the value of Y is ………

10. The mole fraction of urea in an aqueous urea solution 12. Total number of cis N ¾ Mn ¾ Cl bond angles (that is
Mn ¾ N and Mn ¾ Cl bonds in cis positions) present in
containing 900 g of water is 0.05. If the density of the
a molecule of cis [Mn(en)2 Cl 2 ] complex is ………
solution is 1.2 g cm -3 , then molarity of urea solution is
(en = NH 2CH 2CH 2NH 2 )
…… (Given data : Molar masses of urea and water are
60 g mol -1 and 18 g mol -1 , respectively) 13. The amount of water produced (in g) in the oxidation of
1 mole of rhombic sulphur by conc. HNO 3 to a
compound with the highest oxidation state of sulphur
is …… (Given data : Molar mass of water = 18 g mol -1 )

14. Total number of isomers considering


both structural and stereoisomers of cyclic ethers with
the molecular formula C4H 8O is ………
6 JEE Advanced Solved Paper 2019

Section 3 (Maximum Marks : 12)


● This section contains TWO (02) List-Match sets.
● Each List-Match set has TWO (02) Multiple Choice Questions.
● Each List-Match set has two lists : List-I and List-II
● List-I has Four entries (I), (II), (III) and (IV) and List-II has Six entries (P), (Q), (R), (S), (T) and (U).
● FOUR options are given in each Multiple Choice Question based on List-I and List-II and ONLY ONE of these four options
satisfies the condition asked in the Multiple Choice Questions.
● Answer to each question will be evaluated according to the following marking scheme :
Full Marks : + 3 If ONLY the option corresponding to the correct combination is chosen:
Zero Marks : 0 If none of the options is chosen (i.e. the question is unanswered).
Negative Marks : - 1 In all other cases.

w
Answer the following by appropriately matching the lists Which of the following options has the correct
based on the information given in the paragraph. combination considering List-I and List-II?

Flo
(a) (II), (R) (b) (I), (P)
15. Consider the Bohr’s model of a one-electron atom
(c) (I), (T) (d) (II), (Q)
where the electron moves around the nucleus. In the

ree
following List-I contains some quantities for the nth 17. List-I includes starting materials and reagents of
orbit of the atom and List-II contains options showing selected chemical reactions. List-II gives structures of
how they depend on n. compounds that may be formed as intermediate

F
List-I List-II products and/or final products from the reactions of
List-I.
µ n -2

or
(I) Radius of the n th orbit (P)
ur List-I List-II
(II) Angular momentum of the electron in the nth (Q) µ n -1

f
(I) CN (i) DIBAL-H (P)
orbit CHO
(ii) Dil. HCl
ks
(III) Kinetic energy of the electron in the nth orbit (R) µ n0 O (iii) NaBH4 CO2H
Yo
(iv) Conc. H2SO4
(IV) Potential energy of the electron in the nth (S) µ n1 O
oo
orbit OH
(II) (i) O3 (Q)
(T) µ n2 (ii) Zn, H2O
eB

CO2H (iii) NaBH4 OH


(U) µ n1/ 2 (iv) Conc. H2SO4
r

(III) (i) KCN (R)


Which of the following options has the correct Cl
(ii) H3O+,D
ou
ad

combination considering List-I and List-II? (iii) LiAIH4 O


CO2CH3
(a) (III), (P) (b) (III), (S) (iv) Conc. H2SO4
Y

(c) (IV), (U) (d) (IV), (Q)


(IV) CO2Me (S) OH
(iii) LiAIH4
16. Consider the Bohr’s model of a one-electron atom
nd

CO2Me (iv) Conc. H2SO4


Re

where the electron moves around the nucleus. In the CO2H


following List-I contains some quantities for the nth
(T) CO2H
Fi

orbit of the atom and List-II contains options showing


how they depend on n. CO2H
List-I List-II (U)
(I) Radius of the nth orbit (P) µ n -2
O
(II) Angular momentum of the electron in (Q) µ n -1
the nth orbit O
0
(III) Kinetic energy of the electron in the nth (R) µn
orbit Which of the following options has correct
combination considering List-I and List-II?
(IV) Potential energy of the electron in the (S) µ n1
(a) (III), (S), (R)
nth orbit
(b) (IV), (Q), (R)
(T) µ n2
(c) (III), (T), (U)
(U) µ n1/ 2 (d) (IV), (Q), (U)
JEE Advanced Solved Paper 2019 7

18. List-I includes starting materials and reagents of List-I List-II


selected chemical reactions. List-II gives structures of (IV) CO2Me (S) OH
compounds that may be formed as intermediate products (iii) LiAIH4
and/or final products from the reactions of List-I. CO2Me (iv) Conc. H2SO4
CO2H
List-I List-II (T) CO2H
(I) CN (i) DIBAL-H (P)
(ii) Dil. HCl
CHO
CO2H
O (iii) NaBH4 CO2H
(iv) Conc. H2SO4 (U)
O
O
(II) (i) O3 (Q) OH
(ii) Zn, H2O
O
OH

w
CO2H (iii) NaBH4
(iv) Conc. H2SO4
Which of the following options has correct
(III) Cl (i) KCN (R) combination considering List-I and List-II?
(ii) H3O+,D
(a) (II), (P), (S), (U) (b) (I), (Q), (T), (U)

Flo
(iii) LiAIH4 O
CO2CH3 (c) (II), (P), (S), (T) (d) (I), (S), (Q), (R)
(iv) Conc. H2SO4

F ree
Answer with Explanations

or
Paper 1
ur
f
1. (b) Acidic nature depends upon nature of electron D
Na 2B 4O 7 ¾® B 2O 3 + 2NaBO 2
withdrawing group and electronegativity. Electronegativity Boric Sodium
ks
anhydride
further depends on % s character. Higher the s-character, 1 4442metaborat
4443e
Yo
glassy bead
greater will be the electronegativity and hence tendency to
oo
loose H increases thus acidic character also increases. Boric anhydride is non-volalite. When it react with Cr(III) salt
then deep green complex is formed.
O
eB

O H
2Cr 3+ + 3B 2O 3 ¾® 2Cr(BO 2)3
Deep green
(I) H (II) OH
OH H Hence, option (c) is correct.
r

sp- hybridisation sp 2 -hybridisation 3. (d) ZnCO 3-Calamine (zinc ore)


ou

(30-33% s character)
ad

(50% s character)
(pKa = 1.86) (pKa = 4.3) CuCO 3 × Cu(OH)2-Malachite (copper ore)
Fe3O 4 -Magnetite (iron ore)
Y

O OH Na 3AlF6-Cryolite (aluminium ore)


(III) MeO (IV) H3C
OH Thus, option (d) is correct.
O
nd
Re

2
sp -hybridisation 3 4. (b) Key Idea The aqueous solution of ionic surfactant, i.e.
sp -hybridisation – +
(Resonance effect) sodium stearate (C17H 35CO O N a) acts as a strong univalent
Fi

(25% s-character)
(pKa = 4.5) (pKa = 4.8) type of electrolyte in the concentration range below the CMC
and the linear function of dependence of L m on C has a
Hence, acidic order I > II > III > IV. small negative slope.
II is more acidic than III since electron donating group
(¾OCH 3) is attached to benzene ring in III which decreases the
acidic character. CMC of Lm on ÖC has a
Lm small negative slope.
On the other hand, pK a value also determined acidic nature,
lower pK a value gives maximum acidic character.
Hence, option (b) is correct.
¾
2. (c) Borax bead test is performed only for coloured salt. Borax ÖC
(sodium pyroborate), Na 2B 4O 7 ×10H 2O on heating gets fused At normal or low concentration, sodium stearate
and lose water of crystallisation. It swells up into fluffy [CH 3(CH 2)16COO -Na + ] behaves as strong electrolyte and for
white porous mass which melts into a colourless liquid
which later form a clear transparent glassy bead consisting strong electrolyte, molar conductance (Lm) decreases with
of boric anhydride and sodium metaborate. increase in concentration.
D
Na 2B 4O 7 ×10H 2O ¾® Na 2B 4O 7 +10H 2O ­
8 JEE Advanced Solved Paper 2019
Above particular concentration, sodium stearate forms 8. (a, b, d) The explanation of given statements are as follows:
aggregates known as micelles. The concentration is called as
(a) Urms is inversely proportional to the square root of its
CMC. Since, number of ions decreases and hence Lm also
molecular mass.
decreases.
3RT
Hence, option (b) is correct. Urms =
M
5. (c, d) The standard enthalpy of formation is defined as Hence, option (a) is correct.
standard enthalpy change for formation of 1 mole of a (b) When temperature is increased four times then Urms
substance from its elements, present in their most stable become doubled.
state of aggregation.
3R
3 Urms = ´ 4T
O 2(g) ¾® O 3(g) ; M
2
3RT
1 Urms = 2 ´
S8 (s) + O 2(g) ¾® SO 2(g) M
8

w
Hence, option (b) is correct.
In the above two reactions standard enthalpy of reaction is
equal to standard enthalpy of formation. (c) and (d) E av is directly proportional to temperature but does
not depends on its molecular mass at a given temperature as
3
6. (a, d) E av = KT. If temperature raised four times than E av becomes

Flo
Sn 2
– – four time multiple.
SnCl2 + Cl SnCl3 Cl Cl
Tin chloride (X) Cl Thus, option (c) is incorrect and option (d) is correct.

ree
(Q) sp3 (pyramidal)
9. (c, d) The given road map problem is
SnCl -3 has (3s + 1lp) and exist in pyramidal structure.
H—OH

F
O CH3 O MgBr
Me Cl
SnCl2+Me3N SnCl2 × NMe3 Me N Sn
Cl CH3MgBr
(3°amine) Me H2O Hydrolysis

or
(Y)
(Q)
ur
f
Y complex has coordinate bond in between nitrogen and Sn metal. (C6H10O)
Reduction CH3 Cl CH3 OH
ks
H
Br
Yo
+2 +2 +4 +1 Conc. HCl + Mg
oo
SnCl2 + 2CuCl2 SnCl4 + 2CuCl –H2O
(Q) (Z)
OH
Major (Q)
Oxidation
eB

(S)
Z is oxidised product and oxidation state of Sn is +4 in Z
CH3 Br CH3
compound. Structure of SnCl 4 (Z) is
(i) H2,Ni 20% H3PO4,
(Reduction)
r

Cl 360K
+ H2O
ou
ad

Sn (ii) Br2,hn Dehydration


Cl major (free radical major of alcohol
Cl Cl (T) substitution) (R)
Y

Thus, options (a, d) are correct. CH3


Br
7. (a,b,c) Key Idea The lose of one a-particle will decrease the HBr
nd
Re

mass number by 4 and atomic number by 2. On the


other hand, loss of b-particle will increase the atomic Benzoyl peroxide
(anti-Markovnikov's rule) major
number by 1.
Fi

(U)

In decay sequence, Hence, options (c, d) are correct.


234
92U
238
90Th + 2He4 (or a)
X1 particle
10. (a, c, d) The explanation of given statements are as follows:
(a) Two six membered cyclic hemiacetal form of
234
+ (b– or –1e0)
234
+ (b– or –1e0) D-(+)- glucose are called anomers.
92U 91Pa
234 X3 particle X2 particle
Z is isotope CH2OH CH2OH
of uranium
H O OH H O H
230 4 H H
90Th + 2He (or a) OH OH
H H
X4 particle OH H OH OH
X1 particle will deflect towards negatively charged plate due to
H OH H OH
presence of positive charge on a- particles.
a-D(+)-glucopyranose b-D(+)-glucopyranose
Hence, options (a, b, c) are correct.
Both are anomers.
JEE Advanced Solved Paper 2019 9
(b) Oxidation of glucose in presence of Br2 water gives The molecules which gives permanent dipole moment are
gluconic acid. polar in nature.
CHO COOH Cl

H OH H OH S Se
, ,
HO H Br2 HO H O O H H
H OH Water H OH m¹0 m¹0 m¹0
(polar) (polar) (polar)
H OH H OH F
F F
CH2OH CH2OH
m1 m2
D-(+)-glucose Gluconic acid Br
, Cl Be Cl
(c) Monosaccharides can not be hydrolysed into polyhydroxy F F mnet» 0
m¹0
aldehydes and ketones. (non-polar)
(polar)

w
(d) Hydrolysis of sucrose gives D-glucose and L-fructose. Cl
Invertase m1 m2
C12H 22O11 + H 2O ¾¾¾¾® C6 H12O 6 + C6H12O 6 ,
D-glucose L-fructose O C O B Cl
or dextrorotatory or laevorotatory

Flo
mnet 0 m 0
Cl
Hence, options (a, c, d) are correct. (non-polar) (non-polar)

D 1
11. (a, c, d) MnO 2 + 2KOH + O 2 ¾® K 2MnO 4 + H 2O Cl

ree
N N Cl
2 (W) potassium
manganate H C Cl , , , O P Cl ,
O O H H H

F
+ 2– Cl
K2MnO4 (aq) 92K (aq) + MnO 4 (aq) Cl m¹0
m¹0 m¹0 m¹0
(W) (Y) (polar)
(polar) (polar) (polar)
O F

or
3

é Mn
ur
sp- hybridisation,
tetrahedral (manganate ion)
é
é H F
F
F
B O

f

O O Green coloured H C Cl , , , Xe
O complex F F O O
H m¹0
ks
m=0
MnO 2- m¹0 F F
4 ion has one unpaired electrons, therefore it gives (polar) (non-polar)
Yo
(polar) F
d-d transition to form green colour. Y complex has
oo
m»0
paramagnetic nature due to presence of one unpaired electron. (non-polar)
In aqueous solution, Thus, options (a, c) are correct.
eB

Electrolytic oxidation 13. (4.00) N 2O, N 2O 4 , H 2S2O 3 and H 2S2O 8 molecules are
K 2MnO 4 + H 2O ¾¾¾¾¾¾¾®H 2 + KOH + KMnO 4
( W) ( X) containing covalent bond between two atoms.
S
r

O
sp3, tetrahedral
ou
ad

KMnO4(aq)
D
K+ + MnO4- éMn (purple coloured é
é N ºº N ¾® O , S
O O
(Z) (N2O) N¾N ,
O O complex ion)
Y

HO OH O O
O O (N2O4)
MnO -4 ions gives charge transfer spectrum in which a fraction of (H2S2O3)
nd
Re

electronic charge is transferred between the molecular entities. O O


H H
Electrolytic H
Q MnO 24- ¾¾¾® MnO -4 +e -
Fi

(Y) oxidation ( Z) S S , B B
In acidic medium, Y undergoes disproportionation reaction. HO O¾O OH H H
O O H
3MnO 24- (aq) + 4H+ ¾® 2MnO -4 + MnO 2 + 2H 2O (H2S2O8) (B2H6)
(Y) (Z)
H
MnO 2-
4 and MnO -4 both ions form p-bonding between |
(Y) ( Z) H B H
p-orbitals of oxygen and d-orbitals of manganese. N N
Thus, options (a, c, d) are correct.
12. (a,c) Key Idea Dipole moment of a bond depends on the B B
difference in the electronegativities of bonded atoms. H N H
More is the difference in the electronegativities, greater (B3N3H6)
will be the dipole moment. Also,
For symmetrical molecule, m = 0 B 2H 6 and B 3N 6H 6 have polar bond, but do not have same kind
of atom.
For unsymmetrical molecule, m ¹ 0
10 JEE Advanced Solved Paper 2019

14. (1.02) Key Idea First calculate, molar mass of solute using the 16. (8.9) Given, equilibrium constant (K c) at 298 K = 1.6 ´ 1017
p ° - ps nsolute
formula, = and then calculate DTf Fe2+ (aq) + S2- (aq) 1 FeS (s)
p° nsolute + nsolvent
by applying the formula; DTf = K f ´ m. At initial concentration 0.06 M 0.2 M –
(Before mixing)
At initial concentration 0.03 M 0.1 M –
When 0.5 g of non-volatile solute dissolve into 39 gm of (After mixing)
benzene then relative lowering of vapour pressure occurs. At equilibrium 0.03- X 0.1 – 0.03 = 0.07

Hence, vapour pressure decreases from 650 mmHg to
640 mmHg. [Here, K c >>10 , thus limiting reagent will be consumed
3

Given, vapour pressure of solvent (p°) = 650 mmHg almost completely, 0.03 - X = 0 \ X = 0.03]
From equilibrium constant,
Vapour pressure of solution (ps) = 640 mmHg
[FeS]
Weight of non-volatile solute = 0.5 g KC =
[Fe2+][S2- ]
Weight of solvent (benzene) = 39 g
1

w
From relative lowering of vapour pressure, KC = [For FeS(s) = 1 mol L -1 ]
X ´ 0.07
p ° - ps nsolute (Pure solid)
= xSolute = 1
p° nsolute + nsolvent 1.6 ´ 10 =
17

0.5 X ´ 0.07

Flo
650 - 640 molar mass X=
1
= 8.9 ´ 10-17
=
650 0.5 39 1.6 ´ 10 ´ 0.07
17
+

ree
molar mass 78 Given, X = Y ´10-17 = 8.9 ´10-17
0.5 \ Y = 8.9
10 molar mass
= 17. (19) XeF4 reacts with O 2F2 to form XeF6 × O 2F2 is fluoronating

F
650 0.5
+ 0.5 reagent.
molar mass
0.5 + 0.5 ´ molar mass = 65 ´ 0.5
143 K
XeF4 + O 2F2 ¾¾® XeF6 + O 2

or
\ Molar mass of solute = 64 g
ur (Y)

f
From molal depression of freezing point, The structore of XeF6 is
DTf = K f ´ molality F
ks
K f ´ wsolute F
= F
Yo
(MW)solute ´ wsolvent Xe
oo
0.5 ´ 1000 F F
DTf = 512
. ´ Þ DTf = 1.02K
64 ´ 39 F
eB

Y compound (XeF6) has 3 lone pair in each fluorine and one


15. (6.75) Rate = k[A]x [B]y [C]z lone pair in xenon.
(Rate)1 [0.2]x [01
. ]y [01
. ]z 6 ´10-5 Hence, total number of lone pairs electrons is 19.
r

= =
x y z
6 ´10-5
ou

(Rate)2 [0.2] [0.2] [01 .] 18. (4) Scheme -1


ad

Þ y=0 NH2 NH2


6 ´10-5
Y

(Rate)1 [0.2]x [01


. ]y [01
. ]z Br Br
= = (i) Br2(excess) (ii) NaNO2 +HCl
x
(Rate)3 [0.2] [01 y
. ] [0.2] z
1.2 ´10-4 ¾¾¾¾¾¾®
H2O 273K
Þ z =1
nd
Re

Aniline (diazotisation
(Rate)1 [0.2]x [01 . ]z 6 ´10-5
. ]y [01 Br of aniline )
= = (P)
x
(Rate)4 [0.3] [01 y
. ] [01. ]2 9 ´10-5 2,4,6-tribromoaniline
Fi

+ –
N 2 Cl
Þ x =1 CN
Br Br
So, rate = k[A]1 [C]1 Br Br
(iii) CuCN/KCN
®

+ N2 –
From exp-Ist, Cl is displace
Rate = 6.0 ´ 10-5 mol dm -3 s -1 by CN–ion
Br
6.0 ´ 10-5 = k[0.2]1 [01
. ]1 Br 2, 4, 6 tribromo
¾¾¾®

(complete hydrolysis of diazonium chloride salt


k = 3 ´ 10-3 (iv) H3O
+
¾ CN group which
Given, [A] = 015mol
. dm -3 convert into ¾COOH) O
-3
[B] = 0.25 mol dm COOH C ¾Cl
-3
[C] = 015
. mol dm Br Br (v) SOCl2, Br Br
Pyridine
\Rate = (3 ´ 10-3) ´ [015 . ]1 = 3 ´ 10-3 ´ 015
. ]1 [0.25]0 [015 . ´ 015
. ¾¾¾¾®
Rate = 6.75 ´ 10-5 mol dm -3 s -1
Br Br
Thus, Y = 6.75 Major
(Q)
JEE Advanced Solved Paper 2019 11
Scheme-2
SO3H SO3Na OH OH OH
+ Br
(i) Oleum (ii) NaOH, D (iii) H Hydrolysis Br2
¾¾¾¾® ¾¾¾¾® ¾¾¾¾¾¾¾® + NaHSO3 ¾¾¾¾® +
H2S2O7 CS2, 273K
(R ) (H2SO4+ SO3) (Bromination)
minor
(Sulphonation)
2- bromophenol
Br
major (S)
4- bromophenol
Scheme-3 O
Br
C¾Cl O
Br Br O
OH O–Na+ C

w
Br Br
(i) NaOH, D (Q) Br
¾¾¾® + NaCl

Br

Flo
Br Br
(S) (T)
(Major)
T compound has total number of Br atom =4

ree
Paper 2

F
1. (a, b)

or
C C CH2 CHO C CH CH2 CHO
ur (i) Hg2+, dil. H2SO4
Hydration of Alkyne

f
OH
CH3O CH3O
ks
Keto–enol
Tautomerism
Yo
(ii) AgNO3+NH4OH
C CH2 CH2 C O– C CH2 CH2 CHO
oo
(Tollen's reagent)

O O (¾CHO converts into COO–) O


eB

CH3O CH3O
CH2 CH2
(iii) Zn – Hg + Conc. HCl CH2 (i) SOCl2+Pyridine CH2
r

(Clemmensen reduction) (ii) AlCl3


CH3O
ou

CH3O CH2 H
ad

C ¾ reduce to ¾ CH2 Cl CH2


(Q) C
COOH
Y

O
O
CH2 CH2
CH2 CH2
nd

Zn-Hg/Conc. HCl Electrophilic


Re

CH2
Clemmensen reduction CH2 substitution reaction
CH3O CH2 CH3O C
Fi

(S)
O
(R)
2. (c, d) Key Idea An aromatic compound must be cyclic and planar. It must follow ( 4 n + 2 ) e - rule and have the conjugated system in it.

(a) Br OEt
NaOEt Dimerise
+
Substitution product Ellimination product (Non-aromatic)

(b) Benzene react with Cl 2 (excess) in presence of UV light and 500 K of temperature to form benzene hexachloride (non-aromatic).
Cl
Cl Cl
UV
+ Cl2 (excess) 500 K

Cl Cl
Cl
(Non-aromatic)
12 JEE Advanced Solved Paper 2019
(c) CH3 0 +3
Au+ HNO 3 +4HCl ¾® AuCl –4 + H 3O + + NO + H 2O
(i) Alc. KOH (iii) Red hot iron tube
CH3C CH
Br (ii) NaNH2 873 K (Trimerisation)
Br Oxidation
CH3
Hence, option (c) is correct.
(d) Reaction of gold with aqua-regia produces NO gas in
absence of air.
CH3 CH3 Hence, option (d) is incorrect.
(Aromatic)
6. (a, d) The explanation of given statements are as follows:
H (a) Teflon is prepared by heating tetrafluoroethene in
s Na+ presence of persulphate catalyst at higher pressure.
Persulphate
(d) NaOMe nCF2 ==CF2 catalyst
CF2 CF2
–MeOH
Thermoplastic Polymer
(Aromatic ion) (PTFE)

w
Thus, (c) and (d) options are correct. Thus, option (a) is correct.
(b) Natural rubber is polyisoprene containing cis alkene units.
3. (a, c) Given, ground state energy of hydrogen atom = - 136
. eV
CH2 CH2 n
Energy of He + = - 34

Flo
. eV, Z = 2
nCH2 C CH CH2 C C
. ´ Z2
136
Energy of He + , E = - eV CH3 H
n2

ree
CH3 cis-ployisoprene unit
- 136. ´ (2)2 . ´4
136 Isoprene (natural rubber)
. eV =
- 34 Þ n= Þ n= 4
n2 34
. Thus, option (b) is incorrect.

F
Given, azimuthal quantum number () l = 2(d – subshell (c) Cellulose has only b-D-glucose units that are joined together
by glycosidic linkages as shown in the following structure:
Magnetic quantum number (m) = 0

or
\ Angular nodes ()l =2
ur OH OH OH OH
Radial node = n - l - 1 = 4 - 2 - 1 = 1

f
O O O
nl = 4d state O OH O
1 4 OH
ks
O OH OH O
Hence, options (a), (c) are correct. O
,
Yo
4. (b, c, d) When Zn react with hot conc. H 2 SO4 then SO2 is
oo
OH OH OH O
n OH
released and ZnSO 4 is obtained. (cellulose)
(1, 4, b -linkage)
eB

Zn + 2H 2 SO4 ¾® ZnSO4 + SO2 ­ + 2H 2 O


Thus, option (c) is incorrect.
(Hot + Conc.) (G) (R) (X)
(d) Nylon-6 has amide linkages.
R(SO2) molecule is V-Shaped O
r

O
ou

S C
ad

NH D, H2O –nH2O
[H2N(CH2)5COOH] C (CH2)5 NH
O O D
Y

Nylon-6 n
Thus, option (b) is correct.
Caprolactam
When Zn is react with conc. NaOH then H 2 gas is evolved and
Thus, option (d) is correct.
nd

Na2ZnO2 is obtained.
Re

Zn + 2NaOH (conc.) ¾® Na2ZnO2 + H 2 ­ 7. (a, b, c) Cyanide process of gold extraction involves leaching
out gold from its ore with CN - in the presence of O 2 (Q) in
( T) ( Q)
Fi

In ground state, H—H (Q) (bond order = 1)


Thus, option (c) is correct. water to form [Au(CN)2 ]- (R).
The oxidation state of Zn in T(Na 2ZnO 2) is +2 When [Au(CN)2 ]- reacts with Zn (T), it froms [Zn(CN)4 ]2- (Z)
Thus, option (a) is incorrect. and Au.
ZnSO4 + H 2 S + NH 4 OH ¾® ZnS¯ + 2H 2 O + (NH 4 )2 SO4 The corresponding reactions are as follows :
( G) ( Z) ( X) (Y)
ZnS (Z) compound is dirty white coloured. H O + O2 ( Q)
4 Au (s) + 8 CN - (aq) ¾¾¾¾®
2
4[Au(CN)2 ]- + 4OH - (aq)
Thus, option (d) is correct. (R)

5. (a, b, c) The explanation of given statements are as follows: 2[Au(CN)2]- (aq) + Zn(s) ¾® [Zn(CN)4 ]2- (aq) + 2 Au(s)
( R) ( T)
(a) Aqua-regia is prepared by mixing conc. HCl and conc.
HNO 3 in 3:1 (v/v) ratio and is used in oxidation of gold Hence, options (a, b, c) are correct.
and platinum. Hence, option (a) is correct.
8. (a, c) The given reactions takes place as follows:
(b) Yellow colour of aqua-regia is due to its decomposition
into NOCl (orange yellow) and Cl 2 (greenish yellow). (1)
Hence, option (b) is correct. Cl Zn, dil. HCl
CH3 CH2 CH3
(c) When gold reacts with aqua-regia then it produces AuCl -4 (a) CH3 (Propane)
anion complex in which Au has +3 oxidation state.
JEE Advanced Solved Paper 2019 13
Br
11. (6)
Br Zn
CH3 CH CH2 + ZnBr2
(b) CH D (i) H2, Pd–BaSO4
3
(Elimination (Propene)
reaction) Quinoline (reduction)
(Triple bond converts into
CH3 NaOH, CaO, D
CH3 CH2 CH3+ CO2 double bond)
Electrolysis
(c) COONa (Propane)

(d) CH3 Electrolysis


CH3 (CH2)4 CH3
COONa n-hexane
CH2 CH2
Thus, options (a, c) are correct. (ii) Dil. KMnO 4
(excess) 273K
C C
9. (2.3) At constant V, T H H
D OH
2N 2 O5 (g) ¾® 2N 2 O4 (g) + O2(g)

w
H H
At initial t = 0 1 0 0
OH
t = Y ´103 sec 1 - 2p 2p p C C CH2
OH
pTotal = 1 - 2p + 2p + p

Flo
OH OH OH
1. 4 = 1 + p
(P) (Major product)
p = 0.45 atm

ree
Compound (P) has total number of hydroxyl groups = 6
According to first order reaction,
2303
. pi 12. (6) The structure of cis-[Mn(en)2Cl 2] complex is
k= log

F
t pi - 2p Cl (a) Cl (a)
Cl N1 Cl (b)
pi = 1atm (given) (b)
Mn Mn

or
2p = 2 ´ 0.45 = 0.9 atm
ur
On substituting the values in above equation,
en en N2 N4
N

f
1 3
2k × t = 2.303log
1 - 0.9 Bond angles (Mn—N and Mn—Cl bond in cis positions)
ks
1 Cl (a) —— Mn —— N(1)
Yo
2 ´ 5 ´ 10-4 ´ y ´ 103 = 2.303log
oo
01
. Cl (a) —— Mn —— N(2)
y = 2.303 = 2.3 Cl (a) —— Mn —— N(4)
eB

Note Unit of rate constant (k), i.e. s-1 represents that it is a first Cl (b) —— Mn —— N1
order reaction. Cl (b) —— Mn —— N3
Cl (b) —— Mn —— N4
10. (2.98 mole) Number of cis Cl—Mn—N = 6
r
ou

Number of moles of solute ´ 1000


ad

Key Idea Molarity ( M) = 13.(288) Key Idea Rhombic sulphur (S 8 ) gets oxidised into
Volume of solution (in mL)
sulphuric acid and water, NO 2 gas is released on reaction
Y

with conc. HNO 3.


Mass
Also, volume =
Density When rhombic sulphur (S8 ) is oxidised by conc. HNO3 then
nd
Re

Given, mole fraction of urea (c urea) = 0.05 H 2 SO4 is obtained and NO 2 gas is released.
Mass of water = 900g S8 + 48HNO3 ¾® 8H 2 SO4 + 48NO2 + 16 H 2O
Fi

Density = 1.2 g/cm 3 1 mole of rhombic sulphur produces = 16 moles of H 2 O


nurea 900 \ Mass of water = 16 ´ 18 (molar mass of H 2 O)
c urea = [Q Moles of water = = 50]
nurea + 50 18 = 288 g
nurea 14. (10.0) The structure of cyclic ether with molecular formula,
0.05 = Þ 19nurea = 50
nurea + 50 C4H 8O are as follows:
nurea = 2.6315 moles (chiral carbon)
wurea = nurea ´ (M × wt)urea = (2. 6315 ´ 60)g
(1) O (2) *
2.6315 ´ 60 + 900 é Mass of solution ù
V= Q Density = R isomer
1.2 êë Volume of solution úû = 2 isomer
, O S isomer
= 881.57 mL
Now, molarity O
1000 (3) (4) R isomer = 2 isomer
= Number of moles of solute ´ O , * S isomer
Volume of solution (mL)
2.6315 ´ 1000 (1-chiral carbon)
= = 2.98 M
881.57
14 JEE Advanced Solved Paper 2019
O O R, R isomer O
(5) (6) R, S isomer = 3 isomer
* * , S, S isomer C OMe
(i) LiAlH4
(2-chiral carbon) (IV) CH2OH
C OMe
Total number of isomers of cyclic ether with molecular CH2 (Q)
formula, C4 H 8 O are 10. CH2OH
O
15. (a) (III) Kinetic energy of the electron in nth orbit, (ii) conc. H2SO4
Z2 + H2 O
K.E. = + 136 . ´ 2 O
Dehydration
n (R)
1
or K.E. µ 2 or K.E. µ n-2 Hence, correct match of IV is Q, R.
n
Hence, correct matching from list-I and list II on the basis of
From list-II, correct match is (III P).

w
given option is (IV), Q, R.
(IV) Potential energy of the electron in the nth orbit,
18. (a) O
Z2
P.E. = - 2 ´ 136. ´ 2
n CH2 C H

Flo
1 CN
P.E. µ 2 (I) (i) DIBAL–H
n O (ii) Dil. HCl OH
P.E. µ n-2 CH

ree
O OH
From List II, correct match is (IV P). –H2O

Hence, correct matching from list-I and list-II on the basis of O

F
given option is (III, P).
CH2OH (iii) NaBH4 CH
16. (c) (I) Radius of the nth orbit, (reduction) H

or
n2
ur (Q) CH2 OH C
r = 0.529 ´

f
Z O
r µ n2
ks
Here, (iv) Conc. H2SO4
(dehydration)
Yo
From list-II, correct match is (I, T) O
oo
(II) Angular momentum of the electron, (R)
nh Hence, correct match of (I) are (Q, R)
mvr = or mvr µ n
eB

2p
O
From list-II, correct match (II, S)
Hence, correct matching from list-I and list-II on the basis of CH2 C H
(i) O3, (ii) Zn + H2O
r

given option is (I, T). (II)


ou

(Ozonolysis)
ad

17. (b) COOH COOH


(S) (P)
Y

(III) Cl (i) KCN CN


Nucleophilic (iii) NaBH4
substitution reaction
C OCH3 (Cl– replaces by CN– ion) C OCH3
nd
Re

(iv) Conc. H2SO4 CH2 OH


O O –H2O
O
Fi

C OH
CH3OH + COOH
(ii) H3O+ O
D (U) O
LiAlH4

C OH (Complete hydrolysis
(T ) Hence, correct match of II is (P , S ,U).
of —CN give —COOH
O and ester also get hydrolysed
into —COOH and alcohol ) Hence, correct matching from list-I and list-II on the basis of
CH2 CH2 OH given option is (II), P , S , U.
(iv) conc. H2SO4
H2O +
D O
(Q) CH2 OH
(R)
Hence, correct match of (III) are T , Q, R.

You might also like